openev.debatecoaches.org€¦  · Web view1AC . Observation 1: If I could find the spot where...

452
1AC Observation 1: If I could find the spot where truth echoes I would stand there and whisper memories of my children's future I would let their future dwell in my past So that I might live a brighter now Now is the essence of my domain and it contains All that was and will be And I am as I was and will be because I am and always will be That nigga I am that nigga I am that nigga I am that timeless nigga that swings on pendelums like vines Through mines of boobytrapped minds that are enslaved by time I am the life that supersedes lifetimes, I am It was me with serpentine hair and a timeless stare That with immortal glare turned mortal fear into stone time capsules They still exist as the walking dead, as I do The original sufferhead , symbol of life and matriarchy Severed head Medusa, I am I am that nigga I am that nigga! I am that nigga!! I am a negro! Yes negro, negro from _necro_ meaning death I overcame it so they named me after it And I be spitting at death from behind

Transcript of openev.debatecoaches.org€¦  · Web view1AC . Observation 1: If I could find the spot where...

Page 1: openev.debatecoaches.org€¦  · Web view1AC . Observation 1: If I could find the spot where truth echoesI would stand there and whisper memories of my children's future. I would

1AC Observation 1:

If I could find the spot where truth echoesI would stand there and whisper memories of my children's futureI would let their future dwell in my pastSo that I might live a brighter nowNow is the essence of my domain and it containsAll that was and will beAnd I am as I was and will be because I am and always will beThat niggaI am that niggaI am that niggaI am that timeless nigga that swings on pendelums like vinesThrough mines of boobytrapped minds that are enslaved by timeI am the life that supersedes lifetimes, I amIt was me with serpentine hair and a timeless stareThat with immortal glare turned mortal fear into stone time capsulesThey still exist as the walking dead, as I doThe original sufferhead, symbol of life and matriarchySevered head Medusa, I amI am that niggaI am that nigga!I am that nigga!!I am a negro! Yes negro, negro from _necro_ meaning deathI overcame it so they named me after itAnd I be spitting at death from behindAnd putting "Kick Me" signs on it's backBecause I am not the son of Sha-Clack-ClackI am before that, I am beforeI am before beforeBefore death is eternity, after death is eternityThere is no death there's only eternityAnd I be riding on the wings of eternityLike HYAH! HYAH! HYAH! Sha-Clack-ClackBut my flight doesn't go undisturbedBecause time makes dreams deferAnd all of my time fears are turning my days into daymares

Page 2: openev.debatecoaches.org€¦  · Web view1AC . Observation 1: If I could find the spot where truth echoesI would stand there and whisper memories of my children's future. I would

And I live daymares reliving nightmaresOf what taunted my pastSha-Clack-Clack, time is beatin my assAnd I be havin dreams of chocolate covered watermelonsFilled with fried chickens like pinatasWith little pickaninny sons and daughtersStanding up under them with big sticks and aluminum foilHittin em, tryin to catch pieces of fallin fried chicken wingsAnd Aunt Jemima and Uncle Ben are standing in the cornersWith rifles pointed at the heads of the little children"Don't shoot the children," I shout, "don't shoot the children!"But they say it's too lateThey've already been infected by timeBut that shit is before my timeI need more timeI need more timeBut it's too lateThey start shooting at children and killing them!One by one, two by two, three by three, four by fourFive by five, six by six, butMy spirit is growing seven by sevenFaster than the speed of lightCause light only penetrates the darkness that's already thereAnd I'm already thereI'm here at the end of the roadWhich is the beginning of the road beyond time, butWhere my niggas at? (Oh shit!)

Oh shit, don't tell me my niggas got lost in timeMy niggas are dying before their timeMy niggas are serving unjust timeMy niggas are dying because of.. time

Page 3: openev.debatecoaches.org€¦  · Web view1AC . Observation 1: If I could find the spot where truth echoesI would stand there and whisper memories of my children's future. I would

Observation 2 is the gun.Dem crackers see us as some violent criminals, they use our blackness to criminalize us, it’s like our blackness is the weapon of the white master, ya feel me?Chaney ’15 (http://www.jpanafrican.org/docs/vol8no4/8.4-5-CCRR.pdf Cassandra Chaney, Ph.D. Associate Professor, College of Human Sciences and Education, School of Social Work, Department of Child and Family Studies, Louisiana State University//Dewayne Martin)

While this film has been characterized as a cinematic masterpiece, it greatly advanced the aims of White Supremacy. In particular, this film had a deleterious effect on African American males because they were depicted as brutes whose sole intent in life was to rape White women and destroy the White man’s way of life (Loewen, 2007). As an effective Ku Klux Klan recruitment tool, this film galvanized support for the Klan as the preeminent savior of White southern civilization. So far-reaching was the negative effects of this movie for African American people, during a private White House screening by-then President Woodrow Wilson, he reportedly described the film as painting an accurate portrait of African Americans (Loewen, 2007). In short, this film was a catalyst for most of the early 20th century media caricatures of African Americans that still exist today (Baker, 1996; Menzel- Baker, Motley, & Henderson, 2004). The aforementioned images of African Americans have contributed to what Armour (1997) calls Negrophobia, or an irrational fear of African Americans. This irrational fear has contributed to Whites’ general desensitization to African American suffering of all types as well as decreased support for social safety nets. Thus, African American males are devalued and seen as expendable (Burrell, 2010). Unfortunately, Black as a metaphor for criminality is so deeply embedded in the minds of societal members that Whites have reported seeing an African American criminal suspect at the scene of a crime when none was actually present (Chaney & Robertson, 2013b; Leverentz, 2012; Oliver & Fonash, 2002). In a study to determine the effect of network news images on viewer perceptions, Dixon (2008) found that exposure to network news depressed estimates of African American income, increased the endorsement of stereotypes of African Americans as poor and intimidating, and were associated with higher racism scores. Media characterizations of violent criminals as Black has been deeply etched in the psyche of many viewers (Dixon & Maddox, 2005; Leverentz, 2012; Oliver & Fonash, 2002; Oliver, Jackson, Moses, & Dangerfield, 2004). Such negative portrayals of African Americans in media have resulted in wanton stereotyping, extreme fear of African Americans, and African Americans with darker complexions and more Afrocentric phenotypic features being perceived as more worthy of the death penalty in research experiments (Blair, Judd, & Chapleau 2004; Chaney & Robertson, 2013b; Dixon, 2008; Eberhardt, Davies, Purdie-Vaughns, & Johnson, 2006; Maddox & Gray, 2002; Peffley & Hurwitz, 2013). All of the aforementioned serve to legitimize White supremacy, legitimize White life, and de-legitimize African American life so that incidents of police violence against African Americans are not punished nor viewed as a larger societal problem.

Page 4: openev.debatecoaches.org€¦  · Web view1AC . Observation 1: If I could find the spot where truth echoesI would stand there and whisper memories of my children's future. I would

We protest and dem white folks try to make it seem like our protest ain’t do shit, tryna stop us from fighting for freedom, shit we aint letting that happen, dem motherfuckers use their big ass guns to shoot us down like it aint nuthin, Money-hungry companies that make guns that circulate the country And then wind up in the hood, making bloody clothes, dem bullets hit a young boy with a snotty nose.Coyne and Hall-Blanco ‘16 (Christopher J.Coyne, Assc. Prof of Economics @ GMU & Abigail R. Hall-Blanco, Asst Prof of Economics @ Univ of Tampa; Foreign Intervention, Police Militarization, and Minorities; PEACE REVIEW: A Journal of Social Justice, 28:165-170, No. 6, April-June; http://web.b.ebscohost.com/ehost/detail/detail?vid=0&sid=9a5a192e-47bd-49ca-a16a-24a522d91cb0%40pdc-v-sessmgr02&bdata=JnNpdGU9ZWhvc3QtbGl2ZQ%3d%3d#db=aph&AN=116270270) \\EG

The issues of police militarization and the disproportionate use of force against historically disadvantaged racial groups are inextricably linked. The goal here is to provide insight into the origins of domestic police militarization. To do so, we build on our previous work to discuss how proactive U.S. foreign policy generates undesirable domestic consequences, which threaten the liberties and freedoms of U.S. citizens. In the context of police militarization, past foreign military interventions led directly to the militarization of U.S. police. The undesirableconsequences have fallen disproportionately on minorities and disadvantaged groups. The main takeaway is that a proactive, imperialistic foreign policy can impose significant costs on domestic citizens due to expansions in the scope of state power. Special Weapons and Tactics (SWAT) teams are police units possessing highly specialized military equipment and training. These groups have become a fixture in many police departments in the United States. In the mid-1980s, approximately 20 percent of police departments had a SWAT team. By the year 2000, nearly 90 percent of police departments serving populations of 50,000 or more had a SWAT team. An estimated 3,000 SWAT deployments occurred in 1980. By the early 2000s, SWAT teams deployed 45,000 times every year. Current estimates place the number of SWAT deployments as high as 80,000 annually. Although now common, SWAT teams are a relatively new innovation, one that can be directly tied to U.S. foreign intervention and to two combat veterans, former Los Angeles police chief and World War II veteran Daryl Gates and former Los Angeles police officer and Marine John Nelson. As a Marine, Nelson served in Vietnam in an elite Force Recon unit. Although originally designed to gather intelligence, these Force Recon teams saw extended combat and were recognized for their use of lethal force. They became well known for being experts at skillfully eliminating enemy targets. For example, the "kill ratio," or number of enemies killed per every soldier lost, was about 7.6 enemies per Marine for regular Marine infantries during the Vietnam War. The kill ratio for the Force Recon units, meanwhile, was about 34 enemies for every man lost in action. The Force Recon units were also more aggressive. Regular Marine units were the aggressors in combat only 20 percent of the time they saw action. The Force Recon teams, in contrast, were the aggressors in an astounding 95 percent of their operations. Stated differently, the Force Recon units were trained to gather information, engage enemy combatants, and kill. They did so efficiently. These experiences were integral in developing John Nelson's skills, knowledge, and abilities regarding methods for controlling large groups, gathering information, and eliminating enemies. He brought this unique human capital with him when he returned to the United States and joined the Los Angeles Police Department (LAPD). In the 1960s, Nelson was presented an opportunity to use the unique skills of social control that he had developed abroad. In 1965, racial tensions set off the Watts riots in Los Angeles. The riots left the LAPD

Page 5: openev.debatecoaches.org€¦  · Web view1AC . Observation 1: If I could find the spot where truth echoesI would stand there and whisper memories of my children's future. I would

feeling unprepared. In response to the perceived crisis surrounding the riots, leaders of the LAPD were anxious to develop new ways to effectively control the large crowds regularly in attendance at race rallies. Drawing from his experiences with the Force Recon team, Nelson suggested the development of a similar unit within the LAPD. "A small squad of highly trained police officers armed with special weapons," he suggested, "would be more effective in a riotous situation than a massive police response." To take effect, however, Nelson's idea would need administrative support. It was here that Inspector (later Police Chief) Daryl Gates was essential. Like Nelson, Gates was a veteran of foreign conflict, serving aboard the USS Ault during World War II. And like Nelson, Gates returned home from war with a unique skill set and had joined the LAPD. By the time Nelson proposed his idea, individuals like Gates had successfully worked their way into the administration of the LAPD. The presence of war veterans created an administrative and cultural openness to the use of military tactics. In fact, Gates himself wanted the LAPD be more militaristic and "aggressive, intimidating, and confrontational by design." It was within this environment that Nelson proposed the idea of a Force Recon style unit within the LAPD. With Gates' support, Nelson's idea was quickly adopted. The link between the newly formed paramilitary unit and the ongoing war in Vietnam was readily apparent. Initially called the "Special Weapons and Attack Team," it was decided that the word "attack" was politically unpalatable. Gates subsequently changed the name to "Special Weapons and Tactics" and the first SWAT team was born. The first SWAT unit consisted of 60 of the LAPD's top marksmen divided into teams consisting of five men—a leader, marksman, observer, scout, and a rearguard. The men selected for the first SWAT team further illustrate how the skills developed in foreign intervention influenced domestic police operations. According to the LAPD, each member of the original SWAT unit had specialized experience and prior military service. Moreover, the new SWAT team continued to incorporate new military tactics in counterinsurgency and guerrilla warfare, hiring military personnel to teach the SWAT unit. The use of SWAT teams throughout the country expanded rapidly as a result of the War on Drugs and War on Terror. As we have written previously, these conflicts served as catalysts to spread police militarization as local police departments became intertwined with the federal government's efforts to combat drugs and terror. This relationship between the political periphery (state and local governments) and the political center (the national government) allowed for the expansion of SWAT operations through the transfer of military-grade equipment and training. For example, the DOD 1208 Program, implemented in 1990, allowed the Department of Defense to transfer military equipment, such as aircraft, armor, watercraft, and weapons, to state and local police to use in their efforts to combat drugs. In 2013, a successor program, Program 1033, transferred almost $500 million in military weapons and gear to domestic law enforcement agencies for the purposes of fighting drugs and terror. The influx of military equipment into local law enforcement, combined with the adoption of military tactics like those employed by SWAT teams, created an arena in which the liberties and freedoms of U.S. citizens were jeopardized. Those most likely to suffer from these changes were those least likely to have the means to avoid the enhanced coercive power of the state—the poor, politically unconnected, and historically marginalized groups. Just as racial minorities are more likely to die while in police custody, so too are SWAT teams more likely to be used against minority groups. According to a recent report by the American Civil Liberties Union (ACLU), SWAT raids primarily impact persons of color. Between 2011 and 2012, approximately 50 percent of all SWAT raids were conducted against Black or Hispanic individuals while only 20 percent of raids involved white suspects. The difference is more pronounced when looking at particular types of SWAT raids. For example, some 68 percent of drug raids studied by the ACLU were conducted against minority suspects compared to a much lower rate for

Page 6: openev.debatecoaches.org€¦  · Web view1AC . Observation 1: If I could find the spot where truth echoesI would stand there and whisper memories of my children's future. I would

whites, even though rates of drug use and selling are similar across racial groups. In some localities, Blacks and Latinos are much more likely to be impacted by SWAT raids than their white counterparts. In Allentown, Pennsylvania, for example, Latinos are 29 times more likely to be affected by a SWAT raid than whites, while Blacks are 23 times more likely. Blacks are 37 times more likely to be the victim of a SWAT raid in Huntington, West Virginia, than their white counterparts. Blacks in Ogden, Utah, are 39 times more likely to be subjected to a SWAT raid and Blacks in Burlington, North Carolina, are 47 times more likely as compared to whites. The question remains why such groups are more likely to be disproportionally affected by police militarization. Albert Hirschman offers one explanation. He argues that individuals are faced with two options when they confront problems within organizations to which they belong, including governments. First, individuals may "exit," or withdraw from the relationship. Alternatively, they can "voice" their grievances in an attempt to address and repair the problem. For those groups most likely to be adversely affected by police militarization, however, both of these options may be weak or altogether nonexistent for a variety of reasons. For many, exiting a community where police militarization is prevalent may be unviable due to financial constraints. Consider that Hispanics are more than twice as likely, and Blacks almost three times more likely, to live in deep poverty as whites. Given these circumstances, the option to exit is not feasible for many. As a result, these individuals become even more vulnerable to state tools of social control. There is reason to believe that the "voice" mechanism is also weak for racial minorities. One study, for instance, foundthat increased racial segregation leads to a decrease in black civic efficacy. The authors note that relatively segregated black communities are often represented by politicians who fail to vote for policies favored by black constituents. Ferguson illustrates this concept well. While 67 percent of residents are black, there are nearly no Black political figures. Taken together, the lack of voice and exit opportunities means that minority groups are often the least able to avoid the adverse costs and consequences of militarized police forces. Foreign and domestic policies are often seen as distinct and separate. In reality, however, a proactive foreign policy (i.e., military intervention) generates unseen domestic costs, including the importation of techniques, methods, and tools of state-produced social control. The militarization of domestic policing is one illustration of how innovations in social control developed through foreign interventions can boomerang back to the homeland. When these innovations return home they lower the cost for the political elite of exerting control over the domestic populace. The costs of these expanded powers often fall disproportionally on those least able to move or effectively voice their dissatisfaction with government behaviors. What can be done? Changing the status quo requires a fundamental reconsideration of the scope and scale of government by U.S. citizens. Instead of remaining passive regarding the role of the state in their daily lives, citizens must become skeptical regarding the net benefits of the projection of state power, both domestically and abroad. This is especially important for minority and marginalized groups who are least able to avoid the abusive hand of the state. Unfortunately, many U.S. citizens currently view the state as a solution to their problems when in fact it is a contributing cause to many economic, political, and social ills.

Page 7: openev.debatecoaches.org€¦  · Web view1AC . Observation 1: If I could find the spot where truth echoesI would stand there and whisper memories of my children's future. I would

Like you shoot me now. I’m gonna start a riot! Us niggas are tired of letting this white ass society kill us everyday with no consequence. I ain’t seeing no policy do shit about this, them crackers aint ever gon care. We needa burn this shit down, just like how they did in Ferguson. YA HURD!!!Makalani’17(Minkah Makalani is associate professor of African and African diaspora studies at the University of Texas at Austin. "Black Lives Matter and the Limits of Formal Black Politics," The South Atlantic Quarterly 116:3 2017)NAE

Giorgio Agamben’s (1998: 10–11) discussion of “bare life” as reflecting not so much basic biological life but the premise of political life that reveals a condition to which “we are all potentially exposed” helps in unraveling the intimacies between modern democratic governance and totalitarian rule, although his turn to the Nazi camp as the paradigmatic example of the moment of collapse elides the very modes of coloniality that remain unre- marked in his

thinking. I will return to this point of Agamben’s thinking later. What I want to highlight here is that the declaration “Black lives matter” echoes a longer-running commentary on, and a rejection of, the protocols of sovereign rule, where thinking about the exception presents colonialism, plantation slavery, and genocidal violence in Africa, the Americas, and India as long-standing features of the modern. From this tradition of black thought, we can see the state of exception not as something arrived at and thus not a state of exception at all. Rather, it appears as something akin to what

Achille Mbembe (2001: 29–30) calls a régime d’exception, where there is not so much a departure from but an absence of common law, where political rule is designed merely to command subjects, rather than negotiate with citizens within a context of consensual governance. In such a setting, there is no regard for the means by which the modern, the rulers who command, upon encountering their constitutive other, generally in various “states of nature,” might bring about their civilization. For Mbembe, this involves a collapsing of the means and ends of rule as civilizing, which, as others have noted of modernity, may necessarily entail violence. We are therefore confronted with the question of what forms of black politics and political thought open up to us if we view an emergency as representing less the bringing about of an exception than that which illuminates an already operative exception. To draw on Walter Benjamin’s (1968: 257) observation that “the tradition of the oppressed teaches us that the ‘state of emergency’ in which we live is not the exception but the rule ,” what insights might we gain by viewing Ferguson, Baltimore, and Milwaukee as issuing from a tradition for which the

paradigmatic emergency was a prior, longer-standing reality? What if these protests reflect a form of political discourse largely unintelligible to black elites and the larger political structures , for whom this is merely lawlessness and rioting, yet still one whose perspectives insist, if we can follow Benjamin a bit further, that we “attain to a con- ception of history that is in keeping with this insight” (257)? It seems that it is primarily in thinking through black political thought that we find the conceptual tools for addressing these questions. Head, Apologies, and Apologia In discussing the problem of the human in

contemporary critical thought, Anthony Bogues (2012: 34) observes that “it is a critical task of radical thought today to work with the speech of those who have been historically excluded from the history of thought ,” a practice that requires “coming to grips with the speech and practices” of those peoples who are marginalized within or excluded from the social order as productive of thought. This involves taking seriously those claims, actions, and practices that seemingly depart from or stand outside of what is acceptable given prevailing social norms. I want to extend Bogues’s thinking, particularly his explicit departure from Agamben’s approach to the

exception through a foregrounding of the colonial, wherein he focuses on those who exist in a Fanonian zone of nonbeing. While I am not suggesting that Ferguson and Baltimore are colonial contexts, the sovereign practices of those municipalities nonetheless operate within the matrix of coloniality, where the black residents of Ferguson and Baltimore are those on whom “practices of violence [are] conducted that ma[ke] them not sites of exception but rather sites in which regularized performances of violence as power [are] enacted”

(Bogues 2012: 34). In other words, I am after how the actions and speech of those in Baltimore and Ferguson reveal

Page 8: openev.debatecoaches.org€¦  · Web view1AC . Observation 1: If I could find the spot where truth echoesI would stand there and whisper memories of my children's future. I would

the seeming exception as, in fact, the rule of rule. Within days of Ferguson police officer Darren Wilson’s shooting and killing of Brown, an eighteen-year-old unarmed black man who had recently graduated high school, local residents and activists from around the country descended on West Florissant Avenue in the St. Louis suburb. The response of the local police department was to have officers in military-grade riot gear and armored vehicles, using ash grenades and tear gas, confront protesters. One week after Brown’s body lay lifeless in the street for four and a half hours, after police repeatedly destroyed memorials that local residents had erected to Brown on the spot where he was killed, Missouri governor Jay Nixon declared a state of emergency in Ferguson. Although the state of emergency imposed a curfew that made it illegal to be out after midnight, protests continued, as did clashes with local and state police, even after Nixon called in the National Guard. Yet, in what many saw as an unprece- dented move, on November 17, 2014, Nixon declared another state of emer- gency, this time in anticipation of the St. Louis County grand jury’s decision on whether to indict Wilson. Some might have considered Nixon’s second state of emergency prescient. Indeed, on November 25, when it was announced that the grand jury had decided against handing down an indict- ment

of Wilson, Ferguson erupted. After witnessing their son’s body lying in the blistering August heat, handling his transition, burial, and an unimaginable level of grief, Brown’s mother, Lezley McSpadden , and stepfather, Louis Head , had to hear not only that there would be no indictment but, possibly more troubling, that St. Louis County prosecutor Robert McCulloch had purposefully undermined any possibility of an indictment. Their range of emotions came together in a moment of grief, anger, and rage when McSpadden, in tears and atop a car, addressed a crowd gathered outside the Ferguson courthouse. McSpadden made it clear that after having endured the loss of her son, she felt that the grand jury’s decision demonstrated a complete lack of care. “ They ain’t never gonna care. I been here my whole life, I ain’t never had to go through nothing like this,” she told the crowd . As McSpadden began to break down in tears, cries of “fuck the police” were mixed in with a young woman yelling, “ We love you, Ms. Lezley,” a uniquely Southern black form of address for an honored community member, especially a mother (McDonald 2014). McSpadden’s justifiable anger and indignation helps us bring into view the central paradox of turning to the state as the agent offering repair for the actions of the state . When CNN reporter Sunny Hostin asked McSpad- den why she addressed the crowd, she explained, “We heard this and it was just like, like I

had been shot. Like you shoot me now” (quoted in Ford and Levs 2014). McSpadden refused to consider police officer Wilson, county prosecutor McCulloch, and the grand jury as separate from one another or apart from the state . Police killings are not, in such a formulation, the action of individual police officers who represent the state but do not constitute the state as

such. Rather, they are considered parts of a whole. When McSpadden says, “Like you shoot me now,” she brings McCulloch’s handling of the grand jury into the same frame as Wilson, where she identifies both as the “you” who shot her son and her. The “you” here works to capture the state as the party causing injury. To confront such a scenario, one has to ask how a private “citizen” might reconcile the incongruity of turning to the state to carry out repair of an injury that the state caused. It was in this vortex of

emotion that Head climbed onto the car to comfort his wife and then screamed out, “ Burn this bitch down ,” delivering it at a pitch that many would all too easily explain away as an “excited utter- ance.” Yet Head repeated his incendiary plea eleven times, which, while unintentional, echoed Garner’s plea, also uttered eleven times, “I can’t breathe.” The resonance of Head’s “excited utterance” with Garner’s pleas is that it resists the disciplining impulse to render it as thoughtless anger. Indeed, Head acted somewhat deliberately. An Al Jazeera America camera recorded him saying

moments before he climbed onto the car, “If I get up there, I’m gonna start a riot. Burn this bitch down ” (quoted in America Tonight Digital Team 2014). As Head would later describe his “outburst,” “I screamed words that I shouldn’t have screamed in the heat of the moment” (quoted in Alcindor and Bacon 2014), an apparent apology born of the necessities of our political present, where the legitimacy of one’s grief and anger, one’s claim to injury, especially if one is black, depends on one’s expression of that grief on the terms sanctioned by the social order. Central to such a demand is that people design their expression of grief so as to cause minimal disruption to that order. Head, McSpadden, and their advisers certainly understood this in having Head issue an apology, particularly given the efforts to charge him with

inciting a riot. The response from black political elites revealed the continued sway of a civil rights approach that seeks to maintain a moral ground by putting forth the unimpeachable citizen exemplar of sacrifice. Such an approach requires a disavowal of Head’s “outburst” as a mistake, mere thoughtless anger. Rather

than attempt to explain away Head’s anger, I want to suggest that we can see in his shouting “burn this bitch down” an adequate response to his slain son and to Eric Garner’s pleas; his is an anger that, as if speaking to Frantz Fanon’s observation, “ we revolt simply because, for many reasons, we can no longer breathe ,” expressed

Page 9: openev.debatecoaches.org€¦  · Web view1AC . Observation 1: If I could find the spot where truth echoesI would stand there and whisper memories of my children's future. I would

a desire for the total destruction of the racial state. Doing so enables us to dwell on the possibilities that his defiant call holds for thinking through a mode of black politics that refuses the pre- vailing protocols and logics governing public demands for reparative justice . My interest is not to explain Head’s or any other seemingly

“extreme” expres- sion of anger as a departure from black politics. What we find in such expressions, which include the uprisings in Ferguson and Baltimore, is a critique that exceeds the sensibilities of liberal black political discourse precisely in their suggestion of the incompatibility , or possibly the constitutive exclusion, of the black as noncitizen , the paradigmatic other. By taking Head as engaged in political discourse, I want to see in his apology more than an attempt to negotiate the protocols of legitimate or acceptable speech. Given the social function of apology—admission of a violation and an agreement or promise to avoid any such future violation— those apologizing recognize their transgression of a social or political norm and locate guilt for that violation in their own actions or words.2 This was certainly part of what Head intended when he said, “I humbly apologize to all of those who read my pain and anger as a true desire for what I want for our community.” Still, we might better view his

statement as an apologia—a defense or justi cation of his “outburst.”3 Head went on: “But to place blame solely on me for the conditions of our community, and country, after the grand jury decision, goes way too far and is as wrong as the decision itself. To declare a state of emergency and send a message of war, and not peace, before a grand jury decision was announced is also wrong” (quoted in Alcin- dor and Bacon 2014). I want to consider Head’s apologia as centering on a critique of the racial state through his identification of Governor Nixon’s November 17, 2014, state of

emergency order as the context in which he expressed his pain and anger. The fundamental injury in this instance becomes the actions of the state itself, though not simply because the grand jury refused to indict Wilson, or even because county prosecutor McCulloch worked to ensure that the grand jury would not return an indictment. Whereas Agamben might consider Nixon’s two state of

emergency orders the moment an exception appeared, Head invites us to bring into view a longer-running exception already in place. As such, his original “ outburst” brings to mind not so much a desire to destroy property or “our community” but the institutional orderings of the local municipality in which black people experienced a perpetual exception . Head’s invective thus reflects a knowledge born from experiencing Ferguson as a régime d’exception. Unlike in Mbembe’s (2001) framing, where the departure from common law occurs when privileged individuals and companies constitute for themselves a “form of sovereignty,” in Ferguson there was no departure as such, as the posture that the municipality of Ferguson assumed toward its black residents was that of a sovereign toward its subjects who existed outside the compact of consensual governance. The Department of Justice’s (DOJ) report on Ferguson offers insight into how the city’s revenue-generating strategy allowed local police and courts to elaborate an exception. As the report outlines, generating revenue through policing black residents “fostered practices in the two central parts of Ferguson’s law enforcement system—policing and the courts—that are themselves unconstitutional or that contribute to constitutional violations” (DOJ 2015: 27). Where for the police these policies led to practices that “resulted in a pattern and practice of constitutional violations” (27) of black residents’ First and Fourth Amendment rights, the local courts assumed as their primary goal maximizing revenue rather than “administering justice or protecting the rights of the accused” (68). The DOJ (2016: 24) investiga- tion of the Baltimore police discovered a similar “pattern or practice of mak- ing stops, searches, and arrests in violation of the Fourth and Fourteenth Amendments” where they lacked “reasonable suspicion of people who [were] lawfully present on Baltimore streets,” which in turn “escalate[d] street encounters and contribute[d] to o cers making arrests without probable cause.” The DOJ’s (2015: 27) concern in both instances lay with the erosion of “police legitimacy and community trust” that made “policing in Ferguson less fair, less e ective at promoting public safety, and less safe,” a concern the DOJ (2016: 29) repeated in its Baltimore report, where it identi ed the practice of unlawful stops and searches as working to “erode public con - dence in law enforcement and escalate street encounters.” The DOJ reports on Ferguson and Baltimore take as a core concern the erosion of black trust in the police, where the level of distrust helps to esca- late encounters between black people and the police. This is no minor prob- lem. Amy E. Lerman and Vesla Weaver (2014) explain that this eroding trust in the police and the state more generally gives rise to what they call custo- dial citizenship, where citizenship is transformed from a mode of civic engagement through democratic structures into a series of maneuvers designed to minimize encounters with the state. This sense of needing to evade encountering the state certainly seemed to have guided Garner’s, Gray’s, and Brown’s respective encounters with the police o cers who killed them. The DOJ (2016) report on Baltimore is littered with incidents where black people breaking no laws attempt to evade and avoid encounters with the police. What the DOJ reports do not question, and it would have been surprising if they had, is whether the erosion of black people’s trust in the agents of the state, and thus the state itself, was of recent provenance. In light of Joel Olson’s (2004: xv) observation that “American democracy is a white democracy, a polity ruled in the interests of a white citizenry and char- acterized by simultaneous relations of equality and privilege . . . in relation to those who are not white,” we might question whether there is a basis for black citizenship to be anything other than custodial, with a corresponding fugitive orientation toward (or away from) state apparatuses.4 To put the question di erently, is it useful to approach the problem of the “erosion” of black trust in the state as one that the state can solve? To answer this question demands that we give attention to how the state approaches the issue of police violence. One feature of both the DOJ’s (2015: 48; 2016: 74) reports on Ferguson and Baltimore that deserves atten- tion is their reliance on the US Supreme Court’s 1989 Graham v. Connor decision that provides the legal framework for how to assess police o cers’ use of force. The key passage from that ruling (Graham v. Connor, 490 U.S. 386, 396 (1989)), quoted in both reports, holds that police o cers’ use of force must be judged based on whether “the o cers’ actions are ‘objectively

Page 10: openev.debatecoaches.org€¦  · Web view1AC . Observation 1: If I could find the spot where truth echoesI would stand there and whisper memories of my children's future. I would

reasonable’ in light of the facts and circumstances confronting them,” wherein the central element of such an assessment is “the perspective of a reasonable o cer on the scene,” rather than “the 20/20 vision of hindsight.” The part of the ruling that neither report references, however, holds that such an assessment is to be made “without regard to [the o cer’s] underly- ing intent or motivation.” The e ect of this ruling, which allows for the use of force if the o cer determines that a “suspect poses an immediate threat to the safety of the o cers or others,” is that the o cer is the sole body capable of determining whether force, and what degree of force, is necessary—“even if it may later seem unnecessary in the peace of a judge’s chambers” (Gra- ham, 490 U.S. at 396). Graham thus provides the legal justi cation for police o cers to exercise violence, but outside the judgment or oversight of any other governmental body; the police become, in e ect, the sovereign, and they are the ones who decide. This is not the suspension of law that Agam- ben describes as a moment that democracies arrive at—thus marking their historiographical distinction from totalitarian regimes. Rather, the context in which black people exist is itself the exception. Notwithstanding the insights we might gain from the DOJ’s investiga- tions into the Ferguson and Baltimore police departments, these reports fol- low a pattern whereby the state is the agent o ering repair for actions of the state. In finding fault with local police departments or municipal courts, the issue is rendered a strictly local one. That the police practices described in Ferguson and Baltimore are also found in Philadelphia, Cleveland, New Orleans, and Milwaukee (all police departments under consent decrees) never seems to present the possibility that what the DOJ has “uncovered” in any of these departments is less an aberration from the norm than the norm itself. Indeed, rarely have such ndings resulted in

the prosecution of local police o cers for corruption. Under Graham, police o cers are virtually unprosecutable. We are confronted, then, with the Ferguson and Baltimore uprisings mounting what we might describe as an ontological challenge to the guiding premise of black politics as a liberal engagement that pursues redress so that black life can matter within US liberal democracy. Rather than the suspension of law during an emergency, or an aberration from legitimate modes of governance, what Ferguson and Baltimore bring into sharp relief is that the police, even before the juridical armature of Graham, exercised the sovereign right to kill. It is in this context that we should read Head’s “outburst,” his unacceptable, nonsensical speech, as making a fundamental claim about the nature of Western democracies. Head’s words reveal how Ferguson is less a zone of exception, an aberration arrived at, than an example of a constitutive element in the operation of democracy. Head thus rejects a politics largely rejected in dominant black political discourse, yet one that suggests something of the nature of US democratic governance as constituted through an exception.

CIA tryna stop our niggas in Africa from shinin, shit! Dey tryna recolonize Africa with some shit called AFRICOM. Wtf are these niggas on? White as government wanna stop us from burning it down here and they tryna do the same shit in Africa. This AFRICOM shit is the same organization giving these motherfuckers guns thru the 1033 program. These niggas are really wild bro. they don’t wanna see us free.Freeman ‘18 (Netfa Freeman is an organiser in Pan-African Community Action, a member organization in the Black Alliance for Peace, as well as an analyst at the Institute for Policy Studies. Netfa was Director of the Institute’s Social Action & Leadership School for Activists (SALSA) from 2000 to 2010 and is now the coordinator for events of the other IPS projects. SALSA provided affordable workshops covering all aspects of grassroots activism, ” Dual US war on Black people,” https://www.pambazuka.org/pan-africanism/dual-us-war-black-people, 10/22/18)//pshah

Parallel tracks of United States government policy against the Black working class in the US and on the African continent expose much more than incidental similarity, but a concerted fatal conspiracy. For the US, African people globally have no economic value short of being unwitting consumers whose labour-use has expired, and whose resistance to social injustice must be repressed at all costs. This conflict of interests reveals a natural contradiction between North American versus African or Black identities.

Page 11: openev.debatecoaches.org€¦  · Web view1AC . Observation 1: If I could find the spot where truth echoesI would stand there and whisper memories of my children's future. I would

African-American on many levels is an oxymoron. This month marks the 10th anniversary of the United States Africa Command (AFRICOM), created on 1 October 2008. AFRICOM is the re-colonisation of Africa by the US and constitutes the new scramble for Africa equivalent to when, in the 1800s, the colonial powers fought over which of them would dominate which parts of the resource-rich continent. Pre-dating AFRICOM by ten years is its domestic counterpart, the “National Defense Authorisation Act of 1997” signed into law by Bill Clinton and more commonly known as the 1033 Programme. The 1033 Programme facilitates the transfer of excess US Department of Defense supplies and equipment to state and local law enforcement agencies, which are invariably used against Black and Brown communities in the US. The Programme has allowed police departments to acquire vehicles (land, air, and sea), weapons, computer equipment, fingerprint equipment, night vision equipment, radios and televisions, first aid equipment, tents and sleeping bags, photographic equipment and more. There is no more glaring proof that the US has been waging war against both Black people within its borders and those in Africa than a cursory examination of the responses by the US national security state to Black movements for decolonisation and self-determination inside the US and on the continent. A parallel history in form and essence unfolds when comparing what took place from the 1950s to the 70s in the US Black Power, Civil Rights movements with the independence, anti-colonial movement in Africa. Documented evidence vividly illustrates that the Federal Bureau of Investigation (FBI)’s infamous Counterintelligence Programme, also known as COINTELPRO, orchestrated operations to “infiltrate, intimidate, imprison, and assassinate” the leaders of Black movements for social justice in the US. In Africa, the US executed identical and chronologically aligned repression through the Central Intelligence Agency (CIA) against the independent African governments and liberation movements sweeping the continent. No matter where in the world African people are, our organising for social justice is treated as a threat to the political, economic, and cultural interest of the US ruling class that actually constitutes the essence of Americanism. Democratically elected leaders of the new African states were subjected to coup d’états and incessant assassination attempts including that of Kwame Nkrumahin Ghana, the successful assassination of Patrice Lumumba in the Congo, and today we can add the 2011 brutal murder of Muammar Gaddafi in Libya. The need to feed a growing and insatiable military-industrial complex and to guarantee that no radical Black movements emerge within US borders or on the African continent has given rise to increased and better-coordinated militarisation both in Africa through AFRICOM and Black communities in the US through the 1033 Programme. US state agents continue tantamount treatment, like spying on Black Lives Matter activists, monitoring their social media and creating the bogus FBI designation Black Identity Extremists to malign them as responsible for violence against police. All the while tolerating organised, criminal infiltration of law enforcement by violent white supremacists. AFRICOM is the US response to economic competition with China and its increased influence on the continent. AFRICOM is also to prevent the emergence of any independent African influence or force. It is not to fight drug trafficking or terrorism as stated in their promotional materials. The US military presence is a destabilising presence demonstrated by events like the 2012 overthrow of a democratically elected government in Mali by an AFRICOM trained Captain Amadou Haya Sanogo and the 2015 coup in Burkina Faso led by AFRICOM trained Colonel-Major Gilbert Diendere. During this month the Black Alliance for Peace has been rolling out its campaign US Out of Africa: Shut Down AFRICOM urging all peace and justice loving people to sign the campaign’s petition directed to the House Armed Services Committee and the Congressional Black Caucus. In his last year of office, instead of doing what he could to abolish the 1033 Programme, Barack Obama put minor restrictions on it, which the Trump administration immediately reversed within its first year. The Bush administration, progenitor of AFRICOM, was rebuked across the African continent when attempting to establish the headquarters for AFRICOM on the continent, forcing the new command to work out of Europe. Then came the Obama administration that paved the way for the proliferation of AFRICOM on the continent, as quisling African leaders fell over themselves to cooperate with the first Black US President. A now

Page 12: openev.debatecoaches.org€¦  · Web view1AC . Observation 1: If I could find the spot where truth echoesI would stand there and whisper memories of my children's future. I would

acceptable scenario has resulted in 46 various forms of US bases as well as military-to-military relations between almost all of the 54 African countries and the United States. US Special Forces troops now operate in more than a dozen African nations reflecting a 1,900 percent increase in the US military presence in Africa. What impact has increased militarisation abroad had on US Black and Brown communities? Since 1990, about US $6 billion worth of US Department of Defense property has been transferred to local, state, federal and tribal law-enforcement agencies while communities are suffering from austerity cuts. The world saw this deployed against Black rebellions in response to the police shooting of an unarmed Michael Brown in Ferguson, Missouri. The first actual militarisation of the US police started 28 years before the 1033 Programme in the 1960s with the creation of the Special Weapons And Tactics units –commonly known as SWAT. In South Africa (Azania) the same thing was developed at exactly the same time. The Special Task Force (STF) was an elite police tactical unit of the white settler regime’s South African Police Service (SAPS). The first significant deployments of SWAT and STF units were to repress African/Black movements for liberation. On 9 December 1969 SWAT was deployed for a four-hour confrontation with members of the Black Panthers in a densely populated area of Los Angeles. In 1967 about 2,000 STF forces were deployed to guard the northern border of Rhodesia (modern-day Zimbabwe) to assist Rhodesian security forces against the liberation forces there. Roger Morris, former National Security staffer for Henry Kissinger, admits that of the dozens of coup d’états that have taken place in post-colonial Africa only two or three were free of the hidden hand of US destabilisation. The sale of small arms to “Black Africa” was a policy initiated through Nixon’s National Security Study Memorandum 201: Increased Arms sales to Black Africa to stop the “threat of communism.” Today the conditions they have created are dishonestly used as the pretext for AFRICOM. Likewise in the US, it has been shown that the rise of so-called street gangs like the Bloods and Crips reflect a generation of radicalised youth misguided after a decimated 1960s Black Power movement. In spite of gun control laws like California’s Mulford Act supported even by the National Rifle Association to curtail the Black Panther Party’s armed stand to defend the Black community from police terror, by the 1980s semi-automatic weaponry and other sophisticated small arms found its way into US Black communities and countries in Africa. US officials reject as unauthentic the “Study Response To Presidential Security Review Memorandum NSC-46” that outlines a deliberate and sophisticated policy of fostering political divisions between ”Black Africa and the US Black Movement.” Measures like: 1. Specific steps should be taken with the help of appropriate government agencies to inhibit coordinated activity of the Black Movement in the United States. 2. Special clandestine operations should be launched by the CIA to generate mistrust and hostility in American and world opinion against joint activity of the two forces… 3. US embassies to Black African countries specially interested in southern Africa must be highly circumspect in view of the activity … opposing the objectives and methods of US policy toward South Africa…” are all arguably what has been US policy. The double standards and hypocrisy of the US government are clear, making it imperative that the ideological schizophrenia perpetuated by Americanism is broken. A mass movement must emerge that exposes AFRICOM, confronting the powers that be about it; and that makes it inseparable from the concerns we have over the militariatsion in our communities in the United States. All peace and justice loving people can start by signing the US Out of Africa! Shut Down AFRICOM petition, spreading it to others and then getting involved.

The feds should stop tripping and reduce the sales of arms used to exterminate niggas.

Page 13: openev.debatecoaches.org€¦  · Web view1AC . Observation 1: If I could find the spot where truth echoesI would stand there and whisper memories of my children's future. I would

Observation 3 is re-taking yourself.

Let’s engage in a lil bit of namin and claimin. We are word warriors—learnin from the shonuff sista from the hood, Docta G, who is cognizant of what it means to be African. Nommo operates within performative “lanuaging”, a space of agency, contestation, and combat. We member where we came from, we got no desire to front cuz our medium is the message. Dis is a question of linguistic freedom, we ain’t no prisoners of da academy. You dig what I’m saying? Can I get an Amen? We fightin against linguistic erasure Ya feel Me?Yancy ’04 (George Yancy, George Dewey Yancy is an American philosopher who has been a professor of philosophy at Emory University since fall 2015. He is also a distinguished Montgomery Fellow at Dartmouth College, 2004, The Journal of Speculative Philosophy, New Series, Volume 18, Number 4, pp. 273-299, “Geneva Smitherman: The Social Ontology of African-American Language, the Power of Nommo, and the Dynamics of Resistance and Identity Through Language”) \\EG

It is my contention that African American linguist Geneva Smitherman is working within the rich situated practices of Africana philosophy and should be acknowledged as such. She is self-consciously aware of the metareflective analysis that is necessary to make sense of what it means for Black people to have forged

an identity through the muck and mire of white racism. After all, Black folk were deemed inherently inferior , cultureless , without Geist. Yet, Black agency survived the tortuous African blood-stained water of the Atlantic. Like Jazz (with its improvisatory

structure and chromatic form), the Blues (with its ontology of lyrically holding at a distance incredible pain and sorrow), and rap music (with its phat

beats, lyrical braggadocio, and in yo face street reality), African American Language is a significant site of Black cultural innovation, syncretism, and survival , laden with situated epistemological insights . There is no other way to honor

the work of Geneva Smitherman, to explore the “ language- gaming ” of everyday Black folk , without directly and unapologetically

entering into the dynamic, rhythmic, ritual, and cognitive spaces of African American linguistic expressiveness. Hence, from the very giddayup, that is, befo I bees gittin into some really dope cultural, historical, philosophical, and linguistic analyses , let’s engage in a lil bit of naming and claiming . Word! The power of Nommo . Geneva Smitherman (a.k.a. Docta G) is an educational activist, a word warrior, a language rights fighter , a linguist-activist, and a linguistic democratizer. Can I get a witness? Yeah, that’s right. She is the legitimizer of African American Language ( AAL ) (Smitherman

2001, 347). The shonuff sista from the hood who is cognizant of what it means to be a New World African, to be linked to that shonuff Black space of talkin and testifyin, stylin and profilin. AAL is the language of her nurture (343). She was, after all, baptized “in the linguistic fire of Black Folk” (Smitherman 1997, 242). Believe me, for if I’m lyin, I’m flyin, she knows the source of those deleted copulas (“The coffee cold”), those post-vocalic –r sounds (“My feet be tied,” not “tired”), redundant past

tense markings (“I likeded her,” not “liked”), few consonant pairs (“Those tesses was hard,” not “tests”), stylizations, and rhetorical devices. Docta G operates within that unique African American space of performative “languaging,” a space of agency, contestation, self-definition , poiesis, and hermeneutic combat . She is all up in the cultural sphere of ashy knees, nappy hair, and how we be actin so saddity. Ah, yes, and she knows about the hot comb as a cultural artifact of self-hatred, a form of hatred instilled through the

power of colonial white aesthetics. She member where she come from. She got no desire to front . Docta G’s medium is the message. She avoids what linguist- philosopher John L. Austin refers to as the “descriptive fallacy,” which involves the assumption that the main

Page 14: openev.debatecoaches.org€¦  · Web view1AC . Observation 1: If I could find the spot where truth echoesI would stand there and whisper memories of my children's future. I would

function of language is to describe things. Through the incorporation of AAL flava in her written works (and no doubt in her lectures), Docta G is doing something

with those words and phrases. Her writings, in short, are demonstrative e nactments of the historical, stylistic, political, communicative, cognitive, and social ontological power of African American Language. Docta G is the lion who has learned how to write, how to narrate a counterhistorical narrative , and how to recognize and theorize a counterlanguage. A metalinguist , she is a cultural, ethical, and political theorist . If push comes to shove, she’ll “choose goodness over grammar” (349). She knows that the politics of language policy is a larger question of the politics of reality construction, historical structuring of society, race, class, and Anglo-linguistic imperialism. As such, she moves between both the sociolinguists (who stress social and ethnic

language) and the Cartesian or Chomskyan linguists (who stress “ideally competent” language). She knows that the right to speak AAL is a question of linguistic freedom, agency, and justice . A womanist, moving within that bold, self-assertive , and we-affirming space of sistas like Angela Davis, Alice Walker, Patricia Hill Collins, Sojourner Truth, and Harriet Tubman, Docta G is responsible, in charge, and serious. She’s no prisoner of the academy ; rather, she is existentially and politically committed to the Black community, its survival, and the continual actualization of its cultural generative force. Smitherman maintains that a womanist denotes an “African-American woman who is rooted in the Black community and committed to the development of herself and the entire community” (Smitherman 1996, 104). African American women, empowered by their womanist consciousness, were well-aware that white feminists had failed to critique, self-reflexively, the normativity of their own whiteness. Epistemologically, Black womanists occupied their own subject positions, positions that did

not square with the theorizations of white middleclass women. You dig (Wolof: dega) what I’m sayin? Can I get an Amen? Docta G,

“daughter of the hood,” raised in Brownsville, Tennessee, was culturally immersed in the rich locutionary acts of Black folk (Smitherman 2000, xiii). Y’all wit me? I’m pointing to the significant links between Docta G’s biographical location and how this influenced her later theorizations about AAL. Consistent with feminist, womanist, social constructionist, and postmodernist insights, one’s social location is a significant hermeneutic lens through which to understand one’s

theorizations. By emphasizing one’s social location, one is able to avoid the obfuscating process of reification . As social constructionists Peter L. Berger and Thomas Luckmann maintain, “Reification is the apprehension of human phenomena as if they were things, that is, in non-human or possibly suprahuman terms” (Berger and Luckmann 1995, 36). In short, the lived-context, as existential phenomenology stresses, is always already presupposed in relationship to epistemological claims. Docta G, daughter of the Black ghetto, daughter of Reverend Napoleon, was an early witness to the

illocutionary and perlocutionary acts of a linguistically rich Black family, a sharecropping community, and a Traditional Black Church. For example, she knows the power of tonal semantics, a significant feature of AAL that moves the listener through melodic structure and poignant rhetorical configuration . She relates that her father once expressed the following theme in one of his sermons: “I

am nobody talking to Somebody Who can help anybody” (Smitherman 2001, 222). Geneva, monolingual and from the sociolinguistic margins, was well-aware of what it meant to be deemed a problem, to endure the pain of being told that her speech was “ pathological ,” “ wrong ,” “ inferior ,” “bad,” and “derelict.” Having gone North (or was it simply up South?) where she attended college, Geneva had to pass a test “in order to qualify for the teacher preparation program” (1). Given the then oppressive and racist language policy, a policy that

stressed the importance of teachers being able to speak the language of those who “carry on the affairs of the English-speaking people” (2), Geneva did not pass the speech test . Docta G explains: We found ourselves in a classroom with a speech therapist who wasn’t sure what to do with us. Nobody was dyslexic. No one was aphasic. There was not even a stutterer among us. I mean here was this young white girl, a teaching assistant at the university, who was just trying to get her Ph.D., and she was presented with this perplexing problem of people who didn’t have any of the communication disorders she had been trained to deal with. (2) Although Geneva eventually passed the test by simply memorizing the pronunciation of particular sounds she needed to focus on, she came to interpret this experience as key to stimulating her politico-linguistic consciousness. She relates that “it aroused the fighting spirit in me, sent me off into critical linguistics and I eventually entered the lists of the language wars” (3). Clearly, this experience created in Geneva a powerful sense of agency and praxis. On the strength! You know it. As Kenyan philosopher and literary figure Ngu~gi~ wa Thiong’o argues, “There is no history which is purely and for all time that of actors and

those always acted upon” (Thiong’o 1993, 131). Uhm talkin bout a Womanist, Docta G. You know, the chief expert witness for the linguistic intelligence of Black children, the one who has made it her political project to challenge effectively the totalizing systems of Euro /Anglo- linguistic cultural normativity . You betta act like you

know. She fightin against African American linguistic erasure . Naw, even more so , she fightin for African American hue-manity . Docta G reveals that AAL is not some broken, ersatz sign system relegated to the confines of ghetto life; rather, AAL is the language of Black America (Smitherman 2001, 350). Docta G is up on it; she operates in that deeply deep space of African American signs and symbols, a semiotic space where individual words and phrases carry the weight of an entire world -view . As Frantz Fanon asserts, “To speak a language is to take on a world, a culture” (Fanon

1967, 38). I’m talkin bout an entire life-world where folk gotta live under conditions of much oppression , at the bottom, where Black bodies and souls constantly struggle to move within a compressing and collapsing social cosmo s. Sendin out an S.O.S. call . There appears to be a blue shift in the Black existential universe. But as we move to the center of it all, to the heart and soul of these historicizing, proud Black people , we notice a dynamic process of reconstitution, reinterpretation , being, and becoming. We be a praxis oriented

Page 15: openev.debatecoaches.org€¦  · Web view1AC . Observation 1: If I could find the spot where truth echoesI would stand there and whisper memories of my children's future. I would

people who are defined by our communicative acts, our existential improvisatory modes of being, forms of world-making, and ways of re-narrating , over and over again, our historical and spiritual links to Africa and the Americas.

Our black knowledge and language aint okay so they want us to use their white ass language that dem white slaveowners forced our ancestors to learn. We say fuck the colonizers language and imma use my nigga language to fuck up dem power structures that see my language as a problem, and let my presence be known, FUCK 12! Get w the medium g ya feel me?Yancy ’04 (George Yancy, George Dewey Yancy is an American philosopher who has been a professor of philosophy at Emory University since fall 2015. He is also a distinguished Montgomery Fellow at Dartmouth College, 2004, The Journal of Speculative Philosophy, New Series, Volume 18, Number 4, pp. 273-299, “Geneva Smitherman: The Social Ontology of African-American Language, the Power of Nommo, and the Dynamics of Resistance and Identity Through Language”) \\EG

It is here that one might ask: “Are Anglo-American and European philosophical forms of discourse inadequate for re-presenting the complexity of Black experiences?” After all, not any form of discursivity will do. My experiences were in excess of what “Standard” American English could capture. Some forms of knowledge become substantially truncated and distorted , indeed, erased, if not expressed through the familiar linguistic media of those who have possession of such knowledge. In a passage rich with issues concerning the lack of power and effectiveness of “Standard” American English to capture the personal identity and personal experiences of a young Black boy, writer R. DeCoy asks:

How … my Nigger Son, can you ever hope to express what you are, who you are or your experiences with God, in a language so limited , conceived by a people who are quite helpless in explaining themselves? How can you, my Nigger Son, find your identity , articulate your experiences, in an order of words ? (quoted in Brown 2001, 59–60) Regarding white philosophers (or even Black ones) who simply fail to understand the importance of African American

Language as a rich cultural and philosophical site of expression, I believe that it is the job of knowledgeable and responsible Black philosophers—at least for those who are willing to admit that they speak both the Language of Wider Communication (or LWC) and the powerful vernacular

shaped by African retentions and African American linguistic nuances—to invite them to enter African American semiotic spaces of discursive difference and overlap. We should keep in mind that being Black or African American in North America does not ipso facto mean that one is familiar with the subtle complexities and power of African American Language. After all, there are Black philosophers from middle-class (and lower-class) backgrounds whose linguistic assimilation of “Standard” American English, a form of cultural capital ownership and privilege, functions both as a badge of white

acceptance and as an antidote for reducing white anxiety and fear. The invitation, though, should not be a plea, but an honest gesture to explore the language on its own terms. This is why it was so very important that this present article be written unapologetically in the language of my nurture: the medium had to be the message .

Keep in mind that an invitation is not the same as a forced introduction. This was the situation that Blacks of African descent faced; they were forced to learn the language of the colonizer , forced to split, to multiply in so many different cultural, psychological, linguistic, and spiritual directions against their will . The fact that this article appears in the Journal of Speculative Philosophy invites a certain level of cultural and linguistic splitting on the part of its readers, perhaps not very different from what is required when reading Kant or Heidegger, particularly given their penchant for neologism. Let’s be honest, articles that typically appear in the Journal of Speculative Philosophy have no doubt been written in “Standard” American English and by predominantly white male philosophers, philosophers who have been trained to engage in

“proper” philosophical prose. I, too, can write in this language. To write in this language is to reproduce the professional culture of philosophy, to perpetuate lines of power, and to show that you have been “properly” educated and worthy of hire. Moreover, to engage in this discourse is to perform linguistically before an audience of gatekeepers who probably fear too much fat in their discourse, too much play, too much signifying, too much indirection, too much ambiguity, too much vagueness, too much concrete,

everyday reality. Like African American Language within the larger context of “Standard” American Language, by appearing in this journal this article also enters into a space of established norms of linguistic propriety, calling into question and perhaps rupturing the authority of “Standard” philosophical prose, that unhip discourse of professional philosophers. Of course, having this article published in this journal could turn out to be a curse or a blessing. Realizing the degree to which “proper” philosophical discourse is required by philosophy journals, and how such discourse in turn shapes and legitimates philosophy journals, many readers of the journal

Page 16: openev.debatecoaches.org€¦  · Web view1AC . Observation 1: If I could find the spot where truth echoesI would stand there and whisper memories of my children's future. I would

may read the article with contempt. Some may approach the article as a piece of exotica. Some may even view my use of African American Language as a disgraceful “Stepin Fetchit” performance that does a disservice to Black philosophers who are all too eager to perform well in the presence of white power, to show the world that we be real philosophers because we speak the language of Mister Charlie. Then, of course, there are some philosophers who are open to creative possibilities, differences, and alternatives to hegemonic linguistic territorialism, who believe in plural experiences and multiple discourses for articulating them. It’s

here, of course, that I got to give props to my man JJS for being hip to the importance of multiple philosophical voices and the different and complex existential spaces within which they emerge. When the medium is the message, one has got to get wit da medium. It will take more than this article to create a significant impact on a certain linguistic-philosophical reference point that is buttressed by so much history and power, but the Journal of Speculative Philosophy is a good place to begin, right in the midst

of professional philosophy-speak. To best articulate that Black existential space where the real world (not that abstract possible world) is filled with pain , struggle, blood , tears, and laughter— where death follows a minute of joy, where so much is improvisatory and surreal , moving with the quickness, where the streets are hot, dangerous , and familial, where love is abundant and hate smiles in yo face , where melodic sounds fight to stay above the sounds of gunfire , where babies cry all night long , because mama done gone and hit the pipe, where a brotha gots to be down, where brothas be runnin game, talkin that talk, keepin it real , and showin much loyalty—requires fluency in the language that partly grows out of the nitty-gritty core of the epistemology and ontology of that space.

Page 17: openev.debatecoaches.org€¦  · Web view1AC . Observation 1: If I could find the spot where truth echoesI would stand there and whisper memories of my children's future. I would

Vs. Africom + Other VersionsTheir place as the stakeholders of the movement against US imperialism qua AFRICOM fosters leftist white supremacy – their organizing turns in on micro-struggles against the sovereign force of the law by rendering the protest a fantasy space of white liberal pageantry which merely folds the AFF into state-recognition.King ‘14 Tiffany Lethabo King, Assisant Professor of Women’s, Gender and Sexuality Studies at Georgia State, PhD in American Studies from the University of Maryland at College Park. “Puppets, Pageantry and Protest Politics White People and the Anti-War Movement” from Our Culture, Our Resistance: People of Color Speak Out on Anarchism, Race, Class and Gender, 2014) ipartman

Reflecting on my own participation as a person of color in the 2003 protest marches of the anti-war movement, I am now aware my presence is being manipulated and abused . I have been rendered a puppet for white liberal pageantry . Any time I have attended a march I find I subject myself to objectification, marginalization and exploitation. Beyond the personal offenses I have incurred, I now truly believe the presence of people of color in anti-war and other so called " global justice " protest marches led and organized by whites legitimizes tactics that undermine a true pursuit for justice . On February 15, 2003 grappling with my own frustration, anger and feelings of impotency as

our country charged towards war; I attended a protest march. The march was organized by the usual suspects, A.N.S.W.E.R, Not in Our Name, Unite for Peace and Justice and the other white led coalitions here in Philly. As the Bush Administration moved closer and closer to dropping the first bombs on Iraq , I caved in and decided I had to go regardless of who organized this thing. I thought to myself, that if there was ever a time to momentarily get over the racism of the white left and past scars from previous interactions, it was now. My justification for my attendance was we were facing war , and at least the illusion of a collective and unified voice would be "more powerful" than the efforts of isolated communities of color or my own for that matter. In that moment, I had just given into white supremacist systems of domination. White supremacy asserts its own agenda by absorbing , subverting and negating any dissent and resistance to its domination. The structures of white supremacy demand submission by professing to be both the norm and alternative. In the book, Black Anti-Ballistic

Missives, activist, author and poet Ewuare Osayande argues white supremacy is the critical component that the anti- war movement fails to address, thus rendering it irrelevant . The racist anti- war movement is devoid of any self-critical process to acknowledge or address how white supremacy contributes to the oppression of people of color within the movement, so how could it possibly have an analysis of how white supremacy oppresses people of color around the world? Given the reality the anti-war movement does not address white

supremacy, it is to be expected that people of color would be objectified and exploited at one of its protest marches. On February 15, the people of color contingency I attended the march with decided to create a feeder march that would join the larger march on

Broad Street. I assume this was an attempt to empower the pac who would be participating that day. We would temporarily march on our own terms , citing white supremacist imperialism as the true evil. Momentarily, I had the feeling that we would reclaim the political act of protest as a relevant and meaningful tool used by self-determining people of color around the world who have and continue to resist white supremacist imperialism. Yet I realized once I got to the march that this would not be possible. We would chant Black, Latino, Muslim, Asian and... so forth to evoke a feeling of camaraderie and equal partnership. Yet the unequal distribution of power and privilege amongst people of color, which played a part in determining the convening groups ability to organize this very effort, stared us in the face and was left unaddressed . Still,

we feigned a content and empowered united front and proceeded to Broad Street to meet up with the rest of the protestors. As soon as we

Page 18: openev.debatecoaches.org€¦  · Web view1AC . Observation 1: If I could find the spot where truth echoesI would stand there and whisper memories of my children's future. I would

approached the sea of white folks and they became aware of our presence we became a sideshow. White people started to clap and cheer as if we were the long awaited people of color parade float they could awe and point at. They appreciated our presence and danced to our drumming as long as we were their entertainment. However, as soon as members of the contingency started to pick up our bullhorns and speak to the white su- premacist imperialism that murdered people of color they became offended. We were suddenly the recipients of annoyed stares, shushes and interrup- tions because we were talking over the "slated" speakers. We kept on speaking, however our attempts to define and adhere to our own agenda did nothing. We did not create self-determining space that would allow our particular analysis of the war and white supremacy , as the global terrorist, to challenge the shallow, racist analysis of the white activists and

organizers. We were not even able to make the white folks aware of their own racism at the protest itself. We became a pawn, a mere prop, one of those larger than life puppets (often rendered to depict op- pressed people of

color) that white groups make for "protests." I don't know how many times I saw and heard white people look at us and say... "It's sooo good to finally see some color here." I could see them patting themselves on the back for the good work they had done to reach out to people of color and educate us or make us feel welcome to join them. Our presence only legitimized the work of whites , which is to stay in posi- tions of power and control the discourse , action and direction of so called progressive politics. By participating we allow them to delude themselves that , 1) their particular analysis of the war and imperialism is a legitimate one and 2) the power and resources at their disposal to lead " social change " movements are legitimately earned. Our presence at the march made the statement that we support the white supremacy of the left.

An Other Version.

The 1AC relies on a futural hope of a “not-yet-realized” future in which they can fantasize about contingent solutions that never come. This hope creates a cruel optimism for black folk that forces them to invest in the pursuit of our own death because it crowds out all non-politically recognizable alternatives. This model for politics only re-entrenches the anti-black world and is an independent reason to reject the aff. Warren 15 (Calvin L., Black Nihilism and the Politics of Hope; Surce: CR: The New Centennial Review, Vol. 15, No. 1, Derrida and French Hegelianism (Spring 2015), XMT, pp. 215-248 Published by: Michigan State University Press Stable URL: http://www.jstor.org/stable/10.14321/crnewcentrevi.15.1.0215 . Accessed: 30/03/2015)

The politics of hope , then, constitutes what Lauren Berlant would call “ cruel optimism” for blacks (Berlant 2011). It bundles certain promises about redress, equality, freedom, justice, and progress into a political object that always lies beyond reach. The objective of the Political is to keep blacks in a relation to this political object—in an unending pursuit of it. This pursuit, however, is detrimental because it strengthens the very anti-black system that would pulverize black being . The pursuit of the

Page 19: openev.debatecoaches.org€¦  · Web view1AC . Observation 1: If I could find the spot where truth echoesI would stand there and whisper memories of my children's future. I would

object certainly has an “irrational” aspect to it, as Farred details, but it is not mere means without expectation; instead, it is a means that

undermines the attainment of the impossible object desired. In other words, the pursuit marks a cruel attachment to the means of subjugation and the continued widening of the gap between historical reality and fantastical ideal. Black nihilism is a “demythifying” practice, in the Nietzschean vein, that uncovers the subjugating strategies of

political hope and de-idealizes its fantastical object. Once we denude political hope of its axiological and ethical veneer, we see that it operates through certain strategies: 1) positing itself as the only alternative to the problem of anti-blackness, 2) shielding this alternative [End Page 221] from rigorous historical/philosophical critique by placing it in an unknown

future, 3) delimiting the field of action to include only activity recognized and legitimated by the Political, and 4) demonizing critiques or different philosophical perspectives. The politics of hope masks a particular cruelty under the auspices of “happiness” and “life.” It terrifies with the dread of “no alternative .” “Life” itself needs the security of the alternative, and, through this logic, life becomes untenable without it.

Political hope promises to provide this alternative—a discursive and political organization beyond extant structures of violence and destruction. The construction of the binary “alternative/no-alternative” ensures the hegemony and dominance of political hope within the onto-existential horizon. The terror of the “no alternative”—the ultimate space of decay, suffering, and death—depends on two additional binaries: “problem/solution” and “action/inaction.” According to this politics, all problems have solutions, and hope provides the accessibility and realization of these solutions. The solution establishes itself as the elimination of “the problem”; the solution, in fact, transcends the problem and realizes Hegel’s aufheben in its constant attempt to sublate the dirtiness of the “problem” with the pristine being of the

solution. No problem is outside the reach of hope’s solution—every problem is connected to the kernel of its own eradication. The politics of hope must actively refuse the possibility that the “solution” is, in fact, another problem in disguised form; the idea of a “solution” is nothing more than the repetition and disavowal of the problem itself. The solution relies on what we might call the “trick of time”

to fortify itself from the deconstruction of its binary. Because the temporality of hope is a time “not-yet-realized,” a future tense unmoored from present-tense justifications and pragmatist evidence, the politics of hope cleverly shields its “solutions” from critiques of impossibility or repetition. Each insistence that these solutions stand up against the lessons of history or the rigors of analysis is met with the rationale that these solutions are not subject to history or analysis because they do not reside within the horizon of the “past” or “present.” Put differently, we can never ascertain the efficacy of the proposed solutions because they escape the temporality of the moment, always retreating to a “not-yet” and “could-be” temporality. This “trick” of time offers a promise of possibility that can only be realized in an indefinite future, and this promise is a bond of uncertainty that can never be redeemed, only imagined. In this sense, the politics of hope is an instance of the psychoanalytic notion of desire: its sole

purpose is to reproduce its very condition of possibility, never to satiate or bring fulfillment. This politics secures its hegemony through time by claiming the future as its unassailable property and excluding (and

devaluing) any other conception of time that challenges this temporal ordering. The politics of hope, then, depends on the incessant (re)production and proliferation of problems to justify its existence. Solutions cannot really exist within the politics of hope, just the illusion of a different order in a future tense . The “trick” of time and political solution converge on the site of “action.” In critiquing the politics of hope, one encounters the rejoinder of the dangers of inaction. “But we can’t just do nothing! We have to do something.” The field of permissible action is delimited and an unrelenting binary between action/ inaction silences critical engagement with political hope. These exclusionary operations rigorously

reinforce the binary between action and inaction and discredit certain forms of engagement, critique, and protest. Legitimate action takes place in the political—the political not only claims futurity but also action as its property. To “do something” means that this doing must translate into recognizable political activity; “something” is a stand-in for the word “politics”—one must “do politics” to address any problem. A refusal to “do politics” is equivalent to “doing nothing”—this nothingness is constructed as the antithesis of life, possibility, time, ethics, and morality (a “zero-state” as Julia Kristeva [1982] might call it).

Black nihilism rejects this “trick of time” and the lure of emancipatory solutions. To refuse to “do politics” and to reject

Page 20: openev.debatecoaches.org€¦  · Web view1AC . Observation 1: If I could find the spot where truth echoesI would stand there and whisper memories of my children's future. I would

the fantastical object of politics is the only “hope” for blackness in an antiblack world .

The only possible demand is one that calls for the end of the world itself—the affirmative represents a conflict within the paradigm of America but refuses to challenge the foundational antagonism that produces the violence that undergirds that same paradigm Wilderson, ’10 [2010, Frank B. Wilderson is an Associate Professor of African-American Studies at UC Irvine and has a Ph.D. from UC Berkeley, “Red, White & Black: Cinema and the Structure of U.S. Antagonisms,”]

Leaving aside for the moment their state of mind, it would seem that the structure , that is to say the rebar, or better still the grammar of their demands —and, by extension, the grammar of their suffering—was indeed an ethical grammar. Perhaps their grammars are the only ethical grammars available to modern politics and modernity writ large , for they draw our attention not to the way in which space and time are used and abused by enfranchised and violently powerful interests, but to the violence that underwrites the modern world ’ s capacity to think, act, and exist spatially and temporally . The violence that robbed her of her body and him of his land provided the stage upon which other

violent and consensual dramas could be enacted. Thus, they would have to be crazy , crazy enough to call not merely the actions of the world to account but to call the world itself to account , and to account for them no less! The woman at Columbia was not demanding to be a participant in an unethical network of distribution: she was not demanding a place within capital, a piece of the pie (the demand for her sofa notwithstanding). Rather, she was articulating a triangulation between, on the one hand, the loss of her body, the very dereliction of her corporeal integrity, what Hortense Spillers charts as the transition from being a being to becoming a “being for the captor” (206), the drama of value (the stage upon which surplus value is extracted from labor power through commodity production and sale); and on the other, the corporeal integrity that, once ripped from her body, fortified and extended the corporeal integrity of everyone else on the street. She gave birth to the commodity and to the Human, yet

she had neither subjectivity nor a sofa to show for it. In her eyes, the world — and not its myriad discriminatory practices, but the world itself — was unethical . And yet, the world passes by her without the slightest inclination to stop and disabuse her of her claim . Instead, it calls her “crazy.” And to what does the world attribute the Native American man’s insanity? “He’s crazy if he thinks he’s getting any money out of us”? Surely, that doesn’t make him crazy.

Rather it is simply an indication that he does not have a big enough gun . What are we to make of a world that responds to the most lucid enunciation of ethics with violence ? What are the foundational questions of the ethico-political? Why are these questions so scandalous that they are rarely posed politically, intellectually, and cinematically—unless they are

posed obliquely and unconsciously, as if by accident? Return Turtle Island to the “Savage.” Repair the demolished subjectivity of the Slave . Two simple sentences, thirteen simple words, and the structure of U.S. ( and perhaps global ) antagonisms would be dismantled . An “ ethical modernity ” would no longer sound like an oxymoron . From there we could busy ourselves with important conflicts that have been promoted to the level of antagonisms: class struggle, gender conflict, immigrants rights. When pared down to thirteen words and two

sentences, one cannot but wonder why questions that go to the heart of the ethico-political , questions of political ontology, are so unspeakable in intellectual meditations, political broadsides , and even socially and politically engaged feature films. Clearly they can be spoken, even a child could speak those lines, so they would pose no problem for a scholar,

an activist, or a filmmaker. And yet, what is also clear—if the filmographies of socially and politically engaged directors, the archive of progressive scholars , and the plethora of Left-wing broadsides are anything to go by — is that what can so easily be spoken is now (five hundred years and two hundred fifty million Settlers/Masters on) so ubiquitously unspoken that these two simple sentences, these thirteen words not only render their speaker “ crazy ” but become themselves impossible to imagine . Soon it will be forty years since radical politics, Left-

leaning scholarship, and socially engaged feature films began to speak the unspeakable. In the 1960s and early 1970s the questions asked by radical politics and scholarship were not “ Should the U.S. be overthrown? ” or even “ Would it

Page 21: openev.debatecoaches.org€¦  · Web view1AC . Observation 1: If I could find the spot where truth echoesI would stand there and whisper memories of my children's future. I would

be overthrown? ” but rather when and how —and, for some, what—would come in its wake. Those steadfast in their conviction that there remained a discernable quantum of ethics in the U.S. writ large (and here I am speaking of everyone from Martin Luther King, Jr., prior to his 1968 shift, to the Tom Hayden wing of SDS, to the Julian Bond and Marion Barry

faction of SNCC, to Bobbie Kennedy Democrats) were accountable , in their rhetorical machinations, to the paradigmatic zeitgeist of the Black Panthers, the American Indian Movement, and the Weather Underground . Radicals and progressives could deride , reject, or chastise armed struggle mercilessly and cavalierly with respect to tactics and the possibility of “ success, ” but they could not dismiss revolution-as-ethic because they could not make a convincing case —by way of a paradigmatic analysis—that the U.S. was an ethical formation and still hope to maintain credibility as radicals and progressives . Even Bobby Kennedy (a U.S. attorney general and

presidential candidate) mused that the law and its enforcers had no ethical standing in the presence of Blacks . One could (and many did) acknowledge America’s strength and power. This seldom, however, rose to the level of an ethical assessment, but rather remained an assessment of the so-called “ balance of forces. ” The political discourse of Blacks, and to a lesser extent Indians, circulated too widely to credibly wed the U.S. and ethics. The raw force of COINTELPRO put an end to this

trajectory toward a possible hegemony of ethical accountability. Consequently, the power of Blackness and Redness to pose the question — and the power to pose the question is the greatest power of all — retreated as did White radicals and progressives who “ retired ” from struggle. The question ’ s echo lies buried in the graves of young Black Panthers, AIM Warriors, and Black Liberation Army soldiers , or in prison cells where so many of them have been rotting (some in solitary confinement) for ten, twenty, thirty years, and at the gates of the academy where the “ crazies ” shout at passers-by . Gone are not only the young and vibrant voices that affected a seismic shift on the political landscape , but also the intellectual protocols of inquiry, and with them a spate of feature films that became authorized, if not by an unabashed revolutionary polemic, then certainly by a revolutionary zeitgeist . Is it still possible for a dream of unfettered ethics, a dream of the Settlement and the Slave estate ’ s destruction , to manifest itself at the ethical core of cinematic discourse, when this dream is no longer a constituent element of political discourse in the streets nor of intellectual discourse in the academy? The answer is “no” in the sense that, as history has shown, what cannot be articulated as political discourse in the streets is doubly foreclosed upon

in screenplays and in scholarly prose; but “yes” in the sense that in even the most taciturn historical moments such as ours, the grammar of Black and Red suffering breaks in on this foreclosure , albeit like the somatic compliance of hysterical symptoms—it registers in both cinema and scholarship as symptoms of awareness of the structural antagonisms. Between 1967 and 1980, we could think cinematically and intellectually of Blackness and Redness as having the coherence of full-blown discourses. But from 1980

to the present, Blackness and Redness manifests only in the rebar of cinematic and intellectual (political) discourse, that is, as unspoken grammars . This grammar can be discerned in the cinematic strategies (lighting,

camera angles, image composition, and acoustic strategies/design), even when the script labors for the spectator to imagine social turmoil through the rubric of conflic t (that is, a rubric of problems that can be posed and conceptually solved) as opposed to the rubric of antagonism ( an irreconcilable struggle between entities , or positionalities, the resolution of which is not dialectical but entails the obliteration of one of the positions ). In other words, even when films narrate a story in which Blacks or Indians are beleaguered with problems that the script insists are conceptually coherent (usually having to do with poverty or the absence of “family values”), the non-narrative, or cinematic, strategies of the film often disrupt this coherence by posing the irreconcilable questions of Red and Black political ontology—or non-ontology.

The grammar of antagonism breaks in on the mendacity of conflict . Semiotics and linguistics teach us that when we

speak, our grammar goes unspoken. Our grammar is assumed. It is the structure through which the labor of speech is possible. Likewise, the grammar of political ethics —the grammar of assumptions regarding the ontology of suffering — which underwrite Film Theory and political discourse (in this book, discourse elaborated in direct relation to radical action), and which underwrite

cinematic speech (in this book, Red, White, and Black films from the mid-1960s to the present) is also unspoken . This notwithstanding,

film theory, political discourse , and cinema assume an ontological grammar, a structure of suffering . And the structure of suffering which film theory, political discourse, and cinema assume crowds out other structures of suffering, regardless of the sentiment of the film or the spirit of unity mobilized by the political discourse in

Page 22: openev.debatecoaches.org€¦  · Web view1AC . Observation 1: If I could find the spot where truth echoesI would stand there and whisper memories of my children's future. I would

question. To put a finer point on it, structures of ontological suffering stand in antagonistic, rather then conflictual, relation to one another (despite the fact that antagonists themselves may not be aware of the ontological positionality from which they speak). Though this is perhaps the most controversial and out-of-step claim of this book, it is, nonetheless, the foundation of the close reading of feature films and political theory that follows.

Blackness is the tool of civil society to avoid blame for everything it fucks up.Warren ’18 (Calvin Warren, Calvin Warren is an Assistant Professor in WGSS. He received his B.A. in Rhetoric/Philosophy (College Scholar) from Cornell University and his MA and Ph.D. in African American/American Studies from Yale University, 2018, Ontological Terror: Blackness, Nihilism, and Emancipation) \\EG

This lack of properness and metaphysical truth is a symptom of the nothing, for nothing lacks any proper place in metaphysics and cannot be

understood through its episteme. Black being as nothing, then, will always be out of place and improper in an antiblack world. It is the terror of the metaphysical infrastructure, and one can never be a true or proper man when one bears the weight of nothing. Through this analysis, we can understand the anxiety concerning black being, placement, and nothing in antebellum culture. In August 1842, for example, the free black population of Philadelphia held a parade commemorating the abolition of slavery in the West Indies.

A n angry mob of white citizens disrupted the parade, attacked participants, and commenced to destroy black homes and property. Seeking redress through the courts for loss of property and injury, the free black population realized that justice within such a context was impossible, as the grand jury acquitted the rioters and blamed free blacks for inciting this violence. Robert Purvis, a leader in the free black population of Philadelphia, responded to the grand jury’s decision with dismay:49 “The measure of our suffering is full. . . . From the most painful and minute investigation, in the feelings, views and acts of this community — in regard to us — I am convinced of our utter and complete nothingness in public estimation [emphasis mine].”50 What sparked the riot, this devastating expression of antiblackness? We can locate this eruption of

violence at the metaphysical fault line between necessity and hatred. Black being is both a necessary instrument for the human’s self-constitution and an object of ferocious hatred , since it bears the nothing of a metaphysical order. In other words, the riot is the symptom of a metaphysical problem: the public celebration of black freedom sparks a terror in that ontological boundaries are challenged and the transformation from black being, as invention/instrument, to human being, as free, is not only

considered but celebrated. It is also no surprise that the grand jury blamed the victims for the riot, since black freedom is a form of violence for the human , a violence that must be met with extreme force. The riot is a response to ontological terror. “Free,” when paired with “black,” is recast as a weapon against the human and the metaphysical structure that sustains the human. We are dealing with two registers of violence — one is an ontological violence and another is a physical form of antiblack destruction. But Purvis’s response to the violence is perspicuous. He is “convinced of our utter and complete nothingness in public estimation.” If we read this statement as a mere political lamentation, that blacks constitute a political cypher (nothingness) within the law and political processes, then we limit our understanding of the riot as event. The riot, within this reading, is just a form of cruelty or irrational intolerance or a political-economic strategy of subjection. With political readings of antiblack violence, violence is not gratuitous but must be linked to some type of recognizable transgression; when antiblack violence cannot be linked to recognizable transgression, it is considered cruel or irrational — a form of individual pathology and not systemic necessity. If, however, the essence of politics is nothing political, then we might read Purvis’s political commentary as a response to the proper metaphysical question.

His answer is that black being is nothingness in public estimation. We can understand nothingness as the condition (-ness) of

bearing nothing in an antiblack world. Antiblack violence, then, constitutes the structure of this nothing. Black being is always already under attack ; peace, within an antiblack world, is a fallacy (much like freedom). The metaphysical infrastructure

that supports the fiction of the white human is sustained by antiblack violence. The riot is an ontological necessity , not just political cruelty. We can understand the grand jury’s decision philosophically: Being black is both the cause and effect of violence , and when this being claims freedom, extreme violence is always justified and necessary.

The alternative is black nihilism --- refusing political hope is the only metaphysically coherent response to the constant slaughter of black bodies Warren 15 [Calvin K., Assistant Professor of American Studies at George Washington University, “Black Nihilism and the Politics of Hope,” CR: The New Centennial Review, Volume 15, Number 1, Spring 2015]

Page 23: openev.debatecoaches.org€¦  · Web view1AC . Observation 1: If I could find the spot where truth echoesI would stand there and whisper memories of my children's future. I would

V. Conclusion Throughout this essay, I have argued that the Politics of hope preserve metaphysical structures that sustain black suffering. This preservation amounts to an exploitation of hope—when the Political colonizes the spiritual principle of hope and puts it in the service of extending the “will to power” of an anti-black organization of existence. The Politics of hope, then, is bound up with metaphysical violence, and

this violence masquerades as a “solution” to the problem of anti-blackness. Temporal linearity, perfection, betterment, struggle, work, and utopian futurity are conceptual instruments of the Political that will never obviate black suffering or anti-black violence; these concepts only serve to reproduce the conditions that render existence unbearable for blacks . Political theologians and black optimists avoid the immediacy of black suffering, the horror of anti-black pulverization, and place relief in a “not-yet-but-is (maybe)-to-come-social order” that, itself,

can do little more but admonish blacks to survive to keep struggling . Political hope becomes a vicious and abusive cycle of struggle —it mirrors the Lacanian drive, and we encircle an object (black freedom, justice, relief, redress,

equality, etc.) that is inaccessible because it doesn’t really exist. The political theologian and black optimist, then, propose a collective Jouissance as an answer to black suffering—finding the joy in struggle, the victory in toil, and the

satisfaction in inefficacious action. We continue to “struggle” and “work” as black youth are slaughtered daily, black bodies are incarcerated as forms of capital, black infant mortality rates are soaring, and hunger is disabling the bodies, minds, and spirits of desperate black youth. In short, these conditions are deep metaphysical problems—the sadistic pleasure of metaphysical domination—and “work ” and “struggle” avoid the terrifying fact that the world depends on black death to sustain itself . Black nihilism attempts to break this “drive”—to stop it in its tracks, as it were—and to end the cycle of insanity that political hope perpetuates. The question that remains is a question often put to the black nihilist: what is the point? This compulsory geometrical structuring of thought— all knowledge must submit to, and is reducible to, a point —it is an epistemic flicker of certainty, determination, and, to put it bluntly, life. “The point” exists for life; it enlivens, enables, and sustains knowledge. Thought outside of this mandatory point is illegible and useless. To write outside of the “episteme of life” and its grammar will require a position outside of this point , a position somewhere in the infinite horizon of thought (perhaps this is what Heidegger wanted to do with his reconfiguration of thought). Writing in this way is inherently subversive and refuses the geometry of thought. Nevertheless, the [End Page 243] nihilist is forced to enunciate his refusal through a “point,” a point that is contradictory and paradoxical all at once. To say that the point of this essay is that “the point” is fraudulent—its promise of clarity and life are inadequate—will not satisfy the hunger of disciplining the nihilist and insisting that one

undermine the very ground upon which one stands. Black nihilistic hermeneutics resists “the point ” but is subjected to it to have one’s voice heard within the marketplace of ideas. The “point” of this essay is that political hope is pointless . Black suffering is an essential part of the world, and placing hope in the very structure that sustains metaphysical violence , the Political, will never resolve anything . This is why the black

nihilist speaks of “exploited hope,” and the black nihilist attempts to wrest hope from the clutches of the Political . Can we think of hope outside the Political? Must “salvation” translate into a political grammar or a political program? The nihilist , then, hopes for the end of political hope and its metaphysical violence. Nihilism is not antithetical to hope; it does not extinguish hope but reconfigures it. Hope is the foundation of the black nihilistic hermeneutic. In “Blackness and Nothingness,” Fred Moten (2013) conceptualizes blackness as a “pathogen” to metaphysics, something that has the ability to unravel, to disable, and to destroy anti-blackness. If we read Vattimo through Moten’s brilliant analysis, we can suggest that blackness is the limit that Heidegger and Nietzsche were really after. It is a “blackened” world that will ultimately end metaphysics, but putting an end to metaphysics will also put an end to the world itself—this is the nihilism that the black nihilist must theorize through. This is a far cry from what we call “anarchy,” however. The black nihilist has as little faith in the metaphysical reorganization of society through anarchy than he does in traditional forms of political

existence. The black nihilist offers political apostasy as the spiritual practice of denouncing metaphysical violence, black suffering, and the idol

of anti-blackness. The act of renouncing will not change political structures or offer a political program ; instead,

it is the act of retrieving the spiritual concept of hope from the captivity of the Political . Ultimately, it is impossible to end metaphysics without ending blackness, and the black nihilist will never be able to

Page 24: openev.debatecoaches.org€¦  · Web view1AC . Observation 1: If I could find the spot where truth echoesI would stand there and whisper memories of my children's future. I would

withdraw from the Political completely without a certain death-drive or being-toward-death. This is the essence of black suffering : the lack of reprieve from metaphysics, the tormenting complicity in the reproduction of violence, and the lack of a coherent grammar to articulate these dilemmas . After contemplating these issues for some time in my office, I decided to take a train home. As I awaited my train in the station, an older black woman asked me about the train schedule and when I would expect the next train headed toward Dupont Circle. When I told her the trains

were running slowly, she began to talk about the government shutdown. “They don’t care anything about us, you know,” she

said. “We elect these people into office, we vote for them, and they watch black people suffer and have no intentions of doing anything about it.” I shook my head in agreement and listened intently. “I’m going to stop voting,

and supporting this process; why should I keep doing this and our people continue to suffer,” she said. I looked at her

and said, “I don’t know ma’am; I just don’t understand it myself.” She then laughed and thanked me for listening to her—as if our conversation were somewhat cathartic. “You know, people think you’re crazy when you say things like this,” she said giving me a wink. “Yes they do,” I said. “But I am a free woman,” she emphasized “and I

won’t go back.” Shocked, I smiled at her, and she winked at me; at that moment I realized that her wisdom and courage penetrated my mind and demanded answers. I’ve thought about this conversation for some time, and it is for this reason I had

to write this essay. To the brave woman at the train station, I must say you are not crazy at all but thinking outside of metaphysical time, space, and violence . Ultimately, we must hope for the end of political hope.

Case Black arguments cannot be debated without the experience of pain and suffering of black people. The aff is a appropriation of black culture in order to win the debate, re-entrenching whiteness due to the benefits of black suffering.WILDERSON IN 10 [Frank B., Associate Professor of African American Studies and Drama at the University of California & former member of the Umkhonto we Sizwe, Red, White and Black: Cinema and the Structure of US Antagonisms, p. 10, C.A.]Again, if accumulation and fungibility are the modalities through which Blackness is positioned as incapacity, then genocide is that modality through which embodied Redness is positioned as incapacity. Ontological incapacity, I have inferred and here state forthright, is the constituent element of ethics . Put another way, one cannot embody capacity and be, simultaneously, ethical. Where there are Slaves it is unethical to be free . The Settler/Master’s capacity, I have argued, is a function of exploitation and alienation, and the Slave’s incapacity is elaborated by accumulation and fungibility. But the “Savage” is positioned, structurally, by subjective capacity and objective incapacity, by sovereignty and genocide, respectively. The Indian’s liminal status in political economy, how her and his position shuttles between the incapacity of a genocided object and the capacity of a sovereign subject, coupled with the fact that Redness does not overdetermine the thanatology of libidinal economy (this liminal capacity within political economy and complete freedom from incapacity within libidinal economy) raises serious

doubts about the status of “Savage” ethicality vis-à-vis the triangulated structure (Red, White, and Black) of antagonisms. Clearly, the coherence of Whiteness as a structural position in modernity depends on the capacity to be free from genocide, perhaps not as a historical experience, but at least as a positioning modality . This embodied capacity (genocidal immunity) of Whiteness jettisons the White/Red relation from that of a conflict and marks it as an antagonism: it stains it with irreconcilability. Here, the Indian comes into being and is positioned by an a priori violence of genocide. Whiteness can also

Page 25: openev.debatecoaches.org€¦  · Web view1AC . Observation 1: If I could find the spot where truth echoesI would stand there and whisper memories of my children's future. I would

experience this kind of violence but only a fortiori: genocide may be one of a thousand contingent experiences of Whiteness but it is not a

constituent element, it does not make Whites white (or Humans human). Whiteness can grasp its own capacity, be present to itself, coherent, by its unavailability to the a priori violence of Red genocide, as well as by its unavailability to the a priori violence of Black accumulation and fungibility. If it experiences accumulation and fungibility, or genocide, those experiences must be named , qualified, that is, “White

slavery,” or the Armenian massacre, the Jewish Holocaust, Bosnian interment, so that such contingent experience is not confused with ontological necessity . In such a position one can always say, “Im not a ‘Savage’” or “I’m being treated like a nigger.” One can assert one’s Humanity by refusing the ruse of analogy. Regardless of Whites’ historical, and brief, encounters with the modalities of the “Savage” and the Slave, these modalities do not break in on the position of Whiteness with such a force as to replace

exploitation and alienation as the Settler/Master’s constituent elements. We might think of exploitation and alienation as modalities of suffering which inoculate Whiteness from death . If this is indeed the case, then perhaps Whiteness has no constituent elements other than the immanent status of immunity. Still, this immunity is no small matter, for it is the sine qua non of Human capacity.

Commodification DA: The aff commodifies the suffering of black people in exchange for your ballot in the debate economy---playing a game where we move scenarios of suffering around like chess pieces for our own personal enjoyment is the most unethical form of intellectual imperialism.Baudrillard 94 [Jean, “The Illusion of the End” p. 66-71]

We have long denounced the capitalistic, economic exploitation of the poverty of the 'other half of the world' [['autre monde]. We must today denounce the moral and sentimental exploitation of that poverty - charity cannibalism being worse than oppressive violence. The extraction and humanitarian reprocessing of a destitution which has become the equivalent of oil deposits and gold mines. The extortion of the spectacle of poverty and , at the same time, of

our charitable condescension: a worldwide appreciated surplus of fine sentiments and bad conscience . We should, in fact, see this not as the extraction of raw materials, but as a waste-reprocessing enterprise. Their destitution and our bad conscience are, in effect, all part of the waste-products of

history- the main thing is to recycle them to produce a new energy source.¶ We have here an escalation in the psychological balance of terror. World capitalist oppression is now merely the vehicle and alibi for this other, much more ferocious, form of moral predation. One might

almost say, contrary to the Marxist analysis, that material exploitation is only there to extract that spiritual raw material that is the misery of peoples , which serves as psychological nourishment for the rich countries and media nourishment for our daily lives . The 'Fourth World' (we are no longer dealing with a 'developing' Third World) is once again beleaguered, this time

as a catastrophe-bearing stratum. The West is whitewashed in the reprocessing of the rest of the world as waste and residue. And the white world repents and seeks absolution - it, too, the waste-product of its own history.¶ The South is a natural producer of raw materials, the latest of which is

catastrophe. The North, for its part, specializes in the reprocessing of raw materials and hence also in the reprocessing of catastrophe.

Bloodsucking protection, humanitarian interference , Medecins sans frontieres, international solidarity, etc. The last phase of colonialism: the New Sentimental Order is merely the latest form of the New World Order . Other people's destitution becomes our adventure playground . Thus, the humanitarian offensive aimed at the Kurds - a

show of repentance on the part of the Western powers after allowing Saddam Hussein to crush them - is in reality merely the second phase of the war, a phase

in which charitable intervention finishes off the work of extermination . We are the consumers of the ever delightful spectacle of poverty and catastrophe, and of the moving spectacle of our own efforts to alleviate it (which, in fact, merely function to secure the conditions of reproduction of the catastrophe market ); there, at least, in the order of moral profits, the Marxist analysis is wholly applicable: we see to it that extreme poverty is reproduced as a symbolic deposit, as a fuel essential to the moral and sentimental equilibrium of the West. ¶ In our defence, it might be said that this extreme poverty was largely of our own making and it is therefore normal that we should profit by it. There can be no finer proof that the distress of the rest of the world is at the root of Western power and that the spectacle of that distress is its crowning glory than the inauguration, on the roof of the Arche de la Defense, with a sumptuous buffet laid on by the Fondation des Droits de l'homme, of an exhibition of the finest photos of world poverty. Should we be surprised that spaces are set aside in the Arche d' Alliance. for universal suffering hallowed by caviar and champagne? Just as the economic crisis of the West will not be

complete so long as it can still exploit the resources of the rest of the world, so the symbolic crisis will be complete only when it is no longer able to feed on the other half's human and natural catastrophes (Eastern Europe, the Gulf, the Kurds, Bangladesh, etc.).

We need this drug, which serves us as an aphrodisiac and hallucinogen . And the poor countries are the best

Page 26: openev.debatecoaches.org€¦  · Web view1AC . Observation 1: If I could find the spot where truth echoesI would stand there and whisper memories of my children's future. I would

suppliers - as, indeed, they are of other drugs. We provide them, through our media, with the means to exploit this paradoxical resource, just as we give them the means to

exhaust their natural resources with our technologies. Our whole culture lives off this catastrophic cannibalism, relayed in cynical mode by the news media, and carried forward in moral mode by our humanitarian aid, which is a way of encouraging it and ensuring its continuity, just as economic aid is a strategy for perpetuating under-development. Up to now, the financial sacrifice has been compensated a hundredfold by the moral gain. But when the catastrophe market itself reaches crisis point, in accordance with the implacable logic of the market, when distress becomes scarce or the marginal returns on it fall from overexploitation, when we run out of disasters from elsewhere or when they can no longer be traded like coffee or other commodities, the West will be forced to produce its own catastrophe for itself , in order to meet its need for spectacle and that voracious appetite for symbols which characterizes it even more than its voracious appetite for food. It will reach the point where it devours itself. When we have finished sucking out the destiny of others, we shall have to invent one for ourselves. The Great Crash, the symbolic crash, will come in the end from us Westerners, but only when we are no longer able to feed on the hallucinogenic misery which comes to us from the other half of the world.¶ Yet they do not seem keen to give up their monopoly. The Middle East, Bangladesh, black Africa and Latin America are really going flat out in the distress and catastrophe stakes, and thus in providing symbolic nourishment for the rich world. They might be

said to be overdoing it: heaping earthquakes, floods, famines and ecological disasters one upon another, and finding the means to massacre each other most of the time. The 'disaster show' goes on without any let-up and our sacrificial debt to them far exceeds their economic debt. The misery with which they generously overwhelm us is something we shall never be able to repay. The sacrifices we offer in return are laughable (a tornado or two, a few tiny holocausts on the roads , the odd financial sacrifice) and, moreover, by some infernal logic, these work out as much greater gains for us, whereas our kindnesses have merely added to the natural catastrophes another one immeasurably worse: the demographic catastrophe, a veritable

epidemic which we deplore each day in pictures.

Page 27: openev.debatecoaches.org€¦  · Web view1AC . Observation 1: If I could find the spot where truth echoesI would stand there and whisper memories of my children's future. I would

Vs. SabotageThe 1AC relies on a futural hope of a “not-yet-realized” future in which they can fantasize about contingent solutions that never come. This hope creates a cruel optimism for black folk that forces them to invest in the pursuit of our own death because it crowds out all non-politically recognizable alternatives. This model for politics only re-entrenches the anti-black world and is an independent reason to reject the aff. Warren 15 (Calvin L., Black Nihilism and the Politics of Hope; Surce: CR: The New Centennial Review, Vol. 15, No. 1, Derrida and French Hegelianism (Spring 2015), XMT, pp. 215-248 Published by: Michigan State University Press Stable URL: http://www.jstor.org/stable/10.14321/crnewcentrevi.15.1.0215 . Accessed: 30/03/2015)

The politics of hope , then, constitutes what Lauren Berlant would call “ cruel optimism” for blacks (Berlant 2011). It bundles certain promises about redress, equality, freedom, justice, and progress into a political object that always lies beyond reach. The objective of the Political is to keep blacks in a relation to this political object—in an unending pursuit of it. This pursuit, however, is detrimental because it strengthens the very anti-black system that would pulverize black being . The pursuit of the object certainly has an “irrational” aspect to it, as Farred details, but it is not mere means without expectation; instead, it is a means that

undermines the attainment of the impossible object desired. In other words, the pursuit marks a cruel attachment to the means of subjugation and the continued widening of the gap between historical reality and fantastical ideal. Black nihilism is a “demythifying” practice, in the Nietzschean vein, that uncovers the subjugating strategies of

political hope and de-idealizes its fantastical object. Once we denude political hope of its axiological and ethical veneer, we see that it operates through certain strategies: 1) positing itself as the only alternative to the problem of anti-blackness, 2) shielding this alternative [End Page 221] from rigorous historical/philosophical critique by placing it in an unknown

future, 3) delimiting the field of action to include only activity recognized and legitimated by the Political, and 4) demonizing critiques or different philosophical perspectives. The politics of hope masks a particular cruelty under the auspices of “happiness” and “life.” It terrifies with the dread of “no alternative .” “Life” itself needs the security of the alternative, and, through this logic, life becomes untenable without it.

Political hope promises to provide this alternative—a discursive and political organization beyond extant structures of violence and destruction. The construction of the binary “alternative/no-alternative” ensures the hegemony and dominance of political hope within the onto-existential horizon. The terror of the “no alternative”—the ultimate space of decay, suffering, and death—depends on two additional binaries: “problem/solution” and “action/inaction.” According to this politics, all problems have solutions, and hope provides the accessibility and realization of these solutions. The solution establishes itself as the elimination of “the problem”; the solution, in fact, transcends the problem and realizes Hegel’s aufheben in its constant attempt to sublate the dirtiness of the “problem” with the pristine being of the

solution. No problem is outside the reach of hope’s solution—every problem is connected to the kernel of its own eradication. The politics of hope must actively refuse the possibility that the “solution” is, in fact, another problem in disguised form; the idea of a “solution” is nothing more than the repetition and disavowal of the problem itself. The solution relies on what we might call the “trick of time”

to fortify itself from the deconstruction of its binary. Because the temporality of hope is a time “not-yet-realized,” a future tense unmoored from present-tense justifications and pragmatist evidence, the politics of hope cleverly shields its “solutions” from critiques of impossibility or repetition. Each insistence that these solutions stand up against the lessons of history or the rigors of analysis is met with the rationale that these solutions are not subject to history or analysis because they do not reside

Page 28: openev.debatecoaches.org€¦  · Web view1AC . Observation 1: If I could find the spot where truth echoesI would stand there and whisper memories of my children's future. I would

within the horizon of the “past” or “present.” Put differently, we can never ascertain the efficacy of the proposed solutions because they escape the temporality of the moment, always retreating to a “not-yet” and “could-be” temporality. This “trick” of time offers a promise of possibility that can only be realized in an indefinite future, and this promise is a bond of uncertainty that can never be redeemed, only imagined. In this sense, the politics of hope is an instance of the psychoanalytic notion of desire: its sole

purpose is to reproduce its very condition of possibility, never to satiate or bring fulfillment. This politics secures its hegemony through time by claiming the future as its unassailable property and excluding (and

devaluing) any other conception of time that challenges this temporal ordering. The politics of hope, then, depends on the incessant (re)production and proliferation of problems to justify its existence. Solutions cannot really exist within the politics of hope, just the illusion of a different order in a future tense . The “trick” of time and political solution converge on the site of “action.” In critiquing the politics of hope, one encounters the rejoinder of the dangers of inaction. “But we can’t just do nothing! We have to do something.” The field of permissible action is delimited and an unrelenting binary between action/ inaction silences critical engagement with political hope. These exclusionary operations rigorously

reinforce the binary between action and inaction and discredit certain forms of engagement, critique, and protest. Legitimate action takes place in the political—the political not only claims futurity but also action as its property. To “do something” means that this doing must translate into recognizable political activity; “something” is a stand-in for the word “politics”—one must “do politics” to address any problem. A refusal to “do politics” is equivalent to “doing nothing”—this nothingness is constructed as the antithesis of life, possibility, time, ethics, and morality (a “zero-state” as Julia Kristeva [1982] might call it).

Black nihilism rejects this “trick of time” and the lure of emancipatory solutions. To refuse to “do politics” and to reject the fantastical object of politics is the only “hope” for blackness in an antiblack world .

The only possible demand is one that calls for the end of the world itself—the affirmative represents a conflict within the paradigm of America but refuses to challenge the foundational antagonism that produces the violence that undergirds that same paradigm Wilderson, ’10 [2010, Frank B. Wilderson is an Associate Professor of African-American Studies at UC Irvine and has a Ph.D. from UC Berkeley, “Red, White & Black: Cinema and the Structure of U.S. Antagonisms,”]

Leaving aside for the moment their state of mind, it would seem that the structure , that is to say the rebar, or better still the grammar of their demands —and, by extension, the grammar of their suffering—was indeed an ethical grammar. Perhaps their grammars are the only ethical grammars available to modern politics and modernity writ large , for they draw our attention not to the way in which space and time are used and abused by enfranchised and violently powerful interests, but to the violence that underwrites the modern world ’ s capacity to think, act, and exist spatially and temporally . The violence that robbed her of her body and him of his land provided the stage upon which other

violent and consensual dramas could be enacted. Thus, they would have to be crazy , crazy enough to call not merely the actions of the world to account but to call the world itself to account , and to account for them no less! The woman at Columbia was not demanding to be a participant in an unethical network of distribution: she was not demanding a place within capital, a piece of the pie (the demand for her sofa notwithstanding). Rather, she was articulating a triangulation between, on the one hand, the loss of her body, the very dereliction of her corporeal integrity, what Hortense Spillers charts as the transition from being a being to becoming a “being for the captor” (206), the drama of value (the stage upon which surplus value is extracted from labor power through commodity production and sale); and on the other, the corporeal integrity that, once ripped from her body, fortified and extended the corporeal integrity of everyone else on the street. She gave birth to the commodity and to the Human, yet

she had neither subjectivity nor a sofa to show for it. In her eyes, the world — and not its myriad discriminatory practices, but the world itself — was unethical . And yet, the world passes by her without the

Page 29: openev.debatecoaches.org€¦  · Web view1AC . Observation 1: If I could find the spot where truth echoesI would stand there and whisper memories of my children's future. I would

slightest inclination to stop and disabuse her of her claim . Instead, it calls her “crazy.” And to what does the world attribute the Native American man’s insanity? “He’s crazy if he thinks he’s getting any money out of us”? Surely, that doesn’t make him crazy.

Rather it is simply an indication that he does not have a big enough gun . What are we to make of a world that responds to the most lucid enunciation of ethics with violence ? What are the foundational questions of the ethico-political? Why are these questions so scandalous that they are rarely posed politically, intellectually, and cinematically—unless they are

posed obliquely and unconsciously, as if by accident? Return Turtle Island to the “Savage.” Repair the demolished subjectivity of the Slave . Two simple sentences, thirteen simple words, and the structure of U.S. ( and perhaps global ) antagonisms would be dismantled . An “ ethical modernity ” would no longer sound like an oxymoron . From there we could busy ourselves with important conflicts that have been promoted to the level of antagonisms: class struggle, gender conflict, immigrants rights. When pared down to thirteen words and two

sentences, one cannot but wonder why questions that go to the heart of the ethico-political , questions of political ontology, are so unspeakable in intellectual meditations, political broadsides , and even socially and politically engaged feature films. Clearly they can be spoken, even a child could speak those lines, so they would pose no problem for a scholar,

an activist, or a filmmaker. And yet, what is also clear—if the filmographies of socially and politically engaged directors, the archive of progressive scholars , and the plethora of Left-wing broadsides are anything to go by — is that what can so easily be spoken is now (five hundred years and two hundred fifty million Settlers/Masters on) so ubiquitously unspoken that these two simple sentences, these thirteen words not only render their speaker “ crazy ” but become themselves impossible to imagine . Soon it will be forty years since radical politics, Left-

leaning scholarship, and socially engaged feature films began to speak the unspeakable. In the 1960s and early 1970s the questions asked by radical politics and scholarship were not “ Should the U.S. be overthrown? ” or even “ Would it be overthrown? ” but rather when and how —and, for some, what—would come in its wake. Those steadfast in their conviction that there remained a discernable quantum of ethics in the U.S. writ large (and here I am speaking of everyone from Martin Luther King, Jr., prior to his 1968 shift, to the Tom Hayden wing of SDS, to the Julian Bond and Marion Barry

faction of SNCC, to Bobbie Kennedy Democrats) were accountable , in their rhetorical machinations, to the paradigmatic zeitgeist of the Black Panthers, the American Indian Movement, and the Weather Underground . Radicals and progressives could deride , reject, or chastise armed struggle mercilessly and cavalierly with respect to tactics and the possibility of “ success, ” but they could not dismiss revolution-as-ethic because they could not make a convincing case —by way of a paradigmatic analysis—that the U.S. was an ethical formation and still hope to maintain credibility as radicals and progressives . Even Bobby Kennedy (a U.S. attorney general and

presidential candidate) mused that the law and its enforcers had no ethical standing in the presence of Blacks . One could (and many did) acknowledge America’s strength and power. This seldom, however, rose to the level of an ethical assessment, but rather remained an assessment of the so-called “ balance of forces. ” The political discourse of Blacks, and to a lesser extent Indians, circulated too widely to credibly wed the U.S. and ethics. The raw force of COINTELPRO put an end to this

trajectory toward a possible hegemony of ethical accountability. Consequently, the power of Blackness and Redness to pose the question — and the power to pose the question is the greatest power of all — retreated as did White radicals and progressives who “ retired ” from struggle. The question ’ s echo lies buried in the graves of young Black Panthers, AIM Warriors, and Black Liberation Army soldiers , or in prison cells where so many of them have been rotting (some in solitary confinement) for ten, twenty, thirty years, and at the gates of the academy where the “ crazies ” shout at passers-by . Gone are not only the young and vibrant voices that affected a seismic shift on the political landscape , but also the intellectual protocols of inquiry, and with them a spate of feature films that became authorized, if not by an unabashed revolutionary polemic, then certainly by a revolutionary zeitgeist . Is it still possible for a dream of unfettered ethics, a dream of the Settlement and the Slave estate ’ s destruction , to manifest itself at the ethical core of cinematic discourse, when this dream is no longer a constituent element of political discourse in the streets nor of intellectual discourse in the academy? The answer is “no” in the sense that, as history has shown, what cannot be articulated as political discourse in the streets is doubly foreclosed upon

in screenplays and in scholarly prose; but “yes” in the sense that in even the most taciturn historical moments such as

Page 30: openev.debatecoaches.org€¦  · Web view1AC . Observation 1: If I could find the spot where truth echoesI would stand there and whisper memories of my children's future. I would

ours, the grammar of Black and Red suffering breaks in on this foreclosure , albeit like the somatic compliance of hysterical symptoms—it registers in both cinema and scholarship as symptoms of awareness of the structural antagonisms. Between 1967 and 1980, we could think cinematically and intellectually of Blackness and Redness as having the coherence of full-blown discourses. But from 1980

to the present, Blackness and Redness manifests only in the rebar of cinematic and intellectual (political) discourse, that is, as unspoken grammars . This grammar can be discerned in the cinematic strategies (lighting,

camera angles, image composition, and acoustic strategies/design), even when the script labors for the spectator to imagine social turmoil through the rubric of conflic t (that is, a rubric of problems that can be posed and conceptually solved) as opposed to the rubric of antagonism ( an irreconcilable struggle between entities , or positionalities, the resolution of which is not dialectical but entails the obliteration of one of the positions ). In other words, even when films narrate a story in which Blacks or Indians are beleaguered with problems that the script insists are conceptually coherent (usually having to do with poverty or the absence of “family values”), the non-narrative, or cinematic, strategies of the film often disrupt this coherence by posing the irreconcilable questions of Red and Black political ontology—or non-ontology.

The grammar of antagonism breaks in on the mendacity of conflict . Semiotics and linguistics teach us that when we

speak, our grammar goes unspoken. Our grammar is assumed. It is the structure through which the labor of speech is possible. Likewise, the grammar of political ethics —the grammar of assumptions regarding the ontology of suffering — which underwrite Film Theory and political discourse (in this book, discourse elaborated in direct relation to radical action), and which underwrite

cinematic speech (in this book, Red, White, and Black films from the mid-1960s to the present) is also unspoken . This notwithstanding,

film theory, political discourse , and cinema assume an ontological grammar, a structure of suffering . And the structure of suffering which film theory, political discourse, and cinema assume crowds out other structures of suffering, regardless of the sentiment of the film or the spirit of unity mobilized by the political discourse in question. To put a finer point on it, structures of ontological suffering stand in antagonistic, rather then conflictual, relation to one another (despite the fact that antagonists themselves may not be aware of the ontological positionality from which they speak). Though this is perhaps the most controversial and out-of-step claim of this book, it is, nonetheless, the foundation of the close reading of feature films and political theory that follows.

Blackness is the tool of civil society to avoid blame for everything it fucks up.Warren ’18 (Calvin Warren, Calvin Warren is an Assistant Professor in WGSS. He received his B.A. in Rhetoric/Philosophy (College Scholar) from Cornell University and his MA and Ph.D. in African American/American Studies from Yale University, 2018, Ontological Terror: Blackness, Nihilism, and Emancipation) \\EG

This lack of properness and metaphysical truth is a symptom of the nothing, for nothing lacks any proper place in metaphysics and cannot be

understood through its episteme. Black being as nothing, then, will always be out of place and improper in an antiblack world. It is the terror of the metaphysical infrastructure, and one can never be a true or proper man when one bears the weight of nothing. Through this analysis, we can understand the anxiety concerning black being, placement, and nothing in antebellum culture. In August 1842, for example, the free black population of Philadelphia held a parade commemorating the abolition of slavery in the West Indies.

A n angry mob of white citizens disrupted the parade, attacked participants, and commenced to destroy black homes and property. Seeking redress through the courts for loss of property and injury, the free black population realized that justice within such a context was impossible, as the grand jury acquitted the rioters and blamed free blacks for inciting this violence. Robert Purvis, a leader in the free black population of Philadelphia, responded to the grand jury’s decision with dismay:49 “The measure of our suffering is full. . . . From the most painful and minute investigation, in the feelings, views and acts of this community — in regard to us — I am convinced of our utter and complete nothingness in public estimation [emphasis mine].”50 What sparked the riot, this devastating expression of antiblackness? We can locate this eruption of

violence at the metaphysical fault line between necessity and hatred. Black being is both a necessary instrument for the human’s self-constitution and an object of ferocious hatred , since it bears the nothing of a metaphysical order. In other words, the riot is the symptom of a metaphysical problem: the public celebration of black freedom sparks a terror in that ontological boundaries are challenged and the transformation from black being, as invention/instrument, to human being, as free, is not only

considered but celebrated. It is also no surprise that the grand jury blamed the victims for the riot, since black freedom is a form of violence for the human , a violence that must be met with extreme force. The riot is a response to ontological terror. “Free,” when paired with “black,” is recast as a weapon against the

Page 31: openev.debatecoaches.org€¦  · Web view1AC . Observation 1: If I could find the spot where truth echoesI would stand there and whisper memories of my children's future. I would

human and the metaphysical structure that sustains the human. We are dealing with two registers of violence — one is an ontological violence and another is a physical form of antiblack destruction. But Purvis’s response to the violence is perspicuous. He is “convinced of our utter and complete nothingness in public estimation.” If we read this statement as a mere political lamentation, that blacks constitute a political cypher (nothingness) within the law and political processes, then we limit our understanding of the riot as event. The riot, within this reading, is just a form of cruelty or irrational intolerance or a political-economic strategy of subjection. With political readings of antiblack violence, violence is not gratuitous but must be linked to some type of recognizable transgression; when antiblack violence cannot be linked to recognizable transgression, it is considered cruel or irrational — a form of individual pathology and not systemic necessity. If, however, the essence of politics is nothing political, then we might read Purvis’s political commentary as a response to the proper metaphysical question.

His answer is that black being is nothingness in public estimation. We can understand nothingness as the condition (-ness) of

bearing nothing in an antiblack world. Antiblack violence, then, constitutes the structure of this nothing. Black being is always already under attack ; peace, within an antiblack world, is a fallacy (much like freedom). The metaphysical infrastructure

that supports the fiction of the white human is sustained by antiblack violence. The riot is an ontological necessity , not just political cruelty. We can understand the grand jury’s decision philosophically: Being black is both the cause and effect of violence , and when this being claims freedom, extreme violence is always justified and necessary.

The alternative is an unflinching paradigmatic analysis of the uncivility of civil society – the reclamation of the power to pose the question and analysis of ontological questions to articulate the worldWilderson 10 (Frank Wilderson, Professor of African American Studies at UC Irvine, “Red, White & Black: Cinema and the Structure of U.S. Antagonisms”)

STRANGE AS it might seem, this book project began in South Africa. During the last years of apartheid I worked for revolutionary change in both an underground and above-ground capacity, for the Charterist Movement in general and the ANC in particular. During this period, I began to see how essential an unflinching paradigmatic analysis is to a movement dedicated to the complete overthrow of an existing order. The neoliberal compromises that the radical elements of the Chartist Movement made with the moderate elements were due, in large part, to our inability or unwillingness to hold the moderates' feet to the fire of a political agenda predicated on an unflinching paradigmatic analysis. Instead, we allowed our energies and points of attention to be displaced by and onto pragmatic

considerations. Simply put, we abdicated the power to pose the question—and the power to pose the question is the greatest power of all. Elsewhere, I have written about this unfortunate turn of events (Incognegro: A Memoir of Exile and Apartheid), so I'll not rehearse the details here. Suffice it to say, this book germinated in the many political and academic discussions and debates that I was fortunate enough to be a part of at a historic moment and in a place where the word revolution was spoken in earnest, free of qualifiers and irony. For their past and ongoing ideas and interventions, I extend solidarity and appreciation to comrades Amanda Alexander, Franco Barchiesi, Teresa Barnes, Patrick Bond, Ashwin Desai, Nigel Gibson, Steven Greenberg, Allan Horowitz, Bushy Kelebonye (deceased), Tefu Kelebonye, Ulrike Kistner, Kamogelo Lekubu, Andile Mngxitama, Prishani Naidoo, John Shai, and S'bu Zulu. .¶ [CONTINUES]¶ In the Introduction and chapter 1, we saw how the aporia between Black being and political ontology has existed since Arab and European

enslavement of Africans. The crafting of questions through which one might arrive at an unflinching paradigmatic analysis of political ontology , a language that could express the structural and performative violence of Slave-making, is repeatedly thwarted. Humanist discourse, whose epistemological machinations provide our conceptual frameworks for thinking political ontology, is diverse and contrary. But for all its diversity and contrariness it is sutured by an implicit rhetorical consensus that violence accrues to the Human body as a result of transgressions, whether real or imagined, within the symbolic order. That is to say, Humanist discourse can only think a subject’s relation to violence as a contingency and not as a matrix that positions the subject. Put another way, Humanism has no theory of the Slave because it imagines a subject who has been either alienated in language or alienated from his or her cartographic and temporal capacities. 1 It cannot imagine an object who has been positioned by gratuitous violence and who has no car-

tographic and temporal capacities to lose—a sentient being for whom recognition and incorporation is impossible. In short, political ontology, as imagined through Humanism, can only produce discourse that has as its foundation alienation and exploitation as a grammar of suffering, when what is needed (for the Black, who is always already a Slave) is an ensemble of ontological questions that has as its foundation accumulation and fungibility as a grammar of suffering. 2 A Culture of Politics The violence of the Middle Passage and the Slave estate, 3 technologies of accumulation and fungibility, recompose and reenact their horrors on each succeeding generation of Blacks. This violence is both gratuitous (not contingent on transgressions against the hegemony of civil society) and structural (positioning Blacks

Page 32: openev.debatecoaches.org€¦  · Web view1AC . Observation 1: If I could find the spot where truth echoesI would stand there and whisper memories of my children's future. I would

ontologically outside of Humanity and civil society). Simultaneously, it renders the ontological status of Humanity (life itself ) wholly dependent on civil society’s repetition compulsion: the frenzied and fragmented machinations through which civil society reenacts gratuitous violence on the Black— that civil society might know itself as the domain of Humans— generation after generation. Again, we need a new language of

abstraction to explain this horror. The explanatory power of Humanist discourse is bankrupt in the face of the Black. It is inadequate and inessential to, as well as parasitic on, the ensemble of questions which the dead but sentient thing, the Black, struggles to articulate in a world of living subjects.

Next OFFGlobal Warming is not caused by humn’s writ large—it is caused by the uneven development engendered by Whiteness. The affirmative naturalizes the coercive racial politics at the heart of warming by universalizing its source and projecting its impacts far into the future. The imperial West started the process of warming, and the American racial state perpetuated it in the quest to export Whiteness. The affirmative only notices warming when it might destroy white bodies, invisibilizing millions of non-whites already killed.

Wynter 07 (Sylvia, Professor Emeritus in Spanish and Romance Languages at Stanford Univeristy, “The Human being as noun? Or being human as praxis? Towards the Autopoietic turn/overturn: A Manifesto,” otl2.wikispaces.com/file/view/The+Autopoetic+Turn.pdf)

For if, as Time magazine reported in January 2007 (Epigraph 2), a U.N. Intergovernmental panel of Natural Scientists, were soon to release

"a smoking-gun report which confirms that human activities are to blame for global warming " (and thereby for climate change), and had therefore predicted "catastrophic disruptions by 2100," by April, the issued Report not

only confirmed the above, but also repeated the major contradiction which the Time account had re-echoed. This contradiction , however, has nothing to do in any way with the rigor, and precision of their natural scientific findings , but rather with the contradiction referred to by Derrida's question in Epigraph 3—i.e., But who, we?

That is, their attribution of the non-natural factors driving global warming and climate change to, generic human activities , and/or to "anthropocentric forcings"; with what is, in effect, this mis- attribution then determining the nature of their policy recommendations to deal with the already ongoing reality of global warming and climate change, to be ones couched largely in economic terms. That is, in the terms of our present mode of knowledge production, and its "perceptual categorization system" as elaborated by the disciplines of the Humanities and Social Sciences (or "human sciences") and which are reciprocally enacting of our present sociogenic genre of being human, as that of the West's Man in its second Liberal or bio-humanist reinvented form, as homo oeconomicus; as optimally "virtuous Breadwinner, taxpayer, consumer, and as systemically over-represented as if it, and its behavioral activities were isomorphic with the being of being human, and thereby with activities that would be definable as the human-as-a-species ones.

Consequently, the Report's authors because logically taking such an over-representation as an empirical fact, given that, as highly trained natural scientists whose domains of inquiry are the physical and (purely) biological levels of reality, although their own natural-scientific order of cognition with respect to their appropriate non-human domains of inquiry, is an imperatively self-

correcting and therefore, necessarily, a cognitively open/open-ended one, nevertheless, because in order to be natural scientists , they are therefore necessarily, at the same time, middle class Western or westernized subjects, initiated 15 as such, by means of our present overall education system and its mode of knowledge production t o be the optimal symbolically encoded embodiment of the West's Man, it its second reinvented bio-humanist homo oeconomicus, and therefore bourgeois self-conception, over-represented as if it were isomorphic with the being of being

human, t hey also fall into the trap identified by Derrida in the case of his fellow French philosophers. The trap, that is, of conflating their own existentially experienced (Western-bourgeois or ethno-class) referent "we ," with the "we" of "the horizon of humanity ." This then leading them to attribute the reality of behavioral

Page 33: openev.debatecoaches.org€¦  · Web view1AC . Observation 1: If I could find the spot where truth echoesI would stand there and whisper memories of my children's future. I would

activities that are genre-specific to the West's Man in its second reinvented concept/self-conception as homo oeconomicus, ones that are therefore as such, as a historically originated ensemble of behavioral activitiesas being ostensibly human activities-

in-general. This, in spite of the fact that they do historicize the origin of the processes that were to lead to their recent natural scientific findings with respect to the reality of the non-naturally caused ongoing acceleration of global warming and climate change, identifying this process as having begun with the [West's] Industrial Revolution from about 1750 onwards. That is, therefore, as a process that can be seen to have been correlatedly concomitant in Great Britain, both with the growing expansion of the largely bourgeois enterprise of factory manufacturing, as well with the first stages of the political and intellectual struggles the British bourgeoisie who were to spearhead the Industrial Revolution, to displace the then ruling group hegemony of the landed aristocracy cum gentry, and to do so, by inter alia, the autopoetic reinvention of the earlier homo politicus/virtuous citizen civic humanist concept of Man, which had served to legitimate the latter's traditionally landed, political, social and economic dominance, in new terms. This beginning with Adam Smith and the Scottish School of the Enlightenment in the generation before the American, French, and Haitian (slave) revolutions, as a reinvention tat was to be effected in now specifically bourgeois terms as homo oeconomicus/and virtuous Breadwinner. 116 That is as the now purely secular genre of being human, which although not to be fully (i.e., politically, intellectually, and economically) institutionalized until the mid-nineteenth century, onwards, when its optimal incarnation came to

be actualized in the British and Western bourgeoisie as the new ruling class, was, from then on, to generate its prototype specific ensemble of new behavioral activities, that were to impel both the Industrial Revolution, as well as the West's second wave of imperial expansion, this based on the colonized incorporation of a large majority of the world's peoples, all coercively homogenized to serve its own redemptive material telos, the telos initiating of global warming and climate change. Consequently, if the Report's authors note that about 1950, a steady process of increasing acceleration of the processes of global warming and climate change, had begun to take place, this was not only to be due to the Soviet Revolution's (from 1917 onwards) forced march towards industrialization (if in its still homo oeconomicus conception, since a march spearheaded by the 116 See the already cited essay by J.G.A. Pocock "symbolic capital," education credentials owning and technically skilled Eastern European bourgeoisie)—as a state-directed form of capitalism, nor indeed by that of Mao's then China, but was to be also due to the fact that in the wake of the range of successful anti-colonial struggles for political independence, which had accelerated in the wake of the Second World War, because the new entrepreneurial and academic elites had already been initiated by the Western educational system in Western terms as homo oeconomicus, they too would see political independence as calling

for industrialized development on the "collective bovarysme "117 model of the Western bourgeoisie. Therefore, with the acceleration of global warming and climate change gaining even more momentum as all began to industrialize on the model of

homo oeconomicus, with the result that by the time of the Panel's issued April 2007 Report the process was now being driven by a now planetarily homogenized /standardized transnational "system of material provisioning or mode of techno-industrial economic production based on the accumulation of capital; as the means of production of ever-increasing

economic growth, defined as "development"; with this calling for a single model of normative behavioral activities, all driven by the now globally (post-colonially and post-the-1989-collapse-of-the-Soviet Union), homogenized desire of "all men (and women) to," realize

themselves/ourselves, in the terms of homo oeconomicus. In the terms, therefore, of "its single (Western-bourgeois or ethno-class) understanding" of "man's humanity," over-represented as that of the human; with the well-being and common good of its referent "we"—that, not only of the transnational middle classes but even more optimally, of the corporate multinational business industries and their financial networks, both indispensable to the securing of the Western-bourgeois conception of the common good, within the overall terms of the behavior-regulatory redemptive material telos of ever-increasing economic growth, put forward as the Girardot-type "cure" for the projected Malthusian-Ricardo transumed postulate of a "significant ill" as that, now, ostensibly, of mankind's threatened subordination to [the trope] of Natural Scarcity, this in the reoccupied place of Christianity of its postulate

of that "ill" as that of enslavement to Original Sin."' With the result that the very ensemble of behavioral activities indispensable, on the one hand, to the continued hegemony of the bourgeoisie as a Western and westernized transnational ruling class, is the same ensemble of behaviors that is directly causal of global worming and climate change, as they are, on the other, to the continued dynamic enactment and stable replication of the West's second reinvented concept of Man; this latter in response to the latter's existential imperative of guarding against the entropic disintegration of its genre of being human and fictive nation-state mode of kind. Thereby against the possible bringing to an end, therefore, of the societal order, and autopoetic living Western and westernized macro world system in it bourgeois configuration, which is reciprocally the former's (i.e., its genre of being human, and fictive modes of kind's condition of realization, at a now global level. This, therefore, is the cognitive dilemma, one arising directly from the West's hitherto unresolvable aporia of the secular, that has been precisely captured by Sven Lutticken in a recent essay. Despite, he writes, "the consensus that global warming cannot be ascribed

Page 34: openev.debatecoaches.org€¦  · Web view1AC . Observation 1: If I could find the spot where truth echoesI would stand there and whisper memories of my children's future. I would

to normal fluctuations in the earth's temperature... [the] social and political components of this process have been minimized; man-made nature is re-naturalized, the new (un)natural history presented as fate." And with this continuing to be so because (within the terms, I shall add, of our present "single understanding of man's humanity" and

the unresolvable aporia which it continues to enact), "[ t]he truly terrifying notion is not that [ global warming and climate change] is irreversible, but that it actually might be reversible—at the cost of radically changing the economic and social order ... "119The changing , thereby, of the now globally hegemonic biologically absolute answer that we at present give to the question to who we are, a nd of whose biohumanist homo oeconomicus symbolic life/death (i.e., naturally selected/dysselected) code's intentionality of dynamic

enactment and stable replication, our present "economic and social order " is itself the empirical actualization.

Macropolitical attempts to stop climate change and the institution of diplomacy itself are failing --- try-or-die using our scholarship to demand more radical changes Burke et al 16 [Anthony Burke, Audra Mitchell, Balsillie School of International Affairs/Wilfrid Laurier University, Canada, Simon Dalby, Balsillie School of International Affairs/Wilfrid Laurier University, Canada, Daniel J. Levine, University of Alabama, USA, Stefanie Fishel, The Department of Gender and Race Studies, University of Alabama, “Planet Politics: A Manifesto from the End of IR,” Millennium - Journal of International Studies June 2016 vol. 44 no. 3 499-523]

Arresting dangerous climate change , stemming species extinctions, decarbonising our civilisation: this must be a common political project if life on this planet is to survive. However, such a project will necessarily involve agonism and conflict; it will be achieved through both new forms of cooperation and ongoing contestation, through a ‘cosmopolitics’ that admits (many different) humans, nonhumans and things, present, absent, living, inorganic, powerful and less powerful, by making politics receptive to the disturbances they create. It will involve amplifying marginalised voices and creating new forms of solidarity and governance to confront the dystopian power of big energy, big farming, big finance, and fossil fuel capitalism. Planet Politics must be simultaneously a practice of governance and of subversion, of regulation and resistance, at multiple scales and locales. Indeed as resistance it is already underway, but as governance it is struggling to be born. Planet Politics must be very different from the elitist and state-centric global governance that is today’s handmaiden of extinction. 3. Diplomacy, as an institution, is failing Long ago, Hedley Bull argued that Diplomacy was one of the five key ‘institutions’ of international society, alongside International Law, Great Powers, the Balance of Power, and War.29 Diplomacy is carried out by official representatives of states and transnational institutions also created by states. Corporations have lobbied, bought, and bribed themselves into the game. Everyone else is an NGO, or worse, a person, a nothing. And non-human species, oceans, ecosystems – the very living complexity of the planet – have no status at all. Bull’s ‘institutions’ are the action-actor-artifacts of contemporary international society, the subjects and objects that we abjectly depend on to solve the planet’s problems. Below we speak of what must change in international law; how is diplomacy contributing to acknowledging and addressing the gravity of the imminent ecological collapse? Diplomacy has provided the United Nations Framework Convention (UNFCCC) and the Kyoto Protocol on climate change. These are international society’s sole treaty bulwark against the sixth extinction and a potential future of unchecked climate change that the International Institute of Strategic Studies asserted would be ‘catastrophic – on the level of nuclear war’.30 Negotiated in 1997 but not in force

Page 35: openev.debatecoaches.org€¦  · Web view1AC . Observation 1: If I could find the spot where truth echoesI would stand there and whisper memories of my children's future. I would

until 2005, Kyoto was originally a modest commitment by a small group of countries to cut emissions over four years; it has since been extended to the end of 2020 – just five years away – and has achieved cuts of 29% below business as usual. Yet global emissions as a whole soared by 40% to 2009 and have risen to historically unprecedented levels since. Current emissions trends are tracking towards an average warming of 3.7-5.5°C over pre-industrial levels. If the earth warms 3 degrees, the Arctic ice sheet will melt, triggering a rise in sea levels of 7 metres.31 Leading climate scientists are also warning that international society’s assumed ceiling for emissions and ‘dangerous’ climate change (~1000 gigatonnes of carbon (GtC) or 2°C) will, in fact, ‘spur “slow” feedbacks and eventual warming of 3–4°C with disastrous consequences’. Instead we must limit atmospheric greenhouse gases to ~500 GtC or 350 ppm of CO2 – which means dramatically reducing existing concentrations rather than continuing to emit more, given that measurements of over 400 ppm were recorded in 2015.32 The United Nations Framework Convention on Climate Change (UNFCCC) is reviewing the 2 degree target, but this process remains bogged down in political maneuvering.33 Indeed the miraculous appearance of the ‘safe space’ target of 1-1.5 degrees in the preamble to the 2015 Paris agreement – albeit in a way that is nonbinding and subject to further investigation by the IPCC – exemplifies the profound tensions between the needs of the planet and our global diplomatic mechanisms.34 The 1.5 degree target appeared in the agreement due to the combined pressure of scientists, global civil society, and climate-vulnerable states . Yet it is still not a formal goal, and we fear it may never become one. Furthermore, even the text of the Paris agreement notes that the ‘nationally determined contributions’ volunteered by states will fail to keep global warming within 2 degrees let alone 1.5.35 In short, nearly two decades after Kyoto, states have yet to agree to binding emissions reductions that will prevent dangerous climatic change according to the best contemporary advice of earth system scientists. Objects in this mirror are closer than they appear. We can acknowledge the diplomatic achievement of France, the United States and China in preventing the Paris meeting from being a complete failure, and for opening a door to more stringent action in the future.36 At the same time, the UN-based system has presided over an alarming increase in emissions in the three decades since the gravity of the climate crisis was comprehensively identified; emissions that may lock in devastating changes to the biosphere that will be difficult to avoid. This system is still based on consensus and delay , allowing spoilers enormous influence ; i t still keeps the voices of scientists, civil society and indigenous peoples on the margins; it fails to integrate environmental, security and economic governance , or harmonise them in normative terms; and it has no effective mechanisms to admit the claims of the nonhuman. And beyond the problem of climate, a coordinated, accountable, and democratic global machinery to protect crucial ecosystems, restore oceans, end deforestation, and ensure breathable air remains far off – fractured between states, corporate lobbies, and weak and fragmented international organisations kept separate from the ‘real business’ of global economic and security governance.37 Policy elites still talk and think as if their agency matters and is morally unproblematic; as if, clad in the armature of the state and striding purposefully though his own institutions, Cartesian ‘Man’ can continue to dictate to the planet. This attitude was exemplified by comments by the Indian environment minister after the 2014 Lima Accord,38 who argued for the Paris agreement to provide developing countries ‘equitable carbon space to achieve sustainable development’39 – as if the atmosphere can be divided up according to the principles of state sovereignty, as if there is any atmospheric space left. The biosphere cannot be traded, divided or bargained away. It is not a product, nor a monetary or diplomatic artifact, amenable to state compromises and quantification. When earth system scientists are warning that the safe limit for atmospheric carbon concentrations is 50 ppm less than current levels, the continued commitment of the

Page 36: openev.debatecoaches.org€¦  · Web view1AC . Observation 1: If I could find the spot where truth echoesI would stand there and whisper memories of my children's future. I would

UNFCCC to market mechanisms is fetishistic and bizarre. When there can be no emissions to trade, there can be no global emissions trading system. While it is possible to count tonnes of emissions and parts per million of CO2, it is not possible to count non-linear events and unpredictable feedbacks, and the cascading ecosystem and social damage that will ensue. The true moral disaster of extinction cannot be measured in numbers of species lost and billions of animal dead, but in the irreversible devastation to worlds that it represents . In the near term, we will have to work with flawed institutions, but the gravity of this crisis means that it is right to demand more profound and systemic change , and to explore, in politics and in scholarship, what that change should be. Diplomacy will remain in some form a part of global solutions, but as an established institution it is failing us because the crisis we face demands fundamental change in the underlying system and its commitments, of which diplomacy is an epiphenomenon. Diplomacy is the visible hands of a watch running down, when what must be done is not merely expose its interior workings, but to re-imagine our entire structure of social-political time. This time is both much longer than our contemporary horizons, taking in hundreds and thousands of years, and much shorter, requiring ameliorative action that should have begun yesterday. The planet’s watch is ticking ever louder, and too many diplomats and statesmen seem deaf to it; deaf to the running down of the world and the voices of those most affected by melting glaciers, rising waters, and drying continents.

Vote negative on presumption – even if we banned all CO2 emissions right now, we would still face the most cataclysmic effects of climate change for centuries. Successful economic decarbonization is irreconcilable with global industrial capitalism as its lust for profit. It would mean turning off 80% of the world’s power, dooming us further. Even if all of that were effective, it’d take too long to create the global energy infrastructure to make a difference anyhow.Scranton 15. Jeremy Scranton, acclaimed journalist, activist and author, PhD in English from Princeton, currently teaching at Notre Dame, New York Times contributor, Iraq War veteran, Learning To Die in the Anthropocene: Reflections on the End of Civilization, City Lights Books, 2015, online

“The only sure way to keep global warming from accelerating out of control would be to stop dumping waste carbon dioxide immediately. In the stern words of the IPCC: “Climate change can only be mitigated and global temperature be stabilized when the total amount of CO2 is limited and emissions eventually approach zero.”45 With just the CO2 in the atmosphere and oceans today, we are already set for at least 2 to 3 degrees Fahrenheit warming above pre-industrial levels , ” “and it might be more like 5 or 6 degrees Fahrenheit . Any more CO2 we put in from now on (by starting a car, for instance, or charging a phone) is only going to amplify that. So even if we banned dumping CO2 right now, this very instant, we would still be facing serious climate impacts for centuries . Unfortunately for us, given the realities of global politics, a comprehensive, enforceable, worldwide ban on CO2 is sheerest fantasy.

But what about other solutions? What about mitigation? What about decarbonizing our economy, replacing coal and oil with renewable energy or nuclear power? What about a carbon tax? What about cap-and-trade, carbon capture and sequestration, carbon extraction, and geoengineering? Might these

Page 37: openev.debatecoaches.org€¦  · Web view1AC . Observation 1: If I could find the spot where truth echoesI would stand there and whisper memories of my children's future. I would

strategies help us end, reduce, or at least mitigate our CO2 emissions before we hit a tipping point and it’s too late?

Ending our reliance on carbon-based fossil fuels— decarbonizing the global economy—would be the most reliable path to limit and eventually stop dumping waste CO2. The problem is that global decarbonization is effectively irreconcilable ” “with global capitalism . Capitalism needs to produce profit in order to spur investment. Profit . Global economic growth, even basic economic stability, depends on cheap, efficient energy.

Decarbonizing the global economy without a replacement energy source would mean turning off approximately 80 percent of our power, causing a worldwide economic meltdown that would make the Great Depression look like a sluggish sales season. While not nearly as dire, worldwide decarbonization with replacement energy still looks pretty unpalatable. The most reliable studies suggest that even stabilizing CO2 at a relatively low but still unsafe level would require long-term economic austerity. According to the Potsdam Institute for Climate Impact Research, stabilizing carbon dioxide levels at 450–500 ppm (which is 100–200 ppm over the upper limit for keeping warming anywhere near 3.6 degrees Fahrenheit [ 2 degrees Celsius]) calls for slowing and probably even contracting the global economy indefinitely , basically extending the Great Recession into the indefinite future. No population on the planet today is going to willingly trade economic growth for lower carbon emissions, especially since economic power remains the key index of global status.47 The political paroxysms of forced austerity rocking countries across Europe today are only a taste of what we would have to look forward to under a carbon-austerity regime.

Offering a glimmer of hope, the IPCC ’s 2014 report on mitigation argues that we can avoid the worst of global warming with what would be only a slight decrease in global economic growth—about .06 percent.48 The report claims that shifting investment from oil and coal production to research and development of renewable energies, nuclear power, and carbon capture and sequestration could make it possible to decarbonize within thirty or forty years with only a slight cost to global gross domestic product. The report may be right, but the IPCC’s numbers are speculative, not predictive: The renewable energy and carbon capture tech nologies the report’s numbers depend on are still emerging , and we don’t know yet whether they can work on a large ” “enough scale to make a difference, or how much more they might cost than carbon fuels do now. As well, critics have pointed out a variety of problems with the IPCC’s numbers on decarbonization, ranging from excessive political influence affecting data to biases in models underestimating economic impacts .49

Adding to our troubles, it’s no simple thing to completely renovate worldwide energy infrastructure that has taken many years to build. Vaclav Smil, one of the world’s leading energy analysts , observes: “There are five major reasons that the transition from fossil to nonfossil supply will be much more difficult than is commonly realized: scale of the shift; lower energy density of replacement fuels ; substantially lower power density of the renewable energy extractions; intermittence of renewable flows; and uneven distribution of renewable energy resources .”50 It would take decades to develop and implement new systems of carbon-free or carbon-minimal energy infrastructure, if it’s even possible, and we don’t have decades.51 Even 2035, a mere twenty years from now, will be too late. It’s very likely already too late now .

Page 38: openev.debatecoaches.org€¦  · Web view1AC . Observation 1: If I could find the spot where truth echoesI would stand there and whisper memories of my children's future. I would

The Alternative is to reject the 1AC in favor of a teleological suspension of whiteness – our alternative is a process of refusal. Refusing to allow the production of whiteness. Headley 4 (Clevis Headley, “Delegitimizing the Normativity of ‘Whiteness’”, from What White Looks Like: African-American Philosophers on the Whiteness Question, edited by George Yancy)

I want to conclude this essay by briefly considering the future of whiteness. Some thinkers have called for a deconstruction of whiteness, claiming that its contingent historical status will be exposed and its reified status eradicated. Of course, revealing the arbitrary ontological and axiological status of whiteness and its relational dependence upon the other concepts does not automatically translate into an undermining of whiteness. That concepts survive attempted semantic executions is uncontested; we cannot eliminate a concept from our language simply by showing it to be the product of an economy of linguistic difference. There is also the abolitionist position of whiteness and white identity.

Abolitionists confidently call for this abolition on the grounds that whiteness is best defined negatively, that is, by what it is not. Instead of viewing whiteness as designating independent content, it is seen as designating lack and, accordingly, as fake. Because of the void at the heart of whiteness, whites fill in the blanks by devouring the Other . This consuming of the Other takes place because white identity is empty, void of any

content aside from its mediation as difference from non-whiteness. As Roediger writes: “It is not simply that whiteness is oppressive and false. . . . It is the empty and terrifying attempt to build an identity based on what one isn’t and on whom one can hold back.” Although I agree with the critical thrust of the abolitionist program, I must express some skepticism about its ultimate impact. The idea of the abolition of whiteness requires that one clearly state the precise nature of this abolition program. Is it simply going to be a matter of the legal abolition of whiteness? Certainly, one can respond by stating that the civil rights revolution was, in part, intended to eliminate whiteness through legal strategies. Of course, what

was accomplished was the promotion of a procedural formalist notion of equality that serves as a camouflage for whiteness. But with regard to the technical notion of abolition, we still need to know how the voluntary abolition of racial identity is possible beyond the courageous acts of a few isolated individuals, particularly since white identity is a majoritarian and is reinforced by tremendous economic, cultural, social, legal, and political power. From an Africana philosophical perspective, I argue for the teleological suspension of whiteness, realizing that

whiteness must be transcended but not by rational argument. The point here is not to license irrationalism, but,

rather, to underscore the fact that rational argumentation proves blunt in the face of centuries-old social privilege. Here, the notion of the suspension of whiteness. Let us consider a suggestive Africana appropriation of the Kierkegaardian existential dialect. The aesthetic stage of whiteness represents the stage when whiteness and its tremendous benefits were overtly constituted. Of course, this situation generated opposition in that those excluded from the privileges of whiteness openly challenged the rightness of

whiteness and exposed the immorality of white supremacy. Overt and blatant rejoicing in the privilege of whiteness could not escape challenge. The ethical stage of whiteness is similarly dominated with a concern to do the morally right thing. This concern takes the form of bold appeals to objectivity, formal equality of

opportunity, and procedural notions of justice. In this stage, there is a strong appeal to the moral worth of each individual and an equally confident emphasis on the notion of rewarding individuals strictly on merit. In conjunction with a forceful advocacy of color blindness, policies that recognize race, such as affirmative action, are considered to be immoral because they favor groups and not individuals.

Consequently, race-conscious policies are denounced as flagrantly violating both moral and constitutional principles of fairness. We are quickly approaching a situation in which appealing to formal principles of equality and opportunity and to universal principles of justice cannot effectively remedy the tremendous differential in power between blacks and whites . As this realization becomes more and more evident, there will be a need to move on to a third stage.

Unlike Kierkergaard’s third stage, the religious stage made possible by an unconditional act of faith, the third stage of whiteness will more closely resemble his notion of the teleological suspension of the ethical and the inability to rationally inscribe this suspension in a universal discourse. Not being concerned with ultimate meaning or with religious

affirmation, the third stage of whiteness will not be a deconstruction or abolition of whiteness but, rather, a teleological suspension of whiteness. Whiteness cannot be dismantled through rational or

analytical means. Its suspension must come in the form of a continuously affirmed refusal to prolong

Page 39: openev.debatecoaches.org€¦  · Web view1AC . Observation 1: If I could find the spot where truth echoesI would stand there and whisper memories of my children's future. I would

the ontological and existential project of whiteness. The project of whiteness must be suspended for the greater good of human liberation beyond whiteness. The project of whiteness has proven too costly for human

existence. The existential price is simply too costly for those who are forced to involuntarily participate in this project. Hence, teleologically suspending whiteness is a solution, a counter-project that cannot be rationally stated. It defies neutral conceptualization because there is no direct and rationally persuasive way of linguistically describing the urgency of this cause. The reason why the question of the suspension of whiteness cannot be framed in the language of our legal and political system is because our language itself is infected with the project of whiteness . After all, what would it mean to argue in favor of renouncing whiteness and its benefits by using a discourse imbued with the categories that perpetuate whiteness? To the extent that whiteness, understood as white supremacy, is a global system of white racial domination similar to patriarchy, Africana philosophers can call attention to

racial bias in language in the same manner that feminist philosophers have called attention to gender bias in language. For example, the alleged neutral and universal notions of political discourse, which should function neutrally, promote racial outcomes that benefit whites. Lipsitz claims that “the language of liberal individualism serves as a cover for coordinated collective group interests.” Robyn Wiegman, calling attention to Cheryl Harris’s distinction between corrective justices, which seeks “compensation for discrete and ‘finished’ harm done to minority groups members or their ancestors,” and distributive justice, which is “the claim an individual or group would have been awarded under fair conditions,” affirms Harris’s claim that “the goals of affirmative action—to address the harms done to those people minoritized by racial . . . oppression—are undermined when corrective justice is the interpretive frame because not only is the harm assumed to be finished but the practices through which harm has been done are individualized, confined to the one who perpetrated it and the one who endured it. In this context, whites can claim to be innocent and

therefore in need of counterlegislative protection because they have not individually perpetuated harm.” Transcending whiteness, from an Africana philosophical perspective , must of necessity do more than expose the secret career of whiteness in maintaining structures of racial privilege. Such an analysis of whiteness should also serve as an opportunity to acknowledge the affirmative possibilities of conceiving alternative social modes of being. In other words, it should underscore the realization that things can be otherwise. Moving beyond whiteness requires a radically new concept of the human, a new metaphoricity of humanity .

Case

Page 40: openev.debatecoaches.org€¦  · Web view1AC . Observation 1: If I could find the spot where truth echoesI would stand there and whisper memories of my children's future. I would

SAB CASENo inherency Senate voted to end support for Saudi Arabia, this is more recent then their Zheng evidence.AP 12/14 (Associated Press. “Senate Rebukes Trump Response to Khashoggi Death and U.S. Support of Yemen War.” TIME. 14 Dec. 2018, http://time.com/5479319/senate-rebukes-trump-response-khashoggi-death-yemen-war/.) LHSLA LH

Senators voted 56-41 to recommend that the U.S. stop supporting the war in Yemen , a direct affront to the administration’s war powers abilities. Independent Sen. Bernie Sanders of Vermont, who co-sponsored the Yemen resolution with Republican Sen. Mike Lee of Utah, called passage a “historic moment.” Lee said Khashoggi’s death focused attention “on the fact that we have been led into this civil war in Yemen half a world away” and “we’ve done so following the lead” of Saudi Arabia. “What the

Khashoggi event did was to demonstrate, hey, maybe this isn’t a regime that we should just be following that eagerly into battle,” Lee said. As Senate approval loomed, the administration dispatched Pompeo and Defense Secretary Jim Mattis to the House to make the case against the resolutions and warn of damage they could do to the U.S.-Saudi relationship. A congressional aide and an administration official said their appearance was aimed at stopping any House action on the resolutions. Pompeo and Mattis had made a similar entreaty to the Senate late last month.

But it was roundly panned by senators angered by the secretaries’ refusal to accept a CIA determination that assessed the crown prince had ordered Khashoggi’s murder. CIA Director Gina Haspel briefed House leaders Wednesday on the Khashoggi slaying. The journalist, who had lived in the U.S. and wrote for The Washington Post, had been critical of the Saudi regime. He was killed in what U.S. officials have described as an elaborate plot as he visited the consulate for marriage paperwork. Saudi prosecutors have said a 15-man team sent to Istanbul killed Khashoggi and then dismembered his body, which has not been found. Those findings came after Saudi authorities spent weeks denying Khashoggi had been killed in the consulate. Trump has been reluctant to condemn the crown prince. He said the United States “intends to remain a steadfast partner” of the country, touted Saudi arms deals worth billions of dollars to the U.S. and thanked the Saudis for plunging oil prices. But Graham and Sen. Bob Menendez, the top Democrat on the Foreign Relations Committee, have rejected Trump’s economic arguments. They are setting the stage for legislation next year that goes further in

halting arms sales and taking other measures. Menendez says economic concerns do not overpower human rights and the U.S. must send a “global message that killing with impunity” will not be tolerated . Frustration with the crown prince and the White House prompted several Republicans to support the Yemen resolution. Seven Republicans and all Democrats voted for it. Some already had concerns about the war, which human rights groups say is wreaking havoc on the country and subjecting civilians,

many of them children, to deadly disease and indiscriminate bombing. The resolution condemning Saudi Arabia for Khashoggi’s slaying was from Senate Foreign Relations Chairman Bob Corker and Senate Majority Leader Mitch McConnell . Both Republicans opposed the Yemen resolution and voted against it. McConnell said senators have grave concerns about Khashoggi’s killing, but “we also want to preserve a 70-year partnership between the United States and Saudi Arabia, and we want to ensure it continues to serve American interests and stabilizes a dangerous and critical region.” But McConnell encouraged passage of the Khashoggi resolution and said it provided “a clear and unambiguous message about how we feel about what happened to this journalist .” The Senate debate came as the United Nations secretary general announced that Yemen’s warring sides have agreed to a province-wide cease-fire and withdrawal of troops in Hodeida, a contested Red Sea port city. The agreement came during peace talks in Sweden. The brutal four-year-old civil war pits the internationally recognized Yemeni government, supported by a Saudi-led coalition, against the Iran-backed rebels known as Houthis.

Page 41: openev.debatecoaches.org€¦  · Web view1AC . Observation 1: If I could find the spot where truth echoesI would stand there and whisper memories of my children's future. I would

Canada ending arms sales now—fills in gaps left by the U.S. and decimates Saudi military.Burke 12/17 (Burke, Michael. “Trudeau says Canada trying to end arms contract with Saudi Arabia.” The Hill. 17 Dec. 2018, https://thehill.com/policy/international/421673-trudeau-says-canada-trying-to-end-arms-contract-with-saudi-arabia.) LHSLA LH

Canadian Prime Minister Justin Trudeau on Sunday said Canada is looking into ending sales of armored vehic les to Saudi Arabia as pressure builds following the murder of U.S.-based journalist Jamal Khashoggi. "We are engaged with the export permits to try and see if there is a way of no longer exporting these vehicles to Saudi Arabia,” Trudeau told Canadian television network CTV, according to The Globe and Mail. Trudeau's comments come after he said in October that he was unlikely to cancel the sales. “I do not want to leave Canadians holding a billion-dollar bill because we’re trying to move forward on doing

the right thing," he said of the Saudi deal at the time. But Canada and other countries have faced increased pressure to end arms sales to Saudi Arabia following the murder of Khashoggi and amid criticism of the deadly civil war in Yemen. Khashoggi, a Washington Post columnist, was killed inside the Saudi Consulate in Istanbul in October. The CIA has reportedly concluded with high confidence that Saudi Crown Prince Mohammed bin Salman ordered the killing. The U.S. has faced pressure to end its own arm sales to Saudi Arabia since the slaying, but President Trump has maintained that he will not do so. Trump, who has refused to condemn Saudi Arabia or the crown prince over the killing of Khashoggi, has said that ending those sales would result in significant job losses in the U.S.

This means 1. no solvency and 2. They need to isolate why the aff ending arm sales is uniquely different from 1. Canada ending sales, and 2. Congress ending sales in December. 3. Fillin will happen

Saudi-Russia relations thump.Simes Jr. 19 (Dimitri Simes Jr., “Saudi Arabia Is Quietly Cozying Up To Russia: What Does It Mean For The Middle East?”, Daily Caller, 07/08/2019, https://dailycaller.com/2019/07/08/saudi-arabia-quietly-cozying-up-to-russia/)//Shreyas

During the G20 summit earlier this month, Saudi Crown Prince Mohammad bin Salman met with Russian President Vladimir Putin. After the

meeting, Putin announced that Russia and Saudi Arabia had agreed to extend an oil production cut agreement for another nine months. When the extension was formalized several days later, OPEC Secretary General Mohammad

Barkindo touted the deal as a “Catholic marriage” and boasted that it would last an “eternity.” Such cooperation between the two countries would have surprised many international observers just a few years ago. Back then, Saudi Arabia and Russia appeared headed for showdown over Syria and global oil prices. Now however, one of America’s closest allies in the Middle East and one

of its leading adversaries are increasingly finding common ground. At a time when many foreign investors are leaving Russia, Saudi Arabia is investing billions of dollars into the Russian economy. Moreover, Saudi Arabia and Russia are increasingly coordinating to influence global oil prices. In a few instances, Saudi Arabia has even sided with Russia over the United States. For

example, Riyadh has publicly opposed further U.S. sanctions against Moscow and abstained from several UN votes condemning Russian actions in Ukraine. (RELATED: Saudi Arabia Is Trying To Execute And Possibly Crucify Teen Who Participated In Protest When He Was 10 Years Old: Report) Dmitry Frolovsky, an independent Moscow-based geopolitical analyst, told The Daily Caller that the current level of cooperation between Saudi Arabia and Russia is “unprecedented.” “The convergence with Saudi Arabia over the past several years in many ways is surprising even to Russian strategists because no one expected that Russia’s campaign in Syria could help improve relations with the Persian Gulf monarchies so much,” he said. Riyadh and Moscow have historically had a contentious relationship. As recently as 2013, Saudi Intelligence head Prince Bandar bin Sultan tried to persuade Putin to alter his position on Syria by raising the prospect of terrorism at the Sochi Olympics games, according to a report from The Telegraph. However, the Obama administration’s foreign policy in the Middle East made Saudi Arabia rethink its approach to Moscow. Riyadh was alarmed by the Obama administration’s favorable reception of the Arab Spring, which swept away several prominent pro-American dictators in the region. When the United States signed the Iran nuclear deal in 2015, the kingdom regarded the move as a betrayal. On the other hand, Russia’s military intervention in Syria left a strong impression on Saudi

Arabia. Thus, at a time when Saudi Arabia sought to hedge its bets against Washington, Russia seemed likely a formidable option. In 2016, Saudi Arabia and Russia signed their first deal on limiting oil production to combat falling energy

prices. Saudi King Salman visitedMoscow the subsequent year, becoming the first Saudi monarch to set foot in Russia. Economics is

Page 42: openev.debatecoaches.org€¦  · Web view1AC . Observation 1: If I could find the spot where truth echoesI would stand there and whisper memories of my children's future. I would

increasingly at the forefront of Saudi-Russia cooperation . While many Western investors shunned Russia after the 2014 annexation of Crimea, Riyadh instead expanded its portfolio in the country. In 2015, Saudi Arabia pledged to invest $10 billion into the Russian

economy. A quarter of that sum has been invested so far. Last October, Saudi Arabia’s sovereign wealth fund became a partner of a Russo-Chinese investment fund and contributed $500 million to the project. That month Saudi Arabia also promised to provide $5 billion for an LNG project in the Arctic by Russian gas company Novatek. Some in Moscow are optimistic that 2019 could be a breakthrough year for Saudi investment in Russia. Kirill Dmitriev CEO of Russia’s sovereign wealth fund, recently told Saudi website Argaam that Russia and Saudi Arabia were exploring investing $2 billion this year on 25 different joint projects. Alexey Khlebnikov, a Middle East expert at the Russian International Affairs Council stated that Saudi investments help provide Russia with relief from Western sanctions. “They do not fully take away the pain [of Western sanctions], they do not fully resolve the problem,” he said. “But it is one of the alternative sources of foreign direct investment into the Russian economy.” Khlebnikov added, “There is big potential, in the future [Saudi investment] is likely to increase because the dynamics are favorable.” However, Yuri Barmin, Middle East and North Africa Director at Moscow Policy Group, offered a more pessimistic assessment about the long-term prospects for further Saudi investment in Russia. “The problem is that the Saudis are very conservative investors and Russia politically is a very risky market to invest in,” he said. Saudi Arabia’s economic interests in Russia have made it wary of U.S. efforts to increase pressure on the country. In March, Saudi Minister of Energy Khalid Al-Falih publicly opposed a new proposed U.S. sanctions package targeting Moscow. “Russia is a big supplier of gas to Europe, oil to China. If [new sanctions] happen, it will affect Europe, China and the whole world,” the minister stated. Saudi Arabia is also increasingly reluctant to criticize Russian foreign policy. In December 2017 and 2018, the kingdom abstained from two UN General Assembly resolutions condemning Russia conduct towards Ukraine.

This move marked a sharp change of policy. As late as 2016, Saudi Arabia had supported similar resolutions. For its part, Russia has defended Saudi Arabia from international criticism over the kingdom’s feud with Canada and the killing of Saudi dissident Jamal Khashoggi. Khlebnikov explained that Riyadh and Moscow have an unspoken understanding to not criticize one-another’s foreign policy. (RELATED: Behind The Growing Rift Between Russia And Iran) “Russia avoids offering commentary and condemnations of Saudi Arabia’s war in Yemen, and Saudi Arabia likewise takes a neutral position towards Russian policy in Ukraine,” he said. Despite the improved relations between Russia and Saudi Arabia, all the Russian experts The Daily Caller spoke to emphasized that the two countries remained far away from a partnership. Saudi Arabia and Russia still have numerous conflicts of interest. Moreover, Riyadh’s alliance with Washington and Moscow’s strong ties with Tehran limit the ability of both countries to become fully-fledged partners. So far, however,

the two countries have managed to draw closer together despite their differences. All the more is impressive is how far this surprising development has fallen under the radar. At a time when Saudi Arabia has successfully pushed Washington to get more involved against Iran, it has also made significant inroads with a top global competitor of the

United States. By upping its Russia investments and striking oil deals with Moscow, Riyadh has helped the Kremlin avoid international isolation and strengthen its economy. Saudi Arabia and Russia may not be headed for a

partnership, but Washington is unlikely to welcome the current trajectory of their relationship.

Growth key to the environment – tech improvement, link turns case.Daniel Ben-Ami, journalist/author specializing in economics, editor of Fund Strategy, The Independent, 10/20/2010, http://blogs.independent.co.uk/2010/10/20/prosperity-without-growth-a-contradiction-in-terms/

Of course it does not follow from the enormous benefits of growth that the world is perfect. Far from it. But what we need is more growth, rather than less, to overcome the remaining challenges. Take the example of climate change. The conventional green-tinged view is that we need to limit economic growth to stop us destroying the planet. From this perspective the emphasis

must be on individual consumers making sacrifices by consuming less. Such an approach could not be more wrong. To tackle climate change, to the extent it is a problem, we need more resources and better technology rather than less. For example, we can construct more nuclear power stations, build higher sea walls to resist flooding and investigate more high technology solutions. We need an investment in practical engineering-based solutions rather than cheap moralising by politicians and pundits. The end of scarcity is a precondition for human beings to achieve the good life. Economic growth, and social progress more generally, is central to the full realisation of our humanity.

Elitist backlash to transition will kill billions

Page 43: openev.debatecoaches.org€¦  · Web view1AC . Observation 1: If I could find the spot where truth echoesI would stand there and whisper memories of my children's future. I would

Lewis, professor of American Studies at University of Colorado, 1998 (Chris, The Coming Age of Scarcity, ed Dobkowski and Wallimann, p 56-57)

Most critics would argue, probably correctly, that instead of allowing underdeveloped countries to withdraw from the global economy and undermine the economies of the developed world, the United States, Europe, and Japan and others will fight neocolonial wars to force these countries to remain within this collapsing global economy. These neocolonial wars will result in mass death, suffering, and even regional nuclear wars. If First

World countries choose military confrontation and political repression to maintain the global economy, then we may see mass death and genocide on a global scale that will make the deaths of World War II pale in comparison . However, these

neocolonial wars, fought to maintain the developed nations’ economic and political hegemony, will cause the

final collapse of our global industrial civilization . These wars will so damage the complex economic and trading networks and squander material, biological, and energy resources that they will undermine the global economy and its ability to support the earth’s 6 to 8 billion people. This would be the worst-case scenario for the collapse of global civilization.

Transition isn’t coming now – a large effort would have to be made

Florea 8 (Adrian, The Limits of Zero Economic Growth Strategy, University of Oradea, Faculty of Economics, http://imtuoradea.ro/auo.fmte/MIE_files/FLOREA%20ADRIAN%201.pdf)

The deep and the comprehensive character of the ecological crisis, but especially the extremely serious consequences and risks that it generates, maintain and amplify, menacing the existence of economy and human society, the overcoming of ecological crisis, or at

least the diminution of its appearances, represents today an absolute priority. As for the achievement of this goal, we consider that it cannot be left to the sudden mechanisms of nature and its laws, and moreover it cannot be left to the spontaneous mechanisms of the market. On the contrary we think that it will be possible to be done only if the actions done in this respect are based on a huge and deeply based strategy . In the following study we will present some of strategies that we consider as the most efficient, strategies that would have real chances of diminution and even elimination of the ecological crisis.

Econ collapse furthers the poverty gap between the races. White people are armed in the squo, they can redistribute wealth with their weapons, thus Econ collapse is anti-black, leads to factioning and dismantles communities of care

Ehrenreich, author and president of United Professionals, 09 (Barbara, 8/4/09, “The Destruction of the Black Middle Class,” http://www.huffingtonpost.com/barbara-ehrenreich/the-destruction-of-the-bl_b_250828.html, rn)

Left out of the ensuing tangle of commentary on race and class has been the increasing impoverishment -- or, we should say, re-impoverishment -- of African

Americans as a group. In fact, the most salient and lasting effect of the current recession may turn out to be the decimation of the black middle class. According to a study by Demos and the Institute for Assets and Social Policy, 33 percent of the black middle class was already in danger of falling out of the middle class at the start of the recession. Gates and Obama, along with Oprah and Cosby, will no doubt remain in place, but millions of the black equivalents of Officer Crowley -- from factory workers to bank tellers and white collar managers -- are sliding down toward

destitution. For African Americans -- and to a large extent, Latinos -- the recession is over. It occurred between 2000 and 2007, as black employment decreased by 2.4 percent and incomes declined by 2.9 percent. During the seven-year long black recession, one third of black children lived in poverty and black unemployment -- even among college graduates -- consistently ran at about twice the level of white unemployment. That was the black recession. What's happening now is a depression. Black unemployment is now at 14.7 percent, compared to 8.7 for whites. In New York City, black unemployment has been rising four times as fast as that of whites.

Lawrence Mishel, president of the Economic Policy Institute, estimates that 40 percent of African Americans will have experienced unemployment or underemployment by 2010, and this will increase child poverty from one-third of African American children to slightly over half. No one can entirely explain the extraordinary rate of job loss among African Americans, though factors may include the relative concentration of blacks in the hard-hit retail and manufacturing sectors, as well as the lesser seniority of blacks in better-

paying, white collar, positions. But one thing is certain: The longstanding racial "wealth gap" makes African Americans

Page 44: openev.debatecoaches.org€¦  · Web view1AC . Observation 1: If I could find the spot where truth echoesI would stand there and whisper memories of my children's future. I would

particularly vulnerable to poverty when job loss strikes. In 1998, the net worth of white households on average was $100,700 higher than that of African Americans. By 2007, this gap had increased to $142,600. The Survey of Consumer Finances, which is supported by the Federal Reserve Board,

collects this data every three years -- and every time it has been collected, the racial wealth gap has widened. To put it another way: in 2004, for every dollar of wealth held by the typical white family, the African American family had only one 12 cents. In 2007, it had exactly a dime. So when an African American breadwinner loses a job, there are usually no savings to fall back on, no well-heeled parents to hit up, no retirement accounts to raid.

Attempting dedev ensures extinction

Zey, director of the Expansionary Institute, Professor of Management at Montclair State University 1998 (Michael, Seizing the Future, p. 34, pp. 39-40)

However, no outside force guarantees the continued progress of the human species, nor does anything mandate that the human species must even continue to exist. In fact, history is littered with races and civilizations that have disappeared without a trace. So, too, could the human species. There is no guarantee that the human species will survive even if we posit, as many have, a special purpose to the species’ existence. Therefore, the species innately comprehends that it must engage in purposive actions in order to maintain its level of growth and progress . Humanity’s future is conditioned by what I call the Imperative of Growth, a principle I will herewith describe along with its several corollaries. The Imperative of Growth states that in order to survive, any nation, indeed, the human race, must grow, both materially and intellectually . The Macroindustrial Era represents growth in the areas of both technology and human development, a natural stage in the evolution of the species’ continued extension of its control over itself and its environment. Although 5 billion strong, our continued existence depends on our ability to continue the progress we have been making at higher and higher level s . Systems, whether organizations, societies, or cells, have three basic directions in which to move. They can grow, decline, or temporarily reside in a state of equilibrium. These are the choices. Choosing any alternative to growth, for instance, stabilization of production/consumption through zero-growth policies, could have alarmingly pernicious side effects, including extinction . He Continues. . . The fifth corollary of the Imperative of Growth claims that a society can remain in a state of equilibrium only temporarily. In reality, a society seemingly in a phase where it neither improves nor regresses is actually in a transition to either growth or decline. Such periods easily seduce their contemporaries into a false sense of security, that their institutions will last forever, they have all the science they need, and there are no more challenges. In fact, during such periods some imagine that they have reached their “golden age,” perhaps even the “end of history.” During such periods of supposed equilibrium, the population ceases to prepare itself for new challenges and becomes risk averse. Importantly, they reject the idea that growth and progress are necessary for their survival. The sixth corollary evolves from the fifth. If the system chooses not to grow, it will decline and eventually disappear , either because other organisms or systems overtake it or because it is impossible to maintain itself even at static levels without in some way deteriorating. This is the Law of Spiraling Regression. It is indeed a curiosity of the late-twentieth-century culture that this truism has been ignored. In the morass of claims about the risks of technological growth and its impact on the ecosystem, the mainstream media and orthodox a cademics have decided not to consider what harm the full pursuance of zero growth or non growth might inflict on the sociotechnical system, which includes our technological infrastructure, culture, and standard of living.

The worse it is now, the better – the banking sector needs a sustained crisis to cut off failing banks - perception of short-term turnaround only encourages more bubble-inducing speculation

Page 45: openev.debatecoaches.org€¦  · Web view1AC . Observation 1: If I could find the spot where truth echoesI would stand there and whisper memories of my children's future. I would

Morris ‘8 (Charles, Practicing Attorney and former banker, 11-22, 2008, “The upside to a serious downturn,” Houston Chronicle, online: http://www.chron.com/disp/story.mpl/editorial/outlook/6126845.html)

The scale of that bubble is reminiscent of the price-inflation bubble that bedeviled President Carter. So are the policies being used to deal with it. Carter and his hapless Fed chairman, G. William Miller, flooded the economy with dollars even as consumer price inflation spiraled out of control . Volcker took over the Fed in 1979 and, by previous standards, moved aggressively his first year in office. But he made little headway. Finally, in late 1980, he cracked down hard and significantly raised interest rates. Unemployment soared from 5.8 percent to 9.7 percent. Inflation stubbornly held on but finally broke in 1983. For the next several years, Volcker continued to crack down at the slightest hint of price buoyancy, until the markets took for granted that the United States was a low-inflation economy. The 2008 analogue to the Volcker strategy would be to force a harsh, fast marking-down of all bank assets to real values. A one- to two-year bloodbath is far preferable to a decade of death by a thousand cuts. Many banks would fail and would have to be re-equitized by the government — the terms should be neither punitive nor excessively generous — but the weakest and the most irresponsible should simply be let go . The banking system that emerges should be dull — one where credit analysis trumps financial engineering and where everything is on the balance sheet. The big Canadian banks , RBC and TD Bank, have been determinedly dull in the 2000s and have turned in superb profits , far outperforming their supposedly brilliant American cousins . Shrinking the banking sector will curtail bubble-style lending and force the share of GDP represented by consumer spending back down from its current 70 percent to a more sustainable 65-66 percent. It will be very painful, putting many companies in jeopardy, but it is the only way to engineer a transition to a world in which we spend less on houses and TVs and more on infrastructure and health care. Interest rates will be higher to encourage savings and taxes will go up, but debt should go down and the bottom half of the population should be more secure. It will also be very important to shore up our tattered social safety net to cushion the recession's impact on that lower half.

3. Perception of financial crisis is key to drive widespread saving – that’s the most important factor in making growth sustainable

Housel ‘8 (Morgan Housel, Contributor to The Motley Fool, a stock/investment analysis firm, 10-23, 2008, “A Silver Lining to the Financial Crisis,” online: http://www.fool.com/investing/international/2008/10/23/a-silver-lining-to-the-financial-crisis.aspx)

It's hard to find anything to smile about in the economy these days. Portfolios are in tatters. Jobs are being slashed. The market can't find a floor. National debt can't find a ceiling. Inflation, oil, home prices … it's not hard to see why the myriad issues draw comparisons to the Great Depression. Yet beneath the ruckus, one statistic -- perhaps the most vital component of getting back on a sustainable track -- is doing better than it has in years. People are saving money again. And in a big way The Bureau of Economic Analysis reports consumers saved an annualized $297 billion in the second quarter -- more than 10 times higher than the first quarter, and the highest quarterly savings since 1995. From 2004 through 2007, the average rate was around $85 billion. In the third quarter of 2005, things got really out of hand, with a negative savings rate … meaning that as a collective body, living paycheck-to-paycheck meant you were ahead of the pack. Thank goodness those overextended days are gone. It's about time As was the case with oil prices, people don't start changing their ways until the problem is shoved down their throat. It took $4 gasoline to get people to wake up, and to get

Page 46: openev.debatecoaches.org€¦  · Web view1AC . Observation 1: If I could find the spot where truth echoesI would stand there and whisper memories of my children's future. I would

innovation from companies like Sasol (NYSE: SSL) and electric cars from Toyota (NYSE: TM) and General Motors (NYSE: GM) to start being taken seriously. Necessity is the key to innovation , the proverb goes . The same goes for saving. As Lehman Brothers, AIG, and Bear Stearns will tell you, greed clouded by optimism can be an awesomely overwhelming force. Saving went out of style because, at the rate things were going, few saw a need to. Why save when your house went up by 20% every year? That's all changed in the past few months as people are realizing that Visa (NYSE: V), MasterCard (NYSE: MA), and American Express (NYSE: AXP) will make you richer in your brokerage account than they will in your wallet. As reality sets in for millions of Americans blindsided by home foreclosures and job losses without a safety net to fall back on, people are scrambling to rebuild and replenish their personal balance sheets by saving again. And that problem was yours and yours only Which brings up another point: Can we really put all of the blame for our current woes on Wall Street? Of course, the army of Harvard-stupid Wall Streeters (thanks, Bill) were leading the charge, orchestrating the disaster we're now facing. They screwed up. They should pay the price. And they are. But the larger picture is that it wasn't just Wall Street gorging on more debt than they could handle and taking excessive risks. It was nearly everybody. By 2005, over a quarter of Americans' income went toward revolving debt payments. Debt was the lifeblood of the economy, and the chickens have come to roost. I dug up a few headlines from major newspapers and magazines, all published between 2001-2006, that might remind us how overextended we were. Here are a few: * Saving in 2005 Worst Since 1933 * Equity Shrivels as Homeowners Borrow and Buy * As Personal Savings Fall, a Comeuppance Is Due * The Way We Live Now: Home Sweet Debt * Our Vanishing Savings Rate * Overwhelming Majority of Americans Feel Consumers Are Taking on Too Much Debt So, yeah, maybe it wasn't just Wall Street When AIG (NYSE: AIG) gets in over its head, it's a sin, and they're a criminal. When John Q. Citizen gets in over his head, it's a tragedy, and he's a victim. That blame game is an irony to this debacle that'll likely only prolong our recovery. But enough pointing fingers. For once, let's focus on the good news: People are saving again! How does that help the economy? Countries , like people or business, grow wealthier over time by saving money and investing it wisely . It really is about as simple as that. True, our economy managed to grow over the past decade with little or no savings. How? We borrowed. And that's why it's falling apart today. When an economy saves money, it has the resources it needs to invest without borrowing from someone else (Chinese and OPEC nations), and it creates real wealth . Domestic savings creates a pot of money borrowers can turn to for investments in projects that eventually create jobs, keep interest rates low, and push the economy along at a sustainable pace. The important part is that it's our savings -- not savings from foreign nations we became beholden to over the years to sustain our lavish lifestyles. The bottom line is simple: people -- and economies -- that work hard and spend less than they make will grow wealthy. People -- and economies -- that borrow heavily and spend more than they make will grow poor .

Russia is actively looking to fill in – turns case - Russia arms are cheaper, better, and there are no strings attached.Borshchevskaya ’17 - (Anna Borshchevskaya; Senior Fellow at The Washington Institute, focusing on Russia's policy toward the Middle East, PhD candidate; “The Tactical Side of Russia’s Arms Sales to the Middle East.”;

https://jamestown.org/program/tactical-side-russias-arms-sales-middle-east/; December 20th, 2017; HS)

Russia is the world’s top arms exporter, second only to the United States . The Middle East and North Africa (MENA) region has emerged in recent years as Moscow’s second most important arms market after Asia. Moscow has made great strides in this region since Vladimir Putin came to power, and

especially in recent years, after it embarked on major military reform following August 2008. Arms sales matter to the Kremlin because they are a major source of financial

Page 47: openev.debatecoaches.org€¦  · Web view1AC . Observation 1: If I could find the spot where truth echoesI would stand there and whisper memories of my children's future. I would

gain, but these arms sales are also a tactical foreign policy instrument for wielding influence. Russia’s arms —generally speaking—are well made, sometimes on par with the US, and well suited for the region’s needs. These platforms and armaments are also more affordable than Western weaponry. The US simply will not sell weapons to certain countries, which, therefore, turn to Moscow . Politically, Russian arms come with few strings attached and thus are a great choice when a country wants to diversify away from the West , or at least signal such an intent. Moscow has made inroads with traditional clients such as Iran, Syria and Egypt, but also diversified toward countries closer to the West, such as the Arab Gulf states, Morocco

and Turkey. Russia’s overall influence in the region is growing in the context of Western retreat .

Specifically, we’re on the brink of Russia-Ukraine war – the plan emboldens Putin to act further. There is literally zero evidence of Putin wanting to cooperate. Reuters 12/22Reuters, 12-22-2018, "U.S. to provide Ukraine with extra $10m in military aid after Russia attack," haaretz, https://www.haaretz.com/us-news/u-s-to-provide-ukraine-with-extra-10m-in-military-aid-after-russia-attack-1.6767632

The United States will provide an additional $10 million in military financing to Ukraine to bolster its navy after Russia captured three Ukrainian vessels at sea last month , in a “dangerous escalation , ” the State Department said on Friday. The move comes after Lithuania and the United Kingdom increased their security assistance to Ukraine, the

department said, following the November 25 attack near the Kerch Strait. “The United States calls on Russia to immediately return to Ukraine the seized vessels and detained Ukrainian crews, to keep the Kerch Strait and the Sea of Azov open to ships transiting to and from Ukrainian ports, and to respect Ukraine’s sovereignty and territorial integrity,” the State Department added. Meanwhile,

more than a dozen SU-27 and SU-30 fighter jets which Russia is deploying to boost its air force arrived in Crimea on Saturday. A Reuters witness saw the jets land at Belbek air base in Crimea, which was annexed by Russia in 2014 after Moscow-

leaning Ukrainian President Viktor Yanukovich fled Kiev following street clashes and violent protests. Tensions between Moscow and Kiev have risen in the past weeks after Russia seized three Ukrainian navy ships and their crews on Nov. 25 in an incident which Moscow and Kiev have blamed on each other. Russian Foreign Minister Sergei

Lavrov has alleged Ukraine was preparing “a provocation” near Crimea before the end of the year. Russia opened fire on the Ukrainian ships and then seized them and their crews near Crimea - which Russia annexed from Ukraine in March 2014. The Kerch Strait is the only outlet to the Sea of Azov and controls access to two major Ukrainian ports. The incident prompted U.S. President Donald Trump to call off a meeting with Russia’s Vladimir Putin in Argentina to signal Washington’s disapproval of Russian behavior in the naval clash with Ukraine.

OPEC is dead makes their impacts non-unique.Butler 19(Nick Butler, FT energy commentator and chair of The Policy Institute at King’s College London, “The weakness of Opec+ is evident”, 07/08/2019, https://www.ft.com/nick-butler)//ShreyasThe agreement between the oil cartel Opec and a group of non-Opec countries led by Russia to maintain the quotas they established last December for another six to nine months confirms the shift in power within the market away from the producers. In the short term, the defensive deal agreed in Vienna is designed to prop up prices at their current levels. The cartel and its new allies, including Russia, Kazakhstan and Mexico, will continue to take some 1.2m barrels a

day of potential supply off the market. But the immediate market reaction and the further fall in prices in the following days show that the shift in income and wealth away from producers is one that even an agreement by Opec+ — led by Russia and Saudi Arabia — cannot halt. Last week’s formal deal came as no surprise. Nor was it surprising that the once hugely proud and exclusive club Opec agreed to create a permanent alliance with the other countries. The

Page 48: openev.debatecoaches.org€¦  · Web view1AC . Observation 1: If I could find the spot where truth echoesI would stand there and whisper memories of my children's future. I would

need to cling together was evident to everyone — although some, including the Iranians, remain unhappy about the dominance of Russia and Saudi Arabia in the decision-making process. The exporters are being forced to adapt by the impact on the oil market of the one major player not represented in Vienna last week: the US. American shale production — up from almost nothing 10 years ago to 8.5m b/d in May with a further rise predicted over the next five years — has reshaped the entire global market. Because of that rise, the loss of production from Venezuela, Libya and Iran has been shrugged off. In addition, the US has met most of the gradual increase in global demand seen in the past

few years. What happens next in a market over which Opec has lost control depends on the demand side of the equation. The prospect of a serious trade conflict between the US and Beijing remains, and there are significant signs of a slowing of China’s economy and therefore of its oil import requirements. Since

these are running at around 9.5m b/d, any variation in China’s needs swings the physical market and the surrounding speculation. The most recent oil market outlook from the International Energy Agency predicts global demand growth of 1.2m b/d this year but that figure could be revised downward, as it was last month. If demand remains soft, with prices pushed down as Iran tries to beat US sanctions by offering discounts to buyers, the Opec+ quotas will not go far enough. Saudi Arabia, which has already cut its production by some 600,000 b/d beyond its agreed curbs, will have to do even more. It is tough to see which — if any — of the other producers would be willing or able to cut further by any material amount. The intriguing player in the new pattern of relationships is Russia, which seems to have finally accepted that it is a petro-economy whose interests are aligned with those of the Opec states such as Nigeria and Kuwait. The shift is a sign of President Vladimir Putin’s pragmatism and grasp of the dynamics of the market but also shows Russia’s fundamental economic weakness. Oil and gas account for more than 60 per cent of its export earnings and around 50 per cent of state revenues. The diversification of the economy promised when Mr Putin came to power 20 years ago has never happened. Russia’s vulnerability lies in the fact that, for reasons beyond its control, excess supplies of both oil and gas are undermining prices and therefore reducing the flows of revenue through the state companies. The new alliance between Moscow and Riyadh renews a link established in 1926 when the Soviet Union was the first foreign power to recognise Abdul Aziz al-Saud, grandfather of the current king, as the ruler of the Hejaz and Nejd — the forerunner of the modern kingdom of Saudi Arabia. Relations have not always been smooth, particularly when the Saudis were funding the Afghan mujahideen as they drove out the Soviet army in the 1980s. Today’s relationship is one of necessity, and it will be severely tested if oil

prices drop further. Falling prices mean falling revenues, giving every producer in the new grouping — including Russia — the incentive to break their quotas and produce a little more. Last week’s deal is fragile and cannot disguise the fundamental shift of income and wealth away from the oil exporters. Opec’s glory days are over . The world’s call on the cartel’s oil is down to levels not seen since the early 1980s. Over the past five years, the market’s move from being producer-led to a situation where the risks to prices, even after they have fallen from $110 a barrel in the

early months of 2014 to just above $60 now, are still predominantly on the downside. This could be from weakening demand or, through political change, some restoration of production in Venezuela or Iran. Short of a war or revolution in the Middle East taking out substantial supply for a sustained period, Opec+ will need to at least maintain its quotas far beyond March. As the reaction of the market to last week’s announcement showed, there is deep scepticism about the ability of the producers to hold together for a long defensive fight.

Marx et al 7 says climate change must “ facilitate this interaction is through group and participatory decision-making.” Means the aff must allow the neg to choose to not do the aff, meaning no perm

Alexander and Rutherford 14 is totally out of context it’s referring to “eco-anarchists” and advocates in the “deep green school” Using this to perm the neg is a violent reductionist flattening of anti-black violence.

Saudi wouldn’t enact countermeasures – relations high.Al Arabiya English 19 (Staff Writer for the Al Arabiya English, “Saudi Ambassador to the United States presents credentials to President Trump”, Al Arabiya, 07/09/2019, http://english.alarabiya.net/en/News/gulf/2019/07/09/Saudi-Ambassador-to-the-United-States-presents-credentials-to-President-Trump.html)//ShreyasSaudi Ambassador to the United States Princess Reema bint Bandar al-Saud has met with US President Donald Trump and presented her credentials as the Kingdom’s new diplomat to Washington. “Presented

credentials today at the White House. Conveyed the well wishes of Kingdom’s Leadership. Looking forward to working on strengthening and solidifying the historic Saudi - US partnership ,” the ambassador tweeted. “The Saudi-US

Page 49: openev.debatecoaches.org€¦  · Web view1AC . Observation 1: If I could find the spot where truth echoesI would stand there and whisper memories of my children's future. I would

partnership is essential to the interests of both countries and expressed confidence that the two countries are capable of overcoming any challenges at the regional or global levels ,” the embassy quoted the ambassador as saying. Princess Reema bint Bandar al-Saud arrived in Washington DC to begin her duties last week as the ambassador of Saudi Arabia to the United States. She became the 11th Saudi ambassador to the US since 1945 and also the first Saudi woman to hold the post. On her first day, the Saudi ambassador met with the US Assistant Secretary of State for Near Eastern Affairs David Schenker where they discussed

ways to strengthen relations. Princess Reema is the daughter of Prince Bandar bin Sultan , who also served as Saudi Arabia’s ambassador to the US from 1983 to 2005. Prince Bandar was previously the secretary general of Saudi Arabia’s National Security Council and he also served as director general of the General Intelligence Presidency.

Plan gets circumvented – Trump loves Saudi – empirics prove.Billings 19 (Kevin Billings, International Business Times Writer, “US-Saudi Arabia Relations: Pompeo Doesn't Include Saudis On List Of Countries Recruiting Child Soldiers”, International Business Times, 06/18/2019, https://www.ibtimes.com/us-saudi-arabia-relations-pompeo-doesnt-include-saudis-list-countries-recruiting-2801682)//Shreyas

Secretary of State Mike Pompeo blocked Saudi Arabia’s inclusion on a list of countries known to recruit child soldiers, according to a report from Reuters. Pompeo made the decision despite information allegedly provided by security advisors and experts’ findings regarding Saudi Arabia and its connection to the Yemen Civil War. The findings pointed toward the Saudi-backed coalition using underage soldiers in the conflict between the Houthi-led Supreme Political Council and the loyalist Cabinet of Yemen forces. Experts came across this information as they were working on a soon-to-be-released list of countries that are known or suspected to actively recruit and employ child soldiers. Information gathered from news outlets and human rights groups pointed to the Saudi coalition employing child soldiers from Sudan to fight in the war. These findings prompted internal debate between State Department experts and officials as to whether or not to include Saudi Arabia on the list. While experts wanted to include Saudi Arabia, some officials argued that it was not clear whether these were forces under the command of Sudan or were being directed by the Saudi coalition. Pompeo made the final decision that drew immediate blowback from human rights activists. Instead, Sudan will be added back onto the list after being removed in 2018. The Trump administration has previously faced scrutiny for siding with Saudi Arabia despite apparent human rights violations. President Trump faced criticism in the after math of Washington Post journalist Jamal Khashoggi's death at the Saudi consulate in Istanbul, as the president appeared to divert attention from possible Saudi involvement. Criticism heightened when the CIA released findings that confirmed Saudi Crown Prince Mohammad bin Salman had ordered the assassination of Khashoggi at the Saudi consulate in Istanbul.

Reducing military aid to Saudi Arabia would cause a flip towards RussiaBradley 12-1, John R. Bradley ( University College London, Dartmouth College and Exeter College, Oxford), 1 December 2018, Coffee House, 12-1-2018, "Has Saudi Arabia just pivoted towards Russia?," https://blogs.spectator.co.uk/2018/12/has-saudi-arabia-just-pivoted-towards-russia/ [mr.ghs]

For all but the most harried journalist motivated by a need to pay off the mortgage, the annual G20 summit – being held this weekend in Buenos Aires – is typically viewed as a perfect cure for insomnia. Who will stand next to whom in the family photo? Will the wording of a final statement be agreed by all leaders before the

official deadline? Yawn yawn yawn. However, there is an exception to every rule. And yesterday’s opening ceremony proved to be just that. First, a hot mic picked up parts of a tense conversation between the French President and Saudi Crown Prince. While hardly a slanging match, it was the most undiplomatic spat between a Western and Saudi leader ever made public. That was followed by the equally unprecedented spectacle of a bro-five between the

Page 50: openev.debatecoaches.org€¦  · Web view1AC . Observation 1: If I could find the spot where truth echoesI would stand there and whisper memories of my children's future. I would

latter and Russia’s Vladimir Putin. In the space of an hour, we may have witnessed what future history books will say was the moment Saudi Arabia’s relations pivoted from the West towards Russia. The relationship between the democratic West and theocratic Saudi Arabia has often been described as a marriage of convenience. The exchange between President Emmanuel Macron and Crown Prince Mohammed will certainly sound familiar to anyone who has been through a messy but not especially bitter divorce: MbS: Don’t worry. M: I do worry. I am worried, because I am very exp… MbS: He told me. Thank

you. M: I don’t want… MbS: No. M: You never listen to me. MbS: I will listen, of course. M: … MbS: It’s OK. I can deal with it. The official word from the Elysee Palace is that the President was giving the Crown Prince a ‘very firm’ warning over the murder in Istanbul of Saudi dissident Jamal

Khashoggi and the continuing humanitarian crisis in Yemen. In the Western media, the two issues have become inextricably linked. For having failed in their effort to oust the Crown Prince, there is now as a push – led by the Washington Post, which has tripled its coverage of Yemen during the past six weeks – to bring an end to the Yemen conflict as retribution for Khashoggi’s slaughter. And this is especially embarassing for Macron. As recently as April, he pledged France’s ‘full support’ for the Saudi position on Yemen, while slamming the Iran-aligned Houthi rebels the Saudis and their allies are battling against. Predictably, this came on the back of arms agreements worth $18bn. But Macron’s exasperated ‘You never listen to me’ may have been in reference to the Saudi ruler’s decision last year to kidnap Lebanese Prime Minister Saad Hariri during a visit to Riyadh, when he was beaten up and forced to read a resignation letter on Saudi TV. It was only after Macron stepped in that Hariri was finally released, and was subsequently able to resume his role as prime minister. In this way, Macron has been strutting around the world lecturing other countries and leaders how to behave as, his critics argue, a way of diverting attention from his own abysmal

failures at home. Now he is in the unenviable position of being not only the least popular French president in modern history but also aligned most closely abroad with a Saudi ruler who is perhaps the most despised in the world. Putin , of course, is watching all this with relish . He is experienced in the art of bumping off political opponents, and having eradicated almost all traces of meaningful democracy and freedom of expression at home is able to offer the Crown Prince the hand of friendship without fear of reprecussions. He laid the groundwork last year during an official state visit to Moscow by King Salman — the first ever by a reigning monarch. They signed a $3bn arms deal and pledged tens of billions more in bilateral trade. At the time, Russian Foreign Minister Sergei Lavrov described the event as ‘an historical

moment’. Now Saudi Arabia is even negotiating the purchase of Russia’s formiddable S-400 defense missile system, while threatening to cancel arms deals with the Western countires and instead purchase weaponary from Russia and China if sanctions are imposed over the Khashoggi affair . As with Syria and Iran, the Kremlin is demonstrating how, in contrast to the fickle West, it can relied on to remain a steadfast ally to countries in the Middle East it considers friendly, come hell or high water. The partnership between Saudi Arabia and Russia is not going to flourish overnight. But the crucial difference between our political leaders and theirs is that the latter think not in terms of election cycles while feeding at the trough, but rather how to forge deep strategic partnerships that will bear fruit decades, even centuries, in the future.

Russia – US presence in the Middle East is zero sum – Russia historically fills the gapByman 18, 4-11-18, Daniel Byman (foreign policy editor of Lawfare. He is a senior fellow at the Center for Middle East Policy at the Brookings Institution, where he focuses on counterterrorism and Middle East security. He is also a professor at Georgetown University's School of Foreign Service.) "Pushing Back Russia in the Middle East: A Thought Experiment," Lawfare, https://www.lawfareblog.com/pushing-back-russia-middle-east-thought-experiment [mr.ghs]

Russia is increasingly emerging as an enemy of the United States , not just a rival. Although President Trump generally seems to oppose any attempt to confront Russia—with the exception of a tweet this morning in which he warned the Kremlin to “get

ready”—it’s worth considering how a more strategically minded administration might do so, particularly in the Middle East, where Moscow has vastly expanded its influence. During the height of the Cold War, the United States reflexively opposed the Soviet Union and the spread of communism. In addition to shoring up allies in Europe and Japan, the United States often sought to hinder or roll back Moscow’s influence in parts of Africa and Asia, regardless of the minor strategic significance of the areas in question. In the Middle East, the U.S. opposition to the Soviet Union often manifested in efforts to sway and topple governments in Iraq and Syria and a competition for influence in Egypt, among other locations. At the Cold War’s end, the Soviet Union maintained some interest in and influence over Algeria, Iraq, Libya, Syria and South Yemen (the last of which would soon enter history’s dustbin). Yet for the most part, the United States had run the board, with close partnerships with key states such as Saudi Arabia, Jordan, Morocco, and Egypt as well as Israel and Turkey, the region’s military powerhouses. These close relationships continued after the Cold War’s end, and the United States even improved relations with several of Moscow’s former clients. The dominant U.S. position in the Middle East slipped under President Obama. Skeptical of

Page 51: openev.debatecoaches.org€¦  · Web view1AC . Observation 1: If I could find the spot where truth echoesI would stand there and whisper memories of my children's future. I would

intervention in the Middle East and unsympathetic to long-standing allies like Saudi Arabia, the Obama administration tried to keep its distance

from the region. Although the United States intervened in response to the Islamic State’s atrocities against the Yazidis and the broader concerns about terrorism, the Obama team still tried to avoid greater entanglement in Syria, distanced itself from Egypt after the 2013 coup, soured on Netanyahu’s government in Israel and limited efforts to deal with trouble spots like Libya and Yemen to a narrow counterterrorism mission. Even the administration’s successes, like the Iran deal, seemed to alienate many traditional American

friends. U.S. credibility in the region fell , and many leaders were glad to see Obama go. Moscow entered into the void created by the decline in the U.S. position . Russia backed its Syrian ally when Syrian President Bashar al-Assad was on the ropes, helping Assad’s government survive and slowly turning the tide against an opposition that Washington nominally championed. Moscow delivered for its allies and claimed victory as the Islamic State receded and the Syrian state recaptured much of its lost, though now devastated, territory. Moscow has also made inroads with former U.S. stalwarts : President Vladimir Putin recently visited Turkey, probably to coordinate the crushing of the Syrian Kurds who were the tip of the U.S. spear against the Islamic State. Russia also offered to sell Turkey advanced weapons. King Salman of Saudi Arabia, whose country had long scorned Russia, traveled to Moscow last October to talk

about shoring up the price of oil. Ideologically, Russia also fits in well with the Middle East. Putin, an autocrat himself , is comfortable with dictators and resents what he perceives as U .S. democracy promotion —a position that dictators in the Middle East also share. As Steven Cook notes, “Now it’s payback time for almost three decades of Moscow’s

humiliation. And what better place to start than the Middle East , where the United States is already widely resented even among its allies.”

Russian foreign policy exacerbates and retrenches authoritarianism in the Middle EastBarmin 18, Yuri Barmin (expert at the Russian International Affairs Council, covering the Middle East and North Africa, Moscow’s policy towards the region, as well as the conflicts in Syria and Libya. He regularly contributes his analysis to Al Monitor, the Middle East Institute, Al Sharq Forum and FARAS Center. Mr. Barmin holds an MPhil in International Relations from the University of Cambridge.), 3-8-2018, "Russia in the Middle East Until 2024: From Hard Power to Sustainable Influence," Jamestown, https://jamestown.org/program/russia-middle-east-2024-hard-power-sustainable-influence/ [mr.ghs]

Experts who had argued that authoritarianism in the Middle East would maintain stability and keep extremism at bay were proven wrong by the events of the Arab Spring .[xxxv] The Russian leadership , however, still projects its vision of “ autocratic stability ” onto the region . And even though Moscow repeatedly insists that it is up to the Syrian people to decide through a presidential election who will lead the nation into the post-war period, the

Russian government is unlikely to become a supporter of democracy movements in the Middle East. After all, elections have been a crucial legitimization tool of Russia’s own “managed democracy.” The consolidation of power in

the hands of the national leader as well as the securitization of the political agenda have characterized the Russian political system throughout the last 17 years Vladimir Putin has been in power. And they continue to guide him in how he sees regimes in the Middle East . Some of these authoritarian Arab regimes share a long history with Russia: during the Cold War, they proved their ability to maintain order for longer than any democratic regime could sustain it,

not least due to Moscow’s financial and military support. The fact that Bashar al-Assad survived throughout the bloody Syrian conflict , to a large extent due to Russia’s aid, solidifies the idea that authoritarianism in the Middle East guarantees stability and puts a cap on “toxic” democratic values imposed from the outside . In Moscow ’s view, authoritarian tendencies are indigenous to the region , much like they are to Russia, which is why they need not be battled but rather be correctly managed . Russia’s idea of “authoritarian stability” in the Middle East may find a potential supporter in Donald Trump, who notoriously dropped America’s agenda for promoting democracy in the

Middle East. The distinct security focus of Donald Trump’s strategy toward the region has emboldened his allies, Saudi Arabia and Israel, and convinced them that the regional policeman will no longer restrain their geopolitical ambitions. The position of both Russia and the United States is, thus, likely to resonate with many governments in the region that previously had to put on airs of civil society engagement and

Page 52: openev.debatecoaches.org€¦  · Web view1AC . Observation 1: If I could find the spot where truth echoesI would stand there and whisper memories of my children's future. I would

liberalization just to have international political and diplomatic backing. Egypt and Turkey are the two cases in point: the 2017 Human Rights Watch World Report specifically points to them to illustrate how the tide of new authoritarianism is sweeping through the Middle East.[xxxvi] In Turkey, the attempted coup in July 2016 was used by President Recep Tayyip Erdoğan and his Justice and Development Party (AKP) as an excuse to crack down not only on suspected plotters but also on wider circles critical of the government’s

policies. Western powers sharply rebuked Erdoğan over his suspension of the rule of law in the country and mass detentions—but Russia pointedly did not. Putin was the one world leader who gave a call to Erdoğan to tell him Moscow supports his campaign to root out dissent, which the Turkish president described as “anti-constitutional .”[xxxvii] Furthermore Putin hosted his Turkish counterpart in St. Petersburg less than a month after the failed coup, during which Erdoğan explained that Vladimir Putin’s call to him was an important move, “a kind of moral support and display of Russia-Turkey solidarity,” as the Turkish president described the situation.[xxxviii] All this occurred just weeks after Erdoğan’s late

June apology to Russia for the November 2015 downing of a Russian Su-24 jet over Syria; and it goes to show how masterfully

Vladimir Putin uses authoritarian movements to his own political benefit. Egypt is going through a similar wave of authoritarianism, with President Abdel Fattah El Sisi cracking down on dissent that is not necessarily associated with the Muslim Brotherhood. That government campaign is happening against the backdrop of economic instability, currency devaluation and increased poverty rates. However, the army’s grip on power and full control over the public sphere give a semblance of stability in the country. Sisi’s fight to eradicate extremism in the Sinai as well as his crackdown on dissent find support in Moscow, which is reflected in official statements coming from the Kremlin. Egypt reemerged as Russia’s key partner in the Middle East, including in crucial spheres of military-technical cooperation. The two countries signed a protocol on military cooperation in March 2015, significantly ramped up joint military exercises, and are looking to green light an agreement that would allow Russian military aircraft to use Egyptian airspace and infrastructure.[xxxix] With the turmoil and regular attacks in the Sinai Peninsula, counter-terrorism cooperation has become a distinct characteristic of the bilateral relationship. A security-heavy agenda acts as a glue between Moscow and Cairo, not least due to the military and security background of the political elites of the two countries. Both Russians and Egyptians will head to the polls in March 2018 to elect their respective heads of state, while presidential elections in Turkey are to take place in November 2019. The outcome is already known in all three countries; Putin, Sisi and Erdoğan will almost certainly serve out their next terms into the first half of the 2020s, meaning that we are unlikely to witness a disruption in the security-comes-first policy

employed by Moscow in its bilateral relations with both Cairo and Ankara. The cases of Egypt and Turkey illustrate that Vladimir Putin is likely to encourage authoritarian “stability ” across the region through skewed security-heavy policies . Putin’s support for autocratic tendencies will hardly find any resistan ce among other powers in the region and will almost certainly be embraced. Syria’s recovery from the seven-year war is unlikely to happen through the emergence of democratic institutions and freedom, but will probably lead to the creation of a strong regime with an inflated security appara tus to shield a fragile government and keep extremist tendencies at bay . Iraq’s increasingly sectarian policies hint at a similar trend . And as Libya’s internationally recognized government fails to establish control over much of the country’s territory, Libyan National Army Field Marshall Khalifa Haftar represents the type of leader the Kremlin would presumably like to see for a post–civil war Libya. If Russia’s Syria policy is any indication, a highly centralized system will be Moscow’s remedy for extremism throughout the wider region . The fear of a new wave of extremism will push many regimes to seek more control over the population, and a lack of incentives to democratize may bring about new repressive regimes. In other words Russia’s leadership in the Middle East may significantly lengthen and reinforce the era of authoritarianism there.

Page 53: openev.debatecoaches.org€¦  · Web view1AC . Observation 1: If I could find the spot where truth echoesI would stand there and whisper memories of my children's future. I would
Page 54: openev.debatecoaches.org€¦  · Web view1AC . Observation 1: If I could find the spot where truth echoesI would stand there and whisper memories of my children's future. I would

Soft Left UKRussia is continuously depicted as “Oriental” with an “Oriental mentality” by western scholars. Lipatti 17 (Ava Lipatti wrote this article published by the Hampton Institute https://www.newcoldwar.org/russophobia-and-the-logic-of-imperialism/) American cold war discourse about totalitarianism served a double function: in regard to the Soviets, it justified a policy of global anti-communism by reinterpreting all struggles for national self-determination in terms of the geopolitical contest for zones of power against totalitarian Russia; in regard to Nazi Germany, it saved the traditional pre-war faith concerning ‘the values of Western civilization’ held by post-war foreign-policy ‘wise men’ by displacing the human essence of fascism into the non-Western world… The necessary conscience-soothing exorcism was achieved by affirming the equation of Nazi Germany and Soviet Russia, combined with an historical interpretation of the essential Orientalness of the Russian mentality. The basic argument is that ‘totalitarianism’ is nothing other than traditional Oriental despotism plus modern police technology. The appearance of the first truly totalitarian state in the heart of Europe was thus an accident, explainable by the fact that the technology permitting totalitarianism was invented by Western science and was thus first accessible in the West. Moreover, Germany’s totalitarian moment is characterized by Kennan as a ‘relapse’ into barbarism; far from showing a flaw in

Western culture, it proved the need for constant alertness in preserving our distinctly Western values. A supposedly anti-racist theory reveals its racism in its implied upholding of “Western values”, a distinctly fascistic, colonial ideal. As “European civilization” faces an existential threat of “barbarism”, it tightens its ranks and purges itself of all but the purest elements.

According to the Orientalist worldview, Russians have only been able to masquerade as white due to their frequent contact with Europe. However, once the veil is lifted, an essentially Oriental mind is revealed. Pietz again writes: History – specifically the pre-modern geopolitics of the Eurasian ‘ecumene’ which produced the ‘Russian-Asiatic world’ – explains the Oriental essence of the Russian mind. This mentality is distinguished by its ability, after centuries of direct contact with Europe, to appear civilized and to use this facade of civility for its own barbaric ends.

Ukraine is positioned in a unique positions that constantly fluctuates between the East and West and this fluid change allows the west to construct it as the Orient.Habova 18, Yevhenjia Habova, Institute of Oriental Studies, National Academy of Sciences of Ukraine, 2018, “EAST-WEST DICHOTOMY IN THE CONTEXT OF UKRAINIAN CONFLICT RESOLUTION,” accessed 7/11/19, https://periodicals.karazin.ua/cognitiondiscourse/article/view/12270/11672 sfrrpt

Introduction In the trying times of the ongoing Russia-Ukraine conflict, it is natural to expect that Ukraine will seek support from more powerful partners. However, the reaction of international partners to the conflict is increasingly unenthusiastic, despite most recent aggravations in the Azov Sea. A growing awareness of the so-called ‘Ukrainian fatigue’ among international partners invites Ukrainian decision- and policy-makers, as well as general public, to reflect on Ukraine’s own resources, self- support and self-reliance. Alongside the necessity to find an optimal course of development, these issues have become increasingly important as the conflict shows no end. One of the arguments surfacing in this debate is that the East-West division within Ukraine may become internalized and impact the relations within the country, in the present and in the future. As such, analysis of images and narratives on East vs. West within Ukraine may assist in informing and fine-tuning a dialogue in the country and help to seek out compromises and roadmaps for reintegration of the occupied territories. This analysis also asks if

the notions of East and West influence how Ukraine sees its neighbours and the wider world. The ultimate aim of this paper is to identify the imaginary geography of Ukraine and trace Ukraine’s place on the mental map of the world within the coordinates from East to West and in the context of the ongoing conflict . This ‘mental mapping’ is argued to be instrumental in understanding images of external Others as well as images of Self. The data are collected from the interviews of Ukrainian elites – decision-, policy- and opinion- makers – representing different policy fields including media, culture, politics, civil society and business. The semi-structured interviews with Ukrainian elites, conducted face-to-face by pretrained researchers, were held in 2016-17 as a part of the Jean Monnet Network “Crisis, Conflict and Critical Diplomacy: EU Perceptions in Ukraine, Israel and Palestine” (C 3EU) led by the National Centre for Research of

Page 55: openev.debatecoaches.org€¦  · Web view1AC . Observation 1: If I could find the spot where truth echoesI would stand there and whisper memories of my children's future. I would

Europe, University of Canterbury (Christchurch, New Zealand) in cooperation with nine international partners (https://jeanmonnet.nz/c3eu/, see also Chaban and Zhabotynska 2018 in this Issue). In the analysis provided below, the respective references are "Media", "Culture", "Civil", and "Business" followed by a number of the interview, as it is registered in the C 3EU data; for example, Civil5. The questionnaires for the interviews focused on the perceptions of the EU in the context of conflict and crisis in Ukraine. The interviews were conducted in Ukrainian and Russian with responses transcribed verbatim and later translated into English. This paper uses quotations from the English version as a purely linguistic approach is not the goal in this case

study. Due to Human Ethics regulations, all responses will remain fully anonymous, and only the cohort will be identified when the words are quoted. While the questionnaire did not explicitly ask about “East vs. West” images, the interviewees often referenced and compared East vis-à-vis West, typically in order to highlight the differences between Ukraine and other actors. Following this empirical observation, this article sets to trace if there was a clear placement of Ukraine in these “mental mappings”. Where exactly is Ukraine’s place on the imaginary map of the world? Can the opposition “East vs. West” be explained by the influential theory of orientalism [Said, 1978]?

The theory hypothesizes a negative connotation assigned by the West to the East. The article respectively explores if the narrative of unconquerable discrepancies between Ukraine’s East and West regions exist in the imagination of the Ukrainian movers and shakers. The article also aims to map the imaginary geography in terms of “East-West” divide outside the country’s borders. 94 The theoretical framework section describes the theories behind the research, sets the focus of studying Ukraine’s vision of East, West and self. Said’s orientalism and Huntington’s clash of civilizations hypotheses inform the theoretical framework of this study. The analysis is also guided by a set of concepts developed by the scholars of Eastern Europe – Wolff [1994], Pittaway [2003], and Todorova [2009] among others – in highlighting the similarities and differences in the images of East vs. West in neighbouring countries. These theorisations are instrumental to explain the perceptions among Ukrainian elites (traced through the data collected from the interviews and discussed in

detail in the Findings section). The images are categorized into two main frames of “outside Ukraine” and “inside Ukraine” with several sub-categories – a conceptual architecture that explicates an elaborate and complex mental mapping of the world by Ukrainian policy- and decision-makers. The Conclusions section outlines how the

tested theories were partially disproved, showing that the East is not necessarily weak or exotic in Ukraine’s elite perceptions, and that the East vs. West opposition within the country’s borders is not internalized by the interviewed stakeholders. 2. Theoretical frameworks According to the theory articulated by Edward Said in his book Orientalism [1978], the pattern of dissecting the world into East and West has been present since the ancient times. One of the most influential works on the imaginary geography of the post-colonial world, Said’s analysis had become the trend-setter. It invited a generation of scholars to launch into further investigation how the East is perceived by the West and vice versa. Receiving a fair share of criticism for its historical inaccuracies and author’s personal bias, Orientalism nevertheless sparked an ongoing discussion that has only grown since the first edition of the book. This article is informed by Said’s theory of ‘strong West

vs. weak East’ -- as the result of inaccurate cultural representations. In his work, he claims that Orient not only was constructed by the West but also “has helped to define Europe (or the West) as its contrasting image, idea, personality, experience”

[Said, 1978: 13]. He also suggests that for centuries the Orient has been “a place of romance, exotic beings, haunting memories and landscapes, remarkable experiences” [Said, 1978: 2]. Using Ukraine’s experience, this paper explores how these claims have stood against time and the

effects of globalization, if their viability has suffered over the last four decades. Referencing a later work by Huntington -- the much debated Clash of Civilizations [1993] -- Ukraine may be described as one of the ‘cleft countries’ [Huntington, 1993: 30]. On the one hand, it belongs to the so-called Orthodox civilization. On the other, it has a large number of people who are identifying with a different, neighbouring civilization. In Ukraine’s case, the “other” civilisation may be located in the Western regions of the country that are predominately Eastern Rite Catholic or Ukrainian Greek Catholic . It is also possible to speculate whether Ukraine is a Huntington’s ‘torn’ country, i.e. a country that has made a drastic turn to change its civilizational path. Huntington [Huntington, 2013: 44] outlines three requirements for a country to redefine its civilizational identity in a major way: support of its own political and economic elites, approval of the public, and acceptance of the elites of the given civilization that a country is striving to join. So far, it is not clear if conflicted Ukraine can fully become a ‘torn’ country in Huntington’s sense, with EU membership not being on the agenda and without a transparent response on that matter from the West. However, Ukraine’s Association Agreement with the EU may be seen as a sign of at least a beginning of the acknowledgement of Ukraine as a country that belongs with the Western civilization. The image of the West and the Self in Ukraine have been scrutinized by scholars prior to Maidan and the following conflict [Gritsay & Nikolko, 2009; Tarasenko, & Ivanenko, 2004; Yavorska & Bogomolov 2010]. These works reported the emergence of the narrative of the ‘desired yet distant’ Europe and the importance of this narrative in the formulation of the national identity narrative in Ukraine. For Ukraine, Europe has been a desired and seemingly unreachable 95 destination for a long time, the perfect example to follow [Yavorska & Bogomolov, 2010: 86]. The image of Europe is mythological and mostly blended into the image of the West in general [Gritsay & Nikolko, 2009: 176]. Following the dramatic events on Maidan in 2013-14 and the later conflict in the East and annexation of Crimea, literature in the field debates the reasons underlying the Ukrainian crisis [Black et al., 2016; Merry et al., 2016]. While some researchers imply outside influence, including Russian propaganda, as one of the main causes [Kolstø & Blakkisrud, 2018], others point out to internal origins of the problem. Importantly, the latter group of scholars often cites crucial cultural and historical differences between Eastern and Western regions in Ukraine [see e.g. Besier & Stokłosa, 2017]. This article questions whether there is ground to these assumptions and tests them empirically. In this, the article innovatively adds to the discussion on Ukraine’s self-visions and identity in the post-Maidan period. Studying the perceived differences between East and West is by no means a novelty among historians, sociologists, economists, etc. Attempts to define borders within Europe and debating their existence brings a new perspective on putting Ukraine on this “philosophical map”. Norman Davies has set the precedent in the historical studies of shifting the focus from the predominantly Western-centered view of the European history and drawing more attention to the role of the Eastern and Central European countries in shaping the modern Europe [Davies, 2006]. He claims that the so-called East Europe is an inherent part of the Western civilization and should not be seen as subordinate. Mark Pittaway on the other hand goes even further, suggesting that both the internal and external borders of Europe are fluid [Pittaway, 2003] and cannot be defined. He also states that the “former socialist states are both part of outside Europe” [Pittaway, 2003: 156],

which includes Ukraine as a region that previously was on the other side of the “iron curtain”. Using evidence from maps, travellers’ memoirs and works of literature Wolff argues that the imaginary division between Eastern and Western Europe has been present since the Enlightenment and still determines not only the perception of the East as the “other” but the image of

Page 56: openev.debatecoaches.org€¦  · Web view1AC . Observation 1: If I could find the spot where truth echoesI would stand there and whisper memories of my children's future. I would

self in eastern countries. [Wolff, 1994; 16]. The complex and diverse Balkan region especially has been the focus of several studies in the context of new countries joining

the EU [Bideleux & Jeffries, 1998; Petrovic, 2014; Todorova, 2009]. Ukraine’s case, however, may bring an even more profound insight into the matter of these perceived divisions, especially at such a turbulent period of fighting the Russian aggression and striving to find support from its European partners, when finding its own place and stance is vital. 3. Findings Among the first observations is that the imagined geographical positioning of Ukraine in general seems to be very often identified vis-à-vis Russia. This place on the map of the world is seen to be problematic, as it means for Ukraine difficulties in avoiding conflict in the present and securing a peaceful future. A media professional comments, “Ukraine has such a geographical position that places it between the EU and Russia, … a very powerful country, both on the economic and political levels…” (Media7). This geographical position is seen by some to be a trap for Ukraine: ‘You cannot escape Russia, of course. We are just surrounded’ (Civil5), or ‘…we cannot get rid of geography, that is why Russia is important for us’ (Civil9). Yet, some see Ukraine on the move away from the East: ‘We announced the [European] vector, we are leaving. We are still Europeans in our mentality and can not belong to the eastern regions, that want to swallow us, to return us’ (Civil2). Others believe that being a country on the edge of two civilizations may be actually advantageous for Ukraine and the West. This presents Ukraine with an opportunity to play an important role on the international arena. One cultural elite compared Ukraine to a “stumbling block on the way of the eastern and southern and northern hordes” (Culture1). Another representative of the cultural circle stated: 96 Ukrainians are the “resource” for the EU as a civilized, cultural and educated workforce, especially for Eastern Europe - for Poland and Czech Republic. The thing is that Ukrainians have moved to Italy, France, Spain and Portugal earlier and this migration wave is at present moving towards Poland, Czech Republic, a bit less to Hungary. But still nobody denies the logistic importance of Ukraine (Culture7). Perceptions of the notions “East” and “West” among the interviewed elites may be grouped into two main categories: ‘outside Ukraine’ and ‘inside Ukraine’. 3.1. Perceptions of the East: Outside perspective If we consider the “outside Ukraine” perspective, the imaginary geography of East vs. West does not necessarily correspond to the real world mapping conventions, where Europe and Asia are divided by the Ural Mountains. Instead, geopolitical and cultural issues guide the construction of the perceived borders in the imagination of our respondents. Ukraine’s struggle in general is “the issue of us quitting to be part of Asia and moving to Europe” (Media2). The ‘East’ outside Ukraine has its own subdivisions in the eyes of Ukrainian elites: ‘Asia’, ‘Russia’, ‘Eastern Europe’ and ‘Eastern Partnership’ with the latter not a geographical, but a socio- political concept that is rather visible in the interviews. The notion of ‘Asia’ of the ‘outside East’ is comprised in the imagination of Ukrainian policy- and decision-makers of China, Japan and Turkey, while other Asian actors are largely invisible (e.g. “Eastern countries, we’ll name China, and perhaps even others - Japan and the like” (Civil2); “I would say, East - I mean Japan, China and Singapore” (Politics6)). The three visible actors are seen as rapidly developing countries that may potentially offer an alternative pattern of progress if Ukraine’s struggle to be accepted in Europe, or more generally ‘the West’, fails. A civil society representative, for example, argues that instead of relations with the EU, he would rather be talking about some closer relations with Turkey” (Civil4). A politician echoes, “Turkey is just one of the major players in the Black Sea region. ...It is clear that, maybe we need to establish or attempt to establish relations with China, because China is becoming a serious player…” (Politics3). Russia is seen as one of the key representatives of the ‘outside East’. It is often described as one of the great powers, an influential actor in the region along with the EU and the US: (“[...] other serious subjects which are the USA, Russia” (Media1)). Unsurprisingly, its image is ambiguous -- it is both an enemy and a former significant partner. As one media elite argues ‘From the state policy perspective, they [Russia] are our enemies, but from the people’s perspective… we have a million of relatives there, here and there, and they cannot be our enemies.” (Media3). Despite these connections, the path is seen to be changing for Ukraine: “previously, we tried to follow … how it should be in Russia. At present, we try to follow the West.” (Culture9). Another actor in the mental space of the ‘East’ is ‘Eastern Europe’. The interviewees often mentioned Eastern Europe as an important partner for Ukraine. It is typically represented by Poland and the three Baltic states. Consider a rather typical response by a civil society representative: “[They] are our partners on borders with Eastern Europe: it is Poland, the Baltic States, and partly Romania” (Civil9). These states are seen to have historical and cultural ties to Ukraine but at the same time belong to the ‘outside West’ represented by the EU. However, Eastern Europe is seen in a position not dissimilar to Ukraine - they belong to the West, but are not fully accepted there as its rightful members. This is despite having their status of EU member states. Moreover, this region is sometimes seen to be excluded from the very definition of Europe: “The reference of the notion "Europe" in most of its usage does not include the Eastern Europe, many parts. And [certainly] not Ukraine” (Civil5). East European countries are also viewed as advocate for Uk 97 be on their own, not always abiding by the EU rules. In this case the placement of these countries on the imaginary map is particular - regions located to Ukraine’s West are placed in the East and not just by the name, but by cultural proximity. Although the name itself is also seen as an issue. Business respondent comments, “I am very happy that they [young EU MSs like Poland and the Baltic States] are already named not “Eastern Europe”, but “Northern Europe” -- what they actually are” (Business5). Importantly, Ukraine is seen within the circle of the actors of “Eastern Europe”. On the one hand, Ukraine’s people are now a part of the societies in Eastern Europe: “Ukrainians are the “resource” for EU as a civilized, cultural and educated workforce, especially for Eastern Europe - for Poland and Czech Republic” (Culture7). On the other hand, Ukraine is an equal partner to Eastern European countries when it comes to security and defence matters: “[Ukraine is] in the military block of Eastern European countries, and, as a matter of fact, there is also an exchange of experience there. It is like a ‘micro NATO’, let’s call it that” (Business8). Another ‘inverted’ perspective surfaced in the imagining the ‘Eastern Partnership’. It was discussed in the interviews as a tool that is supposed to bring Ukraine closer to the West, but at the same time puts it among the countries that are not yet accepted by the EU as verified partners. The Eastern Partnership in fact is seen as not facilitating the relations with the EU, but making them vaguer and unclear. It is even implied that the very reason for its creation was “that its member countries could never be accepted to the EU (Media7)”. On the whole, ‘Eastern Partnership’ along with ‘Eastern Europe’ is seen to create a connection with the West: “the EU also had deep enough relationship with Ukraine within this Neighbourhood Policy, in the Eastern Partnership framework so it would not be acceptable for them to leave this game completely, well, to ignore the problem” (Civil4). Importantly, the EU’s Eastern Partnership is seen as policy that is designed to make its member “not members, but close friends, close partners” (Politics7) and Europe continues to “co- work with countries, organizations, with institutions … in other countries of Eastern Partnership” (Civil9). Yet, some respondents are less enthusiastic about the policy: “what is to be done with the East Partnership countries…(with) these six countries…[is] absolutely unclear” (Politics4). 3.2. Imagining East: Perspectives inside Ukraine Now, we are turning in our analysis to images of the space “inside Ukraine”. Naturally, war-torn Eastern Ukraine has a high profile in the discussions about the Ukrainian crisis and the EU’s involvement in the peacemaking process. One of the important observations here was that naming the conflict was problematic in itself for a number of interviewees. Some disagreed strongly with the usage of the term ‘conflict’: “Overall role of the EU in the war of Russia against Ukraine - and not a conflict! - Requires increased pressure on Russia and the rejection of double standards of the EU itself” (Culture2). Other descriptors were ‘what we have now going on in the East’ (Civil2), ‘the events’, ‘problems’ (Politics3), ‘the war’ (Media6), ‘attacks of Russia’ (Culture8), ‘warfare or outbreaks’ (Politics6). The situation in Ukraine’s East is frequently mentioned alongside another geographical indicator – the Crimea, or just South in general. Ambiguity of Russia’s image reminds how Huntignton used Ukraine-Russia conflict of 1991 over Crimea as an example of tensions within the same civilization: “Such conflicts, however, are likely to be less intense and less likely to expand than conflicts between civilizations. ...If civilization is what counts, however, the likelihood of violence between Ukrainians and Russians should be low. They are two Slavic, primarily Orthodox peoples who have had close relationships with each other for centuries” [Huntington, 1993: 38]. For some of the interviewees Russia is still a part of the same civilization as Ukraine, but current situation shows, that the conflict is not de-escalating, which may indicate that Ukraine is almost forced to become a “torn country” through this conflict. To sum up, none of the perceptions of the East, inside and outside of the country, appear to support Said’s orientalism theory. Apart from the perception of “Eastern Partnership”, that is seen as ambiguous and sometimes negative, the rest of the images contradict the negative “othering” of the East. Still, Ukraine is a part of the Eastern Partnership which makes it a self-image to a degree. 98 3.3. Perceptions of West: Imagining “Outside West” Diverse responses were observed when interviewees reflected on the concept of West. The West is general is frequently named as an important actor in the region. Yet, it is not seen as infallible and omnipotent. In the eyes of the elite respondents, it lacks integrity when it comes to dealing with external problems. Similarly to the notions of East, the perceptions of the West belong to two general groups: views of the ‘Outside West’ and ‘Western Ukraine’. The former notion has turned out to be a much more complicated and diverse one. With the interviews designed to investigate the image of EU in Ukraine, the ‘outside West’ is seen to be comprised of the EU (often represented as “Europe” or separate EU countries, such as Germany and France), the USA, and sometimes NATO. The notion of the West, however, is typically associated with the USA, and only then with Europe: “the so-called West, metaphorical one, because we are talking about the EU and about America and Canada [...] The West is primarily America.” (Civil4). Importantly, the West is seen to possess “its own values which it tries to disseminate in Ukraine” (Civil 9). One of the most typical visions of the ‘outside West’ was the one of an example for Ukraine to follow. The interviewees also felt that Ukrainians are supposed to belong to the Western society, but are not accepted by it yet because of the current state of events, and internal issues with corruption and slow reform implementation process. The ‘move’ towards the West is recognised in a paradoxical way – it is both inevitable yet seemingly fruitless at the same time. The West is believed to be not ready for Ukraine’s bureaucracy, poverty and territorial disputes. Nevertheless, many elites do not see other alternatives to Ukraine’s orientation towards the West: That is a lot of things are inherent in Western civilization, which unfortunately we do not have, that either they were not here or they were destroyed in totalitarian conditions [...] It is becoming more and more attractive to all those territories to want to return, so that they would fought to return. Just like East Germans fought for a return to West Germany. And I do not see and do not want it any other way (Civil4) Being an important actor, a supporter of Ukraine, the West is not seen to be willing to exert much power to help it or maintain its own interests nonetheless. Still, the interviewees understand the complexity of the situation and do not accuse the EU or the US – as the main representatives of the imagined West – of being neglectful. Still, there is a sentiment shared among elites that the West is not using sanctions or other measures against Russia to the full extent because that would disadvantage the West. These statements are often accompanied by reflections on the necessity of self-reliance and self-dependence, etc. Typical examples of such responses are below: ...the West could have achieved much more than with military force, because it is a tremendous economic power, and it could certainly ... find arguments both for Russia and China in order to persuade them to accept some compromise. ...the West has so many powerful cards that could be used, but for various reasons they are not … used (Civil4) At the same time, the West does not intervene, does not violate the sovereignty of the Russian Federation, but very significantly restricts its economic, political, and diplomatic abilities, because they (Russians) violate human rights. So the West remains holding the position, and the world, the civilized world remains holding the position that human rights are above all (Civil9) 3.4. Perceptions of West: Internal Perspectives Considering the “internal to Ukraine” interpretation of the “West”, Western Ukraine is seen as a link between the country and the ‘outside West’, as well as a region that responds to the European influences more eagerly: “The western region, Western Ukraine is more responsive to the European 99 Union” (Culture1). This is the region which is seen to be well linked with the ‘Eastern European’ part of Europe: “in

Page 57: openev.debatecoaches.org€¦  · Web view1AC . Observation 1: If I could find the spot where truth echoesI would stand there and whisper memories of my children's future. I would

Western Ukraine a certain image of Poland has already been formed, and the relations are clear there, they, nevertheless, began to understand that the same things should be done in the east of Ukraine” (Culture4). Western Ukraine is not mentioned as often as the East of Ukraine, which can be explained by the general context of the interviews that had specific questions about the conflict. It is mentioned only occasionally, either as a contrast to the East, or in a completely unrelated setting. A political interviewee argues, “we have a pro-European population, it mainly resides in Western and Central Ukraine, and certainly there are people who live under the pressure of Russian propaganda and that more are on the eastern Ukraine” (Politics7). Since the focus of the interviews was on the conflict in the East of Ukraine, it is understandable that Western Ukraine was mentioned less frequently. Still, the interviews materials point to a particular framing of this regions of Ukraine. Unlike the East, it appears to be closer to Europe not only geographically, but in terms of common history. The region is seen to become a basis for cooperation and more involvement than with the countries of Central and Western Europe. Some researchers of Ukrainian crisis point out to the discrepancies between its Western and Eastern regions as one of the underlying causes of the conflict [Black et al. 2016; Hahn 2017; Olchawa 2017]. Other speculate that it has been an issue even before the Orange Revolution of 2004 [Portnov2013: 241]. Comparing the perceptions of East vs. West within the country in the responses of interviews in our case does not support this premise. Only one of fifty interviewees, who also happens to be from the East of Ukraine, insisted on ‘Westerners’ being the ones to blame for the conflict in Donbass and justified the separatists’ actions with deeply rooted cultural differences: “many [in Eastern Ukraine] wanted federalization precisely for this reason, because they understood that this is a foreign culture for us, strange values for us, and that once they came to power by armed means, for us it was unacceptable” (Business1), at the same time rejecting Western values decisively: “I do not see a single value that we should have adapted and would be useful for us, not one.” (Business1). This point of view is not shared by other respondents. Moreover, some are rather sarcastic about these sorts of opinion: for some parts of society, particularly under the influence of Russian propaganda, it can also have a different interpretation, namely that ‘that darned EU, darned West started the fire, we used to live so well, peacefully, amicably, had a loving relationship with Russia and suddenly here they spoiled it all’ (Civil4). Regarding the image of the West in all of its variety, empirical findings seem to suggest that Ukraine is indeed a “cleft country”, yet not a “torn country”, as described by Huntigton. Without a clear approval from the imagined “outside West” in form of EU membership or substantial and decisive support in the conflict with Russia, Ukraine does not seem to be able to overcome the perceived limitations imposed by imaginary borders. At the same time, the vision of the West, embodied mostly by the EU, partially goes in line with Said’s “strong West” narrative. Nevertheless, the image of “strong Asia”, discussed earlier in this paper, makes the orientalism hypothesis inconclusive. 4. Conclusions This study traced the images of the East vs. West emerging in the interviews with Ukrainian elites about the image of the EU in the context of the ongoing conflict and crisis in Ukraine. Importantly, the questionnaire did not ask specific questions about the ‘imagined geography’ of Ukraine, and future studies may choose to focus on this research objective exclusively. This article presents an initial attempt to outline the mental map of the world along the East/West divisions in the eyes of Ukrainian people (in this case Ukrainian decision- and policy-makers). It is necessary to stress out 100 that both the notions of East and West are imaginary and their relations are complex and sometimes perplexing. The results can be grouped into two levels: “external frames” and “internal frames”. The “external frame” includes the notions of “East” represented by Asia, Russia, Eastern Europe and Eastern Partnership. The image of Asia has a definitively positive connotation and this finding contradicts Said’s orientalism hypothesis. In addition, Russia – a country to the East of Ukraine -- has an ambiguous image, albeit with an inclination to negativity. This finding does not fully correspond to the image of the exotic and demonised “Other”, proposed by Said. The notion of “Eastern Europe”, on the other hand has mostly positive connotations and is, to an extent, a part of Ukraine’s self-image. The image of “Eastern Partnership” supports Said’s theory, featuring negative associations attached to the notion of the East. These are also partially a component of Ukraine’s self-image. The image of the “Outside West” is complex and does in part resonate with Said’s concept of the ‘strong West’, however from Ukraine’s perspective, the strength remains unrealized due to lack of stern action in dealing with the conflict. The “internal frame” includes two main images - “East of Ukraine”, heavily associated with the conflict in the country, and “Western Ukraine”, seen being closer to Europe not just in a geographical manner. None of these notions is described either as better or worse, or stronger or weaker. While their images are contrasted to a minor extent, they are not seen as rivals or competitors. The basic contours of the ‘imaginary map’ of Ukraine traced through the responses of Ukrainian elites partially dismiss Said’s orientalism maxim of ‘West is strong; East is weak’. In the eyes of the Ukrainian respondents, countries in the Far East were seen as models of economic growth and development in contrast to Europe’s slow but noticeable decline. Moreover, they were seen as a source of alternative models for Ukraine to follow, in case its European orientation does not work out. The interviews were conducted during the period when the visa liberalisation regime for Ukraine had not been implemented yet. At that time, no-visas regime was believed to be unobtainable due to poor fulfilment of reforms on Ukraine’s side and reluctance on the EU’s side. The controversy surrounding the visa-free issue was viewed in the context of Ukraine’s relations with the EU, and more generally with the West. Indeed, it had a symbolic meaning of transcending the borders and getting closer to the West, or rather getting away from the East, and from being the ‘no man’s land’ between the two sides. From the elites’ point of view, the country is not yet a ‘torn’ country in Huntington’s sense, as it lacks the approval and acceptance from the symbolic West -- the EU continues to deny EU membership for Ukraine. But it is indeed seen as a ‘cleft’ country with the growing ties to the West, that are spreading further into Ukraine’s East. The world in the East-West coordinates is not multipolar, but is stretched between two opposing, equally distant epicenters of power, namely the US and China. There is very little visibility to the notions of Center, North or South. These were hardly mentioned in the interviews, and if mentioned then exclusively in the “outside frame”. Notably, Russia seems to have “moved” to the East since the beginning of the conflict: previously it often used to be referred as “the Northern neighbour” of Ukraine, but the interviewees seem to be inclined to associate it with the East (partially supporting Said’s theory). Notably, Ukraine is not the only country that is seen having a marginal status. Countries of Eastern Europe were seen to belong to the same group. While being Western in the definition by geography textbooks, they are not seen to be fully accepted by the West as an intimate part of it. Further research is needed for the perceptions of the imaginary geography of several most mentioned countries, such as Poland, Russia, China and the USA. They had a high visibility in the interviews even though they were not the main focus of them. The fluidity of Europe’s ‘imaginary borders’ provides an opportunity for Ukraine to overcome the perceived differences and to use the historical and cultural ties as an advantage in building new 101 and more far-reaching connections with the EU. Both the current conflict in the East and common history with many Eastern and Central European countries create a potential for fostering even closer mutual relations. However, there is a danger of crossing even those imaginary lines and creating unnecessary tensions, which is more that possible if there are no palpable guidelines. Perhaps the most important conclusion is that the data gathered in the interviews demonstrated that the East-West dichotomy is not fully internalized in Ukraine. Despite some historical discrepancies, in the eyes of Ukrainian elites there is no innate perceived opposition between Eastern and Western regions of the country. This finding is of critical importance to Ukraine. Considering limited external influences, a peaceful dialogue within Ukraine is possible through accentuating common ideals and values.

Orientalism is a process by which Western societies construct ‘Eastern’ cultures as backward and primitive - Ukraine as the backward ‘Other’ is associated with Russian politics Lyubchenko 17, Olena Lyubchenko,, University Karlovy Department of Political Studies Faculty of Social Sciences, 2017, “The Ukrainian Crisis: A Case of ‘New Orientalism’, accessed 7/11/19, http://search.ebscohost.com/login.aspx?direct=true&profile=ehost&scope=site&authtype=crawler&jrnl=18041302&AN=122705982&h=40SlBFAqkbLMjD5umVtsaujTuQ%2FyQH6K6PxJOOE%2B12IRQNznaRLzd3jkUvPcPoK8EKAgBiAKZISJGVFwfQX5tg%3D%3D&crl=c sfrrpt

Abstract: During the 2013 Ukrainian Euromaidan uprising and in its aftermath, many politicians, journalists, as well as academics diagnosed the Ukraine Crisis to be a manifestation of middle-class aspirations for a total social, political, and economic integration with the EU. Although correct in part, this account overlooks the

heterogeneity of the Euromaidan participants and the role played by radical right and nationalist groups. This paper examines the problematic coalition between liberalism and the radical right factions in the Euromaidan protests in Ukraine . More

specifically, it suggests the liberal project that is taking place in Ukraine depends on a specific form of the friend-enemy distinction of ‘New Orientalism’. By doing so, this paper presents a reading of Euromaidan through Carl Schmitt’s critique of liberalism. The discussion concludes with suggesting a political economy analysis of separatism in the Donbass region. Key words: Euromaidan; Ukraine crisis; liberalism; liberal democracy; far right; Donbass; new orientalism; Carl Schmitt; political economy Introduction The ongoing Ukraine crisis raises political issues that

Page 58: openev.debatecoaches.org€¦  · Web view1AC . Observation 1: If I could find the spot where truth echoesI would stand there and whisper memories of my children's future. I would

we thought were long buried in history. The post-communist state model is on trial once again. During the 2013 Ukrainian Euromaidan uprising and in its aftermath, many politicians, journalists, as well as academics2 diagnosed the Ukraine Crisis to be a manifestation of middle-class aspirations for a total social, political and economic integration with the European Union (EU). In a way, the story being told is of the completion of the bourgeois revolution that started in 1991 in the midst of the Soviet Union breakdown and was carried through with the 2004 Orange Revolution. This popular narrative follows the basic trajectory of the modernization theory: capitalism leads to democracy.3 Many academics and commentators claimed that with the end of the Cold War era and its competing worldviews, the spread of liberal democracy would create political homogenization and eliminate social conflict.4 The current Ukraine Crisis appears to be an example of the latter, transitioning Ukraine into a truly European liberal democratic and capitalist society. However, without much theoretical and historical analysis, the terms ‘capitalism’, ‘democracy’, ‘liberalism’ and ‘nationalism’ have been employed 1 Olena Lyubchenko is a doctoral candidate at the Department of Political Science, York University, Toronto. Contact: [email protected] 2 E.g.: Wilson 2014; Riabchuk 2013; Snyder 2015; Gershman 2015. 3 E.g.: Rostow 1956; Lipset 1959; Almond and Coleman 1960; Smelser 1964; Huntington 1968; Inglehart and Welzel 2005. 4 E.g.: Fukuyama 2011; Hoffmann 1987; Friedman 2007. 2017 | Vol. 9 | No. 1 46 interchangeably over the past two years, and the struggle for ‘liberal democracy’ continues to be framed as the ‘Ukrainian nationalist project’. The task of this paper is to demystify and complicate the mainstream narrative of Euromaidan as an organic democratic development. In the context of the Ukraine Crisis, the notion of ‘democracy’ was used rather vaguely. It tended to be associated with the notions of ‘freedom’ and ‘equality’ of the ‘Ukrainian people’; however, none of these terms was properly defined. This paper intends to interrogate the assumed unproblematic convergence between the social forces of liberal democracy5 and those of radical nationalism in the strife for Ukraine’s inclusion into the EU. I suggest that the alliance between the proEuropean liberal forces and the radical right was

enabled by the definition of a common enemy — the external, as well as internal, Russian hegemony. The European homogenization process involves (1) an ideological distancing from Russia and (2) a declaration of an internal enemy, via defining a section of Ukraine’s society as backward people. This has materialized in the ongoing civil war.6 Following Carl Schmitt’s critique of liberalism in The Concept of the Political (1976), i.e. that liberalism merely appears to nullify the friend-enemy distinction, I argue that the political has emerged in Ukraine. The current liberal project in Ukraine relies on a specific form of the friend-enemy distinction, namely that of ‘New Orientalism’: a relational construction of ‘transnational bourgeois identity’, counterposed to an internal ‘Oriental Other’. In this struggle, Ukrainian nationhood has been defined politically and economically as essentially ‘European’ and liberal-democratic; whereas, the ‘Other’, the Donbass separatists in the Eastern-most regions, have been defined in negative terms, as distinctly Russian, Soviet, Asiatic, backward, corrupt, and, ultimately, Oriental. The focus of this paper is one-sided because my motivation is to deconstruct what has gained global currency as a popular example of grassroots democracy by problematizing the ‘democracy’ and the far right’s seemingly comfortable participation in its development. I am interested in discursive, ideological, but also starkly material use of Euromaidan and so attempt to highlight the ultimate contradictions that it is premised upon. Thus, this paper covers a lot of ground – from theories of democratization and transitions from authoritarianism, to critiques of liberalism in political theory, to the knowledge production analysis of the current nationalist discourse, and in connection, a political economy analysis of separatism in Donbass. This paper’s objective does not imply the embrace of the official Russian state politics. Such an adoption of the friend-enemy terms, albeit from the other side, would be counterintuitive to the main theme of this paper. In fact, I hold that “in trying to reify ethnic boundaries by imputing negative political meanings to ethnic and linguistic identities, the Ukrainian far right – ironically – shares the same goal as the Russian government” (Giuliano 2015: 520). For a more complete picture of the history and the causes of the Ukraine Crisis, 5 In this essay, I use ‘liberal’ and ‘neoliberal’ interchangeably. I justify this use historically; I take it to be that ‘liberal democracy’ as a system was first introduced in Ukraine after the fall of the Soviet Union in its already ‘neoliberal’ form through various ‘shock therapy’ economic, social, political policies. This, some could point out, limits my analysis because the differentiation between liberalism and neoliberalism might lead to different results in terms of their construction of an ‘Other’ and possible partnership with the radical right. 6 Although possible, to fully account for the ‘cause’ of the civil war would mean to go over a long history of Ukrainian nation-building, which is not the purpose of this paper. I merely propose to understand the discourse of friend-enemy that has emerged and is becoming mainstream in Ukraine today. 47 The Ukrainian Crisis: A Case of ‘New Orientalism’ a similar analysis ought to be undertaken with regards to the ‘Russian’ side and its specific constructions of Russian nationalism. The Ukraine Crisis and Its Discourse On November 21, 2013, then Ukrainian President Victor Yanukovych refused to sign a free trade agreement with the EU that would signal a step towards Ukraine’s eventual EU membership, and instead agreed to a $15 billion partnership offer from Russia that signified closer cooperation with Ukraine’s neighbour and potential membership in Russia’s Eurasian Customs Union. This act came to be interpreted as anti-democratic and corrupt by the majority of Western media sources and Ukrainian citizens, who saw Yanukovych as advancing a Russian political and economic agenda over the national interests of Ukraine (The Economist 2013a: 59-60; Greene 2014; Balmforth 2013). It is instructive to look at how Yanukovych’s decision, and the Ukraine Crisis more broadly, has been framed thus far by the mainstream media and scholarship in very pro-European terms that naturalize Western liberal-democratic values. Particularly instructive is (a) the definition and use of ‘democracy’ and (b) the way in which the involvement of the far right among pro-Western forces has been defined as democratic, downplayed, and/or suppressed. By rejecting the trade agreement with the EU, the editors of The Economist wrote, Yanukovych “appeared to hand victory to Vladimir Putin in a struggle with the EU over Ukraine’s geopolitical orientation” (The Economist 2013b: 13). Corruption was defined as ‘anti-Western’, and Ukrainian national interests as the interests of the EU. On November 30, 2013, The Economist wrote that the protests were about a desire for Western values and governance: “[…] standing in temperatures of minus 13C, ready to be beaten up, the people on Maidan were defending something far greater than an association agreement with the EU, which was the initial cause. They were standing in the way of a police state, defending fundamental European values and defying the post-Soviet order imposed by Russia.” (2013a: 59) Ukraine’s political path was framed as a choice between two opposites – towards future progress with the EU or back to the corrupt Soviet past with Russia. The outburst of Ukrainian nationalism was represented as the desire to live in Europe under liberal-democratic values. The general response in academia, albeit limited due to the recent nature of the events, has been similar and can be divided into two commonplace views. In the first, scholars downplayed or obscured the role of the radical right at Euromaidan. For example, Olga Onuch approached the Euromaidan phenomenon along the lines of modernization theory. She asked: how can we account for such a popular uprising within the context of struggling democracies everywhere that suffer from low voter turnout and general popular apathy (2014:44)? For Onuch, the popular appeal of Western-style democracy serves as evidence that the liberal subject has developed in Ukraine, despite the Ukrainian state lagging behind. Therefore, the average protester was a liberal subject who revolted against the current state-form in order to achieve the predominance of liberal-democratic European 2017 | Vol. 9 | No. 1 48 values and economic prosperity (Onuch 2014: 48). Meanwhile, the radical-right protester was portrayed as an irrelevant minority (Onuch 2014:46-47). Similarly, Mykola Riabchuk states that what mobilized the protesters was: “their hope for a ‘normal life in a normal country’ which the agreement had envisaged and come to symbolize. Now, as the government had stolen that hope, [people] feel deceived – it’s not just about this single incident, but about their whole lives, the whole development of the country stuck for 22 years in a grey zone between post-Soviet autocracies to the East and increasingly democratizing and prosperous neighbours to the West.” (2013) Here, the West-East dichotomy mirrors the difference between normal and abnormal government and society. Where does the radical right fit into this story of good versus bad? In Ukraine Crisis: What It Means for the West, Andrew Wilson (2014) gave Maidan a ‘truly democratic’ progressive legitimacy by placing it alongside the Occupy movement and the Arab Spring (Ishchenko 2014: 154). While Wilson could not outright deny the presence of radical

Page 59: openev.debatecoaches.org€¦  · Web view1AC . Observation 1: If I could find the spot where truth echoesI would stand there and whisper memories of my children's future. I would

nationalism, he represented its forces at the Maidan as marginal, the argument being that since these far-right parties did not win in the subsequent parliamentary elections, they do not represent a credible threat or significant political faction (Wilson 2014: 86-88, 171-172). In fact, he wrote that “a torch-lit march through Kiev in honour of the World War Two Ukrainian Nationalist hero Stepan Bandera on 1 January was such a stupid idea, it had to be a provocation” (Wilson 2014:86). A Kyiv-based sociologist, Volodymyr Ishchenko, has come to criticize Wilson’s (and others’) view as near-sighted, pointing to the determining role of political groups such as the Organization of Ukrainian Nationalists (OUN), Svoboda Party7 , and Praviy Sektor (the Right Sector) in the movement (Ishchenko 2014: 13). The second general response of academia to the Ukraine Crisis accepts in theory and declares in practice the benefits of the partnership between the radical-right and liberal forces in Ukraine but does so uncritically. Jennifer J. Carroll argues that the beauty of Maidan lies precisely in its unifying nature: the mobilization of elements across class divisions and ideological affiliations – both on the left and the right – through the idea of a nation striving for a European, liberal-democratic future (2014: 9-10, 12). She argues that academics must put semantics aside when analysing the radical-right forces. Instead of analysing the content of radical-right banners (Svoboda and Right Sector), they must remark on the fact that their members were fighting for the same cause as the rest of Ukrainians, i.e. “wanting things to be Ukrainian” (Carroll 2014: 12). While obviously condemning radical-right ideals in themselves, Carroll does not shy away from suggesting that they were useful for the Euromaidan cause in general by radicalizing popular grievances through the idea of the nation. In other words, we can forgive their slogans so long as they support a European Ukraine against Eastern separatism and Russian encroachment. Carroll notes that: “many dedicated members of these so-called ‘radical groups took up arms in cooperation with so-called ‘ordinary’ Ukrainians against a common enemy 7 The literal translation of ‘svoboda’ (‘свобода’) is ‘freedom’. 49 The Ukrainian Crisis: A Case of ‘New Orientalism’ that threatened the very dignity and livelihood of the Ukrainian nation […] numerous protestors told me that radical groups ‘[did] the necessary work of radicalizing ordinary Ukrainians against their oppressors’ and ‘[made] the nation visible, so that people know what they are fighting for’” (2014: 12). In a sense, Carroll tries to sanitize the radical right by fitting it into the bourgeois revolution interpretation of Ukrainian Euromaidan. Thus, her analysis is not too far off from that of Onuch, Riabchuk, and Wilson. However, what Carroll makes explicit is that in the case of Ukraine the project of the radical right and the project of European liberal-democratic integration are not contradictory. Democratization Before turning to the analysis of the far right, I want to address the simple view of ‘Euromaidan’ as a project of completion of the ‘bourgeois revolution’, or a full transition to liberal democracy that began with the fall of the Soviet Union through the 2004 Orange Revolution. It is necessary to pose the question, what is the relationship between the transition to capitalism and the development of democracy? The debates within democratization and democratic transitions scholarship that are often informed by modernization theory have focused on: (a) the ways in which transitions occur and democracies are consolidated, but also (b) around the similarities and differences between the various ‘waves’ of democratization – in Southern Europe, Latin America, and Eastern Europe. Indeed, Euromaidan itself has been presented as another ‘wave’ of democratic transition in Ukraine. Philippe Schmitter and Terry Lynn Karl (1994) maintain that in the post-communist states democratization has, in part, been a strategic politico-economic project with an international dimension. They write: “the regime changes in eastern Europe triggered a major collapse in intraregional trade and the dissolution of the Warsaw Pact. Into this vacuum moved an extraordinary variety of western advisors and promoters binational and multilateral. To a far greater extent than elsewhere, these external actors have imposed political ‘conditionality’ upon the process of consolidation, linking specific rewards explicitly to the meeting of specific norms or even to the selection of specific institutions’.” (Schmitter, Karl 1994: 182) Echoing Schmitter and Karl, Valerie Bunce (1995) notes that in Eastern Europe democratic transition implied a number of inter-related processes of restructuring, both economic and political: the role of the state with regards to its citizens as well as the international system, liberalization of economy and foundations of a capitalist society, re-definition of citizenship via creation of a new capitalist class system (1995: 120-121). Thus, democratization in the post-communist era was not just a political regime change but one based on certain economic preconditions. This view is remnant of the modernization theory model, which suggests a causal relationship between capitalism and democracy. 2017 | Vol. 9 | No. 1 50 Guillermo O’Donnell and Philippe Schmitter (1986), Gerardo Munck (2011) as well as Juan Linz and Alfred Stepan (1996) assume a normative value of minimalist democracy. They differentiate between liberalization and democratization in a manner where economic liberalization can occur without democratization, whereas democratization usually follows, entails, and depends on economic liberalization, but on a wider, more political scale. Their conception of democracy is not very different from Robert Dahl’s “polyarchy”, which refers to an electoral, representative democratic regime only, following the development of a competitive market-oriented economy, not a “true democracy”, which perhaps is more of an ideal than an achievable reality (Dahl 1989:251). This analysis shows that ‘democracy’, and its ‘freedom’ and ‘equality’ is a relative concept. Because the protests at the Maidan have been interpreted in terms of Ukrainian nation’s fight for freedom from totalitarianism, for equality, and for liberal democratic, European values, it is the task of political scientists to demystify those very ideas. A ‘transition’ to a democracy defined as polyarchy might result in establishment of valuable political institutions, but it might not answer all the political and socio-economic demands of the people who protested at the Maidan and in Eastern Ukraine. Far Right The role of the far right was central to the victory of the Maidan and since then has become infiltrated into the new Ukrainian government in Kyiv and its policy-making. The far right was most active in the setting up of self-defence units, as well as attacking and occupying government and administrative buildings, police headquarters and getting access to armament. As Ishchenko clarifies, Maidan participants were not middle-class proper (which, if our criterion is based on consumption patterns, makes up about 10-15 % of the Ukrainian population) but individuals with nothing to lose, largely, “a movement of dispossessed workers” (2014: 11, 19). Peculiarly, their grievances were expressed not in terms of social and economic demands but acute nationalism; aligning with ultra-nationalist and radicalright movements. In this sense, as Ishchenko points out, Svoboda became what we may call a popular, democratic, grassroots party of the Maidan (Ishchenko 2014: 12). Svoboda, originally founded after the fall of the Soviet Union, was officially entitled the Social-National Party of Ukraine (Ishchenko 2014: 14). After the three-month struggle at the Maidan, Svoboda emerged not only as one of the official opposition parties with direct influence over policy-making but also as a party whose members serve as ministers in government. Currently, “Svoboda holds a larger chunk of its nation’s ministries (nearly a quarter, including the prized defense portfolio) than any other far-right party on the [European] continent. Ukraine’s deputy prime minister represents Svoboda (the smaller, even more extreme ‘Right Sector’ coalition fills the deputy National Security Council chair), as does the prosecutor general and the deputy chair of parliament.” (Foxall and Kesslermarch 2014) 51 The Ukrainian Crisis: A Case of ‘New Orientalism’ To be more explicit, Andriy Parubiy was appointed the new secretary of Ukraine’s security council; also a co-founder of Svoboda, his deputy, Dmytro Yarosh, is the leader of the Right Sector, and the highest-ranking right-wing extremist is Deputy Prime Minister Oleksandr Sych, who is also a member of Svoboda (Hughes 2014). Moreover, Western recognition of Svoboda as a legitimate political player has been common in the aftermath of the Euromaidan: “In December, shortly after protests began against Ukraine’s pro-Russian president Viktor Yanukovych, U.S. Senator John McCain shared a platform and an embrace with Svoboda chief Tyahnybok at a mass rally in Kiev, assuring demonstrators, ‘The free world is with you; America is with you.’ In February of this year, France and Germany oversaw a peace deal between Tyahnybok, two other opposition leaders, and Yanukovych (though soon after, protests forced Yanukovych to flee to Russia). And in early March, the U.S. State Department published a debunking of Putin’s ‘False Claims About Ukraine,’ assuring Americans that Ukraine’s farright ‘are not represented’ in parliament.” (Foxall, Kesslermarch 2014) In terms of the concrete influence of the far right over government policy, the magazine Foreign Policy reports that “one of [Svoboda’s] chief demands – that all government business be done in Ukrainian – was passed into law, instantaneously marginalizing the one-third of Ukraine’s citizens (and 60 % of Crimeans) who speak Russian. Then for good measure, the party launched a push to repeal a law against ‘excusing the crimes of fascism’” (Foxall and Kesslermarch 2014). Regardless, the radical right’s greatest success is not its participation in the government but rather that its ideological discourse has come to dominate the conversation around Ukraine’s present and future. This discursive triumph has influenced a general right-wing shift in Ukrainian politics as well as in mainstream and academic thinking, to which Carroll’s piece itself serves as a prime example. For instance, the ideological symbols and slogans such as “Glory to the Nation! Death to the Enemies!” and “Glory to Ukraine! To heroes, glory!” which before the outbreak were traditionally expressed exclusively in ultra-nationalist circles, were transformed into common, quite mainstream parlance (Ishchenko 2014:15; Luhn 2014). On the Memorial Day of the Victims of Repression, the leader of Svoboda, Tyahnybok, proclaimed (in a rough English translation): “For us, the nationalists, it is not enough that Parliament because of certain political situation, of necessity, finally adopted anti-communist laws. By the way, from the first days of its foundation ‘Svoboda’ constantly stressed the need to adopt similar legislation. But now it

Page 60: openev.debatecoaches.org€¦  · Web view1AC . Observation 1: If I could find the spot where truth echoesI would stand there and whisper memories of my children's future. I would

is absolutely essential these laws take an effect in Ukrainian state. It is also important that psychology of all Ukrainians was turned to Ukrainian manner and every Ukrainian looked at the world in the light of Ukrainian’s view but not through bolshevik-communist-Kremlin glasses. Because, unfortunately, nowadays anti-Ukrainian and Moscow’s influence is largely maintained in 2017 | Vol. 9 | No. 1 52 Ukraine. It is important to remember our true heroes, honour their memory, especially in such places like this Memorial complex.” (Svoboda 2015) At the same time, on its official website, Svoboda is also described as: “a modern political party, which is open to cooperation. In our international activities we are looking for those supporting traditional European values, human rights, the rights of nations, respect their sovereignty and oppose imperialism; those partners, whose vision concerning Ukraine, European and global challenges has something in common with our own point of view.” (Svoboda 2015) There appears to be a contradiction of tone between the first and the second statement. With regard to the second statement, we sense the apprehension about the risk of being perceived as a far-right party, and hence, the desire to establish itself as liberal and European and truly legitimate. Whereas, in the first statement, which reveals an organic, naturalistic view of a nation, Tyahnybok places the idea of the ‘nation’ above liberal-democratic values, such as individual freedom of choice and association. On the one hand, the official economic rhetoric of the opposition parties, including Svoboda, has been one of striving for a bourgeois identity, for a ‘European’, globalized Ukraine with open markets, and adoption of the EU’s current neoliberal austerity policies. The new neoliberal government “accepted all the credit conditions imposed by the IMF – increasing public utility tariffs, freezing wages, cutting a whole range of benefits […] that would put the burden of the economic crisis on the poor” (Ishchenko 2014: 22). On the other hand, the underlying ideology of the economic regime has become the radical-right nationalism: freedom for the Ukrainian nation and the right of self-determination of the ‘Ukrainian’ people, based on their ethnic, cultural, linguistic, and historical distinction from the ‘Russian’ nation. The Solidarity Bloc of Petro Poroshenko, the majority party in government, relies for its legitimacy on the Maidan revolution as a victory against both internal and external aggression against Ukraine, against what it calls the “enslavement” of the Ukrainian people (The Solidarity Bloc 2014). Here, enslavement is invoked selectively and excised from socio-economic conditions. What are we to make of this contradictory marriage between liberalism and nationalism? Between, on the one hand liberal universalism and, on the other, Ukrainian national exclusivity? One explanation for this coalition has to do with tactics: the (neo)liberal economic and political Ukrainian elite needed to align themselves with ultra nationalism. The radical-right forces were critical in representing popular sentiments. As Ishchenko points out, “part of the reason why the intelligentsia didn’t take a distance from the far-right may have been that they knew they were objectively weak, and thought that dissociating themselves from Svoboda and Right Sector would mean being sidelined from the movement altogether; the alliance was too important to them” (2014: 16). Thus, the politicians and the intelligentsia adopted radical nationalism as the only successful way to argue for a European Ukraine. An explanation of the local interests should be situated within the global context. Such an account can highlight that the political and economic agenda of Ukraine is partly enforced by powerful 53 The Ukrainian Crisis: A Case of ‘New Orientalism’ external interests, where Ukrainian elites potentially represent global rather than national interests and use nationalist rhetoric to legitimate a global economic and political agenda.8 In line with this interpretation, others suggest that far-right nationalisms in post-Soviet states often perform a compensatory function for those who are the economic ‘losers’ of the transition to a neoliberal capitalist order. They argue, “it is the ‘transition losers’ […] who experience a sense of insecurity, are frustrated with the democratic experience, and seek refuge in nationalist values” (Häusermann and Kriesi 2011). This analysis suggests that in Ukraine, neoliberalism – in its global and local form – relies on the far right’s nationalist discourse for legitimation in lieu of substantial economic security. I return to the political economic analysis after a reading of Carl Schmitt’s critique of liberalism that helps to conceptualize events in Ukraine. Schmitt: The Political In The Concept of the Political, Schmitt characterizes liberalism as a movement for universal pacifism, which negates the political-fundamental status of man – the friend-enemy distinction and a natural inclination to war. There are two dimensions to his critique of liberalism. First, Schmitt shows that the liberal project aims to universalize the human condition, deny and neutralize the political. Liberal democracy, Schmitt writes, appears as “a completely pacified globe, […] without the distinction of friend and enemy and hence a world without politics” (Schmitt 1976: 35). The liberal-bourgeois individual: “rests in the possession of his private property, and under the justification of his possessive individualism he acts as an individual against the totality. He is a man who finds his compensation for his political nullity in the fruits of freedom and enrichment and above all in the total security of its use. Consequently, he wants to be spared bravery and exempted from the danger of a violent death.” (Schmitt 1976: 62-63) Secondly, and more importantly, Schmitt unmasks power, or the political enmity from under liberal pacifism. It serves to reproduce his thoughts verbatim: “the concept of humanity is an especially useful ideological instrument of imperialist expansion, and in its ethical-humanitarian form it is a specific vehicle of economic imperialism […] To confiscate the word humanity, to invoke and monopolize such a term probably has certain incalculable effects, such as denying the enemy the quality of being human and declaring him to be an outlaw of humanity; and a war can thereby be driven to the most extreme inhumanity.” (1976: 54) 8 For a critical account of the history of global capitalism, and specifically the discussion of ‘empire by invitation’, please see Panitch and Gindin (2012). 2017 | Vol. 9 | No. 1 54 Thus, depoliticisation defined as universal humanism is an ideology that covers the true intentions of those who use it for their own (economic or national) advantage. By presenting their values as universally valid human values, the proponents of liberalism first dehumanize their opponents only to then legitimately destroy them as enemies of humanity. For example, in calling the League of Nations an imperialist “alliance” of states, Schmitt argues that international political bodies do not eliminate war. Rather, they legitimize and sanction them against those enemies that could not be incorporated or suppressed (1976: 55-56). In the next section, I attempt to show how the current liberal project in Ukraine (for now) supports Schmitt’s diagnosis. Bringing together a discursive and a political economic analysis, I suggest that the declaration of the ‘enemy’ in the Ukraine Crisis has taken the form of a ‘new Orientalism’. ‘New

Orientalism’ Discourse and Its Critique The term ‘Orientalism’ was first coined by Edward Said to describe a process by which Western societies constructed ‘Eastern’ colonized cultures as backward and primitive, and in doing so named themselves developed, modern, and rational. Anthropologist Michal Buchowski and feminist scholar Sedef Arat-Koc augment Said’s analysis to understand a new phenomenon in Eastern Europe where ‘Orientalist’ thinking is applied internally, to the post-Soviet ‘losers’ of globalization. By using their framework of ‘New Orientalism’, I argue that in Ukraine, as part of the process of homogenization with Europe, we see a construction of a new identity. The ‘Other’ in this context is what Buchowski terms the ‘homo-Sovieticus’, whose existence occurs within national borders but by definition is seen as anachronistic. Whereas previously all of Eastern Europe in Western eyes appeared “as ‘neither fish, nor fowl’, semi-oriental, not fully European, semi-developed, and semicivilized,” in the process of Western incorporation of Eastern Europe after 1989, there has been “a restructuring of the perception of social inequalities by the hegemonic liberal ideology” (Buchowski 2006: 464). Buchowski explains, “the degree to which various countries, authorities, social groups and individuals have embraced the free market and democracy – evaluated by those powerful who set rules of the game –

Page 61: openev.debatecoaches.org€¦  · Web view1AC . Observation 1: If I could find the spot where truth echoesI would stand there and whisper memories of my children's future. I would

has become a yardstick for classifying different regions, countries and groups as fitting more or less into the category of ‘us’ i.e. ‘(post)modern-Western-liberals’.” (2006: 464-465) No longer geographically defined, those who have not embraced ‘Europe’ for ideological or socio-economic reasons are deemed irretrievably backward and, virtually, ethnically different. Arguments from essentialised cultural standpoints dismiss and delegitimize any disagreement about political and economic development. Arat-Koc highlights how ‘culture’ is used to hide political economic issues: “as neoliberal hegemonies exclude theories that demonstrate failure as central and integral to the functioning of capitalism and inevitable, as social, economic, and historical explanations for failure are excluded from 55 The Ukrainian Crisis: A Case of ‘New Orientalism’ hegemonic discourses, ‘culture’ (as a reductionist, essentialized, shrunk, caricatured version of what the term could otherwise mean) becomes the ‘only’ accepted ‘explanation’ in mainstream discourse.” (2014: 329) Echoing Schmitt’s critique of liberalism, this reveals the very undemocratic nature of the ‘liberal-democratic’

project in general and in Ukraine, in particular. In the case of Ukraine, the backward ‘Other’ is ideologically associated with Russian politics, culture, and language, both inside and outside the nation-state. Due to the history of Western and Eastern imperialism, the ideological but nonetheless real and material border between West and East in Ukraine demarcates people by language, culture, ethnic affiliation and even political ideology, i.e. liberalism in the progressive West and socialism in the backward East. As evident from Tyahnybok’s statement above, communism and the working-class movements have been associated with Russian political culture and history, and therefore viewed as a Russian imperial imposition both from within and from without. The oversimplified identification of present-day Russia with the Soviet Union, as well as with communism itself, is ahistorical. First, it obscures the history of Ukraine, as nineteenth-century Ukrainian nationalism was openly leftist, not to mention that it maintained a strong anarchist tradition that was born in central Ukraine under Nestor Makhno (Ischchenko 2014: 16-17). Secondly, that Russian nationality and language, and socialism have become synonymous is wildly inaccurate, since Russia is no longer (and presumably never has been) a socialist state: its economy is now quite obviously structured by capitalist social relations, even if at variance (i.e. the prominent role of ‘oligarchs’ and patrimonialism) with Western models of capitalism. Studies of the local population’s agency in self-determination movements in the Donbass region have been scarce. In order to dismiss Eastern-Ukrainian concerns as illegitimate, the use of ‘New Orientalist’ discourse by politicians, academics, and reporters helps present them as irrational and dehumanized compared to the civilized ‘ethnic’ Western-Ukrainians advocating for proper Western institutions and values. If we look at yet another one of Tiahnybok’s statements, we find another demarcation along friend-enemy lines of ‘civilized’ versus ‘uncivilized’: “In Donbas, gangs of armed terrorists are shouting separatist slogans and – with the support of the Kremlin – carry out physical destruction of the Ukrainian nation. They are, de facto, beginning an ethnic cleansing – people get killed for saying ‘Glory to Ukraine!’, for speaking Ukrainian language and/or wearing ‘vyshyvankas.’9 People are kidnapped just because they are Ukrainians. Three of Svoboda’s representatives have also been abducted. This mess must stop – the state has a duty to its citizens, thus should eliminate terrorism and guarantee security of the people.” (Svoboda 2015) 9 ‘Vyshyvanka’ is a colloquial term for an embroidered peasant blouse in Ukrainian traditional costume. In the aftermath of the Euromaidan, it has been used as a symbol of Ukraine’s freedom, worn in the mainstream, and popularized by the fashion industry in Ukraine and in the West. 2017 | Vol. 9 | No. 1 56 Evidently, the centre of the focus is not the protesters’ practical demands but their ‘character’ and nationality. In fact, there has been a “spread of dehumanizing rhetoric against the movement in Eastern Ukraine […] after the Odessa massacre on 2 May, when thirty people were burned to death in the Trade Union building, [and] some Ukrainian nationalists were exultant” (Ishchenko 2014: 32). The materiality of the shift of the official discourse to the right is striking. While ethnic, cultural, and language differences have been employed to explain the politics of separatism in the East, the three are not so easily discernible into separate and sovereign ‘Ukrainian’ and ‘Russian’ identities. Elise Giuliano explains, “Cultural boundaries between Russians and Ukrainians are fuzzy and faint, as indicated by the high rate of inter-ethnic marriage there. Russians and Ukrainians share, for the most part, a common religion – Eastern Orthodoxy, or a secularism inherited from the Soviet era; a common language (Russian), very similar languages (Ukrainian and Russian), or a mixture of the two (Surzhyk); and a host of social practices and cultural expectations based on their shared experience as Soviet and post-Soviet citizens [...] In general, the porosity of cultural boundaries suggests that ethnic identity by itself provides little information about why many people in the east feel alienated from the Ukrainian state.” (2015: 516-517) Much evidence indicates that cultural and linguistic essentialism have been constructed by and used in favour of a certain political-economic project. However, the association of language and support for separatism has not been a static one throughout Ukraine’s history. For instance, in the Donbass region those citizens who have reported that their national language is Ukrainian still voted for Yanukovych in the 2012 presidential elections (Ibid.). Even after language became politicized in the aftermath of the Maidan by the political elites (via Ukraine’s interim government’s annulment of language law making Russian an official language of the Ukrainian state that was passed under Yanukovych in 2012), residents of the Donbass region did not see it as the main reason behind separation. Giuliano provides statistical evidence: “In an International Republican Institute (IRI) poll, 74% of respondents in east Ukraine (Donestsk; Dnepropetrovsk; Kharkhiv; Luhansk) answered either ‘definitely no’ or ‘not really’ when asked: ‘Do you feel that Russian-speaking citizens of Ukraine are under pressure or threat because of their language?’ A majority of ethnic Russians also answered ‘definitely no’ (49%) and not really (17%).” (2015: 518) Instead, in agreement with both Giuliano and Ishchenko, I propose that it is more useful to understand the alienation experienced by the population in the Donbass region from the new Ukrainian government and Euromaidan in political-economic terms. The Donbass region is the most industrialized and urbanized area of Ukraine. The older Soviet enterprises 57 The Ukrainian Crisis: A Case of ‘New Orientalism’ in the area – mostly coal industry and metallurgy – were a central production hub in the USSR and after its fall; it produced for Russia as its biggest trading partner (Ishchenko 2014). The local economy and social relations, due to a close geographic proximity, are tied to Russia’s. Thus: “in the estimations of ordinary people, joining the Customs Union would maintain trade ties with Russia and other post-Soviet states and therefore preserve jobs and the status quo. A shift in economic orientation toward Europe would bring uncertainty to the region. Such a change, moreover, would affect large numbers of workers since Donetsk and Luhansk are overwhelmingly urban (90% and 10%).” (Giuliano, 2015: 518-519) The region opposed Ukraine’s EU association out of a practical prediction of losing jobs through privatization or shutting down of national enterprises and industries, which are central to employing the urban population and sustaining their lives. The strategic-economic interests of the Donbass region are dismissed as ideological and cultural, as if Eastern Ukrainians must automatically disagree with EU association and values by virtue of being labelled ethnically ‘Russian’ by Western Ukrainians. In connection to the political economic question, Giuliano (2015) emphasizes the political-symbolic alienation of the Donbass residents from the Euromaidan movement and the new Ukrainian government, due to their conception of nationhood in ethnically exclusivist terms. Giuliano explains, “[…] some Ukrainian elites articulated an interpretation of Ukrainian history that, put in stark terms, viewed the past through Stalin’s crimes such as collectivization which produced the Great Famine of 1932 (the Holomodor). In this view, Ukraine is a victim of the

Page 62: openev.debatecoaches.org€¦  · Web view1AC . Observation 1: If I could find the spot where truth echoesI would stand there and whisper memories of my children's future. I would

imperialist Soviet Union which perpetrated genocide and destroyed Ukrainian language and culture. More extreme versions of this view celebrate Stepan Bandera and factions of the Organization of Ukrainian Nationalists – a group that fought the Soviets during World War II in part by making common cause with the Nazis. Advocates of these ideas identify postSoviet Russia and ethnic Russians with the Soviet Union […] Their ideology fit in with a strand of ultranationalist discourse that, over the years, had scapegoated ethnic Russians for the country’s problems and identified them with the Soviet Union and Russia. After Maidan, Ukraine’s interim government did not criticize ultranationalist discourse, but instead appointed a former leader of a neo-fascist party, Andriy Parubiy, to the important post of head of Ukraine’s National Security and Defense Council [...] More dangerously, as the violence heated up, Kiev allowed semi-private paramilitary groups—such as the far right, neo-Nazi Azov Battalion – to fight in east Ukraine.” (2015: 519-520) 2017 | Vol. 9 | No. 1 58 In concluding this section, I want to bring together the discursive and the political economic analyses. The very history of Ukraine, its language, and culture has been reinvented in Western, liberal terms. Aside from the evident materiality of the civil war, which reproduces and reflects certain narratives of national ethnicity as well as global political economic interests, Ukraine is a contemporary example of how knowledge production is a political economic issue. Conclusion Revealing the discourse around the Ukraine Crisis in terms of a ‘New Orientalism’ helps explain the relationship between liberalism and nationalism that became ‘common sense’ at Euromaidan and, currently, in the larger Ukraine Crisis. The modernization school of thought claimed that with the end of the Cold War era of ideology, the spread of capitalist social relations and consequently the transitions to liberal democracy would create political homogenization and render ethnic conflict irrelevant. At a first glance, the current Ukraine Crisis fits this vision as an example of homogenizing political and economic forces at work, transitioning Ukraine into a ‘true’ liberal democratic capitalist society. It was not the purpose of this paper to deny that Euromaidan is a popular, democratic phenomenon. The purpose of this paper was, first, to interrogate what is meant by ‘democracy’ – and whether or not its institutionalization will fulfil the demands of protesters and second, to complicate the romanticized story of a fight for liberal, European values in light of the radical right’s participation in Euromaidan and advance into the new Ukrainian government. In Ukraine, as part of the process of homogenization with Europe, we see a process of the invention of the Ukrainian nation in ethnic, exclusivist terms, which defines a section of Ukraine’s society as a backward ‘ethnicity’. This distinction has concretely manifested in the ongoing violent conflict between the Donbass region and Western and Central Ukraine. Following Schmitt’s critique of liberalism, I argue that the Ukrainian Crisis reveals how liberalism, in this specific historical and geopolitical context, relies on a certain friend-enemy distinction for the advancement of its supposed universalist project. As a way of moving beyond the New Orientalist discourse, I suggest that we understand the political-economic reasons for the separatist politics in Eastern Ukraine, which might point in a useful direction of the re-evaluation of the official Ukrainian as well as the general European political, economic, and social policies.

Impact - The Aff’s orientalist imagery is crucial to perpetuating the violent division between the Orient and the West. Behdad 2010 (“Orientalism Matters,” pg. 711-712, by Ali Behdad on Accessed June 26, 2019. Ali Behdad is a Professor and John Charles Hillis Chair in Literature at UCLA. https://muse.jhu.edu/article/407130)--RSP*

"In contemporary writing about nineteenth-century photography of the Middle East," writes Michelle L. Woodward, "it has become al- most a cliché to describe many ofthese images as 'Orientalist'— that is , reflecting or propagating a system of representation that creates an essentialized difference between the 'Orient' and the 'West'"(363). This claim aptly captures the predominant anti-Saidian sentiment among art historians and curators who work on representations of the Middle East created by both European and indigenous painters and photographers. To be sure, responses to Said's discussion of Oriental- ism as a discourse of colonial power span the critical spectrum , from more rigorous and subtle critiques articulated from the left to the sometimes facile and reactionary from those of an opposing political orientation. On one side are art historians such as Zeynep Celik, Jill Beaulieu and Mary Roberts, and Woodward who argue that "the trend to extend Said's analysis to apply equally to visual representations has . . . been used too broadly, obscuring nuances and inconsistencies, not only between different photographers' bodies of work but also within them" (Woodward 363). These scholars typically aim to con- structively revise Orientalism to encompass "a disparate and disputed set of discursive constructions" while at the same time acknowledging "Orientals" as "participants in the production of counternarratives or resistant images" (Bealieu and Roberts 3). On the other side are writ- ers, such as John MacKenzie and Ken Jacobson, who betray a marked suspicion of theory and seek to return the term "Orientalism" to its prior usage as an art historical term that could be deployed without suggestion of a broader political or ideological critique. In Orientalism: History, Theory, and the Arts, MacKenzie argues against Linda Nochlin that "there is little evidence of a necessary coherence between the imposition of direct imperial rule and the visual arts," claiming that "Orientalism celebrates cultural

Page 63: openev.debatecoaches.org€¦  · Web view1AC . Observation 1: If I could find the spot where truth echoesI would stand there and whisper memories of my children's future. I would

proximity, historical parallelism and religious familiarity [with the Middle East and North Africa] rather than true 'Otherness'" (51). Given such perceived "misconceptions inherent in postcolonialist analysis," Jacobson similarly suggests that "a return to more traditional methods is desirable for the study of 19th- and early 20th-century photography in North Africa and the Near East," urging commentators to focus more single-mindedly on the "notable aesthetic, as well as documentary and historical merit" when analyzing visual representations (88). In what follows, I argue that Orientalism should not be under- stood merely as an ideological discourse of power nor as a neutral art historical term, but rather as a network of aesthetic, economic, and political relationships that cross national and historical boundaries. Understood in this way, Orientalism is indispensible to understanding nineteenth-century photography of the Middle East. Whether consid- ered in the context of their production and dissemination in the nine- teenth century or in relation to their current afterlives as collectable objects or archives, photographs of the Orient become meaningful and legible only if they are considered in terms of the specific geopolitical distinctions, economic interests, and cultural assumptions about the Middle East and its people. While insisting that Orientalism offers a crucial perspective from which to comprehend the meaning and significance of photographic representations of the Middle East, I do not mean to suggest that such images should be understood merely as a reflection of Europeans' racial prejudice against "Orientals," or that these images simply validate European imperial dominance over the region. Nor would I wish to argue that Orientalist photography entails a binary visual structure between the Europeans as active agents and "Orientals" as passive objects of representation. Rather, I hope to provide an alternative view of Orientalistphotography that focuses on nodes and ties that bind artists, collectors, and museums across historical and national boundaries, which are productive of a distinctly exotic vision of the region, a vision at once embraced and perpetuated by the elite in the Middle East .Indigenous photography in and of itself, I maintain, does not constitute an oppositional locus or resistant iconography, for it too belongs to the Orientalist network that mediates its vocabulary and thematics of representation.1 A network theory of Orientalism concerns itself neither with the motivations of individual artists nor with the attributes of art objects, but instead studies the symmetric and asymmetric relations between discrete objects, specific individuals, and concrete practices.

The alternative is to vote negative to construct the ballot as a counter-script against the orientalist discourses of the affirmative. This is key to generating space for subjectivities outside of the construction of the West. Azeez 2k16[ Govand Khalid “Beyond Edward Said: An Outlook on Postcolonialism and Middle Eastern Studies, Social Epistemology” 30:5-6, 710-727, DOI: 10.1080/02691728.2016.1172360 p. 714-716] But Orientalism, as a symbolic revolt, was not just about the critique, deconstruction and exposure of ideological and epistemological Eurocentric frameworks that monopolised the

Page 64: openev.debatecoaches.org€¦  · Web view1AC . Observation 1: If I could find the spot where truth echoesI would stand there and whisper memories of my children's future. I would

realm of discursive production. Appropriating Michel Foucault’s postulation that power is everywhere and that it is diffused and embodied in discourse, knowledge and “regimes of truth”, Said presents the second mode of critique; namely, Orientalism as the ontological nature, structure and a priori condition of power. In this sense, knowledge or the historical materialisation of specific Nietzschean truths were a form of power and power could only be deciphered through critiquing its manufactured meta-narrative and mapping out the transmutation and movement of its anthropomorphisms, metaphors and metonymies. Said demonstrates this through examining Sylvester de Sacy’s role in the Dacier Report (1802). Conducted for examining the state of Orientalist learning, the report was commissioned by Napoleon Bonaparte. Said writes: The importance of the Tableau historique for an understanding of Orientalism’s inaugural phase is that it exteriorizes the form of Orientalist knowledge and its features, as it also describes the Orientalist’s relationship to his subject matter. In Sacy’s pages on Orientalism … he speaks of his own work as having uncovered, brought to light, rescued a vast amount of obscure matter. Why? In order to place it before the student. For like all his learned contemporaries Sacy considered a learned work a positive addition to an edifice that all scholars erected together. Knowledge was essentially the making visible of material, and the aim of a tableau was the construction of a sort of Benthamite panopticon. Scholarly discipline was therefore a specific technology of power …(1978, 127).3 Elsewhere in his analysis of Arthur James Balfour’s speech in the House of Commons on the 13 June 1910, Said declares: England knows Egypt; Egypt is what England knows; England knows that Egypt cannot have self-government; England confirms that by occupying Egypt; for the Egyptians, Egypt is what England has occupied and now governs; foreign occupation therefore becomes “the very basis” of contemporary Egyptian civilization; Egypt requires, indeed, insists upon British occupation (1978, 34). In other words, Said postulated that Orientalism exuded epistemic violence through its inherent relation with European colonial power. And, in turn, European colonial power could only dominate the region through Orientalism functioning as a technique and instrument of power. Here then Said’s third definition of Orientalism, drawing from the two previous epistemological definitions, demonstrates the intrinsically dialectical relation between knowledge and power. Said posits Orientalism is also: (3) The corporate institution for dealing with the Orient—dealing with it by making statements about it, authorizing views of it, describing it, by teaching it, settling it, ruling over it: in short, Orientalism as a Western style for dominating, restructuring, and having authority over the Orient Collectively Said conceptualises Orientalism as epistemological (both as a corpus of scholarly writings and a historical system of thought) and ontological (as in the nature of being, the structure and condition of power). Read critically, for Said the synthesis of Western power and knowledge had a singular ontological and teleological purpose; namely, the thingification or subjectivation of the colonized Oriental subject. This point is best demonstrated by William D. Hart’s analysis of Orientalism: In his description of discourse, Said appropriates Foucault’s ideas of discipline and power/knowledge. By discipline, Foucault [and by default Said] means those methods of modern punitive power that establishes meticulous control over the body, assuring its constant subjection by imposing a relation of “docility-utility”. Discipline, that is, makes human bodies docile and useful [thingifies], and advertises their availability for political, economic, and cultural uses of many kinds (2000, 70). And what of the latter Said’s “voyage in” and the powerful transformative attempt by the native to “write back” in the metropolis? Can

Page 65: openev.debatecoaches.org€¦  · Web view1AC . Observation 1: If I could find the spot where truth echoesI would stand there and whisper memories of my children's future. I would

this “contrapuntal reading” (Said 1994a, 288, 79), allow a discursive space where the native-intellectual, halfinvolved and half-detached (Said 1996), rejects the “hail” and the Althusserian “quadruple system” of interpellation (Althusser 1969)? Does the deliberate attempt in Said’s words to “enter into the discourse of Europe and the West, to mix with it, transform it, to make it acknowledge marginalized or suppressed or forgotten history” (Said 1994, 260) effect this equation? This latter Said, moving away from Foucauldian discursive formation and utilizing Gramsci’s hegemony or direzione culturale [cultural direction], envisions social space alike a Bourdieuean field. Revalorizing culture, political space becomes an arena where subjects engage in competent action and deploy different modes of capital and strategies in order to advance their position within the framework of a shared system of meaning (Bourdieu 1977). In this sense, Said announces in Culture and Imperialism (1994a) that “… no matter how apparently complete the dominance of an ideology or social system, there are always going to be parts of the social experience that it does not cover and control. From these parts very frequently comes opposition both self-conscious and dialectical” (Said 1994, 289). Accordingly, Said posits that the “new receptivity to both liberation movements and post-colonial criticism” not only challenges the “monopoly of discourse by Eurocentric intellectuals and politicians” (Said 1994, 316), but also creates a new form of subjectivity residing at a “median state” whereby power’s Manichean dichotomies are obscured.

Page 66: openev.debatecoaches.org€¦  · Web view1AC . Observation 1: If I could find the spot where truth echoesI would stand there and whisper memories of my children's future. I would

Solft Case:

Trump would cooperate with Putin after the plan to improve relations—arms sales are the key stumbling blockHEER 2017 (Jeet, “The Dangerous Incoherence of Trump’s Russia Policy,” The New Republic, Dec 21, https://newrepublic.com/article/146397/dangerous-incoherence-trumps-russia-policy)

In 2014, the Republican Congress passed the loftily named Ukrainian Freedom Support Act, which permitted the sale of lethal weapons to Ukraine. President Barack Obama signed the bill into law, but as The Washington Post reported, he “never authorized large commercial or government sales, a move widely seen as a de facto decision not to provide lethal weapons to the Ukraine military.” That policy has now changed, as President Donald

Trump has approved a large commercial sale of weapons to the former Soviet state, where the fight against Russia-backed militants in the east has intensified

lately. “The move was heavily supported by top Trump national security Cabinet officials and Congress,” the Post reported, “but may complicate President Trump’s stated ambition to work with Russian President Vladimir Putin.” A senior congressional official told the paper, “We have crossed the Rubicon, this is lethal weapons and I predict more will be coming.”

Some conservatives are arguing that the arms sale proves that Trump didn’t collude with the Russian government to interfere in last year’s presidential election. “NO PUPPET” ran an exultant Twitchy headline.

In reality, the decision on arming Ukraine illustrates something different: The United States has two wildly divergent foreign policies toward Russia. Trump is pursuing a policy of conciliation , while the national security establishment, including

Secretary of Defense James Mattis and Secretary of State Rex Tillerson, have pursued a policy of aggressive containment. Trump sees Russia as a potential ally, while many in his cabinet and in Congress consider Russia a rival that is actively threatening American democracy. This contradictory policy could send mixed signals, leading to war.

Small improvements in relations snowball—the US has to begin by respecting Russian interestsCENTER FOR PREVENTIVE ACTION 2017 (“Managing Global Disorder: Prospects for U.S.-Russian Cooperation,” August 23, https://www.cfr.org/report/managing-global-disorder-prospects-us-russian-cooperation)

Participants largely agreed that taking incremental steps toward cooperation would be the best approach to improving U.S.-Russian relations, with the hope that these steps would build momentum toward further improvement. While legally binding agreements would be difficult to achieve in the current political climate, and legally binding multilateral agreements next to impossible, the United States and Russia could pursue more informal arrangements to work together.

Future cooperation in whatever form is unlikely, however, without a mutual recognition of interests. One participant argued that the United States needed to acknowledge Russia’s legitimate interests, claiming that, currently, “the fact that Russia has legitimate interests is little accepted in the United States.” Other participants pointed out that, while Russia has made some of its red lines clear (such as NATO or EU membership for certain states on its border), it has not officially communicated what its legitimate interests are.

Page 67: openev.debatecoaches.org€¦  · Web view1AC . Observation 1: If I could find the spot where truth echoesI would stand there and whisper memories of my children's future. I would

Improved US-Russian relations right now cause Trump reelection and war with IranSIGOV 2019 (Mike Sigov, a former Russian journalist in Moscow, is a U.S. citizen and a staff writer for The Blade, “On Russia: Keeping a wary eye on improving U.S.-Russian relations,” Toledo Blade, May 19, https://www.toledoblade.com/opinion/columnists/2019/05/19/Keeping-a-wary-eye-on-U-S-Russian-relations/stories/20190518003)

The most immediate and undesirable outcome of such rapprochement is two-fold.

First — regardless of Mr. Pompeo’s over-reported nominal warning to the Russians not to interfere in the upcoming U.S. presidential elections issued in Sochi — the Trump campaign is sure to use any normalization in relations with Russia to undermine U.S. efforts to prevent Russia from influencing the elections in Mr. Trump’s favor the way Russia did in 2016.

Second, Mr. Trump’s threat of war with Iran may well materialize once Mr. Putin signs off on it as a concession to Mr. Trump for anticipated favors, such as relieving the sanctions .

While helping Mr. Trump to rally his base, such a war would be disastrous — it would exact a human, financial, and moral toll on us by far in excess of what the unwarranted second war in Iraq did.

The war also would hurt the cause of nuclear nonproliferation — which Messrs. Pompeo and Lavrov supposedly care so much about — by demonstrating that there is no other effective way to guarantee non-aggression by another state than access to nuclear weapons.

Two directly opposite U.S. policies — one toward nuclear-armed South Korea and the other toward Iran that has no nuclear weapons — attest to that.

As for Mr. Putin, a U.S. military campaign in Iran would be a godsend.

It’s guaranteed to hurt Iran’s oil production, thus affecting the world oil supply and boosting oil prices. That would significantly benefit both the Russian economy and Mr. Putin’s personal coffers, which rely heavily on oil exports.

The bottom line is the American interest mandates that we steer clear of rapprochement with Russia at least until after the 2020 presidential elections.

Trump reelection turns warming, prolif, US-Russian conflict, and nuke warSTARR 2019 (PAUL STARR is a professor of sociology and public affairs at Princeton and a winner of the Pulitzer Prize for General Nonfiction, “Trump’s Second Term,” The Atlantic, May, https://www.theatlantic.com/magazine/archive/2019/05/trump-2020-second-term/585994/)

This is one of those moments. After four years as president, Trump will have made at least two Supreme Court appointments, signed into law tax cuts, and rolled back federal regulation of the environment and the economy. Whatever you think of these actions, many of them can probably be offset or entirely undone in the future. The effects of a full eight years of Trump will be much more difficult, if not impossible, to undo.Three areas—climate change, the risk of a renewed global arms race, and control of the Supreme Court—illustrate the historic significance of the 2020 election. The first two problems will become much harder to address as time goes on. The third one stands to remake our constitutional democracy and undermine the capacity for future change.

In short, the biggest difference between electing Trump in 2016 and reelecting Trump in 2020 would be irreversibility. Climate policy is now the most obvious example . For a long time, even many of the people who acknowledged the reality of climate

change thought of it as a slow process that did not demand immediate action. But today, amid extreme weather events and worsening scientific forecasts, the costs of our delay are clearly mounting, as are the associated dangers. To have a chance at keeping global warming below 1.5 degrees Celsius—the objective of the

Paris climate agreement—the Intergovernmental Panel on Climate Change says that by 2030, CO2 emissions must drop some 45 percent from 2010 levels. Instead of declining, however, they are rising .

Page 68: openev.debatecoaches.org€¦  · Web view1AC . Observation 1: If I could find the spot where truth echoesI would stand there and whisper memories of my children's future. I would

In his first term, Trump has announced plans to cancel existing climate reforms , such as higher fuel- efficiency standards and limits on emissions from new coal-fired power plants, and he has pledged to pull the United States out of the Paris Agreement. His reelection would put off a national commitment to decarbonization until at least the second half of the 2020s, while encouraging other countries to do nothing as well. And change that is delayed becomes more economically and politically difficult. According to the Global Carbon Project, if decarbonization had begun globally in 2000, an emissions reduction of

about 2 percent a year would have been sufficient to stay below 2 degrees Celsius of warming. Now it will need to be approximately 5 percent a year. If we wait another decade, it will be about 9 percent. In the United States, the economic disruption and popular resistance sure to arise from such an abrupt transition may be more than our political system can bear. No one knows, moreover, when the world might hit irreversible tipping points such as the collapse of the West Antarctic Ice Sheet, which would likely doom us to a catastrophic sea-level rise .

The 2020 election will also determine whether the U.S. continues on a course that all but guarantees another kind of runaway global change—a stepped-up arms race, and with it a heightened risk of nuclear accidents and nuclear

war. Trump’s “America first” doctrine, attacks on America’s alliances, and unilateral withdrawal from arms-control treaties have made the world far more dangerous. After pulling the United States out of the Iran nuclear agreement (in so doing, badly damaging America’s reputation as both an ally and a negotiating partner), Trump failed to secure from

North Korea anything approaching the Iran deal’s terms, leaving Kim Jong Un not only unchecked but with increased international standing. Many world leaders are hoping that Trump’s presidency is a blip—that he will lose in

2020, and that his successor will renew America’s commitments to its allies and to the principles of multilateralism and nonproliferation. If he is reelected, however, several countries may opt to pursue nuclear weapons, especially those in regions that have relied on American security guarantees, such as the Middle East and Northeast Asia .

At stake is the global nonproliferation regime that the United States and other countries have maintained over the past several decades to persuade nonnuclear powers to stay that way . That this regime has largely

succeeded is a tribute to a combination of tactics, including U.S. bilateral and alliance-based defense commitments to nonnuclear countries, punishments and incentives, and pledges by the U.S. and Russia—as the world’s leading nuclear powers—to make dramatic cuts to their own arsenals.

In his first term, Trump has begun to undermine the nonproliferation regime and dismantle the remaining arms-control treaties between Washington and Moscow. In October, he announced that the U.S. would withdraw from the Intermediate-Range Nuclear Forces (INF) Treaty signed in 1987 by Ronald Reagan and Mikhail Gorbachev. While the Russian violations of the treaty that Trump cited are inexcusable, he has made no effort to hold Russia to its obligations—to the contrary, by destroying the treaty, he has let Russia off the hook. What’s more, he has displayed no interest in extending New START, which since 2011 has limited the strategic nuclear arsenals of Russia and the United States. If the treaty is allowed to expire, 2021 will mark the first year since 1972 without a legally binding agreement in place to control and reduce the deadliest arsenals ever created.

The prospect of a new nuclear arms race is suddenly very real. With the end of verifiable limits on American and Russian nuclear weapons, both countries will lose the right to inspect each other’s arsenal, and will face greater uncertainty about each other’s capabilities and intentions . Already, rhetoric has taken an

ominous turn: After Trump suspended U.S. participation in the INF Treaty on February 2, Vladimir Putin quickly followed suit and promised a “symmetrical response” to new American weapons. Trump replied a few days later in his State of the Union address, threatening to “outspend and out-innovate all others by far” in weapons development.

The treaties signed by the United States and Russia beginning in the 1980s have resulted in the elimination of nearly 90 percent of their nuclear weapons; the end of the Cold War seemed to confirm that those weapons had limited military utility. Now—as the U.S. and Russia abandon their commitment to arms control, and Trump’s “America first” approach causes countries such as Japan and Saudi Arabia to question the durability of U.S. security guarantees —the stage is being set for more states to go nuclear and for the U.S. and Russia to ramp up weapons development. This breathtaking historical reversal would, like global warming, likely feed on itself, becoming more and more difficult to undo.

Trump circumvents through IsraelAsa Winstanley. 7/4/2018. [Asa Winstanley is an investigative journalist living in London who writes about Palestine and the Middle East.], Israel is arming neo-Nazis in Ukraine, The Electronic Intifada. https://electronicintifada.net/content/israel-arming-neo-nazis-ukraine/24876. EC

Attempts by some in Congress to bar US military aid to Nazis in Ukraine may explain military aid from Israel.

Page 69: openev.debatecoaches.org€¦  · Web view1AC . Observation 1: If I could find the spot where truth echoesI would stand there and whisper memories of my children's future. I would

Israel’s “deepening military -technical cooperation ” with Ukraine and its fascist militias is likely a way to help its partner in the White House , and is another facet of the growing Zionist-White Supremacist alliance.

Israel has historically acted as a useful route through which US presidents and the CIA can circumvent congressional restrictions on aid to various unsavory groups and governments around the world.

Charity Cannibalism DA: The West sustains itself through the cannibalization of the misery of those in the Global South, turning it into a field for the mass production of violence until the point of the west’s total collapseBaudrillard 94 (Jean Baudrillard, greatest living philosopher, 1994, The Illusion of the End, pg. 66-70)

The end of history, being itself a catastrophe, can only be fuelled by catastrophe. Managing the end therefore becomes synonymous with the

management of catastrophe. And, quite specifically, of that catastrophe which is the slow extermination of the rest of the world. We have long denounced the capitalistic, economic exploitation of the poverty of the 'other half of the world'

[l'autre monde]. We must today denounce the moral and sentimental exploitation of that poverty - charity cannibalism being worse than oppressive violence. The extraction and humanitarian reprocessing of a destitution which has become the equivalent of oil deposits and gold mines. The extortion of the spectacle of poverty and, at the same time, of our charitable condescension: a worldwide appreciated surplus of fine sentiments and bad conscience. We

should, in fact, see this not as the extraction of raw materials, but as a waste-reprocessing enterprise. Their destitution and our bad conscience are, in effect, all part of the waste-products of history - the main thing is to recycle them to produce a new energy source. We have here an escalation in the psychological balance of terror. World capitalist oppression is now merely the vehicle and alibi for this other, much more ferocious, form of moral predation. One might almost say, contrary to the Marxist analysis, that material exploitation is only there to extract that spiritual raw material that is the misery of peoples, which serves as psychological nourishment for the rich countries and media nourishment for our daily lives. The 'Fourth World' (we are no longer dealing with a 'developing' Third World) is once again beleaguered, this time as a catastrophe-bearing stratum. The West is whitewashed in the reprocessing

of the rest of the world as waste and residue. And the white world repents and seeks absolution - it, too, the waste-product of its own history. The South is a natural producer of raw materials, the latest of which is catastrophe. The North, for its part, specializes in the reprocessing of raw materials and hence also in the reprocessing of catastrophe. Bloodsucking protection, humanitarian interference, Medecins sans frontieres, international solidarity, etc. The last phase of colonialism: the New Sentimental Order is merely the latest form of the New World Order. Other people's destitution becomes our adventure playground. Thus, the humanitarian offensive aimed at the Kurds - a show of repentance on the part of the Western powers after allowing Saddam Hussein to crush them - is in reality merely the second phase of the war, a phase in which charitable intervention finishes off the work of extermination. We are the consumers of the ever delightful spectacle of poverty and catastrophe, and of the moving spectacle of our own efforts to alleviate it (which, in fact, merely function to secure the conditions of reproduction of the

Page 70: openev.debatecoaches.org€¦  · Web view1AC . Observation 1: If I could find the spot where truth echoesI would stand there and whisper memories of my children's future. I would

catastrophe market); there, at least, in the order of moral profits, the Marxist analysis is wholly applicable: we see to it that extreme poverty is reproduced as a symbolic deposit, as a fuel essential to the moral and sentimental equilibrium of the West. In our defence, it might be said that this extreme poverty was largely of our own making and it is therefore normal that we should profit by it. There can be no finer proof that the distress of the rest of the world is at the root of Western power and that the spectacle of that distress is its crowning glory than the inauguration, on the roof of the Arche de la Defense, with a sumptuous buffet laid on by the Fondation des Droits de l'homme, of an exhibition of the finest photos of world poverty. Should we be surprised that spaces are set aside in the Arche d'Alliance* for universal suffering hallowed by caviar and champagne? Just as the economic crisis of the West will not be complete so

long as it can still exploit the resources of the rest of the world, so the symbolic crisis will be complete only when it is no longer able to feed on the other half's human and natural catastrophes (Eastern Europe, the Gulf, the Kurds, Bangladesh, etc.). We need this drug, which serves us as an aphrodisiac and hallucinogen. And the poor countries are the best suppliers - as, indeed, they are of other drugs. We provide them, through our media, with the means to exploit this paradoxical resource, just as we give them the means to exhaust their natural resources with our technologies. Our whole culture lives off this catastrophic cannibalism, relayed in cynical mode by the news media, and carried forward in moral mode by our humanitarian aid, which is a way of encouraging it and ensuring its continuity, just as economic aid is a strategy for perpetuating under-development. Up to now, the financial sacrifice has been compensated a hundredfold by the

moral gain; But when the catastrophe market itself reaches crisis point, in accordance with the implacable logic of the market, when distress becomes scarce or the marginal returns on it fall from overexploitation, when we run out of disasters from elsewhere or when they can no longer be traded like coffee or other commodities, the West will be forced to produce its own catastrophe for itself, in order to meet its need for spectacle and that voracious appetite for symbols which characterizes it even more than its voracious appetite for food. It will reach the point where it devours itself. When we have finished sucking out the destiny of others, we shall have to invent one for ourselves. The Great Crash, the symbolic crash, will come in the end from us Westerners, but only when we are no longer able to feed on the hallucinogenic misery which comes to us from the other half of the world. Yet they do not seem keen to give up their monopoly. The Middle East, Bangladesh, black Africa and Latin America are really going flat out in the distress and catastrophe stakes, and thus in providing symbolic nourishment for the rich world. They might be said to be overdoing it: heaping earthquakes, floods, famines and ecological disasters one upon another, and finding the means to massacre each other most of the time. The 'disaster show' goes on without any let-up and our sacrificial debt to them far exceeds their economic debt. The misery with which they generously overwhelm us is something we

shall never be able to repay. The sacrifices we offer in return are laughable (a tornado or two, a few tiny holocausts on the roads, the odd financial sacrifice) and, moreover, by some infernal logic, these work out as much greater gains for us, whereas our kindnesses have merely added to the natural catastrophes another one immeasurably worse: the demographic catastrophe, a veritable epidemic which we deplore each day in pictures. In short, there is such distortion between North and South, to the symbolic advantage of the South (a hundred thousand Iraqi dead against casualties numbered in tens on our side: in every case we are the losers), that one day everything will break down. One day, the West will break down if we are not soon washed clean of this shame, if an international congress of the poor countries does not very quickly decide to share out this symbolic privilege of misery and catastrophe. It is of course normal, since we refuse to allow the spread of nuclear weapons, that they should refuse to

allow the spread of the catastrophe weapon. But it is not right that they should exert that monopoly indefinitely. In any case, the under-developed are only so by comparison with the Western system and its presumed success. In the light of its assumed failure, they are not under-developed at all. They are only so in terms of a dominant evolutionism which has always been the worst of colonial ideologies. The argument here is that there is a line of objective progress and everyone is supposed to pass through its various stages (we find the same eyewash with regard to the evolution of

species and in that evolutionism which unilaterally sanctions the superiority of the human race). In the light of current upheavals, which put an end to any idea of history as a linear process, there are no longer either developed or under-developed peoples. Thus, to encourage hope of evolution - albeit by revolution - among the poor and to doom them, in keeping with the objective illusion of progress, to technological salvation is a criminal absurdity. In actual fact, it is their good fortune to be able to escape from evolution just at the point when we no longer

Page 71: openev.debatecoaches.org€¦  · Web view1AC . Observation 1: If I could find the spot where truth echoesI would stand there and whisper memories of my children's future. I would

know where it is leading. In any case, a majority of these peoples, including those of Eastern Europe, do not seem keen to enter this evolutionist modernity, and their weight in the balance is certainly no small factor in the West's repudiation of its own history, of its own utopias and its own modernity. It might be said that the routes of violence, historical or otherwise, are ·being turned around and that the viruses now pass from South to North, there being every chance that, five hundred years after America was conquered, 1992 and the end of the century will mark the comeback of the defeated and the sudden reversal of that modernity. The sense of pride is no longer on the side of wealth but of poverty, of those who - fortunately for them - have nothing to repent, and may indeed glory in being privileged in terms of catastrophes. Admittedly, this is a privilege they could hardly renounce, even if they wished to, but natural disasters merely reinforce the sense of guilt felt towards them by the wealthy - by those whom God visibly scorns since he no longer even strikes them down. One day it will be the Whites themselves who will give up their whiteness. It is a good bet that repentance will reach its highest pitch with the five-hundredth anniversary of the conquest of the Americas. We are going to have to lift the curse of the defeated - but symbolically victorious - peoples, which is insinuating itself five hundred years later, by way of repentance, into the heart of the white race.

Mourning requires the assimilation and cannibalism of the Other – memory and mourning of the Other instrumentalize and incorporate the Other within the self. Instead, we should refuse to mourn the Other as a testament to absolute alterity. Reynolds, 4 [Jack, Professor of Philosophy at the University of Tasmania, Possible and Impossible, Self and Other, and the Reversibility of Merleau-Ponty and Derrida, Spring 2004, Philosophy Today, Vol. 48, No. 1]

Moreover, in his essay "Fors: The Anglish Words of Nicolas Abraham and Maria Torok," Derrida again considers two models of the type of encroachment between self and other that is regularly associated with mourning. Borrowing from post-Freudian theories of mourning, he posits (although later undermines) a difference between introjection, which is love for the other person in me, and incorporation, which involves retaining the other person as a pocket, or a foreign body within one's own body. For Freud, as well as for the psychologists Abraham and Torok whose work Derrida considers, successful mourning is primarily about the introjection of the other person, and where they are consumed within us. The preservation of a discrete and separate other inside the self, as is the case in incorporation, is considered to be where mourning ceases to be a "normal" response and instead becomes pathological. Typically, Derrida reverses this hierarchy and order of Subordination by highlighting that there is a sense in which the supposedly pathological condition of incorporation is actually more respectful of the other's alterity. After all, incorporation means that one has not totally assimilated or digested the other, as there is still a difference and heterogeneity (EO 57). On the other hand, Abraham and Torok's "normal" mourning can be accused of interiorizing the other to such a degree that they have become assimilated and even metaphorically cannibalized .16 Derrida considers this introjection to be an infidelity to the other, although it will be argued that this type of mourning has more to recommend it than he allows. Indeed, it will be shown that Merleau-Ponty's notion of responsibility towards alterity is more "digestive" and appropriative, and it will also be argued that this is not necessarily a bad thing. Before we get ahead of ourselves, however, it is worth recognizing that Derrida's account is not so simple as to unreservedly valorize the incorporation of the other, even if he consistently emphasizes this paradigm in an effort to refute the canonical interpretation of successful mourning. he also acknowledges that the more the self "keeps the foreign element inside itself, the more it excludes it" (Fors xvii). Refusing to engage with the other, we exclude their foreignness from ourselves and hence prevent any transformative interaction with it. When fetishized in their externality in such a manner, the dead other really is lifeless and it is significant that Derrida describes the death of de Man in terms of the loss of exchange and of the transformational opportunities that he presented (MDM xvi).17 Derrida's point hence seems to be that in mourning we must oscillate between

Page 72: openev.debatecoaches.org€¦  · Web view1AC . Observation 1: If I could find the spot where truth echoesI would stand there and whisper memories of my children's future. I would

introjection and incorporation, but the "otherness of the other" nevertheless resists both the process of incorporation as well as the process of introjection. The other can neither be preserved as a foreign entity, nor introjected fully within. Instead, the other must always resist my memory and interiorization of them, and remains outside the grasp of subjectivity-that is, wholly other (tout autre). Towards the end of Memoires: for Paul de Man, Derrida suggests that responsibility towards alterity is precisely about respecting and even emphasizing this resistance of the other, as well as the concomitant impossibility of mourning (MDM 160, 238). It is on this point that he parts company with Merleau-Ponty, and there are again two main responses that Merleau-Ponty's work can help us furnish.

The conflict in Ukraine is not driven by the USREBECCA KHEEL. 03/27/18. [Staff Writer], Congress bans arms to Ukraine militia linked to neo-Nazis, The Hill. https://thehill.com/policy/defense/380483-congress-bans-arms-to-controversial-ukrainian-militia-linked-to-neo-nazis. EC

A little-noticed provision in the 2,232-page government spending bill passed last week bans U.S. arms from going to a controversial ultranationalist militia in Ukraine that has openly accepted neo-Nazis into its ranks. House-passed spending bills for the past three years have included a ban on U.S. aid to Ukraine from going to the Azov Battalion, but the provision was stripped out before final passage each year. This year, though, the $1.3 trillion omnibus spending bill signed into law last week stipulates that “none of the funds made available by this act may be used to provide arms, training or other assistance to the Azov Battalion.” “White supremacy and neo-Nazism are unacceptable and have no place in our world,” Rep. Ro Khanna (D-Calif.), an outspoken critic of providing lethal aid to Ukraine, said in a statement to The Hill on Tuesday. “I am very pleased that the recently passed omnibus prevents the U.S. from providing arms and training assistance to the neo-Nazi Azov Battalion fighting in Ukraine .”

The 1ac perpetuates the norm against noticing the inherent racism associated in ALL modern IR theories….each of them is fundamentally defined in opposition to the concept of ANARCHY, but this Anarchy is built upon racist assumptions and foundations – this failure to foreground race within IR in place of a new defensive realist position guarantees that their scholarship is not only WHOLLY inaccurate, but justifies violence against the majority of the world;s peoplesHenderson 15 (Errol, Race and Racism in International Relations, edited by Anievas Alexander, p. 35-6)

Summary Thus, it is not difficult to trace the historical and conternporary role and impact Of racism in IR theory. Racism has not only informed the paradigms Of world politics, it Was fundamendal to the conceptualisation Of its key theoretical touchstone: anarchy _ The social Contract theorists rooted their conceptualisations Of the State Of nature in a broader contract" that dichotomised humanity racially and established a White supremacist hierarchy in their foundational conceptions Of society _ Late nineteenth and early twentieth—century theorists built on this racist dualism as they constructed their conception Of a global anarchy and the role Of Whites to provide, maintain, and ensure order Within it by a System Of international power relations among Whites — or at minimum, dominated by Whites, and a System Of colonial subjugation for

non—whites — or those nonwhites Who failed to successfully resist their domination militarily. The impact and role Of racism is manifest through the major paradigms operative today — realism, neorealism, liberalism/idealism and

Page 73: openev.debatecoaches.org€¦  · Web view1AC . Observation 1: If I could find the spot where truth echoesI would stand there and whisper memories of my children's future. I would

constructivism — mainly through their continued reliance on a racist conception Of anarchy; in the case Of neorealism through its grounding in African primitivism, while for Marxism, its reliance on ‘normalising’ Of a Eurocentric teleology Of economic

development for the world. TO be sure, the dualism at the broad theoretical level Of paradigms underscores, guides, and informs the more specific dichotomies at the level Of theories, models, and theses that are derived from these paradigms — especially those that are applied to Africa's political processes, and those Of other regions as well. In the Of African international relations, they both and rationalize a black African primitivism juxtaposed to a White Western progressivism, a black African savagery and a White Western universalist humanity, resulting in an enduring African warfare frame Of reference contrasted to an evolved Western democratic peace; in each Case a Static ossified ahistorical contrasted to a dynamic evolving transcendence. One result is that one must endure What are considered to be "meaningful" "appropriate" or or "cutting—edge" discussions Of Africa's and international politics that have as their point Of departure loose and often Obtuse references to "hearts Of darkness" , "greed Versus grievance", "tribal warfare" or a litany Of other metaphors that would not pass the editor's desk at most top—tier journals as legitimate Contexts through which to Observe and examine contemporary armed conflicts in the Western world7. Notably, rarely do those same journals publish work on the and enduring embedded in the major paradigms Of world politics or discuss the implications Of such a condition if it were shown to obtain. In fact, the "norm against noticing" White racism is so intense that it engenders a ‘silencing' Of those Who would raise it; or it ensures against publication in main— Stream outlets for such work except that it provides appropriate euphemisms for the attrocities associated with white racism — especially against blacks o r the requisite "balance " to emphasize the role Of non—whites in their subjugation as if White and the imperialism, colonialism, neocolonialism and internal colonialism that it employed against Africans, Asians, and Native Americans are somehow the responsibility Of groups other than the Whites Who created it, maintained it, and continue to profit from it. Thus, the racist dualism in world polities creates, in turn, a dual quandary for scholars and many Afr1can1StS seeking to publish in Western journals — and many non—Western

ones, too Wherein White racist expectations Of the appropriateness Of certain lines Of inquiry often limits the discourse Of politics to hollow phraseology and meaningless metaphors, while they simultaneously check informed challenges to historical and contemporary expressions, practices, and institutions Of White racism in academia by ensuring that such racism is rarely confronted in the publications in IR "world politics

in Clear and direct terms _ Another result is that it leaves scholars a history Of the development Of i nternational relations

that ignores the salience Of colonialism as the centerpiece in the origins Of the field. That is, in continuing to teach the fiction that the field emerged following the devastation Of World War l, as "idealists" led by Wilson and Others such as Dickinson, and Kerr sought to provide the institutional checks on realpolltlk that Was implicated in the to end all wars". We belie the reality Of the centrality Of colonialism,

race develop— ment, and White racial supremacy to the development Of the field Of Thus, our narrative creates an academic fiction that hovers outside Of its own history. The presence of this narrative is as much a testament to the white supremacism that is a centerpiece of the field given that its role is to ensure a “norm against noticing” the centrality of white racism in world politics while simultaneously “silencing” or making marginal those who would focus on the importance of white racism in the development of the field of IR/world politics, or those who would raise this as a legitimate research focus for the most sensible of reasons: it happens to be true. CONCLUSION: In this chapter I have attempted to address the centrality of racism in international relations (IR) theory. It examines the extent to which

realism, liberalism, and constructivism are oriented by racist precepts grounding in the intellectual foundation of IR. Specifically, a racist dualism inheres within the assumptions informing the foundational construct of IR; namely, anarchy; and due to the centrality of this construct within prominent theses that draw on it, racist precepts have an enduring impact on IR, theory today. In sum, a racist latticework undergirds major theoretical frameworks that inform research and policy in IR . Theses that rest on racist claims are not simply odious, but are untethered to the reality (i.e. world politics itself) that they purport to explain.

Vitalis (2000) is correct that there is a “norm against noticing” white supremacism in mainstream IR discourse. The failure to address it leaves IR analysts ill-equipped to address its intellectual history, its theoretical development, and its prospects for theory building that will generate meaningful research and policy for the vast majority of the world’s people.

Page 74: openev.debatecoaches.org€¦  · Web view1AC . Observation 1: If I could find the spot where truth echoesI would stand there and whisper memories of my children's future. I would

Framing:

Reject their modes of ethics, ethics should not be considered when they were created in a world that sustains modern day slavery and anti blackness.

If the US is responsible for the conflict, then you can cross apply the Baudrillard 1994 Charity Cannibalism evidence to the framing flow, it directly quotes “the West will be forced to produce its own catastrophe for itself, in order to meet its need for spectacle and that voracious appetite for symbols ” This quote just goes to show how fucked up it is that the US created the conflict in Ukraine just to come in and save the day and satisfy its appetite for the exploitation of foreign nations and conflicts, specifically those that the 1AC claims to solve for.

The Olson evidence in the 1AC states that “ Saving the world still should require a debate about whose world is being saved, when, and at what cost”

Page 75: openev.debatecoaches.org€¦  · Web view1AC . Observation 1: If I could find the spot where truth echoesI would stand there and whisper memories of my children's future. I would

Vs. UkraineThe 1AC relies on a futural hope of a “not-yet-realized” future where they fantasize about contingent solutions that never come. This hope creates a cruel optimism for black folk that forces them to invest in the pursuit of our own death because it crowds out all non-politically recognizable alternatives. This model for politics only re-entrenches the anti-black world and is an independent reason to reject the aff. Warren 15 (Calvin L., Black Nihilism and the Politics of Hope; Surce: CR: The New Centennial Review, Vol. 15, No. 1, Derrida and French Hegelianism (Spring 2015), XMT, pp. 215-248 Published by: Michigan State University Press Stable URL: http://www.jstor.org/stable/10.14321/crnewcentrevi.15.1.0215 . Accessed: 30/03/2015)

The politics of hope , then, constitutes what Lauren Berlant would call “ cruel optimism” for blacks (Berlant 2011). It bundles certain promises about redress, equality, freedom, justice, and progress into a political object that always lies beyond reach. The objective of the Political is to keep blacks in a relation to this political object—in an unending pursuit of it. This pursuit, however, is detrimental because it strengthens the very anti-black system that would pulverize black being . The pursuit of the object certainly has an “irrational” aspect to it, as Farred details, but it is not mere means without expectation; instead, it is a means that

undermines the attainment of the impossible object desired. In other words, the pursuit marks a cruel attachment to the means of subjugation and the continued widening of the gap between historical reality and fantastical ideal. Black nihilism is a “demythifying” practice, in the Nietzschean vein, that uncovers the subjugating strategies of

political hope and de-idealizes its fantastical object. Once we denude political hope of its axiological and ethical veneer, we see that it operates through certain strategies: 1) positing itself as the only alternative to the problem of anti-blackness, 2) shielding this alternative [End Page 221] from rigorous historical/philosophical critique by placing it in an unknown

future, 3) delimiting the field of action to include only activity recognized and legitimated by the Political, and 4) demonizing critiques or different philosophical perspectives. The politics of hope masks a particular cruelty under the auspices of “happiness” and “life.” It terrifies with the dread of “no alternative .” “Life” itself needs the security of the alternative, and, through this logic, life becomes untenable without it.

Political hope promises to provide this alternative—a discursive and political organization beyond extant structures of violence and destruction. The construction of the binary “alternative/no-alternative” ensures the hegemony and dominance of political hope within the onto-existential horizon. The terror of the “no alternative”—the ultimate space of decay, suffering, and death—depends on two additional binaries: “problem/solution” and “action/inaction.” According to this politics, all problems have solutions, and hope provides the accessibility and realization of these solutions. The solution establishes itself as the elimination of “the problem”; the solution, in fact, transcends the problem and realizes Hegel’s aufheben in its constant attempt to sublate the dirtiness of the “problem” with the pristine being of the

solution. No problem is outside the reach of hope’s solution—every problem is connected to the kernel of its own eradication. The politics of hope must actively refuse the possibility that the “solution” is, in fact, another problem in disguised form; the idea of a “solution” is nothing more than the repetition and disavowal of the problem itself. The solution relies on what we might call the “trick of time”

to fortify itself from the deconstruction of its binary. Because the temporality of hope is a time “not-yet-realized,” a future tense unmoored from present-tense justifications and pragmatist evidence, the politics of hope cleverly shields its “solutions” from critiques of impossibility or repetition. Each insistence that these solutions stand up against the lessons of history or the rigors of analysis is met with the rationale that these solutions are not subject to history or analysis because they do not reside within the horizon of the “past” or “present.” Put differently, we can never ascertain the efficacy of the

Page 76: openev.debatecoaches.org€¦  · Web view1AC . Observation 1: If I could find the spot where truth echoesI would stand there and whisper memories of my children's future. I would

proposed solutions because they escape the temporality of the moment, always retreating to a “not-yet” and “could-be” temporality. This “trick” of time offers a promise of possibility that can only be realized in an indefinite future, and this promise is a bond of uncertainty that can never be redeemed, only imagined. In this sense, the politics of hope is an instance of the psychoanalytic notion of desire: its sole

purpose is to reproduce its very condition of possibility, never to satiate or bring fulfillment. This politics secures its hegemony through time by claiming the future as its unassailable property and excluding (and

devaluing) any other conception of time that challenges this temporal ordering. The politics of hope, then, depends on the incessant (re)production and proliferation of problems to justify its existence. Solutions cannot really exist within the politics of hope, just the illusion of a different order in a future tense . The “trick” of time and political solution converge on the site of “action.” In critiquing the politics of hope, one encounters the rejoinder of the dangers of inaction. “But we can’t just do nothing! We have to do something.” The field of permissible action is delimited and an unrelenting binary between action/ inaction silences critical engagement with political hope. These exclusionary operations rigorously

reinforce the binary between action and inaction and discredit certain forms of engagement, critique, and protest. Legitimate action takes place in the political—the political not only claims futurity but also action as its property. To “do something” means that this doing must translate into recognizable political activity; “something” is a stand-in for the word “politics”—one must “do politics” to address any problem. A refusal to “do politics” is equivalent to “doing nothing”—this nothingness is constructed as the antithesis of life, possibility, time, ethics, and morality (a “zero-state” as Julia Kristeva [1982] might call it).

Black nihilism rejects this “trick of time” and the lure of emancipatory solutions. To refuse to “do politics” and to reject the fantastical object of politics is the only “hope” for blackness in an antiblack world.

The only possible demand is one that calls for the end of the world itself—the affirmative represents a conflict within the paradigm of America but refuses to challenge the foundational antagonism that produces the violence that undergirds that same paradigm Wilderson, ’10 [2010, Frank B. Wilderson is an Associate Professor of African-American Studies at UC Irvine and has a Ph.D. from UC Berkeley, “Red, White & Black: Cinema and the Structure of U.S. Antagonisms,”]

Leaving aside for the moment their state of mind, it would seem that the structure , that is to say the rebar, or better still the grammar of their demands —and, by extension, the grammar of their suffering—was indeed an ethical grammar. Perhaps their grammars are the only ethical grammars available to modern politics and modernity writ large , for they draw our attention not to the way in which space and time are used and abused by enfranchised and violently powerful interests, but to the violence that underwrites the modern world ’ s capacity to think, act, and exist spatially and temporally . The violence that robbed her of her body and him of his land provided the stage upon which other

violent and consensual dramas could be enacted. Thus, they would have to be crazy , crazy enough to call not merely the actions of the world to account but to call the world itself to account , and to account for them no less! The woman at Columbia was not demanding to be a participant in an unethical network of distribution: she was not demanding a place within capital, a piece of the pie (the demand for her sofa notwithstanding). Rather, she was articulating a triangulation between, on the one hand, the loss of her body, the very dereliction of her corporeal integrity, what Hortense Spillers charts as the transition from being a being to becoming a “being for the captor” (206), the drama of value (the stage upon which surplus value is extracted from labor power through commodity production and sale); and on the other, the corporeal integrity that, once ripped from her body, fortified and extended the corporeal integrity of everyone else on the street. She gave birth to the commodity and to the Human, yet

she had neither subjectivity nor a sofa to show for it. In her eyes, the world — and not its myriad discriminatory practices, but the world itself — was unethical . And yet, the world passes by her without the slightest inclination to stop and disabuse her of her claim . Instead, it calls her “crazy.” And to what does the world

Page 77: openev.debatecoaches.org€¦  · Web view1AC . Observation 1: If I could find the spot where truth echoesI would stand there and whisper memories of my children's future. I would

attribute the Native American man’s insanity? “He’s crazy if he thinks he’s getting any money out of us”? Surely, that doesn’t make him crazy.

Rather it is simply an indication that he does not have a big enough gun . What are we to make of a world that responds to the most lucid enunciation of ethics with violence ? What are the foundational questions of the ethico-political? Why are these questions so scandalous that they are rarely posed politically, intellectually, and cinematically—unless they are

posed obliquely and unconsciously, as if by accident? Return Turtle Island to the “Savage.” Repair the demolished subjectivity of the Slave . Two simple sentences, thirteen simple words, and the structure of U.S. ( and perhaps global ) antagonisms would be dismantled . An “ ethical modernity ” would no longer sound like an oxymoron . From there we could busy ourselves with important conflicts that have been promoted to the level of antagonisms: class struggle, gender conflict, immigrants rights. When pared down to thirteen words and two

sentences, one cannot but wonder why questions that go to the heart of the ethico-political , questions of political ontology, are so unspeakable in intellectual meditations, political broadsides , and even socially and politically engaged feature films. Clearly they can be spoken, even a child could speak those lines, so they would pose no problem for a scholar,

an activist, or a filmmaker. And yet, what is also clear—if the filmographies of socially and politically engaged directors, the archive of progressive scholars , and the plethora of Left-wing broadsides are anything to go by — is that what can so easily be spoken is now (five hundred years and two hundred fifty million Settlers/Masters on) so ubiquitously unspoken that these two simple sentences, these thirteen words not only render their speaker “ crazy ” but become themselves impossible to imagine . Soon it will be forty years since radical politics, Left-

leaning scholarship, and socially engaged feature films began to speak the unspeakable. In the 1960s and early 1970s the questions asked by radical politics and scholarship were not “ Should the U.S. be overthrown? ” or even “ Would it be overthrown? ” but rather when and how —and, for some, what—would come in its wake. Those steadfast in their conviction that there remained a discernable quantum of ethics in the U.S. writ large (and here I am speaking of everyone from Martin Luther King, Jr., prior to his 1968 shift, to the Tom Hayden wing of SDS, to the Julian Bond and Marion Barry

faction of SNCC, to Bobbie Kennedy Democrats) were accountable , in their rhetorical machinations, to the paradigmatic zeitgeist of the Black Panthers, the American Indian Movement, and the Weather Underground . Radicals and progressives could deride , reject, or chastise armed struggle mercilessly and cavalierly with respect to tactics and the possibility of “ success, ” but they could not dismiss revolution-as-ethic because they could not make a convincing case —by way of a paradigmatic analysis—that the U.S. was an ethical formation and still hope to maintain credibility as radicals and progressives . Even Bobby Kennedy (a U.S. attorney general and

presidential candidate) mused that the law and its enforcers had no ethical standing in the presence of Blacks . One could (and many did) acknowledge America’s strength and power. This seldom, however, rose to the level of an ethical assessment, but rather remained an assessment of the so-called “ balance of forces. ” The political discourse of Blacks, and to a lesser extent Indians, circulated too widely to credibly wed the U.S. and ethics. The raw force of COINTELPRO put an end to this

trajectory toward a possible hegemony of ethical accountability. Consequently, the power of Blackness and Redness to pose the question — and the power to pose the question is the greatest power of all — retreated as did White radicals and progressives who “ retired ” from struggle. The question ’ s echo lies buried in the graves of young Black Panthers, AIM Warriors, and Black Liberation Army soldiers , or in prison cells where so many of them have been rotting (some in solitary confinement) for ten, twenty, thirty years, and at the gates of the academy where the “ crazies ” shout at passers-by . Gone are not only the young and vibrant voices that affected a seismic shift on the political landscape , but also the intellectual protocols of inquiry, and with them a spate of feature films that became authorized, if not by an unabashed revolutionary polemic, then certainly by a revolutionary zeitgeist . Is it still possible for a dream of unfettered ethics, a dream of the Settlement and the Slave estate ’ s destruction , to manifest itself at the ethical core of cinematic discourse, when this dream is no longer a constituent element of political discourse in the streets nor of intellectual discourse in the academy? The answer is “no” in the sense that, as history has shown, what cannot be articulated as political discourse in the streets is doubly foreclosed upon

in screenplays and in scholarly prose; but “yes” in the sense that in even the most taciturn historical moments such as ours, the grammar of Black and Red suffering breaks in on this foreclosure , albeit like the somatic compliance of

Page 78: openev.debatecoaches.org€¦  · Web view1AC . Observation 1: If I could find the spot where truth echoesI would stand there and whisper memories of my children's future. I would

hysterical symptoms—it registers in both cinema and scholarship as symptoms of awareness of the structural antagonisms. Between 1967 and 1980, we could think cinematically and intellectually of Blackness and Redness as having the coherence of full-blown discourses. But from 1980

to the present, Blackness and Redness manifests only in the rebar of cinematic and intellectual (political) discourse, that is, as unspoken grammars . This grammar can be discerned in the cinematic strategies (lighting,

camera angles, image composition, and acoustic strategies/design), even when the script labors for the spectator to imagine social turmoil through the rubric of conflic t (that is, a rubric of problems that can be posed and conceptually solved) as opposed to the rubric of antagonism ( an irreconcilable struggle between entities , or positionalities, the resolution of which is not dialectical but entails the obliteration of one of the positions ). In other words, even when films narrate a story in which Blacks or Indians are beleaguered with problems that the script insists are conceptually coherent (usually having to do with poverty or the absence of “family values”), the non-narrative, or cinematic, strategies of the film often disrupt this coherence by posing the irreconcilable questions of Red and Black political ontology—or non-ontology.

The grammar of antagonism breaks in on the mendacity of conflict . Semiotics and linguistics teach us that when we

speak, our grammar goes unspoken. Our grammar is assumed. It is the structure through which the labor of speech is possible. Likewise, the grammar of political ethics —the grammar of assumptions regarding the ontology of suffering — which underwrite Film Theory and political discourse (in this book, discourse elaborated in direct relation to radical action), and which underwrite

cinematic speech (in this book, Red, White, and Black films from the mid-1960s to the present) is also unspoken . This notwithstanding,

film theory, political discourse , and cinema assume an ontological grammar, a structure of suffering . And the structure of suffering which film theory, political discourse, and cinema assume crowds out other structures of suffering, regardless of the sentiment of the film or the spirit of unity mobilized by the political discourse in question. To put a finer point on it, structures of ontological suffering stand in antagonistic, rather then conflictual, relation to one another (despite the fact that antagonists themselves may not be aware of the ontological positionality from which they speak). Though this is perhaps the most controversial and out-of-step claim of this book, it is, nonetheless, the foundation of the close reading of feature films and political theory that follows.

US hegemony is just the racial violence of America gone global –aff claims to benevolence are symptoms of white privilegeRodriguez ‘07 [Dylan, PhD in Ethnic Studies Program of the University of California Berkeley and Associate Proffessor of Ethnic Studies at University of California Riverside, “American Globality And the US Prison regime: State Violence And White Supremacy from Abu Ghraib to Stockton to bagong diwa”, Ateneo de Manila University, 2007, Kritika Kultura 9 (2007): 022-048]

In fact, the notion of American globality I have begun discussing here already exceeds negri and Hardt’s formulation to the extent that it is a global racial formation, and more pointedly a global mobilization of a white supremacist social formation (read: a united States of America formed by the social-economic geographies of racial chattel slavery and their recodification through the post-13th

Amendment innovation of other technologies of criminalization and imprisonment). The US prison regime’s production of human immobilization and death composes some of the fundamental modalities of American national coherence .

It inscribes two forms of domination that tend to slip from the attention of political theorists, including Negri and Hardt: first, the prison regime

strategically institutionalizes the biopolitical structures of white racial/nationalist ascendancy—it quite concretely provides a definition for white American personhood, citizenship, freedom, and racialized patriotism. Second, the prison regime reflects the moral, spiritual, and cultural inscription of Manifest Destiny (and its descendant material cultural and state-building articulations of racist and white supremacist conquest, genocide, and population control) across different historical moments. to invoke and

critically rearticulate negri and Hardt’s formulation, the focal question becomes: How does the right of the US-as-global police to kill, detain, obliterate become voiced, juridically coded, and culturally recoded? the structure of presumption—and therefore relative political silence—enmeshing the prison’s centrality to the logic of American globality is precisely evidence of the fundamental power of the uS prison regime within the larger schema of American hegemony. In this sense the uS prison regime is ultimately really not an “institution.” rather it is a formulation of world order (hence, a dynamic and perpetual labor of institutionalization rather than a definitive modernist institution) in which massively scaled, endlessly strategized technologies of human immobilization address (while never fully resolving) the socio-political crises of globalization. The US prison regime defines a global logic of social organization that constitutes, mobilizes, and prototypes across various localities. What would it mean, then, to consider state-crafted, white supremacist modalities of

Page 79: openev.debatecoaches.org€¦  · Web view1AC . Observation 1: If I could find the spot where truth echoesI would stand there and whisper memories of my children's future. I would

imprisonment as the perpetual end rather than the self-contained means of American globality? I am suggesting a conception of the prison regime that focuses on what cultural and political theorist Allen Feldman calls a “formation of violence,” which anchors the contemporary articulation of white supremacy as a global technology of coercion and hegemony. Feldman writes, the growing autonomy of violence as a self-legitimating sphere of social discourse and transaction points to the inability

of any sphere of social practice to totalize society. Violence itself both reflects and accelerates the experience of society as an incomplete project, as something to be made. As a formation of violence that self-perpetuates a peculiar social project through the discursive structures of warfare, the US prison regime composes an acute formation of racial and white supremacist violence, and thus houses the capacity for mobilization of an epochal (and peculiar) white supremacist global logic. This contention should not be confused with the sometimes parochial (if not politically chauvinistic) proposition that American state and state-sanctioned regimes of bodily violence and human immobilization are somehow self-contained “domestic” productions that are exceptional to the united

States of America, and that other “global” sites simply “import,” imitate, or reenact these institutionalizations of power. In fact, I am suggesting the opposite: the US prison regime exceeds as it enmeshes the ensemble of social relations that cohere uS civil society, and is fundamental to the geographic transformations, institutional vicissitudes, and militarized/economic mobilizations of “globalization” generally . to assert this, however, is to also argue that the constituting violence of the US prison regime has remained somewhat undertheorized and objectified in the overlapping realms of public discourse, activist mobilization, and (grassroots as well as professional) scholarly praxis.

Here I am arguing that it is not possible to conceptualize and critically address the emergence and global proliferation of the (uS/global) prison industrial complex outside a fundamental understanding of what are literally its technical and technological premises: namely, its complex organization and creative production of racist and white supremacist bodily violence . It is only in this context, I would say, that we can examine the problem of how “the Prison” is a modality (and not just a reified product or outcome) of American statecraft in the current political moment. It is only a theoretical foregrounding of the white supremacist state and social formation of the united States that will allow us to understand the uS prison regime as an American globality that materializes as it prototypes state violence and for that matter, “state power” itself through a specific institutional site.

Institutional structures of domination create everyday holocausts—you should reject singular focused impacts in favor of working against the ongoing extinctions of people of color [international conflict impacts] Omolade 89, [1989, Barbara Omolade is a historian of black women for the past twenty years and an organizer in both the women’s and civil rights/black power movements, “We Speak for the Planet” in “Rocking the ship of state : toward a feminist peace politics”, pp. 172-176]

Recent efforts by Soviet leader Mikhail Gorbachev and President Ronald Reagan to limit nuclear testing, stockpiling, and weaponry, while still protecting their own arsenals and selling arms to countries and factions around the world, vividly demonstrate how "peace" can become an abstract concept within a culture of war. Many peace activists are similarly blind to the constant wars and threats of war being waged against people of color and the planet by those who march for "peace" and by those they march against. These pacifists , like Gorbachev and Reagan , frequently want people of color to fear what they fear and define peace as they define it . They are unmindful that our lands and peoples have already been and are being destroyed as part of the "final solution" of the "color line." It is difficult to persuade the remnants of Native American tribes , the starving of African deserts , and the victims of the Cambodian "killing fields" that nuclear war is the major danger to human life on the planet and that only a nuclear "winter" embodies fear and futurelessness for humanity . The peace movement suffers greatly from its lack of a historical and holistic perspective, practice, and vision that include the voices and experiences of people of color; the movement's goals and messages have therefore been

Page 80: openev.debatecoaches.org€¦  · Web view1AC . Observation 1: If I could find the spot where truth echoesI would stand there and whisper memories of my children's future. I would

easily coopted and expropriated by world leaders who share the same culture of racial dominance and arrogance. The peace movement's racist blinders have divorced peace from freedom, from feminism, from education reform, from legal rights, from human rights, from international alliances and friendships, from national liberation, from the particular (for example, black female, Native American male) and the general (human being). Nevertheless, social movements such as the civil rights-black power movement in the United States have always demanded peace with justice, with liberation, and with social and economic reconstruction and cultural

freedom at home and abroad. The integration of our past and our present holocausts and our struggle to define our own lives and have our basic needs met are at the core of the inseparable struggles for world peace and social betterme nt . The Achilles heel of the organized peace movement in this country has always been its whiteness. In this multi-racial and racist society, no allwhite movement can have the strength to bring about basic changes. It is axiomatic that basic changes do not occur in any society unless the people who are oppressed move to make them occur. In our society it is people of color who are the most oppressed. Indeed our entire history teaches us that when people of color have organized and struggled-most especially, because of their particular history, Black people-have moved in a more humane direction as a society, toward a better life for all people.1 Western man's whiteness, imagination, enlightened science, and movements toward peace have developed from a culture and history mobilized against women of color. The political advancements of white men have grown directly from the devastation and holocaust of people of color and our lands. This technological and material progress has been in direct proportion to the undevelopment of women of color. Yet the dayto- day survival, political struggles, and rising up of women of color, especially black women in the United States, reveal both complex resistance to

holocaust and undevelopment and often conflicted responses to the military and war. The Holocausts Women of color are survivors of and remain casualties of holocausts , and we are direct victims of war-that is, of open armed conflict between countries or between factions within the same country. But women of color were not soldiers, nor did we trade animal pelts or slaves to the white man for guns, nor did we sell or lease our lands to the white man for wealth. Most men and women of color resisted and fought back , were slaughtered , enslaved , and force marched into plantation labor camps to serve the white masters of war and to build their empires and war machines. People of color were and are victims of holocausts-that is, of great and widespread destruction, usually by fire. The world as we knew and created it was destroyed in a continual scorched earth policy of the white man. The experience of Jews and other Europeans under the Nazis can teach us the value of understanding the totality of destructive intent, the extensiveness of torture, and the demonical apparatus of war aimed at the human spirit. A Jewish father pushed his daughter from the lines of certain death at Auschwitz and said, "You will be a remembrance-You tell the story. You survive." She lived. He died. Many have criticized the Jews for forcing non-Jews to remember the 6 million Jews who died under the Nazis and for etching the names Auschwitz and Buchenwald, Terezin and Warsaw in our

minds. Yet as women of color, we, too, are "remembrances" of all the holocausts against the people of the world. We must remember the names of concentration camps such as Jesus, Justice, Brotherhood, and Integrity, ships that carried millions of African men, women, and children chained and brutalized across the ocean to the "New World." We must remember the Arawaks, the Taino, the Chickasaw, the Choctaw, the Narragansett, the Montauk, the Delaware, and the other Native American names of thousands of U.S. towns that stand for tribes of people who are no more. We must remember the holocausts visited against the Hawaiians , the aboriginal peoples of Australia , the Pacific Island peoples, and the women and children of Hiroshima and Nagasaki. We must remember the slaughter of men and women at Sharpeville, the children of Soweto, and the men of Attica. We must never, ever, forget the children disfigured, the men maimed, and the women broken in our holocausts -we must remember the names, the numbers, the faces, and the stories and teach them to our children and our children's children so the world can

never forget our suffering and our courage. Whereas the particularity of the Jewish holocaust under the Nazis is over, our holocausts continue. We are the madres locos (crazy mothers) in the Argentinian square silently demanding news of our missing kin from the fascists who rule. We are the children of El Salvador who see our mothers and fathers shot in front of our eyes. We are the Palestinian and Lebanese women and children overrun by Israeli, Lebanese, and U.S. soldiers. We are the women and children of the bantustans and refugee camps and the prisoners of Robbin Island . We are the starving in the Sahel , the poor in Brazil , the sterilized in Puerto Rico. We are the brothers and sisters of Grenada who carry the seeds of the New Jewel Movement in our hearts, not daring to speak of it with our lipsyet. Our holocaust is South Africa ruled by men who loved Adolf Hitler, who have developed the Nazi techniques of terror to more sophisticated levels. Passes replace the Nazi badges and stars. Skin color is the ultimate badge of persecution. Forced removals of women, children, and the elderly-the "useless appendages of South Africa"-into barren, arid bantustans without resources for survival have replaced the need for concentration camps. Black sex-segregated barracks and cells attached to work sites achieve two objectives: The work camps destroy black family and community life, a presumed source of resistance, and attempt to create human automatons whose purpose is to serve the South African state's drive toward wealth and hegemony. Like other fascist regimes, South Africa disallows any democratic rights to black people; they are denied the right to vote, to dissent, to peaceful assembly, to free speech, and to

Page 81: openev.debatecoaches.org€¦  · Web view1AC . Observation 1: If I could find the spot where truth echoesI would stand there and whisper memories of my children's future. I would

political representation. The regime has all the typical Nazi-like political apparatus: house arrests of dissenters such as Winnie Mandela; prison murder of protestors such as Stephen Biko; penal colonies such as Robbin Island. Black people, especially children, are routinely arrested without cause, detained without limits, and confronted with the economic and social disparities of a nation built around racial separation. Legally and economically, South African apartheid is structural and institutionalized racial war. The Organization of African Unity's regional intergovernmental meeting in 1984 in Tanzania was called to review and appraise the achievements of the United Nations Decade for Women. The meeting considered South Africa's racist apartheid regime a peace issue. The "regime is an affront to the dignity of all Africans on the continent and a stark reminder of the absence of equality and peace, representing the worst form of institutionalized oppression and strife." Pacifists such as Martin Luther King, Jr. and Mahatma Gandhi who have used nonviolent resistance charged that those who used violence to obtain justice were just as evil as their oppressors. Yet all successful revolutionary movements have used organized violence. This is especially true of national liberation movements that have obtained state power and reorganized the institutions of their nations for the benefit of the people. If men and women in South Africa do not use organized violence, they could remain in the permanent violent state of the slave. Could it be that pacifism and nonviolence cannot become a way of life for the oppressed? Are they only tactics with specific and limited use for protecting people from further violence? For most people in the developing communities and the developing world consistent nonviolence is a luxury; it presumes that those who have and use nonviolent weapons will refrain from using them long enough for nonviolent resisters to win political battles. To survive, peoples in developing countries must use a varied repertoire of issues, tactics, and approaches. Sometimes arms are needed to defeat apartheid and defend freedom in South Africa; sometimes nonviolent demonstrations for justice are the appropriate strategy

for protesting the shooting of black teenagers by a white man, such as happened in New York City. Peace is not merely an absence of 'conflict that enables white middleclass comfort, nor is it simply resistance to nuclear war and war machinery. The litany of "you will be blown up, too" directed by a white man to a black woman obscures the permanency and institutionalization of war, the violence and holocaust that people of color face daily. Unfortunately, the holocaust does not only refer to the mass murder of Jews, Christians, and atheists during the Nazi regime; it also refers to the permanent institutionalization of war that is part of every fascist and racist regime. The holocaust lives. It is a threat to world peace as pervasive and thorough as nuclear war.

The alternative is an unflinching paradigmatic analysis of the uncivility of civil society – the reclamation of the power to pose the question and analysis of ontological questions to articulate the worldWilderson 10 (Frank Wilderson, Professor of African American Studies at UC Irvine, “Red, White & Black: Cinema and the Structure of U.S. Antagonisms”)

STRANGE AS it might seem, this book project began in South Africa. During the last years of apartheid I worked for revolutionary change in both an underground and above-ground capacity, for the Charterist Movement in general and the ANC in particular. During this period, I began to see how essential an unflinching paradigmatic analysis is to a movement dedicated to the complete overthrow of an existing order. The neoliberal compromises that the radical elements of the Chartist Movement made with the moderate elements were due, in large part, to our inability or unwillingness to hold the moderates' feet to the fire of a political agenda predicated on an unflinching paradigmatic analysis. Instead, we allowed our energies and points of attention to be displaced by and onto pragmatic

considerations. Simply put, we abdicated the power to pose the question—and the power to pose the question is the greatest power of all. Elsewhere, I have written about this unfortunate turn of events (Incognegro: A Memoir of Exile and Apartheid), so I'll not rehearse the details here. Suffice it to say, this book germinated in the many political and academic discussions and debates that I was fortunate enough to be a part of at a historic moment and in a place where the word revolution was spoken in earnest, free of qualifiers and irony. For their past and ongoing ideas and interventions, I extend solidarity and appreciation to comrades Amanda Alexander, Franco Barchiesi, Teresa Barnes, Patrick Bond, Ashwin Desai, Nigel Gibson, Steven Greenberg, Allan Horowitz, Bushy Kelebonye (deceased), Tefu Kelebonye, Ulrike Kistner, Kamogelo Lekubu, Andile Mngxitama, Prishani Naidoo, John Shai, and S'bu Zulu. .¶ [CONTINUES]¶ In the Introduction and chapter 1, we saw how the aporia between Black being and political ontology has existed since Arab and European

enslavement of Africans. The crafting of questions through which one might arrive at an unflinching paradigmatic analysis of political ontology , a language that could express the structural and performative violence of Slave-making, is repeatedly thwarted. Humanist discourse, whose epistemological machinations provide our conceptual frameworks for thinking political ontology, is diverse and contrary. But for all its diversity and contrariness it is sutured by an implicit rhetorical consensus that violence accrues to the Human body as a result of transgressions, whether real or imagined, within the symbolic order. That is to say, Humanist discourse can only think a subject’s relation to violence as a contingency and not as a matrix that positions the subject. Put another way,

Page 82: openev.debatecoaches.org€¦  · Web view1AC . Observation 1: If I could find the spot where truth echoesI would stand there and whisper memories of my children's future. I would

Humanism has no theory of the Slave because it imagines a subject who has been either alienated in language or alienated from his or her cartographic and temporal capacities. 1 It cannot imagine an object who has been positioned by gratuitous violence and who has no car-

tographic and temporal capacities to lose—a sentient being for whom recognition and incorporation is impossible. In short, political ontology, as imagined through Humanism, can only produce discourse that has as its foundation alienation and exploitation as a grammar of suffering, when what is needed (for the Black, who is always already a Slave) is an ensemble of ontological questions that has as its foundation accumulation and fungibility as a grammar of suffering. 2 A Culture of Politics The violence of the Middle Passage and the Slave estate, 3 technologies of accumulation and fungibility, recompose and reenact their horrors on each succeeding generation of Blacks. This violence is both gratuitous (not contingent on transgressions against the hegemony of civil society) and structural (positioning Blacks ontologically outside of Humanity and civil society). Simultaneously, it renders the ontological status of Humanity (life itself ) wholly dependent on civil society’s repetition compulsion: the frenzied and fragmented machinations through which civil society reenacts gratuitous violence on the Black— that civil society might know itself as the domain of Humans— generation after generation. Again, we need a new language of

abstraction to explain this horror. The explanatory power of Humanist discourse is bankrupt in the face of the Black. It is inadequate and inessential to, as well as parasitic on, the ensemble of questions which the dead but sentient thing, the Black, struggles to articulate in a world of living subjects.

Next OffThe aff’s fear of nuclear war is predicated off a citizenship preparedness that constructs whiteness – educational activities like debate are where these constructions are formed and must be challengedPreston 09 (John, Professor of Education at the University of East London, “Preparing for emergencies: citizenship education, ‘whiteness’ and pedagogies of security”, Citizenship Studies 13:2, April 2009, Routledge)

This is an exercise of state power in a time of extreme crisis and there are obvious parallels between what the state considered in times of nuclear war and recent citizenship policies in the ‘new normalcy’ of the war on terror. However, Muller (2004) considers that, even given no major threat to the state in the US, there has already been a retraction of citizenship rights which has been recolonized by ‘identity management’ as part of the states desire to construct a permanent state of emergency (Agamben 1998). It is tempting in these circumstances to figure an epistemological break between pre and post 9/11 notions of citizenship but there is a danger of reifying this event.

Rather, there are continuities, particularly in terms of what Omi and Winant (1994) call ‘racial projects’ .

Brahander (2004), for example, considers that the ‘war on terror’ has led to new modes of ‘biovalue’ (Brahnder 2004, p. 270) where citizenship is differently allocated on the basis of varieties of biotechnical ‘accumulation strategy’ (Haraway 1997). Brahnder considers that this is not a ‘new’ form of citizenship, but one which ‘renormalizes’ and renews discourses of race and citizenship. These biotechnical accumulation strategies can in turn be considered by adapting Jenson and Phillips’ (1996) use of the term ‘citizenship regime’ to a regime of racial accumulation, where Harris’ (1993) conception of ‘whiteness as property’ is used by the state for security purposes . A citizenship regime interconnects various aspects of citizen formation and accords differential degrees of citizenship rights and representation according to state priorities and group contestation. It is not simply that the accumulation of capital and the discipling of labour require the maintenance of security in both times of preparing for and during crisis but also that modified forms of biological accumulation are interpellated. ‘Race’ is key to understanding these new modes and in particular the centrality of ‘whiteness’ to notions of security, preparedness and threat . The ‘collapsible state’ therefore

Page 83: openev.debatecoaches.org€¦  · Web view1AC . Observation 1: If I could find the spot where truth echoesI would stand there and whisper memories of my children's future. I would

can be seen to refer to a citizenship regime in which biological power, in terms of the respecification and revaluing of ‘whiteness’ is key to understanding new forms of control . These are often pedagogical. In the contexts

discussed in this paper, pedagogy and the idea of the pedagogical (t hat citizens can be taught to mobilize affects , conduct behaviours and operationalize cognitions) is central to citizen formation. Not only are pedagogical notions involved in citizen formation but the notion of the pedagogical as a concept underwrites the formation of citizens. The citizenship regimes discussed are ones in which Bernstein’s (2001) conception of the total pedagogical society is becoming concretized.

Citizens learn to be prepared for an emergency and to recognize threats to security whether that is a missile warning or a suspect package or person. The state seeks to deterritorialize and reterritorialize conceptions of security and threat as part of the regime of citizenship. The use of pedagogical schemes by the state in such regimes is extensive and to limit the extent of ‘ citizenship education’ to formal education offered in schools as part of a discrete curriculum belies the pedagogical character of the state . Activities such as public information campaigns, activities in post-compulsory education and (particularly) family and community education can be considered to be elements of the ‘education’ of citizens . These pedagogical schemes attempt to effect affects, behaviours and cognitions and to ‘call up’ various forms of character. Affect is mobilized through these pedagogies and citizens are instructed not only how they should ‘feel’ about events or security but also how these affects should be used

productively. The mobilization of affect and the way in which this is locked into various bodies in turn produces various embodiments and institutionalizations o f embodiments ( in this context of ‘whiteness’ ) which become fixed on certain actors (Ahmed 2007). It is insufficient to state that individuals were (and are) ‘ afraid’ of nuclear war without considering the social and contextual meaning of that ‘fear’, how it was operationalized and used and the work which it did in terms of fixing and institutionalizing both notions of the other and notions of whiteness.

Global Warming is not caused by humn’s writ large—it is caused by the uneven development engendered by Whiteness. The affirmative naturalizes the coercive racial politics at the heart of warming by universalizing its source and projecting its impacts far into the future. The imperial West started the process of warming, and the American racial state perpetuated it in the quest to export Whiteness. The affirmative only notices warming when it might destroy white bodies, invisibilizing millions of non-whites already killed.

Wynter 07 (Sylvia, Professor Emeritus in Spanish and Romance Languages at Stanford Univeristy, “The Human being as noun? Or being human as praxis? Towards the Autopoietic turn/overturn: A Manifesto,” otl2.wikispaces.com/file/view/The+Autopoetic+Turn.pdf)

For if, as Time magazine reported in January 2007 (Epigraph 2), a U.N. Intergovernmental panel of Natural Scientists, were soon to release

"a smoking-gun report which confirms that human activities are to blame for global warming " (and thereby for climate change), and had therefore predicted "catastrophic disruptions by 2100," by April, the issued Report not

only confirmed the above, but also repeated the major contradiction which the Time account had re-echoed. This contradiction , however, has nothing to do in any way with the rigor, and precision of their natural scientific findings , but rather with the contradiction referred to by Derrida's question in Epigraph 3—i.e., But who, we?

That is, their attribution of the non-natural factors driving global warming and climate change to, generic human activities , and/or to "anthropocentric forcings"; with what is, in effect, this mis- attribution then determining the nature of their policy recommendations to deal with

Page 84: openev.debatecoaches.org€¦  · Web view1AC . Observation 1: If I could find the spot where truth echoesI would stand there and whisper memories of my children's future. I would

the already ongoing reality of global warming and climate change, to be ones couched largely in economic terms. That is, in the terms of our present mode of knowledge production, and its "perceptual categorization system" as elaborated by the disciplines of the Humanities and Social Sciences (or "human sciences") and which are reciprocally enacting of our present sociogenic genre of being human, as that of the West's Man in its second Liberal or bio-humanist reinvented form, as homo oeconomicus; as optimally "virtuous Breadwinner, taxpayer, consumer, and as systemically over-represented as if it, and its behavioral activities were isomorphic with the being of being human, and thereby with activities that would be definable as the human-as-a-species ones.

Consequently, the Report's authors because logically taking such an over-representation as an empirical fact, given that, as highly trained natural scientists whose domains of inquiry are the physical and (purely) biological levels of reality, although their own natural-scientific order of cognition with respect to their appropriate non-human domains of inquiry, is an imperatively self-

correcting and therefore, necessarily, a cognitively open/open-ended one, nevertheless, because in order to be natural scientists , they are therefore necessarily, at the same time, middle class Western or westernized subjects, initiated 15 as such, by means of our present overall education system and its mode of knowledge production t o be the optimal symbolically encoded embodiment of the West's Man, it its second reinvented bio-humanist homo oeconomicus, and therefore bourgeois self-conception, over-represented as if it were isomorphic with the being of being

human, t hey also fall into the trap identified by Derrida in the case of his fellow French philosophers. The trap, that is, of conflating their own existentially experienced (Western-bourgeois or ethno-class) referent "we ," with the "we" of "the horizon of humanity ." This then leading them to attribute the reality of behavioral activities that are genre-specific to the West's Man in its second reinvented concept/self-conception as homo oeconomicus, ones that are therefore as such, as a historically originated ensemble of behavioral activitiesas being ostensibly human activities-

in-general. This, in spite of the fact that they do historicize the origin of the processes that were to lead to their recent natural scientific findings with respect to the reality of the non-naturally caused ongoing acceleration of global warming and climate change, identifying this process as having begun with the [West's] Industrial Revolution from about 1750 onwards. That is, therefore, as a process that can be seen to have been correlatedly concomitant in Great Britain, both with the growing expansion of the largely bourgeois enterprise of factory manufacturing, as well with the first stages of the political and intellectual struggles the British bourgeoisie who were to spearhead the Industrial Revolution, to displace the then ruling group hegemony of the landed aristocracy cum gentry, and to do so, by inter alia, the autopoetic reinvention of the earlier homo politicus/virtuous citizen civic humanist concept of Man, which had served to legitimate the latter's traditionally landed, political, social and economic dominance, in new terms. This beginning with Adam Smith and the Scottish School of the Enlightenment in the generation before the American, French, and Haitian (slave) revolutions, as a reinvention tat was to be effected in now specifically bourgeois terms as homo oeconomicus/and virtuous Breadwinner. 116 That is as the now purely secular genre of being human, which although not to be fully (i.e., politically, intellectually, and economically) institutionalized until the mid-nineteenth century, onwards, when its optimal incarnation came to

be actualized in the British and Western bourgeoisie as the new ruling class, was, from then on, to generate its prototype specific ensemble of new behavioral activities, that were to impel both the Industrial Revolution, as well as the West's second wave of imperial expansion, this based on the colonized incorporation of a large majority of the world's peoples, all coercively homogenized to serve its own redemptive material telos, the telos initiating of global warming and climate change. Consequently, if the Report's authors note that about 1950, a steady process of increasing acceleration of the processes of global warming and climate change, had begun to take place, this was not only to be due to the Soviet Revolution's (from 1917 onwards) forced march towards industrialization (if in its still homo oeconomicus conception, since a march spearheaded by the 116 See the already cited essay by J.G.A. Pocock "symbolic capital," education credentials owning and technically skilled Eastern European bourgeoisie)—as a state-directed form of capitalism, nor indeed by that of Mao's then China, but was to be also due to the fact that in the wake of the range of successful anti-colonial struggles for political independence, which had accelerated in the wake of the Second World War, because the new entrepreneurial and academic elites had already been initiated by the Western educational system in Western terms as homo oeconomicus, they too would see political independence as calling

for industrialized development on the "collective bovarysme "117 model of the Western bourgeoisie. Therefore, with the acceleration of global warming and climate change gaining even more momentum as all began to industrialize on the model of

homo oeconomicus, with the result that by the time of the Panel's issued April 2007 Report the process was now being driven by a now planetarily homogenized /standardized transnational "system of material provisioning or mode of techno-industrial economic production based on the accumulation of capital; as the means of production of ever-increasing

economic growth, defined as "development"; with this calling for a single model of normative behavioral activities, all driven by the now globally (post-colonially and post-the-1989-collapse-of-the-Soviet Union), homogenized desire of "all men (and women) to," realize

themselves/ourselves, in the terms of homo oeconomicus. In the terms, therefore, of "its single (Western-bourgeois or

Page 85: openev.debatecoaches.org€¦  · Web view1AC . Observation 1: If I could find the spot where truth echoesI would stand there and whisper memories of my children's future. I would

ethno-class) understanding" of "man's humanity," over-represented as that of the human; with the well-being and common good of its referent "we"—that, not only of the transnational middle classes but even more optimally, of the corporate multinational business industries and their financial networks, both indispensable to the securing of the Western-bourgeois conception of the common good, within the overall terms of the behavior-regulatory redemptive material telos of ever-increasing economic growth, put forward as the Girardot-type "cure" for the projected Malthusian-Ricardo transumed postulate of a "significant ill" as that, now, ostensibly, of mankind's threatened subordination to [the trope] of Natural Scarcity, this in the reoccupied place of Christianity of its postulate

of that "ill" as that of enslavement to Original Sin."' With the result that the very ensemble of behavioral activities indispensable, on the one hand, to the continued hegemony of the bourgeoisie as a Western and westernized transnational ruling class, is the same ensemble of behaviors that is directly causal of global worming and climate change, as they are, on the other, to the continued dynamic enactment and stable replication of the West's second reinvented concept of Man; this latter in response to the latter's existential imperative of guarding against the entropic disintegration of its genre of being human and fictive nation-state mode of kind. Thereby against the possible bringing to an end, therefore, of the societal order, and autopoetic living Western and westernized macro world system in it bourgeois configuration, which is reciprocally the former's (i.e., its genre of being human, and fictive modes of kind's condition of realization, at a now global level. This, therefore, is the cognitive dilemma, one arising directly from the West's hitherto unresolvable aporia of the secular, that has been precisely captured by Sven Lutticken in a recent essay. Despite, he writes, "the consensus that global warming cannot be ascribed

to normal fluctuations in the earth's temperature... [the] social and political components of this process have been minimized; man-made nature is re-naturalized, the new (un)natural history presented as fate." And with this continuing to be so because (within the terms, I shall add, of our present "single understanding of man's humanity" and

the unresolvable aporia which it continues to enact), "[ t]he truly terrifying notion is not that [ global warming and climate change] is irreversible, but that it actually might be reversible—at the cost of radically changing the economic and social order ... "119The changing , thereby, of the now globally hegemonic biologically absolute answer that we at present give to the question to who we are, a nd of whose biohumanist homo oeconomicus symbolic life/death (i.e., naturally selected/dysselected) code's intentionality of dynamic

enactment and stable replication, our present "economic and social order " is itself the empirical actualization.

In Dreazen 2019, they cite that North Korea having bioweapons is uniquely bad, they literally said in cross ex, that North Korea is lead by a crazy little ass Asian man, this represents the US’s view of Asia as a whole and is a fucking link because…

Security anxieties are manufactured by white fears of emasculation – particular true in the context of AsiaPark 12 (Hijin Park – Ph.d Associate Professor Brock University, Canada, “Interracial Violence, Western Racialized Masculinities, and the Geopolitics of Violence Against Women,” Sage Publications, 2012 page 499-500, 492-509)

Liz Philipose (2007), using the work of Sironi and Branche, stresses that torture is not about destroying an individual’s defences; it is violence intended to defeat a group and nation. Torture is not meant to make people reveal themselves and their knowledge; it is meant to keep them quiet, and render them isolated and humiliated (Philipose, 2007: 1064–1065). The humiliation and silencing is directed towards the group(s) the individual represents, rather than the individual herself. Rather than being seen as a man’s desire to dominate a particular group of women because of Orientalist cultural

Page 86: openev.debatecoaches.org€¦  · Web view1AC . Observation 1: If I could find the spot where truth echoesI would stand there and whisper memories of my children's future. I would

misrepresentations, acts of violence against Asian women become ‘ performative means by which relationships of structural inequality are ensured’ (Ahmed, 2001: 348–349). In other words – as violence and desire are racialized, gendered, classed and sexualized – targeted stranger violence against Asian women in the west is part of how the boundaries of race, gender, class and sexuality are constituted and reinforced (see Agathangelou and Ling, 2004: 536). The violence is part of how white hegemonic masculinity is supported in white settler states, since heterosexual masculinity, racial anxieties and national anxieties are inseparable. Philipose (2007: 1067) states it well: ‘racialized anxieties are national anxieties, and if we take the nation as the embodiment of masculine values of virility and potency ... , these are also anxieties about the state of masculinity in the twenty-first-century ... ’. Asian women play a starring role in assisting disaffected (white heterosexual) men assert their masculinity in a women’s movement, post-civil rights, and ‘Asia rising’ era. Hobbs, Na Pattalung and Chandler (2011) contend that western men travel to Thailand as sex tourists to (re)construct, (re)affirm and (re)establish their masculinity (2011: 89–90). Less able to enjoy their masculinity at home amidst the formal legitimation of rights-seeking minority groups (albeit tenuous and limited), western men travel to Asia to be men with Othered women who are structurally and ideologically exploitable. This narrative of decline that haunts white western masculinity is prevalent in predatory heterosexuality. In his provocative essay on how the logics of hunting by North American white men mirror the logics of predatory heterosexuality, Brian Luke (1998) reveals that men who rape often report feeling threatened by their victim, and define the rape as permissible revenge and the restoration of lost control (1998: 633). Importantly, white men’s violence against Asian women restores lost control by attacking women meant to be impregnated, protected and provided for by Asian men and nations (irrespective of the woman’s citizenship and history in the west). This emasculinization of Asian men and nations asserts a global , imperial colour line as well as a national, colonial colour line (see Philipose, 2009). In 1910, W.E.B. DuBois announced the emergence of a transnational whiteness. DuBois stated, ‘the world, in a sudden emotional conversion, has discovered that it is white, and, by that token, wonderful’ (quoted in Lake and Reynolds, 2008: 1–2). In the face of the rise of the ‘tide of colour’ in a neo-colonial era , and the mass migration of colour to the new world, white elite men in Canada, the United States, Australia, Britain and South Africa supported each others’ efforts to solidify the dominance of a newly constituted whiteness (Lake and Reynolds, 2008: 184–209). At the turn of the twentyfirst century there is a renewed reassertion of transnational whiteness through constructs of western civilization and the clash of civilizations, democracy and human rights. Frequently constituted through discourses of war on terrorism and Islamophobia, transnational whiteness is also affirmed through anxieties surrounding the economic and political rise of parts of Asia. The economic rise of Japan in the 1990s and the more recent rise of China and India have destabilized what is central to white power: being exclusively more human and less deficient than other races (see Philipose, 2007: 1051–1052). Understood within the logic of Orientalism/War, Asian difference can threaten white supremacy in neighbourhoods, workplaces, schools and boardrooms at home and globally. This threat is represented not only through military stealth and vociferousness, but also through foreign capital and civilizational might.

Page 87: openev.debatecoaches.org€¦  · Web view1AC . Observation 1: If I could find the spot where truth echoesI would stand there and whisper memories of my children's future. I would

The Alternative is to reject the 1AC in favor of a teleological suspension of whiteness – our alternative is a process of refusal. Refusing to allow the production of whiteness. Headley 4 (Clevis Headley, “Delegitimizing the Normativity of ‘Whiteness’”, from What White Looks Like: African-American Philosophers on the Whiteness Question, edited by George Yancy)

I want to conclude this essay by briefly considering the future of whiteness. Some thinkers have called for a deconstruction of whiteness, claiming that its contingent historical status will be exposed and its reified status eradicated. Of course, revealing the arbitrary ontological and axiological status of whiteness and its relational dependence upon the other concepts does not automatically translate into an undermining of whiteness. That concepts survive attempted semantic executions is uncontested; we cannot eliminate a concept from our language simply by showing it to be the product of an economy of linguistic difference. There is also the abolitionist position of whiteness and white identity.

Abolitionists confidently call for this abolition on the grounds that whiteness is best defined negatively, that is, by what it is not. Instead of viewing whiteness as designating independent content, it is seen as designating lack and, accordingly, as fake. Because of the void at the heart of whiteness, whites fill in the blanks by devouring the Other . This consuming of the Other takes place because white identity is empty, void of any

content aside from its mediation as difference from non-whiteness. As Roediger writes: “It is not simply that whiteness is oppressive and false. . . . It is the empty and terrifying attempt to build an identity based on what one isn’t and on whom one can hold back.” Although I agree with the critical thrust of the abolitionist program, I must express some skepticism about its ultimate impact. The idea of the abolition of whiteness requires that one clearly state the precise nature of this abolition program. Is it simply going to be a matter of the legal abolition of whiteness? Certainly, one can respond by stating that the civil rights revolution was, in part, intended to eliminate whiteness through legal strategies. Of course, what

was accomplished was the promotion of a procedural formalist notion of equality that serves as a camouflage for whiteness. But with regard to the technical notion of abolition, we still need to know how the voluntary abolition of racial identity is possible beyond the courageous acts of a few isolated individuals, particularly since white identity is a majoritarian and is reinforced by tremendous economic, cultural, social, legal, and political power. From an Africana philosophical perspective, I argue for the teleological suspension of whiteness, realizing that

whiteness must be transcended but not by rational argument. The point here is not to license irrationalism, but,

rather, to underscore the fact that rational argumentation proves blunt in the face of centuries-old social privilege. Here, the notion of the suspension of whiteness. Let us consider a suggestive Africana appropriation of the Kierkegaardian existential dialect. The aesthetic stage of whiteness represents the stage when whiteness and its tremendous benefits were overtly constituted. Of course, this situation generated opposition in that those excluded from the privileges of whiteness openly challenged the rightness of

whiteness and exposed the immorality of white supremacy. Overt and blatant rejoicing in the privilege of whiteness could not escape challenge. The ethical stage of whiteness is similarly dominated with a concern to do the morally right thing. This concern takes the form of bold appeals to objectivity, formal equality of

opportunity, and procedural notions of justice. In this stage, there is a strong appeal to the moral worth of each individual and an equally confident emphasis on the notion of rewarding individuals strictly on merit. In conjunction with a forceful advocacy of color blindness, policies that recognize race, such as affirmative action, are considered to be immoral because they favor groups and not individuals.

Consequently, race-conscious policies are denounced as flagrantly violating both moral and constitutional principles of fairness. We are quickly approaching a situation in which appealing to formal principles of equality and opportunity and to universal principles of justice cannot effectively remedy the tremendous differential in power between blacks and whites . As this realization becomes more and more evident, there will be a need to move on to a third stage.

Unlike Kierkergaard’s third stage, the religious stage made possible by an unconditional act of faith, the third stage of whiteness will more closely resemble his notion of the teleological suspension of the ethical and the inability to rationally inscribe this suspension in a universal discourse. Not being concerned with ultimate meaning or with religious

affirmation, the third stage of whiteness will not be a deconstruction or abolition of whiteness but, rather, a teleological suspension of whiteness. Whiteness cannot be dismantled through rational or

analytical means. Its suspension must come in the form of a continuously affirmed refusal to prolong the ontological and existential project of whiteness. The project of whiteness must be suspended for

Page 88: openev.debatecoaches.org€¦  · Web view1AC . Observation 1: If I could find the spot where truth echoesI would stand there and whisper memories of my children's future. I would

the greater good of human liberation beyond whiteness. The project of whiteness has proven too costly for human

existence. The existential price is simply too costly for those who are forced to involuntarily participate in this project. Hence, teleologically suspending whiteness is a solution, a counter-project that cannot be rationally stated. It defies neutral conceptualization because there is no direct and rationally persuasive way of linguistically describing the urgency of this cause. The reason why the question of the suspension of whiteness cannot be framed in the language of our legal and political system is because our language itself is infected with the project of whiteness . After all, what would it mean to argue in favor of renouncing whiteness and its benefits by using a discourse imbued with the categories that perpetuate whiteness? To the extent that whiteness, understood as white supremacy, is a global system of white racial domination similar to patriarchy, Africana philosophers can call attention to

racial bias in language in the same manner that feminist philosophers have called attention to gender bias in language. For example, the alleged neutral and universal notions of political discourse, which should function neutrally, promote racial outcomes that benefit whites. Lipsitz claims that “the language of liberal individualism serves as a cover for coordinated collective group interests.” Robyn Wiegman, calling attention to Cheryl Harris’s distinction between corrective justices, which seeks “compensation for discrete and ‘finished’ harm done to minority groups members or their ancestors,” and distributive justice, which is “the claim an individual or group would have been awarded under fair conditions,” affirms Harris’s claim that “the goals of affirmative action—to address the harms done to those people minoritized by racial . . . oppression—are undermined when corrective justice is the interpretive frame because not only is the harm assumed to be finished but the practices through which harm has been done are individualized, confined to the one who perpetrated it and the one who endured it. In this context, whites can claim to be innocent and

therefore in need of counterlegislative protection because they have not individually perpetuated harm.” Transcending whiteness, from an Africana philosophical perspective , must of necessity do more than expose the secret career of whiteness in maintaining structures of racial privilege. Such an analysis of whiteness should also serve as an opportunity to acknowledge the affirmative possibilities of conceiving alternative social modes of being. In other words, it should underscore the realization that things can be otherwise. Moving beyond whiteness requires a radically new concept of the human, a new metaphoricity of humanity .

Page 89: openev.debatecoaches.org€¦  · Web view1AC . Observation 1: If I could find the spot where truth echoesI would stand there and whisper memories of my children's future. I would

BDS CASEThe aff’s humanitarianism is the twenty-first century white man’s burden – the US uses it to justify taking down anyone it deems unworthy of sovereignty and to install the white supremacist order worldwideBaraka 2013 (Ajamu, Associate Fellow @ the Institute of Policy Studies, a long time human rights activist, write and veteran of the Black Liberation, anti-war, anti-apartheid and Central American solidarity Movement in the United States, "Syria and the Sham of "Humanitarian Intervention", June 4 https://www.commondreams.org/view/2013/06/04)

I believe part of their success has been due to the fact that they have used the concept of humanitarian intervention as one of their main tools.

In my article, I made the argument that humanitarian intervention, along with the concept of the “right to protect” (R2P) has developed into the most effective ideological weapon the liberal human rights community provided Western imperialism since the fall of the Soviet State. Humanitarian intervention has proven to be an even

more valuable propaganda tool than the “war on terror,” because as the situation in Libya and now Syria has demonstrated, it provides a moral justification for imperialist intervention that can also accommodate the presence of the same “terrorist” forces the

U.S. pretends to be opposed to. And of course, in the eyes of the U.S. government, tyrannical and dictatorial

governments that need to be deposed are only those that present an obstacle to the realization of U.S. geo/political interests—never those paragons of freedom and morality like Saudi Arabia and Israel. As I said in my earlier article:Humanitarian intervention provided the U.S. State the perfect ideological cover and internal rationalization to continue as the global “gendarme” of the capitalist order. By providing the human rights rationale for the assertion that the “international community” had a moral

and legal responsibility to protect a threatened people, mainstream human rights activists effectuated a shift in the

discourse on international human rights that moved the R2P assertion from a contested legal and moral augment to a common-sense assumption. And because of their limited perspective, it did not occur to any of these theoreticians that what they propagated was a thinly updated version of the “white man’s burden.” The NATO intervention in Bosnia and Kosovo, the assault on Iraq to “save” the Iraqi people from Saddam Hussein, and most recently the NATO attack on Libya that brought to power a rag-tag assortment of anti-African racists, have solidified the idea among many in the U.S. that humanitarian intervention to protect

human rights through aggressive war is justifiable. The consequence of this for U.S. policy makers and for the likely targets of U.S.

aggression in the global South is that if properly framed, war could be moved back to the center of strategic options without much fear of a backlash from the American people—a development especially important for a declining power that appears to have concluded that it will use military means to attempt to maintain its global empire. The propagandists of the U.S. war strategy have been spectacularly successful in inculcating this shift in consciousness in the general population and the self-muting

of the anti-war and anti-imperialist movements in the West, with the exception of a few organizations. The assertion of the right to unilaterally attack any State that it deems unfit for sovereignty is not a new articulation of White supremacist, imperialist ideology but in this current period where there are few constraints on the global exercise of “White power,” the internalization of this position by the European and U.S. publics, irrespective of ethnicity or race, has made the world a much more dangerous place for Black and Brown people: 50,000 killed in Libya, 80,000 in Syria, 1,000,000 in Iraq,

and 30,000 in Afghanistan. The normalization of war as a contemporary expression of the West’s responsibility to bring liberal democracy and capitalist freedom to the non-White hordes, and the fact that

most of the people being killed in the process of “being saved” by the West are non-European, is a graphic confirmation of the White supremacist assumptions of humanitarian intervention.

Page 90: openev.debatecoaches.org€¦  · Web view1AC . Observation 1: If I could find the spot where truth echoesI would stand there and whisper memories of my children's future. I would

Charity Cannibalism DA: The West sustains itself through the cannibalization of the misery of those in the Global South, turning it into a field for the mass production of violence until the point of the west’s total collapseBaudrillard 94 (Jean Baudrillard, greatest living philosopher, 1994, The Illusion of the End, pg. 66-70)

The end of history, being itself a catastrophe, can only be fuelled by catastrophe. Managing the end therefore becomes synonymous with the

management of catastrophe. And, quite specifically, of that catastrophe which is the slow extermination of the rest of the world. We have long denounced the capitalistic, economic exploitation of the poverty of the 'other half of the world'

[l'autre monde]. We must today denounce the moral and sentimental exploitation of that poverty - charity cannibalism being worse than oppressive violence. The extraction and humanitarian reprocessing of a destitution which has become the equivalent of oil deposits and gold mines. The extortion of the spectacle of poverty and, at the same time, of our charitable condescension: a worldwide appreciated surplus of fine sentiments and bad conscience. We

should, in fact, see this not as the extraction of raw materials, but as a waste-reprocessing enterprise. Their destitution and our bad conscience are, in effect, all part of the waste-products of history - the main thing is to recycle them to produce a new energy source. We have here an escalation in the psychological balance of terror. World capitalist oppression is now merely the vehicle and alibi for this other, much more ferocious, form of moral predation. One might almost say, contrary to the Marxist analysis, that material exploitation is only there to extract that spiritual raw material that is the misery of peoples, which serves as psychological nourishment for the rich countries and media nourishment for our daily lives. The 'Fourth World' (we are no longer dealing with a 'developing' Third World) is once again beleaguered, this time as a catastrophe-bearing stratum. The West is whitewashed in the reprocessing

of the rest of the world as waste and residue. And the white world repents and seeks absolution - it, too, the waste-product of its own history. The South is a natural producer of raw materials, the latest of which is catastrophe. The North, for its part, specializes in the reprocessing of raw materials and hence also in the reprocessing of catastrophe. Bloodsucking protection, humanitarian interference, Medecins sans frontieres, international solidarity, etc. The last phase of colonialism: the New Sentimental Order is merely the latest form of the New World Order. Other people's destitution becomes our adventure playground. Thus, the humanitarian offensive aimed at the Kurds - a show of repentance on the part of the Western powers after allowing Saddam Hussein to crush them - is in reality merely the second phase of the war, a phase in which charitable intervention finishes off the work of extermination. We are the consumers of the ever delightful spectacle of poverty and catastrophe, and of the moving spectacle of our own efforts to alleviate it (which, in fact, merely function to secure the conditions of reproduction of the catastrophe market); there, at least, in the order of moral profits, the Marxist analysis is wholly applicable: we see to it that extreme poverty is reproduced as a symbolic deposit, as a fuel essential to the moral and sentimental equilibrium of the West. In our defence, it might be said that this extreme poverty was largely of our own making and it is therefore normal that we should profit by it. There can be no finer proof that the distress of the rest of the world is at the root of Western power and that the spectacle of that distress is its crowning glory than the inauguration, on the roof of the Arche de la Defense, with a sumptuous buffet laid on by the Fondation des Droits de l'homme, of an exhibition of the finest photos of world poverty. Should we be surprised that spaces are set aside in the Arche d'Alliance* for universal suffering hallowed by caviar and champagne? Just as the economic crisis of the West will not be complete so

long as it can still exploit the resources of the rest of the world, so the symbolic crisis will be complete only when it is no longer able to feed on the other half's human and natural catastrophes (Eastern Europe, the Gulf, the Kurds, Bangladesh, etc.). We need this drug, which serves us as an aphrodisiac and hallucinogen. And the poor countries are the best suppliers - as, indeed, they are of other drugs. We provide them, through our media, with the means to exploit this paradoxical resource, just as we give them the means to exhaust their natural resources with our technologies. Our whole culture lives off this catastrophic

Page 91: openev.debatecoaches.org€¦  · Web view1AC . Observation 1: If I could find the spot where truth echoesI would stand there and whisper memories of my children's future. I would

cannibalism, relayed in cynical mode by the news media, and carried forward in moral mode by our humanitarian aid, which is a way of encouraging it and ensuring its continuity, just as economic aid is a strategy for perpetuating under-development. Up to now, the financial sacrifice has been compensated a hundredfold by the

moral gain; But when the catastrophe market itself reaches crisis point, in accordance with the implacable logic of the market, when distress becomes scarce or the marginal returns on it fall from overexploitation, when we run out of disasters from elsewhere or when they can no longer be traded like coffee or other commodities, the West will be forced to produce its own catastrophe for itself, in order to meet its need for spectacle and that voracious appetite for symbols which characterizes it even more than its voracious appetite for food. It will reach the point where it devours itself. When we have finished sucking out the destiny of others, we shall have to invent one for ourselves. The Great Crash, the symbolic crash, will come in the end from us Westerners, but only when we are no longer able to feed on the hallucinogenic misery which comes to us from the other half of the world. Yet they do not seem keen to give up their monopoly. The Middle East, Bangladesh, black Africa and Latin America are really going flat out in the distress and catastrophe stakes, and thus in providing symbolic nourishment for the rich world. They might be said to be overdoing it: heaping earthquakes, floods, famines and ecological disasters one upon another, and finding the means to massacre each other most of the time. The 'disaster show' goes on without any let-up and our sacrificial debt to them far exceeds their economic debt. The misery with which they generously overwhelm us is something we

shall never be able to repay. The sacrifices we offer in return are laughable (a tornado or two, a few tiny holocausts on the roads, the odd financial sacrifice) and, moreover, by some infernal logic, these work out as much greater gains for us, whereas our kindnesses have merely added to the natural catastrophes another one immeasurably worse: the demographic catastrophe, a veritable epidemic which we deplore each day in pictures. In short, there is such distortion between North and South, to the symbolic advantage of the South (a hundred thousand Iraqi dead against casualties numbered in tens on our side: in every case we are the losers), that one day everything will break down. One day, the West will break down if we are not soon washed clean of this shame, if an international congress of the poor countries does not very quickly decide to share out this symbolic privilege of misery and catastrophe. It is of course normal, since we refuse to allow the spread of nuclear weapons, that they should refuse to

allow the spread of the catastrophe weapon. But it is not right that they should exert that monopoly indefinitely. In any case, the under-developed are only so by comparison with the Western system and its presumed success. In the light of its assumed failure, they are not under-developed at all. They are only so in terms of a dominant evolutionism which has always been the worst of colonial ideologies. The argument here is that there is a line of objective progress and everyone is supposed to pass through its various stages (we find the same eyewash with regard to the evolution of

species and in that evolutionism which unilaterally sanctions the superiority of the human race). In the light of current upheavals, which put an end to any idea of history as a linear process, there are no longer either developed or under-developed peoples. Thus, to encourage hope of evolution - albeit by revolution - among the poor and to doom them, in keeping with the objective illusion of progress, to technological salvation is a criminal absurdity. In actual fact, it is their good fortune to be able to escape from evolution just at the point when we no longer know where it is leading. In any case, a majority of these peoples, including those of Eastern Europe, do not seem keen to enter this evolutionist modernity, and their weight in the balance is certainly no small factor in the West's repudiation of its own history, of its own utopias and its own modernity. It might be said that the routes of violence, historical or otherwise, are ·being turned around and that the viruses now pass from South to North, there being every chance that, five hundred years after America was conquered, 1992 and the end of the century will mark the comeback of the defeated and the sudden reversal of that modernity. The sense of pride is no longer on the side of wealth but of poverty, of those who - fortunately for them - have nothing to repent, and may indeed glory in being privileged in terms of catastrophes. Admittedly, this is a privilege they could hardly renounce, even if they wished to, but natural disasters merely reinforce the sense of guilt felt towards them by the wealthy - by those whom God visibly scorns since he no longer even strikes them down. One day it will be the Whites themselves who will give up their whiteness. It is a good bet that repentance will reach its highest pitch with the five-hundredth anniversary of the conquest of the Americas. We are going to have to lift the curse of the defeated - but symbolically victorious - peoples, which is insinuating itself five hundred years later, by way of repentance, into the heart of the white race.

Page 92: openev.debatecoaches.org€¦  · Web view1AC . Observation 1: If I could find the spot where truth echoesI would stand there and whisper memories of my children's future. I would

Their representations of suffering in Puar 2017, place white bodies as the arbiters of racialized suffering who can deploy their compassion as they see fit

- it’s not distance but racialization that affects whose deaths we see as valuable/mattering (ie the aff prioritizes american lives)- we “look” at other lives (colonised) lives from a supposedly “neutral” viewpoint, but what the aff calls neutral isn’t – it’s the white

gaze, always out of the frame of reference, viewed as neutral- reps of suffering essentialize it, creating it (discursively) as inevitable/natural/a property of those racialized bodies- makes bodies perform their suffering (black bodies on display, etc) as “evidence” – this creates the white viewing position of the

affirmative, it’s always based on racialized images- comes to define who can act/choose to stop that suffering -> savior-type-complex

Szorenyi 09 (Anna, Lecturer in Gender Studies and Social Analysis at the University of Adelaide, “Distanced suffering: photographed suffering and the construction of white in/vulnerability”, Social Semiotics 19:2, June 2009, Routledge)

l emphasise this point because many discussions of the problem of distant suffering take a literal interpretation of "distance" for granted. For example, Terence Wright describes the workings of the mass media`s presentation of suffering: “There is an inverse relationship between numbers and distance. To gain news coverage, not only are larger numbers of victims required as one moves further away from home, but the sensitivity in representing the plight of others diminishes as geographical and cultural distance

increases.” (2004, 99) Thus it has been said of the US media that "One dead fireman in Brooklyn is worth five English bobbies, who are worth 50 Arabs, who are worth 500 Africans " (Mueller 1999, 22, quoted in Wright 2004,

99). According to this theory, differentials in the ways in which suffering and death are represented are often taken to be organised around geographical distance ; those who are near and like get more attention and their stories are told more "sensitively" than those who suffer or die on the other side of the world . But this posited relationship between physical proximity and media visibility (and between physical

proximity and cultural familiarity) is missing something . The routes along which images of suffering are circulated on this map are far

more complex than simple linear measures of degrees of proximity to a centre. This should be obvious from an Australian perspective, where “ Australia" followed by “America, England, the Middle East and Africa" is certainly not a list of increasing geographical distances (and Asia does not even figure in the list). Rather , this list could be described being organised according to what Goldie Osuri and Subhabrata Bobby Bannerjee have called " white diasporas ", a loyalty expressed at the level of the nation-state between those who feel they share a common, “ Anglo" heritage (Osuri and Bannerjee 2004, 151). "Distance" thus seems often to work a way of obliquely referring not

simply to mileage, but to the structure of the world instituted and maintained by colonialism and its accompanying racial discourses. Such racial discourses, while inevitably manifesting slightly differently in different locations, share across the white diasporas an understanding of "white" culture civilised and superior, constructed in opposition to "primitive" racialised others. As theorists of whiteness have pointed out, a recurrent set of looking relations accompanies such shared constructions . Observations of the colonised proliferate if from a neutral observer`s viewpoint, a viewpoint that itself never appears within the frame of vision, taking for granted its separation from the field of view and its right to look, categorise, and order . Stuart Hall (1990) calls this the "white eye". The white eye relentlessly produces essentialist and stereotypical views of racialised others, and also relentlessly reproduces its own status master of the field of view, though the world were its territory. This Hall points out, "the history of slavery and conquest, written, seen, drawn and photographed by The Winners" (1990, 14). As the position from which other stereotypes have been

constructed and elaborated, the stereotype of whiteness itself is left outside the frame. This works to disavow the specificity of whiteness, positioning it the unmarked "norm" against which other are measured (Dyer

1997), and disavowing the way its construction depends on images of the "other" (Mercer 1991). In the realm of "concerned photography", such racialised-looking relations are mapped onto essentialised relations to suffering. The result is neatly summarised by Arthur and Joan Kleinman, who draw attention to the imperialist structure of the viewing position offered by famine pictures: “One message that comes across from viewing suffering from a distance is that for all the havoc in Western society, we are somehow better than this African society. We gain in moral status and some of our organizations gain financially and politically, while those whom we represent, or appropriate, remain where they are, moribund, surrounded by vultures. This "consumption" of suffering in an era of so-called "disordered capitalism" is not so very different from the late nineteenth-century view that the savage barbarism in pagan lands justified the valuing of our own civilization at a higher level of development a view that authorized colonial exploitation.” (Kleinman and Kleinman 1997, 8) As Kleinman

Page 93: openev.debatecoaches.org€¦  · Web view1AC . Observation 1: If I could find the spot where truth echoesI would stand there and whisper memories of my children's future. I would

and Kleinman imply, the mass reproduction of variations on this theme of the suffering "other" has a strongly essentialising effect , working to create an impression that suffering in the "third world" is inevitable, expected, and somehow intrinsic to life in certain regions . As Sontag puts it in considering the photographic record of famine and genocide in postcolonial Africa, "These sights carry a double message. They show a suffering that is outrageous, unjust and should be repaired. They confirm that this is the sort of thing that happens in that place" (2003, 64). Allan Feldman`s work on the "trauma aesthetic"

(2004) is instructive here. Feldman notes that there is an extensive historical tradition of putting the bodies of black people on display in order to invoke compassion , which can be traced in the United States to the display of scars on the bodies of slaves during the Abolitionist era, and is currently manifested in the mass circulation of images and stories of trauma as evidence of human rights violations. Helpful as they may be in human

rights campaigns, to the extent that they incorporate individual suffering into regimes of truth and evidence, these commodifications of trauma also participate in a discursive tradition in which certain bodies are made to perform as "evidence" of their own suffering . The problem with this, as Feldman notes, is that it installs a particular

arrangement of visual perspectives, in which "other" bodies become hypervisualised, while viewers are offered a mobile, but always hierarchically superior, viewing position . Feldman reminds us that Edward Said has named this subject position as one of "flexible positional superiority", a defining feature of imperialist subjectivity

(Feldman 2004, 186; Said 1995, 7). Repetition of thi s structure comes to discursively define black bodies as those who suffer, and invisible white bodies as those who watch , and as those who own, and choose whether or not to deploy, compassion.

Awareness raising of the aff does nothingAl-Jaza'iri and Grant 10 (The authors of this article share a growing concern for the direction the Palestinian solidarity movement in Canada has been taking. Isa Al-Jaza'iri's mother was kicked out of her home in Haifa at gunpoint in 1948, along with her entire family. Alex Grant's grandfather wore his Star of David patch at Mauthausen concentration camp. However, they believe their respective backgrounds, although they have a bearing on this issue, are not the key issue inasmuch as this is an issue that has to be taken up by the world working class. They defend the position that only a working class perspective can end the oppression of the Palestinian people while providing a future for the Israeli population. Isa Al-Jaza'iri and Alex Grant, 19 January 2010, https://www.marxist.com/against-blanket-boycott-israel-working-class-solution.htm, accessed 3/19/19, jmg)

A new boycott campaign has been gaining an echo within the Palestinian solidarity movement. This campaign is known as “Boycott, Divestment and Sanctions”, BDS for short. Sometimes it is referred to as the campaign against Israeli Apartheid. We truly believe that the supporters of this campaign are acting from a genuine desire to end the suffering of the Palestinian people. However, we have to frankly say that we do not believe that the methods of this campaign can defeat Israeli imperialism. What is notable about this campaign is that it ignores the question of class in both Israel and Palestine. We believe that only a working class approach can put an end to Israeli imperialism. What is BDS and who

supports it? The campaign for Boycott, Divestment and Sanctions was launched on 9th July, 2005 by a "unified call" drawing together

more than one hundred Palestinian organizations. Amongst these are the Ramallah Chamber of Commerce, some PLO popular

committees, the Palestinian General Federation of Trade Unions, the General Union of Palestinian Teachers, the Federation of Unions of

Palestinian University Professors and Employees, the Palestinian Engineers’ Association, the Palestinian Lawyers' Association, and an alphabet soup of NGOs and "civil society" groups in the occupied territories and Israel itself. These groups have elected a "Boycott National Committee", which has been entrusted to lead the campaign as well as manage the official website, BDSmovement.net. The campaign has been able to gain the support of some trade unions in Canada, such as the Canadian Union of Postal

Workers and the Fédération Autonome du Collégial teachers' union in Quebec. It also has an extensive list of supporters worldwide. Its demands on the Israeli state are laid out in the original call: "1. Ending its occupation and colonization of all Arab

Page 94: openev.debatecoaches.org€¦  · Web view1AC . Observation 1: If I could find the spot where truth echoesI would stand there and whisper memories of my children's future. I would

lands and dismantling the Wall; 2. Recognizing the fundamental rights of the Arab-Palestinian citizens of Israel to full equality;

and 3. Respecting, protecting, and promoting the rights of Palestinian refugees to return to their homes and properties as stipulated in UN resolution 194." The BDS campaign is an international campaign for consumer and academic boycotts, pressure tactics to push corporations and universities to take investments out of Israeli companies and companies with major Israeli interests, and a broader call for economic sanctions against the Israeli state. It is also accompanied by "cultural" and "sports"

boycotts. Furthermore, even though some less informed supporters may claim a more targeted approach, the consumer boycott is quite

clearly intended to be a blanket boycott of all Israeli goods, as pointed out on the official BDSmovement.net website: “Raise awareness among consumers about the consequences of buying Israeli goods and services. Pressurize supermarkets and shops into removing goods with barcode 729 - denoting Israeli origin - from their shelves. Encourage companies who make use of Israeli technology and components to find alternatives and join the boycott. Focus the attention of the world on Israeli occupation and apartheid and expose those who bankroll the Israeli regime; and to foster an environment in which it is unacceptable to promote Israeli policies.” Many young activists are supporting this campaign because of an honest feeling that something must be done. In

discussions with BDS activists, when the point has been raised that this campaign does nothing to weaken Israeli imperialism and in fact pushes Israeli workers and youth into the arms of imperialists , the response has been "well, at least the campaign has raised awareness". We are prepared to concede this point; the campaign has

received much media attention. However, not a single Palestinian family has ever been saved from having their house knocked down thanks to a letter to the Toronto Star or the Montreal Gazette . People were also aware of the Iraq war. 50 million people demonstrated worldwide in March 2003. But this awareness did not stop US imperialism. Neither will it stop Israeli imperialism. What we need is an approach that can defeat imperialism.

Intra state anti-black violenceA) BDS will mirror South African Anti-Apartheid tactics

Wolfe 14 (Sherry Wolf is the author of Sexuality and Socialism: History, Politics and Theory of Gay Liberation (Haymarket Books, 2009). What's behind the rise of BDS? By Sherry Wolf Issue #93: Features, International Socialist Review, https://isreview.org/issue/93/whats-behind-rise-bds, accessed 3/18/19, jmg)

The BDS movement unabashedly has taken a page from the successful playbook of the South African anti-apartheid movement. Like Israeli Jews today, the vast majority of South Africa’s white population opposed a democratic state and clung to their domination over the Black population until the bitter end. But the domestic resistance of Black South Africans combined with the pressure from the international anti-apartheid movement led to a total loss of legitimacy of the apartheid regime. Like today, some argued that the boycott would harm the very people the movement aimed to help. Yet then as now, the indigenous population was both the initiator and defender of the call for boycott, divestment, and sanctions. Today, some of the same figures who led the South African movement are speaking out against Israel’s apartheid and advocating BDS. “The same issues of inequality and injustice today motivate the divestment movement trying to end Israel’s decades-long occupation of Palestinian territory and the unfair and prejudicial treatment of the Palestinian people by the Israeli government ruling over them,”27argues South Africa’s Archbishop Desmond Tutu. South African anti-apartheid activists Suraya Dado and Muhammed Desai insist a debt of gratitude is owed: “It is our duty as South Africans to stand in solidarity with the Palestinian people.”28 The fake shanty towns that college students set up on their campuses to portray the segregation and misery of life in South Africa’s Black slums has an echo in the mock checkpoints BDS activists set up to dramatize daily conditions for Palestinians trying to travel from home to work. The days-long teach-ins of the South African anti-apartheid movement have their match in the annual Israeli Apartheid Week, which celebrated its tenth year this winter. The phony apartheid passbooks identifying people by their race that students made in the eighties have their counterpart in the faux eviction notices passed under dormroom doors to educate students about what the IDF does before violently driving Palestinians from their homes. The examples are growing as an older generation shares its experiences with their young peers in the movement, and workshops on lessons from the South African anti-apartheid movement have become de rigueur at BDS conferences.

Page 95: openev.debatecoaches.org€¦  · Web view1AC . Observation 1: If I could find the spot where truth echoesI would stand there and whisper memories of my children's future. I would

B) This Hides internalized anti-black violence – there are black bodies on both sides of the Palestinian Israeli and South Africa conflicts – their historical understanding promoted in the 1ac and among advocates mystifies anti-black violence

Brady 13 (Nicholas Brady is an activist-scholar from Baltimore, Maryland. He is a former debater and currently is a head coach for the James Baldwin Debate Society, the only collegiate debate team housed in an African-American Studies department. He was also a recent graduate of Johns Hopkins with a bachelor’s degree in Philosophy and currently a doctoral student at the University of California-Irvine Culture and Theory program., “The Void Speaks Back: Black Suffering as the Unthought of the American Studies Association’s Academic Boycott of Israel”, https://outofnowhereblog.wordpress.com/2013/12/23/the-void-speaks-back-black-suffering-as-the-unthought-of-the-american-studies-associations-academic-boycott-of-israel/, accessed 3/18/19, jmg)

To speak of blackness as “unthought” is not to speak of ignorance then, nor is it even to mean unconsidered. We can say very definitively that

black suffering is not simply unknown, but actually remains a central metaphor for how people understand their

solidarity against Israel. People’s statements of support are full of similes: “this is like the boycott of South Africa…”

“Israeli occupation is like segregation…” etc. Black suffering becomes the gold standard of evaluating other people’s suffering and the foundation for our moral outrage and political maneuvering. What is tragic here is

that the metaphor was unnecessary, for the metaphor elides the existence of black bodies on the ground on both sides of the wall catching hell from both sides of this conflict . Black flesh exists as the unknown term in the Israel/Palestine conflict, the void off the map of political contestation.

Page 96: openev.debatecoaches.org€¦  · Web view1AC . Observation 1: If I could find the spot where truth echoesI would stand there and whisper memories of my children's future. I would

BDS The 1AC relies on a futural hope of a “not-yet-realized” future in which they can fantasize about contingent solutions that never come. This hope creates a cruel optimism for black folk that forces them to invest in the pursuit of our own death because it crowds out all non-politically recognizable alternatives. This model for politics only re-entrenches the anti-black world and is an independent reason to reject the aff. Warren 15 (Calvin L., Black Nihilism and the Politics of Hope; Surce: CR: The New Centennial Review, Vol. 15, No. 1, Derrida and French Hegelianism (Spring 2015), XMT, pp. 215-248 Published by: Michigan State University Press Stable URL: http://www.jstor.org/stable/10.14321/crnewcentrevi.15.1.0215 . Accessed: 30/03/2015)

The politics of hope , then, constitutes what Lauren Berlant would call “ cruel optimism” for blacks (Berlant 2011). It bundles certain promises about redress, equality, freedom, justice, and progress into a political object that always lies beyond reach. The objective of the Political is to keep blacks in a relation to this political object—in an unending pursuit of it. This pursuit, however, is detrimental because it strengthens the very anti-black system that would pulverize black being . The pursuit of the object certainly has an “irrational” aspect to it, as Farred details, but it is not mere means without expectation; instead, it is a means that

undermines the attainment of the impossible object desired. In other words, the pursuit marks a cruel attachment to the means of subjugation and the continued widening of the gap between historical reality and fantastical ideal. Black nihilism is a “demythifying” practice, in the Nietzschean vein, that uncovers the subjugating strategies of

political hope and de-idealizes its fantastical object. Once we denude political hope of its axiological and ethical veneer, we see that it operates through certain strategies: 1) positing itself as the only alternative to the problem of anti-blackness, 2) shielding this alternative [End Page 221] from rigorous historical/philosophical critique by placing it in an unknown

future, 3) delimiting the field of action to include only activity recognized and legitimated by the Political, and 4) demonizing critiques or different philosophical perspectives. The politics of hope masks a particular cruelty under the auspices of “happiness” and “life.” It terrifies with the dread of “no alternative .” “Life” itself needs the security of the alternative, and, through this logic, life becomes untenable without it.

Political hope promises to provide this alternative—a discursive and political organization beyond extant structures of violence and destruction. The construction of the binary “alternative/no-alternative” ensures the hegemony and dominance of political hope within the onto-existential horizon. The terror of the “no alternative”—the ultimate space of decay, suffering, and death—depends on two additional binaries: “problem/solution” and “action/inaction.” According to this politics, all problems have solutions, and hope provides the accessibility and realization of these solutions. The solution establishes itself as the elimination of “the problem”; the solution, in fact, transcends the problem and realizes Hegel’s aufheben in its constant attempt to sublate the dirtiness of the “problem” with the pristine being of the

solution. No problem is outside the reach of hope’s solution—every problem is connected to the kernel of its own eradication. The politics of hope must actively refuse the possibility that the “solution” is, in fact, another problem in disguised form; the idea of a “solution” is nothing more than the repetition and disavowal of the problem itself. The solution relies on what we might call the “trick of time”

to fortify itself from the deconstruction of its binary. Because the temporality of hope is a time “not-yet-realized,” a future tense unmoored from present-tense justifications and pragmatist evidence, the politics of hope cleverly shields its “solutions” from critiques of impossibility or repetition. Each insistence that these solutions stand up against the lessons of history or the rigors of analysis is met with the rationale that these solutions are not subject to history or analysis because they do not reside

Page 97: openev.debatecoaches.org€¦  · Web view1AC . Observation 1: If I could find the spot where truth echoesI would stand there and whisper memories of my children's future. I would

within the horizon of the “past” or “present.” Put differently, we can never ascertain the efficacy of the proposed solutions because they escape the temporality of the moment, always retreating to a “not-yet” and “could-be” temporality. This “trick” of time offers a promise of possibility that can only be realized in an indefinite future, and this promise is a bond of uncertainty that can never be redeemed, only imagined. In this sense, the politics of hope is an instance of the psychoanalytic notion of desire: its sole

purpose is to reproduce its very condition of possibility, never to satiate or bring fulfillment. This politics secures its hegemony through time by claiming the future as its unassailable property and excluding (and

devaluing) any other conception of time that challenges this temporal ordering. The politics of hope, then, depends on the incessant (re)production and proliferation of problems to justify its existence. Solutions cannot really exist within the politics of hope, just the illusion of a different order in a future tense . The “trick” of time and political solution converge on the site of “action.” In critiquing the politics of hope, one encounters the rejoinder of the dangers of inaction. “But we can’t just do nothing! We have to do something.” The field of permissible action is delimited and an unrelenting binary between action/ inaction silences critical engagement with political hope. These exclusionary operations rigorously

reinforce the binary between action and inaction and discredit certain forms of engagement, critique, and protest. Legitimate action takes place in the political—the political not only claims futurity but also action as its property. To “do something” means that this doing must translate into recognizable political activity; “something” is a stand-in for the word “politics”—one must “do politics” to address any problem. A refusal to “do politics” is equivalent to “doing nothing”—this nothingness is constructed as the antithesis of life, possibility, time, ethics, and morality (a “zero-state” as Julia Kristeva [1982] might call it).

Black nihilism rejects this “trick of time” and the lure of emancipatory solutions. To refuse to “do politics” and to reject the fantastical object of politics is the only “hope” for blackness in an antiblack world.

Anti-blackness is a structural antagonism that undergirds political life. Their gesture towards non-Black bodies facilitates the expansion of antiblackness, colonialism, and ultimately sustains the ontological foundation and expansion of the current socioeconomic order. Dumas 16 [Michael J. Dumas, Assistant Professor at the University of California, Berkeley in the Graduate School of Education and the Department of African American Studies, “Against the Dark: Antiblackness in Education Policy and Discourse,” Theory Into Practice 55:11–19, 2016, published by The College of Education and Human Ecology, The Ohio State University]

Antiblackness is the central concern and proposition within an intellectual project known as Afro-pessimism.1 Afro-pessimism theorizes that Black people exist in a structurally antagonistic relationship with humanity . That is, the very technologies and imaginations that allow a social recognition of the humanness of others systematically exclude this possibility for the Black. The Black cannot be human , is not simply an Other but is other than human. Thus, antiblackness does not signify a mere racial conflict that might be resolved through organized political struggle and appeals to the state and to the citizenry for redress. Instead, antiblackness marks an irreconcilability between the Black and any sense of social or cultural regard . The aim of theorizing antiblackness is not to offer solutions to racial inequality, but to come to a deeper understanding of the Black condition within a context of utter

contempt for, and acceptance of violence against the Black. Afro-pessimist scholars contend that the Black is socially and culturally positioned as slave, dispossessed of human agency, desire, and freedom. This is not meant to suggest that Black people are currently

enslaved (by whites or by law), but that slavery marks the ontological position of Black people . Slavery is how Black existence is imagined and enacted upon, and how non-Black people—and particularly whites— assert their own right to freedom, and right to the consumption, destruction, and/or simple dismissal of the Black. “Through chattel slavery,” Frank Wilderson (2010) argued, the world gave birth and coherence to both its joys of domesticity and

Page 98: openev.debatecoaches.org€¦  · Web view1AC . Observation 1: If I could find the spot where truth echoesI would stand there and whisper memories of my children's future. I would

to its struggles of political discontent; and with these joys and struggles the Human was born, but not before it murdered the Black, forging a symbiosis between the political ontology of Humanity and the social death of Blacks. (pp. 20 – 21) This “social death” of the slave is introduced most explicitly in the work of Orlando Patterson (1982), who detailed how slavery involves a parasitic relationship between slave owner and slave, such that the freedom of the slave owner is only

secured and understood in relation to power over the slave. For Patterson, slavery is “the permanent, violent domination of natally alienated and generally dishonored persons” (p. 13). Although slavery involves personal relationships between groups, it also operates as an institutionalized system, maintained through social processes that make it impossible for the Slave to live, to be regarded as alive for her- or himself in the social world. This focus on slavery might seem anachronistic in the current historical moment, some 150 years after the (formal) end of the

institution in the United States. However, Wilderson maintained that the relations of power have not changed. He explained: Nothing remotely approaching claims successfully made on the state has come to pass. In other words, the election of a Black

president aside, police brutality, mass incarceration, segregated and substandard schools and housing , astronomical rates of HIV infection, and the threat of being turned away en masse at the polls still constitute the lived experience of Black life . (p. 10) This lived experience serves as a continual reinscribing of the nonhumanness of the Black , a legitimization of the very antiblackness that has motivated centuries of violence against Black bodies . In this sense, even as slavery is no longer official state policy and practice, the slave endures in the social imagination, and also in the everyday suffering experienced by Black people. As Saidiya Hartman (2007) insisted, Americans are living in what she described as “the afterlife of slavery:” Black lives are still imperiled and devalued by a racial calculus and a political arithmetic that were entrenched centuries ago. This is the afterlife of slavery—skewed life chances, limited access to health and

education, premature death, incarceration, and impoverishment. I, too, am the afterlife of slavery. (p. 6) Importantly here, the afterlife of slavery is not only an historical moment, but deeply impressed upon Black flesh, in the embodiment of the Black person as slave. Thus, Hartman maintained, she is also this afterlife of slavery. Salamishah Tillet (2012) made clear the heaviness of the historical memory, the

everpresence of slavery in Black life: Because racial exclusion has become part and parcel of African American political identity since slavery, it cannot simply be willed or wished away. This protracted experience of disillusionment, mourning, and yearning is in fact the basis of African American civic estrangement. Its lingering is

not just a haunting of the past but is also a reminder of the present-day racial inequities that keep African American citizens in an indeterminate, unassimilable state as a racialized ‘Other.’ While the affect of racial melancholia was bred in the dyad of slavery and democracy, it persists because of the paradox of legal citizenship and civic estrangement. (p. 9) To the extent that there is ample evidence of the civic estrangement of Black people—their exclusion from the public sphere—one can theorize that the Black is still socially positioned as the slave, as difficult as it may be to use this frame to understand contemporary “race relations.” Here, “race relations” is necessarily in quotations because there is really no relation to be had between master and slave in the way one might conceptualize human

relationships. For Afro-pessimists, the Black is not only misrecognized, but unrecognizable as human, and therefore there is no social or political relationship to be fostered or restored . As Wilderson argued, Our analysis cannot be approached through the rubric of gains or reversals in struggles with the state and civil society, not unless and until the interlocutor first explains how the Slave is of the world. The onus is not on one who posits the Master/Slave dichotomy but on the one who argues there is a distinction between Slaveness and Blackness. How, when, and where did such a split occur? (p. 11) And this is the broader challenge posed by a

theory of antiblackness: There is no clear historical moment in which there was a break between slavery and

acknowledgement of Black citizenship and Human-ness; nor is there any indication of a clear disruption of the technologies of violence—that is, the institutional structures and social processes—that maintain Black subjugation. Thus, Afro-pessimists suggest that one must consider the Black as (still) incapable of asking for (civil or human) rights. This does not deny the long legacy of Black racial struggle, but it

positions this struggle as an impossibility, because the Black is (still) imagined outside of the citizenship that allows claims for redress to be regarded as legitimate, or even logical. Part of the challenge in theorizing blackness in contemporary race discourse is that Americans are living in an officially antiracist society, in which, as Jodi Melamed has documented, postWorld War II racial liberalisms and neoliberalisms make some space for the participation of multicultural subjects (Melamed, 2011). That is,

even as race continues to structure capitalism, which in turn facilitates white accumulation, the official stance of the state is against racism; blatantly racist laws and government practices have been declared illegal, and the market embraces outreach to a wide multicultural range of consumers. In this context, there is a rush to celebrate the social and economic advancement of select Black individuals and, perhaps more significantly, the success of other groups of people of color. In fact, it is the social and cultural inclusion of non-

Page 99: openev.debatecoaches.org€¦  · Web view1AC . Observation 1: If I could find the spot where truth echoesI would stand there and whisper memories of my children's future. I would

Black people of color that is often offered as evidence of the end of racial animus and racial barriers in the society. Therefore, the failure of large swaths of the Black population is purported to be a result of cultural deficits within the Black . The slave, always suspected of being lazy and shiftless, now must bear primary responsibility for not making it in a society, which—officially, anyway

—thrives on multiracial harmony and civic participation. Jared Sexton (2008, 2010) contended that in this era, multiracialism thrives largely at the expense of, and firmly against, blackness. His argument rests on the premise that the color line is more fluid during periods in which Black freedom is thought to be most contained. Thus, during slavery in the United States, multiracial communities could serve as “buffer classes between whites and blacks” which often “corroborated and collaborated with antiblackness” (Sexton, 2008, p. 12). The

current period is marked by similar dynamics, with little organized Black political movement, resegregation of

neighborhoods and schools, and, in fact, an easy deterritorialization and gentrification of historic Black urban homeplaces. The current Black Lives Matter movement (Garza, 2014), which has emerged in the wake of so many cases of antiBlack violence,

may yet shift Americans into a period of heightened anxiety about Black bodies, but Sexton’s description of the

current period is valid: There is little fear of Black bodies and, arguably, an emboldened antipathy to the Black overall. This, in Sexton’s theorizing, opens up new spaces for multiracial inclusion. In this moment, the Black –white divide is seen as less consequential and not as much the result of white attitudes and behaviors . In these moments, Sexton maintained, the more significant boundary is the one constructed “between blackness and everything else” (2008, p. 13). And this is a boundary seemingly constructed and maintained by recalcitrant Black people against multiracialism, and more to the point,

multiracial progress. Multiracialism, in Sexton’s view, “premises its contribution to knowledge, culture and politics upon an evacuation of the historical richness , intellectual intensity, cultural expansiveness, and political complexity of Black experience, including, perhaps especially, its indelible terrors” (2008, p. 15). Transcending the Black-white binary , multiracialism ostensibly moves people past the narrowness and anachronism of blackness and toward a more profitable global economy and more sophisticated cultural milieu . Embracing non- Black bodies of color thus facilitates, and is facilitated by, antiblackness, and can be justified as antiracist precisely because it is inclusive of more than white. “The [B]lack body,” Lewis Gordon contended, “is confronted by the situation of its absence” (1997, p. 73). This absence—this social death or afterlife of/as the slave—positions Black people as the embodiment of problem, a thing rather than a people suffering from problems created by antiblackness. Part of the aim of Afropessimist scholarship is to insist on the humanity of Black people. “Those of us who seek to understand [B]lack people,” Gordon concluded, need to “bear in mind that [B]lack people are human beings” (p. 78). In an anti-Black world, this is easier said than done. In the end, there may be, as Wilderson suggested, no “roadmap to freedom so extensive it would free us from the epistemic air we breathe” (2010, p.

338). Even so, like Gordon, Wilderson suggested that theorizing antiblackness is important simply as an existential and political recognition of Black humanity, as a means “to say we must be free of air, while admitting to knowing no other source of breath” (p. 338; italics in original).

The BDS campaign is a rallying cry for left leaning liberals who prey upon philia and recouperation of the nation state – this results in inevitable capital colonial exploitation – radical critique not tied upon the nation state is the only way to articulate a new worldSperber 16 (Joshua Sperber is a doctoral candidate in the political science department at the City University of New York (CUNY) Graduate Center. JOURNAL OF PALESTIAN STUDIES, VOL. 45 2015/16, https://www.palestine-studies.org/jps/fulltext/195933, accessed 3/19/19, jmg)

To return to the question posed above, why then do so many critics abandon their conventional analytic frameworks for interpreting the world when it comes to Israel? I argue that such interpretations result from a convergence of two phenomena: the existence of generally unquestioned yet problematic liberal-left ontologies and a widespread erroneous perception of Israel’s uniqueness. Broadly speaking, the Left promotes concepts that are neither designed nor intended to advance its interests. For instance, Judith Butler has written, “The BDS movement has become the most important contemporary alliance calling for an end to forms of citizenship based on racial stratification, insisting on rights of political self-determination for those for whom such basic freedoms are denied or indefinitely suspended, insisting as well on substantial ways of redressing the rights of those forcibly and/or illegally dispossessed of property and land.” [33] But one need only examine the contemporary world to doubt the merits of citizenship.While it is undeniably oppressive to be excluded from citizenship (the

Page 100: openev.debatecoaches.org€¦  · Web view1AC . Observation 1: If I could find the spot where truth echoesI would stand there and whisper memories of my children's future. I would

production of a defining outside is part and parcel of citizenship’s raison d’être), Jewish Israeli citizens are subject to the same domination, vulnerability, and exploitation that nearly all members of modern states are. Not merely neoliberalism but capitalism has maintained the existence of an exploited Jewish Israeli working class that has to appease bosses at the threat of hunger and landlords at the threat of homelessness. [34] Proactively demanding such a status for all does little other than subordinate notions of liberation to a fetishized regressive egalitarianism. Further, the historically and explicitly tractable nature of rights does not prevent Butler from demanding that they be restored to Palestinians. But who grants these rights in the first place, and who has the power to restore them? And who retains the power to suspend them again if they see fit, as they are suspended now? Articulating a historically grounded skepticism of rights, Giorgio Agamben has suggested that: It is time to stop regarding declarations of rights as proclamations of eternal, meta-juridical values binding the legislator (in fact, without much success) to respect eternal ethical principles, and to begin to consider them according to their real historical function in the modern nation state. The same bare life that in the ancien régime was politically neutral and belonged to God as creaturely life . . . now fully enters into the structure of the state and even becomes the earthly foundation of the state’s legitimacy and sovereignty. [35] While the Left tends to invoke citizenship and rights as defenses against institutional power, it by contrast generally acclaims democracy as an unquestioned good. And the Left’s indignation concerning Israel arises from, as much as any other factor, the fact that Israel, notwithstanding restricted Arab representation in the Knesset, is not democratic. Israel, besides having declared itself “a light unto nations,” is one of the last liberal states to grant formal rights to its predominantly European-descended majority while denying these rights (from property and marriage rights to the right of return) to its predominantly indigenous minorities (namely, Arabs and other non-Jews) who constitute 25 percent of the Israeli population proper. The resemblance between this de facto and de jure legal, political, and social discrimination and nineteenth-century European colonialism, as well as apartheid South Africa and the Jim Crow South, encourages the anti-racist Left to view Israel as a particularly hypocritical and offensive relic of an unacceptable and bygone era. It is precisely this relationship between a ruling group of predominantly white and biologically defined Jews and an excluded group of mainly Muslim people of color that makes Israel salient in the Left imagination. There are numerous reasons why there have not been prominent leftist protests of Saudi Arabia or Chad, countries with horrendous social divisions and oppression, but chief among them is that in these countries the rulers by and large look—at least to most observers in the Global North—like the ruled. And it is in part the largely unquestioned fealty to the concept and practice of democracy that prevents the Left from composing a genuinely radical critique of the world, and a fortiori Israel, as it exists today. That the desire for democracy is ubiquitous should, but apparently does not, raise concern among leftists whose protest chant of “This is what democracy looks like” comfortably coexists with warmongering presidents’ vows to bring democracy to the Middle East. While leftists are enamored with the notion of the demos ruling themselves, they spend less time examining the content and purposes of this rule. Fixated on the who and the how rather than the what and the why, leftists valorize mere proceduralism, a form without content. The devastating, if not absurd, consequences of the Left’s fixation on democracy were recently on display in Greece, where the left-wing coalition, Syriza, was seen as scoring a partial victory over the European Union (EU) when the latter permitted the socialists to self-manage the EU’s austerity program, which of course does nothing to challenge poverty as such and on the contrary ideologically reinforces it as an unavoidable fact of life. And leftists of course frequently compare Israel to South Africa and invoke apartheid’s demise as an inspiring model to emulate. It need not defend racist South African rule, however, to note that South African poverty is as devastating today as it has ever been, but because this poverty is administered by putative representatives of the majority population, the Left, and the world as a whole, take far less notice. That leftist supporters of Palestinian nationalism invoke such models of self-administered poverty as examples of radical politics indicates that the Left, beholden in spite of itself to a form of identity politics, has in effect adopted the economic and political interests of a Palestinian ruling class that is eager to exploit Palestinian workers without external interference. At best, such a stance reveals a fatal paucity of political imagination. * * * However, problematic liberal-left assumptions are ubiquitous and therefore cannot of themselves explain the Left’s idiosyncratic treatment of Israel. This treatment results, I argue, from the convergence of these liberal-left presuppositions with a prevailing perception of Israeli nationalism itself, a perception that should be distinguished from the prosaic material reality of the historical development of Zionism and the Israeli nation-state. The modern project of the nation-state fuses the two institutions that compose its name, but while states, whether dynastic, monarchical, or papal, have existed for millennia, the concept of the nation did not develop until the late eighteenth century. And whereas the state is a territorial fact, the nation is socially constructed, an “imagined community” that bases its legitimacy and authority on an ancient “tradition” that paradoxically could only have been “invented” under modernity. The inherently contrived nature of the nation was perhaps best evinced by Massimo d’Azeglio’s well-known pronouncement following Italy’s 1862 unification, “We have made Italy. Now we must make Italians.” Nevertheless, throughout the nineteenth and twentieth centuries, nationalism, defined as real, became “real in its effects.” Nationalism both produced and articulated the collective self-identification and political aims of, most prominently, independence movements that defined themselves in opposition to the multinational Hapsburg, Ottoman, and Romanov empires that collapsed following World War I and the European and Japanese colonial empires that collapsed following World War II. Notwithstanding the role of key economic factors, these

Page 101: openev.debatecoaches.org€¦  · Web view1AC . Observation 1: If I could find the spot where truth echoesI would stand there and whisper memories of my children's future. I would

national independence movements evolved politically in part as reactions to rule by perceived interlopers in particular geographic areas, whether in Serbia, Kenya, Vietnam, or India. It is not difficult to view Zionism as being distinct in this regard. A minority nearly everywhere but a majority nowhere, Jewish nationalists did not fight for control of any of the lands where they resided except for late Mandatory Palestine, where most Jews were relative newcomers. Instead of constructing a national identity within the context of a localized and concrete struggle with a dominant power that they sought to win independence from, Zionism invoked a globally diffused religious-ethnic-cum-national identity whose demand for a state (whether in Argentina, Uganda, or ultimately Palestine) was initially distinct from any single, ongoing, territorially specific struggle. That is, instead of adopting nationalism as the language of its political struggle, Zionism sought a specific and winnable political struggle that would enable it to implement its nationalism. This was a nationalism that, instead of defining itself against the Habsburgs, French, or British, sought in this respect to define itself far more abstractly against all. [36] Zionism in fact exploited this imagined abstractness in order to mystify its ethnic cleansing of indigenous Palestinians whom it universalized as “mere” Arabs. [37] Whereas the contemporary Left indeed criticizes Zionism for having achieved its national independence at the expense of the indigenous population, I am suggesting that it is this notion of Zionism’s abstractness that ironically helps account for the perceived precariousness of Zionism’s legitimacy outside of the conservative biblical imagination. Such abstractness was only reinforced by the explicit contingency accompanying Israel’s creation. Israeli statehood was dramatically established with the UN’s 1947 vote, which was influenced by sympathy from the “international community” (more accurately, the Great Powers) for what remained of European Jews following the Nazi genocide. The problem with such a “gift,” however, is that everyone knows that gifts can be withdrawn, a perception that to some extent characterizes Israel as an anomalously tenuous member of the “family of nations.” There was a revealing presumptuousness in the Great Powers’ resolution of the Jewish Question at the expense of, not European Christians who had historically persecuted Jews as such, but, Muslim Arabs who had historically been far more tolerant of Jews whom they either treated as one minority among others or respected as a fellow “People of the Book.” As Israel’s abuse of Palestinians has become more visible and abhorrent, it is difficult for its critics not to wonder whether Israel is forfeiting the goodwill that enabled it to become a state in the first place. To be sure, the Zionist lesson of the Judeocide at first glance appears convincing, although not in the manner in which the defenders of a Jewish state imagine. If there is a common denominator in the Nazis’ conceptions and persecution of Jews it is that of nationless-ness. The Nazis conceived of Jews as, in the pseudo-scientific language of the day, an unassimilable “virus” that attacks not merely the “host” but the “host’s” very “immune system.” [38] More specifically, Nazi ideology conceived of the Jews as less of an “actual” race than an “anti-race” existing outside of the Nazis’ carefully constructed racial hierarchy. [39] Simultaneously, the Nazis feared Jews as the personification of Judeo-Bolshevism, a historical force that was understood as an existential threat to Germany’s borders, national identity, social relations, and statehood. The common characteristic uniting these seemingly disparate categorizations is that they both define the Jew as intrinsically international or, more precisely, nationless. I argue that it is only through examining the Nazis’ dread of nationless-ness that we can reconcile the apparent contradiction in the Nazis’ description of the Jew as both international communist and global capitalist. Similarly, it is only through the idea of nationless-ness that we can adequately account for the Nazis’ ferocious persecution of their second-most hated enemy: the Gypsies, of whom half a million were murdered. And Nazism, lest we forget, represented nationalism par excellence. It is then mindless if not cruel to conclude that the appropriate response to the Nazis’ genocide is the creation of yet another state rather than the radical criticism of nationalism and the system of nation-states itself. For the creation of Israel has not only led to the oppression of non-Jews but has also failed to protect the security of its Jewish citizens; Netanyahu’s exhortations to European Jews to immigrate to their supposed haven are belied by his perpetual warnings of an impending Iranian nuclear attack. Moreover, it is not only Jews who have been destroyed by nation-constructing or empire-building states but also Native Americans and Armenians in addition to a great many others. In this regard, the “lesson of the Holocaust” is more relevant to the terrible vulnerability of the stateless and putatively unassimilable Palestinians than it is to Israel, which in the final analysis is defined less by any supposed Jewish essence than by the economic and political demands that confront all states. As Nicholas De Genova has asserted, “The heritage of the Holocaust belongs to the Palestinian people. The State of Israel has no claim to the heritage of the Holocaust. The heritage of the oppressed belongs to the oppressed, not the oppressor.” [40] But if it is unreasonable to suggest that the solution to Jewish suffering is a Jewish state, it is just as unreasonable to suggest that the solution to Palestinian suffering is a Palestinian state. Expressing a weariness of the notion that the only alternative to being a victim is to become a persecutor, Jean Genet has asserted, “The day when the Palestinians become institutionalized . . . I will no longer be at their side. The day the Palestinians become a nation like the other nations, I will no longer be there.” [41] Such a stance hardly entails inaction. On the contrary, it expresses an understanding that human liberation will result not from joining the very institutions that are responsible for mass suffering but from criticizing their fundamental purposes and, through this criticism, articulating a world worth fighting for.

Page 102: openev.debatecoaches.org€¦  · Web view1AC . Observation 1: If I could find the spot where truth echoesI would stand there and whisper memories of my children's future. I would

Blackness is the tool of civil society to avoid blame for everything it fucks up.Warren ’18 (Calvin Warren, Calvin Warren is an Assistant Professor in WGSS. He received his B.A. in Rhetoric/Philosophy (College Scholar) from Cornell University and his MA and Ph.D. in African American/American Studies from Yale University, 2018, Ontological Terror: Blackness, Nihilism, and Emancipation) \\EG

This lack of properness and metaphysical truth is a symptom of the nothing, for nothing lacks any proper place in metaphysics and cannot be

understood through its episteme. Black being as nothing, then, will always be out of place and improper in an antiblack world. It is the terror of the metaphysical infrastructure, and one can never be a true or proper man when one bears the weight of nothing. Through this analysis, we can understand the anxiety concerning black being, placement, and nothing in antebellum culture. In August 1842, for example, the free black population of Philadelphia held a parade commemorating the abolition of slavery in the West Indies.

A n angry mob of white citizens disrupted the parade, attacked participants, and commenced to destroy black homes and property. Seeking redress through the courts for loss of property and injury, the free black population realized that justice within such a context was impossible, as the grand jury acquitted the rioters and blamed free blacks for inciting this violence. Robert Purvis, a leader in the free black population of Philadelphia, responded to the grand jury’s decision with dismay:49 “The measure of our suffering is full. . . . From the most painful and minute investigation, in the feelings, views and acts of this community — in regard to us — I am convinced of our utter and complete nothingness in public estimation [emphasis mine].”50 What sparked the riot, this devastating expression of antiblackness? We can locate this eruption of

violence at the metaphysical fault line between necessity and hatred. Black being is both a necessary instrument for the human’s self-constitution and an object of ferocious hatred, since it bears the nothing of a metaphysical order. In other words, the riot is the symptom of a metaphysical problem: the public celebration of black freedom sparks a terror in that ontological boundaries are challenged and the transformation from black being, as invention/instrument, to human being, as free, is not only

considered but celebrated. It is also no surprise that the grand jury blamed the victims for the riot, since black freedom is a form of violence for the human , a violence that must be met with extreme force. The riot is a response to ontological terror. “Free,” when paired with “black,” is recast as a weapon against the human and the metaphysical structure that sustains the human. We are dealing with two registers of violence — one is an ontological violence and another is a physical form of antiblack destruction. But Purvis’s response to the violence is perspicuous. He is “convinced of our utter and complete nothingness in public estimation.” If we read this statement as a mere political lamentation, that blacks constitute a political cypher (nothingness) within the law and political processes, then we limit our understanding of the riot as event. The riot, within this reading, is just a form of cruelty or irrational intolerance or a political-economic strategy of subjection. With political readings of antiblack violence, violence is not gratuitous but must be linked to some type of recognizable transgression; when antiblack violence cannot be linked to recognizable transgression, it is considered cruel or irrational — a form of individual pathology and not systemic necessity. If, however, the essence of politics is nothing political, then we might read Purvis’s political commentary as a response to the proper metaphysical question.

His answer is that black being is nothingness in public estimation. We can understand nothingness as the condition (-ness) of

bearing nothing in an antiblack world. Antiblack violence, then, constitutes the structure of this nothing. Black being is always already under attack ; peace, within an antiblack world, is a fallacy (much like freedom). The metaphysical infrastructure

that supports the fiction of the white human is sustained by antiblack violence. The riot is an ontological necessity, not just political cruelty. We can understand the grand jury’s decision philosophically: Being black is both the cause and effect of violence , and when this being claims freedom, extreme violence is always justified and necessary.

The alternative is black nihilism --- refusing political hope is the only metaphysically coherent response to the constant slaughter of black bodies Warren 15 [Calvin K., Assistant Professor of American Studies at George Washington University, “Black Nihilism and the Politics of Hope,” CR: The New Centennial Review, Volume 15, Number 1, Spring 2015]

V. Conclusion Throughout this essay, I have argued that the Politics of hope preserve metaphysical structures that sustain black suffering. This preservation amounts to an exploitation of hope—when the Political colonizes the spiritual principle of hope and puts it in the service of extending the “will to power” of an anti-black organization of existence. The Politics of hope, then, is bound up with metaphysical violence, and

this violence masquerades as a “solution” to the problem of anti-blackness. Temporal linearity, perfection, betterment, struggle, work, and utopian futurity are conceptual instruments of the Political that will never obviate black suffering or anti-black violence; these concepts only serve to reproduce the conditions that render

Page 103: openev.debatecoaches.org€¦  · Web view1AC . Observation 1: If I could find the spot where truth echoesI would stand there and whisper memories of my children's future. I would

existence unbearable for blacks . Political theologians and black optimists avoid the immediacy of black suffering, the horror of anti-black pulverization, and place relief in a “not-yet-but-is (maybe)-to-come-social order” that, itself,

can do little more but admonish blacks to survive to keep struggling . Political hope becomes a vicious and abusive cycle of struggle —it mirrors the Lacanian drive, and we encircle an object (black freedom, justice, relief, redress,

equality, etc.) that is inaccessible because it doesn’t really exist. The political theologian and black optimist, then, propose a collective Jouissance as an answer to black suffering—finding the joy in struggle, the victory in toil, and the

satisfaction in inefficacious action. We continue to “struggle” and “work” as black youth are slaughtered daily, black bodies are incarcerated as forms of capital, black infant mortality rates are soaring, and hunger is disabling the bodies, minds, and spirits of desperate black youth. In short, these conditions are deep metaphysical problems—the sadistic pleasure of metaphysical domination—and “work ” and “struggle” avoid the terrifying fact that the world depends on black death to sustain itself . Black nihilism attempts to break this “drive”—to stop it in its tracks, as it were—and to end the cycle of insanity that political hope perpetuates. The question that remains is a question often put to the black nihilist: what is the point? This compulsory geometrical structuring of thought— all knowledge must submit to, and is reducible to, a point —it is an epistemic flicker of certainty, determination, and, to put it bluntly, life. “The point” exists for life; it enlivens, enables, and sustains knowledge. Thought outside of this mandatory point is illegible and useless. To write outside of the “episteme of life” and its grammar will require a position outside of this point , a position somewhere in the infinite horizon of thought (perhaps this is what Heidegger wanted to do with his reconfiguration of thought). Writing in this way is inherently subversive and refuses the geometry of thought. Nevertheless, the [End Page 243] nihilist is forced to enunciate his refusal through a “point,” a point that is contradictory and paradoxical all at once. To say that the point of this essay is that “the point” is fraudulent—its promise of clarity and life are inadequate—will not satisfy the hunger of disciplining the nihilist and insisting that one

undermine the very ground upon which one stands. Black nihilistic hermeneutics resists “the point ” but is subjected to it to have one’s voice heard within the marketplace of ideas. The “point” of this essay is that political hope is pointless . Black suffering is an essential part of the world, and placing hope in the very structure that sustains metaphysical violence , the Political, will never resolve anything . This is why the black

nihilist speaks of “exploited hope,” and the black nihilist attempts to wrest hope from the clutches of the Political . Can we think of hope outside the Political? Must “salvation” translate into a political grammar or a political program? The nihilist , then, hopes for the end of political hope and its metaphysical violence. Nihilism is not antithetical to hope; it does not extinguish hope but reconfigures it. Hope is the foundation of the black nihilistic hermeneutic. In “Blackness and Nothingness,” Fred Moten (2013) conceptualizes blackness as a “pathogen” to metaphysics, something that has the ability to unravel, to disable, and to destroy anti-blackness. If we read Vattimo through Moten’s brilliant analysis, we can suggest that blackness is the limit that Heidegger and Nietzsche were really after. It is a “blackened” world that will ultimately end metaphysics, but putting an end to metaphysics will also put an end to the world itself—this is the nihilism that the black nihilist must theorize through. This is a far cry from what we call “anarchy,” however. The black nihilist has as little faith in the metaphysical reorganization of society through anarchy than he does in traditional forms of political

existence. The black nihilist offers political apostasy as the spiritual practice of denouncing metaphysical violence, black suffering, and the idol

of anti-blackness. The act of renouncing will not change political structures or offer a political program ; instead,

it is the act of retrieving the spiritual concept of hope from the captivity of the Political . Ultimately, it is impossible to end metaphysics without ending blackness, and the black nihilist will never be able to withdraw from the Political completely without a certain death-drive or being-toward-death. This is the essence of black suffering : the lack of reprieve from metaphysics, the tormenting complicity in the reproduction of violence, and the lack of a coherent grammar to articulate these dilemmas . After contemplating these issues for some time in my office, I decided to take a train home. As I awaited my train in the station, an older black woman asked me about the train schedule and when I would expect the next train headed toward Dupont Circle. When I told her the trains

were running slowly, she began to talk about the government shutdown. “They don’t care anything about us, you know,” she

Page 104: openev.debatecoaches.org€¦  · Web view1AC . Observation 1: If I could find the spot where truth echoesI would stand there and whisper memories of my children's future. I would

said. “We elect these people into office, we vote for them, and they watch black people suffer and have no intentions of doing anything about it.” I shook my head in agreement and listened intently. “I’m going to stop voting,

and supporting this process; why should I keep doing this and our people continue to suffer,” she said. I looked at her

and said, “I don’t know ma’am; I just don’t understand it myself.” She then laughed and thanked me for listening to her—as if our conversation were somewhat cathartic. “You know, people think you’re crazy when you say things like this,” she said giving me a wink. “Yes they do,” I said. “But I am a free woman,” she emphasized “and I

won’t go back.” Shocked, I smiled at her, and she winked at me; at that moment I realized that her wisdom and courage penetrated my mind and demanded answers. I’ve thought about this conversation for some time, and it is for this reason I had

to write this essay. To the brave woman at the train station, I must say you are not crazy at all but thinking outside of metaphysical time, space, and violence . Ultimately, we must hope for the end of political hope.

Page 105: openev.debatecoaches.org€¦  · Web view1AC . Observation 1: If I could find the spot where truth echoesI would stand there and whisper memories of my children's future. I would

UK Case:The US and Russia won’t go to war—no escalationTSYGANKOV 2016 (Andrey, Professor at the Departments of Political Science and International Relations at San Francisco State University, ‘5 reasons why the threat of a global war involving Russia is overstated,” Russia Direct, Feb 19, http://www.russia-direct.org/opinion/5-reasons-why-threat-great-power-war-involving-russia-overstated)

Today’s world, while threatening and uncertain, is hardly more dangerous than the Cold War, for the following reasons.

First, whatever the rhetoric, major powers are not inclined towards risky behavior when their core interests are at stake. This concerns not only the nuclear superpowers , but also countries such as Turkey. The prospect of confronting Russia's overwhelmingly superior military should give pause even to someone as hot-tempered as Turkish President Tayyip Erdogan. Even if Erdogan wanted to pit Russia against NATO, it wouldn’t work.

So far, NATO has been careful to not be drawn into highly provocative actions, whether it is by responding to Russia seizing

the Pristina International Airport in June 1999, getting involved on Georgia’s side during the military conflict in August 2008 or by providing lethal military

assistance and support for Ukraine. Unless Russia is the clear and proven aggressor, NATO is unlikely to support Turkey and begin World War III.

Second, Russia remains a defensive power aware of its responsibility for maintaining international stability .

Moscow wants to work with major powers, not against them. Its insistence on Western recognition of Russia’s interests must not be construed as a drive to destroy the foundations of the international order , such as sovereignty, multilateralism, and arms control.

Third, the United States has important interests to prevent regional conflicts from escalating or becoming trans-

regional. Although its relative military capabilities are not where they were ten years ago, the U.S. military and diplomatic resources are sufficient to restrain key regional players in any part of the world . Given the power rivalry across several regions, proxy wars are possible and indeed are happening, but they are unlikely to escalate.

Fourth, unlike the Cold War era, the contemporary world has no rigid alliance structure. The so-called Russia-China-Iran axis is hardly more than a figment of the imagination by American neoconservatives and some Russia

conspiracy-minded thinkers. The world remains a space in which international coalitions overlap and are mostly formed on an ad hoc basis.

Fifth, with the exception of the Islamic State of Iraq and the Greater Syria (ISIS), there is no fundamental conflict of values and ideologies. Despite the efforts to present as incompatible the so-called “traditional” and “Western” values by Russia or “democracy” to “autocracy” by the

United States and Europe, the world majority does not think that this cultural divide is worth fighting for .

DESPITE THE DANGERS OF THE WORLD WE LIVE IN, IT CONTAINS A NUMBER OF IMPORTANT, EVEN UNDERAPPRECIATED, CHECKS ON GREAT POWERS’ MILITARISM

Despite the dangers of the world we live in, it contains a number of important , even underappreciated, checks on great powers’ militarism. The threat talk coming from politicians is often deceiving. Such talk may be a way to pressure the opponent into various political and military concessions rather than to signal real intentions. When such pressures do not bring expected results, the rhetoric of war and isolation subsides.

Page 106: openev.debatecoaches.org€¦  · Web view1AC . Observation 1: If I could find the spot where truth echoesI would stand there and whisper memories of my children's future. I would

Improved US-Russian relations right now cause Trump reelection and war with IranSIGOV 2019 (Mike Sigov, a former Russian journalist in Moscow, is a U.S. citizen and a staff writer for The Blade, “On Russia: Keeping a wary eye on improving U.S.-Russian relations,” Toledo Blade, May 19, https://www.toledoblade.com/opinion/columnists/2019/05/19/Keeping-a-wary-eye-on-U-S-Russian-relations/stories/20190518003)

The most immediate and undesirable outcome of such rapprochement is two-fold.

First — regardless of Mr. Pompeo’s over-reported nominal warning to the Russians not to interfere in the upcoming U.S. presidential elections issued in Sochi — the Trump campaign is sure to use any normalization in relations with Russia to undermine U.S. efforts to prevent Russia from influencing the elections in Mr. Trump’s favor the way Russia did in 2016.

Second, Mr. Trump’s threat of war with Iran may well materialize once Mr. Putin signs off on it as a concession to Mr. Trump for anticipated favors, such as relieving the sanctions .

While helping Mr. Trump to rally his base, such a war would be disastrous — it would exact a human, financial, and moral toll on us by far in excess of what the unwarranted second war in Iraq did.

The war also would hurt the cause of nuclear nonproliferation — which Messrs. Pompeo and Lavrov supposedly care so much about — by demonstrating that there is no other effective way to guarantee non-aggression by another state than access to nuclear weapons.

Two directly opposite U.S. policies — one toward nuclear-armed South Korea and the other toward Iran that has no nuclear weapons — attest to that.

As for Mr. Putin, a U.S. military campaign in Iran would be a godsend.

It’s guaranteed to hurt Iran’s oil production, thus affecting the world oil supply and boosting oil prices. That would significantly benefit both the Russian economy and Mr. Putin’s personal coffers, which rely heavily on oil exports.

The bottom line is the American interest mandates that we steer clear of rapprochement with Russia at least until after the 2020 presidential elections.

Trump reelection turns warming, prolif, US-Russian conflict, and nuke warSTARR 2019 (PAUL STARR is a professor of sociology and public affairs at Princeton and a winner of the Pulitzer Prize for General Nonfiction, “Trump’s Second Term,” The Atlantic, May, https://www.theatlantic.com/magazine/archive/2019/05/trump-2020-second-term/585994/)

This is one of those moments. After four years as president, Trump will have made at least two Supreme Court appointments, signed into law tax cuts, and rolled back federal regulation of the environment and the economy. Whatever you think of these actions, many of them can probably be offset or entirely undone in the future. The effects of a full eight years of Trump will be much more difficult, if not impossible, to undo.Three areas—climate change, the risk of a renewed global arms race, and control of the Supreme Court—illustrate the historic significance of the 2020 election. The first two problems will become much harder to address as time goes on. The third one stands to remake our constitutional democracy and undermine the capacity for future change.

In short, the biggest difference between electing Trump in 2016 and reelecting Trump in 2020 would be irreversibility. Climate policy is now the most obvious example . For a long time, even many of the people who acknowledged the reality of climate

change thought of it as a slow process that did not demand immediate action. But today, amid extreme weather events and worsening scientific forecasts, the costs of our delay are clearly mounting, as are the associated dangers. To have a chance at keeping global warming below 1.5 degrees Celsius—the objective of the

Paris climate agreement—the Intergovernmental Panel on Climate Change says that by 2030, CO2 emissions must drop some 45 percent from 2010 levels. Instead of declining, however, they are rising .

Page 107: openev.debatecoaches.org€¦  · Web view1AC . Observation 1: If I could find the spot where truth echoesI would stand there and whisper memories of my children's future. I would

In his first term, Trump has announced plans to cancel existing climate reforms , such as higher fuel- efficiency standards and limits on emissions from new coal-fired power plants, and he has pledged to pull the United States out of the Paris Agreement. His reelection would put off a national commitment to decarbonization until at least the second half of the 2020s, while encouraging other countries to do nothing as well. And change that is delayed becomes more economically and politically difficult. According to the Global Carbon Project, if decarbonization had begun globally in 2000, an emissions reduction of

about 2 percent a year would have been sufficient to stay below 2 degrees Celsius of warming. Now it will need to be approximately 5 percent a year. If we wait another decade, it will be about 9 percent. In the United States, the economic disruption and popular resistance sure to arise from such an abrupt transition may be more than our political system can bear. No one knows, moreover, when the world might hit irreversible tipping points such as the collapse of the West Antarctic Ice Sheet, which would likely doom us to a catastrophic sea-level rise .

The 2020 election will also determine whether the U.S. continues on a course that all but guarantees another kind of runaway global change—a stepped-up arms race, and with it a heightened risk of nuclear accidents and nuclear

war. Trump’s “America first” doctrine, attacks on America’s alliances, and unilateral withdrawal from arms-control treaties have made the world far more dangerous. After pulling the United States out of the Iran nuclear agreement (in so doing, badly damaging America’s reputation as both an ally and a negotiating partner), Trump failed to secure from

North Korea anything approaching the Iran deal’s terms, leaving Kim Jong Un not only unchecked but with increased international standing. Many world leaders are hoping that Trump’s presidency is a blip—that he will lose in

2020, and that his successor will renew America’s commitments to its allies and to the principles of multilateralism and nonproliferation. If he is reelected, however, several countries may opt to pursue nuclear weapons, especially those in regions that have relied on American security guarantees, such as the Middle East and Northeast Asia .

At stake is the global nonproliferation regime that the United States and other countries have maintained over the past several decades to persuade nonnuclear powers to stay that way . That this regime has largely

succeeded is a tribute to a combination of tactics, including U.S. bilateral and alliance-based defense commitments to nonnuclear countries, punishments and incentives, and pledges by the U.S. and Russia—as the world’s leading nuclear powers—to make dramatic cuts to their own arsenals.

In his first term, Trump has begun to undermine the nonproliferation regime and dismantle the remaining arms-control treaties between Washington and Moscow. In October, he announced that the U.S. would withdraw from the Intermediate-Range Nuclear Forces (INF) Treaty signed in 1987 by Ronald Reagan and Mikhail Gorbachev. While the Russian violations of the treaty that Trump cited are inexcusable, he has made no effort to hold Russia to its obligations—to the contrary, by destroying the treaty, he has let Russia off the hook. What’s more, he has displayed no interest in extending New START, which since 2011 has limited the strategic nuclear arsenals of Russia and the United States. If the treaty is allowed to expire, 2021 will mark the first year since 1972 without a legally binding agreement in place to control and reduce the deadliest arsenals ever created.

The prospect of a new nuclear arms race is suddenly very real. With the end of verifiable limits on American and Russian nuclear weapons, both countries will lose the right to inspect each other’s arsenal, and will face greater uncertainty about each other’s capabilities and intentions . Already, rhetoric has taken an

ominous turn: After Trump suspended U.S. participation in the INF Treaty on February 2, Vladimir Putin quickly followed suit and promised a “symmetrical response” to new American weapons. Trump replied a few days later in his State of the Union address, threatening to “outspend and out-innovate all others by far” in weapons development.

The treaties signed by the United States and Russia beginning in the 1980s have resulted in the elimination of nearly 90 percent of their nuclear weapons; the end of the Cold War seemed to confirm that those weapons had limited military utility. Now—as the U.S. and Russia abandon their commitment to arms control, and Trump’s “America first” approach causes countries such as Japan and Saudi Arabia to question the durability of U.S. security guarantees —the stage is being set for more states to go nuclear and for the U.S. and Russia to ramp up weapons development. This breathtaking historical reversal would, like global warming, likely feed on itself, becoming more and more difficult to undo.

Trump circumvents through IsraelAsa Winstanley. 7/4/2018. [Asa Winstanley is an investigative journalist living in London who writes about Palestine and the Middle East.], Israel is arming neo-Nazis in Ukraine, The Electronic Intifada. https://electronicintifada.net/content/israel-arming-neo-nazis-ukraine/24876. EC

Attempts by some in Congress to bar US military aid to Nazis in Ukraine may explain military aid from Israel.

Page 108: openev.debatecoaches.org€¦  · Web view1AC . Observation 1: If I could find the spot where truth echoesI would stand there and whisper memories of my children's future. I would

Israel’s “deepening military -technical cooperation ” with Ukraine and its fascist militias is likely a way to help its partner in the White House , and is another facet of the growing Zionist-White Supremacist alliance.

Israel has historically acted as a useful route through which US presidents and the CIA can circumvent congressional restrictions on aid to various unsavory groups and governments around the world.

Backing down on Ukraine encourages aggression throughout EuropeBARNETT 2015 (Neil Barnett runs Istok Associates (www.istok.co.uk), a corporate intelligence consultancy, and is a Research Fellow at the Centre for Policy Studies, “Why Ukraine Matters,” Center for Policy Studies, January, https://www.cps.org.uk/blog/q/date/2015/01/20/why-ukraine-matters/)

The many reminders of the 1930s in Russia's behaviour in Ukraine and the Baltic region have been well documented. Putin treats weakness as a provocation, and although to a remarkable extent the West has so far shown resolve, measure and unity, now is the moment when he hopes to exploit division ; only yesterday the EU's

new foreign policy chief, Federica Mogherini, tried and failed to float a series of measures to ease the pressure on Russia.

If force can prevail in Crimea and eastern Ukraine, then a very dangerous post-war precedent is set . A local

majority of kin population and historical links cannot be a pretext for annexation and destabilisation. If that were the case, then Hungary could by the same token invade parts of Romania, Croatia could advance into northern Bosnia and Austria might pounce on the Sud Tirol. This is not to mention the chaos Russia could inflict on the Baltic states and Central Asia .

So before we start to consider easing sanctions, it should be clear that the annexation of Crimea will not stand. Similarly, there must be a full withdrawal of Russian forces and support for separatists in eastern Ukraine. Any sordid deal that creates a frozen conflict as in Nagorno-Karabakh or Transdnistria

amounts to a veto on Ukraine's sovereignty and political development ; if the West colludes with Russia in carving up Ukraine this way it will be a disgrace on the

scale of selling out Czechoslovakia.

What is happening in Ukraine may well be a ‘quarrel in a far away country, between people of whom we know nothing.’ If we persist in our ignorance, we diminish ourselves and can blame no one else. Putin believes the west is so weak, decandent and demoralised that it will buckle before Russia does; the West must show that this is not so .

Page 109: openev.debatecoaches.org€¦  · Web view1AC . Observation 1: If I could find the spot where truth echoesI would stand there and whisper memories of my children's future. I would
Page 110: openev.debatecoaches.org€¦  · Web view1AC . Observation 1: If I could find the spot where truth echoesI would stand there and whisper memories of my children's future. I would

Vs. All AmericanAnti-blackness is a structural antagonism that undergirds political life. The 1ac frames oppression as white on to non white and not non black onto black. This seemingly inclusive framing compares human to non-human suffering, justifying the expansion of antiblackness, colonialism, and the ontological foundation and expansion of the current socioeconomic order. Dumas 16 [Michael J. Dumas, Assistant Professor at the University of California, Berkeley in the Graduate School of Education and the Department of African American Studies, “Against the Dark: Antiblackness in Education Policy and Discourse,” Theory Into Practice 55:11–19, 2016, published by The College of Education and Human Ecology, The Ohio State University]

Antiblackness is the central concern and proposition within an intellectual project known as Afro-pessimism.1 Afro-pessimism theorizes that Black people exist in a structurally antagonistic relationship with humanity . That is, the very technologies and imaginations that allow a social recognition of the humanness of others systematically exclude this possibility for the Black. The Black cannot be human , is not simply an Other but is other than human. Thus, antiblackness does not signify a mere racial conflict that might be resolved through organized political struggle and appeals to the state and to the citizenry for redress. Instead, antiblackness marks an irreconcilability between the Black and any sense of social or cultural regard . The aim of theorizing antiblackness is not to offer solutions to racial inequality, but to come to a deeper understanding of the Black condition within a context of utter

contempt for, and acceptance of violence against the Black. Afro-pessimist scholars contend that the Black is socially and culturally positioned as slave, dispossessed of human agency, desire, and freedom. This is not meant to suggest that Black people are currently

enslaved (by whites or by law), but that slavery marks the ontological position of Black people . Slavery is how Black existence is imagined and enacted upon, and how non-Black people—and particularly whites— assert their own right to freedom, and right to the consumption, destruction, and/or simple dismissal of the Black. “Through chattel slavery,” Frank Wilderson (2010) argued, the world gave birth and coherence to both its joys of domesticity and to its struggles of political discontent; and with these joys and struggles the Human was born, but not before it murdered the Black, forging a symbiosis between the political ontology of Humanity and the social death of Blacks. (pp. 20 – 21) This “social death” of the slave is introduced most explicitly in the work of Orlando Patterson (1982), who detailed how slavery involves a parasitic relationship between slave owner and slave, such that the freedom of the slave owner is only

secured and understood in relation to power over the slave. For Patterson, slavery is “the permanent, violent domination of natally alienated and generally dishonored persons” (p. 13). Although slavery involves personal relationships between groups, it also operates as an institutionalized system, maintained through social processes that make it impossible for the Slave to live, to be regarded as alive for her- or himself in the social world. This focus on slavery might seem anachronistic in the current historical moment, some 150 years after the (formal) end of the

institution in the United States. However, Wilderson maintained that the relations of power have not changed. He explained: Nothing remotely approaching claims successfully made on the state has come to pass. In other words, the election of a Black

president aside, police brutality, mass incarceration, segregated and substandard schools and housing , astronomical rates of HIV infection, and the threat of being turned away en masse at the polls still constitute the lived experience of Black life . (p. 10) This lived experience serves as a continual reinscribing of the nonhumanness of the Black , a legitimization of the very antiblackness that has motivated centuries of violence against Black bodies . In this sense, even as slavery is no longer official state policy and practice, the slave endures in the social imagination, and also in the everyday suffering

Page 111: openev.debatecoaches.org€¦  · Web view1AC . Observation 1: If I could find the spot where truth echoesI would stand there and whisper memories of my children's future. I would

experienced by Black people. As Saidiya Hartman (2007) insisted, Americans are living in what she described as “the afterlife of slavery:” Black lives are still imperiled and devalued by a racial calculus and a political arithmetic that were entrenched centuries ago. This is the afterlife of slavery—skewed life chances, limited access to health and

education, premature death, incarceration, and impoverishment. I, too, am the afterlife of slavery. (p. 6) Importantly here, the afterlife of slavery is not only an historical moment, but deeply impressed upon Black flesh, in the embodiment of the Black person as slave. Thus, Hartman maintained, she is also this afterlife of slavery. Salamishah Tillet (2012) made clear the heaviness of the historical memory, the

everpresence of slavery in Black life: Because racial exclusion has become part and parcel of African American political identity since slavery, it cannot simply be willed or wished away. This protracted experience of disillusionment, mourning, and yearning is in fact the basis of African American civic estrangement. Its lingering is

not just a haunting of the past but is also a reminder of the present-day racial inequities that keep African American citizens in an indeterminate, unassimilable state as a racialized ‘Other.’ While the affect of racial melancholia was bred in the dyad of slavery and democracy, it persists because of the paradox of legal citizenship and civic estrangement. (p. 9) To the extent that there is ample evidence of the civic estrangement of Black people—their exclusion from the public sphere—one can theorize that the Black is still socially positioned as the slave, as difficult as it may be to use this frame to understand contemporary “race relations.” Here, “race relations” is necessarily in quotations because there is really no relation to be had between master and slave in the way one might conceptualize human

relationships. For Afro-pessimists, the Black is not only misrecognized, but unrecognizable as human, and therefore there is no social or political relationship to be fostered or restored . As Wilderson argued, Our analysis cannot be approached through the rubric of gains or reversals in struggles with the state and civil society, not unless and until the interlocutor first explains how the Slave is of the world. The onus is not on one who posits the Master/Slave dichotomy but on the one who argues there is a distinction between Slaveness and Blackness. How, when, and where did such a split occur? (p. 11) And this is the broader challenge posed by a

theory of antiblackness: There is no clear historical moment in which there was a break between slavery and

acknowledgement of Black citizenship and Human-ness; nor is there any indication of a clear disruption of the technologies of violence—that is, the institutional structures and social processes—that maintain Black subjugation. Thus, Afro-pessimists suggest that one must consider the Black as (still) incapable of asking for (civil or human) rights. This does not deny the long legacy of Black racial struggle, but it

positions this struggle as an impossibility, because the Black is (still) imagined outside of the citizenship that allows claims for redress to be regarded as legitimate, or even logical. Part of the challenge in theorizing blackness in contemporary race discourse is that Americans are living in an officially antiracist society, in which, as Jodi Melamed has documented, postWorld War II racial liberalisms and neoliberalisms make some space for the participation of multicultural subjects (Melamed, 2011). That is,

even as race continues to structure capitalism, which in turn facilitates white accumulation, the official stance of the state is against racism; blatantly racist laws and government practices have been declared illegal, and the market embraces outreach to a wide multicultural range of consumers. In this context, there is a rush to celebrate the social and economic advancement of select Black individuals and, perhaps more significantly, the success of other groups of people of color. In fact, it is the social and cultural inclusion of non-

Black people of color that is often offered as evidence of the end of racial animus and racial barriers in the society. Therefore, the failure of large swaths of the Black population is purported to be a result of cultural deficits within the Black . The slave, always suspected of being lazy and shiftless, now must bear primary responsibility for not making it in a society, which—officially, anyway

—thrives on multiracial harmony and civic participation. Jared Sexton (2008, 2010) contended that in this era, multiracialism thrives largely at the expense of, and firmly against, blackness. His argument rests on the premise that the color line is more fluid during periods in which Black freedom is thought to be most contained. Thus, during slavery in the United States, multiracial communities could serve as “buffer classes between whites and blacks” which often “corroborated and collaborated with antiblackness” (Sexton, 2008, p. 12). The

current period is marked by similar dynamics, with little organized Black political movement, resegregation of

neighborhoods and schools, and, in fact, an easy deterritorialization and gentrification of historic Black urban homeplaces. The current Black Lives Matter movement (Garza, 2014), which has emerged in the wake of so many cases of antiBlack violence,

may yet shift Americans into a period of heightened anxiety about Black bodies, but Sexton’s description of the

current period is valid: There is little fear of Black bodies and, arguably, an emboldened antipathy to the Black overall. This, in Sexton’s theorizing, opens up new spaces for multiracial inclusion. In this moment, the Black –white divide is seen as less consequential and not as much the result of white attitudes and behaviors . In these moments, Sexton maintained, the more significant boundary is the one constructed “between blackness and everything else” (2008, p. 13). And this is a boundary seemingly constructed and maintained by recalcitrant Black people against multiracialism, and more to the point,

multiracial progress. Multiracialism, in Sexton’s view, “premises its contribution to knowledge, culture and politics upon an evacuation of the historical richness , intellectual intensity, cultural expansiveness, and political complexity of

Page 112: openev.debatecoaches.org€¦  · Web view1AC . Observation 1: If I could find the spot where truth echoesI would stand there and whisper memories of my children's future. I would

Black experience, including, perhaps especially, its indelible terrors” (2008, p. 15). Transcending the Black-white binary , multiracialism ostensibly moves people past the narrowness and anachronism of blackness and toward a more profitable global economy and more sophisticated cultural milieu . Embracing non- Black bodies of color thus facilitates, and is facilitated by, antiblackness, and can be justified as antiracist precisely because it is inclusive of more than white. “The [B]lack body,” Lewis Gordon contended, “is confronted by the situation of its absence” (1997, p. 73). This absence—this social death or afterlife of/as the slave—positions Black people as the embodiment of problem, a thing rather than a people suffering from problems created by antiblackness. Part of the aim of Afropessimist scholarship is to insist on the humanity of Black people. “Those of us who seek to understand [B]lack people,” Gordon concluded, need to “bear in mind that [B]lack people are human beings” (p. 78). In an anti-Black world, this is easier said than done. In the end, there may be, as Wilderson suggested, no “roadmap to freedom so extensive it would free us from the epistemic air we breathe” (2010, p.

338). Even so, like Gordon, Wilderson suggested that theorizing antiblackness is important simply as an existential and political recognition of Black humanity, as a means “to say we must be free of air, while admitting to knowing no other source of breath” (p. 338; italics in original).

Their faith in performance and conceptual rupturing as an act of emancipation trades off with structural analysis and puts false place in subjectivity Wilderson 2010, [2010, Frank B. Wilderson is an Associate Professor of African-American Studies at UC Irvine and has a Ph.D. from UC Berkeley, “Red, White & Black: Cinema and the Structure of U.S. Antagonisms,”]

Unfortunately, cultural studies that theorizes the interface between Blacks and Humans is hobbled in its attempts to (a) expose power

relationships and (b) examine how relations of power influence and shape cultural practice. Cultural studies insists on a grammar of suffering which assumes that we are all positioned essentially by way of the symbolic order , what Lacan

calls the wall of language—and as such our potential for stasis or change (our capacity for being oppressed or free) is overdeter-mined by our "universal" ability or inability to seize and wield discursive weapons. This idea corrupts the explanatory power of most socially engaged films and even the most radical line of political action because it produces a cinema and a politics that cannot account for the grammar of suffering of the Black—the Slave. To put it bluntly, the

imaginative labor5 of cinema, political action, and cultural studies are all afflicted with the same theoretical aphasia. They are speechless in the face of gratuitous violence. This theoretical aphasia is symptomatic of a debilitated ensemble of questions regarding political ontology. At its heart are two registers of imaginative labor. The first register is that of description, the rhetorical labor aimed at explaining the way relations of power are named, categorized, and explored. The second

register can be characterized as prescription, the rhetorical labor predicated on the notion that everyone can be emancipated through some form of discursive, or symbolic, intervention. But emancipation through some form of discursive or symbolic intervention is wanting in the face of a subject position that is not a subject position—what Marx calls "a speaking implement" or what Ronald Judy calls "an interdiction against subjectivity." In other words,

the Black has sentient capacity but no relational capacity. As an accumulated and fungible object, rather than an exploited and alienated subject, the Black is openly vulnerable to the whims of the world, and so is his or her cultural "production." What does it mean—what are the stakes—when the world can whimsically transpose one's cultural gestures, the stuff of symbolic intervention, onto another worldly good, a commodity of style? Frantz Fanon echoes this question when he writes, "I came into the world imbued with the will to find a meaning in things, my spirit filled with the desire to attain to the source of the world, and then I found that I was an object in the midst of other objects." He clarifies this assertion and alerts us to the stakes which the optimistic assumptions of film studies and cultural studies, the counterhegemonic promise of alternative cinema, and the emancipatory project of coalition politics cannot account for, when he writes: "Ontology—once it is finally admitted as leaving existence by the wayside—does not permit us to understand the being of the black."6 This presents a challenge to film production and to film studies given their cultivation and elaboration by the imaginative labor of cultural studies,

Page 113: openev.debatecoaches.org€¦  · Web view1AC . Observation 1: If I could find the spot where truth echoesI would stand there and whisper memories of my children's future. I would

underwritten by the assumptive logic of Humanism; because if everyone does not possess the DNA of culture, that is, (a) time and space transformative capacity, (b) a relational status with other Humans through which one's time- and space-transformative capacity is recognized and incorporated, and (c) a relation to violence that is contingent and not gratuitous, then how do we theorize a sentient being who is positioned not by the DNA of culture but by the structure of gratuitous violence? How do we think outside of the conceptual framework of subalternity—that is, outside of the explanatory power of cultural studies—and think beyond the pale of emancipatory agency by way of

symbolic intervention? I am calling for a different conceptual framework, predicated not on the subject-effect of cultural performance but on the structure of political ontology, a framework that allows us to substitute a culture of politics for a politics of culture. The value in this rests not simply in the way it would help us rethink cinema and performance, but in the way it can help us theorize what is at present only intuitive and anecdotal: the unbridgeable gap between Black being and Human life. To put a finer point on it, such a framework might enhance the explanatory power of theory, art, and politics by

destroying and perhaps restructuring the ethical range of our current ensemble of questions. This has profound implications for non-Black film studies, Black film studies, and African American studies writ large because they are currently entangled in

a multicultural paradigm that takes an interest in an insufficiently critical comparative analysis— that is, a comparative analysis in pursuit of a coalition politics (if not in practice then at least as a theorizing metaphor) which, by its very nature, crowds out and forecloses the Slave's grammar of suffering.

The 1AC discourse outlines a grammar of suffering that is incoherent to blackness- Positioning all subordinated peoples on the same plane conflates the gratuitous violence of antiblackness with white supremacy and other modes of being human.Saucier and Woods 16, P. Khalil Saucier is chair and associate professor of Africana Studies at Bucknell University. Tryon P. Woods is assistant professor of crime and justice studies at the University of Massachusetts, Dartmouth, and teaches Africana studies at Rhode Island College and of Black Studies at Providence College. 10/11/16, Conceptual Aphasia in Black: Displacing Racial Formation, “Introduction: Racial Optimism and the Drag of Thymotics,” NN

The recent efflorescence of work applying affect theory to the study of race and racism includes explicit criticisms and implicit endorsements of racial formation theory. Affect theory is used to account for how to think the “micro” of embodiment and intersubjectivity within neoliberal biopolitical state formations. James

Thomas’s work provides one example of this approach. Thomas claims to improve on racial formation theory through attention to the “affective-cultural assemblages . . . that produce racial order through their coming together with one another” (Thomas 2014, 74). According to Thomas, this emphasis attends to two deficiencies he finds with racial formation theory. The first problem is the undertheorized role of culture that draws a distinction between racial meaning and reality (78). For Thomas, instead of “a world existing outside of expression, with expression mediating that relationship, reality produces itself only through expression. Meaning-making, then, is not a point of access to reality—it is reality” (78). The second problem Thomas identifies is racial formation theory’s “simultaneous deconstruction of race as a real concept and the active

construction of an essentialist political identity of the Other” (81). In this view, the problem is not that identity politics subsumes blackness, but that it forbids interracial coalition . Thomas asks, “How . . . can a politics of solidarity be fostered among working-class Blacks and Whites when their agency is theorized as bounded to their race, class, and gender?” (83). Instead of identity (understood in this case as principally biological), Thomas suggests that affect can serve as a more productive basis for a shared sense of belonging regardless of race, produce “a unified anti-racist platform,” and foster “a practice-oriented enterprise for . . .

collective action, rather than an identity- oriented enterprise” (83, 85). Each of the different items on the additive menu coalesces in the dead zone to reproduce the conceptual aphasia regarding blackness. Each addition brings salient angles into play and yet the resulting geometry regards as tangential the singularity of black positionality. Racial formation theory and its critics cohere as the police power against blackness (Woods forthcoming). This antagonism runs deep and makes plain at least four issues we observe in brief here. First is the ongoing intransigence of white supremacy as an organizing rubric for both analysis and

action. The only legitimate antidote to white supremacy for Omi and Winant and their critics is multiculturalism, multiracial coalition, and racial projects arising from communities of color . Cutting through the sheen of race consciousness, this is simply humanism’s inclusion and liberalism’s integration principles, dressed up as if it were oppositional to colorblindness . From this perspective, these dynamics are prima facie

evidence that the blackwhite binary model of race relations is outmoded and a drag on post–civil rights racial formation. White supremacy thus sets the terms for both white dominance and its repudiation. If whites cannot legitimately stand apart, since autonomy signifies hierarchy, then no racialized group can either. This orientation towards the racial terrain is a power move against the longstanding interventions from black studies

Page 114: openev.debatecoaches.org€¦  · Web view1AC . Observation 1: If I could find the spot where truth echoesI would stand there and whisper memories of my children's future. I would

quarters—not to mention from black nationalist and black power political formations—that have consistently pointed out, in and across the generations, that

black positionality is singular and without analog among other non-black p eople o f c olor . In lending his

support to “the death of white sociology” Ron Walters cogently observed, “Black life has been distinctive and separate enough to constitute its own uniqueness ” and therefore its own analytical frame (1973, 197). Positioning all subordinated peoples on the same plane conflates the gratuitous violence of antiblackness with white supremacy, neoliberalism, capitalism, settler colonialism, heteronormative patriarchy, and other modes of being human: “The same people who recently confronted the police dogs and fire hoses, who crossed the mountains and deserts to reach Phoenix or Los Angeles (or traversed the Mediterranean to reach Madrid or Lisbon or Paris), the same people who resist military occupation in Jenin” (Winant 2012,

605). The reality registered throughout black historical struggle is that black suffering is a qualitatively different kind of problem, not merely a matter of degree or scale vis-à-vis other non-white people color . This point has been extensively elaborated throughout the corpus of black critical thought. To cite but one recent contribution on this score, Charles Mills

disentangles “the major varieties of white Western racism” in order to highlight that “no other nonwhite group has been so enduringly constitutive of their identity and so enduringly central to white racial consciousness and global racial consciousness in general” (2013, 34, 35, emphasis in original). Extend-ing his argument, Mills makes clear that the anti-Arab, anti-Semite, anti- Native American, anti-Latino, and anti-Aborigine do not create the very thing they are against, “ but the Negrophobe does create the Negro” (Mills 2013, 37 fn 10). Perhaps more forcefully, this point has been levied time and again from the

ranks of black social movement. Case in point is Cesaire’s biting resignation letter from the French Communist Party in 1956. He states with great acuity “ the singularity of our ‘situation in the world,’ which cannot be confused with any other . The singularity of our problems, which cannot be reduced to any other problem . The singularity of our history, constructed out of terrible misfortunes that belong to no one else” (Cesaire 2010, 147).20 For this basic reason,

the pitfalls of coalition have been long understood by the black movement . In his characteristic candor, Malcolm X states

in a “Message to the Grassroots”: “It’s just like when you’ve got some coffee that’s too black, which means it’s too strong. What you do? You integrate it with cream, you make it weak. If you pour too much cream in, you won’t even know you ever had coffee. It used to be hot, it becomes cool. It used to be strong, it becomes weak. It used to wake you up, now it’ll put you to sleep” (1965, 16). Walter Rodney, too, explained at length how African people’s development was irrevocably corrupted (what has been done cannot be undone) and that survival depends upon extracting an autonomy from the

oppressor culture (1990). While the push for coalition and the use of analogies by non-black people disavows these black voices across the generations, non-blacks are not shy about making use of black fungibility to promote their own ethical standing and to impose a scalar reversal in the world’s violence (Sexton 2010 and 2015). For instance, scenes of police violence against protestors in Ferguson, Missouri are more legible and coherent to civil society’s understanding of the ethical problems involved when analogized to Palestine, or the crisis of gun violence in black urban spaces can only be understood when related to war zones

elsewhere, as in the reference to Chicago as “Chi-Raq.” Hortense Spillers pointedly cuts to the power differentials recruited to the coalition question when she describes it as a historical “grid of identities running at perpendicular angles: things in serial and lateral array; beings in hierarchical and vertical array” (2003, 314).

The alternative is an unflinching paradigmatic analysis of the uncivility of civil society – the reclamation of the power to pose the question and analysis of ontological questions to articulate the worldWilderson 10 (Frank Wilderson, Professor of African American Studies at UC Irvine, “Red, White & Black: Cinema and the Structure of U.S. Antagonisms”)

STRANGE AS it might seem, this book project began in South Africa. During the last years of apartheid I worked for revolutionary change in both an underground and above-ground capacity, for the Charterist Movement in general and the ANC in particular. During this period, I began to see how essential an unflinching paradigmatic analysis is to a movement dedicated to the complete overthrow of an existing order. The neoliberal compromises that the radical elements of the Chartist Movement made with the moderate elements were due, in large part, to our inability or unwillingness to hold the moderates' feet to the fire of a political agenda predicated on an

Page 115: openev.debatecoaches.org€¦  · Web view1AC . Observation 1: If I could find the spot where truth echoesI would stand there and whisper memories of my children's future. I would

unflinching paradigmatic analysis. Instead, we allowed our energies and points of attention to be displaced by and onto pragmatic

considerations. Simply put, we abdicated the power to pose the question—and the power to pose the question is the greatest power of all. Elsewhere, I have written about this unfortunate turn of events (Incognegro: A Memoir of Exile and Apartheid), so I'll not rehearse the details here. Suffice it to say, this book germinated in the many political and academic discussions and debates that I was fortunate enough to be a part of at a historic moment and in a place where the word revolution was spoken in earnest, free of qualifiers and irony. For their past and ongoing ideas and interventions, I extend solidarity and appreciation to comrades Amanda Alexander, Franco Barchiesi, Teresa Barnes, Patrick Bond, Ashwin Desai, Nigel Gibson, Steven Greenberg, Allan Horowitz, Bushy Kelebonye (deceased), Tefu Kelebonye, Ulrike Kistner, Kamogelo Lekubu, Andile Mngxitama, Prishani Naidoo, John Shai, and S'bu Zulu. .¶ [CONTINUES]¶ In the Introduction and chapter 1, we saw how the aporia between Black being and political ontology has existed since Arab and European

enslavement of Africans. The crafting of questions through which one might arrive at an unflinching paradigmatic analysis of political ontology , a language that could express the structural and performative violence of Slave-making, is repeatedly thwarted. Humanist discourse, whose epistemological machinations provide our conceptual frameworks for thinking political ontology, is diverse and contrary. But for all its diversity and contrariness it is sutured by an implicit rhetorical consensus that violence accrues to the Human body as a result of transgressions, whether real or imagined, within the symbolic order. That is to say, Humanist discourse can only think a subject’s relation to violence as a contingency and not as a matrix that positions the subject. Put another way, Humanism has no theory of the Slave because it imagines a subject who has been either alienated in language or alienated from his or her cartographic and temporal capacities. 1 It cannot imagine an object who has been positioned by gratuitous violence and who has no car-

tographic and temporal capacities to lose—a sentient being for whom recognition and incorporation is impossible. In short, political ontology, as imagined through Humanism, can only produce discourse that has as its foundation alienation and exploitation as a grammar of suffering, when what is needed (for the Black, who is always already a Slave) is an ensemble of ontological questions that has as its foundation accumulation and fungibility as a grammar of suffering. 2 A Culture of Politics The violence of the Middle Passage and the Slave estate, 3 technologies of accumulation and fungibility, recompose and reenact their horrors on each succeeding generation of Blacks. This violence is both gratuitous (not contingent on transgressions against the hegemony of civil society) and structural (positioning Blacks ontologically outside of Humanity and civil society). Simultaneously, it renders the ontological status of Humanity (life itself ) wholly dependent on civil society’s repetition compulsion: the frenzied and fragmented machinations through which civil society reenacts gratuitous violence on the Black— that civil society might know itself as the domain of Humans— generation after generation. Again, we need a new language of

abstraction to explain this horror. The explanatory power of Humanist discourse is bankrupt in the face of the Black. It is inadequate and inessential to, as well as parasitic on, the ensemble of questions which the dead but sentient thing, the Black, struggles to articulate in a world of living subjects.

Page 116: openev.debatecoaches.org€¦  · Web view1AC . Observation 1: If I could find the spot where truth echoesI would stand there and whisper memories of my children's future. I would

The prefacing of representations of suffering as method to achieve justice and alleviate the violence done to victims is part and parcel of a fetishistic politics which herald injury as the center of political subjectivity. The reduction of another’s suffering to an image to be consumed reduces them to a fetishized object to be violently traded for ballots.Abbas 2010

/Asma, Professor and Division Head in Social Studies, Political Science, Philosophy at the Liebowitz Center for International Studies at Bard College at Simon’s Rock, Liberalism and Human Suffering: Materialist Reflections on Politics, Ethics, and Aesthetics, London: Palgrave Macmillan, pg. Pg. 121- 123/

An inquiry into injury as value and commodity, and the attempted recovery of a materialist method that can speak to the laboring and

suffering body in liberal capitalism, converge to reveal some of injury’s fetish qualities. Injuries as fetishes are products of the labor of suffering — the processes of liberal representation that mediate between our sensuousness and our ability to count and be valued as liberal subjects. When injury is installed as a form that allows our suffering to be visible and to count, it becomes a representation that only grants us solidarity as an afterthought. Injury fashions our suffering and our perceptions of others’ suffering in its image.1 Injury is a fetish because it is dead suffering, valuable in its deadness and its detachment from the life and particularity of the sufferer but at the same time actively defining the sufferer’s identity . Under the fetishism of injuries, suffering ceases to be a sensuous life activity because it can only be experienced in the mode of injury and recognized in its value-creating mode. Suffering counts only when it takes this abstract form of injury, equalizing many kinds of abiding inequalities in the abstract. The suffering that comprises the injury, on the one hand, and the experience of that externalized injury by the sufferer and by others, on the other, seem to be independent, unrelated, and unmediated processes. The relation between these alienated sufferings stands in for real, material social relations between individuals. These relations and their experience drop out of vision and are replaced by social relations between interests and injuries to them. The crux of fetishism is in how production and consumption appear to be independent, unrelated, and unmediated processes that they are not. This feigned indifference arises from the fact that social relations between individuals and the relations of production on which they rest drop out of vision and are replaced by social relations between things. The worker’s “external” relation to her

product is not the key or sole problem. The space between production and consumption may even be where the liberatory potential of capital reposes. Fetishism , however, encourages replacing the appropriation of productive activity (in a Marxist sense) by private property in things. Other obligations and virtues get supplanted. Fetishism is problematic not because it merely renders one’s unmistakably individual labor as indistinguishable from everyone else’s labor. The issue is that fetishism makes my labor appear private and individual, and that it does so by alienating, beyond recognition, its social character and my irreducible socialities and contingencies. The i njury itself becomes an autonomous, fantastical object that is oblivious to the real material relations and the labors that have produced it. This autonomy is from history and perhaps even time itself, so the fetish starts to believe it always was, always has been, and always will be. Fetishism is a broader and more trenchant concept for my purposes than, for instance, exploitation, for a few reasons. Despite the concrete ethical effectivity of Marx’s “scientific” concept of exploitation, it has no structural safeguards against victims

turning fetishes. In exploitation, the perpetrator-victim distinction is necessary, even if not framed in terms of the

autonomous willful individual subject. More importantly, in order to make its injustice claims, the concept of exploitation relies on the existing mode of valuation and value creation that suffering (and production) follows in liberalism (and capitalism). Fetishism, though, sees subjects suspended within elaborate, suffusive, and subtle relations that make the language of exploitation possible in the first place. It thus forces us to confront the incorporated laws of value and

representation and the economies of sensuousness and subjectivity in relation to our own and others’ suffering. Fetishism and exploitation differ in the labors of suffering and the work of victims that they allow to be made visible and, ultimately, the very kinds of existences they posit as real and imaginable . The modes of

Page 117: openev.debatecoaches.org€¦  · Web view1AC . Observation 1: If I could find the spot where truth echoesI would stand there and whisper memories of my children's future. I would

personification, identification, memory, and voicing that are fostered within the ascetic theater of the fetishism of injuries together call attention to how representation’s purposive sensings and presencings occasion certain forms of subjectivity from sufferers. If representation is seen as involving sensing and making sensible,

memory and voice are codependently intertwined in this labor (which also fuels Rawls’s injury play). The labor of suffering and its many instantiations within liberalism thus bespeak life channeled to sustain the fetishes of injuries and victims . At stake within any response to liberalism is, then, the honoring of the reality and potentiality of other modes of suffering and valuation beyond liberalism. A befitting response to liberalism must necessarily traverse these components of the labor of suffering and challenge liberalism’s undialectical understanding of them. This

requires going beyond recognition of injury and victim as fetish, to addressing the categories and relations internal to liberal representation and the fetishism of injuries.

The AFFs mode of accounting for violence is the logic of humanism and western imperialism which turns the aff you should refuse those modes of research. Hoofd 2017. Ingrid. Assistant Professor Department of Media and Culture Studies, Higher Education and Technological Acceleration. http://www.palgrave.com/it/book/9781137517517

So coping, so operating at top speed , one accounts and becomes accountable for nothing: not for what happens, not for the reasons to continue assuming responsibilities without a concept. (1992, 6, italics mine) One could read this as a certain call for a slowing down and taking stock, however urgently; and more particularly as an indictment of the ways in which the continuous over-production— one may think here of course of the steep rise in academic publications and journals, the overwhelming number of reviews that need to be done, the increase in global conference and research travels, and the entire emphasis on performance indicators — is fundamentally entangled with the neo-liberal techniques and technologies that make this possible. What is more, due to this slippery nature of academic responsibility, and due to its foundation residing in the fi

ction of reason and logic in the pursuit of total knowledge, Derrida implies that it is only logical that the “factory-like” properties of the university, in which the production of knowledge is treated “like an industry ” (Derrida

refers here to the words of Immanuel Kant), will slide towards an exceeding quantifi cation, in which digital technologies are roped in to help make sense of and organise the huge amount of new information—a case in point being, for instance, the so-called digital humanities. Such a quantifi cation nonetheless becomes exceedingly irresponsible insofar as it exacerbates the essentially “theatrical representation” of the so-called autonomy of the university by way of delegating it increasingly to a cybernetic machinery designed to close off radical alterity (1992, 3). In other words, while the criteria for academic competence and performance were in the past only seemingly independent (via peer reviews and such) but historically ultimately enmeshed with a patriarchal and Eurocentric politics of the nation-state, such a politics and its criteria for evaluation are now near-thoroughly enmeshed with the technologies of global accelerationand simulation. Derrida on this point once again echoes Lyotard’s analysis of the ways in which knowledge becomes a

product of technological power in The Postmodern Condition , when he says that academic performativity, besides being “theatrical,” also entails the “output of a technical system, a place where knowledge and power are no longer distinguished” (1992, 12). It is therefore “the publication of knowledge” or the communication and dissemination of ideas that fi nd themselves in “a doublebind, a demand … intrinsically in confl ict with itself,” as communicating more ultimately amounts to communicating less (1992, 12). I will discuss the ways in

which this contradiction returns in Virilio’s indictment of the ever more sophisticated ‘vision machines’ as a road to ultimate blindness, taking the analyses of Derrida and Lyotard regarding the enmeshment of the current university with cybernetic tools as a central starting point for the rest of this book. For now, I take it from Derrida

that it ultimately is the quest for universal objectivity that is the hallmark of the university that has

Page 118: openev.debatecoaches.org€¦  · Web view1AC . Observation 1: If I could find the spot where truth echoesI would stand there and whisper memories of my children's future. I would

birthed such an entanglement with supposedly ‘objective’ techniques of formalisation and quantification . The very argument for autonomy and independent objectivity therefore paradoxically has spawned its slippage into irresponsible rationalisation and quantifi cation that seek to stabilise the academic project in an extremely problematic fashion, just as Heidegger attempted to do . And if this is indeed the case, we may very well fi nd that it indeed engages in or runs parallel to a new kind of ‘fascism’ accompanying this rationalisation and

quantifi cation both inside and outside the university walls. Derrida’s insightful assessment warns against an academic activism solely based on a too-quick recanting of the usual academic slogans and concepts—democracy, truth, knowledge—for the purposes of minor internal adjustments that eventually wall off the university from the onslaught of neo-liberal disenfranchisement at large. To argue a return to the ivory tower in the

fashion proposed by Zielinski and Dittrich, while seemingly sympathetic, would then run the risk of merely ensuring the survival of a ‘happy (or perhaps not so happy) few’ within those walls, while the true risk of responsibility lies ultimately in affi rming life for all as entailing more than mere survival under a compromised neo-liberal regime . Lyotard in The Inhuman likewise gestures towards a veritable beyond of the academic project that paradoxically cannot

consist of academics and intellectuals simply “resisting the predominant use of time today,” because if they do so… they are not only predestined to disappear, but they … contribute to the making of a cordon sanitaire isolating themselves. In the shelter of this cordon, their destruction is supposed to be able to be put off for a while. But they ‘buy’ this brief and vain delay by modifying their way of thinking and writing in such a way that their works become more or less communicable, exchangeable; in a word, commercializable. But this exchange, the buying and selling of ideas and words, does not fail to contribute contradictorily to the ‘fi nal solution’ of the problem: how to write, how to think? (1991, 76)

The belief in the power of voting aff fails to subvert authority and locks in the university’s mastery over their AFFHoofd 2017. Ingrid. Assistant Professor Department of Media and Culture Studies, Higher Education and Technological Acceleration. http://www.palgrave.com/it/book/9781137517517

Without a doubt, one cannot , indeed one should not, think that our moral responsibilities as university teachers today without the concept of power which grounds itself in the ‘truth’ of a capitalist, patriarchal, materialist, and colonialist account of the world—to do so would surely mean to fall prey to neo-liberal moralism. But paradoxically, the argument that thinks itself as opposing this neo-liberalisation falls prey to such a moralism, too. The assumption in the argument of the original university with solid theoretical grounds as well as in my Singapore ethics class is surely that an objective theory of power will lead to a subversion of authority and to revolutionary action ‘from below.’ Yet, the students in my ethics class become even more upwardly mobile through scoring marks by learning to question their own relative positions of privilege. So in essence, my ethics class does precisely what the university presidents would want me to do: to provide these students with the creative and critical thinking abilities, with the ‘development of character’ that the new cosmopolitan kinetic elite needs for their entrepreneurial, ambassadorial, and researchoriented travels, endeavours, and connections. The spectre of a philosophy of praxis is problematically and productively haunting my Singapore ethics class as well as the efforts to rethink the role of humanities education in the Dutch context, making ethical thought possible within the organising principles of a historically Greek homo-erotic and ‘European’ understanding of teaching. How revolutionary can any such an implicit repetition of the homo-erotic and humanist mark on the politics of teaching and the function of the academic community at large then really be? How much of an ethics class, as would be an optimism about the ‘elimination of noise’ via the implementation of clear methods or theories, is an unlearning versus a consolidation of privilege, for student and teacher? Derrida remarks in “The Future of the Profession” while faithfully performing the assumptions and imperatives that underlie the teaching profession: “while many say that performativity creates the event, one should rather say that through performativity ‘nothing worthy

of the name ‘event’ can really ever take place’” (2002, 54). But obviously, this cannot be the final word on performativity, as there would be little motive for Derrida to lay bare the grounding assumptions of (his) pedagogy if it

Page 119: openev.debatecoaches.org€¦  · Web view1AC . Observation 1: If I could find the spot where truth echoesI would stand there and whisper memories of my children's future. I would

were a simple repetition—the objective seems rather to simultaneously render the performance unstable by offering a (re)reading . As in the case of hospitality, one ideally ought to hand over ownership of one’s structure (or place of dwelling) to the possible arrival of any other, yet one must first claim ownership of and make seemingly stable one’s structure before one can receive the other at all. Ortega y Gasset for one ultimately misses to acknowledge the ‘dark’ side of the university when grandiosely proclaiming in the fi nal chapter of the

Mission that “Europe is intelligence. A wonderful power: it is the only power which perceives its own limitations—and thereby proves how intelligent it is!” without really realising that the institutional violence on the social (by way of leaving the rise and consolidation of industrial capitalism unquestioned) consists also of setting up of such a fake stability and of being proud of ‘intelligently’ shedding light on its own limitations (1944, 87). My critical pedagogy, as indeed this critical analysis of its merits and demerits, really also argues for a self-reflexivity that borders on the self-indulgence so typical of a certain ‘Europeanness,’ a self-indulgence which somehow has become the sales pitch of neo-liberalism, celebrated in the UN Declaration of Human Rights as the ‘right to communication’

about which more in Chap. 4 . So again, one should not speak of a necessary retreat of politics from the curriculum, but of an acceleration and intensification of politics in the neo-liberal university—opening up unexpected spaces for critique in the face of its neo-liberalisation, which in turn points to the fundamental instability of its enterprise. The Asian and Dutch universities’ ‘responsible complicities’ in previous colonialism and current neo-liberalisation mark precisely the moment where the promise of philosophy gets magnifi ed as its sites of tension and confusion

multiply for teachers as well as students. This intensification of politics is however no ground for celebration, since it remains also the hallmark of the neo-liberal mode of production of knowledge through the new tele- technologies as ‘excellent,’ regardless its critical content. It might be worth noting here that not many staff

working in NUS (as far as I could gather in the years that I worked there) actually conceived of the 2002 shift to a ‘global knowledge enterprise’ as the demise of the ‘original’ university and its pursuit of ‘true knowledge for social justice’ —after all, in this university and its predecessors, the ideas of ‘truth and knowledge’ always already worked for Empire more overtly, while

European universities, and hence the Dutch ones as well, can and could bathe in the illusion of fostering ‘authentic’ progress and democracy by way of its ‘objective’ methods or dominant theories. The university’s instability mirrors the volatility of a capitalism marked by nonsustainability, a growing feminisation of poverty, the rise of a new global upper class, and highly mediated illusions of cybernetic mastery . Derrida hints at this, but also at ‘the’ university’s elusiveness, in “Mochlos, or: the Confl ict of the Faculties,” when he claims that he “would almost call

[the university] the child of an inseparable couple, metaphysics and technology” (1993, 15, emphasis mine). Both universities are typical examples of that university of metaphysics and technology, and yet they can never be completely that—they cannot fi nalise their authority towards a perfect utopia, just as I cannot fi nalise mine in my Singaporean and Dutch modules and curriculum reforms. As Derrida illustrates—pedagogue that he is—through his generous readings of Levinas, taking the

humanist myth of pure ethics and complete duty to ‘the’ students to its logical extreme paradoxically shows its structural limitations. It is at this impossible intersection of myth and its limitation that the particular moral imperative of thought and questioning, in all its complicity in contemporary violence, becomes once again possible in my classrooms. What happens beyond that is out of my ‘European’ hands and into the hands of the emergent Asian and Dutch speed elite. At the end of each teaching block or semester then, this teacher’s agony problematically sublimates into her hoping that her students, as they come to embody philosophy’s promise, may change things for the better.

Page 120: openev.debatecoaches.org€¦  · Web view1AC . Observation 1: If I could find the spot where truth echoesI would stand there and whisper memories of my children's future. I would

The alternative is a politics of refusal—we refuse the political terms on which the affirmative has posed the question of intervention into violence. Instead the alt posits fugitive study as an immanently black and queer refusal that enables us to live in the underground sociality that the aff’s investment into the liberal democratic surface of institutional life has foreclosed. The aff’s motion towards transparency, inclusion, recognition, and representation is a reaction against the alt’s gestures of opacity and refusal—a reaction designed to turn us towards the surface of the political. Study, as a mode of being-with and being-for one another, is the most unproductive task – it disorients the incorporative mechanisms of the 1ac’s curriculum in favor of an art of living beyond the AFFs professionalized tactics of inclusionHalberstam 2013 (Jack Halberstam, “The Wild Beyond: With and for the Undercommons” | Introduction to “The Undercommons: Fugitive Planning & Black Study” by Stefano Harney and Fred Moten, 2013. pp. 5-12) [NJ]

It ends with love, exchange, fellowship. It ends as it begins, in motion, in between various modes of being and belonging, and on the way to new economies of giving, taking, being with and for and it ends with a ride in a

Buick Skylark on the way to another place altogether. Surprising, perhaps, after we have engaged dispossession, debt, dislocation and violence. But not surprising when you have understood that the projects of “ fugitive planning and black study” are mostly about reaching out to fInd connection; they are about making common cause with the brokenness of being, a brokenness , I would venture to say, that is also blackness , that remains blackness, and will, despite all, remain broken because this book is not a prescription for repair. If we do not seek to fix what has been broken, then what? How do we resolve to live with brokenness, with being broke, which is also what Moten and Harney call “debt.” Well, given that debt is sometimes a history of giving, at other times a history of taking, at all times a history of capitalism and given that debt also signifies a promise of ownership but never delivers on that promise, we have to understand that debt is something that cannot be paid off. Debt, as Harney puts it, presumes a kind of individualized relation to a naturalized economy that is predicated upon exploitation . Can we have, he asks,

another sense of what is owed that does not presume a nexus of activities like recognition and acknowledgement, payment and gratitude. Can debt “become a principle of elaboration”? Moten links economic debt to the brokenness of being in the interview with Stevphen Shukaitis; he acknowledges that some debts should be paid, and that much is owed especially to black people by white people, and yet, he says: “I also know that what it is that is supposed to be repaired is irreparable. It can’t be repaired. The only thing we can do is tear this

shit down completely and build something new.” The undercommons do not come to pay their debts, to repair what has been broken, to fix what has come undone. If you want to know what the undercommons wants, what Moten and Harney want, what black people, indigenous peoples, queers and poor people want, what we (the “we” who cohabit in the space of the undercommons) want, it is this – we cannot be satisfied with the recognition and acknowledgement generated by the very system that denies a) that anything was ever broken and b) that we deserved to be the broken part; so we refuse to ask for recognition and instead we want to take apart, dismantle, tear down the structure that , right now, limits our ability to find each other, to see beyond it and to access the places that we know lie outside its walls .

We cannot say what new structures will replace the ones we live with yet , because once we have torn shit down, we will inevitably see more and see differently and feel a new sense of wanting and being and becoming . What we want after “the break” will be different from what we think we want before the break and both are necessarily different from the desire that issues from being in the break . Let’s come at

Page 121: openev.debatecoaches.org€¦  · Web view1AC . Observation 1: If I could find the spot where truth echoesI would stand there and whisper memories of my children's future. I would

this by another path. In the melancholic and visionary 2009 film version of Maurice Sandak’s Where The Wild Tings Are (1963), Max, the small seeker who leaves his room, his home, his family to find the wild beyond, finds a world of lost and lonely beasts and they promptly make him their king. Max is the first king the wild things have had whom they did not eat and who did not, in turn, try to eat them; and the beasts are the first grown things that Max has met who want his opinion, his judgment, his rule. Max’s power is that he is small while they are big; he promises

the beasts that he has no plans to eat them and this is more than anyone has ever promised them. He promises that he will find ways through and around and will “slip through cracks” and re-crack the cracks if they fill up. He promises to keep sadness at bay and to make a world with the wild creatures that “roared their terrible roars and gnashed their terrible teeth and rolled their terrible eyes and showed their terrible claws.” That Max fails to make the wild things happy or to save them or to make a world with them is less important than the fact that he found them and he recognized in them the end of something and potentially the path to an alternative to his world. The wild things were not the utopian creatures of fairy tales, they were the rejected and lost subjects of the world Max had left behind and, because he shuttles between the Oedipal land where his mother rules and the ruined world of the wild, he knows the parameters of the real – he sees what is included and what is left out and he is now able to set sail for another place, a place that is neither the home he left

nor the home to which he wants to return. Moten and Harney want to gesture to another place , a wild place that is not simply the leftover space that limns real and regulated zones of polite society ; rather, it is a wild place that continuously produces its own unregulated wildness. The zone we enter through Moten and Harney is ongoing and exists in the present and, as Harney puts it, “some kind of demand was already being enacted, fulfilled in the call itself.” While

describing the London Riots of 2011, Harney suggests that the riots and insurrections do not separate out “the request, the demand and the call” – rather, they enact the one in the other : “I think the call, in the way I would

understand it, the call, as in the call and response, the response is already there before the call goes out. You’re already in something .” You are already in it. For Moten too, you are always already in the thing that you call for and that calls you. What’s more, the call is always a call to dis-order and this disorder or wildness shows up in many places: in jazz , in improvisation , in noise . The disordered sounds that we refer to as cacophony will always be cast as “extra-musical,” as Moten puts it, precisely because we hear something in them that reminds us that our desire for harmony is arbitrary and in another world, harmony would sound incomprehensible. Listening to cacophony and noise tells us that there is a wild beyond to the structures we inhabit and that inhabit us. And when we are called to this other place, the wild beyond, “beyond the beyond” in Moten and Harney’s apt terminology, we have to give ourselves over to a certain kind of craziness. Moten reminds us that even as Fanon took an anti-colonial stance, he knew that it “looks crazy” but, Fanon, as a psychiatrist, also knew not to accept this organic division between the rational and the crazy and he knew that it would be crazy for him not to take that stance in a world that had assigned to him the role of the unreal, the primitive and the wild. Fanon, according to Moten, wants

not the end of colonialism but the end of the standpoint from which colonialism makes sense. In order to bring colonialism to an end then, one does not speak truth to power, one has to inhabit the crazy, nonsensical, ranting language of the other, the other who has been rendered a nonentity by colonialism . Indeed, blackness , for Moten and Harney by way of Fanon, is the willingness to be in the space that has been abandoned by colonialism, by rule, by order . Moten takes us there, saying of Fanon finally: “Eventually, I believe, he comes to believe in the world, which is to say the other world, where we inhabit and maybe even cultivate this absence, this place which shows up here and now, in the sovereign’s space and time, as absence, darkness, death, things which are not (as John Donne would say).” The path to the wild beyond is paved with refusal. In The Undercommons if we begin anywhere, we begin with the right to refuse what has been refused to you. Citing Gayatri Spivak, Moten and Harney call this refusal the “first right” and it is a game-changing kind of refusal in that it signals the refusal of the choices as offered . We can understand this refusal in terms that Chandan Reddy lays out in Freedom With Violence (2011) – for Reddy, gay marriage is the option that cannot be opposed in the ballot box. While we can circulate multiple critiques of gay marriage in terms of its institutionalization of intimacy, when you arrive at the ballot box, pen in hand, you only get to check “yes” or “no” and the no, in this case, could be more damning than the yes. And so,

you must refuse the choice as offered. Moten and Harney also study what it would mean to refuse what they term “the call to order.” And what would it mean, furthermore, to refuse to call others to order, to refuse interpellation and the re- instantiation of the law. When we refuse, Moten and Harney suggest, we create dissonance and more importantly, we allow dissonance to continue – when we enter a classroom and we refuse to call it to order, we are allowing study to continue , dissonant study perhaps, disorganized study, but study that

Page 122: openev.debatecoaches.org€¦  · Web view1AC . Observation 1: If I could find the spot where truth echoesI would stand there and whisper memories of my children's future. I would

precedes our call and will continue after we have left the room. Or, when we listen to music, we must refuse the idea that music happens only when the musician enters and picks up an instrument; music is also the anticipation of the performance and the noises of appreciation it generates and the speaking that happens through and around it, making it and loving it, being in it while listening . And so, when we refuse the call to order – the teacher picking up the book, the conductor raising his baton, the speaker asking for silence,

the torturer tightening the noose – we refuse order as the distinction between noise and music, chatter and knowledge, pain and truth. These kinds of examples get to the heart of Moten and Harney’s world of the undercommons – the undercommons is not a realm where we rebel and we create critique; it is not a place where we “take arms against a sea of troubles/and by opposing end them.” The undercommons is a space and time which is always here. Our goal – and the “we” is always the right mode of address here – is not to end the troubles but to end the world that created those particular troubles as the ones that must be opposed. Moten and Harney refuse the logic that stages refusal as inactivity, as the absence of a plan and as a mode of stalling real politics. Moten and Harney tell us to listen to the noise we make and to refuse the offers we receive to shape that noise into “music.” In the essay that many people already know best from this volume, “The University and the Undercommons,” Moten and Harney come closest to

explaining their mission. Refusing to be for or against the university and in fact marking the critical academic as the player who holds the “for and against” logic in place, Moten and Harney lead us to the “Undercommons of the Enlightenment” where subversive intellectuals engage both the university and fugitivity: “where the work gets done, where the work gets subverted, where the revolution is still black, still strong.” Te subversive intellectual, we learn, is unprofessional, uncollegial, passionate and disloyal. The subversive intellectual is neither trying to extend the university nor change the university, the subversive intellectual is not toiling in misery and from this place of

misery articulating a “general antagonism.” In fact, the subversive intellectual enjoys the ride and wants it to be faster and wilder; she does not want a room of his or her own, she wants to be in the world, in the world with others and making the world anew . Moten insists: “Like Deleuze. I believe in the world and want to be in it. I want to be in it all the way to the end of it because I believe in another world in the world and I want to be in that. And I plan to stay a believer , like Curtis Mayfeld. But that’s beyond me, and even beyond me and Stefano, and out into the world, the other thing, the other world, the joyful noise of the scattered, scatted eschaton, the undercommon refusal of the academy of misery .” The mission then for the denizens of the undercommons is to recognize that when you seek to make things better, you are not just doing it for the Other, you must also be doing it for yourself . While men may think they are being “sensitive” by

turning to feminism, while white people may think they are being right on by opposing racism, no one will really be able to embrace the mission of tearing “this shit down” until they realize that the structures they oppose are not only bad for some of us, they are bad for all of us. Gender hierarchies are bad for men as well as women and they are really bad for the rest of us. Racial hierarchies are not rational and ordered, they are chaotic and nonsensical and must be opposed by precisely all those who benefit in any way from them. Or, as Moten puts it: “The coalition emerges out of your recognition that it’s fucked up for you, in the same way that we’ve already recognized that it’s fucked up for us. I don’t need your help. I just need you to recognize that this shit is killing you, too, however much more softly, you stupid motherfucker, you know?” The coalition unites us in the recognition that we must change things or die . All of us. We must all change the things that are fucked up and change cannot come in the form that we think of as “revolutionary” – not as a masculinist surge or an armed confrontation. Revolution will come in a form we cannot yet imagine.

Moten and Harney propose that we prepare now for what will come by entering into study. Study, a mode of thinking with others separate from the thinking that the institution requires of you, prepares us to be embedded in what Harney calls “ the with and for” and allows you to spend less time antagonized and antagonizing. Like all world-making and all world-shattering encounters, when you enter this book and learn how

Page 123: openev.debatecoaches.org€¦  · Web view1AC . Observation 1: If I could find the spot where truth echoesI would stand there and whisper memories of my children's future. I would

to be with and for, in coalition, and on the way to the place we are already making, you will also feel fear, trepidation, concern, and

disorientation. The disorientation , Moten and Harney will tell you is not just unfortunate, it is necessary because you will no longer be in one location moving forward to another, instead you will already be part of “the “movement of things” and on the way to this “outlawed social life of nothing.” The movement of things can be felt and touched and exists in language and in fantasy , it is fight, it is motion, it is fugitivity itself. Fugitivity is not only escape , “exit ” as Paolo Virno might put it, or “exodus” in the terms offered by

Hardt and Negri, fugitivity is being separate from settling. It is a being in motion that has learned that “organizations are obstacles to organising ourselves” (The Invisible Committee in The Coming Insurrection) and that there are spaces and modalities that exist separate from the logical, logistical, the housed and the positioned . Moten and Harney call this mode a “being together in homelessness” which does not idealize homelessness nor merely metaphorize it. Homelessness is the state of dispossession that we seek and that we embrace: “Can this being together in homelessness, this interplay of the refusal of what has been refused, this undercommon appositionality, be a place from which emerges neither self- consciousness nor knowledge of the other but an improvisation that proceeds from somewhere on the other side of an unasked question?” I think this is what Jay-Z and Kanye West (another collaborative unit of study) call “no church

in the wild.” For Fred Moten and Stefano Harney, we must make common cause with those desires and (non)- positions that seem crazy and unimaginable: we must, on behalf of this alignment, refuse that which was first refused to us and in this refusal reshape desire, reorient hope, reimagine possibility and do so separate from the fantasies nestled into rights and respectability. Instead, our fantasies must come from what

Moten and Harney citing Frank B. Wilderson III call “the hold”: “ And so it is we remain in the hold, in the break, as if entering again and again the broken world, to trace the visionary company and join it.” The hold here is the hold in the slave ship but it is also the hold that we have on reality and fantasy, the hold they have on us and the hold we decide to forego on the other, preferring instead to touch, to be with, to love. If there is no church in the wild, if there is study rather than knowledge production, if

there is a way of being together in brokenness, if there is an undercommons, then we must all find our way to it. And it will not be there where the wild things are, it will be a place where refuge is not necessary and you will find that you were already in it all along.

Page 124: openev.debatecoaches.org€¦  · Web view1AC . Observation 1: If I could find the spot where truth echoesI would stand there and whisper memories of my children's future. I would

Vs. Buffalo Soldiers

Page 125: openev.debatecoaches.org€¦  · Web view1AC . Observation 1: If I could find the spot where truth echoesI would stand there and whisper memories of my children's future. I would

Vs. Brazil FemThe 1AC relies on a futural hope of a “not-yet-realized” future in which they can fantasize about contingent solutions that never come. This hope creates a cruel optimism for black folk that forces them to invest in the pursuit of our own death because it crowds out all non-politically recognizable alternatives. This model for politics only re-entrenches the anti-black world and is an independent reason to reject the aff. Warren 15 (Calvin L., Black Nihilism and the Politics of Hope; Surce: CR: The New Centennial Review, Vol. 15, No. 1, Derrida and French Hegelianism (Spring 2015), XMT, pp. 215-248 Published by: Michigan State University Press Stable URL: http://www.jstor.org/stable/10.14321/crnewcentrevi.15.1.0215 . Accessed: 30/03/2015)

The politics of hope , then, constitutes what Lauren Berlant would call “ cruel optimism” for blacks (Berlant 2011). It bundles certain promises about redress, equality, freedom, justice, and progress into a political object that always lies beyond reach. The objective of the Political is to keep blacks in a relation to this political object—in an unending pursuit of it. This pursuit, however, is detrimental because it strengthens the very anti-black system that would pulverize black being . The pursuit of the object certainly has an “irrational” aspect to it, as Farred details, but it is not mere means without expectation; instead, it is a means that

undermines the attainment of the impossible object desired. In other words, the pursuit marks a cruel attachment to the means of subjugation and the continued widening of the gap between historical reality and fantastical ideal. Black nihilism is a “demythifying” practice, in the Nietzschean vein, that uncovers the subjugating strategies of

political hope and de-idealizes its fantastical object. Once we denude political hope of its axiological and ethical veneer, we see that it operates through certain strategies: 1) positing itself as the only alternative to the problem of anti-blackness, 2) shielding this alternative [End Page 221] from rigorous historical/philosophical critique by placing it in an unknown

future, 3) delimiting the field of action to include only activity recognized and legitimated by the Political, and 4) demonizing critiques or different philosophical perspectives. The politics of hope masks a particular cruelty under the auspices of “happiness” and “life.” It terrifies with the dread of “no alternative .” “Life” itself needs the security of the alternative, and, through this logic, life becomes untenable without it.

Political hope promises to provide this alternative—a discursive and political organization beyond extant structures of violence and destruction. The construction of the binary “alternative/no-alternative” ensures the hegemony and dominance of political hope within the onto-existential horizon. The terror of the “no alternative”—the ultimate space of decay, suffering, and death—depends on two additional binaries: “problem/solution” and “action/inaction.” According to this politics, all problems have solutions, and hope provides the accessibility and realization of these solutions. The solution establishes itself as the elimination of “the problem”; the solution, in fact, transcends the problem and realizes Hegel’s aufheben in its constant attempt to sublate the dirtiness of the “problem” with the pristine being of the

solution. No problem is outside the reach of hope’s solution—every problem is connected to the kernel of its own eradication. The politics of hope must actively refuse the possibility that the “solution” is, in fact, another problem in disguised form; the idea of a “solution” is nothing more than the repetition and disavowal of the problem itself. The solution relies on what we might call the “trick of time”

to fortify itself from the deconstruction of its binary. Because the temporality of hope is a time “not-yet-realized,” a future tense unmoored from present-tense justifications and pragmatist evidence, the politics of hope cleverly shields its “solutions” from critiques of impossibility or repetition. Each insistence that these solutions stand up against the lessons of history or the rigors of analysis is met with the rationale that these solutions are not subject to history or analysis because they do not reside

Page 126: openev.debatecoaches.org€¦  · Web view1AC . Observation 1: If I could find the spot where truth echoesI would stand there and whisper memories of my children's future. I would

within the horizon of the “past” or “present.” Put differently, we can never ascertain the efficacy of the proposed solutions because they escape the temporality of the moment, always retreating to a “not-yet” and “could-be” temporality. This “trick” of time offers a promise of possibility that can only be realized in an indefinite future, and this promise is a bond of uncertainty that can never be redeemed, only imagined. In this sense, the politics of hope is an instance of the psychoanalytic notion of desire: its sole

purpose is to reproduce its very condition of possibility, never to satiate or bring fulfillment. This politics secures its hegemony through time by claiming the future as its unassailable property and excluding (and

devaluing) any other conception of time that challenges this temporal ordering. The politics of hope, then, depends on the incessant (re)production and proliferation of problems to justify its existence. Solutions cannot really exist within the politics of hope, just the illusion of a different order in a future tense . The “trick” of time and political solution converge on the site of “action.” In critiquing the politics of hope, one encounters the rejoinder of the dangers of inaction. “But we can’t just do nothing! We have to do something.” The field of permissible action is delimited and an unrelenting binary between action/ inaction silences critical engagement with political hope. These exclusionary operations rigorously

reinforce the binary between action and inaction and discredit certain forms of engagement, critique, and protest. Legitimate action takes place in the political—the political not only claims futurity but also action as its property. To “do something” means that this doing must translate into recognizable political activity; “something” is a stand-in for the word “politics”—one must “do politics” to address any problem. A refusal to “do politics” is equivalent to “doing nothing”—this nothingness is constructed as the antithesis of life, possibility, time, ethics, and morality (a “zero-state” as Julia Kristeva [1982] might call it).

Black nihilism rejects this “trick of time” and the lure of emancipatory solutions. To refuse to “do politics” and to reject the fantastical object of politics is the only “hope” for blackness in an antiblack world .

Narratives of Black suffering, no matter the radical form or intent, will always reify White innocence and recreate Black powerlessness. Hartman and Wilderson ’03 (Saidiya Hartman, Saidiya Hartman is a professor at Columbia University specializing in African American literature and history, Frank Wilderson III, Professor of African American Studies at UC Irvine, Source: Qui Parle, Vol. 13, No. 2 (Spring/Summer 2003), pp. 183-201, Published by: Duke University Press, Stable URL: https://www.jstor.org/stable/20686156, “Position of the Unthought”) \\EG

Frank B. Wilderson, Ill- One of the first things 1 want to say is how thankful I am that you wrote Scenes of Subjection: Terror, Slavery, and Self-Making in Nineteenth-Century America. And I want to say a little bit about how meaningful the book is to me as a black grad uate student - a so-called aspiring academic - and as someone caught in the machine but not of it. Because in general, when one reads the work of black scholars - if one is another black scholar or a black student - one prepares

oneself for a disappointment, or works a disappointment into the reading. And one doesn't have to do that with this particular book. What I mean, is that so often in black scholarship, people consciously or unconsciously peel away from the strength and the terror of their evidence in order to propose some kind of coherent, hopeful solution to things. Your book, in moving through these scenes of subjection as they take place in slavery, refuses to do that. And just as importantly, it does not allow the reader to think that there was a radical enough break to reposition the black body after Jubilee.1 That is a tremendous and courageous move. And I think what's important about it, is that it corroborates the experience of ordinary black people today, and of strange black people like you and me in the academy [Iaughter]. But there's something else that the book does, and I want to talk about this at the level of methodology and analysis. If we think about the registers of subjectivity as being preconscious interest, unconscious identity or identifications, and positionality, then a lot of the work in the social sciences organizes itself around preconscious interest; it assumes a subject of consent, and as you have said, a subject of exploitation, which you reposition as the subject of accumulation.2 Now when this sort of social science engages the issue of positionality - if and when it does - it assumes that it can do so in an un-raced manner. That's the best of the work. The worst of the work is a kind of multiculturalism that assumes we all have analogous identities that can be put into a basket of stories, and then that basket of stories can lead to similar interests. For me, what you've done in this book is to split the hair here. In other words, this is not a book that celebrates an essential Afrocentrism that could be captured by the multicultural discourse. And yet it's not a book that remains on the surface of preconscious interest, which so much history and social science does. Instead, it demands a radical racialization of any analysis of positionality. So. Why don't we talk about that? Saidiya V Hartman - Well! That's a lot, and a number of

things come to mind. I think for me the book is about the problem of craft ing a narrative for the slave as subject, and in terms of positionali ty, asking, "Who does that narrative enable?" That's where the whole issue of empathic identification is central for me. Because it just seems that every attempt to employ the slave in a narrative ultimately resulted in his or her obliteration, regardless of whether it was a leftist narrative of political agency - the slave stepping into someone else's shoes and then becoming a political agent - or whether it was about being able to unveil the slave's humanity by actually finding oneself in that position. In many ways, what I was trying to do as a cultural historian was

Page 127: openev.debatecoaches.org€¦  · Web view1AC . Observation 1: If I could find the spot where truth echoesI would stand there and whisper memories of my children's future. I would

to narrate a certain impossibility, to illuminate those practices that speak to the limits of most available narratives to explain the position of the enslaved. On one

hand, the slave is the foundation of the national order, and, on the other, the slave occupies the position of the unthought . So what does it mean to try to bring that position into view without making it a locus of positive value, or without trying to fill in the void? So much of our political vocabulary/imaginary/desires have been implicitly integrationist even when we imagine our claims are more radical. This goes to the sec ond part of the book - that ultimately the metanarrative thrust is always towards an integration into the national project, and particularly when that project is in crisis, black people are called upon to affirm it . So certainly it's

about more than the desire for inclusion with in the limited set of possibilities that the national project provides. What then does this language - the given language of freedom - enable? And once you realize its limits and begin to see its inexorable investment in certain notions of the subject and subjection, then that language of freedom no longer becomes that which rescues the slave from his or her former condition, but the site of the re-elaboration of that condition , rather than its transformation. F.W - This is one of the reasons why your book has been called "pessimistic" by Anita Patterson.3 But it's interesting that she does n't say what I said when we first started talking, that it's enabling. I'm assuming that she's white - I don't know, but it certainly

sounds like it. S.VH. - But I think there's a certain integrationist rights agenda that subjects who are variously positioned on the color line can take up. And that project is something I consider obscene: the attempt to make the narrative of defeat into an opportunity for celebration , the desire to look at the ravages and the brutality of the last few centuries, but to still find a way to feel good about ourselves . That's not my project at all, though I think it's actually the project of a number of people.

Unfortunately, the kind of social revisionist history undertaken by many leftists in the 1970s , who were trying to locate the agency of dominated groups, resulted in celebratory narratives of the oppressed .4 Ultimately, it bled into this celebration, as if there was a space you could carve out of the ter rorizing state apparatus in order to exist outside its clutches and forge some autonomy. My project is a different one. And in partic ular, one of my hidden polemics in the book was an argument against the notion of hegemony, and how that notion has been taken up in the context of looking at the status of the slave. F W - That's very interesting, because it's something I've been thinking about also in respect to Gramsci. Because Anne Showstack Sassoon suggests that Gramsci breaks down hegemony into three categories: influence, leadership, and consent.5 Maybe we could bring the discussion back to your text then, using the examples of Harriet Jacobs,6 a slave, and John Rankin,7 a white anti-slavery Northerner, as ways in which to talk about this. Now, what's really interesting is that in your chapter "Seduction and the Ruses of Power," you not only explain how the positional

ity of black women and white women differs, but you also suggest how blackness disarticulates the notion of consent , if we are to think of

that notion as universal. You write: "[B]eing forced to submit to the will of the master in all things defines the predicament of slavery" (S, 110). In other words, the female slave is a possessed, accumulated, and fungible object, which is to say that she is ontologically different than a white woman who may, as a house servant or indentured labor er, be a subordinated subject.

You go on to say, "The opportunity for nonconsent [as regards, in this case, sex] is required to establish consent , for consent is meaningless if refusal is not an option.... Consent is unseemly in a context in which the very notion of subjectivity is predicated upon the negation of will " (S, 111). S. V.H. - Once again, trying to fit

into the other's shoes becomes the very possibility of narration. In the chapter "A Perilous Passage in the Slave Girl's Life," the question for Jacobs is how she can tell her story in a way that's going to solicit her white readership when she has to efface her very condition in order to make that story intelligible to them. I look at this messy moment as kind of a vortex in Jacobs' narrative, where in order to fashion herself as a desiring subject, she has to deny the very violence, which elsewhere she said defines her position as a slave: her status as a thing and the negation of her will. In one sense, she has to bracket that so she can tell a story about sexuality that's meaningful in a white dominant frame. And I think this is why someone like Hortense Spillers raises the question of whether gender and sexuality are at all applicable to the condition of the captive community.8 That's what I was working with there, that impossibility or ten sion between Jacobs as an agent versus the objective

conditions in which she finds herself. This is something you talk about in your work as well, this existence in the space of death, where negation is the captive's central possibility for action, whether we think of that as a radical refusal of the terms of the social order or these acts that are

sometimes called suicide or self-destruction, but which are real ly an embrace of death. Ultimately it's about the paradox of agency for those who are in these extreme circumstances. And basically, there are very few political narratives that can account for that. F.W -And we have to ask why. In my own work, obviously I'm not saying that in this space of negation, which is blackness, there is no life. We have tremendous life. But this life is not analogous to those touchstones of cohesion that hold civil society together. In fact, the trajectory of our life (within our terrain of civil death) is bound up in claiming - sometimes individually, sometimes collectively - the violence which Fanon writes about in The Wretched of the Earth, that trajectory which, as he says, is "a splinter to the heart of the world"9 and "puts the settler out of the picture."10 So, it doesn't help us politically or

psychologically to try to find ways in which how we live is analogous to how white positionality lives, because, as I think your book suggests, whites gain their coherence by knowing what they are not . There is tremendous diversity on the side of whiteness and tremendous conflict between white men and white women, between Jews and gentiles, and between classes, but that conflict, even in its articulation, has a certain solidarity. And I think that sol idarity comes from a near or far relation to the black body or bod ies. We give the nation its coherence because we're its underbelly. That's what's so interesting for me about Achille Mbembe's work, the way he thinks about the position of the for merly colonized

Page 128: openev.debatecoaches.org€¦  · Web view1AC . Observation 1: If I could find the spot where truth echoesI would stand there and whisper memories of my children's future. I would

subject along the lines of the slave as an essential way of defining the predicament. Essentially, he says, the slave is the object to whom anything can be done,

whose life can be squandered with impunity.12 F.W. - And he's suggesting that what it means to be a slave is to be subject to a kind of complete appropriation, what you call "property of enjoyment." Your book illustrates the "myriad and nefarious uses of slave property" and then demonstrates how "there was no relation to blackness outside the terms of this use of, entitlement to, and occupation of the captive body, for even the status of free blacks was shaped and compromised by the existence of slavery" (S, 24). So. Not only are formally enslaved blacks proper ty, but so are formally free blacks. One could say that the possibil ity of becoming property is one of the essential elements that draws the line between blackness and whiteness. But what's most intrigu ing about your argument is the way in which you demonstrate how

not only is the slave's performance (dance, music, etc.) the property of white enjoyment , but so is - and this is really key

- the slave's own enjoyment of his/her performance: that too belongs to white people .13 S. VH. - Right. You know, as I was writing Scenes of Subjection, there was a whole spate of books on nineteenth-century culture and on minstrelsy in particular. And there was a certain sense

in which the ability to occupy blackness was considered transgressive or as a way of refashioning whiteness, and there were all these rad ical claims that were being made for it.14 And I thought, "Oh, no, this is just an extension of the master's prerogative." It doesn't mat ter whether you do good or you do bad, the crux is that you can choose to do what you wish with the black body . That's why think ing about the dynamics of enjoyment in terms of the material rela tions of slavery was so key for me. F.W -Yes, that's clarifying. A body that you can do what you want with. In your discussion of the body as the

property of enjoyment, what I really like is when you talk about Rankin. Here's a guy - like the prototypical twentieth-century white progressive - who's anti-slavery and uses his powers of observation to write for its abo lition, even to his slave-owning brother. He's in the South, he's looking at a slave coffle, and he imagines that these slaves being beaten could be himself and his family. Through this process it makes sense to him, it becomes meaningful. His body and his fam ily members' white bodies become proxies for real enslaved black bodies and, as you point out, the actual object of identification, the slave, disappears . S.V.H. - I think that gets at one of the fundamental ethical ques tions/problems/crises for the

West: the status of difference and the status of the other. It's as though in order to come to any recognition of common humanity, the other must be assimilated , meaning in this case, utterly displaced and effaced: "Only if I can see myself in that position can I understand the crisis of that position." That is the logic of the moral and political discourses we see everyday - the need for the innocent black subject to be victimized by a racist state in order to see the racism of the racist state. You have to be exemplary in your goodness, as opposed to .. . F.W. - [laughter] A nigga on the warpath! S. V.H. - Exactly! For me it was those moments that were the most telling - the moments of the sympathetic ally, who in some ways is actually no more able to see the slave than the person who is exploiting him or her as their property. That is the work Rankin does and I think it suggests just how ubiquitous that kind of vio lence, in fact, is. F.W - You've just thrown something into crisis, which is very much on the table today: the notion of allies. What you've said (and I'm so happy that someone has come along to say it!) is that the ally is not a stable category. There's a structural prohibition (rather than merely a willful refusal) against whites being the allies of blacks, due to this - to borrow from Fanon's The Wretched of the Earth again - "species" division between what it means to be a subject and what it means to be an object: a structural antago nism. But everything in the academy on race works off of the ques tion, "How do we help white allies?" Black academics assume that there is enough of a structural commonality between the black and the white (working class) position - their mantra being: "We are both exploited subjects" - for one to embark upon a political pedagogy that will somehow help whites become aware of this "com monality." White writers posit the presence of something they call "white skin privilege," and the possibility of "giving that up," as their gesture of being in solidarity with blacks. But what both gestures disavow is that subjects just can't make common cause with objects. They can only become objects, say in the case of John Brown or Marilyn Buck, or further instantiate their subjectivity through modalities of violence (lynching and the prison industrial complex), or through modalities of empathy. In other words, the essential essence of the white/black relation is that of the master/slave - regardless of its historical or geographic specificity. And masters and slaves, even today, are never allies. S.V.H. - Right. I think of the book as an allegory; its argument is a history of the present. F.W - Thank you! I'm so glad you said it's an allegory of the pre sent. Because now we've got two problems on the table, two crises - or rather, we have many crises, but only two that I can identify at the moment. One is how we deal with the common sense around allies, whether it be in teaching literature to undergraduates or going to hear Cornel West speak with Michael Lerner, or listen ing to KPFA, since, in point of fact, it may be that the progressive community is actually as big an enemy to black revolution as Newt Gingrich. And the other I could put as, "How do you go to the movies?" How does one, knowing what one knows, sit through anything? Because it seems like every film - if it is in any way going to communicate some type of empathy that the audience can walk away with - has to have black death as its precondition. S.VH. - Yes, yes. Monster's Ball is a great example.15 Not only is Leticia's husband executed, but her son must also die as the pre condition for her new life with her husband's executioner. And the death requirement is rendered as a romance. Rather than closing with a note of ambivalence, the film actually ends with her smiling over the romantic music, as if to suggest that she's gotten over it, and the future awaits them. And I think that is the frightening hypocrisy of the context we are living in. There's also the film Unfaithful where the lover has to be mur dered in order to protect the heterosexual family.16 The white bour geois family can actually live with murder in order to reconstitute its domesticity. F.W - Well, why does white supremacy seem to be so bound up in the visual? S.VH. - I think that visually, the threat of blackness is somehow heightened. Fanon's "Look! A Negro": that's the formulation, and within the racial classificatory schema that is how much of the work is done, especially in terms of the way racialization has oper ated: how it disposes of bodies, how it appropriates their products, and how it fixes them in a visual grid. I think those are the three ways I would explore that problem, as well as, again, this whole dimension of the empathic. F.W - One of the things I wanted to bring up is how your book is talking to other very important books. It's talking to Fanon as you've said, and it's talking to Patterson's Slavery and Social Death.17 And you talked about the leftist discourses of the '70s, and the univer salizing of Gramscian hegemony that really falls short of helping us understand a position in civil society, but not of civil society. It has to do, I think, with how the idiom of power that black people expe rience has different kinds of manifestations as we move from slav ery into the era of the Freedmen's Bureau, but there's an umbrella of despotism that remains. And when you suggested earlier that the book is an allegory of the present, it was so refreshing, because one can read this book and begin to metaphorize the manifestations of despotism in the past, and also to think about how it continues in the present. S.VH. - It really is the pressing question of freedom. That's why for me, the last lines of the book summon up that moment of poten tiality between the no longer and the not yet. "Not yet free": that articulation is from the space of the twenty-first century, not the nineteenth, and that's the way it's supposed to carry - the same predicament, the same condition. FW. -And in those terms we might think about how Rodney King was accused of inviting his own beating; you know, he shook his ass in an aggressive manner at a white woman. So maybe you could sketch out the way in which the black woman functions sim ilarly in slavery, as somehow outside the statutory, or inside it: she cannot be raped because she's a non-person yet she is presumed to invite the rapist. S. VH. - Yes. No crime can occur because the slave statutes rec ognize no such crime. Often when I'm looking through the crimi nal record of the nineteenth century, I'm seeing the text of black agency. The people who are resisting their masters and overseers appear in the records as they're prosecuted for their crime, creating this displacement of culpability that enables white innocence. In the case of State of Missouri v. Celia (1855), Celia is raped repeat edly by her owner from the moment she's purchased. She begs him to stop; he doesn't, so she kills him. Her crime is the crime on record: she is the culpable agent.18 So in this formulation of law and its punishment, blackness is on the side of culpability, which makes the crimes of property transparent and affirms the rights to property in captives. And you're right, that displacement functions more generally. Who is the responsible and culpable agent? For the most part, it's always the slave, the native, the black. F.W - Which brings your allegory of the present to the prison industrial complex. S.VH. - Actually, I've got an interesting tidbit. I think that Den mark Vesey was the first person ever imprisoned in the South Carolina Penitentiary. F.W - Really? It's like a seamless transition from slavery to prison. S.V.H. - Right. And this is where the larger narrative of capitalism comes into play. Because, basically, in most places in the world, you have a transition from slavery to other modes of involuntary servitude. In my work, I critique the received narrative about the transition from slavery to freedom in the American context, but we could also look at that same kind of transformation in relation to the anti-slavery rhetoric that comes to legitimize the colonial pro ject in Africa. By the nineteenth century, slavery was the dominant mode of production in West Africa. Eventually, the European nations decided "This is an awful institution and we need to stop it," so we get King Leopold masking his atrocities in the Congo in the discourse of anti-slavery, or British colonial figures in Ghana effectively saying, "Well, we saved you from the slave raider so you should be grateful."19 In both cases, it's the same notion: "We've given you your freedom, so now you're in our debt." F.W - And that brings us to Reconstruction in your book where you're talking about post-jubilee: The good conduct encouraged by such counsels eased the transition from slavery to freedom by imploring the freed to continue in old forms of subservience, which primarily entailed

Page 129: openev.debatecoaches.org€¦  · Web view1AC . Observation 1: If I could find the spot where truth echoesI would stand there and whisper memories of my children's future. I would

remaining on the plantation as faith ful, hardworking, and obedient laborers, but also included manners, styles of comportment in work rela tions, objects of consumption, leisure, and domestic relations. In their emphasis on proper conduct, these schoolbooks resuscitated the social roles of slavery, not unlike the regulation of behavior in labor contracts or the criminalization of impudence in the Black Codes. The pedagogical injunctions to obedience and servility cast the freed in a world starkly similar to the one in which they had suffered under slavery. On the one hand, these texts heralded the natural rights of all men; and on the other, they advised blacks to refrain from enjoying this newly conferred equality. Despite proclamations about the whip's demise, emergent forms of involuntary servitude, the coercive control of black labor, the repressive instrumentality of the law, and the social intercourse of everyday life revealed the entan glements of slavery and freedom. (S, 151) So. There's this whole army of white people - missionaries, edu cators, and the like - who go down South to help rehabilitate the Negro after slavery. And in reading that, a wave of cynicism swept over me, because all of a sudden I thought of Freedom Summer, and the white students in SNCC, which is a blasphemous thought to have.

The alternative is black nihilism --- refusing political hope is the only metaphysically coherent response to the constant slaughter of black bodies Warren 15 [Calvin K., Assistant Professor of American Studies at George Washington University, “Black Nihilism and the Politics of Hope,” CR: The New Centennial Review, Volume 15, Number 1, Spring 2015]

V. Conclusion Throughout this essay, I have argued that the Politics of hope preserve metaphysical structures that sustain black suffering. This preservation amounts to an exploitation of hope—when the Political colonizes the spiritual principle of hope and puts it in the service of extending the “will to power” of an anti-black organization of existence. The Politics of hope, then, is bound up with metaphysical violence, and

this violence masquerades as a “solution” to the problem of anti-blackness. Temporal linearity, perfection, betterment, struggle, work, and utopian futurity are conceptual instruments of the Political that will never obviate black suffering or anti-black violence; these concepts only serve to reproduce the conditions that render existence unbearable for blacks . Political theologians and black optimists avoid the immediacy of black suffering, the horror of anti-black pulverization, and place relief in a “not-yet-but-is (maybe)-to-come-social order” that, itself,

can do little more but admonish blacks to survive to keep struggling . Political hope becomes a vicious and abusive cycle of struggle —it mirrors the Lacanian drive, and we encircle an object (black freedom, justice, relief, redress,

equality, etc.) that is inaccessible because it doesn’t really exist. The political theologian and black optimist, then, propose a collective Jouissance as an answer to black suffering—finding the joy in struggle, the victory in toil, and the

satisfaction in inefficacious action. We continue to “struggle” and “work” as black youth are slaughtered daily, black bodies are incarcerated as forms of capital, black infant mortality rates are soaring, and hunger is disabling the bodies, minds, and spirits of desperate black youth. In short, these conditions are deep metaphysical problems—the sadistic pleasure of metaphysical domination—and “work ” and “struggle” avoid the terrifying fact that the world depends on black death to sustain itself . Black nihilism attempts to break this “drive”—to stop it in its tracks, as it were—and to end the cycle of insanity that political hope perpetuates. The question that remains is a question often put to the black nihilist: what is the point? This compulsory geometrical structuring of thought— all knowledge must submit to, and is reducible to, a point —it is an epistemic flicker of certainty, determination, and, to put it bluntly, life. “The point” exists for life; it enlivens, enables, and sustains knowledge. Thought outside of this mandatory point is illegible and useless. To write outside of the “episteme of life” and its grammar will require a position outside of this point , a position somewhere in the infinite horizon of thought (perhaps this is what Heidegger wanted to do with his reconfiguration of thought). Writing in this way is inherently subversive and refuses the geometry of thought. Nevertheless, the [End Page 243] nihilist is forced to enunciate his refusal through a “point,” a point that is contradictory and paradoxical all at once. To say that the point of this essay is that “the point” is fraudulent—its promise of clarity and life are inadequate—will not satisfy the hunger of disciplining the nihilist and insisting that one

undermine the very ground upon which one stands. Black nihilistic hermeneutics resists “the point ” but is subjected to it to have one’s voice heard within the marketplace of ideas. The “point” of this essay is that political hope is pointless . Black suffering is an essential part of the world, and placing hope in the very structure that sustains metaphysical violence , the Political, will never resolve anything . This is why the black

nihilist speaks of “exploited hope,” and the black nihilist attempts to wrest hope from the clutches of the Political . Can we think of hope outside the Political? Must “salvation” translate into a political grammar or a political program? The nihilist , then, hopes for the end of political hope and its metaphysical violence. Nihilism is not antithetical to hope; it does not extinguish hope but reconfigures it. Hope is the foundation of the black nihilistic hermeneutic. In “Blackness and Nothingness,” Fred Moten (2013) conceptualizes

Page 130: openev.debatecoaches.org€¦  · Web view1AC . Observation 1: If I could find the spot where truth echoesI would stand there and whisper memories of my children's future. I would

blackness as a “pathogen” to metaphysics, something that has the ability to unravel, to disable, and to destroy anti-blackness. If we read Vattimo through Moten’s brilliant analysis, we can suggest that blackness is the limit that Heidegger and Nietzsche were really after. It is a “blackened” world that will ultimately end metaphysics, but putting an end to metaphysics will also put an end to the world itself—this is the nihilism that the black nihilist must theorize through. This is a far cry from what we call “anarchy,” however. The black nihilist has as little faith in the metaphysical reorganization of society through anarchy than he does in traditional forms of political

existence. The black nihilist offers political apostasy as the spiritual practice of denouncing metaphysical violence, black suffering, and the idol

of anti-blackness. The act of renouncing will not change political structures or offer a political program ; instead,

it is the act of retrieving the spiritual concept of hope from the captivity of the Political . Ultimately, it is impossible to end metaphysics without ending blackness, and the black nihilist will never be able to withdraw from the Political completely without a certain death-drive or being-toward-death. This is the essence of black suffering : the lack of reprieve from metaphysics, the tormenting complicity in the reproduction of violence, and the lack of a coherent grammar to articulate these dilemmas . After contemplating these issues for some time in my office, I decided to take a train home. As I awaited my train in the station, an older black woman asked me about the train schedule and when I would expect the next train headed toward Dupont Circle. When I told her the trains

were running slowly, she began to talk about the government shutdown. “They don’t care anything about us, you know,” she

said. “We elect these people into office, we vote for them, and they watch black people suffer and have no intentions of doing anything about it.” I shook my head in agreement and listened intently. “I’m going to stop voting,

and supporting this process; why should I keep doing this and our people continue to suffer,” she said. I looked at her

and said, “I don’t know ma’am; I just don’t understand it myself.” She then laughed and thanked me for listening to her—as if our conversation were somewhat cathartic. “You know, people think you’re crazy when you say things like this,” she said giving me a wink. “Yes they do,” I said. “But I am a free woman,” she emphasized “and I

won’t go back.” Shocked, I smiled at her, and she winked at me; at that moment I realized that her wisdom and courage penetrated my mind and demanded answers. I’ve thought about this conversation for some time, and it is for this reason I had

to write this essay. To the brave woman at the train station, I must say you are not crazy at all but thinking outside of metaphysical time, space, and violence . Ultimately, we must hope for the end of political hope.

NEXT OFFThe prefacing of representations of suffering as method to achieve justice and alleviate the violence done to victims is part and parcel of a fetishistic politics which herald injury as the center of political subjectivity. The reduction of another’s suffering to an image to be consumed reduces them to a fetishized object to be violently traded for ballots.Abbas 2010

/Asma, Professor and Division Head in Social Studies, Political Science, Philosophy at the Liebowitz Center for International Studies at Bard College at Simon’s Rock, Liberalism and Human Suffering: Materialist Reflections on Politics, Ethics, and Aesthetics, London: Palgrave Macmillan, pg. Pg. 121- 123/

An inquiry into injury as value and commodity, and the attempted recovery of a materialist method that can speak to the laboring and

suffering body in liberal capitalism, converge to reveal some of injury’s fetish qualities. Injuries as fetishes are products of the labor of suffering — the processes of liberal representation that mediate between our sensuousness and our ability to count and be valued as liberal subjects. When injury is installed as a form that allows

Page 131: openev.debatecoaches.org€¦  · Web view1AC . Observation 1: If I could find the spot where truth echoesI would stand there and whisper memories of my children's future. I would

our suffering to be visible and to count, it becomes a representation that only grants us solidarity as an afterthought. Injury fashions our suffering and our perceptions of others’ suffering in its image.1 Injury is a fetish because it is dead suffering, valuable in its deadness and its detachment from the life and particularity of the sufferer but at the same time actively defining the sufferer’s identity . Under the fetishism of injuries, suffering ceases to be a sensuous life activity because it can only be experienced in the mode of injury and recognized in its value-creating mode. Suffering counts only when it takes this abstract form of injury, equalizing many kinds of abiding inequalities in the abstract. The suffering that comprises the injury, on the one hand, and the experience of that externalized injury by the sufferer and by others, on the other, seem to be independent, unrelated, and unmediated processes. The relation between these alienated sufferings stands in for real, material social relations between individuals. These relations and their experience drop out of vision and are replaced by social relations between interests and injuries to them. The crux of fetishism is in how production and consumption appear to be independent, unrelated, and unmediated processes that they are not. This feigned indifference arises from the fact that social relations between individuals and the relations of production on which they rest drop out of vision and are replaced by social relations between things. The worker’s “external” relation to her

product is not the key or sole problem. The space between production and consumption may even be where the liberatory potential of capital reposes. Fetishism , however, encourages replacing the appropriation of productive activity (in a Marxist sense) by private property in things. Other obligations and virtues get supplanted. Fetishism is problematic not because it merely renders one’s unmistakably individual labor as indistinguishable from everyone else’s labor. The issue is that fetishism makes my labor appear private and individual, and that it does so by alienating, beyond recognition, its social character and my irreducible socialities and contingencies. The i njury itself becomes an autonomous, fantastical object that is oblivious to the real material relations and the labors that have produced it. This autonomy is from history and perhaps even time itself, so the fetish starts to believe it always was, always has been, and always will be. Fetishism is a broader and more trenchant concept for my purposes than, for instance, exploitation, for a few reasons. Despite the concrete ethical effectivity of Marx’s “scientific” concept of exploitation, it has no structural safeguards against victims

turning fetishes. In exploitation, the perpetrator-victim distinction is necessary, even if not framed in terms of the

autonomous willful individual subject. More importantly, in order to make its injustice claims, the concept of exploitation relies on the existing mode of valuation and value creation that suffering (and production) follows in liberalism (and capitalism). Fetishism, though, sees subjects suspended within elaborate, suffusive, and subtle relations that make the language of exploitation possible in the first place. It thus forces us to confront the incorporated laws of value and

representation and the economies of sensuousness and subjectivity in relation to our own and others’ suffering. Fetishism and exploitation differ in the labors of suffering and the work of victims that they allow to be made visible and, ultimately, the very kinds of existences they posit as real and imaginable . The modes of personification, identification, memory, and voicing that are fostered within the ascetic theater of the fetishism of injuries together call attention to how representation’s purposive sensings and presencings occasion certain forms of subjectivity from sufferers. If representation is seen as involving sensing and making sensible,

memory and voice are codependently intertwined in this labor (which also fuels Rawls’s injury play). The labor of suffering and its many instantiations within liberalism thus bespeak life channeled to sustain the fetishes of injuries and victims . At stake within any response to liberalism is, then, the honoring of the reality and potentiality of other modes of suffering and valuation beyond liberalism. A befitting response to liberalism must necessarily traverse these components of the labor of suffering and challenge liberalism’s undialectical understanding of them. This

requires going beyond recognition of injury and victim as fetish, to addressing the categories and relations internal to liberal representation and the fetishism of injuries.

Page 132: openev.debatecoaches.org€¦  · Web view1AC . Observation 1: If I could find the spot where truth echoesI would stand there and whisper memories of my children's future. I would

Their empathetic narrative as a cry out against the oppression of black people ultimately obliterates their position by flattening black people’s ability to imagine themselves otherwise. Hartman (Sadiya V., Professor at Columbia University specializing in African-American Literature and History) 97 (Scenes of Subjection: Terror, Slavery, and Self-Making in the Nineteenth Century, pg.18-20 C.A.)

The nature of feelings aroused here is rater complicated. While this flight of imagination enables a vicarious firsthand experience of the lash, excoriates the pleasure experienced by the master in this brutal exercise of power, and unleashes Rankin’s fiery indignation and resentment,

the phantasmic vehicle of this identification is complicated, unsettling, and disturbing. Although Rankin’s fantasy culminates in indignant outcries against the institution of slavery and, clearly, the purpose of this identification is to highlight the crimes

of slavery, this flight of imaginations and slipping into the captive’s body unlatches a Pandora’s box and, surprisingly what comes to the fore is the difficulty and slipperiness of empathy. Properly speaking, empathy is a projection into another in order to better understand the other or “the projection of one’s own personality into an object, with the attribution to the object of one’s own emotions.” Yet empathy in important respects confounds Rankin’s efforts to identify with the enslaved because in making the slave’s suffering his own, Ranking begins to feel for himself rather than for those whom this exercise in imagination presumably is designed to teach. Moreover, by exploiting the vulnerability of the captive body as a vessel for the uses, thoughts, and feelings of others, the humanity extended to the slave inadvertently confirms the expectations and desires definitive of the relations of chattel slavery. In other word, the ease of Rankin’s empathetic identification is as much due to his good intentions and heartfelt opposition to slavery as to the fungibility of the captive body. By making the suffering of others his own, has Ranking ameliorated indifference or only confirmed the difficulty of understanding the suffering of the enslaved? Can the white witness of the spectacle of suffering affirm the materiality of black sentience only be feeling for himself? Does this not only exacerbate the idea that black sentience is inconceivable and unimaginable but, in the very ease of possessing the abased and enslaved body, ultimately elide an understanding and acknowledgement of the slave’s pain? Beyond evidence of slavery’s crime, what does this exposure of the suffering body of the bondman yield? Does this not reinforce the “thingly” quality of the captive by reducing the body is evidence in the very effort to establish the humanity of the enslaved? Does it not reproduce the hyperbodiness of the powerless? The purpose of these inquiries is not to cast doubt on Rankin’s motives for recounting these events but to consider the

precariousness of empathy and the thing line between witness and spectator. In the fantasy of being beaten, Rankin must substitute himself and his wife and children for black captive in order that this pain be perceived and experienced . So in fact, Rankin becomes a proxy and the other’s pain acknowledged to the degree that it can be imagined; yet by virtue of the substitution the object of identification threatens to disappear. In order to convince the reader of the horrors of slavery, Ranking must volunteer himself and his family for abasement. Put differently, the effort to counteract the commonplace callousness to black suffering requires that the white body be positioned in the place of the black body in order to make this suffering visible and intelligible. Yet if this violence can become palpable and indignation can be fully aroused only through the masochistic fantasy, then it becomes clear that empathy is double-edged, for in making the other’s suffering one’s own, this suffering is occluded by the other’s obliteration. Given the litany of horrors that fill Rankin’s pages, this recourse to fantasy reveals an anxiety about making the slave’s suffering legible. This anxiety is historically determined by the denial of black sentience, the slave’s status as object of property, the predicament of

witnessing given the legal status of blacks, and the repression of the counterdiscourses on the “peculiar institution.” Therefore, Rankin must supplant the black captive in order to give expression to black suffering, and as a consequence, the dilemmas – the denial of black sentience and the obscurity of suffering – its not attenuated but instantiated. The ambivalent character of empathy – more exactly, the repressive effect of empathy – as Jonathan Boyarin

notes, can be located in the “obliteration of otherness” or the facile intimacy that enables identification with the other only as we “feel ourselves into those we imagine as ourselves.” And as a consequence, empathy fails to expand the space of the other but merely places the self in its stead. This is not to suggest that empathy can be discarded or that Rankin’s desire to exist in the place of the other can be dismissed as a narcissistic exercise but rather to highlight the dangers of a too-easy intimacy, the consideration of the self that occurs at the expense of the slave’s suffering and the evidence of identification.

Institutional liquidation of black thought is a paradigmatic necessity – a death dealing episteme defines the horizon of institutional recognition and folding of blackness in these forums –this beyond a simple commodification claim but is a challenge to the accursed share around blackness that facilitates quotidian violence not as mere process of exchange within the economic terms but a politic of death within the general economy which operates at

Page 133: openev.debatecoaches.org€¦  · Web view1AC . Observation 1: If I could find the spot where truth echoesI would stand there and whisper memories of my children's future. I would

multiple levels including speech acts. Independent reason to reject this forum as a medium for redemption in capacity to the questions risen by black studies

Stein 16. Sharon. "Universities, slavery, and the unthought of anti-Blackness." Cultural Dynamics 28.2 (2016): 169-187.) ipartman

Black Studies interrogates the position of Black persons during slavery as well as during what Hartman (2007) has called the “afterlife of slavery ” wherein “black lives are still imperiled and devalued by a racist calculus and a political arithmetic that were entrenched centuries ago,” affecting “skewed life chances, limited access to healthcare and education, premature death, incarceration, and impoverishment” (p. 6).

At the same time as many Black Studies scholars emphasize that the position of Black enslaved per- sons in modernity is without analogy , they also emphasize that modernity itself would not have been possible without Black slavery

(Sexton, 2015). As Wilderson (2010) put it succinctly, “No slave, no world” (p. 11). Hence, the project of Black Studies has a much broader relevance than to Black people alone (James, [1969] 1993; Silva, 2014). The anti-Blackness of Man

The role of slavery and its afterlife in the ordering of modern social life operates at many levels, but perhaps most notably in the notion of humanity itself. In her work, Sylvia Wynter offers an important distinction between

“Man” and other “ genres of the human .” As modern Man (Euro-descended, middle-class, college educated) claims to be the apex of humanity, he deems all other humans—not only racialized peoples, but also the unemployed, the incarcerated, the homeless—to be sub- or

non-human (Wynter, 1994, 2003). This hierarchy of humanity has been used to justify the over- representation of Man’s interests and the subordination of others to his will. Wynter traces how the foundations of European modernity entailed a transformation, starting in the 15th century, from a worldview that transcendentalized the Church and the desire for salvation, to a worldview

that transcendentalized the State and secularized knowledge (Wynter, 2003). However, the transformations that took place were not internally produced within Europe, but rather were enabled through Indigenous colo- nization and Black enslavement. These provided both the material and conceptual conditions for the emergence of the West , including the architectures of the nation- state, capital, and the modern university. In order to invent himself, Man had to forcibly incorporate his Others into a European (conception of the) world and deny their distinct modes of thinking and being the status of “alternative modes of being human” (Wynter, 2003: 282). According to Wynter, it was to be the figure of the Negro (i.e., the category comprised by all peoples of Black African hereditary descent) that [the West] was to place at the nadir of its Chain of Being; that is, on a rung of the ladder lower than that of all humans. (p. 301) Categorization of Black people as non-human and evolutionarily inferior justified their relegation to what Fanon (2008) famously described as “the zone of nonbeing” (p. xii). In the era of chattel slavery, this translated into a logic according to which Black flesh was inscribed and treated as fungible—that is, interchangeable, accumulable, and objec-

tified as property (Hartman, 1997; King, 2014; Spillers, 1987). In addition to studies of slavery, many Black Studies scholars address post-emancipation Black subjugation , carefully cataloguing the ongoing “material, rhetorical, state, discursive, intimate, violences to which black bodies and psyches are subjected” (Sharpe,

2014b: 206). Particularly in the context of this study , it is imperative to note that Black people’s hard-won increased presence in higher education institutions does not forestall this subjugation. Many universities continue to employ Black people in poorly compen- sated and often-precarious staff position s ( Pettit, 2008) and contribute to the gentrifica- tion of neighborhoods surrounding their campuses, often with significant Black populations (Baldwin, 2015; Bose, 2014). Black faculty, students , and staff regularly experience anti-Black racism from their peers and professors (e.g. Griffin et al., 2014; Gusa, 2010; Harper et al., 2011;

Johnson-Ahorlu, 2012; Patitu and Hinton, 2003; Patton and Catching, 2009; Solórzano et al., 2000) and are interrogated and abused by campus and local police (e.g. McMillan Cottom, 2014; Vest, 2013). Anti-Blackness also inheres in the production of knowledge itself . In spite of the many powerful disruptions enacted through Black Studies and associated fields, Sharpe (2014a) argues that an anti-Black “death-dealing episteme continue[s] to be produced in ‘think tanks’ and in the university, by teachers, lecturers, researchers, and scholars, and then reproduced by the students who have been educated in the classrooms and institu- tions where [Black people] labor” (p. 61). This enduring “death-dealing episteme” is not merely contained within explicitly

Page 134: openev.debatecoaches.org€¦  · Web view1AC . Observation 1: If I could find the spot where truth echoesI would stand there and whisper memories of my children's future. I would

white supremacist knowledge, like the now thor- oughly discredited field of phrenology or even today’s more blatantly

pathologizing strains of mainstream social science. Instead, according to Wynter (1994), “both the issue of ‘race’ and its classificatory logic ” are built into the basic logic of the modern order of knowledge (p. 47). Spillers (1987)

called this order an “American grammar ,” arguing, “the ruling episteme that releases the dynamics of naming and valuation , remains grounded in the originating metaphors of captivity and mutilation” (p. 68). Yet, not only does this grammar extend beyond the United States, it also extends beyond an enduring order of knowing to encompass an enduring order of being as well. Silva (2013) captures both in her notion of “the ontoepistemological grammar that governs post- Enlightenment accounts of existence” (p. 50) and that structures Man’s claims to autonomy, self-determination, and mastery of universal reason. These

claims are then used to justify the imposition of Man’s will on the world and on (and in contrast to) those whom he

deems his irrational and outer-determined racial Others. To consider that this modern / colonial grammar has endured for

over five centuries does not minimize its internal variation, the importance of its ongoing contestation, nor the possibility of its

further rearticulation. However, it does raise questions about the available possibilities for justice within it. Black life According to Moten (2003), There are t hose who act as if the only way to speak or fathom or measure the unspeakable , unfathomable, immeasurable venality of the slavers is by way of the absolute degradation of the enslaved . But such calculation is faulty from the start insofar as we are irreducible to what is done to us... (p. 56) Indeed, although the modern global order situates Blackness as “always already a referent of commodity, an object, and the other, as fact beyond evidence” (Silva, 2014: 81), this in no way delimits what Black life was, is, and can be. Wilderson clarifies the differ- ence thusly: “I’m not saying that in this space of negation, which is blackness, there is no life. We have tremendous life. But this life is not analogous to those touchstones of cohe- sion that hold civil society together” (Hartman and Wilderson, 2003: 187). Many have noted that in excess of both the position of non-being ascribed to Black persons by modernity’s

grammar and of Man’s narrowly imagined possibilities for existence, Blackness has always offered ways of knowing and being otherwise

(Moten, 2003, 2008, 2013; Scott and Wynter, 2010; Sexton, 2011; Silva, 2013, 2014). According to Silva (2014), because Man’s claims to sovereign subjecthood are dependent upon Black affectability and objecthood , the stability of this

relation is constantly under- mined by “the radical potential Blackness hosts” (p. 84). This radical potential contains “another text ... a grammar that exceeds existing articulations of the human as a thing of self-determination” (Silva, 2013: 57). If Man’s conceptual coherence and material continuation are premised on Blackness as a threatening but necessary lack and

nega- tion, then Black life cannot be adequately recognized or represented by or through his grammar without repeating this violence .

The alternative is to refuse the AFF in favor of black opacity. Opacity troubles the text, renders it non-normative, pedagogically unsound and in doing so protects black experience from being pressed into service for the master. the black object can only exist for-itself and not for dominant (master) epistemology. This solves the entirety of the case without risking naturalizing the condition of black subordination. Reeves 12

Reeves [Black Western Thought: Toward a Theory of the Black Citizen-Object, 2012]

‘Trouble’ obscures the text, renders it funky, non-normative, undisciplined, pedagogically unsound —in need of being brought back into order . Deodorized. ‘Trouble’ was trouble because it defied the pedagogy of academic acceptability . Rather than understand my use of the term trouble as a move to nuance my reading of

“Timour the Tarter,” my linguistic choice made the text invisible and aberrant. Rather than reason or grapple with the use of the term, my Romantics professor cast it aside as a type of unknowing or non-knowing,

Page 135: openev.debatecoaches.org€¦  · Web view1AC . Observation 1: If I could find the spot where truth echoesI would stand there and whisper memories of my children's future. I would

a type of noise, a screech and scream , to invoke Édouard Glissant and Fred Moten, that needed to be fashioned into a melody that she could recognize within her own frameworks of melodic structure. However, in her dismissal and denial, she re-inscribes the exclusionary poetics of the Academy , a poetics that shuns discursive bodies of knowledge from traditions and subjectivities marginalized by the Academy. Also,

this dismissal, subtly and not-so subtly, invigorates and reproduces the legacy of cultural disenfranchisement and disappearing that harkens back to slavery. The mishearing of the slave becomes the misreading of the black scholar. The misreading of black scholar becomes the misreading of the black poet. The misreading of the black poet becomes the misreading of the black citizen-object . These mis-es share a continuum, a poetics of relation . However, the relationship between these two mis-es are opaque—one’s a purposeful mis- enacted by a marginalized group trying to eke out a semblance of agency in a space of social annihilation and death while the other is a disciplinary pedagogy in which the black scholar’s body of linguistic knowledge is deemed funky and needs to be brought back into order. What does it mean that one mishearing benefits the enslaved while the later misreading, born out of the first mis- , works against the off-spring of that same group? This question

exemplifies the fraught relationship of translation, subversive citationality, and competing and conflicting iterative traditions that this

dissertation project investigates. This interaction between competing and conflicting discourses and linguistic traditions epitomizes the opaque relationship between a discourse meant to trouble, subvert, and challenge the mastering, the subjugation, the surveillance of a dominant discourse while dis- and re-appearing in hopes of avoiding recognition and incurring more death (both social and real).This negotiation of visibility and invisibility, appearing and disappearing, translation and mistranslation is the negotiation of the subjectivity of black objects . This dissertation project concerns itself with investigating and troubling the recognizable. This project extends the work of Aldon Nielsen, Nathaniel Mackey, Harryette Mullen, Evie Shockley, and Meta DuEwa Jones in area of black poetics and what is recognizably black, what is recognizably black poetry, and who is recognizably a black poet6. This project seeks to account for the odd moment when the black poet is told he or she does not write or sound like a black poet.

Page 136: openev.debatecoaches.org€¦  · Web view1AC . Observation 1: If I could find the spot where truth echoesI would stand there and whisper memories of my children's future. I would

Vs. TaiwanAnti-blackness is a structural antagonism that undergirds political life. Their gesture towards non-Black bodies facilitates the expansion of antiblackness, colonialism, and ultimately sustains the ontological foundation and expansion of the current socioeconomic order. Dumas 16 [Michael J. Dumas, Assistant Professor at the University of California, Berkeley in the Graduate School of Education and the Department of African American Studies, “Against the Dark: Antiblackness in Education Policy and Discourse,” Theory Into Practice 55:11–19, 2016, published by The College of Education and Human Ecology, The Ohio State University]

Antiblackness is the central concern and proposition within an intellectual project known as Afro-pessimism.1 Afro-pessimism theorizes that Black people exist in a structurally antagonistic relationship with humanity . That is, the very technologies and imaginations that allow a social recognition of the humanness of others systematically exclude this possibility for the Black. The Black cannot be human , is not simply an Other but is other than human. Thus, antiblackness does not signify a mere racial conflict that might be resolved through organized political struggle and appeals to the state and to the citizenry for redress. Instead, antiblackness marks an irreconcilability between the Black and any sense of social or cultural regard . The aim of theorizing antiblackness is not to offer solutions to racial inequality, but to come to a deeper understanding of the Black condition within a context of utter

contempt for, and acceptance of violence against the Black. Afro-pessimist scholars contend that the Black is socially and culturally positioned as slave, dispossessed of human agency, desire, and freedom. This is not meant to suggest that Black people are currently

enslaved (by whites or by law), but that slavery marks the ontological position of Black people . Slavery is how Black existence is imagined and enacted upon, and how non-Black people—and particularly whites— assert their own right to freedom, and right to the consumption, destruction, and/or simple dismissal of the Black. “Through chattel slavery,” Frank Wilderson (2010) argued, the world gave birth and coherence to both its joys of domesticity and to its struggles of political discontent; and with these joys and struggles the Human was born, but not before it murdered the Black, forging a symbiosis between the political ontology of Humanity and the social death of Blacks. (pp. 20 – 21) This “social death” of the slave is introduced most explicitly in the work of Orlando Patterson (1982), who detailed how slavery involves a parasitic relationship between slave owner and slave, such that the freedom of the slave owner is only

secured and understood in relation to power over the slave. For Patterson, slavery is “the permanent, violent domination of natally alienated and generally dishonored persons” (p. 13). Although slavery involves personal relationships between groups, it also operates as an institutionalized system, maintained through social processes that make it impossible for the Slave to live, to be regarded as alive for her- or himself in the social world. This focus on slavery might seem anachronistic in the current historical moment, some 150 years after the (formal) end of the

institution in the United States. However, Wilderson maintained that the relations of power have not changed. He explained: Nothing remotely approaching claims successfully made on the state has come to pass. In other words, the election of a Black

president aside, police brutality, mass incarceration, segregated and substandard schools and housing , astronomical rates of HIV infection, and the threat of being turned away en masse at the polls still constitute the lived experience of Black life . (p. 10) This lived experience serves as a continual reinscribing of the nonhumanness of the Black , a legitimization of the very antiblackness that has motivated centuries of violence against Black bodies . In this sense, even as slavery is no longer official state policy and practice, the slave endures in the social imagination, and also in the everyday suffering experienced by Black people. As Saidiya Hartman (2007) insisted, Americans are living in what she described as “the afterlife of

Page 137: openev.debatecoaches.org€¦  · Web view1AC . Observation 1: If I could find the spot where truth echoesI would stand there and whisper memories of my children's future. I would

slavery:” Black lives are still imperiled and devalued by a racial calculus and a political arithmetic that were entrenched centuries ago. This is the afterlife of slavery—skewed life chances, limited access to health and

education, premature death, incarceration, and impoverishment. I, too, am the afterlife of slavery. (p. 6) Importantly here, the afterlife of slavery is not only an historical moment, but deeply impressed upon Black flesh, in the embodiment of the Black person as slave. Thus, Hartman maintained, she is also this afterlife of slavery. Salamishah Tillet (2012) made clear the heaviness of the historical memory, the

everpresence of slavery in Black life: Because racial exclusion has become part and parcel of African American political identity since slavery, it cannot simply be willed or wished away. This protracted experience of disillusionment, mourning, and yearning is in fact the basis of African American civic estrangement. Its lingering is

not just a haunting of the past but is also a reminder of the present-day racial inequities that keep African American citizens in an indeterminate, unassimilable state as a racialized ‘Other.’ While the affect of racial melancholia was bred in the dyad of slavery and democracy, it persists because of the paradox of legal citizenship and civic estrangement. (p. 9) To the extent that there is ample evidence of the civic estrangement of Black people—their exclusion from the public sphere—one can theorize that the Black is still socially positioned as the slave, as difficult as it may be to use this frame to understand contemporary “race relations.” Here, “race relations” is necessarily in quotations because there is really no relation to be had between master and slave in the way one might conceptualize human

relationships. For Afro-pessimists, the Black is not only misrecognized, but unrecognizable as human, and therefore there is no social or political relationship to be fostered or restored . As Wilderson argued, Our analysis cannot be approached through the rubric of gains or reversals in struggles with the state and civil society, not unless and until the interlocutor first explains how the Slave is of the world. The onus is not on one who posits the Master/Slave dichotomy but on the one who argues there is a distinction between Slaveness and Blackness. How, when, and where did such a split occur? (p. 11) And this is the broader challenge posed by a

theory of antiblackness: There is no clear historical moment in which there was a break between slavery and

acknowledgement of Black citizenship and Human-ness; nor is there any indication of a clear disruption of the technologies of violence—that is, the institutional structures and social processes—that maintain Black subjugation. Thus, Afro-pessimists suggest that one must consider the Black as (still) incapable of asking for (civil or human) rights. This does not deny the long legacy of Black racial struggle, but it

positions this struggle as an impossibility, because the Black is (still) imagined outside of the citizenship that allows claims for redress to be regarded as legitimate, or even logical. Part of the challenge in theorizing blackness in contemporary race discourse is that Americans are living in an officially antiracist society, in which, as Jodi Melamed has documented, postWorld War II racial liberalisms and neoliberalisms make some space for the participation of multicultural subjects (Melamed, 2011). That is,

even as race continues to structure capitalism, which in turn facilitates white accumulation, the official stance of the state is against racism; blatantly racist laws and government practices have been declared illegal, and the market embraces outreach to a wide multicultural range of consumers. In this context, there is a rush to celebrate the social and economic advancement of select Black individuals and, perhaps more significantly, the success of other groups of people of color. In fact, it is the social and cultural inclusion of non-

Black people of color that is often offered as evidence of the end of racial animus and racial barriers in the society. Therefore, the failure of large swaths of the Black population is purported to be a result of cultural deficits within the Black . The slave, always suspected of being lazy and shiftless, now must bear primary responsibility for not making it in a society, which—officially, anyway

—thrives on multiracial harmony and civic participation. Jared Sexton (2008, 2010) contended that in this era, multiracialism thrives largely at the expense of, and firmly against, blackness. His argument rests on the premise that the color line is more fluid during periods in which Black freedom is thought to be most contained. Thus, during slavery in the United States, multiracial communities could serve as “buffer classes between whites and blacks” which often “corroborated and collaborated with antiblackness” (Sexton, 2008, p. 12). The

current period is marked by similar dynamics, with little organized Black political movement, resegregation of

neighborhoods and schools, and, in fact, an easy deterritorialization and gentrification of historic Black urban homeplaces. The current Black Lives Matter movement (Garza, 2014), which has emerged in the wake of so many cases of antiBlack violence,

may yet shift Americans into a period of heightened anxiety about Black bodies, but Sexton’s description of the

current period is valid: There is little fear of Black bodies and, arguably, an emboldened antipathy to the Black overall. This, in Sexton’s theorizing, opens up new spaces for multiracial inclusion. In this moment, the Black –white divide is seen as less consequential and not as much the result of white attitudes and behaviors . In these moments, Sexton maintained, the more significant boundary is the one constructed “between blackness and everything else” (2008, p. 13). And this is a boundary seemingly constructed and maintained by recalcitrant Black people against multiracialism, and more to the point,

multiracial progress. Multiracialism, in Sexton’s view, “premises its contribution to knowledge, culture and politics upon an evacuation of the historical richness , intellectual intensity, cultural expansiveness, and political complexity of Black experience, including, perhaps especially, its indelible terrors” (2008, p. 15). Transcending the Black-white

Page 138: openev.debatecoaches.org€¦  · Web view1AC . Observation 1: If I could find the spot where truth echoesI would stand there and whisper memories of my children's future. I would

binary , multiracialism ostensibly moves people past the narrowness and anachronism of blackness and toward a more profitable global economy and more sophisticated cultural milieu . Embracing non- Black bodies of color thus facilitates, and is facilitated by, antiblackness, and can be justified as antiracist precisely because it is inclusive of more than white. “The [B]lack body,” Lewis Gordon contended, “is confronted by the situation of its absence” (1997, p. 73). This absence—this social death or afterlife of/as the slave—positions Black people as the embodiment of problem, a thing rather than a people suffering from problems created by antiblackness. Part of the aim of Afropessimist scholarship is to insist on the humanity of Black people. “Those of us who seek to understand [B]lack people,” Gordon concluded, need to “bear in mind that [B]lack people are human beings” (p. 78). In an anti-Black world, this is easier said than done. In the end, there may be, as Wilderson suggested, no “roadmap to freedom so extensive it would free us from the epistemic air we breathe” (2010, p.

338). Even so, like Gordon, Wilderson suggested that theorizing antiblackness is important simply as an existential and political recognition of Black humanity, as a means “to say we must be free of air, while admitting to knowing no other source of breath” (p. 338; italics in original).

The only possible demand is one that calls for the end of the world itself—the affirmative represents a conflict within the paradigm of America but refuses to challenge the foundational antagonism that produces the violence that undergirds that same paradigm Wilderson, ’10 [2010, Frank B. Wilderson is an Associate Professor of African-American Studies at UC Irvine and has a Ph.D. from UC Berkeley, “Red, White & Black: Cinema and the Structure of U.S. Antagonisms,”]

Leaving aside for the moment their state of mind, it would seem that the structure , that is to say the rebar, or better still the grammar of their demands —and, by extension, the grammar of their suffering—was indeed an ethical grammar. Perhaps their grammars are the only ethical grammars available to modern politics and modernity writ large , for they draw our attention not to the way in which space and time are used and abused by enfranchised and violently powerful interests, but to the violence that underwrites the modern world ’ s capacity to think, act, and exist spatially and temporally . The violence that robbed her of her body and him of his land provided the stage upon which other

violent and consensual dramas could be enacted. Thus, they would have to be crazy , crazy enough to call not merely the actions of the world to account but to call the world itself to account , and to account for them no less! The woman at Columbia was not demanding to be a participant in an unethical network of distribution: she was not demanding a place within capital, a piece of the pie (the demand for her sofa notwithstanding). Rather, she was articulating a triangulation between, on the one hand, the loss of her body, the very dereliction of her corporeal integrity, what Hortense Spillers charts as the transition from being a being to becoming a “being for the captor” (206), the drama of value (the stage upon which surplus value is extracted from labor power through commodity production and sale); and on the other, the corporeal integrity that, once ripped from her body, fortified and extended the corporeal integrity of everyone else on the street. She gave birth to the commodity and to the Human, yet

she had neither subjectivity nor a sofa to show for it. In her eyes, the world — and not its myriad discriminatory practices, but the world itself — was unethical . And yet, the world passes by her without the slightest inclination to stop and disabuse her of her claim . Instead, it calls her “crazy.” And to what does the world attribute the Native American man’s insanity? “He’s crazy if he thinks he’s getting any money out of us”? Surely, that doesn’t make him crazy.

Rather it is simply an indication that he does not have a big enough gun . What are we to make of a world that responds to the most lucid enunciation of ethics with violence ? What are the foundational questions of the ethico-political? Why are these questions so scandalous that they are rarely posed politically, intellectually, and cinematically—unless they are

posed obliquely and unconsciously, as if by accident? Return Turtle Island to the “Savage.” Repair the demolished subjectivity of the Slave . Two simple sentences, thirteen simple words, and the structure of U.S. ( and perhaps global )

Page 139: openev.debatecoaches.org€¦  · Web view1AC . Observation 1: If I could find the spot where truth echoesI would stand there and whisper memories of my children's future. I would

antagonisms would be dismantled . An “ ethical modernity ” would no longer sound like an oxymoron . From there we could busy ourselves with important conflicts that have been promoted to the level of antagonisms: class struggle, gender conflict, immigrants rights. When pared down to thirteen words and two

sentences, one cannot but wonder why questions that go to the heart of the ethico-political , questions of political ontology, are so unspeakable in intellectual meditations, political broadsides , and even socially and politically engaged feature films. Clearly they can be spoken, even a child could speak those lines, so they would pose no problem for a scholar,

an activist, or a filmmaker. And yet, what is also clear—if the filmographies of socially and politically engaged directors, the archive of progressive scholars , and the plethora of Left-wing broadsides are anything to go by — is that what can so easily be spoken is now (five hundred years and two hundred fifty million Settlers/Masters on) so ubiquitously unspoken that these two simple sentences, these thirteen words not only render their speaker “ crazy ” but become themselves impossible to imagine . Soon it will be forty years since radical politics, Left-

leaning scholarship, and socially engaged feature films began to speak the unspeakable. In the 1960s and early 1970s the questions asked by radical politics and scholarship were not “ Should the U.S. be overthrown? ” or even “ Would it be overthrown? ” but rather when and how —and, for some, what—would come in its wake. Those steadfast in their conviction that there remained a discernable quantum of ethics in the U.S. writ large (and here I am speaking of everyone from Martin Luther King, Jr., prior to his 1968 shift, to the Tom Hayden wing of SDS, to the Julian Bond and Marion Barry

faction of SNCC, to Bobbie Kennedy Democrats) were accountable , in their rhetorical machinations, to the paradigmatic zeitgeist of the Black Panthers, the American Indian Movement, and the Weather Underground . Radicals and progressives could deride , reject, or chastise armed struggle mercilessly and cavalierly with respect to tactics and the possibility of “ success, ” but they could not dismiss revolution-as-ethic because they could not make a convincing case —by way of a paradigmatic analysis—that the U.S. was an ethical formation and still hope to maintain credibility as radicals and progressives . Even Bobby Kennedy (a U.S. attorney general and

presidential candidate) mused that the law and its enforcers had no ethical standing in the presence of Blacks . One could (and many did) acknowledge America’s strength and power. This seldom, however, rose to the level of an ethical assessment, but rather remained an assessment of the so-called “ balance of forces. ” The political discourse of Blacks, and to a lesser extent Indians, circulated too widely to credibly wed the U.S. and ethics. The raw force of COINTELPRO put an end to this

trajectory toward a possible hegemony of ethical accountability. Consequently, the power of Blackness and Redness to pose the question — and the power to pose the question is the greatest power of all — retreated as did White radicals and progressives who “ retired ” from struggle. The question ’ s echo lies buried in the graves of young Black Panthers, AIM Warriors, and Black Liberation Army soldiers , or in prison cells where so many of them have been rotting (some in solitary confinement) for ten, twenty, thirty years, and at the gates of the academy where the “ crazies ” shout at passers-by . Gone are not only the young and vibrant voices that affected a seismic shift on the political landscape , but also the intellectual protocols of inquiry, and with them a spate of feature films that became authorized, if not by an unabashed revolutionary polemic, then certainly by a revolutionary zeitgeist . Is it still possible for a dream of unfettered ethics, a dream of the Settlement and the Slave estate ’ s destruction , to manifest itself at the ethical core of cinematic discourse, when this dream is no longer a constituent element of political discourse in the streets nor of intellectual discourse in the academy? The answer is “no” in the sense that, as history has shown, what cannot be articulated as political discourse in the streets is doubly foreclosed upon

in screenplays and in scholarly prose; but “yes” in the sense that in even the most taciturn historical moments such as ours, the grammar of Black and Red suffering breaks in on this foreclosure , albeit like the somatic compliance of hysterical symptoms—it registers in both cinema and scholarship as symptoms of awareness of the structural antagonisms. Between 1967 and 1980, we could think cinematically and intellectually of Blackness and Redness as having the coherence of full-blown discourses. But from 1980

to the present, Blackness and Redness manifests only in the rebar of cinematic and intellectual (political) discourse, that is, as unspoken grammars . This grammar can be discerned in the cinematic strategies (lighting,

camera angles, image composition, and acoustic strategies/design), even when the script labors for the spectator to imagine social turmoil through the rubric of conflic t (that is, a rubric of problems that can be posed and

Page 140: openev.debatecoaches.org€¦  · Web view1AC . Observation 1: If I could find the spot where truth echoesI would stand there and whisper memories of my children's future. I would

conceptually solved) as opposed to the rubric of antagonism ( an irreconcilable struggle between entities , or positionalities, the resolution of which is not dialectical but entails the obliteration of one of the positions ). In other words, even when films narrate a story in which Blacks or Indians are beleaguered with problems that the script insists are conceptually coherent (usually having to do with poverty or the absence of “family values”), the non-narrative, or cinematic, strategies of the film often disrupt this coherence by posing the irreconcilable questions of Red and Black political ontology—or non-ontology.

The grammar of antagonism breaks in on the mendacity of conflict . Semiotics and linguistics teach us that when we

speak, our grammar goes unspoken. Our grammar is assumed. It is the structure through which the labor of speech is possible. Likewise, the grammar of political ethics —the grammar of assumptions regarding the ontology of suffering — which underwrite Film Theory and political discourse (in this book, discourse elaborated in direct relation to radical action), and which underwrite

cinematic speech (in this book, Red, White, and Black films from the mid-1960s to the present) is also unspoken . This notwithstanding,

film theory, political discourse , and cinema assume an ontological grammar, a structure of suffering . And the structure of suffering which film theory, political discourse, and cinema assume crowds out other structures of suffering, regardless of the sentiment of the film or the spirit of unity mobilized by the political discourse in question. To put a finer point on it, structures of ontological suffering stand in antagonistic, rather then conflictual, relation to one another (despite the fact that antagonists themselves may not be aware of the ontological positionality from which they speak). Though this is perhaps the most controversial and out-of-step claim of this book, it is, nonetheless, the foundation of the close reading of feature films and political theory that follows.

Social death has no ethical considerations behind it – none of their impacts matter to those who are already dead - the Slave’s subject position is one of non-ontology – civil society checks violence against any non-Black, but guarantees it against the SlaveWilderson- 2002

Frank Wilderson- The Prison Slave as Hegemony's (Silent) Scandal-Presented a t #Imprisoned Intellectuals # Conference Brown University, April 13th 2002, NN

Civil society is not a terrain intended for the Black subject. It is coded as waged and wages are White . Civil society is the terrain where hegemony is produced, contested, mapped. And th e invitat ion to p articipate in hegemony's gestures of influence, leadership, and consent is not ext ended to t he unwaged. We live in the world , but ex ist out side of civil s ociety. This structurally impossible position is a paradox, because the Black subject, the slave, is vital to political economy: s/he kick-starts capital at its genesis and rescues it from its over-accumulation crisis at its end. But Marxism has no account of this phenomenal birth and life-saving role played by the Black subject: from Marx and Gr amsci we have con sistent s ilence. In taking Foucau lt to ta sk for a ssum ing a univ ersal s ubject in r evolt ag ainst d iscipline, in the same s pirit in which I have t aken Gr amsci to ta sk for as suming a u niversal sub ject, the subject of civil societ y in revolt a gainst capita l, Joy Jam es writes : The U.S. carceral network kills, however, and in its prisons, it kills more blacks than any other ethnic group. American prisons constitute an "outside" in U.S. political life. In fact, our society displays waves of concentric outside circles with increasing distances from bourgeois self-policing. The state routinely polices the14 unassim ilable in the hell of lockdow n, deprivat ion tanks , control units , and holes for political prisoners (Resisting State Violence 1996: 34 ) But this peculiar preoccupation is not Gramsci's bailiwick. His concern is with White folks; or with folks in a White (ned) enough subject position that they are confronted by, or threat ened by th e remova l of, a wag e -- be it monetary or social. But Black subjectivity itself disarticulates the Gramscian dream as a ubiquitous emancipatory strategy , because Gramsci, like most White activists, and radical American movements like the prison abolition movement, has no theory of the unwaged, no solidarity with the slave If we are to take Fanon at his

Page 141: openev.debatecoaches.org€¦  · Web view1AC . Observation 1: If I could find the spot where truth echoesI would stand there and whisper memories of my children's future. I would

word when he writes, #Decolonization, which sets out to change the order of the world, is, obviously, a program of complete disorder # (37) then we must accept the fact that no other body functions in the Imaginary, the Symbolic, or the Real so completely as a repository of complete disorder as the Black body. Blackness is the site of absolute dereliction at the level of the Real, for in its magnetizing of bullets the Black body functions as the map of gratuitous violence through which civil society is possible: namely, those other bodies for which violence is, or can be, contingent. Blackness is the site of absolute dereliction at the level of the Symbolic, for Blackness in America generates no categories for the chromosome of History, no data for the categories of Immigration or Sovereignty; it is an experience without analog # a past, without a heritage. Blackness is the site of absolute dereliction at the level of t he Imaginary for # whoever says #rape # says Black , # (Fanon) , whoever says #prison # says Black , and whoever says #AIDS # says Black (Sexton) # the #Negro is a phobogenic object # (Fanon). Indeed &a phobogenic object &a past without a heritage &the map of gratuitous violence &a program of complete disorder. But whereas this realization is, and should be cause for alarm, it should not be cause for lament, or worse, disavowal # not at least, for a true revolutionary, or for a truly revolutionary movement such as prison a bolition. 15 If a social movement is to be neither social democratic, nor Marxist, in terms of the structure of its political desire then it should grasp the invitation to assume the positionality of subjects of social death that present themselves; and, if we are to be honest with ourselves we must admit that the “Negro “ has been inviting Whites, and as well as civil society #s junior partners, to the dance of social death for hundreds of years, but few have wanted to learn the steps. They have been, and remain today # even in the most anti-racist movements, like the prison abolition movement # invested elsewhere. This is not to say that all oppositional political desire today is pro-White, but it is to say that it is almost always “ anti-Black ” which is to say it will not dance with death. Black liberation, as a prospect, makes radicalism more dangerous to the U.S. Not because it raises the specter of some alternative polity (like socialism, or community control of existing resources) but because its condition of possibility as well as its gesture of resistance functions as a negative dialectic: a politics of refusal and a refus al to affirm , a program of complete disorder. One mus t embrace its disorder, its in coherence and allow oneself to be elaborated by it, if indeed one's politics are to be underwritten by a desire to take this country down. If this is not the desire which underwrites one #s politics then through what strategy of legitimation is the word #prison # being linked t o the wo rd #abolition #? Wh at ar e this movem ent #s lines of po litical a ccount abilit y? There #s nothing foreign, frightening, or even unpracticed about the embrace of disorder and incoherence. The desire to be embraced, and elaborated, by disorder and incoherence is not anathema in and of itself: no one, for example, has ever been known to say #gee-whiz, if only my orgasms would end a little sooner, or maybe not come at all. # But few so-called radicals desire to be embraced, and elaborated, by the disorder and incoherence of Blackness # and the state of politica l movemen ts in A merica to day is ma rked by t his very N egroph obogen isis: #gee-whiz, if only Black rage could be more coherent, or maybe not come at all. # Perhaps there #s something more terrifying about the joy of Black, then there is about the joy of sex (unless one is talking sex wit h a Negr o). Perhaps coalitions today p refer to remain in- orgas mic in the fa ce of civilsociety # with hegemony as a handy prophylactic, just in case. But if, through this stasis, or paralysis , they tr y to do t he work of pr ison a bolit ion # that work will fail; because it is always work from a position of coherence (i.e. the worker) on behalf of a position of incoherence, the Black subject, or prison slave. In this way, social formations on the Left remain blind to the contradictions of coalitions bet ween worker s and s laves. T hey remain coalitions opera ting with in the logic of civil society; and function less as revolutionary promises and more as crowding out scenarios of Black antagonisms # they simply feed our frustration. Whereas the positionality of the worker # be s/he a factory worker demanding a monetary wage or an immigrant or White woman demanding a social wage # gestures toward the reconfiguration of civil society, the positionality of the Black subject # be s/he a prison-slave or a prison-slave-in-waiting # gestures toward the disconfiguration of civil society: from the

coherence of civil society, t he Black subject beckons with the in coherence of civil war. A civil war which reclaims Blackness not as a positive value , but as a politically enabling site, to quote Fanon, of “absolute dereliction“: a scandal which rends civil society asunder. Civil war , then, becomes that unthought, but never forgotten understudy of hegemony. A Black specter waiting in the wings, an endless antagonism that cannot be satisfied (via reform or reparation) but must nonetheless be pursued to the death.

Page 142: openev.debatecoaches.org€¦  · Web view1AC . Observation 1: If I could find the spot where truth echoesI would stand there and whisper memories of my children's future. I would

R SPEC:

A. INTERPRETATION – THE AFF HAS TO SPECIFY THE MANNER IN WHICH THEY STAND RESOLVED IN RELATION TO THE STATEMENT OF THE RESOLUTION

DEFINITION – RESOLVED IS A DETERMINATION REGARDING AN OPINION AND PROPOSITIONLaw Dictionary: What is RESOLUTION? definition of RESOLUTION (Black's Law Dictionary)

http://thelawdictionary.org/resolution/#ixzz2XdvQGb50,13

The determination or decision, in regard to its opinion or intention , of a deliberative or legislative body, public assembly, town

council, board of directors or the like. Also a motion or formal proposition offered for adoption by such a body .

CONTEXT – INTENT DOES NOT MATTER. IT IS NOT ENOUGH TO DETERMINE THAT ONE IS RESOLVED BUT THE POLITICAL SITUATEDNESS OF THE TOPIC WITHIN CONSIDERATION. ALL LEGAL DEFINITIONS OF RESOLVE ARE WRITTEN WITHOUT THE CONTEXT OF HOW ONE HAS TO RESOLVE. TO RESOLVE WITHOUT CONSIDERING THE CONTEXT IN WHICH IT IS DONE IS WHERE INSISTENCE REPLACE IMPOSITION, A WORLD-MAKING OF VIOLENCE THAT WRITES OFF OR WRITES OVER BLACK CONCERNS WITHIN A BLACK POLITICAL IMAGINATION. Sexton The Social Life of Social Death.pg 31. 2011

If the intimacy of power suggests the sheer difficulty of difference , the trouble endemic to determining where the white imagination ends and the black imagination begins, then the power of intimacy suggests , with no less tenacity and no less significance, that our grand involvement across the color line is structured like the figure of an envelope, folds folded within folds: a black letter law whose message is obscured, enveloped, turned about, reversed. Here a structure of violence is inscribed problematically in narrative, an inscription that can only struggle and fail to be something other than a writing-off, or a writing-over . The massive violence that founds and opens a structure of vulnerability, a world-making enjoyment of that violence of enjoyment disappears into the telos of resolution, the closure of family romance, the drive for kinship, where insistence replaces imposition. Black rage converts magically to black therapeutics, a white mythology that disavows its points of origin in the theft that creates the crime and its alibi at once. This illegible word, where affect drops away only to remain, is what Sharpe terms “monstrous intimacy,” “a memory for forgetting .” xv And what would we do without it? Indeed, what might we do?

B. VIOLATION – THE AFF DID NOT SPECIFY THEIR RESOLVE. THE AFF ENGAGE IN A NORMATIVE FORM OF DEBATE WHICH SUSPENDS THE QUESTION OF HOW

Page 143: openev.debatecoaches.org€¦  · Web view1AC . Observation 1: If I could find the spot where truth echoesI would stand there and whisper memories of my children's future. I would

WE RELATE TO THE RESOLUTIONAL STATEMENT WHICH STRUCTURES THE DEBATE SPACE. THIS IS AN ASSUMPATION OF PRIVILEDGE THAT IS BAD FOR DEBATE AND DEBATERS. THIS QUESTION IS APRIORI BECAUSE HOW ONE IS RESOLVED TOWARD STRUCTURE WHAT IS DEBATABLE.

C. BLACK DEBATABILITY – FOR BLACK PEOPLE GUN CONTROL IS THE CORE OF THE TOPIC WHEN DISCUSSING POLICY THAT IN CONCERNED WITH REGULATING THE SALE OF WEAPONS

Livingston, Lindsay Adamson Picking Up the Gun Spectacular Performances of Firearm Ownership in the Long Civil Rights Movement. 264.2017

While guns function by projecting a small piece of metal into flesh, they can, alternately, work in performative ways that draw

on centuries of fabricated and exaggerated racialized threat. This becomes particularly clear in examples like those discussed above,

where gun legislation, both pro- and anti-gun control, is used to subjugate Black communities. The gun control legislation of the 1960s and 1970s was in direct response to armed Black resistance; scholars have argued that earlier

pro-gun legislation supported state militias that participated first in slave patrols and later, following emancipation, in the continued oppression of newly freed Black Americans.3 These imagined threats comprise some of the material archive that guns

contain, preserving a racialized past while simultaneously providing potential futures. When those futures are so closely tied to violence perpetrated by firearms, it can create a dire situation indeed. While the militarization of US American culture has

accelerated in the mid- to latetwentieth century, the militarized surveillance and control of Black bodies in North America has been underway since long before the United States of America was founded (Browne 2015). It is this pernicious effect of militarization that is often elided, as Catherine Lutz suggests. “Militarization is intimately connected not only to the obvious increase in the size of armies and resurgence of militant nationalisms and militant fundamentalisms,” she argues, “but also the less visible deformation of human potentials into the hierarchies of race, class, gender, and sexuality, and to the shaping of national histories in ways that glorify and legitimate military action” (Lutz 2002:723). The long history of gun rights in the United States—who can openly perform the right to carry a firearm and whom the armed person is threatening—has functioned as a state-sponsored tool aiding in this project of deformation, even as the gun performs its own meanings and contains its own histories and futures.

AND, SUSPENSION OF THE RESOLVED OF THE TOPIC IS ANTI-BLACK Wilderson 10’( Frank Red White and Black: Cinema and the structure of U.S. Antagonisms) pg. 57 I am calling for a different conceptual framework , predicated not on the subject-effect of cultural performance but on the structure of political ontology, a framework that allows us to substitute a culture of politics for a politics of culture. The value in this rests not simply in the way it would help us rethink cinema and performance, but in the way it can help us theorize what is at present only intuitive and anecdotal: the unbridgeable gap between Black being and Human life. To put a finer point on it, such a framework might enhance the explanatory power of theory, art, and politics by destroying and perhaps restructuring the ethical range of our current ensemble of questions. This has profound implications for non-Black film studies, Black film studies, and African American studies writ large because they are currently entangled in a multicultural paradigm that takes an interest in an insufficiently critical comparative analysis— that is, a comparative analysis in pursuit of a coalition politics (if not in practice then at least as a theorizing metaphor) which, by its very nature, crowds out and forecloses the Slave's grammar of suffering.

Page 144: openev.debatecoaches.org€¦  · Web view1AC . Observation 1: If I could find the spot where truth echoesI would stand there and whisper memories of my children's future. I would

D. STANDARDS 1. LIMITS A. R-SPEC PROVIDES THE BEST LIMITS FOR MULTIPLE FORMS OF DEBATE – WE CHECK ABUSIVE PERFORMANCES LIKE THE IRONY OF THE AFF. THIS MEANS UNDER OUR INTERPRETATION WE PROVIDE THE BEST DISTRIBUTION OF GROUND FOR TRADITIONAL AND PERFORMANCE TEAMSB. R-SPEC IS KEY TO K GROUND. OTHERWISE, THE AFF’S RELATIONSHIP TO THE TOPIC CAN BECOME A MOVING TARGET THAT SPIKES OUT OF OUR K LINKSC. AFF ALWAYS GET PERM. R-SPEC IS KEY TO CHECKING BACK AFF BIAS IN CLASH OF CIVILIZATION DEBATE D. CROSS EX DOESN’T CHECK – THEY CAN REARTICULATE THEIR RELATIONSHIP TO THE TOPIC THROUGHOUT THE DEBATE. CAN’T CHECK BACK MOVING TARGET. 2. EDUCATION AND LIMITA. BVA( Black version of the aff a.k.a better version of the aff b.k.a Ethical version of the aff) is required given the context of the aff current directional resolve of the topic. The resolution states: The United States federal government should substantially reduce Direct Commercial Sales and/or Foreign Military Sales of arms from the United States.” We believe a black interpretation of the resolution is that debate must defend that The US should give arms to Black people as a way to reduce substantially Direct Commercial Sales and/or Foreign Military Sales of arms from the United States. It is the only ethical policy resolution the US can take as well as debaters .Livingston, Lindsay Adamson Picking Up the Gun Spectacular Performances of Firearm Ownership in the Long Civil Rights Movement. 262. 2017

Newton argued that, in the militarized culture of the United States, Black Americans had a right , guaranteed by the constitution, to keep individual arms and to publicly display them in self-defense (Bloom and Martin 2013:47; Strain 2005:153). He and Seale characterized the struggle between the US government and its Black citizens as a “politicalmilitary whole” that required Black Americans to think of the police as “occupying our community like a foreign troop that occupies territory” (in Seale 1970:117, 97).

B. Moving Target destroys pragmatic education. Politics is always about our relation to power. R-Spec ensures we learn pragmatic portable advocacy skills. Blackness is intrinsic to any discussion of a reduction in US Arm Sales. Blackened resolution key to meaningful international geo-political frame that is able to combat anti-blackness.VARGAS, JOÃO H. COSTA .GENOCIDE IN THE AFRICAN DIASPORA United States, Brazil, and the Need for a

Holistic Research and Political Method. 2005.

Page 145: openev.debatecoaches.org€¦  · Web view1AC . Observation 1: If I could find the spot where truth echoesI would stand there and whisper memories of my children's future. I would

The contemporary scholarship on the theory and prevention of genocide, from the perspective of those of us working to understand and oppose the myriad of anti-Black racisms in the African diaspora, is disappointing. In this literature, you will be pressed to find any reference to We Charge Genocide and the plight of Blacks in countries outside Africa. Important authors writing on the theory of genocide such as Andreopoulos (1994), Chalk and Jonassohn (1990), Charny (1991), Fein (1993), and Kuper (1981, 1994) seldom, if at all, mention the systematic and massive terror and premature deaths Blacks are subjected to in countries such as the US and Brazil. It is interesting, however, that Black genocides occurring in Africa are often presented as paradigmatic cases—the mass killing of Tutsis in Rwanda (Taylor, 2002) and Hutus in Burundi (Lemarchand and Martin, 1974), for example—as are the massacres of Indians in the US and Brazil. There is no denial that such occurrences merit all our outrage, analyses, and effort to prevent them from happening again . But what about the genocide of Blacks in polities outside Africa? Why is the moral outrage, and analytical and political will, generally absent when confronting the fates of, say, Afro-

Brazilians or African Americans? Why the silence? What does this silence around anti-Black genocide outside of Africa mean? What makes We Charge Genocide so irrelevant to those studying (and allegedly wanting to redress) genocides? I provide some of the possible answers to these questions. Horowitz (1982) perceived that, among sociologists, genocide is often seen as unfit for scientific inquiry, given its emotional content and the embarrassment that it causes. This would disqualify not only Black genocide, but all genocide from deemed serious academic agendas. Even when genocide is accepted as worthy object of study, it sometimes excludes or (implicitly at least) downplays events not related to the Jewish Holocaust (Charny, 1994: 72) . Moreover, there is a marked backlash in genocide studies against the proliferation and seeming banalization of the term, especially since the 1970s, when the interest in genocide, following the lead of Holocaust studies, proliferated. In this respect, Porter (1982: 9–10, quoted in Fein, 1993: 5) made the following remark: . . . genocide has been applied to all of the following: ‘race mixing’ (integration of Blacks and non-Blacks); drug distribution; methadone programs; the practice of birth control and abortions among Third World people; sterilization and ‘Mississippi appendectomies’ (tubal ligations and hysterectomies); medical treatment of Catholics; and the closing of synagogues in the Soviet Union. Fein (1994: 95) continued the argument, stressing that The wave of misuse and rhetorical abuse parallels the alphabet: abortion, bisexuality, cocaine addiction, and dieting have also been labeled as examples of genocide—as well as suburbanization. At times such labeling verges on the paranoid and incendiary, as when Westerners or Jews are accused of genocide by giving African-Americans AIDS. Banalization is certainly a danger when the genocide concept seems to be applied to a variety of contexts without the appropriate rigor and analysis that is required if a compelling, acceptable case is to be made. I am not making an argument for the uncritical widespread use of the definition of genocide. Rather, I want to point out that the very concern against trivialization often prevents the possibility that the definition of genocide may be applicable, especially to specific, quantifiable, and recurring social processes in the African diaspora whose results are the disproportionate victimization of Black people. ‘Definitionalism’, or the . . . damaging style of intellectual inquiry based on perverse, fetishistic involvement with definitions to the point at which the reality of the subject under discussion is ‘lost,’ that is, no longer experienced emotionally by the scholars conducting the inquiry, to the point that the real enormity of the subject no longer guides or impacts deliberation (Charny, 1994: 91) is yet another problem contributing to silence to which Black genocide outside of Africa has been relegated. In their efforts to circumscribe the scientific field, genocide theorists have glaringly excluded obvious mass killings of defined groups from their field. Such a case can be made for Black genocide in the US and other countries of the African diaspora. Kuper remarked on a related obstacle for

genocide studies, especially those concerned with producing legal and political mechanisms enforcing punishment and prevention. While the United Nations’ Convention on the Prevention and Punishment of the Crime of Genocide does provide a ‘workable definitional core for the interdisciplinary analysis and application, and it is the legally accepted definition that has been incorporated in a convention ratified by the great majority of member states of the UN’ (Kuper, 1994: 31), its applicability is seriously hampered by a core contradiction. The contradiction is the following: UN member states are constrained

to charge genocide in the International Court of Justice because, after all, states are usually the responsible for the crime of genocide. Not only do state members feel restrained in accusing other states of genocide, but also the very states that are supposed to combat genocide are usually the perpetrators. Kuper (1994: 32) gives an example: This avoidance is carried to such extremes that not a single member state was prepared to sponsor a carefully prepared memorandum to the International Court which would have declared Democratic Kampuchea to have breached its obligations under Articles I through V of the UN Genocide Convention, and this at a time when there was a threat of a return to power by the annihilatory Khmer Rouge regime. This contradiction could be interpreted as yet another cause for the reticence of Black genocide studies and preventive actions outside Africa. The United States, whose Senate took 40 years to ratify the UN resolution on genocide, historically has not been compelled to respond to accusations of its own responsibility regarding internal genocidal processes, nor would

Page 146: openev.debatecoaches.org€¦  · Web view1AC . Observation 1: If I could find the spot where truth echoesI would stand there and whisper memories of my children's future. I would

there be a member state willing to challenge such a powerful nation in times of post-9/11 US-dominated world order of increasing economic and military hegemony (Sinavandan, 2003). This may explain why We Charge Genocide is one

of the few documents accusing the US of mass terrorism against a national racialized group in the postcolonial period. There are many studies that chronicle the genocide of American Indians “We do not endorse this language ” (e.g. Chalk and Jonassohn, 1990; Legters, 1988; Porter, 1982; Thornton, 1987), but few extend their analyses to the present . While this

reasoning provides probable causes for the absence of Black genocide (outside Africa) from genocide studies, they are not sufficient. Is the fact that Black lives are noteworthily undervalued related to the silence to which the charge of their genocide is relegated? Notwithstanding the paradigmatic cases cited in the genocide bibliography of well-known massacres in Africa, domestic racism is of a different nature, one that calls into question more directly the genocide disciplines’ complicity with hegemonic common-sense and racialized knowledge. Cedric Robinson (2000), Patricia Hill Collins (1998), Robin Kelley (1997), Kimberlé Crenshaw (1995) and Gayatri Spivak (1999), among many others, have written on the close connection between western academic disciplines and their white-centric, exclusionary, and dehumanizing assumptions. I am pointing to the fact that the social sciences in the western world, and their practitioners, willingly or not, further hegemonic common sense about Blacks, even by not paying attention to their plight and the works produced by them. How many and how seriously do graduate and undergraduate students in the US and other countries of the African diaspora read and engage with the works of Black scholars such as W.E.B. Du Bois, C.L.R. James, Frantz Fanon, James Baldwin, Audre Lorde, Barbara Smith, Molefi Asante, and Angela Davis ? Not many and often not as seriously as so-called white classics would have been read — and this fact alone is a good indication of the white bias of disciplines such as sociology, political science, and anthropology, just to remain within the ‘social sciences’.

C.Ethical Black political solutions are only possible under the Neg’s interpretation.James, Joy. The Dead Zone: Stumbling at the Crossroads of Party Politics, Genocide, and Postracial Racism.473-476. 2009.

In The Black Jacobins, C. L. R. James remarks of historians, “They write so well because they see so little.”33 What is often unspoken and so unseen is the pervasiveness of violence. If genocide is taken off the table for discussion, then there is no immediacy in the struggle. Here’s the point: time does not exist, but genocidal violence does . The golden age is

American mythology. There is no evolutionary future, only the immediate struggle. Resisting violence is a mandate. If

our writing suffers because we see more than we can articulate, that’s fine. At least we tried. Residing in the dead zone, at the nexus where the flight from violence meets the deeper immersion within it, our only achievement will be to stop fetishizing achievement and romanticizing or condemning dysfunction and despair. The crossroads’ dead zone becomes a threshold, a potential site for working for emancipation. Crossing back and forth over the threshold, thought is freed from a prevalent cultural drug, U.S. exceptionalism. Globally exported as the deification of democracy abstracted from context, U.S. exceptionalism is justified by some by money and militarism; the more thoughtful point to the U.S. Constitution and the Bill of Rights. Yet either rationalization posits the religious belief that

democracy is an evolutionary trek toward freedom.34 Given the Thirteenth Amendment, the convict lease system, and the modern prison industrial complex, the United States has never known democracy severed from captivity.35 Yet democracy is rarely contextualized within systems (e.g., socialism, capitalism, consumerism, or [multiracial] white supremacy). As a freestanding idealization, it issues its own mandate: history must usher in its own golden age. Thus, the training of a truncated political imagination begins. U.S. exceptionalism positions party politics and the transcendent political leader to overshadow the agency of genocidal survivors and political resistance . Is there

anything more exceptional than a nation of white supremacy deigning to elect a black man as its chief executive? Hence, at the place where black achievement intersects with black genocide, there is a void that elicits little analytical interest among academic achievers. Most ignore the presence of the intersection, looking past the void with its supposedly obscure or muted signals—there are no yellow, orange, or red flares to indicate national black security threats—of state violence or state-incited genocide in Western democracies. Thinkers see only what is intellectually compatible with our paradigms, shaped and filtered by the dominant ideologies of the “dominant culture” (the latter phrase used by Jeremiah Wright elicited derision within mainstream media).36 In Africana thought, some ask: how could

Page 147: openev.debatecoaches.org€¦  · Web view1AC . Observation 1: If I could find the spot where truth echoesI would stand there and whisper memories of my children's future. I would

a black or Africana man ascend to the presidency while antiblack racism and genocide flourish? Despite or because of George W. Bush’s 2000 presidency as a bequest from the U.S. Supreme Court following felon disenfranchisement, racially driven intimidation at the polls, and faulty voting machinery for impoverished neighborhoods, people more clearly see the process of electing a progressive black president in the United States. Most seem befuddled by the process through which antiblack racism leads to genocide. In an era in which a black man can be elected president in a non-African nation founded on slavery and white supremacy, what is the meaning of black genocide embedded in domestic and foreign policies, and how might multiracial white supremacy mask that meaning?37 American exceptionalism has infiltrated Africana thought. This semantic infiltration shapes a discourse of entitlement. Americans and African Americans, including the newly arrived immigrants from Africa and the diaspora, are entitled to a future that appears to look like the future that whites sought to craft under capitalism and racism and sexism—the “American dream.” In a democratic state, the entitled are those (deserving) Americans and blacks, whose coronation occurs through the electoral process. John Mbiti could point out that such a concept of time (and space) is localized and not universal (quantum physics and Vedanta philosophy concur). Harold Cruise could warn that for African Americans to forget historical antecedents—steeped in genocidal policies implemented by a democratic state—would mean essentially the loss of our souls if not our minds. But neither of these arguments has much impact on politics in contemporary Africana thought. The academy’s neoliberal mandate underscores black and Africana studies as well as other critical studies (ethnic, women and gender, queer, community engagement). Africana thought that circulates as intellectual property is largely produced and disseminated in university or college programs and departments, part of the government or corporate sectors, or all of the above. Given the endowments of elite colleges and universities, Congress has increasingly questioned whether such schools deserve taxexempt status. Of course, state universities are extensions of the government and are regulated as such. Africana studies and thought may function as political parties in an academic environment with our own versions of the Republican National Committee, the Democratic National Committee, and the centrist Democratic Leadership Council, which attempts to emulate the past victories of archconservatives and reactionaries in the Grand Old Party. Academics embedded in “political parties” (that is, political agents operating only within the confines of systems dominated by elites) often do not reject achievement or Mbiti’s “concept of history moving forward towards a future climax.” In the absence of an intellectual promise or progress culminating in tangible liberation, there is no apparent (political) purpose or mission statement for Africana thought , outside of gathering more data for those dedicated to alleviating suffering, intellectual investigations, or “opportunities” and career advancement. Grappling with the issue of black genocide outside of a liberal framework is seen as the kiss of death for career-minded academics. The real and symbolic battles waged during the 2008 primaries have spun out symbolic gestures and performances that captivate a global audience and inspire loyal followers.38 Yet how do the loyalists—the new political class—perceive and respond to antiblack genocide in all of its nuanced and blatant manifestations? Sacrifices and struggles to create, institutionalize, and preserve Africana studies would promise, one hopes, a future, stable ground for further movement toward liberation. Yet we might be living in a sci-fi novel, one in which—as in the works of Butler, whose stumble on a Bay

Area curb yielded yet another ancestor—we find the convergence of the scientific and the imaginative, of the empirical and the theoretical. All have the possibility of fashioning freedom. Resisting party politics and postracial racism, Africana thought may (re)invent itself acknowledging a past that cannot be fully celebrated, a present that cannot be adequately explained in conventional terms, and a future that cannot be fully trusted to promise anything like a utopia . Dystopia? As Butler’s work suggests, dystopia is entirely possible. Yet in terms of liberation in the pursuit of (re)invention, we shall find that it is impossible to adequately contextualize any of this if, as Some of Us Are Brave asserts, the invisible woman sitting squarely in the crossroads remains unseen.39

D. Public discourse for Black people in politics is a death sentence that tenderizes Black bodies as they are subject to the differences in Anti-black violence. SO, locking the discussion of Blackness into the Neg is not only performatively anti-Black but also meaning destroys must needed social and political education.

James, Joy. The Dead Zone: Stumbling at the Crossroads of Party Politics, Genocide, and Postracial Racism. 463-5. 2009

Of course, political leaders, their parties, voters, the opposition, and nonvoters discuss genocide. That painful topic is also studied and debated in Africana thought, academia, media, and policy forums. Yet the topic of genocide is usually restricted to foreign policies and foreign countries or past histories from which we have

Page 148: openev.debatecoaches.org€¦  · Web view1AC . Observation 1: If I could find the spot where truth echoesI would stand there and whisper memories of my children's future. I would

evolved. When it comes to contemporary expressions of repression, histories are forgotten or curtailed, while the current context is vaguely rendered. For example, in the United States, calls for reparations to offset enslavement and apartheid are thought of as revisiting the past and as “divisive” and “counterproductive” to “going forward.” Those who forget that past centuries of enslavement and colonialism shape contemporary Africana crises reject Harold Cruise while embracing John Mbiti: when it comes to black disenfranchisement and exploitation, only the present day (villain) matters. Among nonvictims, genocide is an inflammatory subject because there is no ideological or moral justification for the continuance of a state that countenances genocide. The citizenry complicit in it is considered to be aberrant to the civilized world. The charge of genocide is the touchstone for allegiance or rebellion. A genocidal state is not only immoral and unlawful, but it has regressed to savagery. According to international law and the U.S. Constitution, it must be disciplined and restored to the “rule of law.” Few speak of U.S. policies as genocidal because the dominant tendency is to analyze national policies as the byproduct of specific administrations or political parties not as the consequence of a state apparatus built on and seeped in racial animus. In the land of the First Amendment, one is free to argue that, irrespective of the political party, the state manifests an antiblack (or anti-indigenous) animus that promotes premature social and physical death for its most marginalized peoples. However, in a land in which (neo)liberalism and (neo)conservatism dominate intellectual thought, that argument opens one up to being caricatured as paranoid or a buffoon—a Jeremiah Wright with footnotes. The trajectories in Africana thought are clearly delineated as they intersect and clash, and are repudiated as they take leave of each other. Prominent Africana writers such as Orlando Patterson and William Julius Wilson embody black achievement and shoulder a discourse that normalizes and validates the state by ignoring the context and its murderous excesses. One can critique legislation, party politics, and elected leaders and the legislative promises or debacles they sponsor without ever uttering the incendiary word genocide.6 Harvard scholars have published tracts on the word nigger, tracing the etymology and reflecting on emotional connotations. Yet genocide, which has a much more fearful impact on national consciousness and material well-being, is less rigorously analyzed as part of the black condition. If you don’t name it and shun the language, then you veil the phenomenon. What is also obscured is state violence, as conventional language maintains that only dictatorships, not democracies, practice racial genocide. Convention assumes that electoral democracies have a failsafe mechanism—an enlightened and empowered citizenry—that prevents their participation (except as liberators) in genocidal practices. It is thus not surprising that those most targeted by historical and contemporary state excesses are those most likely to crash into its apparatuses: racially fashioned policing and the prison industrial complex, homelessness, substandard schools and housing, foster care for children marred by indifference, inadequate oversight and resources, the poverty draft into an immoral war, and “shoot-to-kill” edicts for (black) survivors of New Orleans’s substandard levees designed by the Army Corps of Engineers.7 It’s no wonder that some stumble at the intersection where elites undertheorize contradictory conditions marked by class and opportunity (or opportunism, more properly phrased) as they are carried off by the trajectory of black achievement and greater America’s expanding embrace. In the United States, we are routinely asked to fall and genuflect at the crossroads: to acknowledge the positive in U.S. and global “race relations” without dwelling on the negative—the continuance of racial repression and disenfranchisement. But 2008 is the first time that we may likely (in)voluntarily stumble from our own frustrated desires and longings. For the ascendancy of a liberal black to the U.S. presidency must mean something profound, if democracy’s future is to culminate in a “golden age” and blackness is to have a place at the table. If antiblack genocide remains a feature of that utopian democracy, the profound becomes profoundly disappointing— although, of course, not for everyone in a democratic state. Racial genocide has been a historical fixture in Western democracies as citizens amassed existential wealth (white privileges) and material wealth (capital and militarism) through antiblack policies. But those realities tend to be muted in public discourse, where blacks and other people of color are invited to sit at the table of accumulation as national and global narratives note progress.

Page 149: openev.debatecoaches.org€¦  · Web view1AC . Observation 1: If I could find the spot where truth echoesI would stand there and whisper memories of my children's future. I would

E. Deliberative Education Good – Real deliberation is always tied to the material and the everyday. (Matt Stannard, Department of Communication and Journalism, University of Wyoming, Faculty Senate Speaker Series Speech, April 18, 2006, http://legalcommunication.blogspot.com/2006/08/deliberation-debate-

and-democracy-in.html)

We tend to think reason will prevail—or that if it doesn’t, we can explain its failure discursively. This blindness concerning materiality is precisely why deliberative politics must include the voices of the materially disadvantaged. It is why the "perspective of the oppressed" is not only morally necessary, but epistemologically necessary . Within Habermas’s communicative ethics is found both the classic Rawlsian test of how policies and arrangements affect the least advantaged members of society, and the Marxian imperative for

emancipation from the artificial and enforced scarcity and silence of economics. This is vital to making what we do relevant—because even if democratic

legitimacy depends on discursive justification, such justification occurs in a "dirty" material world, the "excrement" of which Marx wrote as a metaphor for the day-to-day material challenges of ordinary people. The aggregate of those material challenges constitutes the very conditions of humanity itself , and awareness of those conditions in their totality requires a commitment to deliberation in all levels of the social world.The complexity and interdependence of human society, combined with the control of political decision making—and political conversation itself—in the hands of fewer and fewer technological "experts," the gradual exhaustion of material resources and the organized circumvention of newer and more innovative resource development, places humanity, and perhaps all life on earth, in a precarious position .

3. ETHICS A. Responsibility starts in the building of the 1AC. B. Blackness is never germane to the topic unless we specify our relation to it.

This is a unique reason to vote neg. C. Our educational standard is revolutionary. If we aren’t learning how to

challenge systems of oppression then we need to stop talking because under the aff interpretation of the assumed no aff can be inherent or produce any decision of their own. Revolutionary decision making key. (Yellow is for a Non-Black reading of the card) (Green + Yellow is read a Black reading of the card)

Anthony Paul Farley, 2005. “Perfecting Slavery.” Loyola University Chicago Law Journal, Volume 36 2005. Pps. 112-115

We who have slavery with us still are made up of memory and forgetting. Freedom is our calling. Slaves are not called. Education is required to

pursue our calling. Education is dangerous to slavery, to the system of white-over-black . James Baldwin, speaking

to Harlem teachers, noted: The paradox of education is precisely this—that as one begins to become conscious one begins to examine the society in which he is being educated. The purpose of education , finally, is to create in a person the ability to look at the world for her/himself, to make his own decisions, to say to her/himself this is black or this is white, to decide for her/himself whether there is a God in heaven or not.35 Baldwin continued:[ I]f I were a teacher in this school, or any Negro school . . . dealing with Negro children, who were in my care only a few hours of every day and would then return to their homes and to the streets, children who have an apprehension of their

future which with every hour grows grimmer and darker, I would try to teach them—I would try to make them know— that those streets, those houses, those dangers, those agonies by which they are surrounded are criminal. I would try to make each child know that these things are a result of a criminal conspiracy to destroy her/him. I would teach her/him that if she/he intends to be [an adult], she/he must at once decide that she/he is stronger than this conspiracy and that she/he must never make her/ his peace with it. And that one of his weapons for refusing to make his peace with it and for destroying it depends on what he decides he is worth.36

Page 150: openev.debatecoaches.org€¦  · Web view1AC . Observation 1: If I could find the spot where truth echoesI would stand there and whisper memories of my children's future. I would

D. . There is In-Round and Out-of-Round Abuse: What we justify matters for our wellbeing in debates and what becomes possible in the debate community. It’s too late for the AFF to spec their R now. Damage is done.

E. Voter for Ethics, Limits and Education

Page 151: openev.debatecoaches.org€¦  · Web view1AC . Observation 1: If I could find the spot where truth echoesI would stand there and whisper memories of my children's future. I would

Vs. Set Col

The 1AC’s attempt to wage a war of position on behalf of indigenous political community only secures the jurisprudential dream-work of civil society by granting native bodies spatial and temporal recognition – this politics is parasitic on black social death as political community itself is only imaginable through a politics of symbolic mediation where the slave remains a fungible objectWilderson 16 (Frank B Wilderson III, associate professor of African American Studies and Drama at UC Irvine, PhD in Rhetoric and Film Studies from UC Berkeley, March 2016, “Doing time in the (psychic) commons: Black insurgency and the unconscious,” in Time, Temporality and Violence in International Relations: (De)fatalizing the Present, Forging Radical Alternatives, pp 92-4) gz

I am humbled by the courage it took to use the space and time allotted to read atrocities into the public record, often at the expense of

adjudicating the charges levied against them. But the reportage of atrocities is just that, reportage: laden with spectacle and lights on sustained meditations on trauma . How can a sense of redress (juridical or political) emerge from a context where sustained meditations on trauma have no purchase , where the spatial and temporal resonances of the trauma Black people experience is barred from being recognized by and incorporated into the corpus of Human events ?

There are important continuities ¾tween the ethical dilemmas raised when a Slave stood before the bar in the nineteenth century and when the BLA stood before the bar in the 1970s and early 1980s. The Dred Scott trials are exemplary of this. Arguably, Dred Scott was pushing in the opposite direction from the BLA; he wanted to depoliticize the court so it would focus on a narrow (and just) interpretation of existing law. But I am not asserting historical continuity of courtroom strategies. The historical continuity of the Dred Scott case and the BLA trials isn’t a continuity of performance but a continuity of position.

Chief Justice Taney’s 1857 majority decision was an early rejoinder to the BLA’s demand 124 years later that their standing before the court be recognized as political rather than juridical. Taney returned Dred Scott to slavery by arguing in the opposite direction, from the juridical to the

political. Taney argues that Dred Scott has no standing as a juridical subject because he has no standing as a political subject. ‘The question is simply this,’ Taney writes, ‘Can a negro whose ancestors were imported and sold as slaves, become a

member of the political community?’ 7 Taney is compelled to compare the Black to the Indian as a necessary prerequisite to legitimating the court’s decision to re-enslave Dred Scott . In so doing, he triangulates the dyad between the Human and the Black with the Indian:

The situation of [the Black] population was altogether unlike that of the Indian race. The latter, it is true, formed no part of the

colonial communities and never amalgamated with them in social connections or in government. But although they were uncivilized, they were yet free and independent people, associated together in nations or tribes, and governed by their own laws. Many of the political communities were situated in territories to which the white race claimed the ultimate right of dominion. 8

From the opening of Taney’s tangential pursuit of Native Americans, it seems they constitute a defeated and denigrated identity within the Human race, devalued Humanity as opposed to the embodiment of social death (Blacks). Taney’s writing speaks of a being with subjective presence and of a community with the capacity for ‘perspective of consciousness,’ ‘[u]ncivilized . . . yet free and independent . . . associated together in nations or tribes, and governed by their own laws’ (Gordon 1995: 183). Furthermore, Indians are not natally alienated because their claims to their offspring are recognized by and incorporated into the world. By extension, their right to govern is acknowledged beyond their circle ( temporal recognition ), just as their place names have resistance in the eyes of the Other (spatial recognition):

Page 152: openev.debatecoaches.org€¦  · Web view1AC . Observation 1: If I could find the spot where truth echoesI would stand there and whisper memories of my children's future. I would

‘Many of the political communities were situated in territories to which the white race claimed the ultimate right of dominion’ (Gordon 1995: 184).

Taney imposes imaginary and fantastic formulations on what heretofore in the ruling has been sober and realist prose buttressed by relational (albeit racist) logic:

Indian Governments were regarded and treated as foreign Governments, as much so as if an ocean had separated the red man from the white; and their freedom has constantly been acknowledged, from the time of the first emigration to the English colonies to the present day, by the different Governments which succeeded each other.

(Neale 1990: 27)

Through a process of condensation and displacement, or jurisprudential dream-work , Taney maps the imagery of settlerism onto the body of Indigenism . Like the dreamer who brings his own water to the beach, Justice Taney has to manufacture an ocean out of dry land, lest the analogy between Whites and Indians crumble. The declaration that Africa is void of political community, coupled with the fantasy of immigration the court ’s unconscious mobilized to situate the Indian within political community , was a vital intervention that reminded the lower courts that general dishonor and natal alienation are two of the three constitutive elements of slavery, not proprietary claims. Dred Scott has no juridical standing because he is not a member of political community ; he is not a member of political community because he is a genealogical isolate ; his status as a genealogical isolate is an effect of his subsumption by structural violence unique to his paradigmatic position . To hear his case on the basis of proprietary claims or, more to the point, to hear it at all , is to breach the divide between the living and the dead – those who can mediate their existence through transindividual objects and those who , at best, can only be a mediating object.

This paradox of existence in which one can never be the subject of symbolic mediation even though one is always already an object of symbolic mediation is the quandary encountered by Kuwasi Balagoon and Dred Scott when they went to trial. The textual heat of Kuwasi Balagoon’s poem is not cathected by transindividual concepts like land or labor power, but dispersed throughout an array of bodily violations, horrifying images indexical of the absence of his capacity to lay claim to mediating concepts. In Balagoon’s case, we do not get a picture of someone whose native land has been stolen , whose labor power has been usurped, or whose culture has been quashed and corrupted, but of someone whose condition of possibility is elaborated by violence too comprehensive to comprehend , violence so totalizing it reenacts its prehistory, repeatedly, in the present (Patterson 1982).

The 1AC stakes itself on its subjective capacity to generate friction in the face of violence which monumentalizes subjectivity against black incapacity—the very remains of symbolic redress of familial and cultural ties underlies relations that pit indigeneity as antagonistic to blackness. The issue is not cartographic redress, but that the call for resignification is symptomatic of antiblack integration. Wilderson et al 2016 (Interview of Frank Wilderson by Samira Spatzek, and Paula von Gleich. Frank Wilderson is a Professor of Drama and African American Studies at the University of California, Irvine, and the author of two books: the memoir Incognegro (2008) and the monograph Red, White and Black: Cinema and the Structure of U.S. Antagonisms (2010). Samira Spatzek and Paula von Gleich, who conducted the interview, are Ph.D. candidates in American Studies at the University of Bremen. They first met Frank Wilderson when he was a senior research fellow of the Alexander von Humboldt Foundation on a research stay in Bremen in 2013 and 2014. “‘The Inside-Outside of Civil Society’: An Interview with Frank B. Wilderson, III.” Black Studies Papers 2.1: 4–22.: http://nbn-resolving.de/urn:nbn:de:gbv:46-00105247-16 (2016)) ipartman

Page 153: openev.debatecoaches.org€¦  · Web view1AC . Observation 1: If I could find the spot where truth echoesI would stand there and whisper memories of my children's future. I would

P: I have a follow-up question on that. You just talked about the position of the Jew and how the Holocaust is perceived in Europe and in Germany specifically. And in your book Red, White, and Black… you give a specific position to Native Americans as the “Red” or the “Savage.” Could you say a little more about that? Why do you

give that specific position to this group of people and not to any other group? F: As soon as Red, White & Black was published, Jared Sexton critiqued the error of my generosity towards Native peoples in the Americas. My book came out in 2010, and Jared Sexton has been teaching courses since then which have been tweaking that as an error. And I think he’s right . To footnote this, if Germans feel guilty about the Jews, everybody here in the United States feels guilty about American Indians. Part of my guilt coming through—because I was a graduate student when I wrote this book— was that, you know, here are a people who, just in the United States alone , were 12 to 18 million, and now they’re 1.6 million, so they’ve been slaughtered. But what Jared points out is that, in pure genocide numbers, the genocide of Africans dwarfs that, for one. Number two, I was dealing with modernity, the 1600s, and anti-Blackness—and I’m not actually doing this myself, two or three graduate students are working on how anti- Blackness is essential to the ability to say the word ‘Arab

family’—goes back to 625 A.D., it’s a project that the Arabs begin in order to turn Africa into the place of social death, and they hand this project over to the Por Portuguese in 1452. And if you think of it like that, what you’re able to see is that there is real conflict between Native people in the Americas and Europeans, but it’s a conflict, it’s not so much an antagonism . It looks more like the Jewish Holocaust and less like African slavery , that is, less like the Maafa (or

Holocaust of Enslavement). Since writing Red, White & Black at least two or three books have come out which are history books, which do a very good job of explaining how the enslavement of Blacks was central to Native American civil society as it was to white civil society , especially in the Southeast. So it’s a complicated thing: Native Americans stand in what seems to be an antagonistic relationship to whites because the word ‘Indian’ is not a word prior to Columbus. It’s like the word ‘Africa’ with the Arabs. ‘Indian’ is a word that is implicated with genocide: you can’t think

‘Indian’ outside of genocide. And so, ‘Indian’ is a positional nomenclature, positioned in a paradigm, whereas Choctaw or Lakota or Apache marks a cultural name of an identity within the position of ‘Indian. ’ And so they stand in what seems to be an antagonistic relationship to Europeans, but they also stand in an antagonistic relationship to Blacks. In my chapter, “Savage Negrophobia,” I went into that and I should have written more about that. But

the laws of the B.I.A., the Bureau of Indian Affairs, seem to work in exactly the opposite way: in other words, there seems to be a blanket imperative that American Indians marry whites in order to advance, and that is very interesting, because civil society and its murderous juggernaut polices interracial marriages by making sure that Blacks can’t marry whites, but it also encourages interracial marriages by encouraging Indians to marry whites. In the libidinal economy, American Indians are not the kind of absolute contaminant that Blacks are, because in the state of South Dakota , where many American Indians were pushed off their lands, you 1 also have a very large reservation, the Pine Ridge Indian reservation . The government came in , and they said: “If you want to own land on Pine Ridge, you only qualify if you’re married to a white person.” And they also go into the homes to correct the behavior of Native Americans— how they eat using utensils, forks and knives, how they clean their houses, etc.—to whiten that process to help them get more integrated. The socalled (derogatorily) ‘half-breeds,’ people who have an American Indian parent and a White parent, have more privileges in terms of land ownership, voting rights, etc. In some ways , American Indians are a liminal category, and in other ways they are more profoundly on the side of “junior partners” and antagonistic to Blacks.

The collective unconscious of civil society locates Blackness as a phobic object within an external superviolence that exceeds rational utility and exists only to preserve the psychic health of Human subjectsWilderson 14 (Frank B Wilderson III, associate professor of African American Studies and Drama at UC Irvine, PhD in Rhetoric and Film Studies from UC Berkeley, October 2014, ““We’re trying to destroy the world”: Anti-Blackness & Police Violence After Ferguson: An Interview with Frank B. Wilderson, III,” http://sfbay-anarchists.org/wp-content/uploads/2015/01/frank-b-wilderson-iii-were-trying-to-destroy-the-world-antiblackness-police-violence-after-ferguson.pdf, modified) gz

FW: That was at Haile Gerima’s bookstore in DC, and it was an all- Black audience, so I didn’t have my guard up. I might have said it differently in a classroom, who knows. What I meant there was, well it was a bit tongue in cheek, but of course I hate police brutality. I haven’t been brutalized in the past ten years, but when I was brutalized I did hate that. I hate the harassment However, I feel that what my critical work is

trying to contribute is to say that Black people in the US and worldwide are the only people -- and I say this

categorically -- for whom it is not productive to speak in terms of ‘police brutality’. I know that we have to,

because we’re forced to speak in these terms, and there is a way in which all Black speech is always coerced speech, in that

Page 154: openev.debatecoaches.org€¦  · Web view1AC . Observation 1: If I could find the spot where truth echoesI would stand there and whisper memories of my children's future. I would

you’re always in what Saidiya Hartman would call a context of slavery: anything that you say, you always have to think, ‘what are the consequences of me speaking my mind going to be?’ The world -- and this goes for Democracy Now, it goes for our post-colonial comrades, etc. -- is not ready to think about the way in which policing affects Black people. And so what we have to do is ratchet-down the scale of abstraction , so that we don’t present the world with the totality of our relation to the police, which is that we are policed all the time , and everywhere . We have to give the world some kind of discourse, some kind of analysis in bite- size pieces that they are ready to accept, so that they can have some kind of empathy for us, some kind of political or legal adjudication . That is why police brutality becomes the focal point of the problem .

Police brutality has never identified our problem. Our problem is one of complete captivity from birth to death , and coercion as the starting point of our interaction with the State and with ordinary white citizens (and with ordinary Latino, Mexican, Asian citizens, Native Americans). And so when I was in that room and I said ‘I don’t hate police brutality, I hate the police’, I think most of the people in that room immediately understood what I was saying, but also understood the problems with going outside and saying that. Here’s one little example of how this conundrum or paradox affects the way we

can speak to White people and our so-called ‘allies of color’. In Tulia, TX, in 1999, 45 Black people and about two Latinos were arrested

in a one-night drug bust. In other words, roughly 10 percent of the Black population were arrested in one night. All of them were convicted. There is a film about this that people can find online. What’s interesting to me is not the celebratory political and emancipatory nature of the film, which ends by saying ‘at the end of the day we were able to get most of the convictions overturned,

because the undercover agent did not have evidence’. There was one undercover agent who indicted 45 Black people and two Latinos. But he did not come to court with cocaine. He came to court with this word. And what was

interesting to me about that was that when jurors were interviewed about that, and people said to them, ‘So you convicted these kids, some to 200 or 300 years, on no evidence, but on the word of one police officer .

Would you want that to happen to your child?’, one of the jurors said—without any sense of irony—‘if it was my child, we’d need evidence’. So the problem then is not where the film situates the problem, or where the media

situates it, i.e. in the rogue actions of the police . The problem is in the libidinal economy , which is to say in the collective unconscious of everybody else. And if we were to actually understand that better, we’d understand that

Blackness is always-already criminalized in the collective unconscious. The only problem for white supremacy and anti-Blackness when it’s happening to Black people in Mexico for example, is one of logistics , of mechanics , which is to say, ‘how can we make the criminalization stick?’ It’s not a question of something wrong

taking place, that these Black people are suffer or exist under police brutality. Policing—policing Blackness—is what keeps everyone else sane. And if we can start to see the policing and the mutilation and the aggressivity towards Blackness not as a form of discrimination , but as being a form of psychic health and well-being for the rest of the world , then we can begin to reformulate the problem and begin to take a much more iconoclastic response to it .

JB - This idea that there is a sort of necessity, for the quality of life—i.e. that the existence of an anti-Black perspective is life for those who are involved in the mutilating, torturing, terrorizing Black people...what’s preventing Black people from understanding this? Some folks, such as Fanon, Frances Cress Welsing, etc., have attempted to grasp the psychic relation between the terrorizer and the terrorized, but most folks won’t go so far as to say that there is a health and even a sense of pleasure in that libidinal economy for Whites to enact an anti-Black perspective. What’s preventing folks from understanding that? FW - Although my work is fine, I would really encourage listeners to read two Black authors, Hortense Spillers and Fanon’s Black Skin White Masks, in particular for those moments where they are at a loss to address what they have come up against. What we tend to do -- and I’m not criticizing this, we have to help Black people make it through the day, which is the job of

Black psychologists and Black psychoanalysts -- but we also need people like me, who point to the failures of what Fanon called the “healthy infrastructure of the psyche”. And then I’d also suggest moving to the more uncompromising literature of David Marriot and Jared Sexton, who will deal with psychoanalysis but will not offer any cure. Here’s the deal: in a nutshell, every other group lives in a context of violence which has what I

would call a sort of psychological grounding wire , which means that they can write a sentence about why they are experiencing that violence. Native Americans can write a sentence that says ‘I’m experiencing

Page 155: openev.debatecoaches.org€¦  · Web view1AC . Observation 1: If I could find the spot where truth echoesI would stand there and whisper memories of my children's future. I would

violence because this is an ongoing tactic within a strategy of colonization’ . White feminists can say the

same, that ‘this is an ongoing tactic within a strategy of patriarchy’. For a Black person to try and emulate that kind of interpretive lens, the problem becomes a lot bigger. For us this is the ongoing tactic of a strategy for human renewal. The violence against us becomes a tactic within a strategy to secure Humanity’s place. It’s not a tactic in an ongoing strategy to take our land away, or to take our rights away. We never had any rights.The other thing is that our psyche does not obey the objective laws of the structure. The simple way of putting that would be to say that we exist in an external superviolence , and we exist in an internal soup which has self-hatred as one of its main components. One of the things that Marriot and Fanon

each say is that, generically speaking, the structure by which human beings are recognized by other human beings and incorporated into a community of human beings, is anti-slave . And slaveness is something that has consumed Blackness and Africanness , making it impossible to divide slavery from Blackness . Even if I say to myself, “I am not a Slave”, we don’t make our own way in the world . So we know every day, before walking out of the house—and I think the American Black knows it quicker, like say at age 3, the Caribbean and

African Black might know it a little bit later on in life, like Fanon says, ‘I was 18 when I learned it’—that we cannot enter into a structure of recognition as a being , an incorporation into a community of beings, without recognition and incorporation being completely destroyed . We know that we are the antithesis of recognition and incorporation. And sometimes we build to a point that we can’t even call it political because it’s bigger than politics, a point of mobilization and organization and theorization that is in some way informed by this, and we just set it off, and I think that Harriet Tubman, Nat Turner, and the Black Liberation Army are episodes of that. But the response to these moments, where we recognize that we cannot be recognized and we move on that, the response is so overwhelmingly violent that it doesn’t seek to end the conflict -- say in Algeria or Vietnam -- it seeks to crush us to the point that nobody ever gets that idea in their head again. Normally people are not radical, normally people are not moving against the system: normally people are just trying to live, to have a bit of romance and to feed their kids . And what people want is to be recognized, to be incorporated. And when we understand that recognition and incorporation are generically anti-Black, then we don’t typically pick up the gun and move against the system, we typically try to find ways to be recognized, to be incorporated, even though that’s impossible. And I think that our language is symptomatic of that when we say that ‘I don’t like police brutality’. Because, here we are saying to the world, to our so-called ‘people of color allies’ and to the white progressives, ‘we’re not going to bring all the Black problems down on you today . If you could just help us with this little thing, I won’t tell you about the whole deal that is going on with us .’

TB: If we agree with your thesis, then what is the framework of resistance? How do we resist, either physically or psychologically?

FW: Your question makes me admit something. Whenever a Black person comes to speak to other Black people, it’s incumbent upon the Black people who are listening to decide how useful this person is to me in what they are saying, and what aspects of my problem can what this person’s saying address. I think more specifically, that professors are by and large categorically disqualified or unqualified to make pronouncements on resistance. I think that when Fanon talks about hallucinatory whitening, I think that whether you’re talking like me, or like an afro-centrist, or an integrationist, that this is so much a part of what it means to be a professor. I feel like cussing people out all the time. But if I do that, and I violate University of California’s civility laws, tenure or not I’m out the door, right? And that tempers my speech. So I think that

what I have to offer is not a way out . What I have to offer is an analysis of the problem . And I don’t trust me as much as I trust Black people on the ground. So having said that, that’s one part of my answer.

The other part is that, as Saidiya Hartman has said, Black liberation presents us with the prospect of a kind of liberation that is so totalizing (i.e. that it is what Fanon says on page 100, quoting Aimé Césaire: ‘ the end of the world’), that it can’t be ratcheted down and put into political language . If I’m right that the problem that Black people are in is not colonial exploitation and not racism but social death -- which is not to say that Black people don’t experience racism and that Black poor people are not exploited, but that once

Page 156: openev.debatecoaches.org€¦  · Web view1AC . Observation 1: If I could find the spot where truth echoesI would stand there and whisper memories of my children's future. I would

all that’s over, we’re still going to be socially dead -- then I think that we actually don’t have a political framework to deal with that , certainly not in Marxism, Feminism, and post-Colonialism . I’m writing about this

now [1]. The beautiful actions of the BLA are bigger than the political discourse of Marxist-Leninism or New Afrikan discourse through which they tried to make sense of that. But as has been pointed out elsewhere, one of the problems that the BLA always had was that they were not only coerced by the police but were at the whims of white radical allies. When Fanon says that the Black person is a ‘stimulus to anxiety’ , and that this is very different from the Jew, since the latter is a stimulus to anxiety because of x, y or z : in the fantasy world of anti-Semitism, he or she is going to do well on all the exams, and there won’t be any space for my kid in the French university; or

they’re going to take over the world economic system, etc. All that stuff, where you can put language to their anxiety, makes the Jew, the Native American, the post-colonial subject, a much more malleable phobic object than the Black. The Black is a phobic object because he or she [they] presents me with a problem that is beyond language , that leaves me with no way to redress what this person represents . This person is the antithesis of Humanity. And there are moments in which we have seized that esprit de corps even if we are not able to speak to each other in that stark of terms, and we move. I mentioned a couple a few minutes ago. I think that we need to keep with those moments. I also

think we need, in our political organizing, to be aware of how impoverished our articulated agenda is in comparison with the suffering that we actually experience

Vote negative to move beyond the event of land dispossession as to dwell in the historical stillness of slavery as an absolute event-horizon. The 1AC’s attempt to return to the site of the colonial event reinvests in a redemptive fantasy that ruses slaveness as an object-event projected within settler/master grammars. The alternative is a corrective to this parasitic desire for relationality and coherent register of sovereignty, which monumentalizes lush capacities against black flesh, in favor of a gesture to dance in the death of blackness. Welcome to the end of the world, please take your seats. Wilderson 15 (Dr. Frank B. Wilderson III, University of California, Irvine, "Afro-Pessimism and The End of Redemption", https://humanitiesfutures.org/papers/afro-pessimism-end-redemption/) ipartman

At first blush an exegesis might be seduced into emphasizing what the poems have in common—the ravages of structural violence on two oppressed populations of color. But another look reveals that the two poems are actually

symptomatic of the fact that violence against Native Americans is not analogous to the violence by which Blacks are elaborated and positioned. The violence of social death (that violence which elaborates and reproduces the slave) is fundamentally different from the violence which usurps Native American land and attempts to destroy the Indian’s cultural and territorial sovereignty . The imaginative labor of these poems is symptomatic of this difference. In the first section of Sand Creek, the poem establishes the filial integrity of the people who are being massacred ("men [who] think…[and] women who grab hold of children, loving them, and saving them for the generations

who would enjoy the rain…") So, what we have is an intuition on the part of the poet that even though the people being killed are seen as a degraded form of humanity , their humanity is fundamentally acknowledged ; and, in addition , there is a symbiosis (a kind of cruel interdependence) between the genocided victims (in the

opening part of the poem) and the descendants of those committing the genocide ("skilled butchers, aerospace engineers,

physicists…"). In other words, the relational status of both the Indian victims and the White oppressors is established—a reciprocal dynamic is acknowledged (between degraded humanity, Indians, and exalted humanity , White settlers) .This reciprocal dynamic is based on the fact that even though one group is massacring the

Page 157: openev.debatecoaches.org€¦  · Web view1AC . Observation 1: If I could find the spot where truth echoesI would stand there and whisper memories of my children's future. I would

other, both exist within the same paradigm of recognition and incorporation. Their relation is based on a mutual recognition of sovereignty . At every scale of abstraction, body, family, community, cosmology, physical terrain, Native American sovereignty is recognized and incorporated into the consciousness of both Indians and settlers who destroyed them . The poem’s coherence is sustained by structural capacity for reciprocity between the genociders and the genocided. This structural reciprocity gives the poem a vision of hope amid the violence, manifested in a sense of spatial presence (images of land and weather) and in Ortiz’s sense that for both groups a future is possible.

This means the violence the Indians suffer has a utility (confiscation and occupation of land) that makes it legible and coherent."Law Abiding" is predicated on the absence of reciprocity, utility, and contingency that Simon Ortiz’s poem takes for granted. Absence of humanity. In fact, the poem suggests that a family of murdering, inanimate bullets could have its grief and

loss processed as grief and loss more readily than the family of a Black murder victim. "Law Abiding" doesn’t assume that the touchstones of cohesion which make filiation legible will or can be extended to Blacks. There is — in this

poem— no mutual futurity into which Blacks and others will find themselves . The future belongs to the bulle t . Filiation belongs to the bullet. Our caring energies will be reserved not for the Black but for the bullet . Reciprocity is not a constituent element of the struggle between beings who are socially dead and those who are socially alive—the struggle between Blacks and the world. We need to apprehend the profound and irreconcilable difference between White supremacy (the colonial utility of the Sand Creek massacre)

and anti-Blackness (the human race’s necessity for violence against Black people). The antagonism between the postcolonial subject and the settler (the Sand Creek massacre, or the Palestinian Nakba) cannot —and should not —be analogized with the violence of social death: that is the violence of slavery , which did not end in 1865 for the simple reason that slavery did not end in 1865. Slavery is a relational dynamic— not an event and certainly not a place in space like the South; just as colonialism is a relational dynamic—and that relational dynamic can continue to exist once the settler has left or ceded governmental power. And

these two relations are secured by radically different structures of violence . Afro-Pessimism offers an analytic lens that labors as a corrective to Humanist assumptive logic . It provides a theoretical apparatus that allows Black people to not have to be burdened by the ruse of analogy—because analogy mystifies , rather than clarifies, Black suffering. Analogy mystifies Black peoples’ relationship to other people of color. Afro-Pessimism labors to throw this mystification into relief — without fear of the faults and fissures that are revealed in the process . Let me state the proposition differently: Human Life is dependent on Black death for its existence and for its conceptual coherence. There is no World without Blacks , yet there are no Blacks who are in the World . The Black is indeed a sentient being, but the hobble of Humanist thought is a constitutive disavowal of Blackness as social death , a disavowal that theorizes the Black as degraded human entity (i.e., as an oppressed worker, a vanquished postcolonial subaltern, or a non-Black woman

suffering under the disciplinary regime of patriarchy). The Black is not a sentient being whose narrative progression has been circumscribed by racism, colonialism, or even slavery for that matter. Blackness and Slaveness are inextricably bound in such a way that whereas Slaveness can be disimbricated from Blackness, Blackness cannot exist as other than Slaveness

Page 158: openev.debatecoaches.org€¦  · Web view1AC . Observation 1: If I could find the spot where truth echoesI would stand there and whisper memories of my children's future. I would

The alternative lodges a radical demand to inhabit the radical groundlessness of unsovereignty in order to disrupt the ontological basis of anti-blackness and its accompanying settler colonial ambitions – this requires rejecting the affirmative in order to push their demand for recognition into dissolution – this radicalizes the affirmative project at the very point where it is no longer recognizable to itselfSexton 16 (Jared Sexton, associate professor of African American Studies at UC Irvine, associate professor of Film and Media Studies at UC Irvine, PhD in ethnic studies from UC Berkeley, July 2016, “The Vel of Slavery: Tracking the Figure of the Unsovereign,” Critical Sociology Volume 42 Numbers 4-5, modified) gz

We might contrast the unsuspecting theoretical status of the concept of settler colonialism in Native Studies with its counterpart in Black Studies: racial slavery. I remarked above that any politics of resurgence or recovery is bound to regard the slave as the position of the unthought. This does not suggest, however, that Black Studies is the field in which slavery is, finally, thought in an adequate way. The field of Black Studies is as susceptible to a politics of resurgence or recovery as any other mode of critical inquiry. Which is to say

that the figure of the slave and the history of the emergence of the relational field called racial slavery remains the unthought ground of thought within Black Studies as well . The difference, provisionally, between these enterprises is that whereas Native Studies sets out to be the alternative to a history of settler colonialism and to pronounce the decolonial intervention, Black Studies dwells within an un-inheritable, in-escapable history and muses upon how that history intervenes upon its own field , providing a sort of untranscendable horizon for its discourse and imagination. The latter is an endeavor that teaches less through pedagogical instruction than through exemplary transmission : rather than initiation into a form of living , emulation of a process of learning through the posing of a question , a procedure for study, for black study , or black studies , wherever they may lead . Native Studies scholars are right to insist upon a synthetic gesture that attempts to shift the terms of engagement .

The problem lies at the level of thought at which the gesture is presented. The settler colonial studies critique of colonial studies must be repeated, this time with respect to settler colonialism itself, in a move that returns us to the body in relation to land, labor, language, lineage – and the capture and commodification of each – in order to ask the most pertinent questions about capacity , commitment , and concept . This might help not only to break down false dichotomies, and perhaps pose a truer one, but also to reveal the ways that the study of slavery is already and of necessity the study of capitalism , colonialism and settler colonialism , among other things; and that the struggle for abolition is already and of necessity the struggle for the promise of communism , decolonization , and settler decolonization , among other things . Slavery is the threshold of the political world , abolition the interminable radicalization of every radical movement. Slavery, as it were, precedes and prepares the way for colonialism , its forebear or fundament or support . Colonialism, as it were, the issue or heir of slavery , its outgrowth or edifice or monument . This is as true of the historic colonization of the Third World as it is the prior and ongoing settler colonization of the Fourth.23

‘ The modern world owes its very existence to slavery’ (Grandin, 2014a).24 What could this impossible debt possibly entail? Not only the infrastructure of its global economy but also the architecture of its theological and philosophical discourses , its legal and political institutions , its scientific and technological practices , indeed, the whole of its semantic field (Wilderson, 2010: 58). A politics of abolition could never finally be a politics of resurgence, recovery, or recuperation. It could only ever begin with degeneration , decline , or dissolution . Abolition is the interminable radicalization of every radical movement , but a radicalization through the perverse affirmation of deracination , an uprooting of the natal, the nation, and the notion , preventing any order of determination from taking root, a politics without claim , without demand even, or a politics whose demand is ‘too radical to be formulated in advance of its deeds’ (Trouillot, 2012: 88).25The field of Black Studies consists in ‘tracking the figure of the un sovereign’ (Chandler, 2013: 163) in order to meditate upon the paramount question: ‘ What if the problem is sovereignty as such’ (Moten, 2013)? Abolition , the political dream of Black Studies, its unconscious thinking, consists in the affirmation of the unsovereign slave – the affectable , the derelict , the monstrous , the wretched 26 – figures of an order altogether different from (even when they coincide or cohabit with) the colonized native – the occupied, the undocumented, the unprotected, the oppressed. Abolition is beyond (the restoration of) sovereignty . Beyond

Page 159: openev.debatecoaches.org€¦  · Web view1AC . Observation 1: If I could find the spot where truth echoesI would stand there and whisper memories of my children's future. I would

the restoration of a lost commons through radical redistribution (everything for everyone), there is the unimaginable loss of that all too imaginable loss itself (nothing for no one).27 If the indigenous relation to land precedes and exceeds any regime of property, then the slave’s inhabitation of the earth precedes and exceeds any prior relation to land – landlessness. And selflessness is the correlate . No ground for identity , no ground to [be] stand (on). Everyone has a claim to everything until no one has a claim to anything. No claim. This is not a politics of despair brought about by a failure to lament a loss, because it is not rooted in hope of winning . The flesh of the earth demands it: the landless inhabitation of selfless existence .

Page 160: openev.debatecoaches.org€¦  · Web view1AC . Observation 1: If I could find the spot where truth echoesI would stand there and whisper memories of my children's future. I would

SC Case Decolonization can not occur within the academyMorgensen 12 , “Destabilizing the Settler Academy: The Decolonial Effects of Indigenous Methodologies” Scott Lauria Morgensen (assistant professor of gender studies at Queenrsquo;s University in Kingston, Ontario. He is coeditor ofQueer Indigenous Studies: Critical Interventions in Theory, Politics, and Literature.) American Quarterly, Volume 64, Number 4, December 2012, pp. 805-808 (Article) Published by Johns Hopkins University Press DOI: https://doi.org/10.1353/aq.2012.0050 HCH

Indigenous methodologies thus demand the interrogation of colonial academic procedures . Many critiques exist of the incapacity of universities to support Indigenous research , a situation that all too often follows explicit re- strictions on such work.8 Yet by contrast, my experience in Canada illuminates growing efforts by settler states and universities to accommodate Indigenous research under governmental logics of inclusion or reconciliation, in a moment Elizabeth Povinelli has diagnosed as “liberal settler multiculturalism.”9 A key site of this effect is ethics review, wherein state law and university employees govern Indigenous research under the aegis of “protection from harm.” Ethical imperatives have informed Indigenous critiques of colonial research, as well as non-Indigenous scholars’ advocacy for ethics review as a counter to scientific violences.10

Yet if Indigenous methodologies are rooted in decolonization, then their “recognition,” which is to say their incorporation within the governance structures of a settler academy, presents a creative adaptation of colonial power that sustains colonial rule. I write this as a member of and applicant to my university’s ethics review boards and as a mentor for Indigenous students who propose research to boards on which I have sat.11 My participation has revealed to me that colonial prin- ciples set the legal standard for determining the

nature or evidence of “harm” in research with Indigenous people. For instance, if one succeeds in convinc- ing reviewers (almost always non-

Indigenous) that one’s work bears an ethical relationship to Indigenous research subjects, the work then must be introduced to subjects under the university’s authority, not that of any Indigenous body. A semblance that Indigenous people possess this authority appears when applicants are asked if their research requires “Band Council approval.” The framing of the question effectively invites applicants to devise a reason not to seek such approval, given that this would omit an additional stage of review. Yet myriad reasons may lead Indigenous researchers to question band councils as proper evaluators for their work. Hundreds of councils exist in Canada in the wake of two centuries of divide-and-conquer tactics that thieved territories by dividing peoples into “bands” as governmental targets of the Indian Act. Alfred eloquently explains that with Canadian law as its horizon, band council authority often suppresses traditional governance and obstructs its resurgence.12 To establish band councils as Indigenous authority within ethics review thus mediates knowledge about Indigenous people using two apparatuses of liberal settler multiculturalism: “self-governance,” devolved within colonial law; and “protection” as a means of entering the academy and its funder, the state. By contrast, Indigenous methodologies envision the termination of colonial rule, by fomenting the knowledges of sovereign and decolonizing peoples: that is, knowledges that subvert the ontology of a perpetually colonial society toward radical transformations of land, life, and governance. Under these conditions, the pursuit of Indigenous methodologies bears activist implications; and, as a counter to colonial research,

it directly affects “the academy.” Yet any sense that the terms activism and academia posit an intelligible distinction implodes once Indigenous methodologies demand, in the first and last instance, decolonization. Reframed in this way, we find that the academy cannot contain the aspirations of Indigenous knowledge pro- duction even while it remains a key site of critical engagement—one where non-Indigenous critics also are held responsible to allied work . Simultaneously, activism no longer presents as an “option,” as it may appear if we limit our- selves to asking whether academia is or should be engaged in activism. Such distinctions dissolve once the question on the table becomes: what did you contribute today to decolonization?

Settler colonialism is refied as the aff lacks learning about and of the otherRegan 10 , Paulette, The Director of Research for the Truth and Reconciliation Commission of Canada. She holds a PhD from the Indigenous Governance Program at the University of Victoria. “Unsettling the Settler Within”, UBC Press, Book, 2010 (HCH)

More recently, some scholars have pointed out that even those research-ers who attempt to know the Other empathically run the risk of simply perpetuating an imperial belief that their status as researchers entitles them to

acquire such knowledge. A more preferable approach, they say, is one in which non-Indigenous researchers fully embrace the uncomfortable epistemological tension that comes with the realization that they can never fully know the Other; nor should they aspire to do so. From this stance, a more nuanced reworking of non-

Indigenous positioning is evident in the literature. For example, Alison Jones (Pakeha) and Kuni Jenkins (Maori) describe their intercultural collaboration as educators "working the Indigene-Colonizer hyphen," in which they

Page 161: openev.debatecoaches.org€¦  · Web view1AC . Observation 1: If I could find the spot where truth echoesI would stand there and whisper memories of my children's future. I would

"attempt to create a research and writing relationship based on the tension of difference, not on its erasure."17 Wanda D. McCaslin (Metis) and Denise C. Breton (Euro-American) collaborate in ways that acknowledge this tension as they critique restorative justice models that they assert have failed to live up to their decolonizing poten-tial within Indigenous communities and in the field of restorative justice itself . These shortcomings reveal the need to get to the root of the problem, which is colonialism, and illustrate why "decolonization is critical for both Indigenous and non-Indigenous people." Both Jones and Breton, as non-Native scholars, emphasize the importance of continually interrogating their own colonial position within their work. Jones describes her effort to engage in collaborative dialogue while being ever mindful of the need to examine her own attitudes and actions: "To rethink collaboration be-tween indigene and colonizer is both to desire it and to ask troubling questions about it ... Interrogating the logic of (my own) White/settler enthusiasm for dialogic collaboration, I consider how this desire might be an unwitting imperialist demand — and thereby in danger of strengthening the very impulses it seeks to combat. I do not argue for a rejection of col-laboration. Rather I unpack its difficulties to suggest a less dialogical and more uneasy, unsettled relationship, based on learning (about difference) from the Other, rather than learning about the Other."19 In a similar vein, Breton notes that her very sense of identity is rooted in "all the mental, emotional, and material

habits" associated with taken-for-granted white privilege, which support ongoing oppression and are often invisible to her. Thus, "the decolonizing work begins here with nam-ing these dynamics, so that I can engage in the lifelong work of breaking their hold."20 Acknowledging that decolonization threatens their own privileged position, Jones and Breton seek to make visible to themselves and others the ease with which the colonizer unconsciously reasserts herself . Viewed from a colonizer-ally's perspective, the challenge of learning from rather than about the Other, from "an unfamiliar space of not know-ing," seems a particularly appropriate standpoint for a study that focuses on the personal and socio-political unsettling of settlers . It is also congru-ent with a broader Indigenous research agenda that supports decoloniza-tion and self-determination in ways that confront the historical and theoretical foundations of Western research paradigms and practices that privilege objectivity and neutrality over subjectivity and engagement. Like Jones and Breton, I aim to interrogate my own positionality as both colonizer-perpetrator and colonizer-ally as I work through the complex-ities of settler participation in truth-telling and reconciliation efforts. Therefore, I make settlers (myself and others) the subject of this study, linking it to my own practice as a former claims resolution manager through critical self-reflective storytelling.

There is no feasibility of reconciliation Edmonds 16, Peneolope is an Australian Research Council Future Fellow, and Associate Professor, School of Humanities, University of Tasmania. She has a PhD from the Department of History, University of Melbourne. Her research and teaching interests include colonial histories, postcolonialism, Indigenous histories, rights and activism, humanitarianism and human rights, Australian and Pacific-region contact and transnational histories, cultural heritage, performance and museums. Penny is the co-editor of Australian Historical Studies journal (2015-2018), and a member of the editorial board of Settler Colonial Studies journal. She has published in journals including Postcolonial Studies, Commonwealth and Imperial History, Journal of Colonialism and Colonial History, Australian Historical Studies, and Journal of Australian Studies. file:///Users/hannahchristianson/Downloads/Penelope-Edmonds-auth.-Settler-Colonialism-and-Reconciliation-Frontier-Violence-Affective-Performances-and-Imaginative-Refoundings%20(1).pdf “Settler Colonialism and (Re)Conciliation” HCH

Conciliation may be utopic in its promise of an imagined refounding, yet it can also be coercive and repressive. Conciliation in settler socie- ties is historically and perpetually marked by this critical ambivalence. Tasmanian Aboriginal artist and scholar Julie Gough has rejected concili- ation as a unifying, desirable or indeed possible process. ‘We are sick of being “conciliated”’, she argues, and points to Tasmania’s ‘genocidal his- tory, [where] ... Tasmanian Aboriginal people

have been subjected to the term “conciliation” for generations, but not its practice’.6 Here, Gough is referring to the ‘Great Conciliation’ of 1832, a moment of settler tri- umph which marked the end of the Black Wars in Van Diemen’s Land (now Tasmania) between settlers and Tasmanian Aboriginal peoples . She makes explicit the connection between this colonial ‘conciliation’ and the rhetoric of the contemporary Australian national reconciliation movement, thus rejecting any notion of a break between the past and the present in

Page 162: openev.debatecoaches.org€¦  · Web view1AC . Observation 1: If I could find the spot where truth echoesI would stand there and whisper memories of my children's future. I would

settler strategies of political diplomacy directed toward Indigenous peoples in Australia . Tasmania is notorious in the popular imagination as a small island that was rapidly and aggressively colonized as a British penal and pastoral settlement and beset by extreme frontier violence, which led to the near annihilation of Tasmanian Aboriginal peoples.7 The conciliatory handshake was depicted on ‘proclamation boards’, objects of diplomacy that were handed out to Aboriginal peo- ples and attached to trees on the island’s forests, at a time of extreme frontier violence and martial law (Figure 3). They reveal the central con- ceit of the conciliation of Aboriginal peoples in colonial Van

Diemen’s Land, one of many nineteenth-century savage wars of peace waged by the British in newly colonized Indigenous lands.8 It is little wonder, then, that when Tasmanian Aboriginal peoples are called upon to engage in reconciliation, the historical and emotional stakes are so high.

Decolonization DA: its not a metaphor and treating it as such reifies settler colonialismTuck & Yang 12-their author “Decolonization is not a metaphor” Eve Tuck State University of New York at New Paltz K. Wayne Yang University of California, San Diego Decolonization: Indigeneity, Education & Society Vol. 1, No. 1, 2012, pp. 1-40 HCH

Alongside this work, we have been thinking about what decolonization means, what it wants and requires. One trend we have noticed, with growing apprehension, is the ease with which the language of decolonization has been superficially adopted into education and other social sciences, supplanting prior ways of talking about social justice, critical methodologies, or approaches which decenter settler perspectives. Decolonization, which we assert is a distinct project from other civil and human rights-based social justice projects, is far too often subsumed into the directives of these projects, with no regard for how decolonization wants something different than those forms of justice . Settler scholars swap out prior civil and human rights based terms, seemingly to signal both an awareness of the significance of Indigenous and decolonizing theorizations of schooling and educational research, and to include Indigenous peoples on the list of considerations - as an additional special (ethnic) group or class. At a conference on educational research, it is not uncommon to hear speakers refer, almost casually, to the need to “decolonize our schools ,” or use

“decolonizing methods,” or “decolonize student thinking.” Yet, we have observed a startling number of these discussions make no mention of Indigenous peoples, our/their1 struggles for the recognition of our/their sovereignty, or the contributions of Indigenous intellectuals and activists to theories and frameworks of decolonization. Further, there is often little recognition given to the immediate context of settler colonialism on the North American lands where many of these conferences take place. Of course, dressing up in the language of decolonization is not as offensive as “Navajo print” underwear sold at a clothing chain

store (Gaynor, 2012) and other appropriations of Indigenous cultures and materials that occur so frequently. Yet, this kind of inclusion is a form of enclosure, dangerous in how it domesticates decolonization. It is also a foreclosure, limiting in how it recapitulates dominant theories of social change. On the occasion of the inaugural issue of Decolonization:

Indigeneity, Education, & Society, we want to be sure to clarify that decolonization is not a metaphor. When metaphor invades decolonization, it kills the very possibility of decolonization; it recenters whiteness, it resettles theory, it extends innocence to the settler, it entertains a settler future . Decolonize (a verb) and decolonization (a noun) cannot easily be grafted onto pre-existing discourses/frameworks, even if they are critical, even if they are anti-racist, even if they are justice frameworks. The easy absorption, adoption, and transposing of decolonization is yet another form of settler appropriation. When we write about decolonization, we are not offering it as a metaphor; it is not an approximation of other experiences of oppression. Decolonization is not a swappable term for other things we want to do to improve our societies and schools. Decolonization doesn’t have a synonym.

Page 163: openev.debatecoaches.org€¦  · Web view1AC . Observation 1: If I could find the spot where truth echoesI would stand there and whisper memories of my children's future. I would

Settlers can never have an accurate depiction of native oppression – the reading of the 1ac created an instantly negative discourse where settlers commodify genocide for the ballot – a vote for the aff is an affirmation that natives need settlers to act as their savior and that they cannot speak for themselves Bell 95Linda Bell, 1995, The Problem of Speaking For Others, published in Cultural Critique (Winter 1991-92), pp. 5-32; revised and reprinted in Who Can Speak? Authority and Critical Identity edited by Judith Roof and Robyn Wiegman, University of Illinois Press, 1996; and in Feminist Nightmares: Women at Odds edited by Susan Weisser and Jennifer Fleischner, (New York: New York University Press, 1994); and also in Racism and Sexism: Differences and Connections eds. David Blumenfeld and Linda Bell, Rowman and Littlefield, 1995. sfrjfl page 1 to 5

. Anne Cameron, a very gifted white Canadian author, writes several first person accounts of the lives of Native Canadian women. At the 1988 International Feminist Book Fair in Montreal, a group of Native Canadian writers ask Cameron to, in their words, "move over" on the grounds that her writings are disempowering for Native authors. She agrees.2 2. After the 1989 elections in Panama are overturned by Manuel Noriega, U.S. President George Bush declares in a public address that Noriega's actions constitute an "outrageous fraud" and that "the voice of the Panamanian people have spoken." "The Panamanian people," he tells us, "want democracy and not tyranny, and want Noriega out." He proceeds to plan the invasion of Panama. 3. At a recent symposium at my university, a prestigious theorist was invited to give a lecture on the political problems of post-modernism. Those of us in the audience, including many white women and people of oppressed nationalities and races, wait in eager anticipation for what he has to contribute to this important discussion. To our disappointment, he introduces his lecture by explaining that he can not cover the assigned topic, because as a white male he does not feel that he can speak for the feminist and post-colonial perspectives which have launched the critical interrogation of postmodernism's politics. He lectures instead on architecture. These examples demonstrate the range of current practices of speaking for others in our society. While the prerogative of speaking for others remains unquestioned in the citadels of The Problem of Speaking For Others http://alcoff.com/content/speaothers.html 2 of 26 1/30/08 11:09 AM colonial administration, among activists and in the academy it elicits a growing unease and, in some communities of discourse, it is being rejected. There is a strong, albeit contested, current within feminism which holds that speaking for others---even

for other women---is arrogant, vain, unethical, and politically illegitimate. Feminist scholarship has a liberatory agenda which almost requires that women scholars speak on behalf of other women, and yet the dangers of speaking across differences of race, culture , sexuality, and power are becoming increasingly clear to all. In feminist magazines such as Sojourner, it is common to find articles and letters in which the author states that she can only speak for herself. In her important paper, "Dyke Methods," Joyce Trebilcot offers a philosophical articulation of this view. She renounces for herself the practice of speaking for others within a lesbian feminist communitfsy, arguing that she "will not try to get other wimmin to accept my beliefs in place of their own" on the grounds that to do so would be to practice a kind of discursive coercion and even a violence.3 Feminist discourse is not the only site in which the problem of speaking for others has been acknowledged and addressed. In anthropology there is similar discussion about whether it is possible to speak for others either adequately or justifiably. Trinh T.

Minh-ha explains the grounds for skepticism when she says that anthropology is "mainly a conversation of `us' with `us' about `them,' of the white man with the white man about the primitive-nature man...in which `them' is silenced. `Them' always stands on the other side of the hill, naked and speechless...`them' is only admitted among `us', the discussing subjects, when accompanied or introduced by an `us'..."4 Given this analysis, even ethnographies written by

progressive anthropologists are a priori regressive because of the structural features of anthropological discursive practice. The recognition that there is a problem in speaking for others has followed from the widespread acceptance of two claims .

First, there has been a growing awareness that where one speaks from affects both the meaning and truth of what one says, and thus that one cannot assume an ability to transcend her location . In other words, a speaker's location (which I take here to refer to her social location or social identity) has an epistemically significant impact on that speaker's claims , and can serve either to authorize or dis- authorize one's The Problem of Speaking For Others http://alcoff.com/content/speaothers.html 3 of 26 1/30/08 11:09 AM speech. The creation of

Women's Studies and African American Studies departments were founded on this very belief: that both the study of and the advocacy for the oppressed must come to be done principally by the oppressed themselves, and that we must finally acknowledge that systematic divergences in social location between speakers and those spoken for will have a significant effect on the content of what is said. The unspoken premise here is simply that a

speaker's location is epistemically salient. I shall explore this issue further in the next section. The second claim holds that not only is location epistemically salient, but certain privileged locations are discursively dangerous.5 In particular, the practice of privileged persons speaking for or on behalf of less privileged persons has actually resulted (in

Page 164: openev.debatecoaches.org€¦  · Web view1AC . Observation 1: If I could find the spot where truth echoesI would stand there and whisper memories of my children's future. I would

many cases) in increasing or reenforcing the oppression of the group spoken for. This was part of the argument made against Anne Cameron's speaking for Native women: Cameron's intentions were never in question, but the effects of her writing were argued to be harmful to the needs of Native authors because it is Cameron rather than they who will be listened to and whose books will be bought by readers interested in Native women. Persons from dominant groups who speak for others are often treated as authenticating presences that confer legitimacy and credibility on the demands of subjugated speakers; such speaking for others does nothing to disrupt the discursive hierarchies that operate in public spaces. For this reason, the work of privileged authors who speak on behalf of the oppressed is becoming increasingly criticized by members of those oppressed groups themselves .6 As social theorists, we are authorized by virtue of our academic positions to develop theories that express and encompass the ideas, needs, and goals of others. However, we must begin to ask ourselves whether this is ever a legitimate authority, and if so, what are the criteria for legitimacy? In particular, is it ever valid to speak for others who are unlike me or who are less privileged than me? We might try to delimit this problem as only arising when a more privileged person speaks for a less privileged one. In

this case, we might say that I should only speak for groups of which I am a member. But this does not tell us how groups themselves should be delimited. For example, can a white woman speak for all women simply by virtue of being a woman? If not, how narrowly should we draw the The Problem of Speaking For Others http://alcoff.com/content/speaothers.html 4 of 26 1/30/08 11:09 AM categories? The complexity and multiplicity of group identifications could result in "communities" composed of single individuals. Moreover, the concept of groups assumes specious notions about clear-cut boundaries and "pure" identities. I am a Panamanian-American and a person of mixed ethnicity and race: half white/Angla and half Panamanian mestiza. The criterion of group identity leaves many unanswered questions for a person such as myself, since I have membership in many conflicting groups but my membership in all of them is problematic. Group identities and boundaries are ambiguous and permeable, and decisions about demarcating identity are always partly arbitrary. Another problem concerns how specific an identity needs to be to confer epistemic authority. Reflection on such problems quickly reveals that no easy solution to the problem of speaking for others can be found by simply restricting the practice to speaking for groups of which one is a member. Adopting the position that one should only speak for oneself raises similarly difficult questions. If I don't speak for those less privileged than myself, am I abandoning my political responsibility to speak out

against oppression, a responsibility incurred by the very fact of my privilege? If I should not speak for others, should I restrict myself to following

their lead uncritically? Is my greatest contribution to move over and get out of the way? And if so, what is the best way to do this---to keep silent or to deconstruct my own discourse? The answers to these questions will certainly depend on who is asking them. While some of us may want to undermine, for example, the U.S. government's practice of speaking for the "Third world," we may not want to undermine someone such as Rigoberta Menchu's ability to speak for Guatemalan Indians.7 So the question arises about whether all instances of speaking for should be condemned and, if not, how we can justify a position which would repudiate some speakers while accepting others. In order to answer these questions we need to become clearer on the epistemological and metaphysical issues which are involved in the articulation of the problem of speaking for others, issues which most often remain implicit. I will attempt to make these issues clear before turning to discuss some of the possible responses to the problem and advancing a provisional, procedural solution of my own. But first I need to explain further my framing of the problem. The Problem of Speaking For Others http://alcoff.com/content/speaothers.html 5 of 26 1/30/08 11:09 AM In the examples used above, there may appear to be a conflation between the issue of speaking for others and the issue of speaking about others. This conflation was intentional on my part, because it is difficult to distinguish speaking about from speaking for in all cases. There is an ambiguity in the two phrases: when one is speaking for another one may be describing their situation and thus also speaking about them. In fact, it may be impossible to speak for another without simultaneously conferring information about them. Similarly, when one is speaking about another, or simply trying to describe their situation or some aspect of it, one may also be speaking in place of them, i.e. speaking for them. One may be speaking about another as an advocate or a messenger if the person

cannot speak for herself. Thus I would maintain that if the practice of speaking for others is problematic, so too must be the practice of speaking about others.8 This is partly the case because of what has been called the "crisis of representation." For in both the practice of speaking for as well as the practice of speaking about others, I am engaging in the act of representing the other's needs, goals, situation, and in fact, who they are, based on my own situated interpretation. In post-structuralist terms, I am participating in the construction of their subject-positions rather than simply discovering their true selves. Once we pose it as a problem of representation, we see that, not only are speaking for and speaking about analytically close, so too are the practices of speaking for

others and speaking for myself. For, in speaking for myself, I am also representing my self in a certain way, as occupying a specific subject-position, having certain characteristics and not others, and so on . In speaking for myself, I (momentarily) create my self---just as much as when I speak for others I create them as a public, discursive self, a self which is more unified than any subjective experience can support. And this public self will in most cases have an effect on the self experienced as interiority. The point here is that the problem of representation underlies all cases of speaking for, whether I am speaking for myself or for others. This is not to suggest that all representations are fictions: they have very real material effects, as well as material origins, but they are always mediated in complex ways by discourse, power, and location. However, the problem of speaking for others is more specific than the problem of representation generally, and requires its own The Problem of Speaking For Others http://alcoff.com/content/speaothers.html 6 of 26 1/30/08 11:09 AM particular analysis.

Reading settler colonialism as a strategic argument within debate creates disingenuous engagement and turns all aff offenseMedak- Saltzman 15 (Danika Medak-Saltzman (Turtle Mountain Chippewa) is thrilled to be joining the Women’s and Gender Studies Department, and her work focuses on Indigenous Feminisms, Native histories, Indigenous thought and theory, transnational Indigeneity, Indigenous futurisms, and visual culture—including film and

Page 165: openev.debatecoaches.org€¦  · Web view1AC . Observation 1: If I could find the spot where truth echoesI would stand there and whisper memories of my children's future. I would

cultural production. She also examines the transnational movement of American colonial policies–particularly in the case of Japan—which is a subject explored in her forthcoming book, Specters of Colonialism: Native Peoples, Visual Cultures, and Colonial Projects in the U.S. and Japan, with the University of Minnesota Press. Her articles have appeared in American Quarterly and The Journal of Critical Ethnic Studies, and Studies in American Indian Literature. In her scholarship, as in her teaching, Medak-Saltzman seeks to reevaluate representations of Native people to underscore how Native peoples have always worked to negotiate difficult situations and visualize/create/manifest Indigenous futures in spite of persistent colonial actions and narratives that mandate Native disappearance. She is a member and co-founder of the “Just Futures Project,” and alongside Iyko Day and Antonio T. Tiongson, Jr. she is co-editor of the “Critical Race, Indigeneity and Relationality” book series for Temple University Press, Empire’s Haunted Logics Comparative Colonialisms and the Challenges of Incorporating Indigeneity, the Journal of Critical Ethnic Studies. Minneapolis: University of Minnesota Press; 1(2) 2015: 11-32, acp)

In no uncertain terms, Indigenous populations remain entrenched in fundamentally different situations than those faced by other racialized groups. Although the truth of it is inconvenient and unpalatable, the reality that settler colonialism—which Patrick Wolfe tells us is a process rather than an event1 —is ongoing makes it all the more conspicuous that the function and effects of settler colonialism remain largely unseen by those who benefit from them. Yet, as incisive as Jodi A. Byrd’s observation is, we must also recognize that the resistance to and failure to see Indigeneity, or the critical contexts central to understanding Indigenous peoples’ historical and contemporary experiences, is decidedly not limited to thinking from “new world geographies” nor to those who insist that our world is a “postcolonial pluralistic multiculture.”2 For these reasons, one might expect that the recent and increasingly common invocation of the term Indigeneity in crossdisciplinary and interdisciplinary scholarly production would be a movement immediately welcomed by Indigenous studies scholars. However, inasmuch as such references to “Indigeneity” may be intended to help finally account for and draw attention to the way academics contribute to the pervasive vanishing and diminishment of Native

presence, significance, and realities, these mentions remain largely, and troublingly, cursory. The invocation of Indigeneity across disciplines cannot yet be counted on as an indication of an author’s fluency with the legal, epistemological, and political particularities of Native peoples’ experiences with settler colonial realities—which one might reasonably otherwise expect from scholarly engagement with intellectual subjects. Instead, this trend seems to be the most recent example of a familiar , and superficial, pattern of (non)engagement with Native peoples, histories, and Indigenous studies scholarship in settler colonial societies writ large. “Indigeneity” is all too often invoked as a term— rather than a concept—which reduces it to jargon, removes it from its vital context, and embeds it in writing that otherwise betrays a very limited intellectual and scholarly understanding of Native experiences, issues, and histories. This is to say (to borrow from Emma Perez’s Decolonial

Imaginary3 ) that Indigeneity is often invoked in a manner that renders it as merely an “appendage” to the real stories, which is markedly different from engaging it as a subject with its own goals. One can make arguments about why this has been the case: perceived paucity of sources (with its emphasis on a very specific and elite colonial print archive), lamentations and assumptions that “we will never know” (which justify not looking for what is there), and the difficulty of combatting the already entrenched Indigenous absence in existing scholarship, among other oft-cited reasons. But such excuses normalize how power is exercised in the production of knowledge and how colonial ideologies have become enmeshed in certain (haunted) ways of thinking about the world. This tendency cannot be blamed on the perceived “newness” of such interventions, for the practice and discipline of Indigenous studies has been building globally since the 1960s. To be clear, the argument here is

not that everyone can or should become Indigenous studies experts.4 However, we would do well to recognize that when invocations of “Indigeneity” do not anticipate a complicated engagement with Native studies scholarship—even though the field represents half a century of scholarly engagement, theoretical interventions, and actual practice of Indigenous studies— it becomes difficult to see such usage as anything more than self- serving. As long as the superficial invocation of “Indigeneity” is intended to showcase that a scholar has kept up with scholarly trends in the field, rather than an attempt to critically engage with or further Indigenous studies scholarship in a meaningful way , then this tendency ought to be viewed as what it is: a twentyfirst-century intellectual example of the time-honored Western tradition of pilfering (lands,

resources, ideas, knowledge, theoretical frames, artistic traditions, etc.) from Native peoples for the benefit of the self/nation, with utter disregard to any resulting consequences borne by Indigenous peoples .

Page 166: openev.debatecoaches.org€¦  · Web view1AC . Observation 1: If I could find the spot where truth echoesI would stand there and whisper memories of my children's future. I would

Settlers can never have an accurate depiction of native oppression – the reading of the 1ac created an instantly negative discourse where settlers commodify genocide for the ballot, it is net better to exclude the settler. Bell 95Linda Bell, 1995, The Problem of Speaking For Others, published in Cultural Critique (Winter 1991-92), pp. 5-32; revised and reprinted in Who Can Speak? Authority and Critical Identity edited by Judith Roof and Robyn Wiegman, University of Illinois Press, 1996; and in Feminist Nightmares: Women at Odds edited by Susan Weisser and Jennifer Fleischner, (New York: New York University Press, 1994); and also in Racism and Sexism: Differences and Connections eds. David Blumenfeld and Linda Bell, Rowman and Littlefield, 1995. //Jazmyn page 1 to 5

. Anne Cameron, a very gifted white Canadian author, writes several first person accounts of the lives of Native Canadian women. At the 1988 International Feminist Book Fair in Montreal, a group of Native Canadian writers ask Cameron to, in their words, "move over" on the grounds that her writings are disempowering for Native authors. She agrees.2 2. After the 1989 elections in Panama are overturned by Manuel Noriega, U.S. President George Bush declares in a public address that Noriega's actions constitute an "outrageous fraud" and that "the voice of the Panamanian people have spoken." "The Panamanian people," he tells us, "want democracy and not tyranny, and want Noriega out." He proceeds to plan the invasion of Panama. 3. At a recent symposium at my university, a prestigious theorist was invited to give a lecture on the political problems of post-modernism. Those of us in the audience, including many white women and people of oppressed nationalities and races, wait in eager anticipation for what he has to contribute to this important discussion. To our disappointment, he introduces his lecture by explaining that he can not cover the assigned topic, because as a white male he does not feel that he can speak for the feminist and post-colonial perspectives which have launched the critical interrogation of postmodernism's politics. He lectures instead on architecture. These examples demonstrate the range of current practices of speaking for others in our society. While the prerogative of speaking for others remains unquestioned in the citadels of The Problem of Speaking For Others http://alcoff.com/content/speaothers.html 2 of 26 1/30/08 11:09 AM colonial administration, among activists and in the academy it elicits a growing unease and, in some communities of discourse, it is being rejected. There is a strong, albeit contested, current within feminism which holds that speaking for others---even

for other women---is arrogant, vain, unethical, and politically illegitimate. Feminist scholarship has a liberatory agenda which almost requires that women scholars speak on behalf of other women, and yet the dangers of speaking across differences of race, culture , sexuality, and power are becoming increasingly clear to all. In feminist magazines such as Sojourner, it is common to find articles and letters in which the author states that she can only speak for herself. In her important paper, "Dyke Methods," Joyce Trebilcot offers a philosophical articulation of this view. She renounces for herself the practice of speaking for others within a lesbian feminist communitfsy, arguing that she "will not try to get other wimmin to accept my beliefs in place of their own" on the grounds that to do so would be to practice a kind of discursive coercion and even a violence.3 Feminist discourse is not the only site in which the problem of speaking for others has been acknowledged and addressed. In anthropology there is similar discussion about whether it is possible to speak for others either adequately or justifiably. Trinh T.

Minh-ha explains the grounds for skepticism when she says that anthropology is "mainly a conversation of `us' with `us' about `them,' of the white man with the white man about the primitive-nature man...in which `them' is silenced. `Them' always stands on the other side of the hill, naked and speechless...`them' is only admitted among `us', the discussing subjects, when accompanied or introduced by an `us'..."4 Given this analysis, even ethnographies written by

progressive anthropologists are a priori regressive because of the structural features of anthropological discursive practice. The recognition that there is a problem in speaking for others has followed from the widespread acceptance of two claims .

First, there has been a growing awareness that where one speaks from affects both the meaning and truth of what one says, and thus that one cannot assume an ability to transcend her location . In other words, a speaker's location (which I take here to refer to her social location or social identity) has an

Page 167: openev.debatecoaches.org€¦  · Web view1AC . Observation 1: If I could find the spot where truth echoesI would stand there and whisper memories of my children's future. I would

epistemically significant impact on that speaker's claims , and can serve either to authorize or dis- authorize one's The Problem of Speaking For Others http://alcoff.com/content/speaothers.html 3 of 26 1/30/08 11:09 AM speech. The creation of

Women's Studies and African American Studies departments were founded on this very belief: that both the study of and the advocacy for the oppressed must come to be done principally by the oppressed themselves, and that we must finally acknowledge that systematic divergences in social location between speakers and those spoken for will have a significant effect on the content of what is said. The unspoken premise here is simply that a

speaker's location is epistemically salient. I shall explore this issue further in the next section. The second claim holds that not only is location epistemically salient, but certain privileged locations are discursively dangerous.5 In particular, the practice of privileged persons speaking for or on behalf of less privileged persons has actually resulted (in many cases) in increasing or reenforcing the oppression of the group spoken for. This was part of the argument made against Anne Cameron's speaking for Native women: Cameron's intentions were never in question, but the effects of her writing were argued to be harmful to the needs of Native authors because it is Cameron rather than they who will be listened to and whose books will be bought by readers interested in Native women. Persons from dominant groups who speak for others are often treated as authenticating presences that confer legitimacy and credibility on the demands of subjugated speakers; such speaking for others does nothing to disrupt the discursive hierarchies that operate in public spaces. For this reason, the work of privileged authors who speak on behalf of the oppressed is becoming increasingly criticized by members of those oppressed groups themselves .6 As social theorists, we are authorized by virtue of our academic positions to develop theories that express and encompass the ideas, needs, and goals of others. However, we must begin to ask ourselves whether this is ever a legitimate authority, and if so, what are the criteria for legitimacy? In particular, is it ever valid to speak for others who are unlike me or who are less privileged than me? We might try to delimit this problem as only arising when a more privileged person speaks for a less privileged one. In

this case, we might say that I should only speak for groups of which I am a member. But this does not tell us how groups themselves should be delimited. For example, can a white woman speak for all women simply by virtue of being a woman? If not, how narrowly should we draw the The Problem of Speaking For Others http://alcoff.com/content/speaothers.html 4 of 26 1/30/08 11:09 AM categories? The complexity and multiplicity of group identifications could result in "communities" composed of single individuals. Moreover, the concept of groups assumes specious notions about clear-cut boundaries and "pure" identities. I am a Panamanian-American and a person of mixed ethnicity and race: half white/Angla and half Panamanian mestiza. The criterion of group identity leaves many unanswered questions for a person such as myself, since I have membership in many conflicting groups but my membership in all of them is problematic. Group identities and boundaries are ambiguous and permeable, and decisions about demarcating identity are always partly arbitrary. Another problem concerns how specific an identity needs to be to confer epistemic authority. Reflection on such problems quickly reveals that no easy solution to the problem of speaking for others can be found by simply restricting the practice to speaking for groups of which one is a member. Adopting the position that one should only speak for oneself raises similarly difficult questions. If I don't speak for those less privileged than myself, am I abandoning my political responsibility to speak out

against oppression, a responsibility incurred by the very fact of my privilege? If I should not speak for others, should I restrict myself to following

their lead uncritically? Is my greatest contribution to move over and get out of the way? And if so, what is the best way to do this---to keep silent or to deconstruct my own discourse? The answers to these questions will certainly depend on who is asking them. While some of us may want to undermine, for example, the U.S. government's practice of speaking for the "Third world," we may not want to undermine someone such as Rigoberta Menchu's ability to speak for Guatemalan Indians.7 So the question arises about whether all instances of speaking for should be condemned and, if not, how we can justify a position which would repudiate some speakers while accepting others. In order to answer these questions we need to become clearer on the epistemological and metaphysical issues which are involved in the articulation of the problem of speaking for others, issues which most often remain implicit. I will attempt to make these issues clear before turning to discuss some of the possible responses to the problem and advancing a provisional, procedural solution of my own. But first I need to explain further my framing of the problem. The Problem of Speaking For Others http://alcoff.com/content/speaothers.html 5 of 26 1/30/08 11:09 AM In the examples used above, there may appear to be a conflation between the issue of speaking for others and the issue of speaking about others. This conflation was intentional on my part, because it is difficult to distinguish speaking about from speaking for in all cases. There is an ambiguity in the two phrases: when one is speaking for another one may be describing their situation and thus also speaking about them. In fact, it may be impossible to speak for another without simultaneously conferring information about them. Similarly, when one is speaking about another, or simply trying to describe their situation or some aspect of it, one may also be speaking in place of them, i.e. speaking for them. One may be speaking about another as an advocate or a messenger if the person

cannot speak for herself. Thus I would maintain that if the practice of speaking for others is problematic, so too must be the practice of speaking about others.8 This is partly the case because of what has been called the "crisis of representation." For in both the practice of speaking for as well as the practice of speaking about others, I am engaging in the act of representing the other's needs, goals, situation, and in fact, who they are, based on my own situated interpretation. In post-structuralist terms, I am participating in the construction of their subject-positions rather than simply discovering their true selves. Once we pose it as a problem of representation, we see that, not only are speaking for and speaking about analytically close, so too are the practices of speaking for

others and speaking for myself. For, in speaking for myself, I am also representing my self in a certain way, as occupying a specific subject-position, having certain characteristics and not others, and so on . In speaking for myself, I (momentarily) create my self---just as much as when I speak for others I create them as a public, discursive self, a self which is more unified than any subjective experience can support. And this public self will in most cases have an effect on the self experienced as interiority. The point here is that the problem of representation underlies all cases of speaking for, whether I am speaking for myself or for others. This is not to suggest that all representations are fictions: they have very real material effects, as well as material origins, but they are always mediated in complex ways by discourse, power, and location. However, the problem

Page 168: openev.debatecoaches.org€¦  · Web view1AC . Observation 1: If I could find the spot where truth echoesI would stand there and whisper memories of my children's future. I would

of speaking for others is more specific than the problem of representation generally, and requires its own The Problem of Speaking For Others http://alcoff.com/content/speaothers.html 6 of 26 1/30/08 11:09 AM particular analysis.

Page 169: openev.debatecoaches.org€¦  · Web view1AC . Observation 1: If I could find the spot where truth echoesI would stand there and whisper memories of my children's future. I would

Stick Up Counter Advocacy:

The Counter Advocacy is to steal from the university, this creates the conceptualization of the stick up artist which is the only way to deconstruct a world engendered in whiteness. It specifically specifies in their evidence that they have to collapse the university in order to solve for their impacts “the university , as such, can never be unsnarled from its colonial origin and operation, and must be destroyed in order to build educational spaces that support thriving indigenous communities .” Mayorga, Leidecker, Gutierrez 19, and the only way to destroy the university without being caught is to become the stick up artist. Moten and Harney ’13 (Fred Moten, Moten is professor of performance studies at New York University, Stefano Harney, Harney is Professor of Strategic Management Education at Singapore Management University, The Undercommons: Fugitive Planning and Black Study, 2013) \\EG

“To the university I’ll steal, and there I’ll steal,” to borrow from Pistol at the end of Henry V, as he would surely borrow from

us. Tis is the only possible relationship to the American university today. Tis may be true of universities everywhere . It may have to be true of the university in general . But certainly, this much is true in the United States: it cannot be denied that the university is a place of refuge, and it cannot be accepted that the university is a place of enlightenment.

In the face of these conditions one can only sneak into the university and steal what one can. To abuse its hospitality , to spite its mission, to join its refugee colony , its gypsy encampment, to be in but not of – this is the path of the subversive intellectual in the modern university. Worry about the university. Tis is the injunction today in the United States , one with a long history. Call for its restoration like Harold Bloom or Stanley Fish or Gerald

Graf. Call for its reform like Derek Bok or Bill Readings or Cary Nelson. Call out to it as it calls to you. But for the subversive intellectual, all of this goes on upstairs, in polite company, among the rational men. After all, the subversive intellectual came under false pretenses , with bad documents , out of love . Her labor is as necessary

as it is unwelcome. Te university needs what she bears but cannot bear what she brings. And on top of all that, she disappears. She disappears into the underground , the downlow lowdown maroon community of the university,

into the undercommons of enlightenment, where the work gets done, where the work gets subverted , where the revolution is still black, still strong. What is that work and what is its social capacity for both reproducing the university and producing fugitivity? If one were to say teaching , one would be performing the work of the university. Teaching is merely a profession and an operation of that onto-/auto-encyclopedic circle of the state” that Jacques

Derrida calls the Universitas. But it is useful to invoke this operation to glimpse the hole in the fence where labor enters, to glimpse its hiring hall, its night quarters . Te university needs teaching labor , despite itself, or as itself, self-identical with and thereby erased by it . It is not teaching that holds this social capacity, but something that

produces the not visible other side of teaching, a thinking through the skin of teaching toward a collective orientation to the knowledge object as future project, and a commitment to what we want to call the prophetic organization . But it is teaching that brings us in. Before there are grants, research, conferences, books, and journals there is the experience of being taught and of teaching. Before the research post with no teaching, before the graduate students to mark the exams, before the string of sabbaticals, before the permanent reduction in teaching load, the appointment to run the Center, the consignment of pedagogy to a discipline called education, before the course designed to be a new book, teaching happened. Te moment of

teaching for food is therefore often mistakenly taken to be a stage, as if eventually one should not teach for food. If the stage persists,

Page 170: openev.debatecoaches.org€¦  · Web view1AC . Observation 1: If I could find the spot where truth echoesI would stand there and whisper memories of my children's future. I would

there is a social pathology in the university. But if the teaching is successfully passed on, the stage is surpassed , and teaching is consigned to those who are known to remain in the stage, the sociopathological labor of the university. Kant interestingly calls such a stage “self-incurred minority.” He tries to contrast it with having the “determination and courage to use one’s intelligence without being guided by another.” “Have the courage to use your own

intelligence.” But what would it mean if teaching or rather what we might call “the beyond of teaching ” is precisely what one is asked to get beyond, to stop taking sustenance? And what of those minorities who refuse, the tribe of moles who will not come back from beyond (that which is beyond “the beyond of teaching”), as if they will not be subjects, as if they want to think as objects, as minority? Certainly, the perfect subjects of communication, those successfully beyond teaching,

will see them as waste. But their collective labor will always call into question who truly is taking the orders of the enlightenment. Te waste lives for those moments beyond teaching when you give away the unexpected beautiful phrase – unexpected, no one has asked, beautiful, it will never come back. Is being the biopower of the enlightenment truly better than this? Perhaps

the biopower of the enlightenment knows this, or perhaps it is just reacting to the objecthood of this labor as it must. But even as it depends on these moles, these refugees, it will call them uncollegial , impractical, naive, unprofessional. And one may be given one last chance to be pragmatic – why steal when one can have it all, they will ask. But if one hides from this interpellation , neither agrees nor disagrees but goes with hands full into the underground of the university , into the Undercommons – this will be regarded as theft, as a criminal act. And it is at the same time, the only possible act. In that undercommons of the university one can see that it is not a matter of teaching versus

research or even the beyond of teaching versus the individualisation of research. To enter this space is to inhabit the ruptural and enraptured disclosure of the commons that fugitive enlightenment enacts, the criminal , matricidal, queer, in the cistern, on the stroll of the stolen life, the life stolen by enlightenment and stolen back , where the commons give refuge, where the refuge gives commons. What the beyond of teaching is really about is not

fnishing oneself, not passing, not completing; it’s about allowing subjectivity to be unlawfully overcome by others, a radical passion and passivity such that one becomes unfit for subjection , because one does not possess the kind of agency that can hold the regulatory forces of subjecthood , and one cannot initiate the auto-interpellative

torque that biopower subjection requires and rewards. It is not so much the teaching as it is the prophecy in the organization of the act of teaching . Te prophecy that predicts its own organization and has therefore passed, as commons, and the

prophecy that exceeds its own organization and therefore as yet can only be organized. Against the prophetic organization of the undercommons is arrayed its own deadening labor for the university, and beyond that, the negligence of professionalization, and the professionalization of the critical academic . Te undercommons is therefore always an unsafe neighborhood . As Fredric Jameson reminds us, the university depends upon “Enlightenment-type critiques and demystifcation of belief and committed ideology, in order to clear the ground for unobstructed planning and ‘development.’” Tis is the weakness of the university, the lapse in its homeland security. It needs labor power for this

“enlightenmenttype critique,” but, somehow, labor always escapes. Te premature subjects of the undercommons took the call seriously, or had to be serious about the call. Tey were not clear about planning, too mystical , too full of belief. And yet this labor force cannot reproduce itself , it must be reproduced. Te university works for the day when it will be able to rid itself, like capital in general, of the trouble of labor. It will then be able to reproduce a labor force that understands itself as not only unnecessary but dangerous to the development of capitalism. Much pedagogy and scholarship is already dedicated in this direction. Students must come to see themselves as the problem, which, counter to the complaints of restorationist critics of the university, is precisely what it means to be a customer, to take on the burden of realisation and always necessarily be inadequate to it. Later, these students will be able to see themselves properly as obstacles to society, or perhaps, with lifelong learning, students will return having successfully diagnosed themselves as the problem. Still, the dream of an undiferentiated labor that knows itself as superfuous is interrupted precisely by the labor of clearing away the burning roadblocks of ideology. While it is better that this police function be in the hands of the few, it still raises labor as diference, labor as the development of other labor, and therefore labor as a source of wealth. And although the enlightenment-type critique, as we suggest below, informs on, kisses the cheek of, any autonomous development as a result of this diference in

labor, there is a break in the wall here, a shallow place in the river, a place to land under the rocks. Te university still needs this clandestine labor to prepare this undiferentiated labor force , whose increasing specialisation and managerialist tendencies, again contra the restorationists, represent precisely the successful integration of the division of labor with the universe of exchange

that commands restorationist loyalty. Introducing this labor upon labor, and providing the space for its development, creates risks . Like the colonial police force recruited unwittingly from guerrilla

Page 171: openev.debatecoaches.org€¦  · Web view1AC . Observation 1: If I could find the spot where truth echoesI would stand there and whisper memories of my children's future. I would

neighborhoods, university labor may harbor refugees, fugitives, renegades, and castaways . But there are

good reasons for the university to be confident that such elements will be exposed or forced underground. Precautions have been taken , book lists have been drawn up, teaching observations conducted , invitations to contribute made. Yet against these precautions stands the immanence of transcendence, the necessary deregulation and the possibilities of criminality and fugitivity that labor upon labor requires. Maroon communities of composition teachers, mentorless graduate students, adjunct Marxist historians, out or queer management professors, state college ethnic studies

departments, closed-down flm programs, visaexpired Yemeni student newspaper editors, historically black college sociologists, and feminist engineers. And what will the university say of them? It will say they are unprofessional . Tis is not an arbitrary charge. It is the charge against the more than professional . How do those who exceed the profession, who exceed and by exceeding escape , how do those maroons problematize themselves, problematize the university, force the university to consider them a problem, a danger? Te undercommons is

not, in short, the kind of fanciful communities of whimsy invoked by Bill Readings at the end of his book. Te undercommons, its maroons, are always at war, always in hiding.

The only way out of a world structured through antiblackness, is an end to the White World itself. If we are engaging in this war then resistance needs an “unspoken dynamic”, a form of guerrilla linguistics, an undercommon communication. A stick-up artist is any decentralized vigilante willing to rupture the system of semiotics through a project of total disorder. To transfigure the symbolic stakes of this academic space, Black terrorism steals in an act of fugitivity and gratuitous freedom. In the age of Trump, the perfection of slavery reaches its horizon. The only choice we will have is to fight. Not because we want to, but because we have to.Gillespie ’17 (John Gillespie, PhD Student at UC Irvine, “On the Prospect of Weaponized Death”, 2017, Propter Nos, https://uci.academia.edu/JohnGillespie) \\EG

Afro-Pessimist thinkers, in favor of a diagnostic analysis, tend to veer away from the tradition of critical social theory that prescribes solutions

to the analysis in the conclusion of their work. However, one finds throughout Afro-Pessimist literature a battle cry, a prophetic vision , a pulsing pessimist hope for the “end of the World.” For if Whiteness ended Worlds through its

colonial simulations and violent transmutations of Africans into Blacks, then the only way out is an end to the White World . White Being is irredeemable, and so is the World it fosters . Sexton says, “In a world structured by the twin axioms of

white superiority and black inferiority, of white existence and black non-existence, a world structured by a negative categorical imperative —‘above all, don’t be black’ —in this world, the zero degree of transformation is the turn toward blackness, a turn toward the shame, as it were, that ‘resides in the idea that 'I am thought of as less than human.’”16 It’s only through black vigilance that the simulacra of White Being is made clear and the spectacle of gratuitous freedom is made visible. It is somewhere in this structural antagonism, that on the one hand conditions the possibility of the World, and on the other hand conditions the possibility of its end, its limitations, its disorientation, that we found the language to say the unsayable and do the

undoable. As Frank Wilderson reminds us: Black Studies in general and Afro-Pessimism in particular present non-Black academics with more than an intellectual problem. It presents them with an existential problem. The reason is because there’s an aspect of Afro-Pessimism that we don’t talk about…which is that were you to follow it to its logical conclusion, it’s calling for the end of the world…it wants the death of everyone else in the same way that we experience our death, so that one could not liberate Blacks through AfroPessimism and be who one was

on the other side of that. That’s the unspoken dynamic of Afro-Pessimism.17 If we are engaging in a war in which the symbolic value, the semiotics of this World itself, positions “the Black as death personified, the White as personification of diversity, of

Page 172: openev.debatecoaches.org€¦  · Web view1AC . Observation 1: If I could find the spot where truth echoesI would stand there and whisper memories of my children's future. I would

life itself,”18 then resistance needs an “unspoken dynamic.” It needs a space where “words don’t go”— a form of guerrilla linguistics , a submarined syntax, an undercommon communication . Perhaps, here, where the

conversation is blackened, and the theory is phobogenic, and the journal is Propter Nos, we can allow ourselves to excavate insurgent dictions still lost in the lingua franca of White Being, but full of the specter of black terror, black disorientation. If the Black is death personified, then what might happen if we weaponized our death? What might happen if we recognized the inevitability of that death? What if we began to think that the non-uniqueness of that death was an opening towards the “ end of Humanity ?” In The Spirit of Terrorism, Jean Baudrillard writes, “When global power monopolizes the situation to this extent, when there is such a formidable condensation of all functions in the technocratic machinery,

and when no alternative form of thinking is allowed, what other way is there but a terroristic situational transfer? ” 19 Terrorism consists of the militaristic tactics used by those who are facing globalized White Being with asymmetrical technologies of terror, violence, intimidation and war. A terrorist is any armed vigilante willing to rupture the system of semiotics through an equally cofounding semiotic. A semiotic that returns one to the “desert of the [Black] Real”— where a “project of total disorder ” is unleashed upon the semiotic system.20 Black terrorism is a violence that re-appropriates the death embedded in the Black’s ontological incapacity in order to enable the possibility of a radical capacity —gratuitous freedom. White Being itself is a decentralized ontoepistemic deployment

of violence, and if violent insurgency is necessary, then the decentralized approach of the black terrorist is necessary to counter the terror of White Being. This being said, black terrorism is perhaps better understood as counter- terror terrorism . We do not have the power to end the World with life. We only have the power to end the World through death. As Baudrillard writes, “The radical difference is that the terrorist, while they have at their disposal weapons that are the system’s own, possess a further lethal weapon: their own deaths.”21 The United States has an international military force, a storehouse of nuclear arms, and the capacity, within their police state alone, to “terrorize” not just one block in Baltimore, but the whole entire world. Black terrorism is what

happens when we heed the Afro-Pessimist call that “A living death is as much a death as it is a living,”22 it is what happens when we take

seriously the unsayable in Afro-Pessimism. Black Terrorism is (non)ontological fugitivity that disavows any need to focus on social life—black terrorism steals black death itself from White Being . It is for this reason that Baudrillard speaks to his own White Being and the specter of terror when he says: When Western culture sees all of its values extinguished one by one, it turns inward on itself in the very

worst way. Our death is an extinction, an annihilation. Herein lies our poverty. When a singularity throws its own death into the ring, it escapes this slow extermination , its dies its own natural death. This is an immense game of double or

quits. In committing suicide, the singularity suicides the other at the same time— we might say that the terrorist acts

literally ‘suicided’ the West. A death for a death , then, but transfigured by the symbolic stakes. ‘We have already

devastated our world, what more do you want?’ says Muray. But precisely, we have devastated this world, it still has to be destroyed. Destroyed symbolically . This is not at all the same undertaking. And though we did the first part, only others are going

to be able to do the second.23 We are the others. Tasked with the (un)fortunate task of ending White hyper-realism, the White World, and White Being. Well aware that if White Fascism continues the project of black annihilation, the only choice we will have is to fight . Not because we want to, but because we have to. But, ultimately, we must

remember the words of Huey Newton: “[T]he first lesson a revolutionary must learn is that he is a doomed man.”24 In the age of Trump, the perfection of slavery reaches its horizon. 25 The disavowal of the lives of refugees is White Being attempting to reconcile the “NationState” simulation with the free track and flow of bodies it’s been attempting to murder; the deportation of undocumented

immigrants in conjunction with the materialization of borders is White Being attempting to secure its linguistic and economic integrity; the rise of the private prison and the militarization of the police force is White Being attempting to innovate the system of enslavement and necropolitics for the 21st Century; the plundering of indigenous land and bodies is White Being attempting to finish off the project of genocide; the disregard for the Earth is White Being ensuring the Anthropocene will also be the

Apocalypse. Trump is a reinvigoration, a call to arms, for White Being, and White Being can only be “destroyed symbolically.” Black terrorism transfigures the symbolic stakes because it steals away that condition of White Being’s possibility in a kind of fugitivity that is a zero-transformation into Blackness. This being said, we all know that the only thing that follows the absolute loss of hope is this Black Spring, this Neo-Fanonian violence, this blackened terroristic situational transfer. In

Baudrillard’s words, in the Age of Trump, let us remember the gift of immorality, “Terrorism is immoral . The World Trade

Page 173: openev.debatecoaches.org€¦  · Web view1AC . Observation 1: If I could find the spot where truth echoesI would stand there and whisper memories of my children's future. I would

Center event, that symbolic challenge, is immoral, and it is a response to a globalization which is itself immoral. So, let us be immoral …”26

Page 174: openev.debatecoaches.org€¦  · Web view1AC . Observation 1: If I could find the spot where truth echoesI would stand there and whisper memories of my children's future. I would

IDK WHAT THE FUCK THESE ARE FOR ASK SAM😊

Link: The alt does not challenge materially challenge institutions – Occupy proves state action is needed to avoid individualism Cutler 11/15/16 (Sophia, writer and dancer residing in Montreal, Occupying Trump, https://www.jacobinmag.com/2016/11/donald-trump-occupy-wall-street-zuccotti-park/)

Others correctly noted that Occupy wasn’t a traditional political movement that made demands upon the state. It instead turned inward, believing the consensual and nonhierarchical community it was building in Zuccotti constituted a demand in itself . Sympathizers touted the camp as the instantiation of a new social awareness, a new way of relating to one another. “[W]e are literally laying the framework for a new world by building it here and

now — and it works,” the New York City encampment proclaimed. This consensual social experience centered on the autonomy of the individual. Occupiers were not held accountable to any particular agenda or leadership. As one participant put it at a general assembly meeting, “None of us are leaders; [therefore] we are all leaders.” Today, as high

school students walk out en masse and anti-Trump protests sweep the country , the memory of Occupy should give us pause. Instead of creating a political movement that materially attacked the institutions of the 1 percent, many occupiers vowed to transform themselves and raise awareness at the individual level . Some responses to Trumpism have fallen into the same trap — treating the election of a far-right demagogue as an opportunity for soul-searching or a reason to rail against individual Trump voters . But just as tending urban gardens didn’t fundamentally challenge the prerogatives of capital, safety pins won’t bring down the reactionary figures taking the reins of government . To stop Trump, we’ll have to reject some of Occupy’s central precepts . Occupy Individualism Occupy’s emphasis on the individual sprang from its horizontalist antecedents — particularly the May 1968 occupations in France. Kalle Lasn and Micah White, who launched Occupy with their famous Adbusters meme, explicitly cited the leaderless, anti-hierarchical student demonstrations in Paris as their inspiration. Yet it’s telling that the sixties critique of uniformity and authority was appropriated not only by subsequent social movements, but by capitalism. Calls “to live, to express oneself, to be free” now survive as tag lines for soft-drinks and SUVS. In their book The New Spirit of Capitalism, Luc Boltanski and Eve Chiapello go so far as to argue that capitalism co-opted the ’68 demands for autonomy to create our contemporary postindustrial economy of worker instability and commanded flexibility. The ease with which the sixties ethos of autonomy was co-opted by capital didn’t seem to trouble Lasn and White, however. They saw the rise of autonomous politics as a source of potential rather than a sign of a weakened and splintered left. In their famous July 2011 call to occupy, Lasn and White celebrated a worldwide shift in revolutionary tactics. They quoted Raimundo Viejo, an activist with the 15-M movement in Spain: “[B]ack then our model was to attack the system like a pack of wolves. There was an alpha male, a wolf who led the pack, and those who followed behind. Now the model has evolved. Today we are one big swarm of people.” Following this leaderless model, Lasn and White took no ownership of the movement. In fact, neither even visited Zuccotti Park that fall. Instead, they shared their idea with a listserv of ninety thousand “redeemers, rebels, and radicals” and allowed it to evolve on its own. While they suggested ending Citizens United as one unifying demand, they left it to the occupiers themselves to formulate “one demand in a plurality of voices.” But the one-plank platform never materialized. As the protest expanded to include thousands of occupiers in hundreds of cities, the mainstream media increasingly pressed Occupy to lay out a set of policy desires. The fast-approaching winter months exacerbated these pressures, as camps across the country scrambled to figure out how to sustain an outdoor occupation in the cold. Occupy’s system of consensus-based decision-making turned this into a tiring and time-consuming , if not effectively

impossible, task. Inspired by Spain’s acampadas, the occupiers opened general assemblies to everyone. If someone vehemently disagreed with a proposal, they could block it, and it would have to be adapted until a super-majority of ninety percent supported the proposal. It did not matter if the dissenter was new to a movement others had been invested in for weeks; they had just as much of a say as the original Occupiers. As a result, a simple decision like how to take care of laundry could take hours to make. The Occupy’s New York City General Assembly explained why the process was so important to the movement: The care we take in a consensus process to hear everyone’s opinions and weave them into a whole is a living demonstration that each one of us is important. It’s a counter to systems that tell us some people count while others don’t. In consensus, everyone matters. Consensus-based decision-making, then, was understood as a way to include marginalized voices in a movement that claimed to represent the 99 percent. But Occupy fostered inclusivity through individualism rather than organized collective action . In countless interviews about the movement,

Page 175: openev.debatecoaches.org€¦  · Web view1AC . Observation 1: If I could find the spot where truth echoesI would stand there and whisper memories of my children's future. I would

participants cautioned, “I can only speak for myself.” The ubiquitous caveat frustrated attempts to understand Occupy as a whole and to advance a collective project . Discussion became confined to personal experiences and opinions. Instead of a rallying point of unison, “We are the 99%” became a Twitter hashtag for the individual to express his personal struggles. In casting individual experiences as fundamentally unknowable to others, occupiers ignored the feminist insistence that the “personal is political.” The second-wave slogan emerged as a reaction to the capitalist credo of personal responsibility, pointing to the ways in which individual experiences were not merely private but in fact defined and shaped by political and social institutions. Distinct, yes — but still fundamentally collective. As Occupy gradually took shape, it moved in the opposite direction, loading the burden on the individual. Too often, the political was nothing but the personal. Too frequently, occupiers focused on “kill[ing] the one-percenter within” instead of pressuring the state to rein in the 1 percent. All of this was fine with Adbusters. As White’s latest book makes clear, the anti-consumerist, culture-jamming magazine still regards resistance as a matter of changing one’s thought patterns and lifestyle choices. In The End of Protest — a self-styled “new playbook for revolution” — the Adbusters editor touts a “subjectivist” theory of protest, arguing that by transforming one’s inner reality, one can transform one’s external material reality. “In our global struggle to liberate humanity,” he writes, “the most significant battles will be fought on the spiritual level — inside our heads, within our imagination and deep in our collective unconscious.” While it’s easy to laugh at his claim that “climate change is happening because of the state of our minds ,” this sort of magical thinking survives in the recent outpouring of praise for Occupy as consciousness-raising. It also lives on in the call to check one’s privilege , which often imagines that self- edification — rather than structural change — can solve inequality . But realizing one’s individual privilege does nothing to dismantle the prison industrial complex , or block environmentally destructive legislation, or improve wages for workers. As Barbara J. Fields remarks: Ideology is not a set of attitudes that people can “have” as they have a cold, and throw off the same way. Human beings live in human societies by negotiating a certain social terrain, whose map they keep alive in their minds by the collective, ritual repetition of the activities they must carry out in order to negotiate the terrain. If the terrain changes, so must their activities, and therefore so must the map. Fields’ definition of ideology puts the power of personal

autonomy in its place. Rather than a “law unto itself,” the individual is situated in the prevailing system of authority. Before the individual can be changed, the system must be restructured. Lessons From Occupy Both Fields’ conception of ideology and the shortcomings of Occupy are worth keeping in mind as we mobilize against Trump and his racism, sexism, and Islamophobia. An effective opposition movement cannot be about privilege checking or soul searching . It must challenge the structures and forces that brought Trump to power. Campus safe spaces won’t do this. Neither will safety pins. Instead we must stand against the disastrous effects of deregulation, privatization, and a disintegrating social safety net. We must organize collectively against an economic system that uses racism, xenophobia, and misogyny to pit worker against worker. Taking on an entire economic and political system sounds daunting. Especially in the face of a rising far right , smaller victories — like moral denunciations and attitudinal shifts — may seem like our only option.

The first impact is democracy- empirics show the attitude of the affirmative drives down participation, decreasing the position of the people in our decisionmakingGustavsson 07 http://www.diva-portal.org/smash/get/diva2:54566/FULLTEXT01.pdf Individualism is often thought to constitute the cultural core of the American dream. Indeed, individualism has been called “the dominant ideology of American life”.24 Ever since Alexis de Tocqueville, this mythical individualism has invoked both fear and awe. The fear has been

grounded in concerns over the potentially eroding effects that individualism might have on America n society ; concerns

that are now becoming perhaps more articulate than ever. There is now extensive evidence that political engagement in the Un ited S tates , or at least traditional forms of it, is rapidly declining. This change seems to have taken place sometime between the

1970’s and the late 1990’s. During this time, there seems to have been a considerable decrease in both voter turnout , petition signing, attendance of town meetings and political rallies, serving on committees , making political

Page 176: openev.debatecoaches.org€¦  · Web view1AC . Observation 1: If I could find the spot where truth echoesI would stand there and whisper memories of my children's future. I would

speeches, writing political articles and participating in groups concerned with governmental reform .25

Meanwhile, however, other radical changes have taken place. The number of protest and civil rights movements has steadily grown higher than ever. And during roughly the same two or three decades, the American population has also become much more tolerant and supportive of diversity. Compared to the 1970’s, the American population of the late 1990’s showed a much greater support for gender equality, civil liberties and racial integration.26

Impact two is soft despotismFoundation for Constitutional Government no date http://thegreatthinkers.org/tocqueville/introduction/

Tocqueville fears that democratic individualism would produce what he called, in his first volume, “tyranny of the majority,”

and, in the second, “soft despotism.” As the English liberal John Stuart Mill understood, this does not refer to a majority imposing its will

on a minority, but, rather, to the propensity of democratic peoples to develop highly abstract political ideas and

erect bureaucratic structures that rob them of the need to act or think for themselves except on the most trivial matters.

Democracy could thus pose a danger to both intellectual and political freedom . Tocqueville notes that this new despotism

is perhaps more insidious than the traditional despotism, because it threatens to enslave the souls of men rather than simply their bodies.

Trump loses in 2020: wall consolidates opposition of key ambivalent votersBrownstein 1/11

[(Ronald Brownstein is Atlantic Media's Editorial Director for Strategic Partnerships, in charge of long-term editorial strategy, The Atlantic, “Trump’s Wall Could Cost Him in 2020,” Pub: 10 Jan 2019, SFR//ew, Acc: 11 Jan 2018, https://www.theatlantic.com/politics/archive/2019/01/trumps-border-wall-could-cost-him-2020-votes/579952/)]

Trump’s Wall Could Cost Him in 2020 The president’s relentless effort to cement the loyalty of his base is alienating him from the ambivalent voters who provided him critical support in 2016. President Donald Trump may now be talking more about steel than cement, but his proposed border wall remains the Rosetta Stone for understanding both his conception of the presidency and his political strategy. Nothing better illustrates Trump’s

political calculus than his determination to build the wall, a goal that most Americans consistently oppose in polls, even at the cost of shutting down the federal government , a tactic that surveys show most Americans also consistently reject. Politically, the

showdown over the shutdown demonstrates how much more Trump prioritizes energizing and mobilizing his passionate base, often

with messages that appeal to anxiety about demographic and cultural change, over broadening his support toward anything that approaches a majority of the country. It sends the same message about his priorities in executing his office. Trump makes no pretense of governing as president of the entire nation. Instead, he governs as the champion of his slice of America against all the forces in the country his supporters dislike or distrust—an instinct he displayed again this week with his latest threats to cut off disaster-relief funding for California. For a president to consistently steer his governing agenda and political messaging toward a demonstrable minority of the country is, to put it mildly, a novel strategy. But Trump may feel comfortable playing on the short side of the field because it’s worked for him before. In the 2016 election, a majority of voters said they had an unfavorable view of him personally and did not believe he had the experience or temperament to succeed as president, according to

the Election Day exit poll conducted by Edison Research. And yet Trump, of course, won a slim Electoral College majority despite losing the popular vote by nearly 3 million. A critical question looming over 2020 is whether he could squeeze out another victory while facing opposition from a majority of the country. The struggle over the border wall actually provides a revealing gauge of

Trump’s prospects on that front. From the start of his presidential campaign, immigration, more than any other issue, captured the potential benefits of Trump’s strategy of pursuing depth of support over breadth . Even during the 2016 Republican primaries, a majority of voters opposed deporting all undocumented immigrants in all but two of the 20 states where exit polls asked their opinion. Yet the minority of voters that supported deportation backed Trump in such preponderant numbers that they provided a majority of his votes in all but two of the 20 states. The same pattern was evident in the general election. In the exit poll, just 41 percent of voters supported Trump’s border wall, while a solid majority of 54 percent opposed it. Yet Trump won a much higher share of the wall’s supporters (85 percent) than Hillary Clinton did of the wall’s opponents (76 percent). Roughly one-fourth of the wall’s opponents either voted for Trump (16 percent) or drifted away to a third-party candidate (8 percent). That disparity reflected a clear trend in the exit poll. On questions about Trump’s personal characteristics—such as whether he had the experience or temperament to succeed as president—he consistently won a higher percentage of those who said “yes” than Clinton won among those who said “no.” That pattern may have reflected doubts about Clinton, a willingness to roll the dice on a political outsider, or

Page 177: openev.debatecoaches.org€¦  · Web view1AC . Observation 1: If I could find the spot where truth echoesI would stand there and whisper memories of my children's future. I would

a desire for change. But whatever the cause, the pattern was decisive: The pivotal votes that made Trump president came from voters ambivalent at best about him and key elements of his agenda . After two years of arguing for the wall as

president, Trump has shown no ability to expand its popularity. In 10 national polls conducted during his presidency, Quinnipiac University has never found support for the wall higher than 43 percent. With his Oval Office address on Tuesday night, Trump may have further consolidated support among Republicans and conservatives, which could raise that number slightly. But his focus on grisly portrayals of undocumented immigrants is unlikely to dent the preponderant opposition to the wall among all the groups that

powered the Democratic takeover of the House last fall: minorities, young people, and college-educated white voters. Depending on the survey, the wall usually faces opposition from at least two-thirds of minorities and young people, and between three-fifths and two-thirds of college-educated whites. Looking back at 2016, Trump may conclude that lopsided opposition from those groups doesn’t matter so long as he maintains

strong support for the wall from his base. But there’s evidence that the voters hostile to the wall , and to many other aspects of Trump’s tenure, are less willing to give him the benefit of the doubt now than they were in 2016 . To test that

proposition, I compared results from the 2016 exit poll with findings from Quinnipiac’s latest national survey on the wall in December. That comparison shows that Trump ’s position among wall opponents has eroded dramatically . Overall attitudes about the wall in the December poll remained similar to 2016, with 43 percent supporting and 54 percent opposing. (Other surveys, including Quinnipiac’s, have usually found support slightly lower, at about 40 percent.) Among whites both with and without a four-year college degree, opinions on the wall in the Quinnipiac survey showed only minimal change from the exit poll, with nearly three-fifths of whites with a degree opposing the wall and nearly three-fifths of those without a degree supporting it. But while overall attitudes about the wall haven’t changed much, attitudes toward Trump have deteriorated significantly among the wall’s critics. Opposition to the wall, just like doubts about Trump’s personal characteristics, was not a deal breaker for a significant share of voters in the presidential election. In the exit poll,

18 percent of the college-educated whites who opposed the wall voted for Trump anyway, according to figures provided by Edison Research. But now, far fewer express support for Trump in general. In the latest Quinnipiac poll, just 3 percent of these voters approved of Trump’s job performance, according to data provided by Quinnipiac. Ninety-two percent disapproved. Likewise, just over one-fourth of non-college-educated whites who opposed the wall still voted for Trump in 2016. But in the latest Quinnipiac survey, only 9 percent of these whites approved of Trump’s performance, while 83 percent disapproved. In all, fully

88 percent of Americans who oppose the wall say they disapprove of Trump’s performance as president. Approval ratings correlate closely with the reelection vote for incumbent presidents . So that huge disapproval number suggests that the 2020 Democratic nominee could win a considerably higher share of wall opponents than the 76 percent who voted for Clinton in 2016. By contrast, the share of wall supporters who approve of Trump in Quinnipiac’s latest poll is about the same (83

percent) as his vote among that group in 2016 (85 percent). That means opponents of the wall are now consolidating against Trump’s overall performance at an even higher rate than its supporters are coalescing behind him . That’s a very different landscape than in 2016, and one that springs directly from two years of governing in a volatile, confrontational manner aimed almost entirely at his base, the strategy that he’s escalated again by shuttering the government over the wall. It’s possible, of course, that with slim victories in key Rust Belt and Sun Belt states, Trump can squeeze out another Electoral College victory even if he loses the popular vote again. His chances will rise if a third-party alternative again divides the voters who are resistant to him. But these

patterns of public reaction suggest that Trump’s relentless effort to cement the loyalty and stoke the outrage of his strongest supporters, compounded by his volatile behavior in office, is building a wall between him and the ambivalent voters who provided him critical support in 2016 (or at least withheld it from Clinton by

splintering to third-party candidates). The sharp movement toward Democrats in the midterm elections among independents and college-educated white voters,

both groups that broke closely for Trump in 2016, points toward the same conclusion. Trump’s monomania on the border wall shows that he remains fixated on the priorities and resentments of his core coalition . But even a 30-foot barrier probably wouldn’t protect him in 2020 if he allows the waves of discontent to continue rising among the majority of Americans who don’t consider themselves part of that ardent club .

The plan’s a populist win that Trump takes credit for and gives him the election in 2020.Yglesias 18 [Matthew Yglesias is an American blogger and journalist who writes about economics and politics. He co-founded Vox. (Matthew Yglesias, published 9-26-2018, "How Trump could win in 2020," Vox, https://www.vox.com/2018/9/26/17905752/trump-2020-strategy, accessed 10-24-2018)]

What if President Trump did popular stuff on policy ? In particular, what if Democrats do well in the midterms and take the House and start passing bills , and then Trump takes their most popular ideas and agrees with them? Or at least with watered-down versions of them? So maybe Trump signs a bill to hike the minimum wage to

$12 an hour, a deficit-financed surge in infrastructure spending, some version of a DREAM Act paired to wall funding, and maybe

even something like the Manchin-Toomey background check bill. Up until now, Trump has governed like a very hardline

conservative except on trade. But except for authoritarian views on immigration and crime, Trump doesn’t

Page 178: openev.debatecoaches.org€¦  · Web view1AC . Observation 1: If I could find the spot where truth echoesI would stand there and whisper memories of my children's future. I would

have any personal history of consistency as a conservative. And even during his 2016 campaign, he put forth a much more eclectic, heterodox version of himself than how he’s governed. Of course, maybe that won’t change no matter what happens in 2018. Maybe he’s a true prisoner of

the conservative movement. Maybe he’s always harbored Heritage Foundation sympathies and they are just now blooming. But I think a reasonable person should have some humility about his ability to foresee the future and admit that this bipartisan, populist Trump is at least a possibility . Trump’s path to victory One immediate consequence of this would be that it would give guys like Ben Sasse and Mike Lee, who sometimes like to position themselves as more high-minded than Trump, the opportunity to actually vote against the president sometimes. Any Trump-Pelosi deal could easily weather a dozen or so defections from the right that would allow that crew to own the brand of “true conservatives” without needing to do anything to check Trump’s corruption or authoritarianism. Conversely, it would greatly empower the left of the Democratic Party because it would cost more moderate factions their

resistance branding. Trump’s electoral base wouldn’t mind a handful of ideological betrayals since rank-and- file Republicans are really here for the culture war stuff and not for the concrete policy anyway . So Trump would enter the 2020 campaign with his base intact but also with the brand as a freethinking moderate who’s at odds with the right wing of congressional Republicans . Democrats would end up nominating

someone with a relatively extreme rejectionist profile, and Trump would be in a good position to improve his approval ratings and get reelected . Think of it as basically a rerun of Bill Clinton in 1995-’96 with “ triangulation .” Now, of course, there are other historical models. Richard Nixon signed a lot of moderately progressive legislation to try to salvage his presidency, and it didn’t work. And the fairly productive Bush-Pelosi 110th Congress didn’t restore the GOP’s popularity. Anyway, who knows? But I think the odds of a Trump rebound are underrated.

A Trump re-election would be four more years of ableist hell.Harnish 17 (Andrew Harnish, Disability & Society, 2/15/17, “Ableism and the Trump phenomenon,” p. 424-425, https://www.tandfonline.com/doi/pdf/10.1080/09687599.2017.1288684?needAccess=true)//jy

But reprehensible as it is, Trump’s ableist rhetoric plainly has purchase. One reason is simple: many in the white working class – and the working class writ large – are deeply fearful of disability. Until recently, life for rural, white, able-bodied Americans was broadly gratifying. The automation of agriculture, the outsourcing of manufacturing, and the neoliberal assault on social services have left many rural Americans unable to find jobs in their communities and debilitated by the great distances they must travel to find employment. Life in rural America is increasingly precarious, yet rural American culture does not have a strong tradition of valuing bodily difference or conceiving of people as having worth beyond their labor value. While the urban working class has access to a range of (often inadequate) government services, the rural working class often lacks even this limited access. Low population density, limited access to public transportation, and few social service offices make rural communities difficult to navigate for those with disabilities. Yet, at least electorally, rural communities have not generally embraced the services that might make the neoliberal disruptions of the last quarter century more endurable. Part of this is because the culture of rural America remains independent, rooted in family and contemptuous of the costs and diversity of the metropolis, scornful of social investments, the ‘welfare’ that always seems to benefit ‘someone else’ (Williams 2016). Part, too, is due to a powerful conservative media apparatus that is quick to denounce urban entitlements, but seldom, if ever, highlights the government subsidies that benefit its largely rural, white audience. In the face of its own increasing economic and even bodily difference, much of rural America has redoubled its isolation. This is the environment Trump preyed upon with his ableist rhetoric. Rural electoral complicity with the neoliberal assaults on the welfare state has created an environment where disabled working-class voters have few resources to fall back on. It is no wonder that many rural, working-class whites are

Page 179: openev.debatecoaches.org€¦  · Web view1AC . Observation 1: If I could find the spot where truth echoesI would stand there and whisper memories of my children's future. I would

frightened of disability. The federal government still offers people with disabilities limited financial assistance, $1070 dollars per month for ‘non-blind persons’ and $1800 dollars monthly for ‘blind persons’ (‘Facts and Figures’ 2014), but these sums are so low that many Americans cannot imagine living on them, especially as, in much of the United States, poverty is treated as a moral flaw. So far, the left has not done a good job persuading rural America that more accessible and inclusive communities will increase its flourishing. Hillary Clinton tried to make this case – she spoke directly to people with disabilities. But Donald Trump eschewed this vision, denouncing racial and disabled minorities, and promising to create rural, white working-class jobs, and trounced Clinton in rural communities. Employment seems to promise the ‘independence’ prized in rural, working-class culture. Conveniently for conservatives, these promises are also a substitute for economically costly investments in the built environment of rural communities: in public transportation, social services, and child care, investments that would make those communities more livable for people with disabilities and those with limited economic means. The rural, white working class embraced Donald Trump, but his policies are liable to make all rural Americans more vulnerable, and especially those with disabilities. His ableist rhetoric, capitalizing on an ableist culture, will increase the structural ableism that already obtains in the United States.

A Trump re-election would be four more years of anti queer hellJohnson 2019, (Chris Johnson is Chief Political & White House Reporter for the Washington Blade. Johnson attends the daily White House press briefings and is a member of the White House Correspondents' Association, 6/5/2019, https://www.washingtonblade.com/2019/06/05/all-of-trumps-anti-lgbt-actions-since-last-pride-plus-a-few-welcome-moves/, 7/17/2019, SG)

President Trump acknowledged Pride month via Twitter last week, but his well wishes for the LGBT community fell on skeptical ears following

the extensive anti-LGBT actions of his administration. In just the year since last Pride, the tally of anti-LGBT actions from the Trump administration dwarf the number of good things that have come from his presidency for the LGBT community. With Pride celebrations underway, the Blade presents a list in no particular order of Trump’s positive and negative

actions with direct impact on the LGBT community since 2018’s Pride celebration. (-) 1. Embracing the Masterpiece Cakeshop decision When the U.S. Supreme Court issued a narrow ruling last year in favor of Colorado baker Jack Phillips, many observers saw the

decision as limited. After all, justices declined to find the First Amendment right Phillips asserted to refuse to make custom-made wedding

cakes for same-sex couples. But the Trump administration fully embraced the decision as a win for “religious freedom.” White House Press

Secretary Sarah Huckabee Sanders said the court “rightly concluded” the Colorado Civil Rights Commission “failed to show tolerance and

respect” for Phillips’ religious beliefs. Soon after, the Labor Department issued guidance to ensure enforcement of LGBT non-discrimination

rules complied with the ruling’s deference to religious freedom, even though the Trump administration wasn’t required to take that action. (-)

2. White House meeting with Ginni Thomas President Trump continues to meet with anti-LGBT activists in the White House, including a recent high-profile discussion with Ginni Thomas, the wife of conservative U.S. Associate Justice

Clarence Thomas. The New York Times reported Trump met in January with anti-LGBT activists led by Thomas in the Roosevelt Room of the

White House. As Trump was reportedly “listening quietly,” members of the group denounced transgender people serving in the U.S. military. In

addition to decrying transgender military service, the anti-LGBT activists said women shouldn’t serve in the military “because they had less

muscle mass and lung capacity than men.” They also said the Supreme Court ruling for marriage equality is “harming the fabric of the United

States” and sexual assault isn’t pervasive in the military, according to the New York Times. (-) 3. Coming out against the Equality Act In the same

week the U.S. House voted to approve the Equality Act, legislation that would amend the Civil Rights Act of 1964 to ban anti-LGBT

Page 180: openev.debatecoaches.org€¦  · Web view1AC . Observation 1: If I could find the spot where truth echoesI would stand there and whisper memories of my children's future. I would

discrimination, Trump came out against the bill. In an exclusive statement to the Blade, a senior administration official said Trump opposes the

Equality Act based on unspecified “poison pill” amendments to the legislation. “The Trump administration absolutely opposes discrimination of

any kind and supports the equal treatment of all; however, this bill in its current form is filled with poison pills that threaten to undermine

parental and conscience rights,” the official said via email. (+) 4. AIDS advisory council restaffed One year after firing all members of the

Presidential Advisory Council on HIV/AIDS without explanation as first reported by the Blade, Trump restaffed the advisory body with 11 new

appointees. Carl Schmid, deputy director of the AIDS Institute, and John Wiesman, secretary of health in Washington State, were named as co-

chairs for the advisory council. Months later, the Department of Health & Human Services named nine additional members to PACHA from a

variety of professions, including the pharmaceutical industry, activism and academia. (-) 5. Trans military ban implemented After the

U.S. Supreme Court essentially green lighted Trump’s ban on transgender people in the military, the Defense Department implemented the

policy in April. Denying the transgender ban is, in fact, a ban, the policy prohibits anyone who has undergone gender reassignment surgery from

enlisting in the military and requires anyone who identifies as transgender to serve in their biological sex (which would be a small number of

transgender people.) Although transgender people who were already serving openly won an exemption, individuals who are diagnosed in the

future with gender dysphoria or obtain transition-related care would be discharged. (-) 6. Brief against trans protections under Title VII In a brief

urging the U.S. Supreme Court not to take up a case seeking clarification on whether anti-trans discrimination is a form of sex discrimination

under federal law, the Trump administration asserted the U.S. Sixth Circuit Court of Appeals wrongly decided transgender people have

protections under Title VII of the Civil Rights Act. “The court of appeals’ conclusion that gender-identity discrimination categorically constitutes

sex discrimination under Title VII is incorrect,” the filing says. “As discussed above, the ordinary meaning of ‘sex’ does not refer to gender

identity…The court’s position effectively broadens the scope of that term beyond its ordinary meaning. Its conclusion should be rejected for

that reason alone.” (-) 7. List of anti-LGBT appointments grows The U.S. Senate continues to confirm Trump’s appointments, many of whom

have long anti-LGBT records. The latest will reportedly be former Virginia Attorney General Ken Cuccinelli, who once said homosexual acts are

“against nature and are harmful to society,” for a position at the Department of Homeland Security Other confirmations include U.S. District

Judge Howard Nielson of Utah, who as an attorney argued a gay judge shouldn’t be able to decide the case against California’s Proposition 8,

and U.S. District Judge Chad Readler of Ohio, who as acting assistant U.S. attorney general penned his name to briefs in favor of the transgender

military ban and against LGBT protections under Title VII. (+) 8. But a few are from the LGBT community A handful of Trump’s appointments are

from the LGBT community. Among them is former Log Cabin Republicans executive director R. Clarke Cooper, whom Trump appointed to a

senior position at the State Department for political-military affairs. The Senate confirmed Cooper in April. Other new LGBT appointments are

Mary Rowland, a lesbian with ties to the LGBT group Lambda Legal whom Trump named to a federal judgeship in Illinois; and Patrick Bumatay,

a gay federal prosecutor whom Trump named for a seat on the U.S. District Court for the Southern District of California. Both nominations are

pending before the Senate. (-) 9. Draconian anti-trans memo leaked An explosive report in the New York Times last year exposed a planned

memo within the Department of Health & Human Services that would effectively erase transgender people from federal law, igniting a massive

outcry among transgender rights supporters. The proposal reportedly asserts Title IX of the Education Amendments of 1972, which bars sex

discrimination in schools, doesn’t apply to transgender people and calls for government agencies to adopt an explicit and uniform definition of

sex “on a biological basis that is clear, grounded in science, objective and administrable.” A dispute about one’s sex, the New York Times

reported, would have to be clarified using genetic testing. (-) 10. Anti-trans ‘conscience rule’ is final The memo as described by the

Times never came to light, but months later HHS did implement an anti-trans “conscience rule” allowing health care providers to opt out of

procedures over which they have religious objections, including abortions or gender reassignment surgery. Trump announced the rule was final

during a speech in the White House Rose Garden on the National Day of Prayer. (-) 11. HHS seeks to undo trans health rule HHS wasn’t done.

Weeks after the conscience rule was final, the department announced a proposed rule seeking to undo regulations in health care against anti-

trans discrimination. The Obama-era regulations asserted Section 1557 of the Affordable Care Act, which bars sex discrimination in health care,

also covers discrimination on the basis of gender identity. Under the Trump rule, HHS would disavow those protections. (The Obama-era rule

was already enjoined by a federal judge.) (-) 12. Ending visas for unmarried partners of diplomats The State Department last

year cancelled visas for the unmarried same-sex partners of diplomats to the United States. By canceling these visas for these partners, the

State Department forced these partners to either marry or get out, which complicated matters if these diplomats are from countries where

same-sex marriage isn’t legal. At the time of the decision, only 25 countries recognized same-sex marriage. (-) 13. Proposal to gut trans protections at homeless shelters Despite assurances from Secretary of Housing & Urban Development Ben Carson LGBT non-

discrimination rules for federally funded housing would remain in place, HUD has proposed a rule that would gut transgender protections at

homeless shelters. The HUD proposal would allow homeless shelters with sex-segregated facilities — such as bathrooms or shared sleeping

Page 181: openev.debatecoaches.org€¦  · Web view1AC . Observation 1: If I could find the spot where truth echoesI would stand there and whisper memories of my children's future. I would

quarters — to establish policy consistent with state and local laws in which operators consider a range of factors when determining where to

place individuals looking to stay, including “religious beliefs.” (+) 14. Trump announces HIV plan in State of the Union Trump

in his State of the Union address announced an initiative to end the HIV epidemic by 2030, asserting “remarkable progress in the fight against

HIV and AIDS” in recent years. “Scientific breakthroughs have brought a once-distant dream within reach,” Trump said. “My budget will ask

Democrats and Republicans to make the needed commitment to eliminate the HIV epidemic in the United States within 10 years. We have

made incredible strides. Incredible. Together, we will defeat AIDS in America and beyond.” The plan seeks to reduce new HIV diagnoses by 75

percent within five years, and by 90 percent within 10 years. Efforts will focus on 48 counties, D.C., and San Juan, Puerto Rico and seven states

where the epidemic is mostly in rural areas. (+) 15. And the budget follows through with that request Trump’s budget request for fiscal year

2020 made good on his pledge in the State of the Union address, seeking $300 million in new funds for domestic HIV programs. The bulk of the

$300 million figure is an additional $140 million requested for HIV prevention at the Centers for Disease Control & Prevention, which is a 19

percent increase in its overall budget from fiscal year 2019. The rest is $70 million for the Ryan White Health Care Program, $50 million for PrEP

services at HRSA centers and $25 million to screen for HIV and treat Hepatitis C. (-) 16. But NIH and global AIDS programs slashed But the same

budget sought to slash funds for the National Institutes for Health, which conducts HIV research, and global AIDS programs like PEPFAR.

Moreover, the plan sought to make Medicaid a block-grant program, even though 40 percent of people with HIV rely on it. Congress ended up

rejecting the cuts, fully funding NIH and global AIDS programs. (-) 17. Giving Pete Buttigieg nickname of ‘Alfred E. Neuman’ Consistent with his

track record of giving his political opponents nicknames, Trump gave an unflattering moniker to Pete Buttigieg, the gay presidential candidate

with growing support in the Democratic primary. Trump dubbed him “Alfred E. Neuman,” the Mad Magazine character famous for the phrase,

“What Me Worry?” In a dog whistle that perhaps gay people could hear, Trump said, “Alfred E. Neuman cannot become president of the United

States.” (+) 18. Recognizing global initiative to end anti-gay laws In his tweet recognizing June as Pride Month, Trump also acknowledged his

global initiative to decriminalize homosexuality. Currently, same-sex relations are illegal in 71 countries. The project is spearheaded by U.S.

Ambassador to Germany Richard Grenell, the highest-ranking openly gay person in the Trump administration. Previously, Trump seemed

unaware of the project. Asked about it by reporters, Trump said, “I don’t know which report you’re talking about. We have many reports.” (-)

19. No State Dept. recognition of Pride Month, IDAHO In contrast to Trump, the State Department in 2019 issued no

statement recognizing Pride Month, nor weeks before did it recognize the International Day Against Homophobia & Transphobia. In 2018,

Secretary of State Mike Pompeo issued statements recognizing Pride Month and IDAHO. Coming off a confirmation process in which he was

criticized as homophobic, Pompeo said “too many governments continue to arrest and abuse their citizens simply for being lesbian, gay,

bisexual, transgender or intersex.” (-) 20. Refusing to recognize birthright of child to gay couple Consistent with the policy of cracking down on

immigration, the Trump administration refused to recognize the birthright citizenship of the son of U.S.-citizen Andrew Dvash-Banks and his

Israeli husband Elad Dvash-Banks. The couple had two twin boys conceived via a surrogate mother in Canada. The State Department, however,

required a DNA test to prove the children were related to the couple to provide them U.S. passports. One child, Aiden, was deemed a citizen

because he’s the biological son of Andrew, but the other, Ethan, wasn’t because he’s the biological son of Elad. (-) 21. And appealed a court

ruling for the couple When the couple sued the Trump administration, a court sided with the couple in granting birthright citizenship to Ethan.

However, the State Department refused to accept the decision and appealed the ruling to the U.S. Ninth Circuit Court of Appeals, where the

case remains pending. A mediation document reveals the State Department insists on its policy of “a biological relationship between a U.S.

citizen parent and a child born outside the United States” to grant citizenship. (-) 22. LGBT protections watered-down in USMCA An initial version of the USMCA trade agreement with Canada and Mexico contained at the behest of Canadian Prime Minister Justin

Trudeau language a call for countries to adopt policies “against sex-based discrimination, including on the basis of sexual orientation and

gender identity.” But Trudeau publicly buckled when asked about his commitment. After additional negotiations with the Trump

administration, a footnote was added to USMCA stating Title VII in the United States, which bars discrimination on the basis of sex in the

workforce, was sufficient to meet the requirements of the deal. (-) 23. DOJ’s ‘Religious Liberty Task Force’ Before he was sacked by Trump,

former U.S. Attorney General Jeff Sessions held a summit at the Justice Department on religious freedom featuring Masterpiece Cakeshop’s

Jack Phillips and Catholic leaders. At the summit, Sessions established the Religious Liberty Task Force. The goal of the task force was to ensure

his memo on “religious freedom” — widely seen as guidance in support of anti-LGBT discrimination — was being implemented throughout the

federal government. (+) 24. Hailing PrEP deal with Truvada as ‘great news’ The Department of Health & Human Services reached a deal with

Gilead to make PrEP available for generic production one year earlier and to secure a donation of the medication for up to 200,000 individuals

each year for up to 11 years. Trump took to Twitter to hail the agreement: “Great news today: My administration just secured a historic

donation of HIV prevention drugs from Gilead to help expand access to PrEP for the uninsured and those at risk. Will help us achieve our goal of

Page 182: openev.debatecoaches.org€¦  · Web view1AC . Observation 1: If I could find the spot where truth echoesI would stand there and whisper memories of my children's future. I would

ending the HIV epidemic in America!” (-) 25. Deleting trans employee guidance on OPM website In a little-noticed development over the

holidays, guidance on the Office of Personnel Management’s website for federal workers who are transgender was deleted without

explanation. The Obama-era guidance spelled out the definition of terms for transgender identities and expectations for respecting transgender

workers. The guidance ensured transgender people could dress according to their gender identity, be addressed by their preferred gender

pronouns and use restrooms and locker rooms consistent with their gender identity. (+) 26. U.S. joins OSCE in calling for Chechnya investigation

Under the Trump administration, the United States joined 15 allied countries at the U.S. Organization for Security & Cooperation in Europe in

the creation of a probe to investigate alleged anti-gay human rights abuses in Chechnya. The report concluded, as the United States and human

rights organizations long believed, Chechen government officials engaged in human rights violations, including “harassment and persecution,

arbitrary or unlawful arrests or detentions, torture, enforced disappearances and extrajudicial executions.” Victims were LGBT people, human

rights defenders, journalists and members of civil society. (-) 27. But U.S. didn’t sign U.N. statement against atrocities Months later, the United

States was nowhere to be found on a United Nations statement signed by more than 30 countries calling for a thorough investigation of the

Chechnya atrocities. The State Department said the United States didn’t sign because it withdrew from the U.N. Human Rights Council “and no

longer participates in its sessions.” (-) 28. State Department proposes ‘natural law’ commission LGBT rights supporters are viewing with

skepticism a State Department proposal to create a “natural law” commission, which is set to “provide fresh thinking about human rights

discourse where such discourse has departed from our nation’s founding principles of natural law and natural rights.” The term “natural law”

has been used to express condemnation of LGBT identities in religious discourse. (-) 29. Eliminating LGBT youth data question in foster care The Trump administration has proposed eliminating requirements for case workers to ask LGBT youth in foster care about their

sexual orientation of youth for data collection purposes. Although the Department of Health & Human Services concluded it was “intrusive and

worrisome,” LGBT rights advocates say the questions are necessary to ascertain disparities facing LGBT youth in the foster care and adoption

systems. (-) 30. Trump stands with anti-LGBT adoption agencies In a speech at the National Prayer Breakfast, Trump expressed solidarity with religious-affiliated adoption agencies, who are bristling over LGBT non-discrimination

requirements to obtain federal funding. “My administration is working to ensure that faith-based adoption agencies are able to help vulnerable

children find their forever families while following their deeply held beliefs,” Trump said. (-) 31. And defends Karen Pence teaching at anti-LGBT school In the same speech, Trump also defended second lady Karen Pence for her decision to teach art at a Christian school in Virginia, which has a policy against employing LGBT teachers or admitting

LGBT students. “She just went back to teaching art classes at a Christian school,” Trump said, “Terrific woman.”

Orientalism is the structuring force behind both anti-Semitism and Israeli oppression of Palestinians.

Khazzoom 2003 [Aziza, “The Great Chain of Orientalism: Jewish Identity, Stigma Management, and Ethnic Exclusion in Israel” American Sociological Review Aug, 2003 accessed July 6, 2019 https://www.jstor.org/stable/1519736

Aziza’s work traces the formation of ethnic inequality among Jews in Israel, combining quantitative and qualitative methods. She is the author of Shifting Ethnic Boundaries and Inequality in Israel, Or: How the Polish Peddler Became a German Intellectual, published by Stanford University Press in 2008. The book focuses on why ethnic discrimination occurred in Israel, and argues that concerns over producing the state as western centrally determined who was excluded and who was included. Other work on ethnic formation in Israel has appeared in the American Sociological Review, Social Forces, and Signs. She is currently collecting life stories of Polish and Iraqi Jews who immigrated to Israel in the 1950s. She has held NSF and ISF grants and postdoctoral fellowships from Tel Aviv University and the Van Leer Institute.]

 

Several features of race and ethnicity are now abundantly clear to sociologists. First, racial and ethnic boundaries are mutable social constructions. Second, they are relational constructs in the describing a “self” by implication describes an “other.” Third, these moving boundaries that form ethnic groups are real in their consequences as they are fundamental building blocks of social hierarchies. But

Page 183: openev.debatecoaches.org€¦  · Web view1AC . Observation 1: If I could find the spot where truth echoesI would stand there and whisper memories of my children's future. I would

while sociologists care about shifting boundaries largely because they are so often related to exclusion, precisely this connection between boundaries and exclusion has received little empirical attention. Currently, the literatures on immigration and racialization are concerned primarily with charting the process by which group boundaries are created, strengthened, or weakened. Within this work it is not always clear why definitional moves are initiated, why boundaries take a particular form when others are available, or how the emergence of ethnic

inequality. Here, I use one instance of rapid transformation in ethnic boundaries to explore these issues. Contemporary Jewish Israeli society was formed in the 1950s as the result of a massive immigration of Jews from many countries.

Despite significant heterogeneity among the immigrants, a bifurcated social structure emerged in which Ashkenazim (European, primarily East European Jews) were the dominant group and Mizrahim (Middle Eastern and North African Jews) were

subordinate (Amit 2001; Nahon 1987). It has been argued that one of the ways Ashkenazim acquired their domination of Jewish Israeli society was to “Orientalize” Mizrahim. That is, Ashkenazim used the previously-existing east/west dichotomy to advance a binary construction of ethnicity in Israel in which the heterogeneity of the arriving cultures was simplified into two, homogeneous categories: Ashkenazim, who were fully “eastern” (Shohat 1988, 1989). The bifurcation in the realm of representation, it is argued, both shaped and justified an unequal distribution of resources

(Bernstein and Swirski 1982). As a result, the binary ethnic division has become entrenched, that that across a range of important contexts Israel ethnic groups. The imposition of new ethnic boundaries in Israel was clearly a first step in the process of exclusion. However, the dynamics most often invoked to explain these moves- competition for scarce material resources and he natural tendency to help one’s own- cannot account for the choice of these particular ethnic

boundaries over others or for their emergence as central aces of exclusion. I suggest that a far better explanation lies in an expanded and historicized account of orientalism in the Jewish world. Based on a review of secondary source

materials, and building on the works of Goffman (1963) and Said (1978), I argue that the past two centuries of Disapora Jewish history in Europe and the Middle East can be conceptualized as a series of orientalizations. Through the history, Jews came to view Jewish tradition as oriental, developed intense commitments to westernization as a form of self-improvement, and became threatened by elements of Jewish culture that represented the oriental past. Self-classification then drove how others were classified- perceived levels of

westernization became the primary determinant for evaluation other Jewish communities , and exclusion and

resulted when a putatively less western group threatened the westernization project of another. In Israel , I argue , the Ashkenazi move to create Mizrahim, and move to exclude them and Middle Eastern non-Jews, both developed from this historical need to manage a “spoiled identity.” (Goffman 1963). Thus it was not the formation of ethnic groups per se that accounts for the emergence of ethnic closure but the content of the identities around which they were formed.

Anti-Semitism and Palestinian liberation are constantly defined in a relation of opposition structured through orientalism, Roosevelts politics provesLittle 08(Douglas Little, Professor of History and International Relations at Clark University., "American Orientalism: The United States and the Middle East since 1945 ", University of North Carolina Press, 3rd edition, 2008, DMW)

Americanizing the Middle East The Middle East began to loom larger on America’s diplomatic and cultural horizon during what Mark Twain called “the Gilded Age,” not only because U.S. missionaries sought to save more souls but also because U.S. merchants sought to expand trade . By the 1870s American entrepreneurs were buying

nearly one-half of Turkey’s opium crop for resale in China while providing the Ottoman Empire with everything from warships to kerosene.

“Even the sacred lamps over the Prophet’s tomb at Mecca,” one U.S. diplomat gloated in 1879, “are fed with oil from Pennsylvania.”19

Meanwhile a new generation of American missionaries made their way to Armenia, Syria, and other corners of the Ottoman realm, spreading not

Page 184: openev.debatecoaches.org€¦  · Web view1AC . Observation 1: If I could find the spot where truth echoesI would stand there and whisper memories of my children's future. I would

only the gospel but also subversive New World ideas. Indeed, by the 1890s two in- stitutions of higher learning established by U.S. missionaries

three decades earlier —Robert College just outside Constantinople and the Syrian Protestant College in Beirut — had become notorious anti-

Turkish hotbeds, where Arabs, Kurds, and Armenians began to dream of and scheme for national independ- ence.20 “Quite without intention,”

British orientalist and adventurer T. E. Lawrence observed a generation later, these two colleges had actually “taught revolution” to subject

peoples throughout the Turkish empire.21 While most U.S. observers seem to have agreed that the Christians of Ar- menia and Syria might profit

enormously from these lessons, few churchmen or diplomats expected such revolutionary teachings to spell anything but dis- aster in the Muslim

world. When angry mobs of Iranian students and peas- ants toppled the royal government and forced the shah to proclaim a consti- tutional

monarchy in August 1906, for example, Ambassador Richmond Pearson offered a bleak forecast laced with orientalism: “History does not re-

cord a single instance of successful constitutional government in a country where the Mussulman religion is the state religion.”22 Ambassador

John Leish- man, Pearson’s counterpart in Constantinople, was no more sanguine about the prospects for constitutional rule in Turkey, where

reformist military officers —“the Young Turks”— staged a coup and curbed the sultan’s powers in July 1908. “The fanatical element” among

Muslim students, soldiers, and mullahs, Leishman reported nine months later, had triggered antigovernment riots, an army mutiny, and “a reign

of terror and a succession of murders.”23 President Theodore Roosevelt, who had appointed both Pearson and Leish- man, was even more skeptical about the possibility of reform and progress in the Middle East. A firm believer in a hierarchy of race in which “civilized na- 14 orientalism, american style tions” like the United States must shoulder “the White Man’s Burden” and at- tempt to westernize the “benighted” peoples of Asia, Africa, and Latin Amer- ica, Roosevelt confessed privately in 1907 that “it is impossible to expect moral, intellectual and material well-being where Mohammedanism is su- preme.” The Egyptians, for example, were “a

people of Moslem fellahin who have never in all time exercised any self-government whatever.” Britain’s Lord Cromer, Roosevelt added, “is one

of the greatest modern colonial administra- tors, and he has handled Egypt just according to Egypt’s needs”— military oc- cupation, foreign

tutelage, and Christian patience.24 If Roosevelt ranked Muslims near the bottom of his hierarchy of race, he placed Jews closer to the top. To be sure, like many other members of the pa- trician elite that still ruled America at the turn of the

century, Roosevelt har- bored some patronizingly offensive stereotypes of Jewish Americans. 25 But he was

also highly critical of the wave of anti-Semitism that swept Turkey and Russia during the First World War, and he was an early supporter of the

idea of establishing a Jewish state in the Holy Land. The United States and its al- lies, Roosevelt observed in July 1918, should “pledge

themselves never to make peace until the Turk is driven from Europe, and . . . the Jews [are] given control of Palestine.” It seemed, he added two

months later, “entirely proper to start a Zionist State around Jerusalem.”26 As the war to end all wars drew to a close, the establishment of a Jewish homeland in Palestine had become a goal widely shared on both sides of the Atlantic. Famous mainly for its biblical ruins and its fruit exports, Palestine had until very recently remained little more than a sleepy

backwater con- trolled by the dying Ottoman Empire. Overwhelmingly Muslim, Palestine had counted just 25,000 Jews among a total population

of 300,000 as late as 1880.27 Fifteen years later, however, Theodore Herzl, a thirty-five-year-old Jewish lawyer-turned-journalist born in

Budapest, published what might be called the first Zionist manifesto. Outraged by the pogroms in Russia and Poland and appalled by the resurgence of anti-Semitism farther west in France, Herzl warned his brethren in the pages of The Jewish

State that only by es- tablishing a national home in Palestine could they be safe from persecution. Working

tirelessly, Herzl brought together Jews from seventeen countries, in- cluding the United States, in Basel, Switzerland, where in August 1897 they

founded the World Zionist Organization committed to accelerating Jewish im- migration to Palestine by purchasing land from the Arabs.

Zionist efforts bore fruit in short order and helped swell the Jewish community in Palestine to 85,000 ,

12 percent of the total population, on the eve of the First World War.28

American intervention in the conflict over Palestine privileges pits the contested notion of Jewish orientalism in opposition to the Arab oriental Little 08

Page 185: openev.debatecoaches.org€¦  · Web view1AC . Observation 1: If I could find the spot where truth echoesI would stand there and whisper memories of my children's future. I would

(Douglas Little, Professor of History and International Relations at Clark University., "American Orientalism: The United States and the Middle East since 1945 ", University of North Carolina Press, 3rd edition, 2008, DMW)

As enthusiasm for a Jewish state in Palestine faded at the White House and at Whitehall during the 1920s, an

upsurge of nativism eroded support on Main Street for the Zionist dream. From Atlanta to Anaheim the Ku Klux Klan burned crosses and staged

rallies to intimidate African Americans, Cath- olics, and Jews, while on the banks of the Potomac Congress was erecting re- strictive quotas to

stem the flow of Jews and other “undesirable” groups from Eastern Europe. Fearful that a Zionist success in Palestine might inadvertently call into question the loyalty of the entire Jewish community in the United States, influential American Jews such as New York Times publisher Arthur Hays Sulzberger distanced themselves from lobbying efforts on Capitol Hill. By the early 1930s membership in the Zionist Organization of America, an orientalism, american style 19 umbrella agency founded by Brandeis and Wise a generation earlier, had plum- meted from a

postwar high of 175,000 to just 25,000, convincing the State De- partment’s elitist and sometimes anti-Semitic Middle Eastern experts that they

could safely ignore this first Jewish foray into interest group politics.44 By late 1936 the State Department’s Wallace Murray had convinced his supe- riors to do nothing that might “entangle us in any other way in the most del- icate problem of Palestine.”45 Murray’s brief for U.S. noninvolvement could not have come at a worse time for American Zionists or their comrades in Europe. Claiming that the influx of 250,000 European Jews during the decade and a half since the First World War was more than the overloaded Palestinian economic and political system could bear, in 1936 the Arabs launched a violent revolt to resist Zion- ism.

While Palestinian militias battled the Haganah, the Jewish underground army, in the streets of Jerusalem and the foothills of Nablus, even more

omi- nous events were unfolding in Germany, where Adolf Hitler’s anti-Semitic policies were growing ever more blatant. Since coming to power

in early 1933 the Nazi dictator had tarred German Jews with the brush of communism, stripped them of their civil rights, and branded them

scapegoats for the Third Reich’s economic woes. After Nazi tanks rolled into Vienna in March 1938 and after Hitler’s storm troopers went on an

anti-Semitic rampage in Berlin eight months later, thousands of German and Austrian Jews sought refuge abroad, some in Britain and America,

but most in Palestine.46 At the time when European Jewry was most desperate for a safe haven in a national homeland, however, the British government moved to reduce Jewish immigration to Palestine sharply.

Having just completed a costly two-year campaign to suppress the Arab revolt, Whitehall issued a White Paper on 17 May 1939 limiting the total

number of Jewish refugees permitted to enter the Holy Land to just 75,000 during the next five years; after that, all further im- migration would

be subject to Palestinian approval. Among the most outspo- ken critics of the 1939 White Paper was fifty-three-year-old David Ben Gu- rion, the

charismatic unofficial leader of the Yishuv, as the 350,000-member Jewish community in Palestine was now known. Convinced that persuading

Whitehall to rescind the White Paper would prove an exercise in futility, Ben Gurion and his comrades hoped American Zionists might be more

successful at the White House, where Franklin D. Roosevelt was preparing to seek an unprecedented third term with support from Jewish liberals.47 Long sympathetic to the aims of the Balfour Declaration, fdr was clearly troubled during the

late 1930s by signs that Britain intended to repudiate its commitment to a Jewish homeland. “I was at Versailles,” he recalled in 1938, “and I

know that the British made no secret of the fact that they promised Palestine to the Jews. Why are they now reneging on their promise?”48 Bri-

20 orientalism, american style tain’s actions during the spring of 1939 only raised more questions. “I have read with interest and a good deal of

dismay the decisions of the British Gov- ernment regarding its Palestine policy,” he told Secretary of State Cordell Hull in mid-May. “This White

Paper,” Roosevelt hastened to add, “is something that we cannot give approval to.”49 During the following eighteen months well-connected

Zionists such as Stephen Wise and Felix Frankfurter, whom fdr had recently tapped to fill Brandeis’s seat on the Supreme Court, quietly

encouraged the president to press Whitehall to honor its commitments re- garding Palestine. At the State Department, however, Hull and his

advisers in- sisted that U.S. meddling would only serve to undermine the U.K. position in the Middle East at a time when Britain, in the wake of

the fall of France in June 1940, was the sole remaining barrier to complete Nazi domination of Eu- rope. Judging geopolitical considerations to be

more important than domestic politics, Roosevelt kept his doubts about the White Paper to himself and still managed to win a third term by a

healthy margin.50 Thousands of European Jews unable to find refuge abroad would soon be among the earliest victims of the Holocaust. During

1939 and 1940 the Nazis had targeted the Jewish population of occupied Europe for relocation to con- centration camps in Poland. After the

Page 186: openev.debatecoaches.org€¦  · Web view1AC . Observation 1: If I could find the spot where truth echoesI would stand there and whisper memories of my children's future. I would

German invasion of the Soviet Union in June 1941, agents of the Gestapo, Hitler’s secret police, began systematically to murder all Russian Jews

who fell into their hands. By the time that Ger- many’s Japanese allies attacked Pearl Harbor on 7 December 1941, rumors of the Gestapo’s anti-

Semitic butchery were already filtering into the United States. In January 1942 Hitler formally approved a “final solution for the Jewish problem”

and authorized the Schutzstaffel, or SS, an elite corps of the German army whose commanders spoke with the voice of Cain, to begin the

wholesale extermination of hundreds of thousands of Jews then imprisoned at Ausch- witz, Buchenwald, and other concentration camps. During

the next three years nearly 6 million Jewish men, women, and children would die.51 The unspeakable slaughter unfolding in Nazi-occupied

Europe removed any remaining doubts among most American Jews about the importance of a Jewish homeland in Palestine. In May 1942, 600

American Zionists gathered at the Biltmore Hotel in New York City and passed a unanimous resolution de- manding “that Palestine be

established as a Jewish Commonwealth integrated in the structure of the new democratic world.” While insiders such as Stephen Wise sought to

win White House support for the Biltmore declaration, out- siders like Abba Hillel Silver, a fiery Cleveland rabbi born in Lithuania and ed-

ucated at Hebrew Union College, founded the American Zionist Emergency Council, whose 200 local chapters funneled a half-million dollars

into national headquarters to finance a lobbying effort in Washington.52 Nineteen forty-four was an election year, and U.S. advocates of a Jewish orientalism, american style 21 homeland in Palestine worked tirelessly to gain bipartisan endorsements for their plans.

Impact - The Aff’s orientalist imagery is crucial to perpetuating the violent division between the Orient and the West. Behdad 2010 (“Orientalism Matters,” pg. 711-712, by Ali Behdad on Accessed June 26, 2019. Ali Behdad is a Professor and John Charles Hillis Chair in Literature at UCLA. https://muse.jhu.edu/article/407130)--RSP*

"In contemporary writing about nineteenth-century photography of the Middle East," writes Michelle L. Woodward, "it has become al- most a cliché to describe many ofthese images as 'Orientalist'— that is , reflecting or propagating a system of representation that creates an essentialized difference between the 'Orient' and the 'West'"(363). This claim aptly captures the predominant anti-Saidian sentiment among art historians and curators who work on representations of the Middle East created by both European and indigenous painters and photographers. To be sure, responses to Said's discussion of Oriental- ism as a discourse of colonial power span the critical spectrum , from more rigorous and subtle critiques articulated from the left to the sometimes facile and reactionary from those of an opposing political orientation. On one side are art historians such as Zeynep Celik, Jill Beaulieu and Mary Roberts, and Woodward who argue that "the trend to extend Said's analysis to apply equally to visual representations has . . . been used too broadly, obscuring nuances and inconsistencies, not only between different photographers' bodies of work but also within them" (Woodward 363). These scholars typically aim to con- structively revise Orientalism to encompass "a disparate and disputed set of discursive constructions" while at the same time acknowledging "Orientals" as "participants in the production of counternarratives or resistant images" (Bealieu and Roberts 3). On the other side are writ- ers, such as John MacKenzie and Ken Jacobson, who betray a marked suspicion of theory and seek to return the term "Orientalism" to its prior usage as an art historical term that could be deployed without suggestion of a broader political or ideological critique. In Orientalism: History, Theory, and the Arts, MacKenzie argues against Linda Nochlin that "there is little evidence of a necessary coherence between the imposition of

Page 187: openev.debatecoaches.org€¦  · Web view1AC . Observation 1: If I could find the spot where truth echoesI would stand there and whisper memories of my children's future. I would

direct imperial rule and the visual arts," claiming that "Orientalism celebrates cultural proximity, historical parallelism and religious familiarity [with the Middle East and North Africa] rather than true 'Otherness'" (51). Given such perceived "misconceptions inherent in postcolonialist analysis," Jacobson similarly suggests that "a return to more traditional methods is desirable for the study of 19th- and early 20th-century photography in North Africa and the Near East," urging commentators to focus more single-mindedly on the "notable aesthetic, as well as documentary and historical merit" when analyzing visual representations (88). In what follows, I argue that Orientalism should not be under- stood merely as an ideological discourse of power nor as a neutral art historical term, but rather as a network of aesthetic, economic, and political relationships that cross national and historical boundaries. Understood in this way, Orientalism is indispensible to understanding nineteenth-century photography of the Middle East. Whether consid- ered in the context of their production and dissemination in the nine- teenth century or in relation to their current afterlives as collectable objects or archives, photographs of the Orient become meaningful and legible only if they are considered in terms of the specific geopolitical distinctions, economic interests, and cultural assumptions about the Middle East and its people. While insisting that Orientalism offers a crucial perspective from which to comprehend the meaning and significance of photographic representations of the Middle East, I do not mean to suggest that such images should be understood merely as a reflection of Europeans' racial prejudice against "Orientals," or that these images simply validate European imperial dominance over the region. Nor would I wish to argue that Orientalist photography entails a binary visual structure between the Europeans as active agents and "Orientals" as passive objects of representation. Rather, I hope to provide an alternative view of Orientalistphotography that focuses on nodes and ties that bind artists, collectors, and museums across historical and national boundaries, which are productive of a distinctly exotic vision of the region, a vision at once embraced and perpetuated by the elite in the Middle East .Indigenous photography in and of itself, I maintain, does not constitute an oppositional locus or resistant iconography, for it too belongs to the Orientalist network that mediates its vocabulary and thematics of representation.1 A network theory of Orientalism concerns itself neither with the motivations of individual artists nor with the attributes of art objects, but instead studies the symmetric and asymmetric relations between discrete objects, specific individuals, and concrete practices.

The alternative is to vote negative to construct the ballot as a counter-script against the orientalist discourses of the affirmative. This is key to generating space for subjectivities outside of the construction of the West. Azeez 2k16[ Govand Khalid “Beyond Edward Said: An Outlook on Postcolonialism and Middle Eastern Studies, Social Epistemology” 30:5-6, 710-727, DOI: 10.1080/02691728.2016.1172360 p. 714-716]

Page 188: openev.debatecoaches.org€¦  · Web view1AC . Observation 1: If I could find the spot where truth echoesI would stand there and whisper memories of my children's future. I would

But Orientalism, as a symbolic revolt, was not just about the critique, deconstruction and exposure of ideological and epistemological Eurocentric frameworks that monopolised the realm of discursive production. Appropriating Michel Foucault’s postulation that power is everywhere and that it is diffused and embodied in discourse, knowledge and “regimes of truth”, Said presents the second mode of critique; namely, Orientalism as the ontological nature, structure and a priori condition of power. In this sense, knowledge or the historical materialisation of specific Nietzschean truths were a form of power and power could only be deciphered through critiquing its manufactured meta-narrative and mapping out the transmutation and movement of its anthropomorphisms, metaphors and metonymies. Said demonstrates this through examining Sylvester de Sacy’s role in the Dacier Report (1802). Conducted for examining the state of Orientalist learning, the report was commissioned by Napoleon Bonaparte. Said writes: The importance of the Tableau historique for an understanding of Orientalism’s inaugural phase is that it exteriorizes the form of Orientalist knowledge and its features, as it also describes the Orientalist’s relationship to his subject matter. In Sacy’s pages on Orientalism … he speaks of his own work as having uncovered, brought to light, rescued a vast amount of obscure matter. Why? In order to place it before the student. For like all his learned contemporaries Sacy considered a learned work a positive addition to an edifice that all scholars erected together. Knowledge was essentially the making visible of material, and the aim of a tableau was the construction of a sort of Benthamite panopticon. Scholarly discipline was therefore a specific technology of power …(1978, 127).3 Elsewhere in his analysis of Arthur James Balfour’s speech in the House of Commons on the 13 June 1910, Said declares: England knows Egypt; Egypt is what England knows; England knows that Egypt cannot have self-government; England confirms that by occupying Egypt; for the Egyptians, Egypt is what England has occupied and now governs; foreign occupation therefore becomes “the very basis” of contemporary Egyptian civilization; Egypt requires, indeed, insists upon British occupation (1978, 34). In other words, Said postulated that Orientalism exuded epistemic violence through its inherent relation with European colonial power. And, in turn, European colonial power could only dominate the region through Orientalism functioning as a technique and instrument of power. Here then Said’s third definition of Orientalism, drawing from the two previous epistemological definitions, demonstrates the intrinsically dialectical relation between knowledge and power. Said posits Orientalism is also: (3) The corporate institution for dealing with the Orient—dealing with it by making statements about it, authorizing views of it, describing it, by teaching it, settling it, ruling over it: in short, Orientalism as a Western style for dominating, restructuring, and having authority over the Orient Collectively Said conceptualises Orientalism as epistemological (both as a corpus of scholarly writings and a historical system of thought) and ontological (as in the nature of being, the structure and condition of power). Read critically, for Said the synthesis of Western power and knowledge had a singular ontological and teleological purpose; namely, the thingification or subjectivation of the colonized Oriental subject. This point is best demonstrated by William D. Hart’s analysis of Orientalism: In his description of discourse, Said appropriates Foucault’s ideas of discipline and power/knowledge. By discipline, Foucault [and by default Said] means those methods of modern punitive power that establishes meticulous control over the body, assuring its constant subjection by imposing a relation of “docility-utility”. Discipline, that is, makes human bodies docile and useful [thingifies], and advertises their availability for political,

Page 189: openev.debatecoaches.org€¦  · Web view1AC . Observation 1: If I could find the spot where truth echoesI would stand there and whisper memories of my children's future. I would

economic, and cultural uses of many kinds (2000, 70). And what of the latter Said’s “voyage in” and the powerful transformative attempt by the native to “write back” in the metropolis? Can this “contrapuntal reading” (Said 1994a, 288, 79), allow a discursive space where the native-intellectual, halfinvolved and half-detached (Said 1996), rejects the “hail” and the Althusserian “quadruple system” of interpellation (Althusser 1969)? Does the deliberate attempt in Said’s words to “enter into the discourse of Europe and the West, to mix with it, transform it, to make it acknowledge marginalized or suppressed or forgotten history” (Said 1994, 260) effect this equation? This latter Said, moving away from Foucauldian discursive formation and utilizing Gramsci’s hegemony or direzione culturale [cultural direction], envisions social space alike a Bourdieuean field. Revalorizing culture, political space becomes an arena where subjects engage in competent action and deploy different modes of capital and strategies in order to advance their position within the framework of a shared system of meaning (Bourdieu 1977). In this sense, Said announces in Culture and Imperialism (1994a) that “… no matter how apparently complete the dominance of an ideology or social system, there are always going to be parts of the social experience that it does not cover and control. From these parts very frequently comes opposition both self-conscious and dialectical” (Said 1994, 289). Accordingly, Said posits that the “new receptivity to both liberation movements and post-colonial criticism” not only challenges the “monopoly of discourse by Eurocentric intellectuals and politicians” (Said 1994, 316), but also creates a new form of subjectivity residing at a “median state” whereby power’s Manichean dichotomies are obscured.

No inherency Senate voted to end support for Saudi Arabia, this is more recent then their Zheng evidence.AP 12/14 (Associated Press. “Senate Rebukes Trump Response to Khashoggi Death and U.S. Support of Yemen War.” TIME. 14 Dec. 2018, http://time.com/5479319/senate-rebukes-trump-response-khashoggi-death-yemen-war/.) LHSLA LH

Senators voted 56-41 to recommend that the U.S. stop supporting the war in Yemen , a direct affront to the administration’s war powers abilities. Independent Sen. Bernie Sanders of Vermont, who co-sponsored the Yemen resolution with Republican Sen. Mike Lee of Utah, called passage a “historic moment.” Lee said Khashoggi’s death focused attention “on the fact that we have been led into this civil war in Yemen half a world away” and “we’ve done so following the lead” of Saudi Arabia. “What the

Khashoggi event did was to demonstrate, hey, maybe this isn’t a regime that we should just be following that eagerly into battle,” Lee said. As Senate approval loomed, the administration dispatched Pompeo and Defense Secretary Jim Mattis to the House to make the case against the resolutions and warn of damage they could do to the U.S.-Saudi relationship. A congressional aide and an administration official said their appearance was aimed at stopping any House action on the resolutions. Pompeo and Mattis had made a similar entreaty to the Senate late last month.

But it was roundly panned by senators angered by the secretaries’ refusal to accept a CIA determination that assessed the crown prince had ordered Khashoggi’s murder. CIA Director Gina Haspel briefed House leaders Wednesday on the Khashoggi slaying. The journalist, who had lived in the U.S. and wrote for The Washington Post, had been critical of the Saudi regime. He was killed in what U.S. officials have described as an elaborate plot as he visited the consulate for marriage paperwork. Saudi prosecutors have said a 15-man team sent to Istanbul killed Khashoggi and then dismembered his body, which has not been found. Those findings came after Saudi authorities spent weeks denying Khashoggi had been killed in the consulate. Trump has been reluctant to condemn the crown prince. He said the United States “intends to remain a steadfast partner” of the country, touted Saudi arms deals worth billions of dollars to the U.S. and thanked the Saudis for plunging oil prices. But Graham and Sen. Bob Menendez, the top Democrat on the Foreign Relations Committee, have rejected Trump’s economic arguments. They are setting the stage for legislation next year that goes further in

halting arms sales and taking other measures. Menendez says economic concerns do not overpower human rights and

Page 190: openev.debatecoaches.org€¦  · Web view1AC . Observation 1: If I could find the spot where truth echoesI would stand there and whisper memories of my children's future. I would

the U.S. must send a “global message that killing with impunity” will not be tolerated . Frustration with the crown prince and the White House prompted several Republicans to support the Yemen resolution. Seven Republicans and all Democrats voted for it. Some already had concerns about the war, which human rights groups say is wreaking havoc on the country and subjecting civilians,

many of them children, to deadly disease and indiscriminate bombing. The resolution condemning Saudi Arabia for Khashoggi’s slaying was from Senate Foreign Relations Chairman Bob Corker and Senate Majority Leader Mitch McConnell . Both Republicans opposed the Yemen resolution and voted against it. McConnell said senators have grave concerns about Khashoggi’s killing, but “we also want to preserve a 70-year partnership between the United States and Saudi Arabia, and we want to ensure it continues to serve American interests and stabilizes a dangerous and critical region.” But McConnell encouraged passage of the Khashoggi resolution and said it provided “a clear and unambiguous message about how we feel about what happened to this journalist .” The Senate debate came as the United Nations secretary general announced that Yemen’s warring sides have agreed to a province-wide cease-fire and withdrawal of troops in Hodeida, a contested Red Sea port city. The agreement came during peace talks in Sweden. The brutal four-year-old civil war pits the internationally recognized Yemeni government, supported by a Saudi-led coalition, against the Iran-backed rebels known as Houthis.

Canada ending arms sales now—fills in gaps left by the U.S. and decimates Saudi military.Burke 12/17 (Burke, Michael. “Trudeau says Canada trying to end arms contract with Saudi Arabia.” The Hill. 17 Dec. 2018, https://thehill.com/policy/international/421673-trudeau-says-canada-trying-to-end-arms-contract-with-saudi-arabia.) LHSLA LH

Canadian Prime Minister Justin Trudeau on Sunday said Canada is looking into ending sales of armored vehic les to Saudi Arabia as pressure builds following the murder of U.S.-based journalist Jamal Khashoggi. "We are engaged with the export permits to try and see if there is a way of no longer exporting these vehicles to Saudi Arabia,” Trudeau told Canadian television network CTV, according to The Globe and Mail. Trudeau's comments come after he said in October that he was unlikely to cancel the sales. “I do not want to leave Canadians holding a billion-dollar bill because we’re trying to move forward on doing

the right thing," he said of the Saudi deal at the time. But Canada and other countries have faced increased pressure to end arms sales to Saudi Arabia following the murder of Khashoggi and amid criticism of the deadly civil war in Yemen. Khashoggi, a Washington Post columnist, was killed inside the Saudi Consulate in Istanbul in October. The CIA has reportedly concluded with high confidence that Saudi Crown Prince Mohammed bin Salman ordered the killing. The U.S. has faced pressure to end its own arm sales to Saudi Arabia since the slaying, but President Trump has maintained that he will not do so. Trump, who has refused to condemn Saudi Arabia or the crown prince over the killing of Khashoggi, has said that ending those sales would result in significant job losses in the U.S.

This means 1. no solvency and 2. They need to isolate why the aff ending arm sales is uniquely different from 1. Canada ending sales, and 2. Congress ending sales in December. 3. Fillin will happen

Saudi-Russia relations thump.Simes Jr. 19 (Dimitri Simes Jr., “Saudi Arabia Is Quietly Cozying Up To Russia: What Does It Mean For The Middle East?”, Daily Caller, 07/08/2019, https://dailycaller.com/2019/07/08/saudi-arabia-quietly-cozying-up-to-russia/)//Shreyas

During the G20 summit earlier this month, Saudi Crown Prince Mohammad bin Salman met with Russian President Vladimir Putin. After the

meeting, Putin announced that Russia and Saudi Arabia had agreed to extend an oil production cut agreement for another nine months. When the extension was formalized several days later, OPEC Secretary General Mohammad

Barkindo touted the deal as a “Catholic marriage” and boasted that it would last an “eternity.” Such cooperation between the two countries would have surprised many international observers just a few years ago. Back then, Saudi Arabia and Russia appeared headed for showdown over Syria and global oil prices. Now however, one of America’s closest allies in the Middle East and one

Page 191: openev.debatecoaches.org€¦  · Web view1AC . Observation 1: If I could find the spot where truth echoesI would stand there and whisper memories of my children's future. I would

of its leading adversaries are increasingly finding common ground. At a time when many foreign investors are leaving Russia, Saudi Arabia is investing billions of dollars into the Russian economy. Moreover, Saudi Arabia and Russia are increasingly coordinating to influence global oil prices. In a few instances, Saudi Arabia has even sided with Russia over the United States. For

example, Riyadh has publicly opposed further U.S. sanctions against Moscow and abstained from several UN votes condemning Russian actions in Ukraine. (RELATED: Saudi Arabia Is Trying To Execute And Possibly Crucify Teen Who Participated In Protest When He Was 10 Years Old: Report) Dmitry Frolovsky, an independent Moscow-based geopolitical analyst, told The Daily Caller that the current level of cooperation between Saudi Arabia and Russia is “unprecedented.” “The convergence with Saudi Arabia over the past several years in many ways is surprising even to Russian strategists because no one expected that Russia’s campaign in Syria could help improve relations with the Persian Gulf monarchies so much,” he said. Riyadh and Moscow have historically had a contentious relationship. As recently as 2013, Saudi Intelligence head Prince Bandar bin Sultan tried to persuade Putin to alter his position on Syria by raising the prospect of terrorism at the Sochi Olympics games, according to a report from The Telegraph. However, the Obama administration’s foreign policy in the Middle East made Saudi Arabia rethink its approach to Moscow. Riyadh was alarmed by the Obama administration’s favorable reception of the Arab Spring, which swept away several prominent pro-American dictators in the region. When the United States signed the Iran nuclear deal in 2015, the kingdom regarded the move as a betrayal. On the other hand, Russia’s military intervention in Syria left a strong impression on Saudi

Arabia. Thus, at a time when Saudi Arabia sought to hedge its bets against Washington, Russia seemed likely a formidable option. In 2016, Saudi Arabia and Russia signed their first deal on limiting oil production to combat falling energy

prices. Saudi King Salman visitedMoscow the subsequent year, becoming the first Saudi monarch to set foot in Russia. Economics is increasingly at the forefront of Saudi-Russia cooperation . While many Western investors shunned Russia after the 2014 annexation of Crimea, Riyadh instead expanded its portfolio in the country. In 2015, Saudi Arabia pledged to invest $10 billion into the Russian

economy. A quarter of that sum has been invested so far. Last October, Saudi Arabia’s sovereign wealth fund became a partner of a Russo-Chinese investment fund and contributed $500 million to the project. That month Saudi Arabia also promised to provide $5 billion for an LNG project in the Arctic by Russian gas company Novatek. Some in Moscow are optimistic that 2019 could be a breakthrough year for Saudi investment in Russia. Kirill Dmitriev CEO of Russia’s sovereign wealth fund, recently told Saudi website Argaam that Russia and Saudi Arabia were exploring investing $2 billion this year on 25 different joint projects. Alexey Khlebnikov, a Middle East expert at the Russian International Affairs Council stated that Saudi investments help provide Russia with relief from Western sanctions. “They do not fully take away the pain [of Western sanctions], they do not fully resolve the problem,” he said. “But it is one of the alternative sources of foreign direct investment into the Russian economy.” Khlebnikov added, “There is big potential, in the future [Saudi investment] is likely to increase because the dynamics are favorable.” However, Yuri Barmin, Middle East and North Africa Director at Moscow Policy Group, offered a more pessimistic assessment about the long-term prospects for further Saudi investment in Russia. “The problem is that the Saudis are very conservative investors and Russia politically is a very risky market to invest in,” he said. Saudi Arabia’s economic interests in Russia have made it wary of U.S. efforts to increase pressure on the country. In March, Saudi Minister of Energy Khalid Al-Falih publicly opposed a new proposed U.S. sanctions package targeting Moscow. “Russia is a big supplier of gas to Europe, oil to China. If [new sanctions] happen, it will affect Europe, China and the whole world,” the minister stated. Saudi Arabia is also increasingly reluctant to criticize Russian foreign policy. In December 2017 and 2018, the kingdom abstained from two UN General Assembly resolutions condemning Russia conduct towards Ukraine.

This move marked a sharp change of policy. As late as 2016, Saudi Arabia had supported similar resolutions. For its part, Russia has defended Saudi Arabia from international criticism over the kingdom’s feud with Canada and the killing of Saudi dissident Jamal Khashoggi. Khlebnikov explained that Riyadh and Moscow have an unspoken understanding to not criticize one-another’s foreign policy. (RELATED: Behind The Growing Rift Between Russia And Iran) “Russia avoids offering commentary and condemnations of Saudi Arabia’s war in Yemen, and Saudi Arabia likewise takes a neutral position towards Russian policy in Ukraine,” he said. Despite the improved relations between Russia and Saudi Arabia, all the Russian experts The Daily Caller spoke to emphasized that the two countries remained far away from a partnership. Saudi Arabia and Russia still have numerous conflicts of interest. Moreover, Riyadh’s alliance with Washington and Moscow’s strong ties with Tehran limit the ability of both countries to become fully-fledged partners. So far, however,

the two countries have managed to draw closer together despite their differences. All the more is impressive is how far this surprising development has fallen under the radar. At a time when Saudi Arabia has successfully pushed Washington to get more involved against Iran, it has also made significant inroads with a top global competitor of the

United States. By upping its Russia investments and striking oil deals with Moscow, Riyadh has helped the Kremlin avoid international isolation and strengthen its economy. Saudi Arabia and Russia may not be headed for a

partnership, but Washington is unlikely to welcome the current trajectory of their relationship.

Page 192: openev.debatecoaches.org€¦  · Web view1AC . Observation 1: If I could find the spot where truth echoesI would stand there and whisper memories of my children's future. I would

Growth key to the environment – tech improvement, link turns case.Daniel Ben-Ami, journalist/author specializing in economics, editor of Fund Strategy, The Independent, 10/20/2010, http://blogs.independent.co.uk/2010/10/20/prosperity-without-growth-a-contradiction-in-terms/

Of course it does not follow from the enormous benefits of growth that the world is perfect. Far from it. But what we need is more growth, rather than less, to overcome the remaining challenges. Take the example of climate change. The conventional green-tinged view is that we need to limit economic growth to stop us destroying the planet. From this perspective the emphasis

must be on individual consumers making sacrifices by consuming less. Such an approach could not be more wrong. To tackle climate change, to the extent it is a problem, we need more resources and better technology rather than less. For example, we can construct more nuclear power stations, build higher sea walls to resist flooding and investigate more high technology solutions. We need an investment in practical engineering-based solutions rather than cheap moralising by politicians and pundits. The end of scarcity is a precondition for human beings to achieve the good life. Economic growth, and social progress more generally, is central to the full realisation of our humanity.

Elitist backlash to transition will kill billions

Lewis, professor of American Studies at University of Colorado, 1998 (Chris, The Coming Age of Scarcity, ed Dobkowski and Wallimann, p 56-57)

Most critics would argue, probably correctly, that instead of allowing underdeveloped countries to withdraw from the global economy and undermine the economies of the developed world, the United States, Europe, and Japan and others will fight neocolonial wars to force these countries to remain within this collapsing global economy. These neocolonial wars will result in mass death, suffering, and even regional nuclear wars. If First

World countries choose military confrontation and political repression to maintain the global economy, then we may see mass death and genocide on a global scale that will make the deaths of World War II pale in comparison . However, these

neocolonial wars, fought to maintain the developed nations’ economic and political hegemony, will cause the

final collapse of our global industrial civilization . These wars will so damage the complex economic and trading networks and squander material, biological, and energy resources that they will undermine the global economy and its ability to support the earth’s 6 to 8 billion people. This would be the worst-case scenario for the collapse of global civilization.

Transition isn’t coming now – a large effort would have to be made

Florea 8 (Adrian, The Limits of Zero Economic Growth Strategy, University of Oradea, Faculty of Economics, http://imtuoradea.ro/auo.fmte/MIE_files/FLOREA%20ADRIAN%201.pdf)

The deep and the comprehensive character of the ecological crisis, but especially the extremely serious consequences and risks that it generates, maintain and amplify, menacing the existence of economy and human society, the overcoming of ecological crisis, or at

least the diminution of its appearances, represents today an absolute priority. As for the achievement of this goal, we consider that it cannot be left to the sudden mechanisms of nature and its laws, and moreover it cannot be left to the spontaneous mechanisms of the market. On the contrary we think that it will be possible to be done only if the actions done in this respect are based on a huge and deeply based strategy . In the following study we will present some of strategies that we consider as the most efficient, strategies that would have real chances of diminution and even elimination of the ecological crisis.

Econ collapse furthers the poverty gap between the races. White people are armed in the squo, they can redistribute wealth with their weapons, thus Econ collapse is anti-black, leads to factioning and dismantles communities of care

Page 193: openev.debatecoaches.org€¦  · Web view1AC . Observation 1: If I could find the spot where truth echoesI would stand there and whisper memories of my children's future. I would

Ehrenreich, author and president of United Professionals, 09 (Barbara, 8/4/09, “The Destruction of the Black Middle Class,” http://www.huffingtonpost.com/barbara-ehrenreich/the-destruction-of-the-bl_b_250828.html, rn)

Left out of the ensuing tangle of commentary on race and class has been the increasing impoverishment -- or, we should say, re-impoverishment -- of African

Americans as a group. In fact, the most salient and lasting effect of the current recession may turn out to be the decimation of the black middle class. According to a study by Demos and the Institute for Assets and Social Policy, 33 percent of the black middle class was already in danger of falling out of the middle class at the start of the recession. Gates and Obama, along with Oprah and Cosby, will no doubt remain in place, but millions of the black equivalents of Officer Crowley -- from factory workers to bank tellers and white collar managers -- are sliding down toward

destitution. For African Americans -- and to a large extent, Latinos -- the recession is over. It occurred between 2000 and 2007, as black employment decreased by 2.4 percent and incomes declined by 2.9 percent. During the seven-year long black recession, one third of black children lived in poverty and black unemployment -- even among college graduates -- consistently ran at about twice the level of white unemployment. That was the black recession. What's happening now is a depression. Black unemployment is now at 14.7 percent, compared to 8.7 for whites. In New York City, black unemployment has been rising four times as fast as that of whites.

Lawrence Mishel, president of the Economic Policy Institute, estimates that 40 percent of African Americans will have experienced unemployment or underemployment by 2010, and this will increase child poverty from one-third of African American children to slightly over half. No one can entirely explain the extraordinary rate of job loss among African Americans, though factors may include the relative concentration of blacks in the hard-hit retail and manufacturing sectors, as well as the lesser seniority of blacks in better-

paying, white collar, positions. But one thing is certain: The longstanding racial "wealth gap" makes African Americans particularly vulnerable to poverty when job loss strikes. In 1998, the net worth of white households on average was $100,700 higher than that of African Americans. By 2007, this gap had increased to $142,600. The Survey of Consumer Finances, which is supported by the Federal Reserve Board,

collects this data every three years -- and every time it has been collected, the racial wealth gap has widened. To put it another way: in 2004, for every dollar of wealth held by the typical white family, the African American family had only one 12 cents. In 2007, it had exactly a dime. So when an African American breadwinner loses a job, there are usually no savings to fall back on, no well-heeled parents to hit up, no retirement accounts to raid.

Attempting dedev ensures extinction

Zey, director of the Expansionary Institute, Professor of Management at Montclair State University 1998 (Michael, Seizing the Future, p. 34, pp. 39-40)

However, no outside force guarantees the continued progress of the human species, nor does anything mandate that the human species must even continue to exist. In fact, history is littered with races and civilizations that have disappeared without a trace. So, too, could the human species. There is no guarantee that the human species will survive even if we posit, as many have, a special purpose to the species’ existence. Therefore, the species innately comprehends that it must engage in purposive actions in order to maintain its level of growth and progress . Humanity’s future is conditioned by what I call the Imperative of Growth, a principle I will herewith describe along with its several corollaries. The Imperative of Growth states that in order to survive, any nation, indeed, the human race, must grow, both materially and intellectually . The Macroindustrial Era represents growth in the areas of both technology and human development, a natural stage in the evolution of the species’ continued extension of its control over itself and its environment. Although 5 billion strong, our continued existence depends on our ability to continue the progress we have been making at higher and higher level s . Systems, whether organizations, societies, or cells, have three basic directions in which to move. They can grow, decline, or temporarily reside in a state of equilibrium. These are the choices. Choosing any alternative to growth, for instance, stabilization of production/consumption through zero-growth policies, could have alarmingly pernicious side effects, including extinction . He Continues. . . The fifth corollary of the Imperative of Growth claims that a society can remain in a state of equilibrium only temporarily. In reality, a society seemingly in a phase where it neither improves nor regresses is actually in a transition to either growth or decline. Such periods easily

Page 194: openev.debatecoaches.org€¦  · Web view1AC . Observation 1: If I could find the spot where truth echoesI would stand there and whisper memories of my children's future. I would

seduce their contemporaries into a false sense of security, that their institutions will last forever, they have all the science they need, and there are no more challenges. In fact, during such periods some imagine that they have reached their “golden age,” perhaps even the “end of history.” During such periods of supposed equilibrium, the population ceases to prepare itself for new challenges and becomes risk averse. Importantly, they reject the idea that growth and progress are necessary for their survival. The sixth corollary evolves from the fifth. If the system chooses not to grow, it will decline and eventually disappear , either because other organisms or systems overtake it or because it is impossible to maintain itself even at static levels without in some way deteriorating. This is the Law of Spiraling Regression. It is indeed a curiosity of the late-twentieth-century culture that this truism has been ignored. In the morass of claims about the risks of technological growth and its impact on the ecosystem, the mainstream media and orthodox a cademics have decided not to consider what harm the full pursuance of zero growth or non growth might inflict on the sociotechnical system, which includes our technological infrastructure, culture, and standard of living.

The worse it is now, the better – the banking sector needs a sustained crisis to cut off failing banks - perception of short-term turnaround only encourages more bubble-inducing speculation

Morris ‘8 (Charles, Practicing Attorney and former banker, 11-22, 2008, “The upside to a serious downturn,” Houston Chronicle, online: http://www.chron.com/disp/story.mpl/editorial/outlook/6126845.html)

The scale of that bubble is reminiscent of the price-inflation bubble that bedeviled President Carter. So are the policies being used to deal with it. Carter and his hapless Fed chairman, G. William Miller, flooded the economy with dollars even as consumer price inflation spiraled out of control . Volcker took over the Fed in 1979 and, by previous standards, moved aggressively his first year in office. But he made little headway. Finally, in late 1980, he cracked down hard and significantly raised interest rates. Unemployment soared from 5.8 percent to 9.7 percent. Inflation stubbornly held on but finally broke in 1983. For the next several years, Volcker continued to crack down at the slightest hint of price buoyancy, until the markets took for granted that the United States was a low-inflation economy. The 2008 analogue to the Volcker strategy would be to force a harsh, fast marking-down of all bank assets to real values. A one- to two-year bloodbath is far preferable to a decade of death by a thousand cuts. Many banks would fail and would have to be re-equitized by the government — the terms should be neither punitive nor excessively generous — but the weakest and the most irresponsible should simply be let go . The banking system that emerges should be dull — one where credit analysis trumps financial engineering and where everything is on the balance sheet. The big Canadian banks , RBC and TD Bank, have been determinedly dull in the 2000s and have turned in superb profits , far outperforming their supposedly brilliant American cousins . Shrinking the banking sector will curtail bubble-style lending and force the share of GDP represented by consumer spending back down from its current 70 percent to a more sustainable 65-66 percent. It will be very painful, putting many companies in jeopardy, but it is the only way to engineer a transition to a world in which we spend less on houses and TVs and more on infrastructure and health care. Interest rates will be higher to encourage savings and taxes will go up, but debt should go down and the bottom half of the population should be more secure. It will also be very important to shore up our tattered social safety net to cushion the recession's impact on that lower half.

3. Perception of financial crisis is key to drive widespread saving – that’s the most important factor in making growth sustainable

Page 195: openev.debatecoaches.org€¦  · Web view1AC . Observation 1: If I could find the spot where truth echoesI would stand there and whisper memories of my children's future. I would

Housel ‘8 (Morgan Housel, Contributor to The Motley Fool, a stock/investment analysis firm, 10-23, 2008, “A Silver Lining to the Financial Crisis,” online: http://www.fool.com/investing/international/2008/10/23/a-silver-lining-to-the-financial-crisis.aspx)

It's hard to find anything to smile about in the economy these days. Portfolios are in tatters. Jobs are being slashed. The market can't find a floor. National debt can't find a ceiling. Inflation, oil, home prices … it's not hard to see why the myriad issues draw comparisons to the Great Depression. Yet beneath the ruckus, one statistic -- perhaps the most vital component of getting back on a sustainable track -- is doing better than it has in years. People are saving money again. And in a big way The Bureau of Economic Analysis reports consumers saved an annualized $297 billion in the second quarter -- more than 10 times higher than the first quarter, and the highest quarterly savings since 1995. From 2004 through 2007, the average rate was around $85 billion. In the third quarter of 2005, things got really out of hand, with a negative savings rate … meaning that as a collective body, living paycheck-to-paycheck meant you were ahead of the pack. Thank goodness those overextended days are gone. It's about time As was the case with oil prices, people don't start changing their ways until the problem is shoved down their throat. It took $4 gasoline to get people to wake up, and to get innovation from companies like Sasol (NYSE: SSL) and electric cars from Toyota (NYSE: TM) and General Motors (NYSE: GM) to start being taken seriously. Necessity is the key to innovation , the proverb goes . The same goes for saving. As Lehman Brothers, AIG, and Bear Stearns will tell you, greed clouded by optimism can be an awesomely overwhelming force. Saving went out of style because, at the rate things were going, few saw a need to. Why save when your house went up by 20% every year? That's all changed in the past few months as people are realizing that Visa (NYSE: V), MasterCard (NYSE: MA), and American Express (NYSE: AXP) will make you richer in your brokerage account than they will in your wallet. As reality sets in for millions of Americans blindsided by home foreclosures and job losses without a safety net to fall back on, people are scrambling to rebuild and replenish their personal balance sheets by saving again. And that problem was yours and yours only Which brings up another point: Can we really put all of the blame for our current woes on Wall Street? Of course, the army of Harvard-stupid Wall Streeters (thanks, Bill) were leading the charge, orchestrating the disaster we're now facing. They screwed up. They should pay the price. And they are. But the larger picture is that it wasn't just Wall Street gorging on more debt than they could handle and taking excessive risks. It was nearly everybody. By 2005, over a quarter of Americans' income went toward revolving debt payments. Debt was the lifeblood of the economy, and the chickens have come to roost. I dug up a few headlines from major newspapers and magazines, all published between 2001-2006, that might remind us how overextended we were. Here are a few: * Saving in 2005 Worst Since 1933 * Equity Shrivels as Homeowners Borrow and Buy * As Personal Savings Fall, a Comeuppance Is Due * The Way We Live Now: Home Sweet Debt * Our Vanishing Savings Rate * Overwhelming Majority of Americans Feel Consumers Are Taking on Too Much Debt So, yeah, maybe it wasn't just Wall Street When AIG (NYSE: AIG) gets in over its head, it's a sin, and they're a criminal. When John Q. Citizen gets in over his head, it's a tragedy, and he's a victim. That blame game is an irony to this debacle that'll likely only prolong our recovery. But enough pointing fingers. For once, let's focus on the good news: People are saving again! How does that help the economy? Countries , like people or business, grow wealthier over time by saving money and investing it wisely . It really is about as simple as that. True, our economy managed to grow over the past decade with little or no savings. How? We borrowed. And that's why it's falling apart today. When an economy saves money, it has the resources it needs to invest without borrowing from someone else (Chinese and OPEC nations), and it creates real wealth . Domestic savings creates a pot of money borrowers can turn to for investments in projects that eventually create jobs, keep interest rates low, and push the economy along at a sustainable pace. The important part is that it's our savings -- not savings

Page 196: openev.debatecoaches.org€¦  · Web view1AC . Observation 1: If I could find the spot where truth echoesI would stand there and whisper memories of my children's future. I would

from foreign nations we became beholden to over the years to sustain our lavish lifestyles. The bottom line is simple: people -- and economies -- that work hard and spend less than they make will grow wealthy. People -- and economies -- that borrow heavily and spend more than they make will grow poor .

Russia is actively looking to fill in – turns case - Russia arms are cheaper, better, and there are no strings attached.Borshchevskaya ’17 - (Anna Borshchevskaya; Senior Fellow at The Washington Institute, focusing on Russia's policy toward the Middle East, PhD candidate; “The Tactical Side of Russia’s Arms Sales to the Middle East.”;

https://jamestown.org/program/tactical-side-russias-arms-sales-middle-east/; December 20th, 2017; HS)

Russia is the world’s top arms exporter, second only to the United States . The Middle East and North Africa (MENA) region has emerged in recent years as Moscow’s second most important arms market after Asia. Moscow has made great strides in this region since Vladimir Putin came to power, and

especially in recent years, after it embarked on major military reform following August 2008. Arms sales matter to the Kremlin because they are a major source of financial gain, but these arms sales are also a tactical foreign policy instrument for wielding influence. Russia’s arms —generally speaking—are well made, sometimes on par with the US, and well suited for the region’s needs. These platforms and armaments are also more affordable than Western weaponry. The US simply will not sell weapons to certain countries, which, therefore, turn to Moscow . Politically, Russian arms come with few strings attached and thus are a great choice when a country wants to diversify away from the West , or at least signal such an intent. Moscow has made inroads with traditional clients such as Iran, Syria and Egypt, but also diversified toward countries closer to the West, such as the Arab Gulf states, Morocco

and Turkey. Russia’s overall influence in the region is growing in the context of Western retreat .

Specifically, we’re on the brink of Russia-Ukraine war – the plan emboldens Putin to act further. There is literally zero evidence of Putin wanting to cooperate. Reuters 12/22Reuters, 12-22-2018, "U.S. to provide Ukraine with extra $10m in military aid after Russia attack," haaretz, https://www.haaretz.com/us-news/u-s-to-provide-ukraine-with-extra-10m-in-military-aid-after-russia-attack-1.6767632

The United States will provide an additional $10 million in military financing to Ukraine to bolster its navy after Russia captured three Ukrainian vessels at sea last month , in a “dangerous escalation , ” the State Department said on Friday. The move comes after Lithuania and the United Kingdom increased their security assistance to Ukraine, the

department said, following the November 25 attack near the Kerch Strait. “The United States calls on Russia to immediately return to Ukraine the seized vessels and detained Ukrainian crews, to keep the Kerch Strait and the Sea of Azov open to ships transiting to and from Ukrainian ports, and to respect Ukraine’s sovereignty and territorial integrity,” the State Department added. Meanwhile,

more than a dozen SU-27 and SU-30 fighter jets which Russia is deploying to boost its air force arrived in Crimea on Saturday. A Reuters witness saw the jets land at Belbek air base in Crimea, which was annexed by Russia in 2014 after Moscow-

leaning Ukrainian President Viktor Yanukovich fled Kiev following street clashes and violent protests. Tensions between Moscow and Kiev have risen in the past weeks after Russia seized three Ukrainian navy ships and their crews

Page 197: openev.debatecoaches.org€¦  · Web view1AC . Observation 1: If I could find the spot where truth echoesI would stand there and whisper memories of my children's future. I would

on Nov. 25 in an incident which Moscow and Kiev have blamed on each other. Russian Foreign Minister Sergei

Lavrov has alleged Ukraine was preparing “a provocation” near Crimea before the end of the year. Russia opened fire on the Ukrainian ships and then seized them and their crews near Crimea - which Russia annexed from Ukraine in March 2014. The Kerch Strait is the only outlet to the Sea of Azov and controls access to two major Ukrainian ports. The incident prompted U.S. President Donald Trump to call off a meeting with Russia’s Vladimir Putin in Argentina to signal Washington’s disapproval of Russian behavior in the naval clash with Ukraine.

OPEC is dead makes their impacts non-unique.Butler 19(Nick Butler, FT energy commentator and chair of The Policy Institute at King’s College London, “The weakness of Opec+ is evident”, 07/08/2019, https://www.ft.com/nick-butler)//ShreyasThe agreement between the oil cartel Opec and a group of non-Opec countries led by Russia to maintain the quotas they established last December for another six to nine months confirms the shift in power within the market away from the producers. In the short term, the defensive deal agreed in Vienna is designed to prop up prices at their current levels. The cartel and its new allies, including Russia, Kazakhstan and Mexico, will continue to take some 1.2m barrels a

day of potential supply off the market. But the immediate market reaction and the further fall in prices in the following days show that the shift in income and wealth away from producers is one that even an agreement by Opec+ — led by Russia and Saudi Arabia — cannot halt. Last week’s formal deal came as no surprise. Nor was it surprising that the once hugely proud and exclusive club Opec agreed to create a permanent alliance with the other countries. The need to cling together was evident to everyone — although some, including the Iranians, remain unhappy about the dominance of Russia and Saudi Arabia in the decision-making process. The exporters are being forced to adapt by the impact on the oil market of the one major player not represented in Vienna last week: the US. American shale production — up from almost nothing 10 years ago to 8.5m b/d in May with a further rise predicted over the next five years — has reshaped the entire global market. Because of that rise, the loss of production from Venezuela, Libya and Iran has been shrugged off. In addition, the US has met most of the gradual increase in global demand seen in the past

few years. What happens next in a market over which Opec has lost control depends on the demand side of the equation. The prospect of a serious trade conflict between the US and Beijing remains, and there are significant signs of a slowing of China’s economy and therefore of its oil import requirements. Since

these are running at around 9.5m b/d, any variation in China’s needs swings the physical market and the surrounding speculation. The most recent oil market outlook from the International Energy Agency predicts global demand growth of 1.2m b/d this year but that figure could be revised downward, as it was last month. If demand remains soft, with prices pushed down as Iran tries to beat US sanctions by offering discounts to buyers, the Opec+ quotas will not go far enough. Saudi Arabia, which has already cut its production by some 600,000 b/d beyond its agreed curbs, will have to do even more. It is tough to see which — if any — of the other producers would be willing or able to cut further by any material amount. The intriguing player in the new pattern of relationships is Russia, which seems to have finally accepted that it is a petro-economy whose interests are aligned with those of the Opec states such as Nigeria and Kuwait. The shift is a sign of President Vladimir Putin’s pragmatism and grasp of the dynamics of the market but also shows Russia’s fundamental economic weakness. Oil and gas account for more than 60 per cent of its export earnings and around 50 per cent of state revenues. The diversification of the economy promised when Mr Putin came to power 20 years ago has never happened. Russia’s vulnerability lies in the fact that, for reasons beyond its control, excess supplies of both oil and gas are undermining prices and therefore reducing the flows of revenue through the state companies. The new alliance between Moscow and Riyadh renews a link established in 1926 when the Soviet Union was the first foreign power to recognise Abdul Aziz al-Saud, grandfather of the current king, as the ruler of the Hejaz and Nejd — the forerunner of the modern kingdom of Saudi Arabia. Relations have not always been smooth, particularly when the Saudis were funding the Afghan mujahideen as they drove out the Soviet army in the 1980s. Today’s relationship is one of necessity, and it will be severely tested if oil

prices drop further. Falling prices mean falling revenues, giving every producer in the new grouping — including Russia — the incentive to break their quotas and produce a little more. Last week’s deal is fragile and cannot disguise the fundamental shift of income and wealth away from the oil exporters. Opec’s glory days are over . The world’s call on the cartel’s oil is down to levels not seen since the early 1980s. Over the past five years, the market’s move from being producer-led to a situation where the risks to prices, even after they have fallen from $110 a barrel in the

early months of 2014 to just above $60 now, are still predominantly on the downside. This could be from weakening demand or, through political change, some restoration of production in Venezuela or Iran. Short of a war or revolution in the Middle East taking out substantial supply for a sustained period, Opec+ will need to at least maintain its quotas far beyond March. As the reaction of the market to last week’s announcement showed, there is deep scepticism about the ability of the producers to hold together for a long defensive fight.

Page 198: openev.debatecoaches.org€¦  · Web view1AC . Observation 1: If I could find the spot where truth echoesI would stand there and whisper memories of my children's future. I would

Marx et al 7 says climate change must “ facilitate this interaction is through group and participatory decision-making.” Means the aff must allow the neg to choose to not do the aff, meaning no perm

Alexander and Rutherford 14 is totally out of context it’s referring to “eco-anarchists” and advocates in the “deep green school” Using this to perm the neg is a violent reductionist flattening of anti-black violence.

Saudi wouldn’t enact countermeasures – relations high.Al Arabiya English 19 (Staff Writer for the Al Arabiya English, “Saudi Ambassador to the United States presents credentials to President Trump”, Al Arabiya, 07/09/2019, http://english.alarabiya.net/en/News/gulf/2019/07/09/Saudi-Ambassador-to-the-United-States-presents-credentials-to-President-Trump.html)//ShreyasSaudi Ambassador to the United States Princess Reema bint Bandar al-Saud has met with US President Donald Trump and presented her credentials as the Kingdom’s new diplomat to Washington. “Presented

credentials today at the White House. Conveyed the well wishes of Kingdom’s Leadership. Looking forward to working on strengthening and solidifying the historic Saudi - US partnership ,” the ambassador tweeted. “The Saudi-US partnership is essential to the interests of both countries and expressed confidence that the two countries are capable of overcoming any challenges at the regional or global levels ,” the embassy quoted the ambassador as saying. Princess Reema bint Bandar al-Saud arrived in Washington DC to begin her duties last week as the ambassador of Saudi Arabia to the United States. She became the 11th Saudi ambassador to the US since 1945 and also the first Saudi woman to hold the post. On her first day, the Saudi ambassador met with the US Assistant Secretary of State for Near Eastern Affairs David Schenker where they discussed

ways to strengthen relations. Princess Reema is the daughter of Prince Bandar bin Sultan , who also served as Saudi Arabia’s ambassador to the US from 1983 to 2005. Prince Bandar was previously the secretary general of Saudi Arabia’s National Security Council and he also served as director general of the General Intelligence Presidency.

Plan gets circumvented – Trump loves Saudi – empirics prove.Billings 19 (Kevin Billings, International Business Times Writer, “US-Saudi Arabia Relations: Pompeo Doesn't Include Saudis On List Of Countries Recruiting Child Soldiers”, International Business Times, 06/18/2019, https://www.ibtimes.com/us-saudi-arabia-relations-pompeo-doesnt-include-saudis-list-countries-recruiting-2801682)//Shreyas

Secretary of State Mike Pompeo blocked Saudi Arabia’s inclusion on a list of countries known to recruit child soldiers, according to a report from Reuters. Pompeo made the decision despite information allegedly provided by security advisors and experts’ findings regarding Saudi Arabia and its connection to the Yemen Civil War. The findings pointed toward the Saudi-backed coalition using underage soldiers in the conflict between the Houthi-led Supreme Political Council and the loyalist Cabinet of Yemen forces. Experts came across this information as they were working on a soon-to-be-released list of countries that are known or suspected to actively recruit and employ child soldiers. Information gathered from news outlets and human rights groups pointed to the Saudi coalition employing child soldiers from Sudan to fight in the war. These findings prompted internal debate between State Department experts and officials as to whether or not to include Saudi Arabia on the list. While experts wanted to include Saudi Arabia, some officials argued that it was not clear whether these were forces under the command of Sudan or were being directed by the Saudi coalition. Pompeo made the final decision that drew immediate blowback from human rights activists. Instead, Sudan will be added back onto the list after being removed in 2018. The Trump administration has previously faced scrutiny for siding with Saudi Arabia despite apparent human rights violations. President Trump faced criticism in the after math

Page 199: openev.debatecoaches.org€¦  · Web view1AC . Observation 1: If I could find the spot where truth echoesI would stand there and whisper memories of my children's future. I would

of Washington Post journalist Jamal Khashoggi's death at the Saudi consulate in Istanbul, as the president appeared to divert attention from possible Saudi involvement. Criticism heightened when the CIA released findings that confirmed Saudi Crown Prince Mohammad bin Salman had ordered the assassination of Khashoggi at the Saudi consulate in Istanbul.

Reducing military aid to Saudi Arabia would cause a flip towards RussiaBradley 12-1, John R. Bradley ( University College London, Dartmouth College and Exeter College, Oxford), 1 December 2018, Coffee House, 12-1-2018, "Has Saudi Arabia just pivoted towards Russia?," https://blogs.spectator.co.uk/2018/12/has-saudi-arabia-just-pivoted-towards-russia/ [mr.ghs]

For all but the most harried journalist motivated by a need to pay off the mortgage, the annual G20 summit – being held this weekend in Buenos Aires – is typically viewed as a perfect cure for insomnia. Who will stand next to whom in the family photo? Will the wording of a final statement be agreed by all leaders before the

official deadline? Yawn yawn yawn. However, there is an exception to every rule. And yesterday’s opening ceremony proved to be just that. First, a hot mic picked up parts of a tense conversation between the French President and Saudi Crown Prince. While hardly a slanging match, it was the most undiplomatic spat between a Western and Saudi leader ever made public. That was followed by the equally unprecedented spectacle of a bro-five between the latter and Russia’s Vladimir Putin. In the space of an hour, we may have witnessed what future history books will say was the moment Saudi Arabia’s relations pivoted from the West towards Russia. The relationship between the democratic West and theocratic Saudi Arabia has often been described as a marriage of convenience. The exchange between President Emmanuel Macron and Crown Prince Mohammed will certainly sound familiar to anyone who has been through a messy but not especially bitter divorce: MbS: Don’t worry. M: I do worry. I am worried, because I am very exp… MbS: He told me. Thank

you. M: I don’t want… MbS: No. M: You never listen to me. MbS: I will listen, of course. M: … MbS: It’s OK. I can deal with it. The official word from the Elysee Palace is that the President was giving the Crown Prince a ‘very firm’ warning over the murder in Istanbul of Saudi dissident Jamal

Khashoggi and the continuing humanitarian crisis in Yemen. In the Western media, the two issues have become inextricably linked. For having failed in their effort to oust the Crown Prince, there is now as a push – led by the Washington Post, which has tripled its coverage of Yemen during the past six weeks – to bring an end to the Yemen conflict as retribution for Khashoggi’s slaughter. And this is especially embarassing for Macron. As recently as April, he pledged France’s ‘full support’ for the Saudi position on Yemen, while slamming the Iran-aligned Houthi rebels the Saudis and their allies are battling against. Predictably, this came on the back of arms agreements worth $18bn. But Macron’s exasperated ‘You never listen to me’ may have been in reference to the Saudi ruler’s decision last year to kidnap Lebanese Prime Minister Saad Hariri during a visit to Riyadh, when he was beaten up and forced to read a resignation letter on Saudi TV. It was only after Macron stepped in that Hariri was finally released, and was subsequently able to resume his role as prime minister. In this way, Macron has been strutting around the world lecturing other countries and leaders how to behave as, his critics argue, a way of diverting attention from his own abysmal

failures at home. Now he is in the unenviable position of being not only the least popular French president in modern history but also aligned most closely abroad with a Saudi ruler who is perhaps the most despised in the world. Putin , of course, is watching all this with relish . He is experienced in the art of bumping off political opponents, and having eradicated almost all traces of meaningful democracy and freedom of expression at home is able to offer the Crown Prince the hand of friendship without fear of reprecussions. He laid the groundwork last year during an official state visit to Moscow by King Salman — the first ever by a reigning monarch. They signed a $3bn arms deal and pledged tens of billions more in bilateral trade. At the time, Russian Foreign Minister Sergei Lavrov described the event as ‘an historical

moment’. Now Saudi Arabia is even negotiating the purchase of Russia’s formiddable S-400 defense missile system, while threatening to cancel arms deals with the Western countires and instead purchase weaponary from Russia and China if sanctions are imposed over the Khashoggi affair . As with Syria and Iran, the Kremlin is demonstrating how, in contrast to the fickle West, it can relied on to remain a steadfast ally to countries in the Middle East it considers friendly, come hell or high water. The partnership between Saudi Arabia and Russia is not going to flourish overnight. But the crucial difference between our political leaders and theirs is that the latter think not in terms of election cycles

Page 200: openev.debatecoaches.org€¦  · Web view1AC . Observation 1: If I could find the spot where truth echoesI would stand there and whisper memories of my children's future. I would

while feeding at the trough, but rather how to forge deep strategic partnerships that will bear fruit decades, even centuries, in the future.

Russia – US presence in the Middle East is zero sum – Russia historically fills the gapByman 18, 4-11-18, Daniel Byman (foreign policy editor of Lawfare. He is a senior fellow at the Center for Middle East Policy at the Brookings Institution, where he focuses on counterterrorism and Middle East security. He is also a professor at Georgetown University's School of Foreign Service.) "Pushing Back Russia in the Middle East: A Thought Experiment," Lawfare, https://www.lawfareblog.com/pushing-back-russia-middle-east-thought-experiment [mr.ghs]

Russia is increasingly emerging as an enemy of the United States , not just a rival. Although President Trump generally seems to oppose any attempt to confront Russia—with the exception of a tweet this morning in which he warned the Kremlin to “get

ready”—it’s worth considering how a more strategically minded administration might do so, particularly in the Middle East, where Moscow has vastly expanded its influence. During the height of the Cold War, the United States reflexively opposed the Soviet Union and the spread of communism. In addition to shoring up allies in Europe and Japan, the United States often sought to hinder or roll back Moscow’s influence in parts of Africa and Asia, regardless of the minor strategic significance of the areas in question. In the Middle East, the U.S. opposition to the Soviet Union often manifested in efforts to sway and topple governments in Iraq and Syria and a competition for influence in Egypt, among other locations. At the Cold War’s end, the Soviet Union maintained some interest in and influence over Algeria, Iraq, Libya, Syria and South Yemen (the last of which would soon enter history’s dustbin). Yet for the most part, the United States had run the board, with close partnerships with key states such as Saudi Arabia, Jordan, Morocco, and Egypt as well as Israel and Turkey, the region’s military powerhouses. These close relationships continued after the Cold War’s end, and the United States even improved relations with several of Moscow’s former clients. The dominant U.S. position in the Middle East slipped under President Obama. Skeptical of intervention in the Middle East and unsympathetic to long-standing allies like Saudi Arabia, the Obama administration tried to keep its distance

from the region. Although the United States intervened in response to the Islamic State’s atrocities against the Yazidis and the broader concerns about terrorism, the Obama team still tried to avoid greater entanglement in Syria, distanced itself from Egypt after the 2013 coup, soured on Netanyahu’s government in Israel and limited efforts to deal with trouble spots like Libya and Yemen to a narrow counterterrorism mission. Even the administration’s successes, like the Iran deal, seemed to alienate many traditional American

friends. U.S. credibility in the region fell , and many leaders were glad to see Obama go. Moscow entered into the void created by the decline in the U.S. position . Russia backed its Syrian ally when Syrian President Bashar al-Assad was on the ropes, helping Assad’s government survive and slowly turning the tide against an opposition that Washington nominally championed. Moscow delivered for its allies and claimed victory as the Islamic State receded and the Syrian state recaptured much of its lost, though now devastated, territory. Moscow has also made inroads with former U.S. stalwarts : President Vladimir Putin recently visited Turkey, probably to coordinate the crushing of the Syrian Kurds who were the tip of the U.S. spear against the Islamic State. Russia also offered to sell Turkey advanced weapons. King Salman of Saudi Arabia, whose country had long scorned Russia, traveled to Moscow last October to talk

about shoring up the price of oil. Ideologically, Russia also fits in well with the Middle East. Putin, an autocrat himself , is comfortable with dictators and resents what he perceives as U .S. democracy promotion —a position that dictators in the Middle East also share. As Steven Cook notes, “Now it’s payback time for almost three decades of Moscow’s

humiliation. And what better place to start than the Middle East , where the United States is already widely resented even among its allies.”

Russian foreign policy exacerbates and retrenches authoritarianism in the Middle EastBarmin 18, Yuri Barmin (expert at the Russian International Affairs Council, covering the Middle East and North Africa, Moscow’s policy towards the region, as well as the conflicts in Syria and Libya. He regularly contributes his analysis to Al Monitor, the Middle East Institute, Al Sharq Forum and FARAS Center. Mr. Barmin holds an MPhil in International Relations from the University of Cambridge.), 3-8-2018, "Russia in the Middle East Until 2024: From Hard Power to Sustainable Influence," Jamestown, https://jamestown.org/program/russia-middle-east-2024-hard-power-sustainable-influence/ [mr.ghs]

Experts who had argued that authoritarianism in the Middle East would maintain stability and keep extremism at bay were proven wrong by the events of the Arab Spring .[xxxv] The Russian leadership , however, still projects its vision of “ autocratic stability ” onto the region . And even though Moscow repeatedly

Page 201: openev.debatecoaches.org€¦  · Web view1AC . Observation 1: If I could find the spot where truth echoesI would stand there and whisper memories of my children's future. I would

insists that it is up to the Syrian people to decide through a presidential election who will lead the nation into the post-war period, the

Russian government is unlikely to become a supporter of democracy movements in the Middle East. After all, elections have been a crucial legitimization tool of Russia’s own “managed democracy.” The consolidation of power in

the hands of the national leader as well as the securitization of the political agenda have characterized the Russian political system throughout the last 17 years Vladimir Putin has been in power. And they continue to guide him in how he sees regimes in the Middle East . Some of these authoritarian Arab regimes share a long history with Russia: during the Cold War, they proved their ability to maintain order for longer than any democratic regime could sustain it,

not least due to Moscow’s financial and military support. The fact that Bashar al-Assad survived throughout the bloody Syrian conflict , to a large extent due to Russia’s aid, solidifies the idea that authoritarianism in the Middle East guarantees stability and puts a cap on “toxic” democratic values imposed from the outside . In Moscow ’s view, authoritarian tendencies are indigenous to the region , much like they are to Russia, which is why they need not be battled but rather be correctly managed . Russia’s idea of “authoritarian stability” in the Middle East may find a potential supporter in Donald Trump, who notoriously dropped America’s agenda for promoting democracy in the

Middle East. The distinct security focus of Donald Trump’s strategy toward the region has emboldened his allies, Saudi Arabia and Israel, and convinced them that the regional policeman will no longer restrain their geopolitical ambitions. The position of both Russia and the United States is, thus, likely to resonate with many governments in the region that previously had to put on airs of civil society engagement and liberalization just to have international political and diplomatic backing. Egypt and Turkey are the two cases in point: the 2017 Human Rights Watch World Report specifically points to them to illustrate how the tide of new authoritarianism is sweeping through the Middle East.[xxxvi] In Turkey, the attempted coup in July 2016 was used by President Recep Tayyip Erdoğan and his Justice and Development Party (AKP) as an excuse to crack down not only on suspected plotters but also on wider circles critical of the government’s

policies. Western powers sharply rebuked Erdoğan over his suspension of the rule of law in the country and mass detentions—but Russia pointedly did not. Putin was the one world leader who gave a call to Erdoğan to tell him Moscow supports his campaign to root out dissent, which the Turkish president described as “anti-constitutional .”[xxxvii] Furthermore Putin hosted his Turkish counterpart in St. Petersburg less than a month after the failed coup, during which Erdoğan explained that Vladimir Putin’s call to him was an important move, “a kind of moral support and display of Russia-Turkey solidarity,” as the Turkish president described the situation.[xxxviii] All this occurred just weeks after Erdoğan’s late

June apology to Russia for the November 2015 downing of a Russian Su-24 jet over Syria; and it goes to show how masterfully

Vladimir Putin uses authoritarian movements to his own political benefit. Egypt is going through a similar wave of authoritarianism, with President Abdel Fattah El Sisi cracking down on dissent that is not necessarily associated with the Muslim Brotherhood. That government campaign is happening against the backdrop of economic instability, currency devaluation and increased poverty rates. However, the army’s grip on power and full control over the public sphere give a semblance of stability in the country. Sisi’s fight to eradicate extremism in the Sinai as well as his crackdown on dissent find support in Moscow, which is reflected in official statements coming from the Kremlin. Egypt reemerged as Russia’s key partner in the Middle East, including in crucial spheres of military-technical cooperation. The two countries signed a protocol on military cooperation in March 2015, significantly ramped up joint military exercises, and are looking to green light an agreement that would allow Russian military aircraft to use Egyptian airspace and infrastructure.[xxxix] With the turmoil and regular attacks in the Sinai Peninsula, counter-terrorism cooperation has become a distinct characteristic of the bilateral relationship. A security-heavy agenda acts as a glue between Moscow and Cairo, not least due to the military and security background of the political elites of the two countries. Both Russians and Egyptians will head to the polls in March 2018 to elect their respective heads of state, while presidential elections in Turkey are to take place in November 2019. The outcome is already known in all three countries; Putin, Sisi and Erdoğan will almost certainly serve out their next terms into the first half of the 2020s, meaning that we are unlikely to witness a disruption in the security-comes-first policy

employed by Moscow in its bilateral relations with both Cairo and Ankara. The cases of Egypt and Turkey illustrate that Vladimir Putin is likely to encourage authoritarian “stability ” across the region through skewed security-heavy policies . Putin’s support for autocratic tendencies will hardly find any resistan ce among other powers in the region and will almost certainly be embraced. Syria’s recovery from the seven-year war is unlikely to happen through the emergence of democratic institutions and freedom, but will probably lead to the creation of a strong regime with an inflated security appara tus to shield a fragile government and keep extremist tendencies at bay . Iraq’s increasingly sectarian policies hint at a similar trend . And as Libya’s internationally recognized government fails to establish control over much of the country’s territory, Libyan National Army Field Marshall Khalifa Haftar represents the type of leader the

Page 202: openev.debatecoaches.org€¦  · Web view1AC . Observation 1: If I could find the spot where truth echoesI would stand there and whisper memories of my children's future. I would

Kremlin would presumably like to see for a post–civil war Libya. If Russia’s Syria policy is any indication, a highly centralized system will be Moscow’s remedy for extremism throughout the wider region . The fear of a new wave of extremism will push many regimes to seek more control over the population, and a lack of incentives to democratize may bring about new repressive regimes. In other words Russia’s leadership in the Middle East may significantly lengthen and reinforce the era of authoritarianism there.

The form of the aff is emblematic of the exact form of representational violence that they critique. Their call for the ballot in the face of violence is morphed by late capitalism into a form of virtue signaling that makes coalitionary politics impossible.ANON 17(#Alt-Woke Manifesto is the work of ANON. We are a collective of “Other.” Some of us are sex workers, some immigrants, many of us queer. There are even a few privileged white cucks amongst us. Nevertheless, ANON is largely the work and brainchild of People of Color (PoC). Our social disciplines are as varied as our identities, from journalists to dominatrixes. ANON are the intellectual cousins of #BlackLivesMatter divorced from liberalism http://tripleampersand.org/alt-woke-manifesto/ cVs)

The moderate Left misappropriated theoretical terms and concepts, divorced from any actual theory. Identity politics, despite its origins in academia, flourishes best on social media — it’s the most accessible concept for moderate liberals to grasp. “Well, if identity is

only a game, if it is only a procedure to have relations, social and sexual-pleasure relationships that create new friendships, it is useful. But if identity becomes the problem of sexual existence , and if people think that they have to ‘uncover’ their ‘own identity,’ and that their own identity has to become the law , the principle, the code of their existence; if the perennial question they ask is ‘Does this thing conform to my identity? ’ then, I think, they will turn back to a kind of ethics very close to the old heterosexual virility. If we are asked to relate to the question of identity, it must be an identity to our unique selves. But the relationships we have to have with ourselves are not ones of identity, rather, they must be relationships of differentiation, of creation, of innovation. To be the same is really boring. We must not exclude identity if people find their pleasure through this identity, but we must not think of this

identity as an ethical universal rule.” — Michel Foucault, “Sex, Power, and the Politics of Identity” (1984) Identity politics became an albatross, however. Both the moderate and radical were too eager to evangelize oppressed identities. There was no room for discussion, no place for debate. Call outs, clap backs , and other reality tv patois replaced dialectics . Representation is the de facto litmus of society’s progress for the moderate liberal — society appeared more inclusive and diverse because “Orange is the New Black” has a female lead and a multiethnic supporting cast. They inhabit a never ending, curated echo chamber of think pieces, listicles, notifications, and retweets. Everyone within their algorithmic ghetto shares their sentiments about society. The algorithm makes their small corner seem far more vast than it actually is, and as a result, the moderate extends this myopia to society at large. The moderate midwifed the birth of the Alt-Right through bipartisan compromises. Moderate liberals are basically content to vest trust in their vaunted Democratic Party as it slides further to the right, thereby underpinning a level of discourse friendly to the far-right. It’s worth remembering that the end of the 20th and beginning of the 21st centuries were a period of diehard cooperation between liberals and conservatives in crafting today’s authoritarianism. Neoconservatism provided socio-political planning that complemented a neoliberal economic agenda. This is why the radical Left blames

liberals as well as conservatives for “command and control policing”, mass surveillance and this century’s rationale for endless warfare. Moderate liberals provided and adopted theoretical frameworks that explained away structural oppression but retained an appearance of caring about racism and equality across intersecting spectrums of gender and sexuality. This was an obvious farce that mystified progress and the far right took advantage of this because they actually suffered no serious political setbacks. Liberalism provided an incubator for the alt right to form by mollifying actual demands for change . “If politics without passion leads to cold-hearted, bureaucratic technocracy,

Page 203: openev.debatecoaches.org€¦  · Web view1AC . Observation 1: If I could find the spot where truth echoesI would stand there and whisper memories of my children's future. I would

then passion bereft of analysis risks becoming a libidinally driven surrogate for effective action. Politics comes to be about feeling of personal empowerment, masking an absence of strategic gains . ” — Nick Srnicek and Alex Williams, “Inventing the Future” (2015)

Music becomes commodified by the nation-state to promote democracy and capitalDamodaran, 16 (Sumangala, PhD in Economics, Jawaharlal Nehru University, “Protest and Music”, Aug 2016, http://politics.oxfordre.com/view/10.1093/acrefore/9780190228637.001.0001/acrefore-9780190228637-e-81, Accessed: 7/8/18, JY)

Despite the recognition of the social importance of music, and that social structures and developments can be be reflected in varied ways in musical structures, music has not been given a fair hearing in the social sciences. If little systematic investigation into the relation of music to culture or society as a whole has been made, even less effort has been expended on understanding the relationship between music and politics. Music has represented a mode of expression for human beings’ interaction with their surroundings, making it a spontaneous medium for expressing their discontent with it as well. This discontent has, in varied historical and geographical contexts, been expressed through words, without words, through appraisal, evaluation, and often rejection of certain canonical forms, and through creation of

new forms. Further, structures of authority have used music as a medium or mode for transmitting political information and values, mobilizing the population, evoking and sustaining pride and identity, and legitimizing patterns of authority. Despite there being a long history of the connection between music and politics, in the role of music as a medium or an instrument of political communication, in critiquing existing social contexts and norms or in expressing protest against those norms, the corpus of work on music and politics has been scant. It is necessary to make a qualification here. This point, about the corpus of work on music and politics not being substantial, is being made for politics and protest as manifest through collective political movements and the music of protest

that arises out of such political mobilizations. Political theory and political philosophy have engaged with the politics of sound and auditory regimes in capitalism, as well as the politics of the emergence of the “visual” against the “auditory” as the dominant sensorial register that acquires prominence, and through which power gets established and played out in society, particularly under capitalism (Attali, 1977; Bull & Back, 2003; Siisiainen, 2012). French theory, through the work of thinkers such as Ranciere (2006) and Nancy (2007), to name two, has been concerned with the relationship of music to society, the place of listening with philosophies and cultures of listening, and the place of the sonorous in being and experience.

Page 204: openev.debatecoaches.org€¦  · Web view1AC . Observation 1: If I could find the spot where truth echoesI would stand there and whisper memories of my children's future. I would

Music link - music labels are inherently capitalist, their performance gets consumed by white America in an attempt to become black, the industry is built off of capitalism and the black body becomes just another commoditiy that can be replaced as soon as there is the next big thing or the next black person that they can sell to the world as another piece of media to be consumed, the music industry takes over any potential solvency, their performance becomes coopted by white capitalist society and molded into the white definition of blackness that reproduces their impacts - this independently turns case because black people will always just become the sterotypical thug whose in a gang or smokes weed, their performance is rooted in capitalist exploitation of black bodies to make a profit, white America will consume their performance through the capitalist consumption of everything that is produced in this structure , picking apart the parts of black culture that are aesthetically pleasing and selling it to white America - this turns case and disproves any perm because they are using capitalism to achieve any form of self care which ultimately coopts their movement by directly feeding into capitalism which will inev. Produce anti black violence onto black people.

Performance fails – it traps resistance at the individual level and cedes the power of surveillance to recognition – this locks in a universalized, neoliberal subject at the center of their movements – only anti-capitalist organizing solvesMonahan 15(Torin Monahan is associate professor in the Department of Human and Organizational Development at Vanderbilt University. “The Right to Hide? Anti-Surveillance Camouflage and the Aestheticization of Resistance” Communication and Critical/Cultural Studies Vol. 12, No. 2, June 2015 http://publicsurveillance.com/papers/Right-to-hide.pdf rvs)

The aestheticization of resistance enacted by anti-surveillance camouflage and fashion ultimately fails to address the exclusionary logics of contemporary state and corporate surveillance. These anti-surveillance practices emerge at this historical juncture because of a widespread recognition of unchecked, pervasive surveillance and popular criticism of government and

corporate overreach. The key to the popularity of these artistic efforts may be that they mobilize the trappings of radical intervention , in highly stylized form, but do so in ways that do not compel people to challenge state visuality projects. They offer hyper-individualized and consumer-oriented adaptations to undesired surveillance. To the extent that such efforts can be seen as critical interventions, they rely on an appeal to the pedagogical potential of art to galvanize meaningful political change. As Jacques Rancière explains: Art is presumed to be effective politically because it displays the marks of domination, or parodies mainstream icons, or even because it leaves the spaces reserved for it and becomes a social practice. . . . The logic of mimesis consists in conferring on the artwork the power of the effects that

it is supposed to elicit on the behavior of spectators.65 In the case of the examples covered in this paper, it is clear that while some of the signifiers of critical art are present, for instance with the Fag Face Mask’s blurring of institutionally imposed identities, the primary message is nonetheless one of accommodating pervasive surveillance and inviting a playful dance with it. Recognition of the violent , unequal, and marginalizing applications of surveillance is bracketed or denied in the presentation of universal, neoliberal subjects in search of a modicum of (fashionable) control over their exposure. This is not to say that play has no place in resistance efforts. As Jeffrey Juris66 has

Page 205: openev.debatecoaches.org€¦  · Web view1AC . Observation 1: If I could find the spot where truth echoesI would stand there and whisper memories of my children's future. I would

illustrated in his ethnography of the anti-corporate globalization movement, play and frivolity can sometimes succeed in ways that oppositional tactics cannot. For example, in spaces of confrontation, people playing music or staging performances while dressed in elaborate costumes are effective because they are symbolically powerful solidarity-building activities that are not physically threatening to the police. Juris writes, “Such playful provocation represents a form of ritual opposition, a symbolic overturning of hierarchy much like medieval carnival…. Play, in particular, reveals the possibility of radically reorganizing current social arrangements.” 67 In the mode of anti-surveillance, groups like the Surveillance Camera Players similarly embody a spirit of play as they stage performances for video surveillance camera operators and spectators in public places like New York City subway stations.68 Perhaps because of the public setting of these performances, which usually end with police or security guards escorting players off public property, these interventions may have the effect of fostering in audiences a critique of policing priorities and the commodification of public space. Play of this sort may be an effective form of resistance that alters public awareness and cultural sensibilities, but it can be a difficult task for such interventions to problematize inequalities that can fester within

assumptions of shared rights. Ultimately, discourses of “the right to hide” are weak variations of “the right to privacy,” both of which depend on conceptually inadequate and empirically deficient mobilizations of universal rights. Indeed, poor and racialized populations subjected to the most invasive forms of monitoring are much more concerned with issues of domination and control, along with the practicalities of survival, than they are with legal or philosophical abstractions like privacy.69 Privacy is also a deeply individualistic concept, poorly suited to forestall discriminatory practices against social groups.70 As Sami Coll explains, “The notion of privacy, as a critique of [the] information society, has been assimilated and reshaped by and

in favour of informational capitalism, notably by being over-individualized through the self-determination principle.” 71 The discourse of the right to hide , as with the right to privacy, accepts the legitimacy of state demands for legible populations and offers symbolic compromises to assert degrees of freedom within those constraints. Instead of being content with artistic forms of hiding, countervisuality projects , by contrast, would “look back” and pursue alternatives to totalizing regimes of state visuality . They would seek to undermine the authority of state control by challenging the capitalist imperatives that lend legitimacy to forms of state violence and oppression.72 What is required is a full engagement with “the political,” which, as Rancière describes, is always in opposition to the police : The police is not a social function but a symbolic constitution of the social. The essence of the

police lies neither in repression nor even in control over the living. Its essence lies in a certain way of dividing up the sensible. . . . Politics, by contrast, consists in transforming this space of “move-along,” of circulation, into a space for the appearance of a subject: the people, the workers, the citizens.73

The aff’s strategy cedes the universal in favor of local, fragmented knowledge – this surrenders the ability to define the future to neoliberal hegemony – the universal is not inherently-oppressive, but it will be under unfettered capitalism

Hester 17(Helen Hester is Associate Professor of Media and Communication at the University of West London. Her research interests include technofeminism, sexuality studies, and theories of social reproduction. She is a member of the international feminist collective Laboria Cuboniks. “Promethean Labors and Domestic Realism” 25 September 2017 HYPERLINK "http://www.e-flux.com/architecture/artificial-labor/140680/promethean-labors-and-domestic-realism/"http://www.e-flux.com/architecture/artificial-labor/140680/promethean-labors-and-domestic-realism/ rvs)

There has been an excess of modesty in the feminist agendas of recent decades. Carol A. Stabile is amongst those who have been critical of an absence of systemic thinking within postmodern feminisms, remarking upon a “growing emphasis on fragmentations and single-issue politics.”1 Stabile dismisses this kind of thinking which, in “so resolutely avoiding ‘totalizing’—the bête noire of contemporary critical theory—[…] ignores or jettisons a structural analysis of capitalism.”2 The difference in scope and scale between that which is being opposed and the strategies being used to oppose it is generative of a

Page 206: openev.debatecoaches.org€¦  · Web view1AC . Observation 1: If I could find the spot where truth echoesI would stand there and whisper memories of my children's future. I would

sense of disempowerment. On the one hand, Stabile argues, postmodern social theorists “accept the systemic nature of capitalism, as made visible in its consolidation of power and its global expansion […] Capitalism’s power as a system is therefore identified and named as a totality”; on the other hand, these theorists “celebrate local, fragmented, or partial forms of knowledge as the only forms of knowledge available” and criticize big-picture speculative thinking for its potentially oppressive tendencies or applications.3 Nancy Fraser, too, has addressed this apparent “shrinking of emancipatory vision at the fin de siècle,” linking this with “a major shift in the feminist imaginary” during the 1980s and 1990s—that is, with a move away from attempting to remake political economy (redistribution) and towards an effort at transforming culture (recognition).4 The legacies of this kind of political theorizing—legacies some might describe as “folk political”—are still being felt today, and continue to shape the perceived horizons of possibility for progressive projects.5 Yet these projects, which are frequently valuable, necessary, and effective on their own terms, are not sufficient as ends in themselves. To the extent that they are conceptualized in detachment from an ecology of other interventions, operating via a diversity of means and across a variety of scales, they cannot serve as a suitable basis for any politics seeking to contest the imaginaries of the right or to contend with the expansive hegemonic project of neoliberal capitalism. It is for this reason that Nick Srnicek and Alex Williams’s work positions itself as somewhat skeptical about fragmentations and single-issue politics, pointing out that problems such as “global exploitation, planetary climate change, rising surplus populations, [and] the repeated crises of capitalism are abstract in appearance, complex in structure, and non-localized.”6 As such, a politics based around the ideas that “the local is ethical, simpler is better, the organic is healthy, permanence is oppressive, and progress is over” is not always the best weapon in an attempt to contend with the complex technomaterial conditions of the world as it stands.7 There is a persistent kind of abstraction anxiety hanging over progressive politics; an anxiety that haunts a contemporary leftist feminism still unwilling or unable to critically reappraise the tendencies that Stabile identified in the 90s. Recently, however, a renewed appetite for ambitious and future-oriented emancipatory politics has begun to make itself felt at the fringes of the left—and indeed, to gather momentum and popular support more broadly.8 Perhaps the most remarkable example of this tendency within philosophically-inflected political theory circles has been accelerationism, with its calls to build an “intellectual infrastructure” capable of “creating a new ideology, economic and social models, and a vision of the good to replace and surpass the emaciated ideals that rule our world today.”9 These so-called “Promethean” ideas have generated widespread interest, arguably both reflecting and contributing to the changing tenor of activist discourse. Interestingly, this term has to some extent emerged in opposition to the pejorative “folk political,” acting as a shorthand for a very different set of values and perspectives. In a recent critical piece, Alexander Galloway suggests that “Prometheanism” could be defined as “technology for humans to overcome natural limit.”10 Peter Wolfendale, meanwhile, sees it as a “politics of intervention”—one that starts from the insistence that nothing be exempted in advance from the enactment of re/visionary processes.11

We have an ethical-political obligation to reject capitalism – the organizing principle of society that necessitates global exclusion and violence which cannot be calculatedZizek and Daly 14

Page 207: openev.debatecoaches.org€¦  · Web view1AC . Observation 1: If I could find the spot where truth echoesI would stand there and whisper memories of my children's future. I would

(Slavoj, senior researcher at the Institute for Sociology and Philosophy at the University of Ljubljana; AND, Glyn, Associate Professor of Sociology, Economics, International Development, International Relations, and Politics at the University of Northamption, “Conversations with Zizek,” Republished by Polity Press)

For Zizek it is imperative that we cut through this Gordian knot of postmodern protocol and recognize that our ethico-political responsibility is to confront the constitutive violence of today’s global capitalism and its obscene naturalization / anonymization of the millions who are subjugated by it throughout the world. Against the

standardized positions of postmodern culture – with all its pieties concerning ‘multiculturalist’ etiquette – Zizek is arguing for a politics that might be called ‘radically incorrect’ in the sense that it break with these types of positions 7 and focuses instead on the very organizing principles of today’s social reality : the principles of global liberal capitalism. This requires some care and subtlety. For far too long, Marxism has been bedeviled by an almost fetishistic economism that has tended towards political morbidity. With the likes of Hilferding and Gramsci, and more recently Laclau and Mouffee, crucial theoretical advances have been made that enable the transcendence of all forms of economism. In this new context, however, Zizek argues that the problem that now presents itself is almost that of the opposite fetish. That is to say, the prohibitive anxieties surrounding the taboo of economism can function as a way of not engaging with economic reality and as a way of implicitly accepting the latter as a basic horizon of existence. In an ironic Freudian-Lacanian twist, the fear of economism can end up reinforcing a de facto economic necessity in respect of contemporary capitalism (i.e. the initial prohibition conjures up the very thing it fears). This is not to endorse any kind of retrograde return to economism. Zizek’s point is rather that in rejecting economism we should not lose sight of the systemic power of capital in shaping the lives and destinies of humanity and our very sense of the possible. In

particular we should not overlook Marx’s central insight that in order to create a universal global system the forces of capitalism seek to conceal the politico-discursive violence of its construction through a kind of gentrification of that system . What is persistently denied by neo-liberals such as Rorty (1989) and Fukuyama (1992) is that the gentrification of global liberal capitalism is one whose ‘universalism’ fundamentally reproduces and depends upon a disavowed violence that excludes vast sectors of the world’s populations . In this way, neoliberal ideology attempts to naturalize capitalism by presenting its outcomes of winning and losing as if they were simply a matter of chance and sound judgment in a neutral market place. Capitalism does indeed create a space for a certain diversity, at least for the central capitalist regions, but it is neither neutral nor ideal and its price in terms of social exclusion is exorbitant. That

is to say, the human cost in terms of inherent global poverty and degraded ‘life -chances’ cannot be calculated within the existing economic rationale and , in consequence, social exclusion remains mystified and nameless (viz. the patronizing reference to the ‘developing world’). And Zizek’s point is that this mystification is magnified through capitalism’s profound capacity to ingest its own excesses and negativity: to redirect (or misdirect)

social antagonisms and to absorb them within a culture of differential affirmation. Instead of Bolshevism, the tendency today is towards a kind of political boutiquism that is readily sustained by postmodern forms of consumerism and lifestyle. Against

this Zizek argues for a new universalism whose primary ethical directive is to confront the fact that our forms of social existence are founded on exclusion on a global scale . While it is perfectly true that universalism can

never become Universal (it will always require a hegemonic-particular embodiment in order to have any meaning), what is novel about Zizek’s universalism is that it would not attempt to conceal this fact or reduce the status of the abject Other to that of a ‘glitch’ in an otherwise sound matrix.

The alternative is to affirm the model of the Communist Party – only the Party can provide effective accountability mechanisms to correct chauvinist tendencies, educate and mobilize marginalized communities, and connect local struggles to a movement for international liberation

Escalante 18(Alyson Escalante is a Marxist-Leninist, Materialist Feminist and Anti-Imperialist activist.

Page 208: openev.debatecoaches.org€¦  · Web view1AC . Observation 1: If I could find the spot where truth echoesI would stand there and whisper memories of my children's future. I would

“PARTY ORGANIZING IN THE 21ST CENTURY” September 21st, 2018 HYPERLINK "https://theforgenews.org/2018/09/21/party-organizing-in-the-21st-century/"https://theforgenews.org/2018/09/21/party-organizing-in-the-21st-century/ cVs)

I would argue that within the base building movement, there is a move towards party organizing, but this trend has not always been explicitly theorized or forwarded within the movement. My goal in this essay is to argue that base building and dual power strategy can be best forwarded through party organizing, and that party organizing can allow this emerging movement to solidify into a powerful revolutionary socialist tendency in the United States. One of the crucial insights of the base building movement is that the current state of the left in the United States is one in which revolution is not currently possible. There exists very little popular support for socialist politics. A century of anticommunist propaganda has been extremely effective in convincing even the most oppressed and marginalized that communism has nothing to offer them. The base building emphasis on dual power responds directly to this insight. By building institutions which can meet people’s needs, we are able to concretely demonstrate that communists can offer the oppressed relief from the horrific conditions of capitalism. Base building strategy recognizes that actually doing the work to serve the people does infinitely more to create a socialist base of popular support than electing democratic socialist candidates or holding endless political education classes can ever hope to do. Dual power is about proving that we have something to offer the oppressed. The question, of course, remains: once we have built a base of popular support, what do we do next? If it turns out that establishing socialist institutions to meet people’s needs does in fact create sympathy towards the cause of communism, how can we mobilize that base? Put simply: in order to mobilize the base which base builders hope to create, we need to have already done the work of building a communist party. It is not enough to simply meet peoples needs. Rather, we must build the institutions of dual power in the name of communism. We must refuse covert front organizing and instead have a public face as a communist party. When we build tenants unions, serve the people programs, and other dual power projects, we must make it clear that we are organizing as communists, unified around a party, and are not content simply with establishing endless dual power organizations. We must be clear that our strategy is revolutionary and in order to make this clear we must adopt party organizing. By “party organizing” I mean an organizational strategy which adopts the party model. Such organizing focuses on building a party whose membership is formally unified around a party line determined by democratic centralist decision making. The party model creates internal methods for holding party members accountable, unifying party member action around democratically determined goals, and for educating party members in communist theory and praxis. A communist organization utilizing the party model works to build dual power institutions while simultaneously educating the communities they hope to serve. Organizations which adopt the party model focus on propagandizing around the need for revolutionary socialism. They function as the forefront of political organizing, empowering local communities to theorize their liberation through communist theory while organizing communities to literally fight for their liberation. A party is not simply a group of individuals doing work together, but is a formal organization unified in its fight against capitalism. Party organizing has much to offer the base building movement. By working in a unified party, base builders can ensure that local struggles are tied to and informed by a unified national and international strategy. While the most horrific manifestations of capitalism take on particular and unique form at the local level, we need to remember that our struggle is against a material base which functions not only at the national but at the international level. The formal structures provided by a democratic centralist party model allow individual locals to have a voice in open debate, but also allow

Page 209: openev.debatecoaches.org€¦  · Web view1AC . Observation 1: If I could find the spot where truth echoesI would stand there and whisper memories of my children's future. I would

for a unified strategy to emerge from democratic consensus. Furthermore, party organizing allows for local organizations and individual organizers to be held accountable for their actions. It allows criticism to function not as one independent group criticizing another independent group, but rather as comrades with a formal organizational unity working together to sharpen each others strategies and to help correct chauvinist ideas and actions. In the context of the socialist movement within the United States, such accountability is crucial. As a movement which operates within a settler colonial society, imperialist and colonial ideal frequently infect leftist organizing. Creating formal unity and party procedure for dealing with and correcting these ideas allows us to address these consistent problems within American socialist organizing. Having a formal party which unifies the various dual power projects being undertaken at the local level also allows for base builders to not simply meet peoples needs, but to pull them into the membership of the party as organizers themselves. The party model creates a means for sustained growth to occur by unifying organizers in a manner that allows for skills, strategies, and ideas to be shared with newer organizers. It also allows community members who have been served by dual power projects to take an active role in organizing by becoming party members and participating in the continued growth of base building strategy. It ensures that there are formal processes for educating communities in communist theory and praxis, and also enables them to act and organize in accordance with their own local conditions. We also must recognize that the current state of the base building movement precludes the possibility of such a national unified party in the present moment. Since base building strategy is being undertaken in a number of already established organizations, it is not likely that base builders would abandon these organizations in favor of founding a unified party. Additionally, it would not be strategic to immediately undertake such complete unification because it would mean abandoning the organizational contexts in which concrete gains are already being made and in which growth is currently occurring. What is important for base builders to focus on in the current moment is building dual power on a local level alongside building a national movement. This means aspiring towards the possibility of a unified party, while pursuing continued local growth. The movement within the Marxist Center network towards some form of unification is positive step in the right direction. The independent party emphasis within the Refoundation caucus should also be recognized as a positive approach. It is important for base builders to continue to explore the possibility of unification, and to maintain unification through a party model as a long term goal. In the meantime, individual base building organizations ought to adopt party models for their local organizing. Local organizations ought to be building dual power alongside recruitment into their organizations, education of community members in communist theory and praxis, and the establishment of armed and militant party cadres capable of defending dual power institutions from state terror. Dual power institutions must be unified openly and transparently around these organizations in order for them to operate as more than “red charities.” Serving the people means meeting their material needs while also educating and propagandizing. It means radicalizing, recruiting, and organizing. The party model remains the most useful method for achieving these ends. The use of the party model by local organizations allows base builders to gain popular support, and most importantly, to mobilize their base of popular support towards revolutionary ends, not simply towards the construction of a parallel economy which exists as an end in and of itself. It is my hope that we will see future unification of the various local base building organizations into a national party, but in the meantime we must push for party organizing at the local level. If local organizations adopt party organizing, it ought to become clear that a unified national party will have to be the long term goal of the base building movement. Many of the already existing organizations within the base building movement already operate according to these principles. I do not

Page 210: openev.debatecoaches.org€¦  · Web view1AC . Observation 1: If I could find the spot where truth echoesI would stand there and whisper memories of my children's future. I would

mean to suggest otherwise. Rather, my hope is to suggest that we ought to be explicit about the need for party organizing and emphasize the relationship between dual power and the party model. Doing so will make it clear that the base building movement is not pursuing a cooperative economy alongside capitalism, but is pursuing a revolutionary socialist strategy capable of fighting capitalism. The long term details of base building and dual power organizing will arise organically in response to the conditions the movement finds itself operating within. I hope that I have put forward a useful contribution to the discussion about base building organizing, and have demonstrated the need for party organizing in order to ensure that the base building tendency maintains a revolutionary orientation. The finer details of revolutionary strategy will be worked out over time and are not a good subject for public discussion. I strongly believe party organizing offers the best path for ensuring that such strategy will succeed. My goal here is not to dictate the only possible path forward but to open a conversation about how the base building movement will organize as it transitions from a loose network of individual organizations into a unified socialist tendency. These discussions and debates will be crucial to ensuring that this rapidly growing movement can succeed.

American soft power is declining now due to Trump

Nye 2/12 [Joseph S. Nye, Jr. is a professor at Harvard and author of The future of power. “Donald Trump and the decline of US soft power.” ASPI 2/2/18 https://www.aspistrategist.org.au/donald-trump-decline-us-soft-power/ //#evilempire]

The evidence is clear. Donald Trump’ s presidency has eroded America’s soft power . Only 30% of people recently polled by Gallup in 134 countries held a favourable view of the United States under Trump’s leadership, a drop of almost 20 points since Barack Obama’s presidency. The Pew Research Center found that China, with 30% approval ratings, had reached near-parity with the US. And a British index, The Soft Power 30, showed America slipping from first place in 2016 to third place last year. Trump’s defenders reply that soft power does not matter. Trump’s budget director, Mick Mulvaney, proclaimed a ‘hard power budget’ as he slashed funds for the State Department and the US Agency for International Development by 30%. For promoters of ‘America First’, what the rest of the world thinks ranks second. Are they right? Soft power rests on attraction rather than coercion or payment. It co-opts people rather than coerces them. At the personal level, wise parents know that their power will be greater and will last

longer if they model sound ethical values for their children, rather than relying only on spankings, allowances or taking away the car keys. Similarly, political leaders have long understood the power that comes from being able to set the agenda and determine the framework of a debate. If I can get you to want to do what I want, then I do not have to force you to do what you do not want. If the US represents values that others want to follow, it can economise on sticks and carrots. Added to hard power, attraction can be a force multiplier. A country’s soft power comes primarily from three sources: its culture (when it is attractive to others), its political values such as democracy and human rights (when it lives up to them ), and its policies (when they are seen as legitimate because they are framed with some humility and awareness of others’ interests). How a government behaves at home (for example, protecting a free press), in international institutions (consulting others and multilateralism) and in foreign policy (promoting development and human rights) can affect others by the influence of its example. In all of these areas, Trump has reversed attractive American policies. Fortunately, America is more than either Trump or the government. Unlike hard-power assets (such as armed forces), many soft-power resources are separate from the government and are only partly responsive to its purposes. In a liberal society, government cannot control the culture. Indeed, the absence of official cultural policies can itself be a source of attraction. Hollywood movies like The Post, which showcase independent women and press freedom, can attract others. So, too, can the charitable work of US foundations or the benefits of freedom of inquiry at American universities. It is true that firms, universities, foundations, churches and other non-governmental groups develop soft power of their own which may reinforce or be at odds with official foreign policy goals. And all of these private sources of soft

power are likely to become increasingly important in the global information age. That is all the more reason for governments

Page 211: openev.debatecoaches.org€¦  · Web view1AC . Observation 1: If I could find the spot where truth echoesI would stand there and whisper memories of my children's future. I would

to make sure that their own actions and policies create and reinforce rather than undercut and squander their soft power. Domestic or foreign policies that appear hypocritical, arrogant, indifferent to others ’ views, or based on a narrow conception of national interests can undermine soft power. For example, the steep decline in the attractiveness of the US in opinion polls conducted after the invasion of Iraq in 2003 was a reaction to the Bush administration and its policies, rather than to the US generally. The Iraq War was not the first government policy that made the US unpopular. In the 1970s, many people around the world objected to the US war in Vietnam, and America’s global standing reflected the unpopularity of that policy. When the policy changed and the memories of the war receded, the US recovered much of its lost soft power. Similarly, in the aftermath of the Iraq War, the US managed to recover much of its soft power in most regions of the world (though less so in the Middle East). Skeptics might still argue that the rise and fall of American soft power does not matter much, because countries cooperate out of self-interest. But this argument misses a crucial point: cooperation is a matter of degree, and the degree is affected by attraction or repulsion. Moreover, the effects of a country’s soft power extend to non-state actors—for example, by aiding or impeding recruitment by terrorist

organisations. In an information age, success depends not only on whose army wins, but also on whose story wins. One of the greatest sources of America’s soft power is the openness of its democratic processes . Even when mistaken policies reduce its attractiveness , America’s ability to criticise and correct its mistakes makes it attractive to others at a deeper level. When protesters overseas were marching against the Vietnam War, they often sang ‘We Shall Overcome’, the anthem of the US civil rights movement. America, too, will almost certainly overcome. Given past experience, there is every reason to hope that the US will recover its soft power after Trump.

The US realigning itself with international ethics would be key to increasing US Soft Power. Their Epstein 2018 evidence specifically claims “the plan makes the US look more legit as a world power” which proves that they link. [Peter Maurer, 12-17-2014, "Rejecting Torture Is the Right Thing to Do," HuffPost, https://www.huffpost.com/entry/rejecting-torture-is-the_b_6336926] //WY

The fact of the matter is that torture is illegal. The prohibition of torture, and all forms of ill-treatment, is absolute — everywhere and at all times . It is strictly forbidden by international human rights law, U.S. jurisprudence, the United Nations Convention against Torture, and the Geneva Conventions — a body of law embraced by the U.S. and its military since the 19th Century. Importantly, torture is morally reprehensible and it should be permanently repudiated by all nations. Torture and other forms of cruel or humiliating treatment are an affront to humanity, and the physical and psychological scars can last a lifetime. It is morally wrong because it dehumanizes the victims, the perpetrators, and the society that

surrounds them. Torture can destroy the social fabric of communities, degrade a society’s institutions, and undermine the integrity of its political systems. I know that no country on Earth is entirely immune to what amounts to an ages-old

temptation, and some people reading this might think, “That’s easy for you to say. Sometimes you have to make tough choices in the name of national security.” I would argue that those who believe that perceived, near term tactical gains are worth risking long term and potentially dangerous consequences, are shortsighted. Experience shows that the reliance on illegal, immoral, and inhumane interrogation techniques is universally a very poor choice. The International Committee of the Red Cross

visits roughly half a million detainees in nearly 100 countries each year. It’s our job to try to prevent and put an end to torture and ill-treatment. During my time as president of the ICRC, I have met in person with detainees, who have experienced ill-treatment at the hands of their captors, and I know for a fact that abuse only grows hatred. It sows the seeds of revenge, and nothing justifies the use of such tactics. I have also observed that

torture has a corrupting effect on its perpetrators and the institutions they serve. Wherever it takes place and whatever the stakes involved, it’s simply wrong. In the wake of the September 11 terrorist attacks, the ICRC actively engaged with the U.S. on issues related to the treatment of detainees held in Iraq, Afghanistan, and Guantanamo Bay. Very early on, we decided to invest in a constructive and sustained dialogue with the American authorities. Even when discussions were sometimes tense over the past 12 years, the ICRC’s commitment to the detainees never wavered. We remain committed to visiting them, monitoring their treatment, connecting them with their families, and consistently reminding the U.S. of their obligations under

international humanitarian law. Today, our relationship with the government is strong. The bottom line is that decisions taken in

Page 212: openev.debatecoaches.org€¦  · Web view1AC . Observation 1: If I could find the spot where truth echoesI would stand there and whisper memories of my children's future. I would

Washington resonate across the globe. The U.S. has a major influence on the world stage and American

detention practices can, for better or for worse, have an impact on the actions of officials , prison directors, guards,

and interrogators in even the most far-flung places on the planet. That’s why this moment in history offers an opportunity to reaffirm clearly, and for the whole world to hear, that torture and ill-treatment are fundamentally wrong, and that unconditionally rejecting such tactics — now and forever — is the right thing to do.

US soft power is key to maintaining the US WORLD WIDE EMPIRE – turns case

Thomas 3/20 [Thomas, Joseph. Geopolitical Analyst, Thailand. U.S. “Soft Power”: How to Dominate Without the Use of Force or Coercion 3/20/18. Global Research Centre. https://www.globalresearch.ca/us-soft-power-how-to-dominate-without-the-use-of-force-or-coercion/5632991 //#evilempire]

Foreign Affairs magazine, published by big-business-funded US policy think tank, the Council on Foreign Relations, would reveal in a review of Joseph Nye’s book, “Soft Power: The Means to Success in World Politics,” that (my emphasis): …the term “soft power” — the ability of a

country to persuade others to do what it wants without force or coercion — is now widely invoked in foreign policy debates. The United States can dominate others, but it has also excelled in projecting soft power , with the help of its companies, foundations, universities, churches, and other institutions of civil society; U.S. culture, ideals, and values have been extraordinarily important in helping Washington attract partners and supporters. And in reality, US domination and its soft power work together to create what is modern day empire and the foundation of US global hegemony. The U nited S tates’ many organisations, from the National Endowment for Democracy (NED) to its Young Leaders Initiatives targeting the Americas (Young Leaders of the Americas Initiative/YLAI), Africa (Young African

Leaders Initiative/YALI) and Southeast Asia (Young Southeast Asian Leaders Initiative/YSELAI), all seek to indoctrinate and co-opt the populations of targeted nations to serve the interests of Wall Street and Washington rather than their own. While the US does this often under the guise of promoting “democracy,” it is clearly engaged in precisely the opposite. While democracy is generally understood as a process of self-determination, through US soft power, the process is co-opted and abused to allow Wall Street and Washington to determine the policies and direction a targeted nation takes rather than its own people. Often times victims of US soft power are youths who are indoctrinated in university programmes or targeted by US-funded fronts posing as nongovernmental organisations (NGOs). They believe they have arrived at their conclusions and adopted their personal set of principles on their own, unaware of the amount of time, money and energy invested in ensuring they adopt a worldview and a set of political proclivities that serve US interests rather than those of their own nation, people and those of the individuals themselves. The use of soft power is not new. It is a practice as old as empire itself. The ancient Romans engaged in sophisticated cultural colonisation we could easily describe as soft power. Ancient Roman historian Tacitus (c. AD 56 – after 117) would adeptly describe the systematic manner in which Rome pacified foreign peoples and the manner in which it would extend its sociocultural and institutional influence over conquered lands. In chapter 21 of his book Agricola, named so after his father-in-law whose methods of conquest were the subject of the text, Tacitus would explain (my emphasis): His object was to accustom them to a life of peace and quiet by the provision of amenities. He therefore gave official assistance to the building of temples, public squares and good houses. He educated the sons of the chiefs in the liberal arts, and expressed a preference for British ability as compared to the trained skills of the Gauls. The result was that instead of loathing the Latin language they became eager to speak it effectively. In the same way, our national dress came into favour and the toga was everywhere to be seen. And so the population was gradually led into the demoralizing temptation of arcades, baths and sumptuous banquets. The unsuspecting Britons spoke of such novelties as ‘civilization’, when in fact they were only a feature of their enslavement. In a very similar manner, youths today in nations targeted by US soft power describe the notions of “democracy” and “human rights’ as well as Western-style neo-liberal politics

Page 213: openev.debatecoaches.org€¦  · Web view1AC . Observation 1: If I could find the spot where truth echoesI would stand there and whisper memories of my children's future. I would

and institutions as “civilisation.” They often seek out every opportunity to disparage the culture and institutions of their own nation, describing them as backwards and demanding they be promptly replaced with new notions and institutions modelled after or directly beholden to those in the US and Europe. Thailand’s “Color Revolution”: US Meddles Abroad, Accuses Others of Meddling at Home. We can see across the whole of Asia this full process of soft power coming to fruition. Years and millions of dollars spent in infiltrating universities, indoctrinating youths through programmes like YSEALI or the British Chevening scholarships and funding and directing fronts posing as NGOs has led to the creation of entire political parties contesting power, comprised of indoctrinated youths beholden both to the notions of Western culture and institutions as well as the money and technical support nations like the US and UK directly provide these parties. Hong Kong’s “Demosisto” political party is made up entirely of youths and NGO representatives that have been created and funded for years by the US, UK and various other European interests. Myanmar’s ruling National League for Democracy has the top echelons of its party run by former journalists, activists and politicians cultivated, funded and trained by US-funded programmes for decades. This includes the current minister of information, Pe Myint. Case Study: Thailand The recently formed “Future Forward” opposition party headed by Thanathorn Juangroongruangkit,the heir of a multi-million dollar auto-parts business, has overtly advertised itself as an amalgamation of Western-style neo-liberal political ideology. While the supposed “founders” of the party appear to fully represent various social issues, the immense amount of money needed to perform “Future Forward’s” campaigning indicates the true founders (and financial sponsors) have chosen to remain behind the scenes. Reuters in its article, “Thai auto heir launches new party, promises to heal political rift,” would admit: Thanathorn introduced other party co-founders on Thursday, including a filmmaker and a number of activists involved in LGBT and environment causes, among other issues. Party co-founder Piyabutr Saengkanokkul, a law lecturer at Bangkok’s Thammasat University, said the party hopes to transcend Thailand’s political divide, a sentiment echoed by the student-led groups that have held anti-junta protests across Bangkok in recent weeks. But some say the party might find it difficult to appeal to grassroots voters. “Will they, academics and NGOs … be able to connect with grassroots people, which is a large part of the electoral base?,” asked one Twitter user. To create that electoral base, the US is currently funding programmes inside Thailand specifically to infiltrate and co-opt local, regional and national concerns. Everything from environmental issues regarding the building of dams and power plants to women’s rights and access to education have been used as vectors by US-funded organisations seeking to co-opt and knit together various genuine individual pragmatic causes into a singular, national political clearinghouse. Part of this singular front’s responsibilities will be to serve as a voting bloc to place parties like “Future Forward” into power. NED and YSEALI are two examples of how single US organisations are targeting and cultivating youths much in the way Tacitus described in Agricola. These individuals are cultivated to be “leaders” who then create their own organisations (often US funded) to begin recruiting and indoctrinating additional members. Like a pyramid scheme, the efforts’ structure enables the US to recruit and indoctrinate Thais faster than any single US organisation could do on its own. While programmes like YSEALI boast of thousands of leaders who undoubtedly have infected thousands more with US-funded indoctrination, its still isn’t likely enough to create a voting bloc big enough to place “Future Forward” into power. But it doesn’t need to be. The US is still depending on existing political machines of politicians like US proxy Thaksin Shinawatra to create the support needed to propel “Future Forward” and other parties like it politically. Future Forward: The Evolution of a US Proxy While Reuters admits that Future Forward has been accused of ties to US proxy Thaksin Shinawatra, the article fails to mention the substantial evidence those making the accusations are citing. Piyabutr, mentioned by Reuters as the party’s co-founder, had previously abused his academic credentials to organise and host an indoor event for Thaksin Shinawatra’s United Front for Democracy Against Dictatorship (UDD) also known as red shirts. The event held at Thammasat University, included Thaksin Shinawatra’s lobbyist Robert Amsterdam given a front row seat during the proceedings. The red shirts are Shinawatra’s street front whose reputation had become a political liability after back-to-back riots and deadly armed violence the front carried out in 2009-2010. Piyabutr and fellow academics endeavoured to rehabilitate the UDD’s public image by transforming it into a more academic movement, papering over the crass populism and demagoguery used to create it in the first place. While the “red shirt” street front is still used to give emerging successors to Shinawatra’s political machinery the numbers they need at public events, protests and rallies, this new, more academic face is what is being presented to the public, and the world. Soft Power’s Final Destination: Consume All, Including Allies The US will continue attempting to create a voting bloc independent of traditional political figures like Thaksin Shinawatra and his own networks of patronage. While Thaksin Shinawatra has been a loyal servant of US interests for years, the US would prefer a political party and a voting bloc it controls entirely on its own. By Shinawatra supporting the creation of parties like “Future Forward” he is in reality sealing his own political fate. Special interests sponsoring “Future Forward’s” political activities

are also creating a monster that will eventually consume them both politically and economically in the future. As demonstrated in nations around the world subjected to the full cycle of US meddling, co-opting, infiltration and domination , even those special interests that eagerly assisted US ambitions find themselves unwelcomed competitors once the US finally succeeds. Those who believe they can “ride the tiger” of US heg emony into power often find themselves the target of the very domestic networks of agitators and activists they helped the US create. Protecting Against US Soft Power Clearly, the soft power process has nothing to do with any genuine interpretation of democracy. It is simply using democratic themes and procedures to lend legitimacy to what is modern day imperialism and the very sort of soft power employed by the Romans against the ancient world centuries ago. Thailand and other nations targeted by US soft power can only defend themselves by being able to both effectively expose US soft power methods, and by countering them through the work of indigenous institutions and genuine NGOs filling Thailand’s political, activist, educational, information and economic space sufficiently enough so that no room remains for foreign-funded alternatives. As to why the US is so interested in co-opting and

Page 214: openev.debatecoaches.org€¦  · Web view1AC . Observation 1: If I could find the spot where truth echoesI would stand there and whisper memories of my children's future. I would

controlling Thailand politically, the answer lies in Washington’s larger Asia-Pacific agenda which includes the encirclement and containment of China with nations that do business with and are entirely under the influence of Washington. A political party run by the products of decades of US cultural colonisation and soft power efforts taking office in Thailand would directly serve Washington’s wider regional ambitions and augment its efforts to co-opt and control Thailand’s Southeast Asian neighbours as well.

Black arguments cannot be debated without the experience of pain and suffering of black people. The aff is a appropriation of black culture in order to win the debate, re-entrenching whiteness due to the benefits of black suffering.WILDERSON IN 10 [Frank B., Associate Professor of African American Studies and Drama at the University of California & former member of the Umkhonto we Sizwe, Red, White and Black: Cinema and the Structure of US Antagonisms, p. 10, C.A.]Again, if accumulation and fungibility are the modalities through which Blackness is positioned as incapacity, then genocide is that modality through which embodied Redness is positioned as incapacity. Ontological incapacity, I have inferred and here state forthright, is the constituent element of ethics . Put another way, one cannot embody capacity and be, simultaneously, ethical. Where there are Slaves it is unethical to be free . The Settler/Master’s capacity, I have argued, is a function of exploitation and alienation, and the Slave’s incapacity is elaborated by accumulation and fungibility. But the “Savage” is positioned, structurally, by subjective capacity and objective incapacity, by sovereignty and genocide, respectively. The Indian’s liminal status in political economy, how her and his position shuttles between the incapacity of a genocided object and the capacity of a sovereign subject, coupled with the fact that Redness does not overdetermine the thanatology of libidinal economy (this liminal capacity within political economy and complete freedom from incapacity within libidinal economy) raises serious

doubts about the status of “Savage” ethicality vis-à-vis the triangulated structure (Red, White, and Black) of antagonisms. Clearly, the coherence of Whiteness as a structural position in modernity depends on the capacity to be free from genocide, perhaps not as a historical experience, but at least as a positioning modality . This embodied capacity (genocidal immunity) of Whiteness jettisons the White/Red relation from that of a conflict and marks it as an antagonism: it stains it with irreconcilability. Here, the Indian comes into being and is positioned by an a priori violence of genocide. Whiteness can also experience this kind of violence but only a fortiori: genocide may be one of a thousand contingent experiences of Whiteness but it is not a

constituent element, it does not make Whites white (or Humans human). Whiteness can grasp its own capacity, be present to itself, coherent, by its unavailability to the a priori violence of Red genocide, as well as by its unavailability to the a priori violence of Black accumulation and fungibility. If it experiences accumulation and fungibility, or genocide, those experiences must be named , qualified, that is, “White

slavery,” or the Armenian massacre, the Jewish Holocaust, Bosnian interment, so that such contingent experience is not confused with ontological necessity . In such a position one can always say, “Im not a ‘Savage’” or “I’m being treated like a nigger.” One can assert one’s Humanity by refusing the ruse of analogy. Regardless of Whites’ historical, and brief, encounters with the modalities of the “Savage” and the Slave, these modalities do not break in on the position of Whiteness with such a force as to replace

exploitation and alienation as the Settler/Master’s constituent elements. We might think of exploitation and alienation as modalities of suffering which inoculate Whiteness from death . If this is indeed the case, then perhaps Whiteness has no constituent elements other than the immanent status of immunity. Still, this immunity is no small matter, for it is the sine qua non of Human capacity.

Page 215: openev.debatecoaches.org€¦  · Web view1AC . Observation 1: If I could find the spot where truth echoesI would stand there and whisper memories of my children's future. I would

3. Pragmatism-Their desire to seek a perfect conceptual understanding of power before reengaging the political makes the good the enemy of the perfect, we can’t afford to wait.Yancy 1 (George Yancy and Cornel West, "Cornel West", p 70) *we do not endorse ableist language

The final component of the political construal of prophetic pragmatism deals with the issue of power. Here, West announces that prophetic pragmatism will unmask the different power structures that present themselves as natural. Along with this, he Ervors a genealogical strategy that will similarly unearth the relevant practices and systems of beliefs inappropriately dependent upon questionable moral values. Confessing to his Foucauldian connections, he states that “Prophetic pragmatism shares with Foucault a preoccupation with the operation of powers. It also incorporates the genealogical mode of

inquiry .... [Plrophetic pragmatism promotes genealogical materialist modes of analysis similar in many respects to those of Foucault.”3" West finally construes prophetic pragmatism as a form of tragic thought. although not a “doomsday” form of thinking. To the extent that prophetic pragmatism evades involvement with a priori epistemological concerns but instead focuses on the everyday concerns of human beings, West cautions that. despite our efforts to better our existential condition, both personal and social evil are persistent realities not immediately submissive to our intentions to do good. Evil’s evasion of the good that we seek to do will continue. West's focus on the presence of evil is not an invitation to concede defeat but rather one to force us to become humble as we realize that our best intentions are not immune to failure. Prophetic pragmatism . according to West, “is a form of trage thought in that it conlionts candidly individual and collective experiences of evil in individuals and institutions - with little expectation of ridding the world of all evil.” 3' As with his other construals. West links his conception of prophetic pragmatism, as a fomi of tragic thought, with democracy. What he seems to be getting at here is the idea that it is misleading to assume that we can totally and completely

realize the absolute ideal of democracy as a historically manifested reality. Democracy remains an ideal that we must constantly pursue while realizing that we are not pursuing an illusion. The dynamics of human existence and the dialectical tensions between human being and the natural world continually give rise to various obstacles and challenges as we seek a more socially enriching existence. Nevertheless, in acknowledging the constant struggle to improve things, West warns against surrendering to pessimism or yielding to the seductive idea of perfectibility. Hence, he maintains: Prophetic pragmatism denies Sisyphean pessimism and utopian perfectionism. Rather. it promotes the possibility of human progress and human impossibility of paradise. Human struggles its at the center of prophetic pragmatism, a struggle guided by a democratic and libertarian vision, sustained by moral courage and existential integrity, and tempered by the recognition of human finitude and fragility.” Clearly. West focuses persistently upon the importance of creating “new possibilities for human agency.” He even briefly describes his prophetic pragmatism as a form of historical consciousness attuned

to the struggles of the past as well as appreciative of the alternative forms of life based upon the best of the past. But he quickly interjects the theme of the tragic, stating that the praxis of prophetic pragmatism “is tragic action with revolutionary intent." 33 Finally, forever straddling two modes of thought at once, West reiterates that prophetic pragmatism is not blind to the fact that utopian schemes QIIIJOK escape the limitation presented by the unfortunate condition of individuals and the structures of injustice normally sustained by social, economic and political institutions. So, even if prophetic pragmatism evades pessimism, it still cautiously embraces utopianism. According to West: Prophetic pragmatism. . . tempets its utopian impulse with a profound sense of the tragic character of life and history. This sense of the tragic highlightst be irreducible predicament of unique individuals who undergo dread, despair, disillusionment, disease, and death and institutional forms of oppression that dehumanize people.” Now that we have some understanding of \Vest's impressionistic conception of prophetic pragmatism. we can offer an evaluation of it. We should keep in mind, however, that thus task is complicated by West's preference for a method of reading that emphasizes moving the reader through passionate and motivational rhetoric rather than the reader through the dispassionate execution of arguments. Hence, in criticizing West, we must focus on the vision at the heart of his prophetic pragmatism and not treat it as a systematic body of truths.

Page 216: openev.debatecoaches.org€¦  · Web view1AC . Observation 1: If I could find the spot where truth echoesI would stand there and whisper memories of my children's future. I would

1. You fail to ground your discourse in materiality – you don’t defend anything and understand the impacts of your inaction – vote neg on presumption Rashad Evans writes while debating Casey Harrigan in 2012: Rashad Evans, 2012 11-20-2012 (http://www.rwesq.com/the-1nc/)

A quare approach to debate includes afro-pessimism. For instance, I am pessimistic that any of the solutions being floated around will meaningfully address anti-blackness. I am pessimistic of changes in MPJ, resolutional wording or tournament practices to address the attitudinal and structural (throwback) barriers to black participation in debate. A quare approach to debate recognizes the importance that color, sex and gender have in structuring our lives and producing differing results for different people . Disidentification is a way by which we can enact change at the structural level through localized acts of resistance . It is a process whereby we can work on and against the system. I started this process at Harvard with the

initiation of my affirmative action policy and have continued on facebook and my blog. This disidentification is important because it recognizes the material consequences of our actions and inactions. It also allows individuals utilize discourse in a subversive way. E. Patrick Johnson explains that: “Foucault himself acknowledges that discourse has the potential to disrupt power: “Discourses are not once and for all subservient to power or raised up against it, anymore than silences are. We must make allowances for complex and unstable process whereby discourse can be both an instrument and effect of power, but also a hindrance, a stumbling block, a point of resistance and a starting point for an opposing strategy. Discourse transmit and produces power; it reinforces it , but also undermines and exposes it, renders it fragile and makes it possible to thwart it. Although people of color, myself included, may not have theorized our lives in Foucault’s terms, we have used discourse in subversive ways because it was necessary for our survival. Failure to ground discourse in materiality is to privilege the position of those whose subjectivity and agency, outside the realm of

gender and sexuality, have never been subjugated.” Therefore, we must look look at the material consequences of our acts and inactions and account for the ways in which certain performances either maintain power or subvert power. In addition, we must account for stylistic differences. It is at the level of style that most black and queer

politics are enacted. In the debate space, stylistic differences is one of the most important ways that Black debaters enact their politics and resistance. Style is an important tool of survival for Black, queer and female

debaters. One cannot speak if she cannot speak with and from her own voice. Style is important and so too is the

value of using the debate space to theorize about life more generally. Debate is a homeplace for its participants. Theorizing about things like anti-blackness, humanism and feminism from within the debate space is important to teaching debaters how to theorize about this issues outside the debate space . We must always be able to be critical in those places where we are most familiar and most comfortable. That is why these discussions regarding debate are so important.

Important to a quare understanding of debate is the work done by Dr. Shanara Reid-Brinkley and her call that we pay specific

attention to how we produce our scholarship. Responsible argumentation should account for multiple forms of knowledge including (1) academic scholars, (2) organic intellectuals and (3) personal experience.

Responsible debaters also engage in knowledge production and not just knowledge consumption.

2. This replicates the revolutionary hero who limits their analysis to one time and place instead of formulating strategy to create replicable solution. This prevents any ability to turn theory to praxis, turns all of caseNewton-1971 –Huey P. Newton- Black Capitalism Reanalysed- Huey P. Newton Reader- 227-228

*we do not endorse gendered language

We see then that power has a dual character and that we cannot simply identify and define phenomena without acting, for to do so is to become an armchair philosopher. And when Bobby and I left Merritt

Page 217: openev.debatecoaches.org€¦  · Web view1AC . Observation 1: If I could find the spot where truth echoesI would stand there and whisper memories of my children's future. I would

College to organize brothers on the block we did so because the college students were too content to sit around and analyze without acting. On the other hand, power includes action , for it is making phenomena perform in the desired manner. But action without thinking and theory is also incorrect. If the social forces at work in the community have not been correctly analyzed and defined, how can you control them in such a way that they act in a desired manner? So the Black Panther Party has always merged theory and practice in such a way as to serve the true interests of the community. In merging theory with practice we recognized that it was necessary to develop a theory which was valid for more than one time and place. We wanted to develop a system of which was good anywhere, thus it had to be rather abstract. Yet our theory would relate to a concrete analysis of concrete conditions so that our actions would always be relevant and profitable to the people. Yet, at the same time, it had to advance their thinking so that they would move toward a transformation of their situation of exploitation and oppression. We have always insisted on good theory and good practice, but we have not always been successful in carrying this through. When the Black Panther Party defected from the Black community; we became, for a while, revolutionary cultists. One of the primary characteristic s of a revolutionary cultist is that he [they] despise s everyone who has not reached his [their] level of consciousness, or the level of consciousness that he [they] thinks he has [they have] reached, instead of acting to bring the people to that level. In that way the revolutionary cultist becomes divided from the people, he [they] defects from the community. Instead of serving the people as a vanguard , he [they] become s a hero. Heroes engage in very courageous actions sometimes, and they often make great sacrifices, including the supreme sacrifice, but they are still isolated from the people. Their courageous actions and sacrifices do not lead the people to a higher level of consciousness, nor do they produce fundamental changes in the exploitation and oppression of the people. A vanguard, however, will guide the people onto higher levels of consciousness and in that way bring them to the point where they will take sterner actions in their own interests and against those who continue to oppress them. As I've said previously; revolution is a process, not a conclusion. A true revolutionist will not only take courageous actions, he [they] will also try to advance the people in such a manner that they will transform their situation. That is, by delivering power to the people the true revolutionist will help them define the social phenomena in their community and lead them to the point where they will seize the time and make these phenomena act in a desired manner.

3. Advocacy Skills - Analysis around material change is essential to provide political training. Ignoring politics only cedes the political to the right and hurts those subject to carceral logic.Themba-Nixon 2k (makani, Changing the rules what public policy means for organizing Colorlines 3.2)

*we do not endorse ableist language

In essence, policies are the codification of power relationships and resource allocation. Policies are the rules of the world we live in. Changing the world means changing the rules. So, if organizing is about changing the rules and building power, how can organizing be separated from policies? Can we really speak truth to power, fight the right, stop corporate abuses, or win racial justice without contesting the rules and the rulers, the policies and the policymakers? The answer is no-and double no for people of color. Today, racism subtly dominates nearly every aspect of policymaking. From ballot propositions to city funding priorities, policy is increasingly about the control, de-funding, and disfranchisement of communities of color. What Do We Stand For? Take the public conversation about welfare

Page 218: openev.debatecoaches.org€¦  · Web view1AC . Observation 1: If I could find the spot where truth echoesI would stand there and whisper memories of my children's future. I would

reform, for example. Most of us know it isn't really about putting people to work. The right's message was framed around racial stereotypes of lazy, cheating "welfare queens" whose poverty was "cultural." But the new welfare policy was about moving billions of dollars in individual cash payments and direct services from welfare recipients to other, more powerful, social

actors. Many of us were too busy to tune into the welfare policy drama in Washington, only to find it washed up right on our doorsteps. Our members are suffering from workfare policies, new regulations, and cutoffs. Families who

were barely getting by under the old rules are being pushed over the edge by the new policies. Policy doesn't get more relevant than this. And so we got involved in policy-as defense. Yet we have to do more than block their punches. We have to start the fight with initiatives of our own. Those who do are finding offense a bit more fun than defense alone. Living wage ordinances, youth development initiatives, even gun control and alcohol and tobacco policies are finding their way onto the public agenda, thanks to focused community organizing that leverages power for community-driven initiatives. - Over 600 local policies have been passed to regulate the tobacco industry. Local coalitions have taken the lead by writing ordinances that address local problems and organizing broad support for them. - Nearly 100 gun control and violence prevention policies have been enacted since 1991. - Milwaukee, Boston, and Oakland are among the cities that have passed living wage ordinances: local laws that guarantee higher than minimum wages for workers, usually set as the minimum needed to keep a family of four above poverty. These are just a few of the examples that demonstrate how organizing for local policy advocacy has made inroads in areas where positive national policy had been stalled by conservatives. Increasingly, the local policy arena is where the action is and where activists are finding success. Of course, corporate interests-which are usually the target of these policies-are gearing up in defense. Tactics include front groups, economic pressure, and the tried and true: cold, hard cash. Despite these barriers, grassroots organizing can be very effective at the smaller scale of local politics. At the local level, we have greater access to elected officials and officials have a greater reliance on their constituents for reelection. For example, getting 400 people to show up at city hall in just about any city in the U.S. is quite impressive. On the other hand, 400 people at the state house or the Congress would have a less significant impact. Add to that the fact that all 400 people at city hall are usually constituents, and the impact is even greater.

Recent trends in government underscore the importance of local policy. Congress has enacted a series of measures devolving significant power to state and local government. Welfare, health care, and the regulation of food and drinking water safety are among the areas where states and localities now have greater rule. Devolution has some negative consequences to be sure. History has taught us that, for social services and civil rights in particular, the lack of clear federal standards and mechanisms for accountability lead to uneven enforcement and even discriminatory implementation of policies. Still, there are real opportunities for advancing progressive initiatives in this more localized environment. Greater local control can mean greater community power to shape and implement important social policies that were heretofore out of reach. To do so will require careful

attention to the mechanics of local policymaking and a clear blueprint of what we stand for. Getting It in Writing Much of the work of framing what we stand for takes place in the shaping of demands. By getting into the policy arena in a proactive manner, we can take our demands to the next level. Our demands can become law, with real consequences if the agreement is broken. After all the organizing, press work, and effort, a group should leave a decision maker with more than a handshake and his or her word. Of course, this work requires a certain amount of interaction with "the suits," as well as struggles with the bureaucracy, the technical language, and the all-too-common resistance by decision makers. Still , if it's worth demanding, it's worth having in writing- whether as law, regulation, or internal policy. From ballot initiatives on rent control to laws requiring worker protections, organizers are leveraging their power into written policies

that are making a real difference in their communities. Of course, policy work is just one tool in our organizing arsenal, but it is a tool we simply can't afford to ignore. Making policy work an integral part of organizing will require a certain amount of retrofitting. We will need to develop the capacity to translate our information , data, and experience into stories that are designed to affect the public conversation. Perhaps most important, we will need to move beyond fighting problems and on to framing solutions that bring us closer to our vision of how things should be. And then we must be committed to making it so.

Page 219: openev.debatecoaches.org€¦  · Web view1AC . Observation 1: If I could find the spot where truth echoesI would stand there and whisper memories of my children's future. I would

Complete rejection of the real world institutional logic of civil society crushes anti-white supremacy projectsKimberle Crenshaw 1988, law at UCLA, “Race, Reform, and Retrenchment: Transformation and Legitimation in Antidscrimination Law 101,” Harvard law review 1331//MMWang

The Critics' product is of limited utility to Blacks in its present form. The implications for Blacks of trashing liberal legal ideology are troubling, even though it may be proper to assail belief structures that obscure liberating possibilities. Trashing legal ideology seems to tell us repeatedly what has already been established - that legal discourse is unstable and relatively indeterminate. Furthermore, trashing offers no idea of how to avoid the negative

consequences of engaging in reformist discourse or how to work around such consequences. Even if we imagine the wrong world when we think in terms of legal discourse, we must nevertheless exist in a present world where legal protection has at times been a blessing - albeit a mixed one. The fundamental problem is that, although Critics criticize law because it functions to legitimate existing institutional arrangements, it is precisely this legitimating function that has made law receptive to

certain demands in this area. The Critical emphasis on deconstruction as the vehicle for liberation leads to the conclusion that engaging in legal discourse should be avoided because it reinforces not only the discourse itself but also the society and the world that it embodies . Yet Critics offer little beyond this observation. Their focus on delegitimating rights rhetoric seems to suggest that, once rights rhetoric has been discarded, there exists a more productive strategy for change, one which does not reinforce existing patterns of domination . Unfortunately, no such strategy has yet been articulated, and it is difficult to imagine that racial minorities will ever be able to discover one . As Frances Fox Piven and Richard Cloward

point out in their Tushnet observes: The conditions of the society define exactly what kind of rights-talk makes sense, and the sort of rights-talk that makes sense in turn defines what the society is. When someone objects to an act as a violation of a right, the ensuing dialogue either involves a claim that the challenged act is inconsistent with some "deeper" commitments that the actor has . . . or deals with what kind of society we ought to have excellent account of the civil rights movement, popular struggles are a reflection of institutionally determined logic and a challenge to that logic . 137 People can only demand change in ways that reflect the logic of the institutions that they are challenging .138 Demands for change that do

not reflect the institutional logic - that is, demands that do not engage and subsequently reinforce the dominant ideology - will probably be ineffective . 139 The possibility for ideological change is created through the very process of legitimation , which is triggered by crisis. Powerless people can sometimes trigger such a crisis by challenging an institution internally , that is, by using its own logic against it . 140 Such crisis occurs when powerless people force open and politicize a contradiction between the dominant ideology and their reality. The political conse- 137 See id. at 22-25. The observation concerning the inability to bring about change in some non-legitimating fashion does not, of course, rule out the possibility of

armed revolution. For most oppressed peoples , however, the costs of such a revolt are often too great . That

is, the oppressed cannot realistically hope to overcome the "coercive" components of hegemony . More

importantly, it is not clear that such a struggle, although superficially a clear radical challenge to the coercive force of the status

quo, would be a lesser reinforcement of the ideology of American society (i.e., the consensual components of

hegemony). Consequences of maintaining the contradictions may sometimes force an adjustment - an attempt to close the gap or to make things appear fair. 141 Yet, because the adjustment is triggered by the political consequences of the contradiction, circumstances will be adjusted only to the extent necessary to close the apparent contradiction. This approach to understanding legitimation and change is applicable to the civil rights movement . Because Blacks were challenging their exclusion from political society, the only claims that were likely to achieve recognition were those that reflected American society's institutional logic: legal rights ideology. Articulating their formal demands through legal rights ideology, civil rights protesters exposed a series of contradictions - the most important being the promised privileges of American citizenship and the practice of absolute racial subordination. Rather than using the contradictions to suggest that American citizenship was itself illegitimate or false, civil rights protestors proceeded as if American citizenship were real, and demanded to

exercise the "rights" that citizenship entailed. By seeking to restructure reality to reflect American mythology, Blacks

Page 220: openev.debatecoaches.org€¦  · Web view1AC . Observation 1: If I could find the spot where truth echoesI would stand there and whisper memories of my children's future. I would

relied upon and ultimately benefited from politically inspired efforts to resolve the contradictions by granting formal rights. Although it is the need to maintain legitimacy that presents powerless groups with the opportunity to wrest concessions from the dominant order, it is the very accomplishment of legitimacy that forecloses greater possibilities. In sum, the potential for change is both created and limited by legitimation.

Strategic resistance through institutional engagement can allow us to understand, expose, and destabilize whitenessJohn Powell 2005, Williams Chair in Civil Liberties and Civil Rights @ Moritz College of Law – Ohio State University, Executive Director @ Kirwan Institute for the Study of Race and Ethnicity, “Dreaming of a Self Beyond Whiteness and Isloation,” Washington University Journal of Law and Policy//MMWang

I am not suggesting that the interrogation of whiteness and the privileges bound up in it is not useful. I have attempted to do some of that in another article,82 but these approaches are both incomplete and likely to be misdirected. Any particular focus on privilege as being something that can possibly be separated from whiteness is likely to leave the structure of whiteness in place with the reinscribing of a new arrangement

of privileges. Any focus on the deconstruction of whiteness through the individual intent of whites risks, as

Wiegman notes, “reproducing . . . the white male rebel as the . . . subject of antiracist struggle.”83 In our

increasingly complicated postmodern world, transgression is much more complicated than deciding “not to be white.” One could point to the Civil War and the Civil Rights Movement as two examples of the attempt to disturb white privilege without striking at its core. But how are we to strike at the core of white racial hierarchy? What I have been asserting throughout this article is that we

must address the ontological question. This is not a retreat from the possible or a retreat to the interior. It is not just race that is socially constructed; so is the self.84 We must better understand and address how this self is constructed and what maintains its attachment to whiteness . The self, and particularly the white self, has a history . Because the self has a history, it is constantly being made and remade. This process goes largely unnoticed and hides behind a veil of naturalness . But it would be a serious error to see this as only an internal undertaking. This is part of the myth of the individual subject, that the self is internal and private. We must expose the social nature of the

subject. This subject is not just held together by other subjects but also by our norms, practices and institutions . This subject is related to other subjects and to the world. Together this creates a context that the subject lives in both externally and internally . But this fix is never perfect. This context always denies some possibility that yearns for expression. This yearning itself is part of the hope. As we think about institutional arrangements, we must think about what they mean for routine expression and the experience of internal space. Unger reminds us that the way institutions are arranged will either mutilate or provide space for the emancipation of our being.85 For those of us with privilege, we must use those privileges we cannot reject to better understand, expose, and

destabilize the structures and cultural norms that support and reinscribe whiteness. We must raise the cost of maintaining whiteness by seeking strategic interventions that reduce racialized disparities across multiple areas, but still seek to better understand and problematize whiteness. We must begin to work for a new set of arrangements that will support a new way of relating, a new way of being . So, part of this answer is in the material world, the arrangements of structures and institutions—not only because we need to address material needs and disparities, but because

structures are not separate from our self. But we must keep an eye on the self that we are trying to call into being. Without working on the interiorization of whiteness, we simply cannot solve the problem of whiteness . There has been some development in this region, but I believe that the ontological question of whiteness remains largely undertheorized.86

The purpose of this understanding is to end the performance of whiteness, not so that whites can be non-white or uncolored people, but so we can all be human with all our social amalgamation and complexity that has so long been denied and dreaded. Feminist theory has developed a view of the self as radically relational. It is this relational self that whiteness is created in, but fearful to acknowledge. How can we create communities of kinship that allow us to explore these connections?

Page 221: openev.debatecoaches.org€¦  · Web view1AC . Observation 1: If I could find the spot where truth echoesI would stand there and whisper memories of my children's future. I would

This replicates the revolutionary hero who limits their analysis to one time and place instead of formulating strategy to take theory and create replicable solution in the community. This prevents any ability to turn theory to praxis Newton-1971 –Huey P. Newton- Black Capitalism Reanalysed- Huey P. Newton Reader- 227-228

*we do not endorse gendered language

We see then that power has a dual character and that we cannot simply identify and define phenomena without acting, for to do so is to become an armchair philosopher. And when Bobby and I left Merritt College to organize brothers on the block we did so because the college students were too content to sit around and analyze without acting. On the other hand, power includes action , for it is making phenomena perform in the desired manner. But action without thinking and theory is also incorrect. If the social forces at work in the community have not been correctly analyzed and defined, how can you control them in such a way that they act in a desired manner? So the Black Panther Party has always merged theory and practice in such a way as to serve the true interests of the community. In merging theory with practice we recognized that it was necessary to develop a theory which was valid for more than one time and place. We wanted to develop a system of which was good anywhere, thus it had to be rather abstract. Yet our theory would relate to a concrete analysis of concrete conditions so that our actions would always be relevant and profitable to the people. Yet, at the same time, it had to advance their thinking so that they would move toward a transformation of their situation of exploitation and oppression. We have always insisted on good theory and good practice, but we have not always been successful in carrying this through. When the Black Panther Party defected from the Black community; we became, for a while, revolutionary cultists. One of the primary characteristic s of a revolutionary cultist is that he [they] despise s everyone who has not reached his [their] level of consciousness, or the level of consciousness that he [they] thinks he has [they have] reached, instead of acting to bring the people to that level. In that way the revolutionary cultist becomes divided from the people, he [they] defects from the community. Instead of serving the people as a vanguard , he [they] become s a hero. Heroes engage in very courageous actions sometimes, and they often make great sacrifices, including the supreme sacrifice, but they are still isolated from the people. Their courageous actions and sacrifices do not lead the people to a higher level of consciousness, nor do they produce fundamental changes in the exploitation and oppression of the people. A vanguard, however, will guide the people onto higher levels of consciousness and in that way bring them to the point where they will take sterner actions in their own interests and against those who continue to oppress them. As I've said previously; revolution is a process, not a conclusion. A true revolutionist will not only take courageous actions, he [they] will also try to advance the people in such a manner that they will transform their situation. That is, by delivering power to the people the true revolutionist will help them define the social phenomena in their community and lead them to the point where they will seize the time and make these phenomena act in a desired manner.

3. Revolutionary education- Contextualizing analysis around material change is essential to provide political training. Ignoring politics only cedes the political to the right which UNIQULY hurts people of color. We must gain the tools to fight power in the halls of power on OUR termsThemba-Nixon 2k (makani, Changing the rules what public policy means for organizing Colorlines 3.2)

*we do not endorse ableist language

Page 222: openev.debatecoaches.org€¦  · Web view1AC . Observation 1: If I could find the spot where truth echoesI would stand there and whisper memories of my children's future. I would

In essence, policies are the codification of power relationships and resource allocation. Policies are the rules of the world we live in. Changing the world means changing the rules. So, if organizing is about changing the rules and building power, how can organizing be separated from policies?

Can we really speak truth to power, fight the right, stop corporate abuses, or win racial justice without contesting the rules and the rulers, the policies and the policymakers? The answer is no-and double no for people of color. Today, racism subtly dominates nearly every aspect of policymaking. From ballot propositions to city funding priorities, policy is increasingly about the control, de-funding, and disfranchisement of communities of color. What Do We Stand For?

Take the public conversation about welfare reform, for example. Most of us know it isn't really about putting people to work. The right's message was framed around racial stereotypes of lazy, cheating "welfare queens" whose poverty was "cultural." But the new welfare policy was about moving billions of dollars in individual cash payments and direct services from welfare

recipients to other, more powerful, social actors. Many of us were too busy to tune into the welfare policy drama in Washington, only to find it washed up right on our doorsteps. Our members are suffering from workfare policies, new regulations, and cutoffs. Families who were barely getting by under the old rules are being pushed over the edge by the new

policies. Policy doesn't get more relevant than this. And so we got involved in policy-as defense. Yet we have to do more than block their punches. We have to start the fight with initiatives of our own. Those who do are finding offense a bit more fun than defense alone. Living wage ordinances, youth development initiatives, even gun control and alcohol and tobacco policies are finding their way onto the public agenda, thanks to focused community organizing that leverages power for community-driven initiatives. - Over 600 local policies have been passed to regulate the tobacco industry. Local coalitions have taken the lead by writing ordinances that address local problems and organizing broad support for them. - Nearly 100 gun control and violence prevention policies have been enacted since 1991. - Milwaukee, Boston, and Oakland are among the cities that have passed living wage ordinances: local laws that guarantee higher than minimum wages for workers, usually set as the minimum needed to keep a family of four above poverty. These are just a few of the examples that demonstrate how organizing for local policy advocacy has made inroads in areas where positive national policy had been stalled by conservatives. Increasingly, the local policy arena is where the action is and where activists are finding success. Of course, corporate interests-which are usually the target of these policies-are gearing up in defense. Tactics include front groups, economic pressure, and the tried and true: cold, hard cash. Despite these barriers, grassroots organizing can be very effective at the smaller scale of local politics. At the local level, we have greater access to elected officials and officials have a greater reliance on their constituents for reelection. For example, getting 400 people to show up at city hall in just about any city in the U.S. is quite impressive. On the other hand, 400 people at the state house or the Congress would have a less significant impact. Add to that the fact that all 400 people at city hall are usually

constituents, and the impact is even greater. Recent trends in government underscore the importance of local policy. Congress has enacted a series of measures devolving significant power to state and local government. Welfare, health care, and the regulation of food and drinking water safety are among the areas where states and localities now have greater rule. Devolution has some negative consequences to be sure. History has taught us that, for social services and civil rights in particular, the lack of clear federal standards and mechanisms for accountability lead to uneven enforcement and even discriminatory implementation of policies. Still, there are real opportunities for advancing progressive initiatives in this more localized environment. Greater local control can mean greater community power to shape and implement important social policies that were heretofore out of reach. To do so will require careful attention to the mechanics of local policymaking and a clear blueprint of what we stand for. Getting It

in Writing Much of the work of framing what we stand for takes place in the shaping of demands. By getting into the policy arena in a proactive manner, we can take our demands to the next level. Our demands can become law, with real consequences if the agreement is broken. After all the organizing, press work, and effort, a group should leave a decision maker with more than a handshake and his or her word. Of course, this work requires a certain amount of interaction with "the suits," as well as struggles with the bureaucracy, the technical language, and the all-too-common resistance by decision makers. Still , if it's worth demanding, it's worth having in writing- whether as law, regulation, or internal policy. From ballot initiatives on rent control to laws requiring worker protections, organizers are

leveraging their power into written policies that are making a real difference in their communities. Of course, policy work is just one tool in our organizing arsenal, but it is a tool we simply can't afford to ignore. Making policy work an integral part of organizing will require a certain amount of retrofitting. We will need to develop the capacity to translate our information , data, and experience into stories that are designed to affect the public conversation. Perhaps most important, we will need to move beyond fighting

Page 223: openev.debatecoaches.org€¦  · Web view1AC . Observation 1: If I could find the spot where truth echoesI would stand there and whisper memories of my children's future. I would

problems and on to framing solutions that bring us closer to our vision of how things should be. And then we must be committed to making it so.

3. Pragmatism-Their desire to seek a perfect conceptual understanding of power before reengaging the political makes the good the enemy of the perfect, something people of color can not afford in this moment of crisis. Our call allows for politics combined with critical analysis of power relations making us the perfect middle ground Yancy 1 (George Yancy and Cornel West, "Cornel West", p 70) *we do not endorse ableist language

The final component of the political construal of prophetic pragmatism deals with the issue of power. Here, West announces that prophetic pragmatism will unmask the different power structures that present themselves as natural. Along with this, he Ervors a genealogical strategy that will similarly unearth the relevant practices and systems of beliefs inappropriately dependent upon questionable moral values. Confessing to his Foucauldian connections, he states that “Prophetic pragmatism shares with Foucault a preoccupation with the operation of powers. It also incorporates the genealogical mode of

inquiry .... [Plrophetic pragmatism promotes genealogical materialist modes of analysis similar in many respects to those of Foucault.”3" West finally construes prophetic pragmatism as a form of tragic thought. although not a “doomsday” form of thinking. To the extent that prophetic pragmatism evades involvement with a priori epistemological concerns but instead focuses on the everyday concerns of human beings, West cautions that. despite our efforts to better our existential condition, both personal and social evil are persistent realities not immediately submissive to our intentions to do good. Evil’s evasion of the good that we seek to do will continue. West's focus on the presence of evil is not an invitation to concede defeat but rather one to force us to become humble as we realize that our best intentions are not immune to failure. Prophetic pragmatism . according to West, “is a form of trage thought in that it conlionts candidly individual and collective experiences of evil in individuals and institutions - with little expectation of ridding the world of all evil.” 3' As with his other construals. West links his conception of prophetic pragmatism, as a fomi of tragic thought, with democracy. What he seems to be getting at here is the idea that it is misleading to assume that we can totally and completely

realize the absolute ideal of democracy as a historically manifested reality. Democracy remains an ideal that we must constantly pursue while realizing that we are not pursuing an illusion. The dynamics of human existence and the dialectical tensions between human being and the natural world continually give rise to various obstacles and challenges as we seek a more socially enriching existence. Nevertheless, in acknowledging the constant struggle to improve things, West warns against surrendering to pessimism or yielding to the seductive idea of perfectibility. Hence, he maintains: Prophetic pragmatism denies Sisyphean pessimism and utopian perfectionism. Rather. it promotes the possibility of human progress and human impossibility of paradise. Human struggles its at the center of prophetic pragmatism, a struggle guided by a democratic and libertarian vision, sustained by moral courage and existential integrity, and tempered by the recognition of human finitude and fragility.” Clearly. West focuses persistently upon the importance of creating “new possibilities for human agency.” He even briefly describes his prophetic pragmatism as a form of historical consciousness attuned

to the struggles of the past as well as appreciative of the alternative forms of life based upon the best of the past. But he quickly interjects the theme of the tragic, stating that the praxis of prophetic pragmatism “is tragic action with revolutionary intent." 33 Finally, forever straddling two modes of thought at once, West reiterates that prophetic pragmatism is not blind to the fact that utopian schemes QIIIJOK escape the limitation presented by the unfortunate condition of individuals and the structures of injustice normally sustained by social, economic and political institutions. So, even if prophetic pragmatism evades pessimism, it still cautiously embraces utopianism. According to West: Prophetic pragmatism. . . tempets its utopian impulse with a profound sense of the tragic character of life and history. This sense of the tragic

Page 224: openev.debatecoaches.org€¦  · Web view1AC . Observation 1: If I could find the spot where truth echoesI would stand there and whisper memories of my children's future. I would

highlightst be irreducible predicament of unique individuals who undergo dread, despair, disillusionment, disease, and death and institutional forms of oppression that dehumanize people.” Now that we have some understanding of \Vest's impressionistic conception of prophetic pragmatism. we can offer an evaluation of it. We should keep in mind, however, that thus task is complicated by West's preference for a method of reading that emphasizes moving the reader through passionate and motivational rhetoric rather than the reader through the dispassionate execution of arguments. Hence, in criticizing West, we must focus on the vision at the heart of his prophetic pragmatism and not treat it as a systematic body of truths.

It’s a voting issue - You fail to ground your discourse in materiality – you don’t defend anything and understand the impacts of your inaction – vote neg on presumption Rashad Evans writes while debating Casey Harrigan in 2012: Rashad Evans, 2012 11-20-2012 (http://www.rwesq.com/the-1nc/)

A quare approach to debate includes afro-pessimism. For instance, I am pessimistic that any of the solutions being floated around will meaningfully address anti-blackness. I am pessimistic of changes in MPJ, resolutional wording or tournament practices to address the attitudinal and structural (throwback) barriers to black participation in debate. A quare approach to debate recognizes the importance that color, sex and gender have in structuring our lives and producing differing results for different people . Disidentification is a way by which we can enact change at the structural level through localized acts of resistance . It is a process whereby we can work on and against the system. I started this process at Harvard with the

initiation of my affirmative action policy and have continued on facebook and my blog. This disidentification is important because it recognizes the material consequences of our actions and inactions. It also allows individuals utilize discourse in a subversive way. E. Patrick Johnson explains that: “Foucault himself acknowledges that discourse has the potential to disrupt power: “Discourses are not once and for all subservient to power or raised up against it, anymore than silences are. We must make allowances for complex and unstable process whereby discourse can be both an instrument and effect of power, but also a hindrance, a stumbling block, a point of resistance and a starting point for an opposing strategy. Discourse transmit and produces power; it reinforces it , but also undermines and exposes it, renders it fragile and makes it possible to thwart it. Although people of color, myself included, may not have theorized our lives in Foucault’s terms, we have used discourse in subversive ways because it was necessary for our survival. Failure to ground discourse in materiality is to privilege the position of those whose subjectivity and agency, outside the realm of

gender and sexuality, have never been subjugated.” Therefore, we must look look at the material consequences of our acts and inactions and account for the ways in which certain performances either maintain power or subvert power. In addition, we must account for stylistic differences. It is at the level of style that most black and queer

politics are enacted. In the debate space, stylistic differences is one of the most important ways that Black debaters enact their politics and resistance. Style is an important tool of survival for Black, queer and female

debaters. One cannot speak if she cannot speak with and from her own voice. Style is important and so too is the

value of using the debate space to theorize about life more generally. Debate is a homeplace for its participants. Theorizing about things like anti-blackness, humanism and feminism from within the debate space is important to teaching debaters how to theorize about this issues outside the debate space . We must always be able to be critical in those places where we are most familiar and most comfortable. That is why these discussions regarding debate are so important.

Important to a quare understanding of debate is the work done by Dr. Shanara Reid-Brinkley and her call that we pay specific

attention to how we produce our scholarship. Responsible argumentation should account for multiple forms of knowledge including (1) academic scholars, (2) organic intellectuals and (3) personal experience.

Responsible debaters also engage in knowledge production and not just knowledge consumption.

Page 225: openev.debatecoaches.org€¦  · Web view1AC . Observation 1: If I could find the spot where truth echoesI would stand there and whisper memories of my children's future. I would

This replicates the revolutionary hero who limits their analysis to one time and place instead of formulating strategy to take theory and create replicable solution in the community. This prevents any ability to turn theory to praxis Newton-1971 –Huey P. Newton- Black Capitalism Reanalysed- Huey P. Newton Reader- 227-228

*we do not endorse gendered language

We see then that power has a dual character and that we cannot simply identify and define phenomena without acting, for to do so is to become an armchair philosopher. And when Bobby and I left Merritt College to organize brothers on the block we did so because the college students were too content to sit around and analyze without acting. On the other hand, power includes action , for it is making phenomena perform in the desired manner. But action without thinking and theory is also incorrect. If the social forces at work in the community have not been correctly analyzed and defined, how can you control them in such a way that they act in a desired manner? So the Black Panther Party has always merged theory and practice in such a way as to serve the true interests of the community. In merging theory with practice we recognized that it was necessary to develop a theory which was valid for more than one time and place. We wanted to develop a system of which was good anywhere, thus it had to be rather abstract. Yet our theory would relate to a concrete analysis of concrete conditions so that our actions would always be relevant and profitable to the people. Yet, at the same time, it had to advance their thinking so that they would move toward a transformation of their situation of exploitation and oppression. We have always insisted on good theory and good practice, but we have not always been successful in carrying this through. When the Black Panther Party defected from the Black community; we became, for a while, revolutionary cultists. One of the primary characteristics of a revolutionary cultist is that he [they] despises everyone who has not reached his [their] level of consciousness, or the level of consciousness that he [they] thinks he has [they have] reached, instead of acting to bring the people to that level. In that way the revolutionary cultist becomes divided from the people, he [they] defects from the community. Instead of serving the people as a vanguard, he [they] becomes a hero. Heroes engage in very courageous actions sometimes, and they often make great sacrifices, including the supreme sacrifice, but they are still isolated from the people. Their courageous actions and sacrifices do not lead the people to a higher level of consciousness, nor do they produce fundamental changes in the exploitation and oppression of the people. A vanguard, however , will guide the people onto higher levels of consciousness and in that way bring them to the point where they will take sterner actions in their own interests and against those who continue to oppress them. As I've said previously; revolution is a process, not a conclusion. A true revolutionist will not only take courageous actions, he [they] will also try to advance the people in such a manner that they will transform their situation. That is, by delivering power to the people the true revolutionist will help them define the social phenomena in their community and lead them to the point where they will seize the time and make these phenomena act in a desired manner.

Page 226: openev.debatecoaches.org€¦  · Web view1AC . Observation 1: If I could find the spot where truth echoesI would stand there and whisper memories of my children's future. I would

2. Minority Participation-Adopting an ethereal view of power based on abstraction turns any attempts at political engagement because minorities NEED to have relatable political analysis. They make debate less relevant for people of color and lead to replicating the intellectual framework that is exclusionary and precludes revolutionary analysis, decreasing participation bell hooks 1990, black feminist and author, postmodern blackness

It is sadly ironic that the contemporary discourse which talks the most about heterogeneity , the decentered subject, declaring breakthroughs that allow recognition of otherness, still directs its critical voice primarily to a specialized audience , one that shares a common language rooted in the very master narratives it claims to challenge. If radical postmodernist thinking is to have a transformative impact then a critical break with the notion of "authority" as "mastery over" must not simply be a rhetorical device, it must be reflected in habits of being, including styles of writing as well as chosen subject matter. Third-world scholars, especially elites, and white critics who passively absorb white supremacist thinking, and therefore never notice or look at black people on the streets, at their jobs, who render us invisible with their gaze in all areas of daily life, are not likely to produce liberatory theory that will challenge racist domination, or to promote a breakdown in traditional ways of seeing and thinking about reality, ways of constructing aesthetic theory and practice. From a different standpoint Robert Storr makes a similar critique in the global issue of _Art in America_ when he asserts: To be sure,

much postmodernist critical inquiry has centered precisely on the issues of "difference" and "otherness." On the purely theoretical plane the exploration of these concepts has produced some important results, but in the absence of any sustained research into what artists of color and others outside the mainstream might be up to, such discussions become rootless instead of radical. Endless second guessing about the latent imperialism of intruding upon other cultures only compounded matters, preventing or excusing these theorists from investigating

what black, Hispanic, Asian and Native American artists were actually doing. Without adequate concrete knowledge of and contact with the non-white "other," white theorists may move in discursive theoretical directions that are threatening to and potentially disruptive of that critical practice which would support radical liberation struggle.

Page 227: openev.debatecoaches.org€¦  · Web view1AC . Observation 1: If I could find the spot where truth echoesI would stand there and whisper memories of my children's future. I would

3. Revolutionary education- Contextualizing analysis around material change is essential to provide political training. Ignoring politics only cedes the political to the right which UNIQULY hurts people of color. We must gain the tools to fight power in the halls of power on OUR termsThemba-Nixon 2k (makani, Changing the rules what public policy means for organizing Colorlines 3.2)

*we do not endorse ableist language

In essence, policies are the codification of power relationships and resource allocation. Policies are the rules of the world we live in. Changing the world means changing the rules. So, if organizing is about changing the rules and building power, how can organizing be separated from policies?

Can we really speak truth to power, fight the right, stop corporate abuses, or win racial justice without contesting the rules and the rulers, the policies and the policymakers? The answer is no-and double no for people of color. Today, racism subtly dominates nearly every aspect of policymaking. From ballot propositions to city funding priorities, policy is increasingly about the control, de-funding, and disfranchisement of communities of color. What Do We Stand For?

Take the public conversation about welfare reform, for example. Most of us know it isn't really about putting people to work. The right's message was framed around racial stereotypes of lazy, cheating "welfare queens" whose poverty was "cultural." But the new welfare policy was about moving billions of dollars in individual cash payments and direct services from welfare

recipients to other, more powerful, social actors. Many of us were too busy to tune into the welfare policy drama in Washington, only to find it washed up right on our doorsteps. Our members are suffering from workfare policies, new regulations, and cutoffs. Families who were barely getting by under the old rules are being pushed over the edge by the new

policies. Policy doesn't get more relevant than this. And so we got involved in policy-as defense. Yet we have to do more than block their punches. We have to start the fight with initiatives of our own. Those who do are finding offense a bit more fun than defense alone. Living wage ordinances, youth development initiatives, even gun control and alcohol and tobacco policies are finding their way onto the public agenda, thanks to focused community organizing that leverages power for community-driven initiatives. - Over 600 local policies have been passed to regulate the tobacco industry. Local coalitions have taken the lead by writing ordinances that address local problems and organizing broad support for them. - Nearly 100 gun control and violence prevention policies have been enacted since 1991. - Milwaukee, Boston, and Oakland are among the cities that have passed living wage ordinances: local laws that guarantee higher than minimum wages for workers, usually set as the minimum needed to keep a family of four above poverty. These are just a few of the examples that demonstrate how organizing for local policy advocacy has made inroads in areas where positive national policy had been stalled by conservatives. Increasingly, the local policy arena is where the action is and where activists are finding success. Of course, corporate interests-which are usually the target of these policies-are gearing up in defense. Tactics include front groups, economic pressure, and the tried and true: cold, hard cash. Despite these barriers, grassroots organizing can be very effective at the smaller scale of local politics. At the local level, we have greater access to elected officials and officials have a greater reliance on their constituents for reelection. For example, getting 400 people to show up at city hall in just about any city in the U.S. is quite impressive. On the other hand, 400 people at the state house or the Congress would have a less significant impact. Add to that the fact that all 400 people at city hall are usually

constituents, and the impact is even greater. Recent trends in government underscore the importance of local policy. Congress has enacted a series of measures devolv ing significant power to state and local government. Welfare, health care, and the regulation of food and drinking water safety are among the areas where states and localities now have greater rule. Devolution has some negative consequences to be sure. History has taught us that, for social services and civil rights in particular, the lack of clear federal standards and mechanisms for accountability lead to uneven enforcement and even discriminatory implementation of policies. Still, there are real opportunities for advancing progressive initiatives in this more localized environment. Greater local control can mean greater community power to shape and implement important social policies that were heretofore out of reach. To do so will require careful attention to the mechanics of local policymaking and a clear blueprint of what we stand for. Getting It

in Writing Much of the work of framing what we stand for takes place in the shaping of demands. By getting into the policy arena in a proactive manner, we can take our demands to the next level. Our demands can become law, with real consequences if the agreement is broken. After all the organizing, press work, and effort, a group should leave a decision maker with more than a handshake and his or her word. Of course, this work requires a certain amount of interaction with "the suits," as well as struggles with the bureaucracy, the technical language, and the all-too-common resistance by decision makers. Still , if it's worth demanding, it's worth having in writing- whether as

Page 228: openev.debatecoaches.org€¦  · Web view1AC . Observation 1: If I could find the spot where truth echoesI would stand there and whisper memories of my children's future. I would

law, regulation, or internal policy. From ballot initiatives on rent control to laws requiring worker protections, organizers are

leveraging their power into written policies that are making a real difference in their communities. Of course, policy work is just one tool in our organizing arsenal, but it is a tool we simply can't afford to ignore. Making policy work an integral part of organizing will require a certain amount of retrofitting. We will need to develop the capacity to translate our information , data, and experience into stories that are designed to affect the public conversation. Perhaps most important, we will need to move beyond fighting problems and on to framing solutions that bring us closer to our vision of how things should be. And then we must be committed to making it so.

Only a complete shattering of European philosophical framework through adopting a focus on material conditions can solveMarimba Ani. Yurugu: An African-Centered Critique of European Cultural Thought and Behavior. March 1994 Pg.329-331

It has been part of the posture of the moral philosophers of European culture to disavow cultural commitment, yet their work has contributed significantly to the survival and intensification of the rhetorical ethic-the hypocrisy and the deception that constitute a vital and definitive part of the content of European cultural imperial- ism-and, therefore, to nationalistic objectives. To begin with the Platonic-influenced utamawazo provides the theoretical basis for a conceptual ethics; an ethical system, the themes of which are considered to be valid, as long as they are consistent in terms of the logic of that system. What is "ethical" becomes what is "rational" and "logical." The most "ethical" statement is the purest abstraction. As Havelock correctly observes, the individual "thinking" psyche becomes the seat of morality and the individual's ability to act ethically is based on his [their] ability to think " rationally"; i.e., "abstractly." The result, again , is "talk." The European idea is that words divorced from action, feeling, commitment, from human involvement can them- selves be relevant to (and properly inform) human interaction-as long as they are part of a consistent syntax; an approved semantical system. This pursuit itself is an exercise in self-deception. Primary cultures are characterized by an "existential ethic" (Stanley Diamond) that is based on and refers to actual behavior. European culture gives rise to semantical systems and instead of being concerned with the inconsistency between "word" and "deed" (which could conceivably be the determinant of ethical behaVior), the moral philosophers are merely concerned with verbal and what they call "logical" inconsistency. One result of this characteristic of the culture is a tendency to make philosophers the most irrelevant of people and to effectively divorce their work from any decision- making capacity or role that in any way influences the ethical behavior of European peoples. What this tradition has done instead is to support the culture in its ability to use words without meaning, and to support Europeans in their quest to deceive others and themselves as well. The body of literature known as "ethical theory" has to a large degree been conducive to the growth of moral hypocrisy in European culture. It is the "liberal" academic tradition in contemporary European/ European American culture that uses the rhetorical ethic best to sup- port the objectives of European chauvinism. Ingeniously these theo- rists use the semantical systems of the moral philosophers, the "brotherhood" rhetoric of the Christian statement and empty abstractions like "humanitarianism" and "universalistic ethics" as evidence of the ideological commitments of the Europeans and therefore as indices of the nature of European culture. They are "critical," because they say that the imperialistic behavior of the European has represented a conflicting theme or "negative" tendency in

Page 229: openev.debatecoaches.org€¦  · Web view1AC . Observation 1: If I could find the spot where truth echoesI would stand there and whisper memories of my children's future. I would

European development . The result of their theories, however , is that they succeed in making the European responsible for everything- the "good" as well as the "bad"- and in the end the good far outweighs the bad and will, of course, triumph along with "reason." Norman F. Cantor provides an excellent example of the subtle chauvinism of the European liberal academician in his work on Western culture. He says, The new ethos of the late 1960's sought to restore to their central place in Western culture the religious, mystical, compassionate , imaginative, and altruistic ideals that had been tarnished or ignored by industrialism and secularism, by the mechanism and bureaucracy of modern life . 22 The new ethos had indigenous roots in some of the central cur- rents of the Western tradition-in Christian mysticism, in the Enlightenment's vision of a happy and peaceful world, in Romanticism's yearning of the union of self and nature and for the union of all individuals in the Absolute Spirit, in anarchism's faith in the spontaneous association of men in a harmonious community when freed from the brutality and oppression of the state, in Nietzsche's life-affirming ethic and Freud's revelation of the pri- macy of erotic impulses, and in the existential philosophy of Camus, Sartre, and Jaspers. 23 The trick is to "claim" ideas that have failed to influence the def- inition of the culture: because they do not fit in with the asili. In this way, \any critique of European ideology informed by a vision of the human that could only have been created either by a rejection of European value or in a culture qualitatively different from European culture itself becomes a "Western" product. And this argument (if "argued" at all) is made on the basis of values that were , for the European, never more than rhetoric! " Christian mysticism" becomes "Western," and the "Enlightenment's vision of a happy world" is not tarnished by the fact that this world was to be defined in terms of and controlled by European "progress." Cantor's characterization of "Western liberalism" is a perfect statement of what I have called the "rhetorical ethic." In the statement that follows, taken from the concluding paragraphs of his three-vol- ume work on European cultural history, Cantor claims, for the cul- ture, its most severe critics. Movements that would seek the destruction of what the West has meant are characterized as expres- sions of Western humanism and of Western ideals. This excerpt is evi- dence of the characteristic of European cultural nationalism that we are here delineating. This particular example is all the more signifi- cant because it represents a fairly recently published text, used to explain and interpret to the European-American college student, the nature and meaning of Western-European history: It is a pernicious misreading of history to identify Western civiliza- tion with the racism, imperialism, and capitalism of the late nine- teenth century. Even in their heyday, these attitudes and institutions were only one side of the Western world view and way of life. The destiny of Western civilization immeasurably transcends the mistakes of one era. The West has had its confusion, horror, and misery, its moments when anti-human doctrine have seemed on the verge of carrying all before them. But it is the glory of Western civilization that it has never stood still and has never neglected for long the quest for institutions that can contribute to the realization of human freedom. Soon its best minds have recalled the highest ideals of the classical and Christian traditions; they have inspired their contemporaries with the vision of a great age of beginning anew, of the establishment of God's kingdom on earth or a secular equivalent in their own time." Cantor concludes his panegyric with the assurance that the "great upheavals of the 1960's were collectively only manifestations of the age-old western tradition by which Western 'civilization' peri- odically 'renews itsel!.'" In this way he debunks the need for revolu- tion; and in fact "claims" the revolutionaries, who, he says, will ineVitably and happily be overshadowed by the "rationalists and moderates, II who have restructured the institutions of the past and redirected the ideas of the present. The result

Page 230: openev.debatecoaches.org€¦  · Web view1AC . Observation 1: If I could find the spot where truth echoesI would stand there and whisper memories of my children's future. I would

has never been perfect justice or absolute truth but sufficient justice and enough truth to satisfy the anxieties of the contemporary era while reestablishing the social peace and political order that the progress of civilization requires. 22 And so ends Cantor's historical study of the "genesis and des- tiny" of Western culture. With its greatest minds as the custodians of "civilization"-not just "European civilization." My interpretation of that history is quite different, as it is informed by an African-centered perspective and methodology. Cantor is concerned lest the students of the "new ethos" would "shatter" and irrevocably separate from what has historically been Western European culture. Our conclusion is that the European tradition must be "shattered" if a truly "new ethos" is to replace the old. This means a new utamaroho to fulfill a different asiii. But then centered in African interest I understand European culture to be identifi ed with anti-Africanism, the imperialistic pursuit, and with a denial of the human spirit ; whereas Cantor finds this identification "pernicious" and makes the claim that the "liberation of the human spirit" has been a "central current" in the Western tradition. Ultimately Cantor's objectives are chauvinistic. He is concerned with influencing students in such a way that they will act to maintain the "peace" and "order" necessary for the continuance of the European conception of "progress," i.e., the persistence of European power .

5. Pragmatism-Their desire to seek a perfect conceptual understanding of power before reengaging the political makes the good the enemy of the perfect, something people of color cannot afford. Our call allows for politics combined with critical analysis of power relations making us the perfect middle ground Yancy 1 (George Yancy and Cornel West, "Cornel West", p 70) *we do not endorse ableist language

The final component of the political construal of prophetic pragmatism deals with the issue of power. Here, West announces that prophetic pragmatism will unmask the different power structures that present themselves as natural. Along with this, he Ervors a genealogical strategy that will similarly unearth the relevant practices and systems of beliefs inappropriately dependent upon questionable moral values. Confessing to his Foucauldian connections, he states that “Prophetic pragmatism shares with Foucault a preoccupation with the operation of powers. It also incorporates the genealogical mode of

inquiry .... [Plrophetic pragmatism promotes genealogical materialist modes of analysis similar in many respects to those of Foucault.”3" West finally construes prophetic pragmatism as a form of tragic thought. although not a “doomsday” form of thinking. To the extent that prophetic pragmatism evades involvement with a priori epistemological concerns but instead focuses on the everyday concerns of human beings, West cautions that. despite our efforts to better our existential condition, both personal and social evil are persistent realities not immediately submissive to our intentions to do good. Evil’s evasion of the good that we seek to do will continue. West's focus on the presence of evil is not an invitation to concede defeat but rather one to force us to become humble as we realize that our best intentions are not immune to failure. Prophetic pragmatism . according to West, “is a form of trage thought in that it conlionts candidly individual and collective experiences of evil in individuals and institutions - with little expectation of ridding the world of all evil.” 3' As with his other construals. West links his conception of prophetic pragmatism, as a fomi of tragic thought, with democracy. What he seems to be getting at here is the idea that it is misleading to assume that we can totally and completely

realize the absolute ideal of democracy as a historically manifested reality. Democracy remains an ideal that we must constantly pursue while realizing that we are not pursuing an illusion. The dynamics of human existence and the dialectical tensions between human being and the natural world continually give rise to various

Page 231: openev.debatecoaches.org€¦  · Web view1AC . Observation 1: If I could find the spot where truth echoesI would stand there and whisper memories of my children's future. I would

obstacles and challenges as we seek a more socially enriching existence. Nevertheless, in acknowledging the constant struggle to improve things, West warns against surrendering to pessimism or yielding to the seductive idea of perfectibility. Hence, he maintains: Prophetic pragmatism denies Sisyphean pessimism and utopian perfectionism. Rather. it promotes the possibility of human progress and human impossibility of paradise. Human struggles its at the center of prophetic pragmatism, a struggle guided by a democratic and libertarian vision, sustained by moral courage and existential integrity, and tempered by the recognition of human finitude and fragility.” Clearly. West focuses persistently upon the importance of creating “new possibilities for human agency.” He even briefly describes his prophetic pragmatism as a form of historical consciousness attuned

to the struggles of the past as well as appreciative of the alternative forms of life based upon the best of the past. But he quickly interjects the theme of the tragic, stating that the praxis of prophetic pragmatism “is tragic action with revolutionary intent." 33 Finally, forever straddling two modes of thought at once, West reiterates that prophetic pragmatism is not blind to the fact that utopian schemes QIIIJOK escape the limitation presented by the unfortunate condition of individuals and the structures of injustice normally sustained by social, economic and political institutions. So, even if prophetic pragmatism evades pessimism, it still cautiously embraces utopianism. According to West: Prophetic pragmatism. . . tempets its utopian impulse with a profound sense of the tragic character of life and history. This sense of the tragic highlightst be irreducible predicament of unique individuals who undergo dread, despair, disillusionment, disease, and death and institutional forms of oppression that dehumanize people.” Now that we have some understanding of \Vest's impressionistic conception of prophetic pragmatism. we can offer an evaluation of it. We should keep in mind, however, that thus task is complicated by West's preference for a method of reading that emphasizes moving the reader through passionate and motivational rhetoric rather than the reader through the dispassionate execution of arguments. Hence, in criticizing West, we must focus on the vision at the heart of his prophetic pragmatism and not treat it as a systematic body of truths.

6. Switch Side DebatePreserving negative ground key to fairness for minorities in debate. You don’t have to suppress your identities to affirm, you must infuse your identity into public discourse and discussion – going negative solves all your offense.

Rashad Evans, 2012. 11-20-2012 (http://www.rwesq.com/the-1nc/)

I will accept this definition, but I think it requires some more refinement. In a practical sense, SSD, at a minimum, requires that the affirmative defend the resolution. For the most part, the negative is unconstrained (except by competition) and most debates in which SSD comes up the question is whether the affirmative team should be required to affirm the topic, including some defense of an action by the United States Federal Government (“USFG”). The essential question posed by SSD is whether the affirmative team should be required to defend the resolution even when they (1) fundamentally disagree with all or part of it and/or (2) are compelled to affirm other ideas because of their “personal convictions.” The answer to this question must be yes. First,

there are multiple ways to skin a cat and even more ways to affirm the resolution in the world of parametric debate because one need only affirm an interpretation of the resolution and not the resolution writ large. Therefore, there are rare instances in which there is no interpretation of the resolution that you cannot defend and these rare instances are most likely the product of lack of creativity and research. So in reality, the question is an even simpler one: should the affirmative team be required to defend an interpretation of the resolution. There are immense constitutive and instrumental benefits to SSD that cannot be otherwise obtained and therefore must be required. I ask that the judges: George, Justin and Kyla evaluate this debate holistically, which requires that they evaluate the benefits of SSD to the speaker (constitutive) and to society

(instrumental). This also requires that they exclude nothing in their analysis of this debate. The Constitutive Debaters benefit from SSD. It requires that they engage in topics that they may not otherwise engage. The resolution poses a unique

Page 232: openev.debatecoaches.org€¦  · Web view1AC . Observation 1: If I could find the spot where truth echoesI would stand there and whisper memories of my children's future. I would

question and the debate round is a unique time to answer that question. One is much more likely to consider their personal convictions outside of the debate round than they are to consider the resolution outside the debate round. Therefore, independently of whether one agrees with the resolution or not, there are benefits to defending an interpretation of the resolution in that it requires you to consider something you may not have otherwise considered. These thought expiriments create more informed and thoughtful debaters and citizens. SSD creates a unique opportunity for self-reflexivity. The idea is to let

your personal convictions guide you in the debate. The resolution doesn’t require that you suppress your identity or convictions, but is instead an opportunity to infuse that identity and those convictions into the resolution and public policy discussion. So, given who you are, what you believe and what you know what say you regarding energy production? What say you regarding democracy assistance? What say you regarding federal control in Indian Country? Therefore, the resolution is an opportunity for you to refine your personal convictions and put your identity in politics instead of relying exclusively on identity politics. SSD allows you to check your own privilege. Here, I will rely on my personal experience. I once participated in a 30 day creative challenge where the challenge leader would pose questions on facebook and the group would answer them in the comments section. One day he asked us to imagine we could be the other gender for 3 days and to discuss what would we do. I thought about this on the A-train from Harlem to Chelsea and again from Chelsea to Harlem and had no answer because I couldn’t imagine being a woman. I explained that as a gay man I don’t need to be a woman to have sex with a man,

I don’t want a baby in me and more importantly I don’t want the physical vulnerability of being a woman. This realization really forced me to reevaluate my previous jokes even that as a gay many I was ontologically indistinct from

a woman. I am not. I am gay and I am black…in many ways I am the the worlds bottom, but even I could would not want to occupy the body of a woman. That thought experiment taught me a lot about myself, privilege and the world. Sometimes, you have to take a walk in someone else’s stiletto to learn who you really are . Seriously CH, have you walked in stilettos? It will change your life.

SSD is both necessary and sufficient to ensure that you debate your personal convictions . Remember, if you are staunchly opposed to the resolution you get to lodge those complaints on the negative. Also,

remember that what is good for the goose is good for the gander and if you don’t have to debate the resolution then no one has to debate the resolution. This loosening of the restrictions on the resolution may mean that no one gets to debate anything they want. I mean lets face it, no one wants to be topical! There’s always a better version of your aff that isn’t topical. Every 2AC considers severance or intrinsicness because the resolution is flawed or insufficient. However, a true debater understands that they both have to debate on the affirmative and the negative and that in order to do either effectively there must be stability in the relationship between the affirmative and the negative. Therefore, committing yourself to the resolution is key preserving your own negative ground to engage in your critique of the resolution. As goes the resolution so goes your personal convictions about that resolution. The Instrumental SSD is the only way to organize a fair debate tournament. We must have a topic. There must be rules and allowing any and everyone go rogue because of their “personal convictions” it is a recipe for disaster. I don’t want to live and

debate in that world. There must be strict requirements on defending the resolution to allow for fair and predictable negative ground. By the way, protecting fair negative ground is easier and more important than all other concerns regarding fairness. All people of all convictions have to be negative at some point. Which means that the affirmative-negative dialiectic is the only intersectional analysis that can be performed because all people of all identities will eventually find themselves on the affirmative or negative side in debate. Preserving negative ground then preserves the ground of women, minorities and hippies. SSD ensures that we have all hands on deck in these extraordinary times. We need all perspectives and all peoples to engage in all debates. SSD facilitates this. Allowing students to pick their own topic because of their “personal convictions” may mean that important issues get ignored all together. I mean, what would happen if we let the students set the curriculum? Would they learn anything, ever? How many times have you heard these questions in a classroom: why do I have to learn this? Will this be on the exam? My major is X, why do I need to learn Y? Blah. Yes, students would almost always like to do what they want to do. If we allowed this where would we be as a country? Who will be there to tackle the important questions pertaining to immigration, energy production, the Arab Spring,

Page 233: openev.debatecoaches.org€¦  · Web view1AC . Observation 1: If I could find the spot where truth echoesI would stand there and whisper memories of my children's future. I would

etc? Engaging the resolution from your specific social location can cause others to change their views. There are several examples of creative interpretations of the resolution that have inspired folk to view the topic and life differently. This is when debate is at its best and

when it’s engaging in important knowledge production. Debate and debaters have the potential to infuse new arguments, ideas and perspectives into the resolution and into the broader discussion.

State engagement is key – Asian feminist jurisprudence is the best starting point to challenged white sexual imperialism – only by engaging macro-level institutions can solve the impact of the 1ACWoan 08 (Sunny Woan – J.D. Public Interest and Social Justice Law, emphasis in Critical Race Theory, Santa Clara University School of Law, “WHITE SEXUAL IMPERIALISM: A THEORY OF ASIAN FEMINIST JURISPRUDENCE’, 14 Wash. & Lee J. Civ. Rts. & Soc. Just. 275 (2008).Page 299-301, pg 275- 301)

The dominance approach to feminist theory frames the question of equality as "a question of the distribution of power., 20 7 Thus, gender equality, as a question of power, scrutinizes "male supremacy and female subordination.",20 8 Following this logic, racial equality scrutinizes White supremacy and non-White subordination. For the Asian woman at the intersection of gender and race, achieving equality means overthrowing not only male supremacy or White supremacy, but specifically White male supremacy . Since "sexuality appears as the interactive dynamic of gender as an inequality''2 09 and "aggression against those with less power is experienced as sexual pleasure, an entitlement of masculinity,, 210 it is the White male's sexual dominance over the Asian female which emerges as the source of inequality that the Asian female suffers. Moreover, for Asian feminist jurisprudence, "colonial and military domination are interwoven with sexual domination." 21 ' The Western military's involvement in Asia , both in colonial and neo-colonial history, has led to Asia's sex tourism industry .1 2 This is an industry where the buyers of bodies for sexual pleasure are predominantly White men and the sellers of their bodies for sexual pleasure are predominantly Asian women . No other fact or condition confirms the imbalanced power relations between the East and the West. This imbalance of power came from White men imperializing Asia2 13 and, in the course of conquest, the taking of Asian women's bodies as their spoils. 214 The pervasiveness of sexual objectification establishes in the minds of Westerners a stereotype of Asian women as hyper-sexualized , since their only utility to Westerners for centuries come from their sexual submission.215 B. Where Do We Go From Here? The U.S. recognizes the profound harms that the institution of slavery caused during the early parts of American history which still endure today. Yet what about imperialism? Students read of it from textbooks in neutral language. No sense of penance comes with the recounts of U.S. occupation in Asia. Considering the general trends of the Asian and diasporic Asian communities enumerated in this essay, chiefly, severe underreporting of violent crimes inflicted upon them and a lack of scholarship examining the role imperialism played in the subjugation of Asian women , it comes as no surprise that history, through America's eyes, would white-wash the imperialized experience Asians endure even well into this century. Asian men feel emasculated from the American media's portrayal of them as effeminate, and many Asian women's subconscious preference for dating White men over Asian men-a trend which has become increasingly popular. White men display the "Asian fetish" syndrome, a symptom of not only the desire for male dominance, but also the imported stereotype that Asian women want to be dominated. The mail-order bride industry flourishes, capitalizing on the "Asian fetish." Then, the overrepresentation of Asian women in pornography perpetuate the entire cycle of White sexual

Page 234: openev.debatecoaches.org€¦  · Web view1AC . Observation 1: If I could find the spot where truth echoesI would stand there and whisper memories of my children's future. I would

imperialism as experienced by Asian women today. The action this Article calls for is humble, but significant: recognition. Recognize the pervasiveness of White sexual imperialism, understand its roots and where the branches pan out, and see how firmly implanted it is in the lives of those in the Asian community. The author asks for little more for now: merely recognition. " Oppressed groups need the

law," said Professor Cynthia 216 Bowman. Thus, recognition of White sexual imperialism begins with the law .V . Conclusion The Western world's desire for imperialistic domination over Asia relates to its desire for sexual domination over Asian women. In Asian feminist jurisprudence, the theoretical principle of White sexual imperialism explains the

inequality Asian and diasporic Asian women face today. Without first undermining the White sexual imperialist regime, Asian feminists cannot effectively achieve sexual-racial equality fo r Asian and diasporic A sian women . Without first undermining the White sexual imperialist regime, violent crimes against Asian victims will continue to be largely perpetrated by White men and, moreover, women of Asian descent will find no peace from the hyper-sexed stereotypes. Knowledge of social conditions as a vehicle for consciousness-raising shows women their situation in way that affirms they can change it.217 Thus, knowledge of White sexual imperialism as a vehicle for consciousness-raising shows Asian feminists just how deep they must dig to uproot the systemic inequality . While much literature has focused on sexual violence against White women, Blacks and Latinas, very few studies have looked at sexual violence against Asian women.218 This Article urges continued and active scholarship in Asian feminist jurisprudence, particularly through the fisheye lens of White sexual imperialism. Understanding intersectionality issues with the historical dimension of colonialism will help expose otherwise latent forces that work at conserving injurious sex and race disparities.

Operating within formal politics can challenge biopower—reform key to engagement Foucault, French Sociologist, 1988 (Michel, “On Criticism” in Michel Foucault: Politics Philosophy Culture Interviews and other writings 1977- 1984)

D.E. You mean it will be possible to work with this government? FOUCAULT: We must escape from the dilemma of being either for or against. After all, it is possible to face up to a government and remain standing. To work with a govern ment implies neither subjection nor total acceptance. One may work with it and yet be restive. I even believe that the two things go together. D.E. After Michel Foucault the critic, are we now going to see Michel Foucault the reformist? After all, the reproach was often made that the criticism made by intellectuals leads to nothing. FOUCAULT First I’ll answer the point about “that leads to nothing.” There are hundreds and thousands of people who have worked for the emergence of a number of problems that are now on the agenda. To say that this work produced nothing is quite wrong. Do you think that twenty years ago people were considering the problems of the relationship between mental illness and psychological normality, the problem of prison, the problem of medical power, the problem of the relationship between the sexes, and so on, as they are doing today? Furthermore, there are no reforms as such. Reforms are not produced in the air, independently of those who carry them out. One cannot not take account of those who will have the job of carrying out this transformation. And, then, above all, I believe that an opposition can be made between critique and transformation, “ideal” critique and “real” transformation. A critique is not a matter of saying that things are not right as they are. It is a matter of pointing out on what kinds of assumptions, what kinds of familiar, unchallenged, unconsidered modes of thought the practices that we accept rest. We must free ourselves from the sacrilization of the social as the only reality and stop regarding as superfluous something so essential in human life and in human relations as thought. Thought exists independently of systems and structures of discourse. It is something that is often hidden, but which always animates everyday

behavior. There is always a little thought even in the most stupid institutions; there is always thought even in silent habits. Criticism is a matter of flushing out that thought and trying to change it : to show that things are not as self-evident as

one believed, to see that what is accepted as self-evident will no longer be accepted as such. Practicing criticism is a matter of making facile gestures difficult. In these circumstances, criticism (and radical criticism) is absolutely indispensable for any transformation. A transformation that remains within the same mode of thought, a transformation that is only a way of adjusting the same thought more closely to the reality of things can merely be a superficial

transformation. On the other hand, as soon as one can no longer think things as one formerly thought them, transformation becomes both very urgent, very difficult, and quite possible. It is not therefore a question of there being a time for criticism and a time for transformation, nor people who do the criticism and others who do the transforming, those who are enclosed in an inaccessible radicalism and those who are forced to

make the necessary concessions to reality. In fact I think the work of deep transformation can only be carried out in a free atmosphere, one constantly agitated by a permanent criticism. D.E. But do you think the intellectual must have a

programmatic role in this transformation? FOUCAULT A reform is never only the result of a process in which there is

Page 235: openev.debatecoaches.org€¦  · Web view1AC . Observation 1: If I could find the spot where truth echoesI would stand there and whisper memories of my children's future. I would

conflict, confrontation, struggle, resistance To say to oneself at the outset: what reform will I be able to carry out? That is not, I believe, an aim for the intellectual to pursue. His role, since he works specifically in the realm of thought, is to see how far the liberation of thought can make those transformations urgent enough for people to want

to carry them out and difficult enough to carry out for them to be profoundly rooted in reality. It is a question of making conflicts more visible, of making them more essential than mere confrontations of interests or mere institutional immobility.

Out of these conflicts , these confrontations, a new power relation must emerge, whose first, temporary expression will be a reform. If at the base there has not been the work of thought upon itself and if, in fact, modes of thought, that is to say modes of action, have not been altered, whatever the project for reform, we know that it will be swamped, digested by modes of behavior and institutions that will always be the same.

Theory is irrelevant absent specific application – must combine theory and political actionFoucault ’82 [Michel, “Politics and Ethics: An Interview,” The Foucault Reader, Trans. Catherine Porter, Ed. Paul Rabinow, 373-4]

Q. There is much talk in America these days comparing your work to that of Jurgen Habermas. It has been suggested that your work is more concerned with ethics and his with politics. Habermas, for example, grew up reading Heidegger as a politically disastrous heir of Nietzsche. He associates Heidegger with German neo-conservatism. He thinks of these people as the conservative heirs of Nietzsche and of you as the anarchistic heir. You don't read the philosophical tradition this way at all, do you? M.F. That's right. When Habermas was in Paris, we talked at some length, and in fact I was quite struck by his observation of the extent to which the problem of Heidegger and of the political implications of Heidegger's thought was quite a pressing and important one for him. One thing he said to me

has left me musing, and it's something I'd like to mull over further. After explaining how Heidegger's thought indeed constituted a political disaster, he mentioned one of his professors who was a great Kantian, very well-known in the '30s, and he explained how astonished and disappointed he had been when, while looking through card catalogues one day, he found some

texts from around 1934 by this illustrious Kantian that were thoroughly Nazi in orientation. I have just recently had the same experience with Max Pohlenz, who heralded the universal values of Stoicism all his life. I came across a text of his from 1934 devoted to Fiihrertum in Stoicism. You should reread the introductory page and the book's closing remarks on the Fuhrersideal and on the true humanism constituted by the Volk under the inspiration of the leader's direction-Heidegger never wrote anything more disturbing. Nothing in this condemns Stoicism or Kantianism, needless to say. But I think that we must

reckon with several facts: there is a very tenuous "analytic" link between a philosophical conception and the concrete political attitude of someone who is appealing to it; the "best" theories do not constitute a very effective protection against disastrous political choices; certain great themes such as "humanism" can be used to any end whatever-for example, to show with what gratitude Pohlenz would have greeted Hitler. I do not conclude from this

that one may say just anything within the order of theory, but, on the contrary, that a demanding, prudent, "experimental" attitude is necesary ; at every moment , step by step, one must confront what one is thinking and saying with what one is doing, with what one is. I have never been too concerned about people who say: "You are bor-rowing ideas from Nietzsche; well,

Nietzsche was used by the Nazis, therefore. . ."; but, on the other hand, I have always been concerned with linking together as tightly as possible the historical and theoretical analysis of power relations , institu - tions , and knowledge, to the movements, critiques , and experiences that call them into question in reality. If I have insisted on all this "practice," it has not been in order to "apply" ideas, but in order to put them to the test and modify them. The key to the Personal poetic attitude of a philosopher is not to be sought in his ideas, as if it could be deduced from them, but rather in his philosophy-as-life, in his philosophicallife, his ethos. Among the French philosophers who participated in the Resistance during the war, one was Cavailles, a historian of mathematics who was interested in the development of its internal structures. None of the philosophers of engagement-Sartre, Simone de Beauvoir, Merleau-Ponty-none of them did a thing .

Policy engagement is vital in effectively challenging the excesses of neoliberalism - rejection is a form of utopianism that stifles progressive politics

Giroux and Polychroniou 14 (Henry and CJ, Global Television Network Chair in English and Cultural Studies at McMaster University + research associate and policy fellow at the Levy Economics Institute of Bard College, "The Specter of Authoritarianism and the Future of the Left: An Interview With Henry A. Giroux," 6/8, http://truth-out.org/news/item/24121-the-specter-of-authoritarianism-and-the-future-of-the-left-an-interview-with-henry-a-giroux)

They are the enemies of democracy and are crucial in creating subjectivities and values that buy into the notion that capital rather than people are the subject of history and that consuming is the only obligation of citizenship. Their goal is to normalize the ideologies, modes of governance and policies that reproduce massive inequities and suffering for the many, and exorbitant and dangerous privileges for the

corporate and financial elite. Moreover, such intellectuals are symptomic of the fact that neoliberalism represents a new historical conjuncture in which cultural institutions and political power have taken on a whole new life in shaping politics. What this suggests is that the left in its various registers has to create its own public intellectuals in higher education,

the alternative media and all of those spaces where meaning circulates. Intellectuals have a responsibility to connect their

Page 236: openev.debatecoaches.org€¦  · Web view1AC . Observation 1: If I could find the spot where truth echoesI would stand there and whisper memories of my children's future. I would

work to important social issues , work with popular movements and engage in the shaping of policies that benefit all people and not simply a few. At the heart of this suggestion is the need to recognize that ideas matter in the battle

against authoritarianism and that pedagogy must be central to any viable notion of politics and collective struggle. Public intellectuals have an obligation to work for global peace, individual freedom , care of others, economic justice and democratic participation, especially at a time of legitimized violence and tyranny. I completely agree with the late Pierre Bourdieu when he insisted that there is enormous political importance "to defend the possibility and necessity of the intellectual, who is firstly critical of the existing state of affairs. There is no genuine democracy without genuine opposing critical power." The very notion of being an engaged public intellectual is neither foreign to nor a violation of what it means to be an academic scholar, but central to its very definition. Put simply,

academics have a duty to enter into the public sphere unafraid to take positions and generate controversy, functioning as moral witnesses, raising political awareness and making connections to those elements of power and politics often hidden from public view. One final question. Are you optimistic about the future of the left and of progressive politics in general? It is impossible to be on the left and at the same time surrender to the normalization of a dystopian vision.

One has to be optimistic, but also realistic . This means that there is no room for a kind of

romanticized utopianism . Instead, one has to be motivated by a faith in the willingness of young people

principally to fight for a future in which dignity, equality and justice matter and at the same time recognize the forces that are preventing such a struggle. More specifically, hope has to be fed by the need for collective

action . Power is never completely on the side of domination and resistance is not a luxury but a

necessity . As Stanley Aronowitz has argued the left has to engage the issue of economic inequality, overcome its fragmentation,

develop an international social formation for radical democracy and the defense of the public good, undertake ways to finance itself, take

seriously the educative nature of politics and the need to change the way people think, and develop a comprehensive notion of

politics and a vision to match . History is open, though the gates are closing fast. The issue for me personally is not whether I am

pessimistic, but how am I going to use whatever intellectual resources I have to make it harder to prevent various events and problems from getting worse while at the same time struggling for a society in which the promise of democracy appears on the horizon of possibility.

Scholarship must engage the existing institution – wishing away policy discussion fails because oppression is institutionally entrenchedJones & Spicer ‘9

(Campbell, Senior Lecturer in the School of Management at U of Leicester, Andre, Associate Professor in the Dept of Industrial Relations @ Warwick Business School U of Warwick, Unmasking the Entrepreneur, pgs. 22-23)

The third strand in our proposed critical theory of entrepreneurship involves questions of the 'extra-discursive' factors that structure the context in which these discourses appear. The result of privileging language often results in losing sight of political and economic relations , and for this reason, a turn to language and a concomitant disavowal of things

extra-discursive have been roundly criticised (Ackroyd and Fleetwood, 2000; Armstrong, 2001; Reed, 1998,2000,2009). An analysis of discourse cannot alone account for the enduring social structures such as the state or capitalism . Mike

Reed has argued that a discursive approach to power relations effectively blinds critical theorists to issues of social structures: Foucauldian discourse analysis is largely restricted to a tactical and localised view of power , as constituted and expressed through situational-specific 'negotiated orders', which seriously underestimates the structural reality of more permanent and hierarchal power relations. It finds it difficult , if not impossible, to deal with institutionalised stabilities and continuities in power relations because it cannot get at the higher levels of social

organisation in which micro-level processes and practices are embedded. (Reed, 2000: 526-7) These institutional stabilities may

include market relations, the power of the state , relations like colonialism , kinship and patriarchy. These are

Page 237: openev.debatecoaches.org€¦  · Web view1AC . Observation 1: If I could find the spot where truth echoesI would stand there and whisper memories of my children's future. I would

the 'generative properties' that Reed (1998: 210) understands as 'mak( ing) social practices and forms - such as discursive formations - what they are and equip(ing) them with what they do'. Equally Thompson and Ackroyd also argue that in discourse analysis 'workers are not disciplined by the market, or sanctions actually or potentially invoked by capital, but their own

subjectivities' (1995: 627). The inability to examine structures such as capitalism means that some basic forms of power are thus uninvestigated. Focusing solely on entrepreneurship discourse within organisations and the workplace would

lead to a situation where pertinent relations that do not enter into discourse are taken to not exist. Such oversights in discursive analyses are that often structural relations such as class and the state have become so reified in social and

mental worlds that they disappear . An ironic outcome indeed. Even when this structural context is considered, it is often examined in broad , oversimplified , and underspecified manners . This attention to social structure can be an important part of developing a critical theory of entrepreneurship, as we remember that the existing structural arrangements at any point are

not inevitable, but can be subjected to criticism and change. In order to deal with these problems, we need to revive the concept of social structure. Thus we are arguing that 'there exist in the social world itself and not only within symbolic systems (language, myths, etc.) objective structures independent of the consciousness and will of agents, which are capable of guiding and constraining their practices or their representations' (Bourdieu, 1990: 122). Objective still means socially constructed, but social constructions that have become solidified as structures external to individual subjects . Examples of these structures may include basic 'organising principals' which are relatively stable and spatially and historically situated such as capitalism , kinship,

patriarchy and the state . Some entrepreneurship researchers, particularly those drawing on sociology and political science, have shown the importance of social structure for understanding entrepreneurship (see for example Swedberg, 2000).

Resistance over participation maintains the status quo – we need genuine political engagement to enact changeJeffrey Pyle 99—Boston College Law School, J.D., magna cum laude, “Race, Equality and the Rule of Law: Critical Race Theory's Attack on the Promises of Liberalism,” 40 B.C.L. Rev. 787, pgs 807-808//MMWang

For all their talk of "realism,"'" race-crits are strangely unrealistic in their proposals for reform. 1 m 7 Most probably realize

that radical measures like racial or ethnic reparations are not likely to be granted, especially by a court. But even unrealistic proposals are rare, because race-crits generally prefer not to suggest solutions, but to "resist" the dominant legal thought, doctrine and policy, whatever that happens to be. '" As Derrick Bell has put it, "most critical race theorists are committed to a program of scholarly resistance , and most hope scholarly resistance will lay the groundwork for wide-scale resistance."'" How this ivory tower oppositionalism would foment grassroots revolt is unclear , because CRT professors rarely suggest anything practical. Rather, their exhortations are meant, as Bell says, to "harass white folks" and • thereby "make life bearable in a society where blacks are a permanent, subordinate class."'" One of the race-erns' few practical programs of "resistance" is Paul Butler's proposal that inner-city juries practice racially-based jury nullification.' 91 jurors of color, Butler argues, have the "moral responsibility" not to apply the criminal law to blacks and whites equally, but to "etnancipate some guilty black outlaws" because "the black community" would be "better off" if there were fewer black men in prison.'" If enough juries were hung or not-guilty verdicts rendered, he imagines, the white-dominated government would change its excessive reliance on incarceration.'" Butler rejects the ordinary democratic process of legal reform.' Democracy, he says, ensures a "permanent, homogenous majority" of whites that "dominat[es]" African Ainericans.w5 Butler is probably correct that occasional acts of jury nullification might well express the resentment that many African Americans justifiably feel towards discriminatory law enforcement.'"`' As Randall Kennedy has pointed out, however, black Americans are disproportionately the victims of crimes,' 97 and therefore tend to favor more, not less, criminal

prosecution and punishment. 1 "8 The race-crits' preference for "resistance "' 99 over democratic participation seems to flow from a fear of losing their status as "oppositional scholars] "200 to the game of mainstream law and politics, which they regard as "an inevitably co-optive process?"' Better to be radically opposed to the "dominant political discourse""2 and remain an outsider than to work within the current system and lose one's "authenticity?" In rejecting the realistic for the "authentic," however, race- crits begin to look like academic poseurs —ideological purists striking the correct radical stance, but doing little within the confines of the real world, so sure are they that nothing much can be done."

Page 238: openev.debatecoaches.org€¦  · Web view1AC . Observation 1: If I could find the spot where truth echoesI would stand there and whisper memories of my children's future. I would

Must start from within dominant discourses—abstract criticisms of feminism fail to bring about real world changeSaloom , 6. JD Univ of Georgia School of Law and M.A. in Middle Eastern Studies from U of Chicago, Fall 2006

[Rachel, A Feminist Inquiry into International Law and International Relations, 12 Roger Williams U. L. Rev. 159, l/n, Stevens]

Tickner's last point that deserves further reflection is the notion that international law and international relations will not become free from gender bias as long as we live in a gendered world. This is not to say that small steps are ineffective, but rather that international law and international relations are merely a small part of the larger systemic problem of unequal gender relations. While it is desirable that more women occupy foreign and military policy making positions, this "desire" does not necessarily transform the way international law and international relations work. To allege that this is the case assume s that women have an essential character that can transform the system. This of course is contrary to the very arguments that most gender theorists forward, because it would mean that women have some unique "feminine" perspective. What is needed then is a release from the sole preoccupation on women and men. The state's masculinist nature that gender theorists critique affects everyone in society. Moving beyond the "add and stir" approach is quite difficult, but there must be a starting point from which gender theorists can work. 105 If everything is problematized, paralysis will inevitably occur. Working within the current framework is truly the only option to bring about change. Lofty abstract criticisms will do nothing to change the practices of international law and international relations. Pragmatic feminist criticisms of international law and international relations, however, should be further developed. Even advocates of realist thought will admit that realism is neither the most accurate nor the only way to view the world. 106 The changing dynamics of world politics make formulating new ways of understanding international relations quite pertinent. Keeping some semblance of realism in tact, while at the same time opening up space for theorizing about other possibilities, is necessary . Critics are quick to note that realism cannot be easily abandoned without some sort of alternative framework. Casting aside realism now, even given the concerns of gender scholars, is not the most promising option. Wayman and Diehl note that [*180] "the abandonment of realism leaves a void, which in the short to medium term is at least as much of a dead end as would be the result of following realism." 107 New possibilities can be envisioned while still adhering to some of the realist ideologies. Wayman and Diehl describe realism as a detour and not a definitive road map. 108 Thus, theorists must admit that realism is not the only way or the correct way to view international law and international relations, but it cannot be totally abandoned. Even given all of the criticisms of feminist theories, there must be space, however, for feminist theorization. A pragmatic approach should not dismiss the benefits of theorizing. Discussions and debates on feminism and international law and relations are extremely important. Yet even where feminist discourses lack the social power to realize their versions of knowledge in institutional practices, they can offer the discursive space from which the individual can resist dominant subject positions... . Resistance to the dominant at the level of the individual subject

Institutional configurations exist independently of and shape micro-politics

Wight 6 – Professor of IR @ University of Sydney – 6

(Colin, Agents, Structures and International Relations: Politics as Ontology, pgs. 48-50

One important aspect of this relational ontology is that these relations constitute our identity as social actors. According to this relational model of societies, one is what one is, by virtue of the relations within which one is embedded. A worker is only a worker by virtue of his/her relationship to his/her employer and vice versa. ‘Our social being is constituted by relations and our social acts presuppose them.’ At any

particular moment in time an individual may be implicated in all manner of relations , each exerting its own peculiar

causal effects. This ‘lattice-work’ of relations constitutes the structure of particular societies and endures despite changes in

the individuals occupying them . Thus, the relations, the structures, are ontologically distinct from the individuals who enter into

them. At a minimum, the social sciences are concerned with two distinct, although mutually interdependent, strata. There is an ontological difference between people and structures: ‘people are not relations, societies are not conscious agents’. Any attempt to explain one in terms of the other should be rejected. If there is an ontological difference between society and people, however, we need to elaborate on the relationship between them. Bhaskar argues that we need a system of mediating concepts, encompassing both aspects of the duality of praxis

into which active subjects must fit in order to reproduce it: that is, a system of concepts designating the ‘point of contact’ between human agency and social structures. This is known as a ‘ positioned practice’ system. In many respects, the idea

Page 239: openev.debatecoaches.org€¦  · Web view1AC . Observation 1: If I could find the spot where truth echoesI would stand there and whisper memories of my children's future. I would

of ‘positioned practice’ is very similar to Pierre Bourdieu’s notion of habitus. Bourdieu is primarily concerned with what individuals do in their daily lives. He is keen to refute the idea that social activity can be understood solely in terms of individual decision-making, or as determined by surpa-individual objective structures. Bourdieu’s notion of the habitus can be viewed as a bridge-building exercise across the explanatory gap between two extremes. Importantly, the notion of a habitus can only be understood in relation to the concept of a ‘social field’. According to

Bourdieu, a social field is ‘a network, or a configuration, of objective relations between positions objectively defined’. A social field, then,

refers to a structured system of social positions occupied by individuals and/or institutions – the nature of which defines the situation for their occupants . This is a social field whose form is constituted in terms of the relations which define it as a field of a certain type. A habitus (positioned practices) is a mediating link between individuals’ subjective worlds and the socio-cultural world into which they are born

and which they share with others. The power of the habitus derives from the thoughtlessness of habit and habituation, rather than

consciously learned rules. The habitus is imprinted and encoded in a socializing process that commences during early childhood. It is inculcated more by experience than by explicit teaching. Socially competent performances are produced as a matter of routine, without explicit reference to a body of codified knowledge , and without the actors necessarily knowing what they are doing (in the sense of being able adequately to explain what they are doing). As such, the habitus can be seen as the site of ‘internalization of reality and the externalization of internality.’ Thus social practices are produced in, and by, the encounter between: (1) the habitus and its dispositions; (2) the constraints and demands of the socio-cultural field to which the habitus is appropriate or within; and (3) the dispositions of the individual agents located within both the socio-cultural field and the habitus. When placed within Bhaskar’s stratified complex social ontology the model we have is as depicted

in Figure 1. The explanation of practices will require all three levels. Society , as field of relations, exists prior to, and is independent of, individual and collective understandings at any particular moment in time ; that is, social

action requires the conditions for action. Likewise, given that behavior is seemingly recurrent, patterned, ordered,

institutionalised, and displays a degree of stability over time, there must be sets of relations and rules that govern it. Contrary to individualist theory, these relations, rules and roles are not dependent upon either knowledge of them by particular individuals,

or the existence of actions by particular individuals; that is, their explanation cannot be reduced to consciousness or to

the attributes of individuals . These emergent social forms must possess emergent powers. This leads on to arguments for the reality of

society baswed on a causal criterion. Society, as opposed to the individuals that constitute it, is, as Foucault has put it, ‘a complex and independent reality that has its own laws and mechanisms of reaction, its regulations as well as its possibility of disturbance. This new reality is society…It becomes necessary to reflect upon it, upon its specific characteristics, its constants and its variables’.

Only political engagement can challenge a heteronormative society on a broad scale and effectuate changeBenjamin Bateman 2006, doctoral candidate in English at the University of Virginia, Spring 2006, “The Future of Queer Theory,” The Minnesota Review//MMWang

Queer theorists more politically programmatic than Edelman frequently neglect this point. Michael Warner, for example, accuses gays and lesbians who aspire to marriage of caving , in assimilationist fashion, to heterosexual norms perceived as demands. But queers never exist completely outside such norms—and thus cannot , logically, succumb to them—and marriage and childrearing might not look the same with gays on board. After all, gays who have been traumatized by their parents' homophobia and lessons of compulsory heterosexuality are probably less likely than their heterosexual counterparts to repeat such

mistakes. Insofar as married gays retain connections to less traditional elements of queer culture, we cannot assume that they will abandon their fights for sexual freedom, conform entirely to all matrimonial traditions, or turn their backs upon their promiscuous peers. Some might, but many will not. Edelman 's book works well as an intensely academic polemic but as a political resource it proves insufficient. If queer theory is to have a social impact, it must interpellate the gay and lesbian audience to whom , after all, it is primarily addressed. Few of these people, we can safely assume, want to live in a void or die Antigone's death. Queer culture should keep insisting that we not sacrifice present, pressing needs to heterosexual fantasies, but to secure its future it must imagine a political order in which the needs of children are not inimical to the interests of queers, and it must celebrate—as Eve Sedgwick does so passionately in "How to Grow Your Kids Up Gay"—that which is most queer, and queer-able , in children.

Page 240: openev.debatecoaches.org€¦  · Web view1AC . Observation 1: If I could find the spot where truth echoesI would stand there and whisper memories of my children's future. I would

Reformism is an effective method of change and may bring revolution closer Richard Delgado 2009, self-appointed Minority scholar, Chair of law at the U of Alabama law schoo, books have won eight national book prizes, including a Pulitzer nomination, teaching and writings focus on race, the legal profession, and social change, “Does Critical Legal Studies have what minorities want, arguing about law,” pgs 588-590//MMWang

2. The CLS critique of piecemeal reform Critical scholars reject the idea of piecemeal reform. Incremental change, they argue, merely postpones the wholesale reformation that must occur to create a decent society. Even worse, an unfair social system survives by using piecemeal reform to disguise and legitimize oppression. Those who control the system weaken resistance by pointing to the occasional concession to, or periodic court victory of, a black plaintiff or worker as evidence that the system is fair and just. In fact, Crits believe that teaching the common law or using the case method in law school is a disguised means of preaching incrementalism and thereby maintaining the current power structure.“ To avoid this, CLS scholars urge law professors to abandon the case method, give up the effort to find rationality

and order in the case law, and teach in an unabashedly political fashion. The CLS critique of piecemeal reform is familiar, imperialistic and wrong . Minorities know from bitter experience that occasional court victories do not mean the Promised Land is at hand. The critique is imperialistic in that it tells minorities and other oppressed peoples how they should interpret events affecting them. A court order directing a housing authority to disburse funds for heating in subsidized housing may postpone the revolution, or it may not. In the meantime, the order keeps a number of poor families warm. This may mean more to them than it does to a comfortable academic working in a warm office. It smacks of paternalism to assert that the possibility of revolution later outweighs the certainty of heat now , unless there is evidence for that possibility. The Crits do not offer such evidence. Indeed, some incremental changes may bring revolutionary changes closer , not push them further away. Not all small reforms induce complacency; some may whet the appetite for further combat.

The welfare family may hold a tenants‘ union meeting in their heated living room. CLS scholars‘ critique of piecemeal reform often misses these possibilities, and neglects the question of whether total change, when it comes, will be what we want.

Project of infiltration—universalist prescriptions that isolate ourselves from the institutions that exercise power militates against revolutionary movementsWilliams 69

[Summer 1969, Robert F. Williams was a civil rights leader and author, best known for serving as president of the Monroe, North Carolina chapter of the NAACP in the 1950s and early 1960s. Black Panther Party founder Huey Newton cited Williams’s Negroes with Guns as a major inspiration. “The Deprived: Rebellion in the Streets”, The Crusader, Volume 10, Number 02, http://freedomarchives.org/Documents/Finder/DOC513_scans/Robert_F_Williams/513.Crusader.Vol.10.2.Summer.1969.pdf

INFILTRATE THE MANS INSTITUTIONS: Black youth should not commit the catastrophic error of seeing things simply in black and white . That is, of seeing things as all good or all bad . It is erroneous to think that one can isolate oneself completely from the institutions of a social and political system that exercises power over the environment in which he resides. Self-imposed and pre- mature isolation , initiated by the oppressed against the organs of a tyrannical establishment, militates against revolutionary move- ments dedicated to radical change. It is a grave error for militant and just-minded youth to reject struggle-serving opportunities to join the mans government services, police forces, armed forces, peace corps and vital organs of the power structure . Militants should become acquainted with the methods of the oppressor. Meaningful change can be more thoroughly effectuated by militant pressure from within as well as without. We can obtain invaluable know-how from the oppressor . Struggle is not all violence. Effective struggle requires tactics , plans, analysis and a highly sophisti- cated

Page 241: openev.debatecoaches.org€¦  · Web view1AC . Observation 1: If I could find the spot where truth echoesI would stand there and whisper memories of my children's future. I would

application of mental aptness. The forces of oppression and tyranny have perfected a highly articulate system of infiltration for undermining and frustrating the efforts of the oppressed in trying to upset the unjust

status quo. To a great extent, the power structure keeps itself informed as to the revolutionary activity of freedom fighters. With the threat of extermination looming menacingly before Black Americans, it is pressingly imperative that our people enter the vital organs of the establishment . FIGHT KANGAROOISM:

Inasmuch as the kangaroo court system constitutes a powerful defense arm of tyranny , extensive and vigorous educational work must be done among our people so that when they serve on jury duty they will not become tools of a legal system dedicated to railroading our people to concentration camps disguised as prisons. The kangaroo court system is being widely used to rid racist America of black militants , non-conformists and effective ghetto leadership . These so-called courts are not protecting the human and civil rights of our people; they are not dis- pensing even-handed justice, but are long- standing instruments of terror and intimidation. Black Americans must be inspired to display the same determination in safeguarding the human and civil rights of our oppressed people as white racists are to

legally lynch us. No matter how much rigmarole is dished out about black capitalism and minority enterprise, the hard cold fact remains that it is as difficult for a Black American militant to receive justice in America's tyrannical courts as it is for a camel to pass through the eye of a needle. Black people must be brought to see their duty as jurors as an opportunity to right legal wrongs not to perpetrate shameful obeisance to tyranny and racism . Youth should mount a campaign relative to this social evil that will by far ex- ceed the campaign of voter registration.

Critiquing existing security structures isn’t enough – political action is necessaryPinar Bilgin (Department of International Relations Bilkent University Ankara) 05 “Regional Security in the Middle East” p. 60-1

Admittedly, providing a critique of existing approaches to security , revealing those hidden assumptions and normative projects

embedded in Cold War Security Studies, is only a first step. In other words, from a critical security perspective, self-reflection, thinking and writing are not enough in themselves. They should be compounded by other forms of practice (that is, action taken on the ground). It is indeed crucial for students of critical approaches to re-think security in both theory and practice by pointing to possibilities for change immanent in world politics and suggesting emancipatory practices if it is going to fulfil the promise of becoming a 'force of change' in world politics. Cognisant of the need to find and suggest alternative practices to meet a broadened security agenda without adopting militarised or zero-sum thinking and practices, students of critical approaches to security have suggested the imagining, creation and nurturing of security communities as emancipatory practices (Booth 1994a; Booth and Vale 1997). Although Devetak's approach to the theory/practice relationship echoes critical approaches' conception of theory as a form of practice, the latter seeks to go further in

shaping global practices. The distinction Booth makes between 'thinking about thinking' and 'thinking about doing' grasps the difference between the two. Booth (1997:114) writes: Thinking about thinking is important, but, more urgently, so is thinking about doing…. Abstract ideas about emancipation will not suffice: it is important for Critical Security Studies to engage with the real by suggesting policies, agents, and sites of change, to help humankind, in whole and in part, to move away

from its structural wrongs. In this sense, providing a critique of existing approaches to security, revealing those hidden

assumptions and normative projects embedded in Cold War Security Studies, is only a first (albeit crucial) step . It is vital for the students of critical

approaches to re-think security in both theory and practice.

Complete rejection of the institutional logic of civil society crushes anti-white supremacy projectsKimberle Crenshaw 1988, law at UCLA, “Race, Reform, and Retrenchment: Transformation and Legitimation in Antidscrimination Law 101,” Harvard law review 1331//MMWang

The Critics' product is of limited utility to Blacks in its present form. The implications for Blacks of trashing liberal legal ideology are troubling, even though it may be proper to assail belief structures that obscure liberating possibilities. Trashing legal ideology seems to tell us repeatedly what has already been established - that legal discourse is unstable and relatively indeterminate. Furthermore, trashing offers no idea of how to avoid the negative

consequences of engaging in reformist discourse or how to work around such consequences. Even if we imagine the wrong world when we think in terms of legal discourse, we must nevertheless exist in a present world where legal

Page 242: openev.debatecoaches.org€¦  · Web view1AC . Observation 1: If I could find the spot where truth echoesI would stand there and whisper memories of my children's future. I would

protection has at times been a blessing - albeit a mixed one. The fundamental problem is that, although Critics criticize law because it functions to legitimate existing institutional arrangements, it is precisely this legitimating function that has made law receptive to

certain demands in this area. The Critical emphasis on deconstruction as the vehicle for liberation leads to the conclusion that engaging in legal discourse should be avoided because it reinforces not only the discourse itself but also the society and the world that it embodies . Yet Critics offer little beyond this observation. Their focus on delegitimating rights rhetoric seems to suggest that, once rights rhetoric has been discarded, there exists a more productive strategy for change, one which does not reinforce existing patterns of domination . Unfortunately, no such strategy has yet been articulated, and it is difficult to imagine that racial minorities will ever be able to discover one . As Frances Fox Piven and Richard Cloward

point out in their Tushnet observes: The conditions of the society define exactly what kind of rights-talk makes sense, and the sort of rights-talk that makes sense in turn defines what the society is. When someone objects to an act as a violation of a right, the ensuing dialogue either involves a claim that the challenged act is inconsistent with some "deeper" commitments that the actor has . . . or deals with what kind of society we ought to have excellent account of the civil rights movement, popular struggles are a reflection of institutionally determined logic and a challenge to that logic . 137 People can only demand change in ways that reflect the logic of the institutions that they are challenging .138 Demands for change that do

not reflect the institutional logic - that is, demands that do not engage and subsequently reinforce the dominant ideology - will probably be ineffective . 139 The possibility for ideological change is created through the very process of legitimation , which is triggered by crisis. Powerless people can sometimes trigger such a crisis by challenging an institution internally , that is, by using its own logic against it . 140 Such crisis occurs when powerless people force open and politicize a contradiction between the dominant ideology and their reality. The political conse- 137 See id. at 22-25. The observation concerning the inability to bring about change in some non-legitimating fashion does not, of course, rule out the possibility of

armed revolution. For most oppressed peoples , however, the costs of such a revolt are often too great . That

is, the oppressed cannot realistically hope to overcome the "coercive" components of hegemony . More

importantly, it is not clear that such a struggle, although superficially a clear radical challenge to the coercive force of the status

quo, would be a lesser reinforcement of the ideology of American society (i.e., the consensual components of

hegemony). Consequences of maintaining the contradictions may sometimes force an adjustment - an attempt to close the gap or to make things appear fair. 141 Yet, because the adjustment is triggered by the political consequences of the contradiction, circumstances will be adjusted only to the extent necessary to close the apparent contradiction. This approach to understanding legitimation and change is applicable to the civil rights movement . Because Blacks were challenging their exclusion from political society, the only claims that were likely to achieve recognition were those that reflected American society's institutional logic: legal rights ideology. Articulating their formal demands through legal rights ideology, civil rights protesters exposed a series of contradictions - the most important being the promised privileges of American citizenship and the practice of absolute racial subordination. Rather than using the contradictions to suggest that American citizenship was itself illegitimate or false, civil rights protestors proceeded as if American citizenship were real, and demanded to

exercise the "rights" that citizenship entailed. By seeking to restructure reality to reflect American mythology, Blacks relied upon and ultimately benefited from politically inspired efforts to resolve the contradictions by granting formal rights. Although it is the need to maintain legitimacy that presents powerless groups with the opportunity to wrest concessions from the dominant order, it is the very accomplishment of legitimacy that forecloses greater possibilities. In sum, the potential for change is both created and limited by legitimation.

Strategic resistance through institutional engagement can allow us to understand, expose, and destabilize whitenessJohn Powell 2005, Williams Chair in Civil Liberties and Civil Rights @ Moritz College of Law – Ohio State University, Executive Director @ Kirwan Institute for the Study of Race and Ethnicity, “Dreaming of a Self Beyond Whiteness and Isloation,” Washington University Journal of Law and Policy//MMWang

Page 243: openev.debatecoaches.org€¦  · Web view1AC . Observation 1: If I could find the spot where truth echoesI would stand there and whisper memories of my children's future. I would

I am not suggesting that the interrogation of whiteness and the privileges bound up in it is not useful. I have attempted to do some of that in another article,82 but these approaches are both incomplete and likely to be misdirected. Any particular focus on privilege as being something that can possibly be separated from whiteness is likely to leave the structure of whiteness in place with the reinscribing of a new arrangement

of privileges. Any focus on the deconstruction of whiteness through the individual intent of whites risks, as

Wiegman notes, “reproducing . . . the white male rebel as the . . . subject of antiracist struggle.”83 In our

increasingly complicated postmodern world, transgression is much more complicated than deciding “not to be white.” One could point to the Civil War and the Civil Rights Movement as two examples of the attempt to disturb white privilege without striking at its core. But how are we to strike at the core of white racial hierarchy? What I have been asserting throughout this article is that we

must address the ontological question. This is not a retreat from the possible or a retreat to the interior. It is not just race that is socially constructed; so is the self.84 We must better understand and address how this self is constructed and what maintains its attachment to whiteness . The self, and particularly the white self, has a history . Because the self has a history, it is constantly being made and remade. This process goes largely unnoticed and hides behind a veil of naturalness . But it would be a serious error to see this as only an internal undertaking. This is part of the myth of the individual subject, that the self is internal and private. We must expose the social nature of the

subject. This subject is not just held together by other subjects but also by our norms, practices and institutions . This subject is related to other subjects and to the world. Together this creates a context that the subject lives in both externally and internally . But this fix is never perfect. This context always denies some possibility that yearns for expression. This yearning itself is part of the hope. As we think about institutional arrangements, we must think about what they mean for routine expression and the experience of internal space. Unger reminds us that the way institutions are arranged will either mutilate or provide space for the emancipation of our being.85 For those of us with privilege, we must use those privileges we cannot reject to better understand, expose, and

destabilize the structures and cultural norms that support and reinscribe whiteness. We must raise the cost of maintaining whiteness by seeking strategic interventions that reduce racialized disparities across multiple areas, but still seek to better understand and problematize whiteness. We must begin to work for a new set of arrangements that will support a new way of relating, a new way of being . So, part of this answer is in the material world, the arrangements of structures and institutions—not only because we need to address material needs and disparities, but because

structures are not separate from our self. But we must keep an eye on the self that we are trying to call into being. Without working on the interiorization of whiteness, we simply cannot solve the problem of whiteness . There has been some development in this region, but I believe that the ontological question of whiteness remains largely undertheorized.86

The purpose of this understanding is to end the performance of whiteness, not so that whites can be non-white or uncolored people, but so we can all be human with all our social amalgamation and complexity that has so long been denied and dreaded. Feminist theory has developed a view of the self as radically relational. It is this relational self that whiteness is created in, but fearful to acknowledge. How can we create communities of kinship that allow us to explore these connections?

Interrogation of power is not enough. We must engage in debates on how best to operationalize change to become effective advocates. If we win any risk of ground offense they can’t win any education claimsDonald S Lutz, 2k (Political Theory and Partisan Politics, pg 36-7)

The position argued here is that to the extent such a discussion between political theorists and politicians does not take place we damage the prospects for marrying justice with power. Since the hope of uniting justice with power was the reason for creating political philosophy in the first place, political theorists need to pursue the dialogue as part of what justifies their intellectual project. Politics is the realm of power. More specifically, i t is the realm where force and violence are replaced by debates and discussion about how to implement power. Without the meaningful injection of considerations of justice, politics tends to become discourse by the most powerful about how to implement their

Page 244: openev.debatecoaches.org€¦  · Web view1AC . Observation 1: If I could find the spot where truth echoesI would stand there and whisper memories of my children's future. I would

preferred regime. Although constitutionalism tends to be disparaged by contemporary political science, a constitution is the very place where justice and power are married. Aristotle first taught us that a constitution must be matched to the realities of the political system – the character, hopes, fears, needs and environment of the people – which requires that constitutionalism be addressed by men and women practiced in the art of the possible.

Aristotle also taught us that a constitution the politeia, or plan for a way of life should address the improvement of people toward the best life possible which requires that constitutionalism be addressed by political theorists who can hold out a vision of justice and the means for advancing toward it. The conversation between politician and political theorist stands at the center of their respective callings, and a constitution, even though it reflects only a part of the reality of a political system, has a special status in this central conversion. Although the focus of this chapter is on a direct conversation between theorist and politician, there is

an important, indirect aspect of the conversation that should not be overlooked – classroom teaching. Too often the conversation between politician and political theorist is described in terms of a direct one between philosophers and those holding power. Overlooked is the central need to educate as many young people as possible . Since it is difficult to predict who will, in fact, hold power, and because the various peoples who take seriously the marriage of justice with power are overwhelmingly committed to a non-elitist, broad involvement of the population, we should not overlook or minimize our importance as teachers of the many. Political leaders drawn from a people who do not understand what is at stake are neither inclined nor equipped to join the conversation. As we teach, we converse with future leaders. Perhaps not everyone who teaches political theory has had the same experience, but of the more than eight thousand students I have taught, I know of at least forty nine who later held a major elective office, and a least eighty more who have become important political activists. This comes down to about five students per teaching year , and I could not have predicted

which five it would be. The indeterminate future of any given student is one argument against directing our efforts at civic education forward the few, best students. A constitutional perspective suggests not only that those in power rely upon support and direction from a broad segment of the public, but also that reliance upon the successful civic education of the elite is not very effective, by itself for marrying justice with power in the long run

Close enough does not count in the context of the resolution. Black participation is uniquely key in nuclear debatesManning Marable. Director, Institute for Research in African-American Studies

Professor of History, Columbia University -1984. Speaking Truth to Power- Essays on Race, Resistance and Radicalism. Page 198-199

Black Americans also comprehend that peace is not the absence of conflict . As long as institutional racism, apartheid, and social class inequality exist, social tensions will erupt into confrontations. Most blacks recognize that peace is the realization of social justice and human dignity for all nations and historically oppressed peoples. Peace more than anything else is the recognition of the oneness of humanity. As Paul Robeson, the great black artist and activist, observed in his autobiographical work Here I Stand, "I learned that the essential character of a nation is determined not by the tipper classes, but by the common people, and that the Common people of all nations are truly brothers in the great family of mankind." Any people who experience generations of oppression gain an awareness of the innate commonalty of all human beings, despite their religions, ethnic, and political differences. In order to reverse the logic of the Cold War, white Americans must begin to view themselves as a distinct minority ill a world dominated by people of color. Peace between the superpowers is directly linked to the evolution of democratic rights, economic development, and social justice in the third world periphery. Black intellectuals, front W. E. B. Du Bois to the present, have also comprehended their unique role in the struggle for peace arid social justice. Cultural and intellectual activity for its is inseparable from politics. All art and aesthetics, scientific inquiry, and social studies are directly or indirectly linked to the material conditions of human beings, and the existing set of power relationships which dictates the policies of the modern state. When intellectual artists fail to combat racial or gender inequality, or the virus of anti-Semitism,

Page 245: openev.debatecoaches.org€¦  · Web view1AC . Observation 1: If I could find the spot where truth echoesI would stand there and whisper memories of my children's future. I would

their creative energies may indirectly contribute to the ideological justification for prejudice and social oppression. This is equally the case for the problem of war and peace. Through the bifurcation of our moral and social consciences against the cold abstractions of research and "value-free" social science, we may console ourselves by suggesting that we play role in the escalation of the Cold War political culture . By hesitating to dedicate ourselves and our work to the pursuit of peace and social justice, we inevitably contribute to the dynamics of national chauvinism, Militarism, and perhaps set the ideological basis necessary for World War III. Paul Robeson, during the Spanish Civil War, expressed the perspective of the black Peace tradition as a passionate belief in humanity: "Every artist, every scientist must decide, now, where he [they] stand s , life has no alternative. There are no impartial observers. The commitment to contest public dogmas, the recognition that we share with the Soviet people a Community of social, economic, and cultural interests, force the intellectual into the terrain of ideological debate. If we fail to do so, and if the peace consensus of black America remains isolated from the electoral mainstream , the results may be the termination of humanity itself.

This education is the best way to rupture systems of power. Revolutions must be trained to infiltration institutions of power. This is the ONLY possible strategy in a world on face of the slow genocide of people of colorWilliams 1970 [Robert F., civil rights leader, promoter of self defense, interviewed by The Black Scholar, “Interviews,” The Black Scholar Volume 1 Number 7] Williams: It is erroneous to think that one can isolate oneself completely from institutions of a social and political system that exercises power over the environment in which he resides. Self-imposed and premature isolation , initiated by the oppressed against the organs of a tyrannical establishment, militates against revolutionary movements dedicated to radical change. It is a grave error for militant and just minded youth to reject struggle-serving opportunities to join the man's [humanity’s] government and the services, police forces, peace corps and vital organs of the power structure. Militants should become acquainted with the methods of the oppressor. Meaningful change can be more thoroughly effectuated by militant pressure from within as well as without. We can obtain valuable know-how from the oppressor. Struggle is not all violence. Effective struggle requires tactics, plans, analysis and a highly sophisticated application of mental aptness. The forces of oppression and tyranny have perfected highly articulate systems of infiltration for undermining and frustrating the efforts of the oppressed in trying to upset the unjust status quo. To a great extent, the power structure keeps itself informed as to the revolutionary activity of freedom fighters. With the looming threat of extermination looming menacingly before black Americans, it is pressingly imperative that our people enter the vital organs of the establishment . Infiltrate the man's [power’s] institutions.

Page 246: openev.debatecoaches.org€¦  · Web view1AC . Observation 1: If I could find the spot where truth echoesI would stand there and whisper memories of my children's future. I would

Western philosophical ethics ignores the material realities of people of color with devastating consequence. This outweighs any of their impacts SISKANNA NAYNAHA - DOCTOR OF PHILOSOPHY -2006 - RACE OF ANGELS: XICANISMA, POSTCOLONIAL PASSIONS, AND RHETORICS OF REACTION AND REVOLUTION- WASHINGTON STATE UNIVERSITY- Department of English May 2006 – Online- http://74.125.155.132/search?q=cache%3A8oZmjXnNUpEJ%3Ahttps%3A%2F%2Fresearch.wsulibs.wsu.edu%3A8443%2Fdspace%2Fbitstream%2F2376%2F492%2F1%2Fs_naynaha_050306.pdf+%22postmodern%22%22ivory+tower%22%22material+reality%22%22people+of+color%22%22racism%22&hl=en&gl=us

At the turn of the twenty-first century, Laclau entered into a critical discussion with cultural theorist Judith Butler and Slovenian psychoanalyst and theorist Slavoj Žižek in their collaborative Contingency, Hegemony, Universality: Contemporary Dialogues in the Left. Using the theoretical lenses provided by Gramsci, Derrida, and Lacan, the three debate the failure of the Left in contemporary politics and, if there has indeed been a failure, its causes. The major contribution of this work to the field of democratic theory is the ways the authors problematize current watchwords deployed in the cultural rhetoric of US democracy such as “multicultural,” “pluralistic,” and “politically correct.” Ultimately, however , the theorists of so-called radical democracy tend to become bogged down in discussions of the discursivity of democracy; what they neglect here is the material, economic realities of poor people of color in the US and around the globe. Kalyan K. Sanyal elaborates on this critique in his “Postmarxism and the Third World: A Critical Response to the Radical Democratic Agenda.” “By emphasizing the discourse of the right,” he argues, the radical democrats link their multiple struggles to the state because it is the state that endows every citizen with right, and the process of realization of the right must refer to the state rather than to any other form of collectivity . . . [but] what are the implications of the radical democratic agenda for the global order, economic and political? (128) In the end Sanyal finds that the implications are devastating . The most salient is that the “Third World” “has to bear a large part of the cost of accommodating rights in the [First World]. To the extent that these rights impinge on the logic of profit and accumulation, capital has a tendency to move to greener pastures in the Third World where such rights hardly exist” (128). In fact, the rhetoric of democracy in the US has grown out of a Eurocentric obsession with “Western” foundations which inspire and perpetuate an obsessive possessive individualism through constant appeals to Enlightenment era thinking and ideals. The deployment of such rhetoric has long obscured the problems of racism, poverty, patriarchal oppression, and heterosexism within the US, and now, given the global expansion of US domination under the logic of late capitalism, that same rhetoric of democracy obfuscates the historical and material realities of US colonialism and imperialism around the world. Debates about individual “rights” and appropriate “procedures” rage on in the US while suffering rages on in poor neighborhoods populated disproportionately by people of color in the US and enrages the devastated Two-Thirds World. It is a travesty that demands a sustained intervention, one that historicizes thepolitical and economic dimensions of the rhetoric of democracy in the US.

Page 247: openev.debatecoaches.org€¦  · Web view1AC . Observation 1: If I could find the spot where truth echoesI would stand there and whisper memories of my children's future. I would

They cannot solve systems of inequality unless we have a political framework contextualizing change through institutions. We must ignore the false choice between revolution and piecemeal reform. The alternative is political isolation creating a hopelessness that is the worst enemy of the oppressedWood 2k (Mark David Wood- MARK david wood is an assistant professor jointly appointed in religious studies and African American studies at Virginia Commonwealth University - "Cornel West and the politics of prophetic pragmatism", p.88 90)

In subsequent publications, however, West did not pursue the working class, internationalist, anticapitalist, anti-imperialist revolutionary socialist theory and politics he elaborated and argued for in his pre-Evasion works. Between the publication of Prophesy Deliverance and Race Matters in 1993, West developed (and has continued to develop) a progressive pragmatist theory and politics. Rather than continue to develop the radical project bequeathed to him by Martin Luther King Ir. and Malcolm X,West has followed a post-Marxist path of theoretical and political development . Following this path involved, above all else, marginalizing and in some cases jettisoning the Marxist theory of class, class struggle, and socialist transformation. Although many academicians made race, class, and gender their conceptual scaffolding, class realities (e.g., exploitation) received the least attention, and sometimes none at all (see Wood 1986; Eagleton 1996). Academicians theorized racism, ethnocentrism, nationalism, sexism. and heterosexism as being fundamentally the consequence of ideological and institutional forces (e.g., racist discourses and the legal system, respectively) but not of capitalist property relations. They thus proposed ending these oppressive phenomena by transforming ideologies and institutions, not by transforming the system of property relations that these super structural forces regulate and reproduce. So, for example, West contends in Race Matters that the “major enemy of black survival in America has been and is neither oppression nor exploitation but rather the nihilistic threat- that is, the loss of hope and absence of meaning” (1993:15). This problem can be “tamed [only] by love and care,” that is, by “a politics of conversion," by “a turning of one’s soul [and] one’s own affirmation of one’s own worth-an affirmation fueled by concern for others” (18, 19).’ Alliteratively, in The War against Parents, he and Hewlett argue that CEO greed, not private ownership of the means of production and global competition for profits, is the fundamental cause of inequalities (Hewlett and West 1998:6\, 87). Having rejected the Marxist analysis he advanced in Prophesy Deliverance and Prophetic Fragments, West no longer seeks to replace private with public control of “the major institutions that regulate |workers'] lives"; rather, he now wants to ensure that “the grid of class, gender, and race weigh less heavily upon our life in common" and to increase “the ability of the determined individual to climb the ladder of class distinctions” (West 1982I1l4; Unger and West 1998:6o, 2.4). indeed, the contrast between the theory and politics of his pre-Evasion works and those of his post-Evasion writings could not be more striking. As West writes in Race Matters, his post-Marxist politics seeks not to build an international movement against corporate-controlled globalization but to save America from racial division, moral dissolution, and political balkanization and to establish “a freer, more efficient, and stable America” (1993a:7). Rather than argue for developing a counter hegemonic working class movement to turn back the business-class assault, Hewlett and West urge us to empower heterosexual parents to raise their children well so as to “increase America‘s store of human capital and help this nation compete with the Germans and Koreans” (1998:93). Rather than propose revolutionizing a system that West previously described as “anti-democratic to the core,” he and Unger

Page 248: openev.debatecoaches.org€¦  · Web view1AC . Observation 1: If I could find the spot where truth echoesI would stand there and whisper memories of my children's future. I would

argue that our overarching goal should be “to democratize the American economy and reenergize American democracy" (1998:93). Not only has West come to accept private property-"the market" and “free enterprise”-as an inevitable fact of life, but he argues explicitly against what he now calls the “paralyzing” belief, recommended by European theories like Marxism, in the existence of a system outtier “capitalism.” for example-with its driving laws, its inner logic, and its indivisible unity. Either you change the whole system, or you merely try to soften its harsh effects through “reformism." This simple and sincere progressive fails to cross the threshold of questioning and reimagining American institutions, or crosses it only under pressure of extreme crisis and with the help of the collective anxieties and enthusiasms crisis generates. The believer in the idea of the “system” places the institutional arrangements of the country beyond effective reach of deliberate, piecemeal reconstruction. (Unger and West 1998-9-30). West is certainly a long way from his earlier claims that Marxism is essential to liberation in that it clarifies in “an extremely clear and convincing way" the causes of racism and what must be done to fight capitalist imperialism and to build a genuinely humane, free, and democratic society. A progressivism that incorporates “the truth about political possibility," as West indicates his now does. rejects “the simple contrast between governmental activism and free enterprise, not because it wants to have a little of each, but because it insists upon having more of both" (Unger and West 199857, 3). Unlike those who believe in a capitalist system “out there,” prophetic pragmatists, as members of a large progressive coalition, do not assume "primacy for one causal factor,” such as property relations, and are not committed to “any preordained historical agent, such as the working class” (West 1989: 2.35). Rather, they take the high road and “condemn oppression anywhere and everywhere,” promote “ an all-embracing democratic and libertarian moral vision, ” and “invite all people of goodwill both here and abroad to fight for an Emersonian culture of creative democracy" in order, ultimately to “alleviate the plight of the wretched of the earth” (135, 231, 235). Whereas the aim of West’s African American revolutionary Christianity was to build a socio-economic system based on “collective control of major institutions,” the aim of his prophetic pragmatism is to “revitalize our republic” and “democratize the market economy” (West t98z:n4; Hewlett and West 1998:258; Unger and West 1998:49). The goal is no longer to develop “a revolutionary praxis beyond capitalist civilization” but rather to improve conditions of labor, lei-sure, and life within the limits of capitalist property relations. Interestingly following West’s earlier Marxist analyses, which he now views as being based on a “ paralyzing " belief in a reified system, nothing could be more utopian.

Switch side debate requires you to affirm the resolution. Fluidity is the strategy of whiteness, able to mold and shape itself to avoid being identified and to remain invisible. Switch side debate is blackness – Conviction based debate is whiteness.Evans, 2012. Rashad. JD Penn. http://www.rwesq.com/the-3nr/ November 21

I agree with this argument. However, you have no warrant to support the claim that SSD prevents the pursuit of social justice. SSD best facilitates social justice. You have already dropped my argument that it protects negative ground and that protecting negative ground is essential to promoting social justice in debate . You also dropped my argument that allowing debaters to go rogue because of their “personal convictions” may mean that we never discuss issues pertaining to social justice because debaters may choose to avoid those issues all together . In short,

Page 249: openev.debatecoaches.org€¦  · Web view1AC . Observation 1: If I could find the spot where truth echoesI would stand there and whisper memories of my children's future. I would

“ conviction based debate” (“CBD”) is neither necessary nor sufficient to produce social justice . You lose this argument. CH says: Rashad has presented a defense of a “Black SSD” which allows individuals to “[engaged] the resolution from [their] specific social location”. This modification to the traditional practice of SSD, he claims, exposes debates to a broad range of topics, encourages reflexivity, forces students to “check their privilege”, and offers creative takes on the topic, all while preserving the “instrumental goal” of topic-based fairness in debate. However, none of the purported

benefits *justify* SSD. Rashad has to thread the needle: First, you dropped my pre-empt to this argument. SSD is Blackness; CBD is whiteness.

Therefore, Black SSD is not a modification but instead a clarification . Your ghetoization of debate that is engaging as opposed to dismissive highlights the problems with CBD and is my first voting issue in this debate. Encouraging debaters to engage the resolution from their social location is not a Black thing; it’s just a thing. It recognizes that white people have a social location as well and that they must begin to situate themselves as merely part of the picture and not the whole picture. In other words, SSD or Black SSD as you

call it, forces you to realize that your view comes from somewhere and is not just a view from nowhere .

Lastly, this argument was gay turned in the 1NC. SSD requires debaters to speak on issues they may not otherwise consider. In an increasingly diverse debate community with increased Black participation in particular, SSD takes on an even more significant meaning as it is a unique opportunity to incorporate Black thought into debate and policy making. This incorporation solves the fundamental problem with all debate: it’s too white, too male and too straight. Again, debate infused by Black and queer thought is the solution to the ills you identify. Your argument has been turned.

Their aff is oppression tourism- engaging with suffering only to think about it versus engaging it. This allows the right to control the institutions of power and political discourse around differenceBrennan 2006 - Timothy Brennan is a professor of English and comparative literature at the University of Minnesota. (Timothy Brennan, "Wars of position", p 26-8)

In an aesthetic rather than political register , theory articulates dissidence and compliance simultaneously. At least one reason why this precarious balance can be maintained is found in the prevailing conditions of the postwar United States in which a fissure has opened up between the site of contestation embodied by the law and unwritten directives governing acceptable dissent. We have all grown accustomed, for example, to the assumption that the U.S. Constitution protects citizens against discrimination on the basis of “race, color, and creed." In fact, unlike the UN’s Universal Declaration of Human Rights, there are no explicit protections for freedom of conscience in the U.S. Constitution apart from article 1, which protects against “prohibiting the free exercise of religion." This has resulted in a series of negative pressures on North American intellectual life. While free speech in the abstract is protected by law, the safeguards never specify the content of acceptable speech, leaving all with the (false) impression that no restrictions are tolerated. Such abstraction does not render free speech meaningless, but it hides its partial character. Freedom of expression is, in this respect, fatally abstract and for that reason severely restricted without being meaningless. One clear institutional exercise of free expression has been the policy of affirmative action, which attempts to allow doing to take place freely, to not be foreclosed by a certain perception of being. On the other hand, affirmative action relies on a shift from identity-as-position to identity-as-location. So, many still stubbornly assume, for example, that American-based scholars whose family background is Lebanese, Nigerian, or Mexican are born specialists on Lebanon, Nigeria, or Mexico. Entire careers, not just the speakers’ lists of conferences or magazine roundtables, operate with a brittle confidence in this questionable homology between knowledge and being, analytical depth and sumame. A cellular epistemology has for several decades covered the thinking and (more important) the policymaking of academic reformers and liberal politicians as well as that of public commentators and mainstream news organizations, thereby opening itself up to a reactionary attack that has won favor with

Page 250: openev.debatecoaches.org€¦  · Web view1AC . Observation 1: If I could find the spot where truth echoesI would stand there and whisper memories of my children's future. I would

sectors of the public. In spite of the crucial points of dissonance between the media and academic progressives on matters ranging from the value of cultural studies to the role of research in undergraduate humanities education, there is no disagreement about the subtle equation between black scholars and black thinking, if one can use such a term. I use it here, at any rate, with intentional irony to refer to that nervous overcompensation by well-intended administrators who still actually believe in what they pretend not to: namely, the radical and essential differences among people on the grounds of race, gender, and ethnicity. In this liberal sentiment there circulates an amalgam of similarly harmful but widespread assumptions by the very people at the cutting edge, supposedly, of curricular reform, affirmative action, and postcolonial sympathy. Such assumptions include the centrality of the English language in a world where only a global minority speaks it, about the vibrancy and longevity of foreign popular cultures provided they are already subsumed into the U.S. media and are therefore translatable, or about the seductions of the European literary canon-a position that must ignore the competitive aesthetic brilliance of, say, Caribbean music when placed alongside European literature or, indeed, Urdu literature when juxtaposed with its European counterparts. In both media and academic venues, the defense of culturally dissonant thinking is given permission to be only in the imaginary form of the nonwhite, the woman, or the diasporic scholar. Very much like the Californian tourist in Mali, the liberal critic cannot wait to return home to a more secure and inviting environment. Beliefs breed ways of being, and the intolerance toward some has less to do with scholarly refutations than with a palpable unease in the presence of lifestyles and worldviews that demand of the stranger a painful, and often impossible, cultural conversion. Outside the relative social fairness and moral conscience of the university, the key sites of opinion forming and decision making -Congress. the Washington talk shows. and somewhat less so, influential news feature and interview programs such as CNN Reports or The O’ Reilly Factor- are so overwhelmingly white (or so predictably toltenized) that equations between affirmative action and alternative values seem almost axiomatic. Nonetheless, a set of arguments against gender and racial oppression is firmly in place, even though those arguments have not achieved anything like racial or gender equality. There is a good deal of evidence, in fact, that the unethical is becoming the ethical, that trends in business and government are actually moving from de facto segregation to an open declaration that minority exclusion is justified.” This represents a shift in policy that mirrors the rise in anti-immigrant violence , the number of police brutality cases, recourse to the death penalty, and extraordinary campaigns by white supremacist organization s best exemplified, perhaps, by the wave of arson against Southern black churches in the late 1990’s, or the Post 9/11 government campaign against Arab men under the rubric of terrorism, or the utter silence regarding the fate of Palestinians at the hands of Israeli troops and settlers. Such campaigns and policy changes are not being enacted, however, without resistance, for the crucial reason that a firm rhetoric of racial and gender inclusions still remains a part of U.S. civic life. People still widely agree that genocide and racism are bad things, however seldom they are roused to protest them.

Page 251: openev.debatecoaches.org€¦  · Web view1AC . Observation 1: If I could find the spot where truth echoesI would stand there and whisper memories of my children's future. I would

2. Policy engagement is vital in effectively challenging institutions - rejection is a form of utopianism that stifles progressive politics

Giroux and Polychroniou 14 (Henry and CJ, Global Television Network Chair in English and Cultural Studies at McMaster University + research associate and policy fellow at the Levy Economics Institute of Bard College, "The Specter of Authoritarianism and the Future of the Left: An Interview With Henry A. Giroux," 6/8, http://truth-out.org/news/item/24121-the-specter-of-authoritarianism-and-the-future-of-the-left-an-interview-with-henry-a-giroux)

*we do not endorse ableist language

They are the enemies of democracy and are crucial in creating subjectivities and values that buy into the notion that capital rather than people are the subject of history and that consuming is the only obligation of citizenship. Their goal is to normalize the ideologies, modes of governance and policies that reproduce massive inequities and suffering for the many, and exorbitant and dangerous privileges for the

corporate and financial elite. Moreover, such intellectuals are symptomic of the fact that neoliberalism represents a new historical conjuncture in which cultural institutions and political power have taken on a whole new life in shaping politics. What this suggests is that the left in its various registers has to create its own public intellectuals in higher education,

the alternative media and all of those spaces where meaning circulates. Intellectuals have a responsibility to connect their work to important social issues , work with popular movements and engage in the shaping of policies that benefit all people and not simply a few. At the heart of this suggestion is the need to recognize that ideas matter in the battle

against authoritarianism and that pedagogy must be central to any viable notion of politics and collective struggle. Public intellectuals have an obligation to work for global peace, individual freedom , care of others, economic justice and democratic participation, especially at a time of legitimized violence and tyranny. I completely agree with the late Pierre Bourdieu when he insisted that there is enormous political importance "to defend the possibility and necessity of the intellectual, who is firstly critical of the existing state of affairs. There is no genuine democracy without genuine opposing critical power." The very notion of being an engaged public intellectual is neither foreign to nor a violation of what it means to be an academic scholar, but central to its very definition. Put simply,

academics have a duty to enter into the public sphere unafraid to take positions and generate controversy, functioning as moral witnesses, raising political awareness and making connections to those elements of power and politics often hidden from public view. One final question. Are you optimistic about the future of the left and of progressive politics in general? It is impossible to be on the left and at the same time surrender to the normalization of a dystopian vision.

One has to be optimistic, but also realistic . This means that there is no room for a kind of

romanticized utopianism . Instead, one has to be motivated by a faith in the willingness of young people

principally to fight for a future in which dignity, equality and justice matter and at the same time recognize the forces that are preventing such a struggle. More specifically, hope has to be fed by the need for collective

action . Power is never completely on the side of domination and resistance is not a luxury but a

necessity . As Stanley Aronowitz has argued the left has to engage the issue of economic inequality, overcome its fragmentation,

develop an international social formation for radical democracy and the defense of the public good, undertake ways to finance itself, take

seriously the educative nature of politics and the need to change the way people think, and develop a comprehensive notion of

politics and a vision to match . History is open, though the gates are closing fast. The issue for me personally is not whether I am

pessimistic, but how am I going to use whatever intellectual resources I have to make it harder to prevent various events and problems from getting worse while at the same time struggling for a society in which the promise of democracy appears on the horizon of possibility.

Page 252: openev.debatecoaches.org€¦  · Web view1AC . Observation 1: If I could find the spot where truth echoesI would stand there and whisper memories of my children's future. I would

Charity Cannibalism DA: The West sustains itself through the cannibalization of the misery of those in the Global South, turning it in;pl to a field for the mass production of violence until the point of the west’s total collapseBaudrillard 94 (Jean Baudrillard, greatest living philosopher, 1994, The Illusion of the End, pg. 66-70)

The end of history, being itself a catastrophe, can only be fuelled by catastrophe. Managing the end therefore becomes synonymous with the management of catastrophe. And, quite specifically, of that catastrophe which is the slow extermination of the rest of the world. We have long denounced the capitalistic, economic exploitation of the poverty of the 'other half of the world' [l'autre monde]. We must today denounce the moral and sentimental exploitation of that poverty - charity cannibalism being worse than oppressive violence. The extraction and humanitarian reprocessing of a destitution which has become the equivalent of oil deposits and gold mines. The extortion of the spectacle of poverty and, at the same time, of our charitable condescension: a worldwide appreciated surplus of fine sentiments and bad conscience. We should, in fact, see this not as the extraction of raw materials, but as a waste-reprocessing enterprise. Their destitution and our bad conscience are, in effect, all part of the waste-products of history - the main thing is to recycle them to produce a new energy source. We have here an escalation in the psychological balance of terror. World capitalist oppression is now merely the vehicle and alibi for this other, much more ferocious, form of moral predation. One might almost say, contrary to the Marxist analysis, that material exploitation is only there to extract that spiritual raw material that is the misery of peoples, which serves as psychological nourishment for the rich countries and media nourishment for our daily lives. The 'Fourth World' (we are no longer dealing with a 'developing' Third World) is once again beleaguered, this time as a catastrophe-bearing stratum. The West is whitewashed in the reprocessing of the rest of the world as waste and residue. And the white world repents and seeks absolution - it, too, the waste-product of its own history. The South is a natural producer of raw materials, the latest of which is catastrophe. The North, for its part, specializes in the reprocessing of raw materials and hence also in the reprocessing of catastrophe. Bloodsucking protection, humanitarian interference, Medecins sans frontieres, international solidarity, etc. The last phase of colonialism: the New Sentimental Order is merely the latest form of the New World Order. Other people's destitution becomes our adventure playground. Thus, the humanitarian offensive aimed at the Kurds - a show of repentance on the part of the Western powers after allowing Saddam Hussein to crush them - is in reality merely the second phase of the war, a phase in which charitable intervention finishes off the work of extermination. We are the consumers of the ever delightful spectacle of poverty and catastrophe, and of the moving spectacle of our own efforts to alleviate it (which, in fact, merely function to secure the conditions of reproduction of the catastrophe market); there, at least, in the order of moral profits, the Marxist analysis is wholly applicable: we see to it that extreme poverty is reproduced as a symbolic deposit, as a fuel essential to the moral and sentimental equilibrium of the West. In our defence, it might be said that this extreme poverty was largely of our own making and it is therefore normal that we should profit by it. There can be no finer proof that the distress of the rest of the world is at the root of Western power and that the spectacle of that distress is its crowning glory than the inauguration, on the roof of the Arche de la Defense, with a sumptuous buffet laid on by the Fondation des Droits de l'homme, of an exhibition of the finest photos of world poverty. Should we be surprised that spaces are set aside in the Arche d'Alliance* for universal suffering hallowed by caviar and

Page 253: openev.debatecoaches.org€¦  · Web view1AC . Observation 1: If I could find the spot where truth echoesI would stand there and whisper memories of my children's future. I would

champagne? Just as the economic crisis of the West will not be complete so long as it can still exploit the resources of the rest of the world, so the symbolic crisis will be complete only when it is no longer able to feed on the other half's human and natural catastrophes (Eastern Europe, the Gulf, the Kurds, Bangladesh, etc.). We need this drug, which serves us as an aphrodisiac and hallucinogen. And the poor countries are the best suppliers - as, indeed, they are of other drugs. We provide them, through our media, with the means to exploit this paradoxical resource, just as we give them the means to exhaust their natural resources with our technologies. Our whole culture lives off this catastrophic cannibalism, relayed in cynical mode by the news media, and carried forward in moral mode by our humanitarian aid, which is a way of encouraging it and ensuring its continuity, just as economic aid is a strategy for perpetuating under-development. Up to now, the financial sacrifice has been compensated a hundredfold by the moral gain; But when the catastrophe market itself reaches crisis point, in accordance with the implacable logic of the market, when distress becomes scarce or the marginal returns on it fall from overexploitation, when we run out of disasters from elsewhere or when they can no longer be traded like coffee or other commodities, the West will be forced to produce its own catastrophe for itself, in order to meet its need for spectacle and that voracious appetite for symbols which characterizes it even more than its voracious appetite for food. It will reach the point where it devours itself. When we have finished sucking out the destiny of others, we shall have to invent one for ourselves. The Great Crash, the symbolic crash, will come in the end from us Westerners, but only when we are no longer able to feed on the hallucinogenic misery which comes to us from the other half of the world. Yet they do not seem keen to give up their monopoly. The Middle East, Bangladesh, black Africa and Latin America are really going flat out in the distress and catastrophe stakes, and thus in providing symbolic nourishment for the rich world. They might be said to be overdoing it: heaping earthquakes, floods, famines and ecological disasters one upon another, and finding the means to massacre each other most of the time. The 'disaster show' goes on without any let-up and our sacrificial debt to them far exceeds their economic debt. The misery with which they generously overwhelm us is something we shall never be able to repay. The sacrifices we offer in return are laughable (a tornado or two, a few tiny holocausts on the roads, the odd financial sacrifice) and, moreover, by some infernal logic, these work out as much greater gains for us, whereas our kindnesses have merely added to the natural catastrophes another one immeasurably worse: the demographic catastrophe, a veritable epidemic which we deplore each day in pictures. In short, there is such distortion between North and South, to the symbolic advantage of the South (a hundred thousand Iraqi dead against casualties numbered in tens on our side: in every case we are the losers), that one day everything will break down. One day, the West will break down if we are not soon washed clean of this shame, if an international congress of the poor countries does not very quickly decide to share out this symbolic privilege of misery and catastrophe. It is of course normal, since we refuse to allow the spread of nuclear weapons, that they should refuse to allow the spread of the catastrophe weapon. But it is not right that they should exert that monopoly indefinitely. In any case, the under-developed are only so by comparison with the Western system and its presumed success. In the light of its assumed failure, they are not under-developed at all. They are only so in terms of a dominant evolutionism which has always been the worst of colonial ideologies. The argument here is that there is a line of objective progress and everyone is supposed to pass through its various stages (we find the same eyewash with regard to the evolution of species and in that evolutionism which unilaterally sanctions the superiority of the human race). In the

Page 254: openev.debatecoaches.org€¦  · Web view1AC . Observation 1: If I could find the spot where truth echoesI would stand there and whisper memories of my children's future. I would

light of current upheavals, which put an end to any idea of history as a linear process, there are no longer either developed or under-developed peoples. Thus, to encourage hope of evolution - albeit by revolution - among the poor and to doom them, in keeping with the objective illusion of progress, to technological salvation is a criminal absurdity. In actual fact, it is their good fortune to be able to escape from evolution just at the point when we no longer know where it is leading. In any case, a majority of these peoples, including those of Eastern Europe, do not seem keen to enter this evolutionist modernity, and their weight in the balance is certainly no small factor in the West's repudiation of its own history, of its own utopias and its own modernity. It might be said that the routes of violence, historical or otherwise, are ·being turned around and that the viruses now pass from South to North, there being every chance that, five hundred years after America was conquered, 1992 and the end of the century will mark the comeback of the defeated and the sudden reversal of that modernity. The sense of pride is no longer on the side of wealth but of poverty, of those who - fortunately for them - have nothing to repent, and may indeed glory in being privileged in terms of catastrophes. Admittedly, this is a privilege they could hardly renounce, even if they wished to, but natural disasters merely reinforce the sense of guilt felt towards them by the wealthy - by those whom God visibly scorns since he no longer even strikes them down. One day it will be the Whites themselves who will give up their whiteness. It is a good bet that repentance will reach its highest pitch with the five-hundredth anniversary of the conquest of the Americas. We are going to have to lift the curse of the defeated - but symbolically victorious - peoples, which is insinuating itself five hundred years later, by way of repentance, into the heart of the white race.

Mourning requires the assimilation and cannibalism of the Other – memory and mourning of the Other instrumentalize and incorporate the Other within the self. Instead, we should refuse to mourn the Other as a testament to absolute alterity. Reynolds, 4 [Jack, Professor of Philosophy at the University of Tasmania, Possible and Impossible, Self and Other, and the Reversibility of Merleau-Ponty and Derrida, Spring 2004, Philosophy Today, Vol. 48, No. 1]

Moreover, in his essay "Fors: The Anglish Words of Nicolas Abraham and Maria Torok," Derrida again considers two models of the type of encroachment between self and other that is regularly associated with mourning. Borrowing from post-Freudian theories of mourning, he posits (although later undermines) a difference between introjection, which is love for the other person in me, and incorporation, which involves retaining the other person as a pocket, or a foreign body within one's own body. For Freud, as well as for the psychologists Abraham and Torok whose work Derrida considers, successful mourning is primarily about the introjection of the other person, and where they are consumed within us. The preservation of a discrete and separate other inside the self, as is the case in incorporation, is considered to be where mourning ceases to be a "normal" response and instead becomes pathological. Typically, Derrida reverses this hierarchy and order of Subordination by highlighting that there is a sense in which the supposedly pathological condition of incorporation is actually more respectful of the other's alterity. After all, incorporation means that one has not totally assimilated or digested the other, as there is still a difference and heterogeneity (EO 57). On the other hand, Abraham and Torok's "normal" mourning can be accused of interiorizing the other to such a

Page 255: openev.debatecoaches.org€¦  · Web view1AC . Observation 1: If I could find the spot where truth echoesI would stand there and whisper memories of my children's future. I would

degree that they have become assimilated and even metaphorically cannibalized .16 Derrida considers this introjection to be an infidelity to the other, although it will be argued that this type of mourning has more to recommend it than he allows. Indeed, it will be shown that Merleau-Ponty's notion of responsibility towards alterity is more "digestive" and appropriative, and it will also be argued that this is not necessarily a bad thing. Before we get ahead of ourselves, however, it is worth recognizing that Derrida's account is not so simple as to unreservedly valorize the incorporation of the other, even if he consistently emphasizes this paradigm in an effort to refute the canonical interpretation of successful mourning. he also acknowledges that the more the self "keeps the foreign element inside itself, the more it excludes it" (Fors xvii). Refusing to engage with the other, we exclude their foreignness from ourselves and hence prevent any transformative interaction with it. When fetishized in their externality in such a manner, the dead other really is lifeless and it is significant that Derrida describes the death of de Man in terms of the loss of exchange and of the transformational opportunities that he presented (MDM xvi).17 Derrida's point hence seems to be that in mourning we must oscillate between introjection and incorporation, but the "otherness of the other" nevertheless resists both the process of incorporation as well as the process of introjection. The other can neither be preserved as a foreign entity, nor introjected fully within. Instead, the other must always resist my memory and interiorization of them, and remains outside the grasp of subjectivity-that is, wholly other (tout autre). Towards the end of Memoires: for Paul de Man, Derrida suggests that responsibility towards alterity is precisely about respecting and even emphasizing this resistance of the other, as well as the concomitant impossibility of mourning (MDM 160, 238). It is on this point that he parts company with Merleau-Ponty, and there are again two main responses that Merleau-Ponty's work can help us furnish.

Page 256: openev.debatecoaches.org€¦  · Web view1AC . Observation 1: If I could find the spot where truth echoesI would stand there and whisper memories of my children's future. I would

PESSSAnti-blackness is a structural antagonism that undergirds political life. Their gesture towards non-Black bodies facilitates the expansion of antiblackness, colonialism, and ultimately sustains the ontological foundation and expansion of the current socioeconomic order. Dumas 16 [Michael J. Dumas, Assistant Professor at the University of California, Berkeley in the Graduate School of Education and the Department of African American Studies, “Against the Dark: Antiblackness in Education Policy and Discourse,” Theory Into Practice 55:11–19, 2016, published by The College of Education and Human Ecology, The Ohio State University]

Antiblackness is the central concern and proposition within an intellectual project known as Afro-pessimism.1 Afro-pessimism theorizes that Black people exist in a structurally antagonistic relationship with humanity . That is, the very technologies and imaginations that allow a social recognition of the humanness of others systematically exclude this possibility for the Black. The Black cannot be human , is not simply an Other but is other than human. Thus, antiblackness does not signify a mere racial conflict that might be resolved through organized political struggle and appeals to the state and to the citizenry for redress. Instead, antiblackness marks an irreconcilability between the Black and any sense of social or cultural regard . The aim of theorizing antiblackness is not to offer solutions to racial inequality, but to come to a deeper understanding of the Black condition within a context of utter

contempt for, and acceptance of violence against the Black. Afro-pessimist scholars contend that the Black is socially and culturally positioned as slave, dispossessed of human agency, desire, and freedom. This is not meant to suggest that Black people are currently

enslaved (by whites or by law), but that slavery marks the ontological position of Black people . Slavery is how Black existence is imagined and enacted upon, and how non-Black people—and particularly whites— assert their own right to freedom, and right to the consumption, destruction, and/or simple dismissal of the Black. “Through chattel slavery,” Frank Wilderson (2010) argued, the world gave birth and coherence to both its joys of domesticity and to its struggles of political discontent; and with these joys and struggles the Human was born, but not before it murdered the Black, forging a symbiosis between the political ontology of Humanity and the social death of Blacks. (pp. 20 – 21) This “social death” of the slave is introduced most explicitly in the work of Orlando Patterson (1982), who detailed how slavery involves a parasitic relationship between slave owner and slave, such that the freedom of the slave owner is only

secured and understood in relation to power over the slave. For Patterson, slavery is “the permanent, violent domination of natally alienated and generally dishonored persons” (p. 13). Although slavery involves personal relationships between groups, it also operates as an institutionalized system, maintained through social processes that make it impossible for the Slave to live, to be regarded as alive for her- or himself in the social world. This focus on slavery might seem anachronistic in the current historical moment, some 150 years after the (formal) end of the

institution in the United States. However, Wilderson maintained that the relations of power have not changed. He explained: Nothing remotely approaching claims successfully made on the state has come to pass. In other words, the election of a Black

president aside, police brutality, mass incarceration, segregated and substandard schools and housing , astronomical rates of HIV infection, and the threat of being turned away en masse at the polls still constitute the lived experience of Black life . (p. 10) This lived experience serves as a continual reinscribing of the nonhumanness of the Black , a legitimization of the very antiblackness that has motivated centuries of violence against Black bodies . In this sense, even as slavery is no longer official state policy and practice, the slave endures in the social imagination, and also in the everyday suffering experienced by Black people. As Saidiya Hartman (2007) insisted, Americans are living in what she described as “the afterlife of slavery:” Black lives are still imperiled and devalued by a racial calculus and a political arithmetic that were entrenched centuries ago. This is the afterlife of slavery—skewed life chances, limited access to health and

Page 257: openev.debatecoaches.org€¦  · Web view1AC . Observation 1: If I could find the spot where truth echoesI would stand there and whisper memories of my children's future. I would

education, premature death, incarceration, and impoverishment. I, too, am the afterlife of slavery. (p. 6) Importantly here, the afterlife of slavery is not only an historical moment, but deeply impressed upon Black flesh, in the embodiment of the Black person as slave. Thus, Hartman maintained, she is also this afterlife of slavery. Salamishah Tillet (2012) made clear the heaviness of the historical memory, the

everpresence of slavery in Black life: Because racial exclusion has become part and parcel of African American political identity since slavery, it cannot simply be willed or wished away. This protracted experience of disillusionment, mourning, and yearning is in fact the basis of African American civic estrangement. Its lingering is

not just a haunting of the past but is also a reminder of the present-day racial inequities that keep African American citizens in an indeterminate, unassimilable state as a racialized ‘Other.’ While the affect of racial melancholia was bred in the dyad of slavery and democracy, it persists because of the paradox of legal citizenship and civic estrangement. (p. 9) To the extent that there is ample evidence of the civic estrangement of Black people—their exclusion from the public sphere—one can theorize that the Black is still socially positioned as the slave, as difficult as it may be to use this frame to understand contemporary “race relations.” Here, “race relations” is necessarily in quotations because there is really no relation to be had between master and slave in the way one might conceptualize human

relationships. For Afro-pessimists, the Black is not only misrecognized, but unrecognizable as human, and therefore there is no social or political relationship to be fostered or restored . As Wilderson argued, Our analysis cannot be approached through the rubric of gains or reversals in struggles with the state and civil society, not unless and until the interlocutor first explains how the Slave is of the world. The onus is not on one who posits the Master/Slave dichotomy but on the one who argues there is a distinction between Slaveness and Blackness. How, when, and where did such a split occur? (p. 11) And this is the broader challenge posed by a

theory of antiblackness: There is no clear historical moment in which there was a break between slavery and

acknowledgement of Black citizenship and Human-ness; nor is there any indication of a clear disruption of the technologies of violence—that is, the institutional structures and social processes—that maintain Black subjugation. Thus, Afro-pessimists suggest that one must consider the Black as (still) incapable of asking for (civil or human) rights. This does not deny the long legacy of Black racial struggle, but it

positions this struggle as an impossibility, because the Black is (still) imagined outside of the citizenship that allows claims for redress to be regarded as legitimate, or even logical. Part of the challenge in theorizing blackness in contemporary race discourse is that Americans are living in an officially antiracist society, in which, as Jodi Melamed has documented, postWorld War II racial liberalisms and neoliberalisms make some space for the participation of multicultural subjects (Melamed, 2011). That is,

even as race continues to structure capitalism, which in turn facilitates white accumulation, the official stance of the state is against racism; blatantly racist laws and government practices have been declared illegal, and the market embraces outreach to a wide multicultural range of consumers. In this context, there is a rush to celebrate the social and economic advancement of select Black individuals and, perhaps more significantly, the success of other groups of people of color. In fact, it is the social and cultural inclusion of non-

Black people of color that is often offered as evidence of the end of racial animus and racial barriers in the society. Therefore, the failure of large swaths of the Black population is purported to be a result of cultural deficits within the Black . The slave, always suspected of being lazy and shiftless, now must bear primary responsibility for not making it in a society, which—officially, anyway

—thrives on multiracial harmony and civic participation. Jared Sexton (2008, 2010) contended that in this era, multiracialism thrives largely at the expense of, and firmly against, blackness. His argument rests on the premise that the color line is more fluid during periods in which Black freedom is thought to be most contained. Thus, during slavery in the United States, multiracial communities could serve as “buffer classes between whites and blacks” which often “corroborated and collaborated with antiblackness” (Sexton, 2008, p. 12). The

current period is marked by similar dynamics, with little organized Black political movement, resegregation of

neighborhoods and schools, and, in fact, an easy deterritorialization and gentrification of historic Black urban homeplaces. The current Black Lives Matter movement (Garza, 2014), which has emerged in the wake of so many cases of antiBlack violence,

may yet shift Americans into a period of heightened anxiety about Black bodies, but Sexton’s description of the

current period is valid: There is little fear of Black bodies and, arguably, an emboldened antipathy to the Black overall. This, in Sexton’s theorizing, opens up new spaces for multiracial inclusion. In this moment, the Black –white divide is seen as less consequential and not as much the result of white attitudes and behaviors . In these moments, Sexton maintained, the more significant boundary is the one constructed “between blackness and everything else” (2008, p. 13). And this is a boundary seemingly constructed and maintained by recalcitrant Black people against multiracialism, and more to the point,

multiracial progress. Multiracialism, in Sexton’s view, “premises its contribution to knowledge, culture and politics upon an evacuation of the historical richness , intellectual intensity, cultural expansiveness, and political complexity of Black experience, including, perhaps especially, its indelible terrors” (2008, p. 15). Transcending the Black-white binary , multiracialism ostensibly moves people past the narrowness and anachronism of blackness and toward a more profitable global economy and more sophisticated cultural milieu . Embracing non-

Page 258: openev.debatecoaches.org€¦  · Web view1AC . Observation 1: If I could find the spot where truth echoesI would stand there and whisper memories of my children's future. I would

Black bodies of color thus facilitates, and is facilitated by, antiblackness, and can be justified as antiracist precisely because it is inclusive of more than white. “The [B]lack body,” Lewis Gordon contended, “is confronted by the situation of its absence” (1997, p. 73). This absence—this social death or afterlife of/as the slave—positions Black people as the embodiment of problem, a thing rather than a people suffering from problems created by antiblackness. Part of the aim of Afropessimist scholarship is to insist on the humanity of Black people. “Those of us who seek to understand [B]lack people,” Gordon concluded, need to “bear in mind that [B]lack people are human beings” (p. 78). In an anti-Black world, this is easier said than done. In the end, there may be, as Wilderson suggested, no “roadmap to freedom so extensive it would free us from the epistemic air we breathe” (2010, p.

338). Even so, like Gordon, Wilderson suggested that theorizing antiblackness is important simply as an existential and political recognition of Black humanity, as a means “to say we must be free of air, while admitting to knowing no other source of breath” (p. 338; italics in original).

There is an ontological difference between slavery and colonialism – we must prefigure slavery in order to understand the ways in which whiteness has dispossessed the Savage – the concept of the law is irredeemable once properly understood for black agency is only ever conceivably legal when it is criminal – the struggle for indigenous liberation is always appropriative of black struggle in a violent analogization that erases black agencySexton 10 the curtain of the sky pg 14-7

That is to say, in the debate about the colonial policy of assimilation and its discontents, a debate in which Mannoni and Fanon

intervene respectively, it is slavery and the particular freedom struggle it engenders that mark the critical difference . Slavery : that which reduces ‘colonial peoples to a molten state’ uniquely enabling the metropolitan power ‘to pour them into a new mould’, a process in which ‘th e personality of the native is first destroyed through uprooting, enslavement, and the collapse of the social structure’ (Mannoni 1990: 27). For Mannoni, ‘assimilation is only practicable where an individual has been isolated from his group, wrenched from his environment and transplanted else- where’ (Mannoni 1990: 27, emphasis added). Fanon’s historical materialist redaction of Mannoni’s psychology of the colonial relation is to refuse the latter’s projection of the ‘affective disorders’ produced by colonization into a pre-colonial cultural eternity. Not so much, perhaps, because such projection would have the Malagasy desire her own colonizer (like the Inca who Mannoni suggests desires her own conquistador in an earlier historical period), but because the contradictions of colonization might provide an even more problematic recommendation for ‘the

introduction of slavery’ (Mannoni 1990: 27). To suffer the loss of political sovereignty, the exploitation of labor, the dispossession of land and resources is deplorable ; yet, we might say in this light that to suffer colonization is unenviable

unless one is enslaved. One may not be free, but one is at least not enslaved. More simply, we might say of the colonized: you may lose your motherland, but you will not ‘lose your mother’ (Hartman 2007). The latter condition, the ‘social death’ under which kinship is denied entirely by the force of law, is reserved for the ‘natal alienation’ and ‘genealogical isolation’

characterizing slavery. Here is Orlando Patterson, from his encyclopedic 1982 Slavery and Social Death: I prefer the term ‘natal alienation’ because it goes directly to the heart of what is critical in the slave’s forced alienation, the loss of ties of birth in both ascending

and descending generations. It also has the important nuance of a loss of native status, of deracination . It was this alienation of the slave from all formal, legally enforceable ties of ‘blood,’ and from any attachment to groups or localities other than those chosen for him [sic] by the master, that gave the relation of slavery its peculiar value to the master. The slave was the ultimate human tool, as imprintable and as disposable as the master wished. And this was true, at least in theory, of all slaves, no matter how elevated. (Patterson 1982: 7–8) True even if elevated by the income and formal education of the mythic American middle class, the celebrity of a Hollywood icon, or the political position of the so-called Leader of the Free World. 4 The alienation and isolation of the slave is not only vertical, canceling ties to past and future generations and rendering thereby the notion of ‘descen- dants of slaves’ as a strict

oxymoron. It is also a horizontal prohibition, canceling ties to the slave’s contemporaries as well. Reduced to a tool, the deracination of the slave, as Mannoni and Fanon each note in their turn, is total, more fundamental even than the displacement of the colonized, whose status obtains in a network of persecuted human relations rather than in a collection or dispersal of a class of things. Crucially, this total deracination is strictly correlative to the ‘absolute submission mandated by

Page 259: openev.debatecoaches.org€¦  · Web view1AC . Observation 1: If I could find the spot where truth echoesI would stand there and whisper memories of my children's future. I would

[slave] law’ discussed rigorously in Saidiya Hartman’s 1997 Scenes of Subjection: the slave estate is the most perfect example of the space of purely formal obedience defining the jurisdictional field of sovereignty (Agamben 2000).

Because the forced submission of the slave is absolute, any signs whatsoever of ‘reasoning … intent and rationality’ are recognized ‘solely in the context of criminal liability’. That is, ‘the slave’s will [is] acknowledged only as it [is] prohibited or punished’ (Hartman 1997: 82, emphasis added). A criminal will, a criminal reasoning, a criminal intent,

a criminal rationality: with these erstwhile human capacities construed as indices of culpability before the law, even the potentiality of

slave resistance is rendered illegitimate and illegible a priori. The disqualification of black resistance by the logic of racial slavery is not unrelated to the longstanding cross-racial phenomenon in which the white bourgeois and

proletarian revolutions on both sides of the Atlantic can allegorize themselves as revolts against slavery, while the

hemispheric black struggle against actually existing slavery cannot authorize itself literally in those same terms. The latter must code itself as the apotheosis of the French and American revolutions (with their themes of Judeo-Christian deliverance) or, later, the Russian and Chinese revolutions (with their themes of secular messianic trans- formation)

or, later still, the broad anti-colonial movements in Africa, Asia, and Latin America of the mid-20th century (with their themes of indigenous reclamation and renaissance). 5 One of the defining features of contemporary political and intellectual culture remains this metaphoric transfer that appropriates black suffering as the template for non-black grievances , while it misrecognizes the singularity of black struggles against racial slavery and what Loïc Wacquant calls its ‘functional surrogates’ or what Hartman terms its ‘afterlife’. Put differently, ‘the occult presence of racial slavery’ continues to haunt our political imagination: ‘nowhere, but nevertheless everywhere, a dead time which never arrives and does not stop arriving’ (Marriott 2007: xxi). Hartman’s notion of slavery’s afterlife and Wacquant’s theorization of slavery’s functional surrogates are two productive recent attempts to name the interminable terror of slavery, but we are still very much within the crisis of language – of thinking and feeling, seeing and hearing – that slavery provokes. Both scholars challenge the optimistic idea of a

residual ‘legacy’ of slavery, precisely because it requires the untenable demarcation of an historic end in Emancipation. The relations of slavery live on, Hartman might say, after the death knell of formal abolition, mutating into ‘the burdened individuality of freedom’.The functions of the chattel system are largely maintained,Wacquant might say, despite the efforts of Reconstruction, preserved in surrogate institutional form under Jim Crow, the ghetto, and the prison. Slavery lives on, it survives, despite the grand attempts on its institutional life forged by the international movements against slavery, segregation and mass imprisonment (Davis 2003). But what if slavery does not die, as it were, because it is immortal, but rather because it is non-mortal, because it has never lived, at least not in the psychic life of power? What if the source of slavery’s longevity is not its resilience in the face of opposition, but the obscurity of its existence? Not the accumulation of its political capital, but the illegibility of its grammar? On this account, for those that bear the mark of slavery – the trace of blackness – to speak is to sound off without foundation, to appear as a ghost on the threshold of the visible world, a spook retaining (only) the negative capacity to absent the presence,ornegatethewilltopresence,ofeveryclaimtohumanbeing,evenperhapsthefugi- tive movement of stolen life explored masterfully by Fred Moten (2008). We might rethink as well the very fruitful notion of ‘fugitive justice’ that shapes the prize-winning 2005

special issue of Representations on ‘Redress’. Co-editors Saidiya Hartman and Stephen Best are posing the right question: ‘How does one compensate for centuries of violence that have as their consequence the impossibility of restoring a prior existence, of giving back what was taken, of repairing what was broken? ’ (Hartman and Best 2005: 2) That is to say, they are thinking about ‘the question of slavery in terms of the incomplete nature of abolition’, ‘the contemporary predicament of freedom’ (2005: 5, emphasis added). Yet, the notion subsequently developed of a fugitive life ‘lived in loss’ – spanning the split difference between grievance and grief, remedy and redress, law and justice, hope and resignation – relies nonetheless on an outside, however

improbable or impossible, as the space of possibility, of movement, of life. Returning to our schematization of Fanon, we can say that the outside is a concept embedded in the problématique of colonization and its imaginary topography , indeed, the fact that it can imagine topographically at all. But, even if the freedom dreams of the black radical imagination do conjure images of place (and to do here does not imply that one can in either sense of the latter word:

able or permitted); what both the fact of blackness and the lived experience of the black name for us , in their

discrepant registers, is an anti-black world for which there is no outside . 6 ‘The language of race developed in the modern

period and in the context of the slave trade’ (Hartman 2007: 5). And if that context is our context and that context is the world, then this is the principal insight revealed by the contemporary predicament of freedom: there is no such thing as a fugitive slave. Malcolm X, by another route, was not far from this formulation in his famous ‘The Ballot or the Bullet’ address, delivered 3 April 1964 at the Cory Methodist Church in Cleveland, Ohio. Speaking to the risks of political confrontation with the structures of racial

domination, he exhorts: ‘If you go to jail, so what? If you black, you were born in jail. If you black, you were

Page 260: openev.debatecoaches.org€¦  · Web view1AC . Observation 1: If I could find the spot where truth echoesI would stand there and whisper memories of my children's future. I would

born in jail, in the North as well as the South. Stop talking about the South. Long as you south of the Canadian border, you’re south.’ For blacks in the USA, the political borders of the nation-state mark the walls of a social incarceration, a political ontology of race uninterrupted by ontic differences of region or legal standing. Of course, Malcolm X did not restrict his commentary to the USA , even if recent devel- opments in

national electoral politics were the focus of this particular address. His evolv- ing analysis accommodated a much larger geographical scale, what he elsewhere designated ‘white world supremacy’. But if there is any weight to his insistence that the Mason-

Dixon Line, demarcating the territories of a still unresolved civil war, or even the prison wall, constituting liberal democracy’s internal hard

edge, are incidental to black life – this from a former prisoner of over six years – should we not extend this reasoning to the ultimate penalty, the absolute master, and stop talking about death as the limit of black life? Not a loss (of life and limb, liberty and property), but a never having had. Not only the figurative ‘nothing to lose but your chains’ of the proletariat, but the literal inability to lose (because unable to own, to accumulate, to have and to hold, to self-possess) at all. Can’t have (even when we got), can’t be (even when we are): a strange freedom in the heart of slavery. ‘The political ontology of race’ is a phrase borrowed from work of political theorist Frank B. Wilderson, III, where it has been elaborated from his 2003 Social Identities article, ‘Gramsci’s Black Marx’, to his 2008 American Book Award-winning memoir, Incognegro: A Memoir of Exile and Apartheid, and his forthcoming Red,White, and Black: Cinema and the Structure of US Antagonisms. Drawing heavily upon Gordon and Fanon, alongside the early Patterson, the ongoing research of Wacquant on the four ‘peculiar institutions’ that have ‘operated to define, confine, and control African Americans in the history of the United States’ (Wacquant 2002: 41), and an array of noted literary critics and historians (e.g. David Eltis, Lindon Barrett, Saidiya Hartman, Ronald A.T. Judy, David Marriott, Hortense Spillers); Wilderson supplants the paradigm of comparative ethnic and racial studies in two principle ways. First, by moving conceptually from the empirical to the structural, especially insofar as the question of differential racialization – or the compli- cations of racial hierarchy – makes recourse to a comparative sociology, measuring relative rates of infant mortality, poverty, illiteracy, high school graduation, hate crimes, impris- onment, electoral participation, and so on. Second, by reframing racism (pace Fanon) as a social relationship that is grounded in anti-blackness rather than white supremacy. What Wilderson demonstrates at length is that ‘the racialization of the globe’ (Dikötter 2008) or the formation of the ‘world racial system’ (Winant 2002) does not adhere strictly to Du Bois’s thesis on the color line – ‘the relation of the darker to the lighter races of men [sic] in Asia and Africa, in America and the islands of the sea’ – in which ‘Negro slavery’ is referred to as but one ‘phase’ of a general problem. Rather, slavery establishes the vestibule of the category of the Human. To be sure, Humans do not live under con- ditions of equality

in the modern world. In fact, modernity is, to a large degree, marked by societies structured in dominance: patriarchy and white supremacy, settler colonialism and extra-territorial conquest, imperialist warfare and genocide ,

class struggle and the international division of labor. Yet, for Wilderson, there is a qualitative difference, an ontological one, between the inferiorization or dehumanization of the masses of people ‘in Asia … in America and the islands of the sea’,

including the colonization of their land and resources, the exploitation of their labor and even their extermination in whole or

in part, and the singular com- modification of human being pursued under racial slavery, that structure of gratuitous violence in which bodies are rendered as flesh to be accumulated and exchanged . 7 On this score, we should note that ‘the absolute submission mandated by law was not simply that of slave to his or her owner, but the submission of the enslaved before all whites’ (Hartman 1997: 83). The latter group is perhaps better termed all non-blacks (or the unequally arrayed category of non-blackness), because it is racial blackness as a necessary condition for enslavement that matters most, rather than whiteness as a sufficient condition for freedom. The structural position of the Indian slaveholder – or, for that matter, the smattering of free black slaveholders in the USA or the slaveholding mulatto elite in the Caribbean – is a case in point (Blackburn 1997; Koger 2006; Miles and Holland 2006). Freedom from the rule of slave law requires only that one be considered non-black, whether that non-black racial designation be ‘white’ or ‘Indian’ or, in the rare case, ‘Oriental’ – this despite the fact that each of these groups have at one point or another labored in conditions similar to or contiguous with enslaved African-derived groups. In other words, it is not labor relations, but property relations that are constitutive of slavery.

Institutional structures of domination create everyday holocausts—you should reject singular focused impacts in favor of working against the ongoing extinctions of people of color [international conflict impacts] Omolade 89, [1989, Barbara Omolade is a historian of black women for the past twenty years and an organizer in both the women’s and civil rights/black power movements, “We Speak for the Planet” in “Rocking the ship of state : toward a feminist peace politics”, pp. 172-176]

Recent efforts by Soviet leader Mikhail Gorbachev and President Ronald Reagan to limit nuclear testing, stockpiling, and weaponry, while still protecting their own arsenals and selling arms to countries and factions around the world, vividly demonstrate how "peace" can become an abstract concept within a

Page 261: openev.debatecoaches.org€¦  · Web view1AC . Observation 1: If I could find the spot where truth echoesI would stand there and whisper memories of my children's future. I would

culture of war. Many peace activists are similarly blind to the constant wars and threats of war being waged against people of color and the planet by those who march for "peace" and by those they march against. These pacifists , like Gorbachev and Reagan , frequently want people of color to fear what they fear and define peace as they define it . They are unmindful that our lands and peoples have already been and are being destroyed as part of the "final solution" of the "color line." It is difficult to persuade the remnants of Native American tribes , the starving of African deserts , and the victims of the Cambodian "killing fields" that nuclear war is the major danger to human life on the planet and that only a nuclear "winter" embodies fear and futurelessness for humanity . The peace movement suffers greatly from its lack of a historical and holistic perspective, practice, and vision that include the voices and experiences of people of color; the movement's goals and messages have therefore been easily coopted and expropriated by world leaders who share the same culture of racial dominance and arrogance. The peace movement's racist blinders have divorced peace from freedom, from feminism, from education reform, from legal rights, from human rights, from international alliances and friendships, from national liberation, from the particular (for example, black female, Native American male) and the general (human being). Nevertheless, social movements such as the civil rights-black power movement in the United States have always demanded peace with justice, with liberation, and with social and economic reconstruction and cultural

freedom at home and abroad. The integration of our past and our present holocausts and our struggle to define our own lives and have our basic needs met are at the core of the inseparable struggles for world peace and social betterme nt . The Achilles heel of the organized peace movement in this country has always been its whiteness. In this multi-racial and racist society, no allwhite movement can have the strength to bring about basic changes. It is axiomatic that basic changes do not occur in any society unless the people who are oppressed move to make them occur. In our society it is people of color who are the most oppressed. Indeed our entire history teaches us that when people of color have organized and struggled-most especially, because of their particular history, Black people-have moved in a more humane direction as a society, toward a better life for all people.1 Western man's whiteness, imagination, enlightened science, and movements toward peace have developed from a culture and history mobilized against women of color. The political advancements of white men have grown directly from the devastation and holocaust of people of color and our lands. This technological and material progress has been in direct proportion to the undevelopment of women of color. Yet the dayto- day survival, political struggles, and rising up of women of color, especially black women in the United States, reveal both complex resistance to

holocaust and undevelopment and often conflicted responses to the military and war. The Holocausts Women of color are survivors of and remain casualties of holocausts , and we are direct victims of war-that is, of open armed conflict between countries or between factions within the same country. But women of color were not soldiers, nor did we trade animal pelts or slaves to the white man for guns, nor did we sell or lease our lands to the white man for wealth. Most men and women of color resisted and fought back , were slaughtered , enslaved , and force marched into plantation labor camps to serve the white masters of war and to build their empires and war machines. People of color were and are victims of holocausts-that is, of great and widespread destruction, usually by fire. The world as we knew and created it was destroyed in a continual scorched earth policy of the white man. The experience of Jews and other Europeans under the Nazis can teach us the value of understanding the totality of destructive intent, the extensiveness of torture, and the demonical apparatus of war aimed at the human spirit. A Jewish father pushed his daughter from the lines of certain death at Auschwitz and said, "You will be a remembrance-You tell the story. You survive." She lived. He died. Many have criticized the Jews for forcing non-Jews to remember the 6 million Jews who died under the Nazis and for etching the names Auschwitz and Buchenwald, Terezin and Warsaw in our

minds. Yet as women of color, we, too, are "remembrances" of all the holocausts against the people of the world. We must remember the names of concentration camps such as Jesus, Justice, Brotherhood, and Integrity, ships that carried millions of African men, women, and children chained and brutalized across the ocean to the "New World." We must remember the Arawaks, the Taino, the Chickasaw, the Choctaw, the Narragansett, the Montauk, the Delaware, and the other Native American names of thousands of U.S. towns that stand for tribes of people who are no more. We must remember the holocausts visited against the Hawaiians , the aboriginal peoples of Australia , the Pacific Island peoples, and the women and children of Hiroshima and Nagasaki. We must remember the slaughter of men and women at Sharpeville, the children of Soweto, and the men of Attica. We must never, ever, forget the children disfigured, the men maimed, and the women broken in our holocausts -we must remember the names, the numbers, the faces, and the stories and teach them to our children and our children's children so the world can

never forget our suffering and our courage. Whereas the particularity of the Jewish holocaust under the Nazis is over, our holocausts continue. We are the madres locos (crazy mothers) in the Argentinian square silently demanding news

Page 262: openev.debatecoaches.org€¦  · Web view1AC . Observation 1: If I could find the spot where truth echoesI would stand there and whisper memories of my children's future. I would

of our missing kin from the fascists who rule. We are the children of El Salvador who see our mothers and fathers shot in front of our eyes. We are the Palestinian and Lebanese women and children overrun by Israeli, Lebanese, and U.S. soldiers. We are the women and children of the bantustans and refugee camps and the prisoners of Robbin Island . We are the starving in the Sahel , the poor in Brazil , the sterilized in Puerto Rico. We are the brothers and sisters of Grenada who carry the seeds of the New Jewel Movement in our hearts, not daring to speak of it with our lipsyet. Our holocaust is South Africa ruled by men who loved Adolf Hitler, who have developed the Nazi techniques of terror to more sophisticated levels. Passes replace the Nazi badges and stars. Skin color is the ultimate badge of persecution. Forced removals of women, children, and the elderly-the "useless appendages of South Africa"-into barren, arid bantustans without resources for survival have replaced the need for concentration camps. Black sex-segregated barracks and cells attached to work sites achieve two objectives: The work camps destroy black family and community life, a presumed source of resistance, and attempt to create human automatons whose purpose is to serve the South African state's drive toward wealth and hegemony. Like other fascist regimes, South Africa disallows any democratic rights to black people; they are denied the right to vote, to dissent, to peaceful assembly, to free speech, and to political representation. The regime has all the typical Nazi-like political apparatus: house arrests of dissenters such as Winnie Mandela; prison murder of protestors such as Stephen Biko; penal colonies such as Robbin Island. Black people, especially children, are routinely arrested without cause, detained without limits, and confronted with the economic and social disparities of a nation built around racial separation. Legally and economically, South African apartheid is structural and institutionalized racial war. The Organization of African Unity's regional intergovernmental meeting in 1984 in Tanzania was called to review and appraise the achievements of the United Nations Decade for Women. The meeting considered South Africa's racist apartheid regime a peace issue. The "regime is an affront to the dignity of all Africans on the continent and a stark reminder of the absence of equality and peace, representing the worst form of institutionalized oppression and strife." Pacifists such as Martin Luther King, Jr. and Mahatma Gandhi who have used nonviolent resistance charged that those who used violence to obtain justice were just as evil as their oppressors. Yet all successful revolutionary movements have used organized violence. This is especially true of national liberation movements that have obtained state power and reorganized the institutions of their nations for the benefit of the people. If men and women in South Africa do not use organized violence, they could remain in the permanent violent state of the slave. Could it be that pacifism and nonviolence cannot become a way of life for the oppressed? Are they only tactics with specific and limited use for protecting people from further violence? For most people in the developing communities and the developing world consistent nonviolence is a luxury; it presumes that those who have and use nonviolent weapons will refrain from using them long enough for nonviolent resisters to win political battles. To survive, peoples in developing countries must use a varied repertoire of issues, tactics, and approaches. Sometimes arms are needed to defeat apartheid and defend freedom in South Africa; sometimes nonviolent demonstrations for justice are the appropriate strategy

for protesting the shooting of black teenagers by a white man, such as happened in New York City. Peace is not merely an absence of 'conflict that enables white middleclass comfort, nor is it simply resistance to nuclear war and war machinery. The litany of "you will be blown up, too" directed by a white man to a black woman obscures the permanency and institutionalization of war, the violence and holocaust that people of color face daily. Unfortunately, the holocaust does not only refer to the mass murder of Jews, Christians, and atheists during the Nazi regime; it also refers to the permanent institutionalization of war that is part of every fascist and racist regime. The holocaust lives. It is a threat to world peace as pervasive and thorough as nuclear war.

The 1AC reads the subjective vertigo of communicative capitalism, but they cannot read the paradigmatic antiblackness that makes communicative capitalism possible. Blackness is marked by OBJECTIVE VERTIGO rather than subjective vertigo – blackness gratituously positioned as Non-Being. The only appropriate response must be a politics of negation. Affirmative politics of the 1AC reelaborates black suffering through the Human Being they seek to overcome. Turns case because they reproduce communicative capitalism and the dialectic, and external impact – they reproduce antiblackness at a metaphysical level. Only the Alternative has any chance of realigning deleuze with objective rather than subjective vertigo

Barber 16 (Daniel Colucciello Barber, researcher at the Humboldt University of Berlin, PhD from Duke University, 2016, “The Creation of Non-Being,” Rhizomes Issue 29, footnotes 1, 2, and 7 included in curly braces, modified) gz

Page 263: openev.debatecoaches.org€¦  · Web view1AC . Observation 1: If I could find the spot where truth echoesI would stand there and whisper memories of my children's future. I would

[1] Anti-blackness operates axiomatically. This is the case, at least, insofar as we speak of what Frank B. Wilderson, III, has called " the world " (Wilderson 2003: 234).[1] {1. "World" here refers not to reality as such, but more precisely to the paradigmatic operations by which reality is structured , positioned, and rendered sensible. Yet this does not mean that one can directly express or pose reality as distinct from the world, for the world governs the very conditions of possibility for expression or position . Even purportedly universal terms, such as humanity, social life, and—to invoke the concern of this essay—being itself, are operations of the world. The Afro-Pessimist thesis, following Wilderson, is that this world constitutes itself and maintains its coherence , at essence, through anti-blackness : the world has being insofar as blackness does not. Since the grammar of this world, or the logic of the aforementioned operations, is so naturalized —enacted and assumed by/from power—that it generally has no need to appear (much less defend itself), the articulation of reality without the anti-black world must begin as an articulation against this world.} The aim of this essay is to address the consequences of this axiomatic operation for some rather classical terms of reference within continental philosophy, such as being, analogy,[2] communication, possibility, and knowledge. {2. Both the reading of Lazzarato I provide below and my general argument, which revolves around the question of negativity and analogy, are deeply shaped by—and only conceivable thanks to—the writings of Wilderson, whose claim about analogy is summarized in the following remark, made in conversation

with Hartman: "In my own work, obviously I'm not saying that in this space of negation, which is blackness, there is no life. We have tremendous life . But this life is not analogous to those touchstones of cohesion that hold civil society together. In fact, the trajectory of our life (within our terrain of civil death ) is bound up in claiming—sometimes individually, sometimes collectively—the violence which Fanon writes about in The

Wretched of the Earth, that trajectory which, as he says, is 'a splinter to the heart of the world' and 'puts the settler out of the picture.' So, it doesn't help us politically or psychologically to try to find ways in which how we live is analogous to how white positionality lives, because, as I think your book suggests, whites gain their coherence by knowing what they are not" (Hartman and Wilderson 2003: 187).} Such terms are the means by which the world claims to grant itself coherence ; they form the grammatical ground , the structuring condition , of the world . If the "gratuitous violence" of anti-blackness extends into the very "grammar" of the world (Wilderson 2010: 38, 131),[3] then the aforementioned terms—far from providing retreat into a "metaphysical" domain unaffected by the historical and material—serve as points for the articulation of antagonism toward anti-blackness. In fact, the gratuity of such violence —its irreducibility to purposive meaning—entails a refusal of the coherent ground that these very terms claim to supply . This is to say that

being—or the possibility thereof—grounds itself not through its own coherence , but through an enactment of power that is staged by anti-black violence . Power precedes grammatical ground.

[2] Maurizio Lazzarato's analysis of contemporary capitalism approaches the anti-blackness analyzed by Wilderson. Lazzarato argues that capitalism is not grounded in any coherent science of economy, but is an enactment of the power to make indebted beings. It is by way of this emphasis on power that he links a purportedly secular capitalism to the theological structure of Christianity—that is, to a being that acts gratuitously, or without ground. Yet Lazzarato, I argue, ultimately wards off an encounter with anti-blackness through reliance on a coherence

implicit in "the indebted man" (Lazzarato 2012: 8). I elaborate this argument by drawing on Gilles Deleuze's concept of "difference in itself" (Deleuze 1994: 36-89). This concept, on my reading, ungrounds the purported coherence of being by way of a logically prior differentiality, which is expressed as non-being . Essential to this argument is the task of articulating such non-being without conversion to an affirmation of the world .

Non-Being: Deleuze Against Affirmation

[3] Deleuze's philosophy has come to be associated with habits of affirmation, where "habits" indicate the practices or operations by which reality is experientially and experimentally enacted .[4] This association could be attributed to Deleuze's invocation of concepts such as the rhizome, which appears to advocate teeming, emergent, multiplicitous movement in excess of all boundaries . In such a landscape of fluidity and flux, Deleuze's notion of creation then becomes associated with the affirmation

Page 264: openev.debatecoaches.org€¦  · Web view1AC . Observation 1: If I could find the spot where truth echoesI would stand there and whisper memories of my children's future. I would

of alternative possibilities. This association may also be attributed to Deleuze's rigorous refusal of the being of negativity. He contends that negative being plays no role in the determination of reality, that it is in fact an illusion that conceals the force of differential immanence . Given the centrality of this contention, any association of Deleuze's thought with habits of affirmation would have to depend on the following claim: the refusal of negative being entails the refusal of habits of negativity, in favor of habits of affirmation .[5]

[4] Yet it is fundamentally mistaken to conflate the refusal of negative being with the refusal of negative habits. The call for habits of affirmation is theoretically illegitimate: if all habits are real, and if reality has no negative being, then all habits—precisely because they are real—do not involve negative being; the reality that is habituated—regardless of whether this habituation is characterized as affirmative or negative—has no negative being. If the call for habits of affirmation is therefore not entailed by Deleuze's refusal of negative being,

then from where does this call arise? If habits of affirmation are imperative, then from where does this imperativity draw its mandate? To begin to answer these questions, one must address the ways in which habits of affirmation are logically consistent —and ultimately politically complicit —with the contemporary conjuncture of capitalism .

[5] This conjuncture, which has been variously described in terms of " late capitalism ," " postfordism ," or " communicative capitalism ," is marked by an affirmation of mobility , innovation , fluidity , possibility , and creativity . Deleuze analyzed this conjuncture in terms of control societies , which he distinguished from disciplinary societies. Control establish es domination not by setting up in advance strict boundaries , but rather by a kind of unending encouragement , or motivated permissiveness : control establishes and expands itself by establishing and expanding possibilities of communication . Domination "no longer operate[s] by confining people but through continuous control and instant communication" (Deleuze 1997: 174). Whereas discipline names the prohibition of excessive mobility and innovation, control names the "modulation" of the possibilities implied in such mobility and innovation (Deleuze 1997: 179).[6]

[6] With control, domination remains not despite, nor in opposition to, but precisely as possibility , which is modulated through a communicability that is ever more fluid and receptive in its listening in order to be ever more innovative in its surveilling .[7] {7. The fundamental insidiousness of control is that it permits and encourages the fluidity , mobility , and possibility implied by the sheer capacity to narrate. Communicative capitalism does not work by mandating what can and cannot be narrated, rather it calls for any-narration -whatever, as long as the possibility of narration is affirmed .} Following Deleuze's analysis of control, habits of affirmation—of multiplicitous possibilities, or of the possibility of being-otherwise—are not resistant to, but actually constitutive of, control's modulation . Control is marked by " endless postponement " (Deleuze 1997: 179), meaning that the future—as that which breaks with the present—never takes place. The present is extended into the future, and so the future becomes a modulation of the present ; an essential incommensurability between present and future remains unthinkable.[8] Given Deleuze's analysis, it is not by accident that he increasingly experimented with habits of negativity. In his last book, What is Philosophy? – co-written with Félix Guattari, and published one year after his analysis of control—

one can observe, for instance, his attentiveness to "shame" (Deleuze and Guattari 1996: 107), which was motivated by his reading of Primo Levi, or his indication of agreement with the negative dialectic of Theodor Adorno.[9]

[7] One finds, in the same book, a polemic against communication and a concomitant positioning of creation as distinct from and incommensurable with the communicative.[10] Simply put, Deleuze's

Page 265: openev.debatecoaches.org€¦  · Web view1AC . Observation 1: If I could find the spot where truth echoesI would stand there and whisper memories of my children's future. I would

increased attention to control, or communication, directly corresponds to his increased attention to the negative—not as being but as experience and experiment, as habit. Thus it is not only that Deleuze's refusal of negative being cannot be conflated with habits of affirmation, it is also that Deleuze , when attending to control, attempts to articulate habits of negativity . What is Philosophy? concludes with an articulation of the No of chaos , the non of thought that enables creation : philosophy must attain "an essential relationship with the No that concerns it "; philosophy does "not need the No as beginning, or as the end in which [it] would be called upon to disappear by being realized, but at every moment of [its] becoming or [its] development" (Deleuze and Guattari 1996: 218).

[8] The creation named by Deleuze's philosophy is thus in immanence with the No , and it is this No- creation immanence that begins to articulate antagonism toward communication : "Creating has always been something different from communicating " (Deleuze 1997: 175). This divergence between communication and the No of creation is utter , essential , and irredeemable . There is no possibility of emancipating communication , nor is there any affirmative basis for creation —for the base is communication. There is nothing to affirm , and so creation is immanent with the negativity of the non : " The key thing may be to create vacuoles of noncommunication " (Deleuze 1997: 175).

The Reality of Non-Being

[9] My argument, drawing on Deleuze, is that the logic of possibility actually serves to modulatively reproduce the anti-black grammar of the world. Creation, defined as a break with the presently given world, is not a possibility. It is rather immanent with an axiomatic No to such possibility , with habits of negativity .

[10] This thesis concerns a key problematic that stems from the Afro-Pessimist analysis of anti-blackness: if blackness stands [is] both within the habitus of modernity, as an organizing principle , and without this habitus, as a perpetually banished subjectivity , then the very articulation of blackness would seem to depend on and reproduce such a habitus. In other words, both being -within and being -without are possibilities governed by modernity's dominative positioning of blackness. The articulation of blackness is in fact bound by this problematic insofar as one remains within the ambit of habits of affirmation . In other words, the presumption of affirmation is co-extensive with the reproduction of the habitus of modernity : that which is presently available for affirmation is already governed by modernity and its articulation of blackness , and so habits of affirmation inevitably participate in and reproduce the double-bind in which modernity positions blackness.

[11] Against such reproduction, it is essential to insist on habits of negativity . Such insistence is total : since it is affirmation as such that entails participation in the being here indexed by modernity, even a modicum of affirmation mitigates the force enacted by negativity. The power of creation therefore resides entirely and essentially on the side of negativity —and not at all on the side of affirmation . Concomitantly, to invoke such power actually entails an unmitigated refusal of habits of affirmation ; affirmation does not name or support, but on the contrary denies, the power of creation . Given the double-bind in which modernity positions blackness, this is to say that the negativity of the non , in virtue of its immanence with a force of creation, indexes blackness as a power of non-being , as that which is without need of—and in fact opposed to —reliance on the affirmative.

[12] It remains necessary to outline the articulation of this immanence of creation and non-being—that is, to theoretically express how an unmitigated insistence on habits of negativity can be both a refusal of affirmation and an enactment of power. This warrants a return to Deleuze's thought by way of some questions: How can habits of negativity, articulated via Deleuze's insistence on the non, gain theoretical consistency with his conceptual refusal of negative being? If negative being is refused, then in what sense can there be insistence on the non?

Page 266: openev.debatecoaches.org€¦  · Web view1AC . Observation 1: If I could find the spot where truth echoesI would stand there and whisper memories of my children's future. I would

[13] Deleuze argues that "being is difference itself. Being is also non-being, but non-being is not the being of the negative . . . non-being is Difference" (Deleuze 1994: 76-77). This makes clear that negative being is refused in virtue of difference ; what is essential is difference in itself . Hence difference is articulated not as the affirmation of affirmative being, nor even as the affirmation of being as such . On the contrary, difference is articulated as "non-being" : negative being is refused, but it is refused in favor of non -being . Difference antecedes both positive being and negative being , thereby displacing their dialectical or conflictual relation. In other words, difference is not between opposed beings but in itself, autonomous from and antecedent to every being or thing ; difference is real, but precisely as a matter of non -being . Its reality is not the being of a thing, it is no -thing .

[14] Such theorization enables the delinking of creation (as force of non-being, or no-thing) from affirmation (as possibility of being). Difference, or non-being, marks a real force of creation that is without, and incommensurable with, being. In virtue of this unanalogizability of non-being with being , creation is articulated as a force stemming from negativity , and not at all from affirmation : affirmation is said of being and its possibilization, whereas creation is said of non-being . Habits of negativity, which antagonize every (positively or negatively described) being, or being as such, are thus coeval with an insistence on the real force of non-being.

[15] This argument can be used to negotiate a tension between the Afro-Pessimist emphasis on irresolvable negativity and the concern of Black Optimism to emphasize a power named by blackness : while the former's emphasis on negativity extends to habits of affirmation as such, this negativity immanently involves —and thus does not abandon—an insistence on the power of creation.

Consequently, the Black Op concern to speak of the power of blackness may be satisfied entirely within the space of negativity, or social death , on which Afro-Pessimism insists . Such satisfaction does not then require recourse to qualifications that would mitigate the negativity of this space, On the contrary, power is immanent to a redoubled negativity , or a negativity toward both being and the affirmation of the possibility of being-otherwise.

[16] Yet even as Deleuze's philosophical efforts may be deployed by and for the articulation of Afro- Pessimist claims , these claims vertiginously intensify Deleuze's theorization of non-being : Deleuze theorizes non-being in terms of a "vertigo" of immanence (Deleuze and Guattari 1996: 48), yet blackness is the historical, material experience of such vertigo. Drawing on a distinction made by Wilderson, this is to say that for Deleuze non-being is a "subjective vertigo," or a vertigo into which Deleuze's thought makes an entrance , while blackness is experienced as "objective vertigo," meaning that vertigo is —historically or materially—always already there (Wilderson 2011: 3). Immanence, or the vertigo of non-being, remains an object for the thought of Deleuze; blackness is historically or materially the objective reality of non-being —the very reality of the vertigo of immanence. Consequently, to think non-being according to blackness entails the reading of Deleuze's theoretical articulation in terms of the operations by which historical, material power is enacted

Page 267: openev.debatecoaches.org€¦  · Web view1AC . Observation 1: If I could find the spot where truth echoesI would stand there and whisper memories of my children's future. I would

The 1AC imagines itself as a break from relation, but only insofar as it attempts to create or elaborate upon a condition of self-creation and subjectivity. This inevitably falls back into a habit of affirmation that fails to understand blackness as a site of absolute and unhinged vertigo – only the demand of absolute negativity has the potential to analyze blackness as perpetually unhinged in objective vertigo – this card is really complicated and if they try to answer it they will loseBarber 16 (Daniel Colucciello Barber is a Researcher at Humboldt University of Berlin. Nearest date given is 2016, Rhizomes Issue 29: Black Holes: Afro-Pessimism, blackness and the discourses of Modernity, “The Creation of Non-Being,” http://www.rhizomes.net/issue29/barber.html )

[9] My argument, drawing on Deleuze, is that the logic of possibility actually serves to modulatively reproduce the anti-black grammar of the world . Creation , defined as a break with the presently given world, is not a possibility . It is rather immanent with an axiomatic No to such possibility, with habits of negativity . [10] This thesis concerns a key problematic that stems from the Afro-Pessimist analysis of anti-blackness: if blackness stands both within the habitus of modernity, as an organizing principle, and without this habitus, as a perpetually banished subjectivity, then the very articulation of blackness would seem to depend on and reproduce such a habitus. In other words, both being-within and being-without are possibilities governed by modernity's dominative positioning of blackness. The articulation of blackness is in fact bound by this problematic insofar as one remains within the ambit of habits of affirmation . In other words, the presumption of affirmation is co- extensive with the reproduction of the habitus of modernity : that which is presently available for affirmation is already governed by modernity and its articulation of blackness, and so habits of affirmation inevitably participate in and reproduce the double-bind in which modernity positions blackness. [11] Against such reproduction, it is essential to insist on habits of negativity. Such insistence is total: since it is affirmation as such that entails participation in the being here indexed by modernity, even a modicum of affirmation mitigates the force enacted by negativity. The power of creation therefore resides entirely and essentially on the side of negativity — and not at all on the side of affirmation . Concomitantly, to invoke such power actually entails an unmitigated refusal of habits of affirmation; affirmation does not name or support, but on the contrary denies, the power of creation. Given the double-bind in which modernity positions blackness, this is to say that the negativity of the non, in virtue of its immanence with a force of creation, indexes blackness as a power of non-being, as that which is without need of—and in fact opposed to—reliance on the affirmative. [12] It remains necessary to outline the articulation of this immanence of creation and non-being—that is, to theoretically express how an unmitigated insistence on habits of negativity can be both a refusal of affirmation and an enactment of power. This warrants a return to Deleuze's thought by way of some questions: How can habits of negativity, articulated via Deleuze's insistence on the non, gain theoretical consistency with his conceptual refusal of negative being? If negative being is refused, then in what sense can there be insistence on the non? [13] Deleuze argues that "being is difference itself. Being is also non-being, but non-being is not the being of the negative . . . non-being is Difference" (Deleuze 1994: 76-77). This makes clear that negative being is refused in virtue of difference; what is essential is difference in itself. Hence difference is articulated not as the affirmation of affirmative being, nor even as the affirmation of being as such. On the contrary, difference is articulated as "non-being": negative being is refused, but it

Page 268: openev.debatecoaches.org€¦  · Web view1AC . Observation 1: If I could find the spot where truth echoesI would stand there and whisper memories of my children's future. I would

is refused in favor of non-being. Difference antecedes both positive being and negative being, thereby displacing their dialectical or conflictual relation. In other words, difference is not between opposed beings but in itself, autonomous from and antecedent to every being or thing; difference is real, but precisely as a matter of non-being. Its reality is not the being of a thing, it is no-thing. [14] Such theorization enables the delinking of creation (as force of non-being, or no-thing) from affirmation (as possibility of being). Difference, or non-being, marks a real force of creation that is without, and incommensurable with, being. In virtue of this unanalogizability of non-being with being, creation is articulated as a force stemming from negativity , and not at all from affirmation: affirmation is said of being and its possibilization, whereas creation is said of non-being . Habits of negativity, which antagonize every (positively or negatively described) being , or being as such, are thus coeval with an insistence on the real force of non-being . [15] This argument can be used to negotiate a tension between the Afro-Pessimist emphasis on irresolvable negativity and the concern of Black Optimism to emphasize a power named by blackness: while the former's emphasis on negativity extends to habits of affirmation as such, this negativity immanently involves—and thus does not abandon—an insistence on the power of creation. Consequently, the Black Op concern to speak of the power of blackness may be satisfied entirely within the space of negativity, or social death, on which Afro-Pessimism insists. Such satisfaction does not then require recourse to qualifications that would mitigate the negativity of this space, On the contrary, power is immanent to a redoubled negativity, or a negativity toward both being and the affirmation of the possibility of being-otherwise. [16] Yet even as Deleuze's philosophical efforts may be deployed by and for the articulation of Afro-Pessimist claims, these claims vertiginously intensify Deleuze's theorization of non-being: Deleuze theorizes non-being in terms of a "vertigo" of immanence (Deleuze and Guattari 1996: 48), yet blackness is the historical, material experience of such vertigo. Drawing on a distinction made by Wilderson, this is to say that for Deleuze non-being is a "subjective vertigo," or a vertigo into which Deleuze's thought makes an entrance, while blackness is experienced as "objective vertigo ," meaning that vertigo is— historically or materially — always already there (Wilderson 2011: 3). Immanence, or the vertigo of non-being, remains an object for the thought of Deleuze; blackness is historically or materially the objective reality of non-being—the very reality of the vertigo of immanence. Consequently, to think non-being according to blackness entails the reading of Deleuze's theoretical articulation in terms of the operations by which historical, material power is enacted

Interrogation of debt and gift asymmetry presupposes a universalization of indebtedness, a global capacity for a relation of inheritability, which relegates blackness to the position of the unthought – as a paradoxical condition of non-being, blackness has no access to recognized relationality because it is the target of a mode of domination prior to and essential for capitalist violence – this means the most radical change the aff can effectuate is an anti-black modulation of civil societyBarber 16 (Daniel Colucciello Barber, researcher at the Humboldt University of Berlin, PhD from Duke University, 2016, “The Creation of Non-Being,” Rhizomes Issue 29, footnotes 13 and 15 included in curly braces) gz[22] One way of addressing this task is to think debt as inheritance—that is, to think the inheritance of Christian debt by capitalism, and in doing so to think how a capacity of being is inherited by capitalism from Christianity. We

Page 269: openev.debatecoaches.org€¦  · Web view1AC . Observation 1: If I could find the spot where truth echoesI would stand there and whisper memories of my children's future. I would

inherit debt, and debt requires that our future be inherited—ahead of time—as the debt enacted in the present. But it is not just that debt is inherited, it is also that debt constitutes its inheritors as something , as beings analogically belonging to a "we." Note, for instance, the collective first-person of Lazzarato's claim: "We are no longer the inheritors of original sin but rather of the debt of preceding generations " (Lazzarato 2012: 32). Who is "we" ?

[23] It is by way of this question that one begins to encounter a limit of Lazzarato's analysis, which I will address in a logical register before returning to the explicitly historical marks of the inheritance that he tracks. This limit, logically speaking, is Lazzarato's focus on asymmetry. He clearly observes the injustice of the debt-relation by articulating the deep asymmetry between creditor and debtor.[11] While this observation is not incorrect, the approach to which it belongs ignores the ways in which asymmetry remains within being . In order for one thing to be communicated as asymmetrical with or disproportionate to another thing, these things must be analogical to one another , possessing a minimal degree of likeness or commonality. Therefore creditor and debtor, despite the extremity of their asymmetry, remain analogous to each other as beings .

[24] This is to say that the debt-relation operates as a domain of analogous being, and that Lazzarato —by presupposing and leaving in place this domain —fails to encounter the negativity of non-being .[12] Whereas asymmetry presupposes the commonality and analogical relation of beings, non-being names that which is without being , and thus without analogical relation to being(s) : something and nothing are not asymmetrical but incommensurable . Lazzarato's critique of debt, by focusing on asymmetry, ignores this absence of analogy between being (whether positive or negative, creditor or debtor) and non-being , and so it can only amount to a modulation of being —that is, a modulation of we .

[25] Returning to Lazzarato's history of Christian-capitalist inheritance, I contend that this modulation is evident as an apparent transmutation within the "we": we were once the inheritors of original sin, whereas we are now the inheritors of debt. Yet a deeper continuity remains, for while we are different, it is we who have undergone—and survived—discontinuity: we are still we . Previously we inherited original sin, whereas now we inherit debt, but we are still those who inherit —and, in virtue of this being-inherited or inheriting-being, we are something . We are we, and we remain we , across any apparent discontinuity of Christianity and capitalism, because what remains, what is constant, is the capacity to inherit . Such inheritance is not so happy, of course, for to inherit sin, or to inherit debt, is to be exploited by God or capital. Yet this structure of exploitation maintains an analogy between exploiter and exploited : we are exploited, but precisely through this exploitation , this inheritance of debt, we still know ourselves as we .

[26] In this sense, "we" names the inherited capacity to be-something , or the capacity to inherit being . What is ultimately inherited is not debt so much as this capacity : the debt that exploits is the debt that gives being, that gives the capacity to be in analogy with other beings , and thus to participate in or

Page 270: openev.debatecoaches.org€¦  · Web view1AC . Observation 1: If I could find the spot where truth echoesI would stand there and whisper memories of my children's future. I would

communicate as we . To frame the inheritance of debt primarily in terms of its asymmetry or exploitation is thus to obscure the fact that inheritability , or the ability to inherit, is the common or communicable being underlying all asymmetry . This is to say that Lazzarato focuses his analysis on the conflictual relation between beings of the anti-black world and thereby fails to address the more essential antagonism between blackness and the world . Lazzarato remains within the being of inheritance , or within the we that underlies and guarantees the "coherence" [13] (Hartman and Wilderson 2003: 187) of asymmetrical relation , whereas any break with the present must be articulated according to blackness , which is without relation . {13. Wilderson, in dialogue with Hartman, remarks: "as I think your book suggests, whites gain their coherence by knowing what they are not ."}

[27] The break , then, must be articulated according to the uninheritability of blackness . For Lazzarato , however, blackness remains in "the position of the unthought" (Hartman and Wilderson 2003: 185), and this is precisely because he adheres to the universalizable horizon of the we . "Everyone is a 'debtor,' accountable to and guilty before. Capital has become the Great Creditor, the Universal Creditor" (Lazzarato 2012: 11).[14] Yet it is clear that there are those who do not participate in the we of the indebted man .[15] {15. In view of this investment in universalization, the gendered nature of "the indebted man" should be observed . More precisely, universalization's taking-distance from the feminine should be connected to its taking-distance from blackness.} Logically prior to the domination articulated via asymmetrical relations of we (inheritance of debt), there is domination articulated as non- being : "the damned of the earth "[16] do not inherit .

We resolve the non antiblack portions of the aff, we can shift his subjective vertigo towards objective vertigo.

The affirmative resistance is just a blueprint of anti-blackness – there move to ground of resistance politics is provided by antiblackness – in Trump era resistance is wanted by anti-blackness while only being absorbed and gutted for potentiality

Sexton ’17 (Jared, Interviewed by Daniel Colucciello Barber, <On Black Negativity, or the Affirmation of Nothing, <http://societyandspace.org/2017/09/18/on-black-negativity-or-the-affirmation-of-nothing/link> pg. 9-29, B.M.)

So, this tear in the world, this tearing of the world, this torn world ... you see indexes of its ramifications within and across the field of black studies itself—which for me is an unlimited field—that being part of the dehiscence I mentioned in my earlier essay. I initially borrowed from the Ecrits (2006) of Jacques Lacan, where dehiscence refers, in part, to the internal splitting of the subject by the imposition of language and the abolition of access, conscious access, to what might be called a real relation to being. Thereafter, one must wrestle with language to articulate the inarticulable and, in the process, live out and live with the effects of such signifying action at the level of embodiment, affect, memory, and the like. But dehiscence has a helpful polyvalence—thanks to Richard Yoder for speaking to this—indicating, in surgical medicine, the opening

Page 271: openev.debatecoaches.org€¦  · Web view1AC . Observation 1: If I could find the spot where truth echoesI would stand there and whisper memories of my children's future. I would

up of a wound along the lines of incision (either because the wound was inadequately sutured or has become infected or subjected to further trauma), or, in botany, the opening up of plants along a seam at the age of maturity as a means of dissemination, or, in otology, the perforation in the inner ear labyrinth causing chronic disequilibrium or vertigo. These themes of wounding, dissemination, and vertigo all loom large in my thinking....In this protracted effort, of course, there is a necessary element of defense against the undertones—or, more usually, the clear

overtones—of anti-blackness that provide both the message and the medium of the radical betrayal of radicality in thought and action, a certain ceding of potentiality (not to be confused with mere possibility) in search of ground, base, foundation. Operating without need of a base, a willingness to operate in a way that is groundless, baseless, without foundation is not just the good old anti-foundationalism that exercised the minds of late-twentieth century philosophers, historians, political theorists, and literary and cultural critics; in part, because there is no rejection of a concept of totality here. Fanon was tracking something like this, I think, in his insistence, late in Black Skin, White Masks, on “introducing invention into existence” (Fanon, 2008a: 179). And on this point I really prefer the Markmann translation to the Philcox translation. Philcox has it as “introducing invention into life” (Fanon, 2008b: 204), which unnecessarily restricts Fanon’s commentary to the realm of life, or the bios/zoe couplet, and leaves aside the entire realm of non-life, or geos (Povinelli, 2016). ..Now, Zizek’s “Bartleby politics” are obviously not quietist, insofar as they are meant to prepare the way for a true political act. (Frédéric Neyrat [2014] has a related

conception: “Rather than its heart, passivity should be the skin of politics. Without passivity, without a ‘negative capability,’ to refer to Keats’s notion, there isn’t any creative imagination, this chaotic imagination that generates the promises of new worlds.” And, not for nothing, Hortense Spillers (2003) makes another, earlier argument for “negative capability” in a pair of essays first published in the 1990s, “The Crisis of the Negro Intellectual: A Post-Date” and “All The Things You Could Be By Now If Sigmund Freud’s Wife Was Your Mother: Psychoanalysis and Race.” But the interregnum that opens up between the

frenetic, aggressively passive “ activism” of the current socio-ideological constellation— in which “the anxious expectation that nothing will happen” competes with “the desperate demand to do something”—and that new constellation brought into being by the introduction of some fundamental indeterminancy—a negativity that is, as you rightly note above, strictly unfathomable —that interregnum would seem to require the cultivation of an oxymoronic passive activity. Does it make sense to speak of a need for “passivism” (not to be confused with the homophonic term “pacifism”)? Think of the performative contradiction of trying to relax; the harder you try to attain it, the more it evades you. As every

athlete worth their salt knows, your best performance requires your least effort. The more you relax, the more intensely you can exert yourself. In this scenario, you do more the less you try... It’s worth thinking about this seriously in the Trump era (using Trump here as a symbol for the consolidation of a whole post-civil rights, post-cold war, post-9/11

dispensation), given how greatly the ongoing reactionary campaign benefits from and requires any and all imagery of protest, political or pedestrian, as evidence —“alternative factual evidence”— supporting a narrative, ultimately, of white victimization and oppressive black power (and all the conflictual transliterations of

this antagonism seen today—from the land and resource battles in the heartland to the travel bans at the borders). Given, that is, how frustratingly ineffective that protest seems to be in the face of an entire infrastructure that not only absorbs resistance, but solicits it too. It makes you nostalgic for the days of good old-fashioned repression and co-optation (days which, of course, never really existed in black ), because at least then you knew you were on to something truly oppositional, subversive, alternate. I don’t think it’s a coincidence that, under such conditions, black (or blackened) artists are drawn with some regularity to paradoxical ideas about fighting anti-blackness by over- identifying with its desire to disappear or distort or disfigure blackness, taking it over and enforcing it hyperbolically, satirically, even vindictively . I think Paul Beatty’s literature has done this to great effect for twenty years or more—from White Boy Shu!e (1996) to The Sellout (2015)—and you could add to that titles like Darius James’s Negrophobia (1992) and Kola Boof’s Sexy Part of the Bible (2011); consider as well the work of Betye Saar and Kara Walker and Michael Ray Charles in the visual arts, the comedy of Dave Chappelle and Leslie Jones, or films like Spike Lee’s Bamboozled (2000) and Lars von Trier’s Manderlay (2005) and Jordan Peele’s Get Out (2017). I don’t want to get too much into the weeds of

the current conjuncture and lose track of the larger structural problem posed by blackness and anti-blackness, a problem that confounds

the very distinction between structure and conjuncture in the first place. It is not just the exigencies of the present moment or the strategic and tactical challenges facing the Movement for Black Lives that raise the question of how to intervene, of how to introduce “invention into existence,” as Fanon put it. What I’ve called “a groundless or baseless politics that does not proceed from a margin of power, a politics with no (final) recourse to foundations of any sort, a politics forged from critical resources immanent to the situation, resources

Page 272: openev.debatecoaches.org€¦  · Web view1AC . Observation 1: If I could find the spot where truth echoesI would stand there and whisper memories of my children's future. I would

from anywhere and anyone, which is to say from nowhere and no one in particular” (Sexton, 2016b: 589); this approach to politics would seem to entail a total rejection of transcendence, a politics of pure immanence without the Archimedean point. And in one sense it is, but I am attuned to the difficulties arising from an abandonment of the negative in our enthusiasm for the affirmative, and so I want to think not so much about transcendence viz. immanence as about the various forms or modalities of transcendence and immanence that can be mobilized

“Feeling others, feeling through you” is not an appositional politic but a taxonomy of racial sentimentality mired and confirmed by a logic of biological racism and the imperialist temporality of the nation-state. The 1AC’s investment in hapticality as a new form of sociality is also an investment in a disciplinary function of sympathy that calcifies the racial superiority of the whites against the torpid sympathy of colonized subjects. Ultimately, they remain bound to biopolitics of sentiment that evokes moral feelings of social good for the sake of liberal individualist subjectivity – turns the AFF’s project. Schuller ‘18 Kyla, Associate Professor and Women's Global Health Leadership (WGHL) Certificate Faculty Director of Women's and Gender Studies at Rutgers University, New Brunswick. She investigates the intersections between race, gender, sexuality, and the sciences. “The Biopolitics of Feeling; Race, Sex, and Science in the Nineteenth Century. Duke University Press, December 2017) ipartman

The volatility of the impressible body, which depended on its impressions from the external world for its own development, requireda strict disciplinary technology. Sentimentwas enlisted for this purpose. Thealleged

supple impressibility of the civilized races ensured that they continuously developed new, hopefully advantageous, physical and mental characteristics. Most important among these was the capacity for sentiment. Cope argued that the advanced “social life and the family relation” of civilization “have developed the benevolent sentiments and the affections,” as

the effects of evolution became the stimuli themselves. Sentiment , in these scientists’ work, involves the ability to make a n

appropriate and sympathetic reaction to an impression , rather than an impulsive and self-serving one. Cope and his

cohort posited that racial progress stems from the ability of the civilized to control the impulses of their body

through the faculty of sentiment . Le Conte surmised that “sympathy, pity, [and] love” thus drive species change among the most advanced races, freeing them from the indignity of struggle .Hence for Cope “evolution is . . . the long process of learning how to bring matter into subserviency to the uses of mind,” or the sublimation of the body to “self-control,

from the material as well as from the mental standpoint.” Sentiment thus functions as an epistemology , an ontology, and a discipline . Scientific articulations of sentimentalism clarify that sympathy has an intercessory and teleological function, in thatit ensures that civilized responses to stimuli benefit racial progress . In her recent analysis of the role of sympathy in late nineteenth-century sciences of the mind, Susan Lanzoni argues that late nineteenth-century

psychologists and philosophers believed sympathy to increase with evolutionary advance. For Herbert Spencer, sympathy is the “ awareness of consequences ,” or an access to the future as opposed to the primitive “impulsivity” of “reflex-oriented”

responses,which are mired in the eternal present.Primitive bodies, Spencer maintained, were capable only of reflexes, not of reflection; they kicked experience off rather than absorbing it over time. In his view, social experience reverberates off black bodies in artful mimicry entertaining to their racial superiors, but arguably useless to

themselves. Otherscientists proposed that a “savage would throw a crying baby to the ground because of ‘torpid sympathy . ’”For Cope, sympathy enables the civilized to transform basic impulses of pleasure or

pain into a moral feeling that considers the social good, yetstillensures individual development . Cope

characterized sympathy as an advanced faculty evolving from sentiment that acts as a gatekeeper between the impressible civilized body—especially the more delicate female constitution—and its environment. This mediating capacity ensures that those who possessed it could

overcome the threats inherent to the impressible body, for sympathy allowed them to transform others’ suffering into opportunities for personal growth rather than for degeneration. On account of its developmental function, Cope

Page 273: openev.debatecoaches.org€¦  · Web view1AC . Observation 1: If I could find the spot where truth echoesI would stand there and whisper memories of my children's future. I would

declared thatsympathy is ultimately in one’s own self-interest : “ The affections or sympathies should be developed sufficiently to produce a desire for the happiness of others, through the pleasure the happiness of others gives us.”Presenting the formula of

the domestic novel as evolutionary doctrine—that making others feel good , especially those beneath you in social stature, brings its own reward —Cope lays bare the function of sympathy as building the actor’s body and character. Cope’s emphasis on the asymmetrical relations of sympathy illustrates the aptness of Glenn Hendler and Elizabeth Barnes’s analysis that sentimental sympathy functions as an “act of imagining oneself in another’s position” that ultimately works to constitute the

self.Sympathybothincreases and regulates the body’s affective experiences. Abolitionists famously drew on the sentimental discourse of shared feeling. Yet sentimental taxonomies of feeling broadly denied a common intensity of feeling and self-possession . As Saidiya Hartman has argued,the sentimental principle that a shared capacity for pain renders all life worthy of political recognition was a process that cut two ways, one that subjected blacks to power far more than it granted liberal individualist subjectivity to the enslaved . The American School’s emphasis on the reflective quality of sentiment, as opposed to the immediate and impulsive acts of sensation, suggests a final way in which these scientists drew on sentimentalism in their account of species change. As Dana Luciano has

argued, nineteenth-century U.S. sentimentalism marks “a way of using deployments of mixed feeling(pleasure and pain) to negotiate problems in time.”For these evolutionists, fundamentally concerned with the narration of temporality, sentimentalism proved a rich resource through which to challenge Darwin’s account of evolutionary time as a ruthless, senseless process. In the

first half of the century, Luciano argues, a wide variety of writers and lecturers understood grief asaway to access sacred,a reflective time that connected the grieving subject to the repetitive cycles of the organic and to offer protection from the linear, relentless, forward-moving temporality of national progress as well as from the impetuousness of sensation. Sensation “ signals a mode of intensified embodiment in which all times but the present fall away —a condition simultaneously desired, in its recollection of the infantile state, and feared, in its negation of social agency ” ; by contrast, “a morally regulated sentimentality,” manifested particularly in the capacity of reflection, “properly disperses feeling across time.”Cope and the American School

adapted sentimentalism’s function as a measured, reflective orientation of the civilized subjectin time into an evolutionary

discourse that gave the civilized the ability to manage the future development of the race . In keeping with their

political paradigm, this entailed reworking affective feeling as a sacred time outside the linear time line of national development into the means by whichthe organic body could be brought in synchrony with national and imperial progress. The American School drew on sentimentalism to assert Anglo-Saxons’ capacity to subjugate the

recursive rhythms of organic time to the service of the linear progress of national development. In the post-Darwinian context, the sentimental premise that refined feeling enables the transcendence of the physical body promised Anglo-Saxons a correlated control over natural time both cyclical and linear .Denied the status of fellow subjects of

the nineteenth century, racialized peoples were understood to be animated fossils of the evolutionary past. The “ great chain of

feeling , ” in historian Martin Pernick’s apt phrase, hierarchized human groups on the basis of their assumed sensibility and extended spatially to the expanding borders of the nation and temporally from the past to the future yet to come.The American School championed the cyclical theory of recapitulation, in which fetuses literally retrace the development of their ancestors in the womb, only fully reaching the evolutionary plane of their parents at puberty. Recapitulationists rearranged the spatial distinctions that polygenesis, as articulated by Agassiz and others in the American School of Ethnology, relied on to conceive of racial difference. Cope and Hyatt interpreted their collections of fossilized dinosaurs and cephalopods as evidence that different species exhibit parallel development, such that evolutionary change is best depicted not as a branching tree but as a common trunk that divides

into multiple parallel lines of differing length. Frozen somewhere near the dawn of civilization, blacks , Native Americans , Asians , and other racial groups formed different stages of “the infancy of civilized man ,” which nonetheless persisted into the present.They were the roots of humanity, the base from which the civilized had branched off and surpassed.The primitive would retrace the evolution of lizards and other animals in the womb: they would become human at birth but remain frozen in the same developmental state as their parents, even as their body seemingly

matured. Biopolitics entails the racialization of temporality . In Foucault’s words, itis thenineteenth-century “recasting of the theme of racial confrontations . . . [within] the theory of evolutionism and the struggle for

Page 274: openev.debatecoaches.org€¦  · Web view1AC . Observation 1: If I could find the spot where truth echoesI would stand there and whisper memories of my children's future. I would

existence,” in whichsome peoples now represent “the past of [the] race”that consolidates modern “ biologico-social racism ” as well as modern political power .As opposed to earlier understandings of human difference in which races were unequivocally distinct entities with diverse origins and were thus fundamentally at odds with one another, evolutionary perspectives conceived of racial difference as “permanently, ceaselessly in ltrating the social body,” and as the lingering prehistory

of the individual body.To be racialized in biopower isnot to be figured as an innately distinct species, as the American School of

Ethnology infamously had it several decades prior, but to be located within the past of civilization itself. Population

management aimed to harness the enemy lurking within the very borders of the settler colonial nation and the matter of the civilized body. Sentiment, in turn, functioned as one of thekey technologies to contain the threat of the biological past that haunted the settler colonial nation-state.

Narratives of Black suffering, no matter the radical form or intent, will always reify White innocence and recreate Black powerlessness. Hartman and Wilderson ’03 (Saidiya Hartman, Saidiya Hartman is a professor at Columbia University specializing in African American literature and history, Frank Wilderson III, Professor of African American Studies at UC Irvine, Source: Qui Parle, Vol. 13, No. 2 (Spring/Summer 2003), pp. 183-201, Published by: Duke University Press, Stable URL: https://www.jstor.org/stable/20686156, “Position of the Unthought”) \\EG

Frank B. Wilderson, Ill- One of the first things 1 want to say is how thankful I am that you wrote Scenes of Subjection: Terror, Slavery, and Self-Making in Nineteenth-Century America. And I want to say a little bit about how meaningful the book is to me as a black grad uate student - a so-called aspiring academic - and as someone caught in the machine but not of it. Because in general, when one reads the work of black scholars - if one is another black scholar or a black student - one prepares

oneself for a disappointment, or works a disappointment into the reading. And one doesn't have to do that with this particular book. What I mean, is that so often in black scholarship, people consciously or unconsciously peel away from the strength and the terror of their evidence in order to propose some kind of coherent, hopeful solution to things. Your book, in moving through these scenes of subjection as they take place in slavery, refuses to do that. And just as importantly, it does not allow the reader to think that there was a radical enough break to reposition the black body after Jubilee.1 That is a tremendous and courageous move. And I think what's important about it, is that it corroborates the experience of ordinary black people today, and of strange black people like you and me in the academy [Iaughter]. But there's something else that the book does, and I want to talk about this at the level of methodology and analysis. If we think about the registers of subjectivity as being preconscious interest, unconscious identity or identifications, and positionality, then a lot of the work in the social sciences organizes itself around preconscious interest; it assumes a subject of consent, and as you have said, a subject of exploitation, which you reposition as the subject of accumulation.2 Now when this sort of social science engages the issue of positionality - if and when it does - it assumes that it can do so in an un-raced manner. That's the best of the work. The worst of the work is a kind of multiculturalism that assumes we all have analogous identities that can be put into a basket of stories, and then that basket of stories can lead to similar interests. For me, what you've done in this book is to split the hair here. In other words, this is not a book that celebrates an essential Afrocentrism that could be captured by the multicultural discourse. And yet it's not a book that remains on the surface of preconscious interest, which so much history and social science does. Instead, it demands a radical racialization of any analysis of positionality. So. Why don't we talk about that? Saidiya V Hartman - Well! That's a lot, and a number of

things come to mind. I think for me the book is about the problem of craft ing a narrative for the slave as subject, and in terms of positionali ty, asking, "Who does that narrative enable?" That's where the whole issue of empathic identification is central for me. Because it just seems that every attempt to employ the slave in a narrative ultimately resulted in his or her obliteration, regardless of whether it was a leftist narrative of political agency - the slave stepping into someone else's shoes and then becoming a political agent - or whether it was about being able to unveil the slave's humanity by actually finding oneself in that position. In many ways, what I was trying to do as a cultural historian was to narrate a certain impossibility, to illuminate those practices that speak to the limits of most available narratives to explain the position of the enslaved. On one

hand, the slave is the foundation of the national order, and, on the other, the slave occupies the position of the unthought. So what does it mean to try to bring that position into view without making it a locus of positive value, or without trying to fill in the void? So much of our political vocabulary/imaginary/desires have been implicitly integrationist even when we imagine our claims are more radical. This goes to the sec ond part of the book - that ultimately the metanarrative thrust is always towards an integration into the national project, and particularly when that project is in crisis, black people are called upon to affirm it . So certainly it's

about more than the desire for inclusion with in the limited set of possibilities that the national project provides. What then does this language - the given language of freedom - enable? And once you realize its limits and begin to see its inexorable investment in certain notions of the subject and subjection, then that language of freedom no longer becomes that which rescues the slave from his or her former condition, but the site of the re-elaboration of that condition , rather than its transformation. F.W - This is one of the reasons why your book has been called "pessimistic" by Anita Patterson.3 But it's interesting that she does n't say what I said when we first started talking, that it's enabling. I'm assuming that she's white - I don't know, but it certainly

Page 275: openev.debatecoaches.org€¦  · Web view1AC . Observation 1: If I could find the spot where truth echoesI would stand there and whisper memories of my children's future. I would

sounds like it. S.VH. - But I think there's a certain integrationist rights agenda that subjects who are variously positioned on the color line can take up. And that project is something I consider obscene: the attempt to make the narrative of defeat into an opportunity for celebration, the desire to look at the ravages and the brutality of the last few centuries, but to still find a way to feel good about ourselves. That's not my project at all, though I think it's actually the project of a number of people.

Unfortunately, the kind of social revisionist history undertaken by many leftists in the 1970s , who were trying to locate the agency of dominated groups, resulted in celebratory narratives of the oppressed .4 Ultimately, it bled into this celebration, as if there was a space you could carve out of the ter rorizing state apparatus in order to exist outside its clutches and forge some autonomy. My project is a different one. And in partic ular, one of my hidden polemics in the book was an argument against the notion of hegemony, and how that notion has been taken up in the context of looking at the status of the slave. F W - That's very interesting, because it's something I've been thinking about also in respect to Gramsci. Because Anne Showstack Sassoon suggests that Gramsci breaks down hegemony into three categories: influence, leadership, and consent.5 Maybe we could bring the discussion back to your text then, using the examples of Harriet Jacobs,6 a slave, and John Rankin,7 a white anti-slavery Northerner, as ways in which to talk about this. Now, what's really interesting is that in your chapter "Seduction and the Ruses of Power," you not only explain how the positional

ity of black women and white women differs, but you also suggest how blackness disarticulates the notion of consent , if we are to think of

that notion as universal. You write: "[B]eing forced to submit to the will of the master in all things defines the predicament of slavery" (S, 110). In other words, the female slave is a possessed, accumulated, and fungible object, which is to say that she is ontologically different than a white woman who may, as a house servant or indentured labor er, be a subordinated subject.

You go on to say, "The opportunity for nonconsent [as regards, in this case, sex] is required to establish consent , for consent is meaningless if refusal is not an option.... Consent is unseemly in a context in which the very notion of subjectivity is predicated upon the negation of will " (S, 111). S. V.H. - Once again, trying to fit

into the other's shoes becomes the very possibility of narration. In the chapter "A Perilous Passage in the Slave Girl's Life," the question for Jacobs is how she can tell her story in a way that's going to solicit her white readership when she has to efface her very condition in order to make that story intelligible to them. I look at this messy moment as kind of a vortex in Jacobs' narrative, where in order to fashion herself as a desiring subject, she has to deny the very violence, which elsewhere she said defines her position as a slave: her status as a thing and the negation of her will. In one sense, she has to bracket that so she can tell a story about sexuality that's meaningful in a white dominant frame. And I think this is why someone like Hortense Spillers raises the question of whether gender and sexuality are at all applicable to the condition of the captive community.8 That's what I was working with there, that impossibility or ten sion between Jacobs as an agent versus the objective

conditions in which she finds herself. This is something you talk about in your work as well, this existence in the space of death, where negation is the captive's central possibility for action, whether we think of that as a radical refusal of the terms of the social order or these acts that are

sometimes called suicide or self-destruction, but which are real ly an embrace of death. Ultimately it's about the paradox of agency for those who are in these extreme circumstances. And basically, there are very few political narratives that can account for that. F.W -And we have to ask why. In my own work, obviously I'm not saying that in this space of negation, which is blackness, there is no life. We have tremendous life. But this life is not analogous to those touchstones of cohesion that hold civil society together. In fact, the trajectory of our life (within our terrain of civil death) is bound up in claiming - sometimes individually, sometimes collectively - the violence which Fanon writes about in The Wretched of the Earth, that trajectory which, as he says, is "a splinter to the heart of the world"9 and "puts the settler out of the picture."10 So, it doesn't help us politically or

psychologically to try to find ways in which how we live is analogous to how white positionality lives, because, as I think your book suggests, whites gain their coherence by knowing what they are not. There is tremendous diversity on the side of whiteness and tremendous conflict between white men and white women, between Jews and gentiles, and between classes, but that conflict, even in its articulation, has a certain solidarity. And I think that sol idarity comes from a near or far relation to the black body or bod ies. We give the nation its coherence because we're its underbelly. That's what's so interesting for me about Achille Mbembe's work, the way he thinks about the position of the for merly colonized subject along the lines of the slave as an essential way of defining the predicament. Essentially, he says, the slave is the object to whom anything can be done,

whose life can be squandered with impunity.12 F.W. - And he's suggesting that what it means to be a slave is to be subject to a kind of complete appropriation, what you call "property of enjoyment." Your book illustrates the "myriad and nefarious uses of slave property" and then demonstrates how "there was no relation to blackness outside the terms of this use of, entitlement to, and occupation of the captive body, for even the status of free blacks was shaped and compromised by the existence of slavery" (S, 24). So. Not only are formally enslaved blacks proper ty, but so are formally free blacks. One could say that the possibil ity of becoming property is one of the essential elements that draws the line between blackness and whiteness. But what's most intrigu ing about your argument is the way in which you demonstrate how

not only is the slave's performance (dance, music, etc.) the property of white enjoyment, but so is - and this is really key -

the slave's own enjoyment of his/her performance: that too belongs to white people .13 S. VH. - Right. You know, as I was writing Scenes of Subjection, there was a whole spate of books on nineteenth-century culture and on minstrelsy in particular. And there was a certain sense in

which the ability to occupy blackness was considered transgressive or as a way of refashioning whiteness, and there were all these rad ical claims that were being made for it.14 And I thought, "Oh, no, this is just an extension of the master's prerogative." It doesn't mat ter whether you do good or you do bad, the crux is that you can choose to do what you wish with the black body . That's why think ing about the dynamics of enjoyment in terms of

Page 276: openev.debatecoaches.org€¦  · Web view1AC . Observation 1: If I could find the spot where truth echoesI would stand there and whisper memories of my children's future. I would

the material rela tions of slavery was so key for me. F.W -Yes, that's clarifying. A body that you can do what you want with. In your discussion of the body as the

property of enjoyment, what I really like is when you talk about Rankin. Here's a guy - like the prototypical twentieth-century white progressive - who's anti-slavery and uses his powers of observation to write for its abo lition, even to his slave-owning brother. He's in the South, he's looking at a slave coffle, and he imagines that these slaves being beaten could be himself and his family. Through this process it makes sense to him, it becomes meaningful. His body and his fam ily members' white bodies become proxies for real enslaved black bodies and, as you point out, the actual object of identification, the slave, disappears . S.V.H. - I think that gets at one of the fundamental ethical ques tions/problems/crises for the

West: the status of difference and the status of the other. It's as though in order to come to any recognition of common humanity, the other must be assimilated , meaning in this case, utterly displaced and effaced: "Only if I can see myself in that position can I understand the crisis of that position." That is the logic of the moral and political discourses we see everyday - the need for the innocent black subject to be victimized by a racist state in order to see the racism of the racist state. You have to be exemplary in your goodness, as opposed to .. . F.W. - [laughter] A nigga on the warpath! S. V.H. - Exactly! For me it was those moments that were the most telling - the moments of the sympathetic ally, who in some ways is actually no more able to see the slave than the person who is exploiting him or her as their property. That is the work Rankin does and I think it suggests just how ubiquitous that kind of vio lence, in fact, is. F.W - You've just thrown something into crisis, which is very much on the table today: the notion of allies. What you've said (and I'm so happy that someone has come along to say it!) is that the ally is not a stable category. There's a structural prohibition (rather than merely a willful refusal) against whites being the allies of blacks, due to this - to borrow from Fanon's The Wretched of the Earth again - "species" division between what it means to be a subject and what it means to be an object: a structural antago nism. But everything in the academy on race works off of the ques tion, "How do we help white allies?" Black academics assume that there is enough of a structural commonality between the black and the white (working class) position - their mantra being: "We are both exploited subjects" - for one to embark upon a political pedagogy that will somehow help whites become aware of this "com monality." White writers posit the presence of something they call "white skin privilege," and the possibility of "giving that up," as their gesture of being in solidarity with blacks. But what both gestures disavow is that subjects just can't make common cause with objects. They can only become objects, say in the case of John Brown or Marilyn Buck, or further instantiate their subjectivity through modalities of violence (lynching and the prison industrial complex), or through modalities of empathy. In other words, the essential essence of the white/black relation is that of the master/slave - regardless of its historical or geographic specificity. And masters and slaves, even today, are never allies. S.V.H. - Right. I think of the book as an allegory; its argument is a history of the present. F.W - Thank you! I'm so glad you said it's an allegory of the pre sent. Because now we've got two problems on the table, two crises - or rather, we have many crises, but only two that I can identify at the moment. One is how we deal with the common sense around allies, whether it be in teaching literature to undergraduates or going to hear Cornel West speak with Michael Lerner, or listen ing to KPFA, since, in point of fact, it may be that the progressive community is actually as big an enemy to black revolution as Newt Gingrich. And the other I could put as, "How do you go to the movies?" How does one, knowing what one knows, sit through anything? Because it seems like every film - if it is in any way going to communicate some type of empathy that the audience can walk away with - has to have black death as its precondition. S.VH. - Yes, yes. Monster's Ball is a great example.15 Not only is Leticia's husband executed, but her son must also die as the pre condition for her new life with her husband's executioner. And the death requirement is rendered as a romance. Rather than closing with a note of ambivalence, the film actually ends with her smiling over the romantic music, as if to suggest that she's gotten over it, and the future awaits them. And I think that is the frightening hypocrisy of the context we are living in. There's also the film Unfaithful where the lover has to be mur dered in order to protect the heterosexual family.16 The white bour geois family can actually live with murder in order to reconstitute its domesticity. F.W - Well, why does white supremacy seem to be so bound up in the visual? S.VH. - I think that visually, the threat of blackness is somehow heightened. Fanon's "Look! A Negro": that's the formulation, and within the racial classificatory schema that is how much of the work is done, especially in terms of the way racialization has oper ated: how it disposes of bodies, how it appropriates their products, and how it fixes them in a visual grid. I think those are the three ways I would explore that problem, as well as, again, this whole dimension of the empathic. F.W - One of the things I wanted to bring up is how your book is talking to other very important books. It's talking to Fanon as you've said, and it's talking to Patterson's Slavery and Social Death.17 And you talked about the leftist discourses of the '70s, and the univer salizing of Gramscian hegemony that really falls short of helping us understand a position in civil society, but not of civil society. It has to do, I think, with how the idiom of power that black people expe rience has different kinds of manifestations as we move from slav ery into the era of the Freedmen's Bureau, but there's an umbrella of despotism that remains. And when you suggested earlier that the book is an allegory of the present, it was so refreshing, because one can read this book and begin to metaphorize the manifestations of despotism in the past, and also to think about how it continues in the present. S.VH. - It really is the pressing question of freedom. That's why for me, the last lines of the book summon up that moment of poten tiality between the no longer and the not yet. "Not yet free": that articulation is from the space of the twenty-first century, not the nineteenth, and that's the way it's supposed to carry - the same predicament, the same condition. FW. -And in those terms we might think about how Rodney King was accused of inviting his own beating; you know, he shook his ass in an aggressive manner at a white woman. So maybe you could sketch out the way in which the black woman functions sim ilarly in slavery, as somehow outside the statutory, or inside it: she cannot be raped because she's a non-person yet she is presumed to invite the rapist. S. VH. - Yes. No crime can occur because the slave statutes rec ognize no such crime. Often when I'm looking through the crimi nal record of the nineteenth century, I'm seeing the text of black agency. The people who are resisting their masters and overseers appear in the records as they're prosecuted for their crime, creating this displacement of culpability that enables white innocence. In the case of State of Missouri v. Celia (1855), Celia is raped repeat edly by her owner from the moment she's purchased. She begs him to stop; he doesn't, so she kills him. Her crime is the crime on record: she is the culpable agent.18 So in this formulation of law and its punishment, blackness is on the side of culpability, which makes the crimes of property transparent and affirms the rights to property in captives. And you're right, that displacement functions more generally. Who is the responsible and culpable agent? For the most part, it's always the slave, the native, the black. F.W - Which brings your allegory of the present to the prison industrial complex. S.VH. - Actually, I've got an interesting tidbit. I think that Den mark Vesey was the first person ever imprisoned in the South Carolina Penitentiary. F.W - Really? It's like a seamless transition from slavery to prison. S.V.H. - Right. And this is where the larger narrative of capitalism comes into play. Because, basically, in most places in the world, you have a transition from slavery to other modes of involuntary servitude. In my work, I critique the received narrative about the transition from slavery to freedom in the American context, but we could also look at that same kind of transformation in relation to the anti-slavery rhetoric that comes to legitimize the colonial pro ject in Africa. By the nineteenth century, slavery was the dominant mode of production in West Africa. Eventually, the European nations decided "This is an awful institution and we need to stop it," so we get King Leopold masking his atrocities in the Congo in the discourse of anti-slavery, or British colonial figures in Ghana effectively saying, "Well, we saved you from the slave raider so you should be grateful."19 In both cases, it's the same notion: "We've given you your freedom, so now you're in our debt." F.W - And that brings us to Reconstruction in your book where you're talking about post-jubilee: The good conduct encouraged by such counsels eased the transition from slavery to freedom by imploring the freed to continue in old forms of subservience, which primarily entailed remaining on the plantation as faith ful, hardworking, and obedient laborers, but also included manners, styles of comportment in work rela tions, objects of consumption, leisure, and domestic relations. In their emphasis on proper conduct, these schoolbooks resuscitated the social roles of slavery, not unlike the regulation of behavior in labor contracts or the criminalization of impudence in the Black Codes. The pedagogical injunctions to obedience and servility cast the freed in a world starkly similar to the one in which they had suffered under slavery. On the one hand, these texts heralded the natural rights of all men; and on the other, they advised blacks to refrain from enjoying this newly conferred equality. Despite proclamations about the whip's demise, emergent forms of involuntary servitude, the coercive control of black labor, the repressive instrumentality of the law, and the social intercourse of everyday life revealed the entan glements of slavery and freedom. (S, 151) So. There's this whole army of white people - missionaries, edu cators, and the like - who go down South to help rehabilitate the Negro after slavery. And in reading that, a wave of cynicism swept over me, because all of a sudden I thought of Freedom Summer, and the white students in SNCC, which is a blasphemous thought to have.

Their faith in performance and conceptual rupturing as an act of emancipation trades off with structural analysis and puts false place in subjectivity Wilderson 2010, [2010, Frank B. Wilderson is an Associate Professor of African-American Studies at UC Irvine and has a Ph.D. from UC Berkeley, “Red, White & Black: Cinema and the Structure of U.S. Antagonisms,”]

Page 277: openev.debatecoaches.org€¦  · Web view1AC . Observation 1: If I could find the spot where truth echoesI would stand there and whisper memories of my children's future. I would

Unfortunately, cultural studies that theorizes the interface between Blacks and Humans is hobbled in its attempts to (a) expose power

relationships and (b) examine how relations of power influence and shape cultural practice. Cultural studies insists on a grammar of suffering which assumes that we are all positioned essentially by way of the symbolic order , what Lacan

calls the wall of language—and as such our potential for stasis or change (our capacity for being oppressed or free) is overdeter-mined by our "universal" ability or inability to seize and wield discursive weapons. This idea corrupts the explanatory power of most socially engaged films and even the most radical line of political action because it produces a cinema and a politics that cannot account for the grammar of suffering of the Black—the Slave. To put it bluntly, the

imaginative labor5 of cinema, political action, and cultural studies are all afflicted with the same theoretical aphasia. They are speechless in the face of gratuitous violence. This theoretical aphasia is symptomatic of a debilitated ensemble of questions regarding political ontology. At its heart are two registers of imaginative labor. The first register is that of description, the rhetorical labor aimed at explaining the way relations of power are named, categorized, and explored. The second

register can be characterized as prescription, the rhetorical labor predicated on the notion that everyone can be emancipated through some form of discursive, or symbolic, intervention. But emancipation through some form of discursive or symbolic intervention is wanting in the face of a subject position that is not a subject position—what Marx calls "a speaking implement" or what Ronald Judy calls "an interdiction against subjectivity." In other words,

the Black has sentient capacity but no relational capacity. As an accumulated and fungible object, rather than an exploited and alienated subject, the Black is openly vulnerable to the whims of the world, and so is his or her cultural "production." What does it mean—what are the stakes—when the world can whimsically transpose one's cultural gestures, the stuff of symbolic intervention, onto another worldly good, a commodity of style? Frantz Fanon echoes this question when he writes, "I came into the world imbued with the will to find a meaning in things, my spirit filled with the desire to attain to the source of the world, and then I found that I was an object in the midst of other objects." He clarifies this assertion and alerts us to the stakes which the optimistic assumptions of film studies and cultural studies, the counterhegemonic promise of alternative cinema, and the emancipatory project of coalition politics cannot account for, when he writes: "Ontology—once it is finally admitted as leaving existence by the wayside—does not permit us to understand the being of the black."6 This presents a challenge to film production and to film studies given their cultivation and elaboration by the imaginative labor of cultural studies, underwritten by the assumptive logic of Humanism; because if everyone does not possess the DNA of culture, that is, (a) time and space transformative capacity, (b) a relational status with other Humans through which one's time- and space-transformative capacity is recognized and incorporated, and (c) a relation to violence that is contingent and not gratuitous, then how do we theorize a sentient being who is positioned not by the DNA of culture but by the structure of gratuitous violence? How do we think outside of the conceptual framework of subalternity—that is, outside of the explanatory power of cultural studies—and think beyond the pale of emancipatory agency by way of

symbolic intervention? I am calling for a different conceptual framework, predicated not on the subject-effect of cultural performance but on the structure of political ontology, a framework that allows us to substitute a culture of politics for a politics of culture. The value in this rests not simply in the way it would help us rethink cinema and performance, but in the way it can help us theorize what is at present only intuitive and anecdotal: the unbridgeable gap between Black being and Human life. To put a finer point on it, such a framework might enhance the explanatory power of theory, art, and politics by

destroying and perhaps restructuring the ethical range of our current ensemble of questions. This has profound implications for non-Black film studies, Black film studies, and African American studies writ large because they are currently entangled in

a multicultural paradigm that takes an interest in an insufficiently critical comparative analysis— that is, a comparative analysis in pursuit of a coalition politics (if not in practice then at least as a theorizing metaphor) which, by its very nature, crowds out and forecloses the Slave's grammar of suffering.

Only a refusal to create a distance from the pathology of blackness, to work inside of it can produce the end of the world, and therefore sociality. Sexton 10 (Jared Sexton, Director, African American Studies School of Humanities , Associate Professor, African American Studies School of Humanities, Associate Professor, Film & Media Studies

Page 278: openev.debatecoaches.org€¦  · Web view1AC . Observation 1: If I could find the spot where truth echoesI would stand there and whisper memories of my children's future. I would

School of Humanities at University of California Irvine, “The Social Life of Social Death: On Afro-Pessimism and Black Optimism”)What I find most intriguing about the timbre of the argument of “The Case of Blackness,” and the black optimism it articulates against a certain

construal of afro-pessimism, is the way that it works away from a discourse of black pathology only to swerve right back into it as an ascription to those found to be taking up and holding themselves in “the stance of the pathologist” in relation to black folks. I say this not only because there is, in this version of events, a recourse to psychoanalytic

terminology (“fetishization,” “obsession,” “repetition,”), but also because there is at the heart of the matter a rhetorical question that establishes both the bad advice of a wild analysis and a tacit diagnosis affording a certain speaker’s benefit: “So why is it repressed?” The “it” that has been afflicted by the psychopathology of obsessional neurosis is the understanding, which is also to say the celebration, of the ontological priority or previousness of blackness relative to the antiblackness that establishes itself against it, a priority or previousness that is also termed “knowledge of freedom” or, pace Chandler, comprehension of “the constitutive force of

the African American subject(s)” (Chandler 2000: 261). [21] What does not occur here is a consideration of the possibility that something might be unfolding in the project or projections of afro-pessimism “knowing full well the danger of a kind of negative reification” associated with its analytical claims to the paradigmatic (Moten 2004: 279). That is to say, it might just be the case that an object lesson in the phenomenology of the thing is a gratuity that folds a new encounter into older habits of thought through a reinscription of (black) pathology that reassigns its cause and relocates its source without ever really getting inside it. In a way, what we’re talking about relates not to a disagreement about “unthought

positions” (and their de-formation) but to a disagreement, or discrepancy, about “unthought dispositions” (and their

in-formation). I would maintain this insofar as the misrecognition at work in the reading of that motley crew listed in the ninth

footnote regards, perhaps ironically, the performative dimension or signifying aspect of a “generalized impropriety” so improper as to appear as the same old propriety returning through the back door . Without sufficient consideration of the gap between statement and enunciation here, to say nothing of quaint notions like context or audience or historical conjuncture, the discourse of afro-pessimism, even as it approaches otherwise important questions, can only seem like a “tragically neurotic” instance of “certain discourse on the relation between blackness and death” (Moten 2007: 9).xiii Fanon and his interlocutors, or what appear rather as his fateful adherents, would seem to have a problem embracing black social life because they never really come to believe in it, because they cannot acknowledge the social life from which they speak and of which they speak—as negation and impossibility—as their own (Moten 2008: 192). Another way of putting this might be to say that they are caught in a performative contradiction enabled by disavowal. I wonder, however, whether things are even this clear in Fanon and the readings his writing might facilitate. Lewis Gordon’s sustained engagement finds Fanon situated in an ethical stance grounded in the affirmation of blackness in the historic antiblack world. In a response to the discourse of multiracialism emergent in the late twentieth-century United States, for instance, Gordon writes, following Fanon, that “there is no way to reject the thesis that there is something wrong with being black beyond the willingness to ‘be’ black – in terms of convenient fads of playing blackness, but in paying the costs of antiblackness on a global scale. Against the raceless credo, then, racism cannot be rejected without a dialectic in which humanity experiences a blackened world” (Gordon 1997: 67). What is this willingness to ‘be’ black, of choosing to be black affirmatively rather than reluctantly, that Gordon finds as the key ethical moment in Fanon? [23] Elsewhere, in a discussion of Du Bois on the study of black folk, Gordon restates an existential phenomenological conception of the antiblack world developed across his first several books: “Blacks here suffer the phobogenic reality posed by the spirit of racial seriousness. In effect, they more than symbolize or signify various

social pathologies—they become them. In our antiblack world, blacks are pathology” (Gordon 2000: 87). This conception would seem to support Moten’s contention that even much radical black studies scholarship sustains the association of blackness with a certain sense of decay and thereby fortifies and extends the interlocutory life of widely accepted political common sense. In fact, it would seem that Gordon deepens the already problematic association to the level of identity. And yet, this is precisely what Gordon argues is the value and insight of Fanon: he fully accepts the definition of himself as pathological as it is imposed by a world that knows itself through that imposition, rather than remaining in a reactive stance that insists on the

(temporal, moral, etc.) heterogeneity between a self and an imago originating in culture. Though it may appear counterintuitive, or rather because it is counterintuitive, this acceptance or affirmation is active; it is a willing or willingness, in other words, to pay whatever social costs accrue to being black, to inhabiting blackness, to living a black social life under the shadow of social death. This is not an accommodation to

Page 279: openev.debatecoaches.org€¦  · Web view1AC . Observation 1: If I could find the spot where truth echoesI would stand there and whisper memories of my children's future. I would

the dictates of the antiblack world. The affirmation of blackness, which is to say an affirmation of pathological being, is a refusal to distance oneself from blackness in a valorization of minor differences that bring one closer to health, to life, or to sociality. Fanon writes in the first chapter of Black Skin, White Masks, “The Black Man and Language”: “A Senegalese who learns Creole to pass for Antillean is a case of alienation. The Antilleans who make a mockery out of him are lacking in judgment” (Fanon 2008: 21). In a world structured by the twin axioms of white superiority and black inferiority, of white existence and black nonexistence, a world structured by a negative categorical imperative—“above all, don’t be black” (Gordon 1997: 63)—in this world, the zero degree of transformation is the turn toward blackness, a turn toward the shame, as it were, that

“resides in the idea that ‘I am thought of as less than human’” (Nyong’o 2002: 389).xiv In this we might create a transvaluation of pathology itself, something like an embrace of pathology without pathos .

Page 280: openev.debatecoaches.org€¦  · Web view1AC . Observation 1: If I could find the spot where truth echoesI would stand there and whisper memories of my children's future. I would

Link - HegemonyUS hegemony is just the racial violence of America gone global –aff claims to benevolence are symptoms of white privilegeRodriguez ‘07 [Dylan, PhD in Ethnic Studies Program of the University of California Berkeley and Associate Proffessor of Ethnic Studies at University of California Riverside, “American Globality And the US Prison regime: State Violence And White Supremacy from Abu Ghraib to Stockton to bagong diwa”, Ateneo de Manila University, 2007, Kritika Kultura 9 (2007): 022-048]

In fact, the notion of American globality I have begun discussing here already exceeds negri and Hardt’s formulation to the extent that it is a global racial formation, and more pointedly a global mobilization of a white supremacist social formation (read: a united States of America formed by the social-economic geographies of racial chattel slavery and their recodification through the post-13th

Amendment innovation of other technologies of criminalization and imprisonment). The US prison regime’s production of human immobilization and death composes some of the fundamental modalities of American national coherence .

It inscribes two forms of domination that tend to slip from the attention of political theorists, including Negri and Hardt: first, the prison regime

strategically institutionalizes the biopolitical structures of white racial/nationalist ascendancy—it quite concretely provides a definition for white American personhood, citizenship, freedom, and racialized patriotism. Second, the prison regime reflects the moral, spiritual, and cultural inscription of Manifest Destiny (and its descendant material cultural and state-building articulations of racist and white supremacist conquest, genocide, and population control) across different historical moments. to invoke and

critically rearticulate negri and Hardt’s formulation, the focal question becomes: How does the right of the US-as-global police to kill, detain, obliterate become voiced, juridically coded, and culturally recoded? the structure of presumption—and therefore relative political silence—enmeshing the prison’s centrality to the logic of American globality is precisely evidence of the fundamental power of the uS prison regime within the larger schema of American hegemony. In this sense the uS prison regime is ultimately really not an “institution.” rather it is a formulation of world order (hence, a dynamic and perpetual labor of institutionalization rather than a definitive modernist institution) in which massively scaled, endlessly strategized technologies of human immobilization address (while never fully resolving) the socio-political crises of globalization. The US prison regime defines a global logic of social organization that constitutes, mobilizes, and prototypes across various localities. What would it mean, then, to consider state-crafted, white supremacist modalities of imprisonment as the perpetual end rather than the self-contained means of American globality? I am suggesting a conception of the prison regime that focuses on what cultural and political theorist Allen Feldman calls a “formation of violence,” which anchors the contemporary articulation of white supremacy as a global technology of coercion and hegemony. Feldman writes, the growing autonomy of violence as a self-legitimating sphere of social discourse and transaction points to the inability

of any sphere of social practice to totalize society. Violence itself both reflects and accelerates the experience of society as an incomplete project, as something to be made. As a formation of violence that self-perpetuates a peculiar social project through the discursive structures of warfare, the US prison regime composes an acute formation of racial and white supremacist violence, and thus houses the capacity for mobilization of an epochal (and peculiar) white supremacist global logic. This contention should not be confused with the sometimes parochial (if not politically chauvinistic) proposition that American state and state-sanctioned regimes of bodily violence and human immobilization are somehow self-contained “domestic” productions that are exceptional to the united

States of America, and that other “global” sites simply “import,” imitate, or reenact these institutionalizations of power. In fact, I am suggesting the opposite: the US prison regime exceeds as it enmeshes the ensemble of social relations that cohere uS civil society, and is fundamental to the geographic transformations, institutional vicissitudes, and militarized/economic mobilizations of “globalization” generally . to assert this, however, is to also argue that the constituting violence of the US prison regime has remained somewhat undertheorized and objectified in the overlapping realms of public discourse, activist mobilization, and (grassroots as well as professional) scholarly praxis.

Here I am arguing that it is not possible to conceptualize and critically address the emergence and global proliferation of the (uS/global) prison industrial complex outside a fundamental understanding of what are literally its technical and technological premises: namely, its complex organization and creative production of racist and white supremacist bodily violence . It is only in this context, I would say, that we can examine the problem of how “the Prison” is a modality (and not just a reified product or outcome) of American statecraft in the current political moment. It is only a theoretical foregrounding of the white supremacist state and social formation of the united States that will allow us to understand the uS prison regime as an American globality that materializes as it prototypes state violence and for that matter, “state power” itself through a specific institutional site.

Page 281: openev.debatecoaches.org€¦  · Web view1AC . Observation 1: If I could find the spot where truth echoesI would stand there and whisper memories of my children's future. I would

Link - Performance Their faith in performance and conceptual rupturing as an act of emancipation trades off with structural analysis and puts false place in subjectivity Wilderson 2010, [2010, Frank B. Wilderson is an Associate Professor of African-American Studies at UC Irvine and has a Ph.D. from UC Berkeley, “Red, White & Black: Cinema and the Structure of U.S. Antagonisms,”]

Unfortunately, cultural studies that theorizes the interface between Blacks and Humans is hobbled in its attempts to (a) expose power

relationships and (b) examine how relations of power influence and shape cultural practice. Cultural studies insists on a grammar of suffering which assumes that we are all positioned essentially by way of the symbolic order , what Lacan

calls the wall of language—and as such our potential for stasis or change (our capacity for being oppressed or free) is overdeter-mined by our "universal" ability or inability to seize and wield discursive weapons. This idea corrupts the explanatory power of most socially engaged films and even the most radical line of political action because it produces a cinema and a politics that cannot account for the grammar of suffering of the Black—the Slave. To put it bluntly, the

imaginative labor5 of cinema, political action, and cultural studies are all afflicted with the same theoretical aphasia. They are speechless in the face of gratuitous violence. This theoretical aphasia is symptomatic of a debilitated ensemble of questions regarding political ontology. At its heart are two registers of imaginative labor. The first register is that of description, the rhetorical labor aimed at explaining the way relations of power are named, categorized, and explored. The second

register can be characterized as prescription, the rhetorical labor predicated on the notion that everyone can be emancipated through some form of discursive, or symbolic, intervention. But emancipation through some form of discursive or symbolic intervention is wanting in the face of a subject position that is not a subject position—what Marx calls "a speaking implement" or what Ronald Judy calls "an interdiction against subjectivity." In other words,

the Black has sentient capacity but no relational capacity. As an accumulated and fungible object, rather than an exploited and alienated subject, the Black is openly vulnerable to the whims of the world, and so is his or her cultural "production." What does it mean—what are the stakes—when the world can whimsically transpose one's cultural gestures, the stuff of symbolic intervention, onto another worldly good, a commodity of style? Frantz Fanon echoes this question when he writes, "I came into the world imbued with the will to find a meaning in things, my spirit filled with the desire to attain to the source of the world, and then I found that I was an object in the midst of other objects." He clarifies this assertion and alerts us to the stakes which the optimistic assumptions of film studies and cultural studies, the counterhegemonic promise of alternative cinema, and the emancipatory project of coalition politics cannot account for, when he writes: "Ontology—once it is finally admitted as leaving existence by the wayside—does not permit us to understand the being of the black."6 This presents a challenge to film production and to film studies given their cultivation and elaboration by the imaginative labor of cultural studies, underwritten by the assumptive logic of Humanism; because if everyone does not possess the DNA of culture, that is, (a) time and space transformative capacity, (b) a relational status with other Humans through which one's time- and space-transformative capacity is recognized and incorporated, and (c) a relation to violence that is contingent and not gratuitous, then how do we theorize a sentient being who is positioned not by the DNA of culture but by the structure of gratuitous violence? How do we think outside of the conceptual framework of subalternity—that is, outside of the explanatory power of cultural studies—and think beyond the pale of emancipatory agency by way of

symbolic intervention? I am calling for a different conceptual framework, predicated not on the subject-effect of cultural performance but on the structure of political ontology, a framework that allows us to substitute a culture of politics for a politics of culture. The value in this rests not simply in the way it would help us rethink cinema and performance, but in the way it can help us theorize what is at present only intuitive and anecdotal: the unbridgeable gap between Black being and Human life. To put a finer point on it, such a framework might enhance the explanatory power of theory, art, and politics by

destroying and perhaps restructuring the ethical range of our current ensemble of questions. This has profound implications for non-Black film studies, Black film studies, and African American studies writ large because they are currently entangled in

a multicultural paradigm that takes an interest in an insufficiently critical comparative analysis— that is,

Page 282: openev.debatecoaches.org€¦  · Web view1AC . Observation 1: If I could find the spot where truth echoesI would stand there and whisper memories of my children's future. I would

a comparative analysis in pursuit of a coalition politics (if not in practice then at least as a theorizing metaphor) which, by its very nature, crowds out and forecloses the Slave's grammar of suffering.

Page 283: openev.debatecoaches.org€¦  · Web view1AC . Observation 1: If I could find the spot where truth echoesI would stand there and whisper memories of my children's future. I would

Link - Queerness Thinking queerness as an identity, performance or set of practices obscures the prior ontological queerness of blackness, which exists as the absolute index of otherness over and against which humanity and modernity vouchsafe their value and coherence. This failure to theorize gender and sexuality from the underside of the human is a failure to interrogate the very template of deviance that haunts queerness, ensuring anti-black homonormative identification.Jackson 2011 [Zakiyyah, PhD candidate in African diaspora studies in the African American studies department at the University of California, Berkeley, “Waking Nightmares,” GLQ: A Journal of Lesbian and Gay Studies, Vol. 17, No. 2-3]

Marriott’s scholarship reminds us that queer theory may unwittingly diminish its criticality if it fails to acknowledge the role antiblack racism plays in shaping the discursive practices of gender and sexuality. The violence that produces blackness necessitates that from the existential vantage point of black lived experience, gender and sexuality lose their coherence as normative categori es .8 Moreover, as queer theory attempts to map a territory that encompasses an increasingly generalized nonnormativity , it may unwittingly overlook the function of blackness in modernity, since the black body has been rendered the “absolute index of otherness.” 9

While particular nonblack sexual and gendered practices may be queered, blackness serves as an essential template of gendered and sexual “deviance” that is limited to the negation not of a particular practice but of a state of being. In other words, there are no practices that an individual black person can take up that will settle once and for all the doubt that accompanies the assertion of a black humanity . Marriott’s texts encourage us to interrogate the subject of feminist and queer theory rather than presume that a subject is always and already there. Marriott’s writing invites us to reflect on aspects of gendered and sexualized racial experience that often go unaccounted for in scholarly work on race, despite the efforts of black feminism and its theories of intersectionality. Feminists of color have encouraged us to think about gender and sexuality as they intersect with the particularities of race and embodiment. In the context of blackness, gender oppression not only circumscribes the life chances of women but also stratifies or suspends the category of manhood.10 Black men are seen as “excessively male and insufficiently masculine.”11 Historically, black men’s “inversion” has served as an alibi for their rape and castration, painful reminders that rape is, as

feminists state, “about power” rather than contingent on an essentialized female vulnerability or an inherent male power. Existential negation , which we refer to as “race” in polite conversation , substantially complicates our theorization of “black

patriarchy” and “black sexuality.” It requires us to theorize gendered and sexual violence from the underside of “the human,” which arguably necessitates that we think about queerness as something other than an identity, gender, or even set of sexual practices . We might think of black queerness as an existential matter rather than as an attribution that accompanies only some black subjectivities . Marriott reminds us that nonbeing is the existential burden facing black people under the conditions of (post)modernity and also the specter that haunts queer

subjectivity. This is fitting considering that the birth of “homosexuality” is inextricable from the rise of scientific authority and its racism. Fantasies of blackness , particularly black female sexuality, are the gendered and racial specters that haunt queerness — that from which homonormative subjects must distance themselves in order to be properly recognized as humans , as citizens, as subjects . Despite prior interdictions on same-sex sexuality, it is only as recently as the late nineteenth century that sexual acts and desires became constitutive of identity: the

homosexual becomes a type.12 In Siobhan Somerville’s “Scientific Racism and the Invention of the Homosexual Body,” she queries, “ is it merely a historical coincidence that the classification of bodies as either ‘homosexual’ or ‘heterosexual’ emerged at the same time that the United States was aggressively policing the imaginary boundary between ‘black’ and ‘white’ bodies?”13 Somerville goes on to suggest the mutually constitutive effects of the bifurcated

categories of race and sexuality, their structural interdependence and mutual production. Structures and methodologies that drove dominant scientific ideologies of race were subsequently taken up in the scientific pursuit of an

Page 284: openev.debatecoaches.org€¦  · Web view1AC . Observation 1: If I could find the spot where truth echoesI would stand there and whisper memories of my children's future. I would

emerging discourse of sexuality. Difference was thought to be a visualizable fact inscribed on the body; according to this logic, interiority could be read on the surface of the body’s anatomical markers. Racial difference seemed to hinge on and be most represented by the

supposed differences of sexual appetites and anatomies, particularly those of the African female. Sexologists drew on fantasies of black female embodiment as their model of sexual deviancy and gender nonconformity. Racial comparative-anatomy methods were used to determine sexual definition, with a presumed similitude between “deviant” white bodies and the black body. The word homosexual itself seemed to conjure some anxieties about miscegenation, as the “barbarously hybrid word” was a mix of Latin and Greek, even referring to “shades of gender” and “sexual half-breeds.” 14 Reading Marriott in the context of feminist and queer theory offers new insight into the gendered and sexualized nature of blackness’s

ontological negation, particularly the nonheteronormativity of race’s reproduction. The negation of blackness is the foundation of ethics and politics, even of modern sociality itself; this negation overdetermines black practices as criminal, queer, nationally polluting, and pathological.15

Page 285: openev.debatecoaches.org€¦  · Web view1AC . Observation 1: If I could find the spot where truth echoesI would stand there and whisper memories of my children's future. I would

ALTSThe alternative is an unflinching paradigmatic analysis of the uncivility of civil society – the reclamation of the power to pose the question and analysis of ontological questions to articulate the worldWilderson 10 (Frank Wilderson, Professor of African American Studies at UC Irvine, “Red, White & Black: Cinema and the Structure of U.S. Antagonisms”)

STRANGE AS it might seem, this book project began in South Africa. During the last years of apartheid I worked for revolutionary change in both an underground and above-ground capacity, for the Charterist Movement in general and the ANC in particular. During this period, I began to see how essential an unflinching paradigmatic analysis is to a movement dedicated to the complete overthrow of an existing order. The neoliberal compromises that the radical elements of the Chartist Movement made with the moderate elements were due, in large part, to our inability or unwillingness to hold the moderates' feet to the fire of a political agenda predicated on an unflinching paradigmatic analysis. Instead, we allowed our energies and points of attention to be displaced by and onto pragmatic

considerations. Simply put, we abdicated the power to pose the question—and the power to pose the question is the greatest power of all. Elsewhere, I have written about this unfortunate turn of events (Incognegro: A Memoir of Exile and Apartheid), so I'll not rehearse the details here. Suffice it to say, this book germinated in the many political and academic discussions and debates that I was fortunate enough to be a part of at a historic moment and in a place where the word revolution was spoken in earnest, free of qualifiers and irony. For their past and ongoing ideas and interventions, I extend solidarity and appreciation to comrades Amanda Alexander, Franco Barchiesi, Teresa Barnes, Patrick Bond, Ashwin Desai, Nigel Gibson, Steven Greenberg, Allan Horowitz, Bushy Kelebonye (deceased), Tefu Kelebonye, Ulrike Kistner, Kamogelo Lekubu, Andile Mngxitama, Prishani Naidoo, John Shai, and S'bu Zulu. .¶ [CONTINUES]¶ In the Introduction and chapter 1, we saw how the aporia between Black being and political ontology has existed since Arab and European

enslavement of Africans. The crafting of questions through which one might arrive at an unflinching paradigmatic analysis of political ontology , a language that could express the structural and performative violence of Slave-making, is repeatedly thwarted. Humanist discourse, whose epistemological machinations provide our conceptual frameworks for thinking political ontology, is diverse and contrary. But for all its diversity and contrariness it is sutured by an implicit rhetorical consensus that violence accrues to the Human body as a result of transgressions, whether real or imagined, within the symbolic order. That is to say, Humanist discourse can only think a subject’s relation to violence as a contingency and not as a matrix that positions the subject. Put another way, Humanism has no theory of the Slave because it imagines a subject who has been either alienated in language or alienated from his or her cartographic and temporal capacities. 1 It cannot imagine an object who has been positioned by gratuitous violence and who has no car-

tographic and temporal capacities to lose—a sentient being for whom recognition and incorporation is impossible. In short, political ontology, as imagined through Humanism, can only produce discourse that has as its foundation alienation and exploitation as a grammar of suffering, when what is needed (for the Black, who is always already a Slave) is an ensemble of ontological questions that has as its foundation accumulation and fungibility as a grammar of suffering. 2 A Culture of Politics The violence of the Middle Passage and the Slave estate, 3 technologies of accumulation and fungibility, recompose and reenact their horrors on each succeeding generation of Blacks. This violence is both gratuitous (not contingent on transgressions against the hegemony of civil society) and structural (positioning Blacks ontologically outside of Humanity and civil society). Simultaneously, it renders the ontological status of Humanity (life itself ) wholly dependent on civil society’s repetition compulsion: the frenzied and fragmented machinations through which civil society reenacts gratuitous violence on the Black— that civil society might know itself as the domain of Humans— generation after generation. Again, we need a new language of

abstraction to explain this horror. The explanatory power of Humanist discourse is bankrupt in the face of the Black. It is inadequate and inessential to, as well as parasitic on, the ensemble of questions which the dead but sentient thing, the Black, struggles to articulate in a world of living subjects.

Page 286: openev.debatecoaches.org€¦  · Web view1AC . Observation 1: If I could find the spot where truth echoesI would stand there and whisper memories of my children's future. I would

Black NihilismThe alternative is black nihilism --- refusing political hope is the only metaphysically coherent response to the constant slaughter of black bodies Warren 15 [Calvin K., Assistant Professor of American Studies at George Washington University, “Black Nihilism and the Politics of Hope,” CR: The New Centennial Review, Volume 15, Number 1, Spring 2015]

V. Conclusion Throughout this essay, I have argued that the Politics of hope preserve metaphysical structures that sustain black suffering. This preservation amounts to an exploitation of hope—when the Political colonizes the spiritual principle of hope and puts it in the service of extending the “will to power” of an anti-black organization of existence. The Politics of hope, then, is bound up with metaphysical violence, and

this violence masquerades as a “solution” to the problem of anti-blackness. Temporal linearity, perfection, betterment, struggle, work, and utopian futurity are conceptual instruments of the Political that will never obviate black suffering or anti-black violence; these concepts only serve to reproduce the conditions that render existence unbearable for blacks . Political theologians and black optimists avoid the immediacy of black suffering, the horror of anti-black pulverization, and place relief in a “not-yet-but-is (maybe)-to-come-social order” that, itself,

can do little more but admonish blacks to survive to keep struggling . Political hope becomes a vicious and abusive cycle of struggle —it mirrors the Lacanian drive, and we encircle an object (black freedom, justice, relief, redress,

equality, etc.) that is inaccessible because it doesn’t really exist. The political theologian and black optimist, then, propose a collective Jouissance as an answer to black suffering—finding the joy in struggle, the victory in toil, and the

satisfaction in inefficacious action. We continue to “struggle” and “work” as black youth are slaughtered daily, black bodies are incarcerated as forms of capital, black infant mortality rates are soaring, and hunger is disabling the bodies, minds, and spirits of desperate black youth. In short, these conditions are deep metaphysical problems—the sadistic pleasure of metaphysical domination—and “work ” and “struggle” avoid the terrifying fact that the world depends on black death to sustain itself . Black nihilism attempts to break this “drive”—to stop it in its tracks, as it were—and to end the cycle of insanity that political hope perpetuates. The question that remains is a question often put to the black nihilist: what is the point? This compulsory geometrical structuring of thought— all knowledge must submit to, and is reducible to, a point —it is an epistemic flicker of certainty, determination, and, to put it bluntly, life. “The point” exists for life; it enlivens, enables, and sustains knowledge. Thought outside of this mandatory point is illegible and useless. To write outside of the “episteme of life” and its grammar will require a position outside of this point , a position somewhere in the infinite horizon of thought (perhaps this is what Heidegger wanted to do with his reconfiguration of thought). Writing in this way is inherently subversive and refuses the geometry of thought. Nevertheless, the [End Page 243] nihilist is forced to enunciate his refusal through a “point,” a point that is contradictory and paradoxical all at once. To say that the point of this essay is that “the point” is fraudulent—its promise of clarity and life are inadequate—will not satisfy the hunger of disciplining the nihilist and insisting that one

undermine the very ground upon which one stands. Black nihilistic hermeneutics resists “the point ” but is subjected to it to have one’s voice heard within the marketplace of ideas. The “point” of this essay is that political hope is pointless . Black suffering is an essential part of the world, and placing hope in the very structure that sustains metaphysical violence , the Political, will never resolve anything . This is why the black

nihilist speaks of “exploited hope,” and the black nihilist attempts to wrest hope from the clutches of the Political . Can we think of hope outside the Political? Must “salvation” translate into a political grammar or a political program? The nihilist , then, hopes for the end of political hope and its metaphysical violence. Nihilism is not antithetical to hope; it does not extinguish hope but reconfigures it. Hope is the foundation of the black nihilistic hermeneutic. In “Blackness and Nothingness,” Fred Moten (2013) conceptualizes blackness as a “pathogen” to metaphysics, something that has the ability to unravel, to disable, and to destroy anti-blackness. If we read Vattimo through Moten’s brilliant analysis, we can suggest that blackness is the limit that Heidegger and Nietzsche were really after. It is a “blackened” world that will ultimately end

Page 287: openev.debatecoaches.org€¦  · Web view1AC . Observation 1: If I could find the spot where truth echoesI would stand there and whisper memories of my children's future. I would

metaphysics, but putting an end to metaphysics will also put an end to the world itself—this is the nihilism that the black nihilist must theorize through. This is a far cry from what we call “anarchy,” however. The black nihilist has as little faith in the metaphysical reorganization of society through anarchy than he does in traditional forms of political

existence. The black nihilist offers political apostasy as the spiritual practice of denouncing metaphysical violence, black suffering, and the idol

of anti-blackness. The act of renouncing will not change political structures or offer a political program ; instead,

it is the act of retrieving the spiritual concept of hope from the captivity of the Political . Ultimately, it is impossible to end metaphysics without ending blackness, and the black nihilist will never be able to withdraw from the Political completely without a certain death-drive or being-toward-death. This is the essence of black suffering : the lack of reprieve from metaphysics, the tormenting complicity in the reproduction of violence, and the lack of a coherent grammar to articulate these dilemmas . After contemplating these issues for some time in my office, I decided to take a train home. As I awaited my train in the station, an older black woman asked me about the train schedule and when I would expect the next train headed toward Dupont Circle. When I told her the trains

were running slowly, she began to talk about the government shutdown. “They don’t care anything about us, you know,” she

said. “We elect these people into office, we vote for them, and they watch black people suffer and have no intentions of doing anything about it.” I shook my head in agreement and listened intently. “I’m going to stop voting,

and supporting this process; why should I keep doing this and our people continue to suffer,” she said. I looked at her

and said, “I don’t know ma’am; I just don’t understand it myself.” She then laughed and thanked me for listening to her—as if our conversation were somewhat cathartic. “You know, people think you’re crazy when you say things like this,” she said giving me a wink. “Yes they do,” I said. “But I am a free woman,” she emphasized “and I

won’t go back.” Shocked, I smiled at her, and she winked at me; at that moment I realized that her wisdom and courage penetrated my mind and demanded answers. I’ve thought about this conversation for some time, and it is for this reason I had

to write this essay. To the brave woman at the train station, I must say you are not crazy at all but thinking outside of metaphysical time, space, and violence . Ultimately, we must hope for the end of political hope.

Page 288: openev.debatecoaches.org€¦  · Web view1AC . Observation 1: If I could find the spot where truth echoesI would stand there and whisper memories of my children's future. I would

PathologyOnly a refusal to create a distance from the pathology of blackness, to work inside of it can produce the end of the world, and therefore sociality. Sexton 10 (Jared Sexton, Director, African American Studies School of Humanities , Associate Professor, African American Studies School of Humanities, Associate Professor, Film & Media Studies School of Humanities at University of California Irvine, “The Social Life of Social Death: On Afro-Pessimism and Black Optimism”)What I find most intriguing about the timbre of the argument of “The Case of Blackness,” and the black optimism it articulates against a certain

construal of afro-pessimism, is the way that it works away from a discourse of black pathology only to swerve right back into it as an ascription to those found to be taking up and holding themselves in “the stance of the pathologist” in relation to black folks. I say this not only because there is, in this version of events, a recourse to psychoanalytic

terminology (“fetishization,” “obsession,” “repetition,”), but also because there is at the heart of the matter a rhetorical question that establishes both the bad advice of a wild analysis and a tacit diagnosis affording a certain speaker’s benefit: “So why is it repressed?” The “it” that has been afflicted by the psychopathology of obsessional neurosis is the understanding, which is also to say the celebration, of the ontological priority or previousness of blackness relative to the antiblackness that establishes itself against it, a priority or previousness that is also termed “knowledge of freedom” or, pace Chandler, comprehension of “the constitutive force of

the African American subject(s)” (Chandler 2000: 261). [21] What does not occur here is a consideration of the possibility that something might be unfolding in the project or projections of afro-pessimism “knowing full well the danger of a kind of negative reification” associated with its analytical claims to the paradigmatic (Moten 2004: 279). That is to say, it might just be the case that an object lesson in the phenomenology of the thing is a gratuity that folds a new encounter into older habits of thought through a reinscription of (black) pathology that reassigns its cause and relocates its source without ever really getting inside it. In a way, what we’re talking about relates not to a disagreement about “unthought

positions” (and their de-formation) but to a disagreement, or discrepancy, about “unthought dispositions” (and their

in-formation). I would maintain this insofar as the misrecognition at work in the reading of that motley crew listed in the ninth

footnote regards, perhaps ironically, the performative dimension or signifying aspect of a “generalized impropriety” so improper as to appear as the same old propriety returning through the back door . Without sufficient consideration of the gap between statement and enunciation here, to say nothing of quaint notions like context or audience or historical conjuncture, the discourse of afro-pessimism, even as it approaches otherwise important questions, can only seem like a “tragically neurotic” instance of “certain discourse on the relation between blackness and death” (Moten 2007: 9).xiii Fanon and his interlocutors, or what appear rather as his fateful adherents, would seem to have a problem embracing black social life because they never really come to believe in it, because they cannot acknowledge the social life from which they speak and of which they speak—as negation and impossibility—as their own (Moten 2008: 192). Another way of putting this might be to say that they are caught in a performative contradiction enabled by disavowal. I wonder, however, whether things are even this clear in Fanon and the readings his writing might facilitate. Lewis Gordon’s sustained engagement finds Fanon situated in an ethical stance grounded in the affirmation of blackness in the historic antiblack world. In a response to the discourse of multiracialism emergent in the late twentieth-century United States, for instance, Gordon writes, following Fanon, that “there is no way to reject the thesis that there is something wrong with being black beyond the willingness to ‘be’ black – in terms of convenient fads of playing blackness, but in paying the costs of antiblackness on a global scale. Against the raceless credo, then, racism cannot be rejected without a dialectic in which humanity experiences a blackened world” (Gordon 1997: 67). What is this willingness to ‘be’ black, of choosing to be black affirmatively rather than reluctantly, that Gordon finds as the key ethical moment in Fanon? [23] Elsewhere, in a discussion of Du Bois on the study of black folk, Gordon restates an existential phenomenological conception of the antiblack world developed across his first several books: “Blacks here suffer the phobogenic reality posed by the spirit of racial seriousness. In effect, they more than symbolize or signify various

social pathologies—they become them. In our antiblack world, blacks are pathology” (Gordon 2000: 87). This conception would seem to support Moten’s contention that even much radical black studies scholarship sustains the association of blackness with a certain sense of decay and thereby fortifies and extends the interlocutory life of widely accepted political common sense. In fact, it would seem that Gordon deepens the already

Page 289: openev.debatecoaches.org€¦  · Web view1AC . Observation 1: If I could find the spot where truth echoesI would stand there and whisper memories of my children's future. I would

problematic association to the level of identity. And yet, this is precisely what Gordon argues is the value and insight of Fanon: he fully accepts the definition of himself as pathological as it is imposed by a world that knows itself through that imposition, rather than remaining in a reactive stance that insists on the

(temporal, moral, etc.) heterogeneity between a self and an imago originating in culture. Though it may appear counterintuitive, or rather because it is counterintuitive, this acceptance or affirmation is active; it is a willing or willingness, in other words, to pay whatever social costs accrue to being black, to inhabiting blackness, to living a black social life under the shadow of social death. This is not an accommodation to the dictates of the antiblack world. The affirmation of blackness, which is to say an affirmation of pathological being, is a refusal to distance oneself from blackness in a valorization of minor differences that bring one closer to health, to life, or to sociality. Fanon writes in the first chapter of Black Skin, White Masks, “The Black Man and Language”: “A Senegalese who learns Creole to pass for Antillean is a case of alienation. The Antilleans who make a mockery out of him are lacking in judgment” (Fanon 2008: 21). In a world structured by the twin axioms of white superiority and black inferiority, of white existence and black nonexistence, a world structured by a negative categorical imperative—“above all, don’t be black” (Gordon 1997: 63)—in this world, the zero degree of transformation is the turn toward blackness, a turn toward the shame, as it were, that

“resides in the idea that ‘I am thought of as less than human’” (Nyong’o 2002: 389).xiv In this we might create a transvaluation of pathology itself, something like an embrace of pathology without pathos .

Page 290: openev.debatecoaches.org€¦  · Web view1AC . Observation 1: If I could find the spot where truth echoesI would stand there and whisper memories of my children's future. I would

Maximum CaptivityWe must theorize slavery as maximum captivity in order to produce a structural analysis capable to think of the resistance necessary to disrupt that system. Sexton 8 (Jared Sexton, Director of African American Studies at UC Irvine, 2008, “Amalgamation Schemes: Antiblackness and the Critique of Multiracialism”, pages 111-114)

FYI: Randall Kennedy is “one of the first black scholars in this generation to pen a sustained argument advocating what he terms ‘a cosmopolitan ethos that welcomes the prospect of genuine, loving interracial intimacy’ ” (page 107-108)

In response to the last question, we examine several comments from Kennedy’s opening chapter, “In the Age of Slavery.” As noted, Kennedy is at pains to counter the claims of a certain black feminist history regarding the “extremity of power” exercised by

the slaveholder and “the absolute submission required of the slave ” (Hartman, quoted in Kennedy 2003, 532fn11). He is, in

other words, attempting to demonstrate, or at least to speculate upon, the limits of the slave system’s power of domination. Beyond this limit—whose locus proves frustratingly obscure—the agency of the slave herself was, we are told, able to affect significantly the conditions of captivity to alternate ends. Kennedy, in other words, proffers a narrative in which evidence of agency (evidence, that is, confirming an assumption of agency), however circumscribed or practically ineffective, is taken as a sign of resistance . More properly,

this is a narrative of resistant affection, an insistence that the dehumanizing social order of racial slavery was unable to achieve its ultimate goal—“the absolute submission of the slave”—because it could not overcome the irresistible force of affection between men and women, “regardless of color.” When all is said

and done, a human is still a human, as it were, and the family romance of normative heterosexuality persists “even within” hierarchies that preclude for the captive all of the recognizable (social, political, economic, cultural,

legal) trappings of “human being” in the modern sense. Here is Kennedy: The slave system failed , however, to

perfect the domination that [ Judge Thomas] Ruffin envisioned. It failed to bind the slaves so tightly as to deprive them of all room to maneuver. It failed to wring from them all prohibited yearnings. Slavery was, to be sure, a horribly oppressive system that severely restricted the ambit within which its victims could make decisions. But slavery did not extinguish altogether the possibility of choice.

(43) We might ask, what is the minimum ambit of decision making ? What sort of system, if not slavery, would bind one so tightly as to deprive one of all “room to maneuver”? Need a system of domination be “perfect” in order for it to be legally binding or socially effective or politically determinant ? Need the captive body be deprived of all room to maneuver for the situation to be considered one of extremity? Need the yearnings of slaves be wrung entirely from them for their prohibition to be considered a constitutive element of life? At what point does the quantitative measure of the slave’s bondage become difference of a qualitative sort? What precisely is the “choice” available under slavery, and is it one worthy of belaboring, one whose sphere of influence is to be considered newsworthy? To put a finer point on it, why is the categorical discrepancy refused between the free and the enslaved, or

more specifically, between the slave and the slaveholder? Is such refusal not tantamount to denying the very existence of slavery as a system that produced slaves rather than free people whose freedom was simply “severely restricted” or whose power was simply “severely limited” or who simply faced “difficult situations”? Kennedy continues: Bondage severely limited the power—including the sexual power—of slaves. But it did not wholly erase their capacity to attract and shape affectionate, erotic attachments of all sorts, including interracial ones. In a hard-to-quantify but substantial number of cases, feelings of affection and attachment between white male masters and their black female slaves somehow survived slavery’s deadening influence. The great difficulty, in any particular instance, lies in determining whether sex between a male master and a female slave was an expression of sexual autonomy or an act of unwanted sex. The truth is that most often we cannot know for sure, since

there exists little direct testimony from those involved, especially the enslaved women. (44) The inability to quantify the “number of

Page 291: openev.debatecoaches.org€¦  · Web view1AC . Observation 1: If I could find the spot where truth echoesI would stand there and whisper memories of my children's future. I would

cases” or, indeed, to “know for sure” anything about them does not prevent the author from considering them nonetheless

“substantial,” and the paucity of direct testimony,6 “especially [from] the enslaved women,” does not stop the author from extrapolating wildly about said “feelings of affection and attachment” between them and their “white male

masters.” In fact, it is the void in its place—the great historic silence—that enables both the reiteration of longstanding alibis for white male sexual violence—what Hartman (1997) discusses skillfully as the “ruses of seduction”—and the projection of this newfangled, though no less menacing, story about a maverick interracial intimacy that , almost undetectably, undermines the injunctions of white supremacy, serving not only as a sign of agency for enslaved women but a moment of their resistance as well . Their “sexual power” is expressed as the “capacity to attract”—and “somehow” to manipulate—the erotic attachments of white male slaveholders. There is here an unsubtle shift in terms: agency is not in itself subversive; indeed, the entire slave system derives, in large part, from the agency of the enslaved (its capture, manipulation, redeployment, etc.) (Chandler 2000).

Agency may be resistant or complicit or both , and it may or may not have practical effects in the world ; all of this can only be determined contextually . Much more troubling than Kennedy’s imprecision here, however, is

his entirely uncritical suggestion about the “sexual power” of slaves . Is not one of the principal conceits of power to suggest that though the dominant may monopolize power political, economic, and social, the dominated nonetheless enjoy a wily aptitude for “getting their way” by other means, namely, the ars

erotica of seduction? Is not one of the most pernicious elements of the proslavery discourse that the “attractiveness” of enslaved black women presents a threat of corruption to civilized white manhood and/or an internal guarantee against the excesses of state-sanctioned violence reserved for white slaveholders? The same quality that served as temptation was also, or alternately, taken to be that which would

forestall the descent of slaveholding into unrestrained brutality, an essential rationalization for the upholding of white (male) impunity toward blacks , whether enslaved or nominally “free” (Hartman 1997).7 Finally, was not the suggestion that enslaved black men might have the power to seduce white women (whether free or, in earlier

periods, indentured) one of the prime alibis for the construction of regulatory or prohibitory statutes around interracial marriage and sexual relations from the seventeenth century onward (Bardaglio 1999)? In each case, the focus on the “sexual power” of slaves was undoubtedly a displacement of the organized violence consistently required of captivity and, further, a dissimulation of the institutionalized sexual power of slaveholders in particular (whose authority not only foreclosed the possibility of prosecution and militated against the extralegal reprisals but also contributed immeasurably to their “capacity to attract and shape

affectionate, erotic attachments of all kinds.” The asymmetry here approaches the incommensurable—how, after all, would a slave go on to “court” a master? How would such an exercise in self-objectification, supplementing structural availability with an affirmation of “willingness,” rightly be called power?).

This is no less the case simply because for Kennedy the “sexual power” of slaves is something to honor or celebrate rather than to fear.

Page 292: openev.debatecoaches.org€¦  · Web view1AC . Observation 1: If I could find the spot where truth echoesI would stand there and whisper memories of my children's future. I would

Burn it Down We must burn civil society to the ground - there is no up from slavery. From segregation to neosegregation, the end of the 1AC’s progress is only the perfection of the slave - The legal system becomes the plantation, and we must kill it. Farley ’05 (Anthony Paul, Professor of Law @ Boston College, “Perfecting Slavery”, 1/27/2005, http://lawdigitalcommons.bc.edu/cgi/viewcontent.cgi?article=1028&context=lsfp –) NN

What is to be done? Two hundred years ago, when the slaves in Haiti rose up, they, of necessity, burned everything : They burned San Domingo flat so that at the end of the war it was a charred desert. Why do you burn everything?

asked a French officer of a prisoner. We have a right to burn what we cultivate because a man has a right to dispose of his own labour , was the reply of this unknown anarchist. The slaves burned everything because everything was against them. Everything was against the slaves, the entire order that it was their lot to follow, the entire order in which they were positioned as worse than senseless things, every plantation, everything. “Leave nothing white behind you,” said Toussaint to those dedicated to the end of white-over black. “God gave

Noah the rainbow sign. No more water, the fire next time.” The slaves burned everything, yes, but, unfortunately, they only burned everything in Haiti. Theirs was the greatest and most successful revolution in the history of the world but the failure of their fire to cross the waters was the great tragedy of the nineteenth century. At the dawn of the twentieth century, W.E.B. Du Bois wrote, “The colorline belts the world.” Du Bois said that the problem of the

twentieth century was the problem of the colorline. The problem, now, at the dawn of the twenty-first century is the problem of the colorline. The colorline continues to belt the world. Indeed, the slave power that is the U nited S tates now threatens an entire world with the death that it has become and so the slaves of yesterday, today, and tomorrow, those with nothing but their chains to lose, must, if they would be free, if they would escape slavery, win the entire world . We begin as children. We are called and we become our response to the

call. Slaves are not called. What becomes of them? What becomes of the broken-hearted? The slaves are divided souls, they are brokenhearted, the slaves are split asunder by what they are called upon to become. The slaves are called upon to become objects but objecthood is not a calling. The slave, then, during its loneliest loneliness, is divided from itself. This is schizophrenia. The slaves are not called, or, rather, the slaves are called to not be.

The slaves are called unfree but this the living can never be and so the slaves burst apart and die. The slaves begin as death, not as children, and death is not a beginning but an end. There is no progress and no exit from the undiscovered country of the slave, or so it seems. We are trained to think through a progress narrative, a grand narrative, the grandest narrative, that takes us up from slavery. There is no up from slavery. The progress from slavery to the end of history is the progress from white-over-black to white-over-black to white-overblack. The progress of slavery runs in the opposite direction of the past present future timeline . The slave only becomes the perfect slave at the end of the timeline, only under conditions of total juridical freedom. It is only under conditions of freedom, of bourgeois legality, that the slave can perfect itself as a slave by freely choosing to bow down before its master. The slave perfects itself as a slave by offering a prayer for equal rights . The system of marks is a plantation. The system of property is a plantation. The system of law is a plantation. These plantations, all part of the same system, hierarchy , produce white-overblack , white-over-black only, and that continually . The slave perfects itself as a slave through its prayers for equal rights. The plantation system will not commit suicide and the slave, as stated above, has knowing non-knowledge of this fact. The slave finds its way back from the undiscovered country only by burning down every

Page 293: openev.debatecoaches.org€¦  · Web view1AC . Observation 1: If I could find the spot where truth echoesI would stand there and whisper memories of my children's future. I would

plantation. When the slave prays for equal rights it makes the free choice to be dead, and it makes the free choice to not be. Education is the call. We are called to be and then we become something. We become that which we make of ourselves. We follow the call, we pursue a

calling. Freedom is the only calling—it alone contains all possible directions, all of the choices that may later blossom into the fullness of our lives. We can only be free. Slavery is death. How do slaves die? Slaves are not born, they are made. The slave must be trained to be that which the living cannot be. The only thing that the living are not free to be is dead. The slave must be trained to follow the call that is not a call. The slave must be trained to pursue the calling that is

not a calling. The slave must be trained to objecthood. The slave must become death. Slavery is white-over-black. White-over-black is death. White-over-black, death, then, is what the slave must become to pursue its calling that is not a calling.

Page 294: openev.debatecoaches.org€¦  · Web view1AC . Observation 1: If I could find the spot where truth echoesI would stand there and whisper memories of my children's future. I would

Ableism

Affective communication is fundamentally tied to a subject’s corporeal existence where the body acts as a medium of one’s acting which is an assumption of ability- for folks with cognitive disabilities the body can rigidify and stand in resistance to one’s political engagement- the impact is the demonization of disabled bodiesSlaby 2012 (Jan[Professor of philosophy at Free University Berlin, Germany. PhD thesis in Cognitive Science, University of Osnabrück. Thesis on the world-directedness of emotion; German title: Gefühl und Weltbezug; Grade: summa cum laude. MA Thesis in Philosophy from Humboldt University], April 2012, “Affective Self-Construal and the Sense of Ability” Freie Universität Berlin, Germany Emotion Review Vol. 4, No.2, sagepub, accessed 10/11) JA

Corporealization and Self-Construal in Depression It makes sense to combine the idea of a changeable existential background with the notion of self-feeling and with the idea that the basic “location” of affective self-construal is in the 154 Emotion Review Vol. 4 No. 2 vicinity of a person’s active

engagement with the world. The basic sense of ability and agency, intimately tied to an agent’s corporeal existence—the lived body as the medium of one’s acting and being acted upon—comprises the basic dimension of affective self-relatedness from which all other dimensions originate . It is here where we have to look for the contents and processes relevant to affective self-construal, and it is this dimension that seems particularly vulnerable to pathological modification. Affective changes occurring in depressive illness seem to pertain to this dimension of corporeal–agentive self-feeling , as many autobiographic patient narratives vividly illustrate.7 Depending on my sense of ability and (potential or actual) agency—a sense that is surely

varied and multiple—and as a consequence thereof, the world appears as a space of specific possibilities and as devoid of other possibilities, or as something removed and unreachable, or as a site of impending disaster, of threat and danger. Likewise, my sense of other people as people depends upon my felt ability to connect , to make contact, to engage in interactions and also on my capacity to be reached or affectively touched by others , and my sense of being part of interpersonal relations or arrangements such as a certain group or community. And when I feel fundamentally unable to connect, other people will as a consequence come to seem awkward, alien, probably even hostile and dangerous, appearing like strangers, and in the end even as not being persons

at all but instead as mere “dummies” or “soulless automata.” Reflected back to myself, this profound disconnectedness from others might in turn lead to feelings of solitude and alonenes s as there are no longer, in a certain sense, any others with whom one could make contact or be together. With this, we have moved right into the core dimension of the experiential world

described by sufferers from severe depression. Even more intimate than the relations described thus far is the link between the sense of ability and the feelings that one has of one’s own body. Once we understand the body phenomenologically as the active and

affectively responsive lived body, we see that there is no gap between our sense of ability and our sense of the body .

When the body , as part of a pathological process, ceases to be experienced as a smoothly operating medium of potential engagement with the world, it will increasingly turn into what is felt as a mere objec t —a transformation that has been called “ corporealization ” (Fuchs, 2003, 2005). In many depressed patients, the lived body seems to “rigidify” and turn into something resistant, into an obstacle and hindrance to one’s projected engagements and activities. One’s taken-for-granted relatedness to the world is altered completely as one no longer finds oneself within a context of seamless activity and amidst routinely encountered possibilities . The objectified, dysfunctional body amounts to a break between oneself and the

Page 295: openev.debatecoaches.org€¦  · Web view1AC . Observation 1: If I could find the spot where truth echoesI would stand there and whisper memories of my children's future. I would

world. Obviously, this marks a fundamental alteration in self-feeling: What we have here are bodily feelings of being trapped within oneself, feelings of being isolated from one’s formerly meaningful surroundings. This might give rise to feelings of being encaged

or imprisoned and of being unable to reach out to make contact with the world or with other people . As a further

consequence, feelings of estrangement, of depersonalization, even of not being bodily existent in the world any more might ensue—amounting to a fundamental sense of self-alienation . This dimension of self feelings—feelings of inability that are inextricable from severely modified, alienated corporeal experience and thus also from a radically altered sense of the reality of the world and of other people—seems to comprise the most basic level

of self-related feelings in depression. One last dimension presumably resulting from the affective changes under discussion is the experience of time. Depressed patients often complain about a radical disruption of everyday temporality. Most notably, the patients’ orientation towards the future as a temporal dimension potentially different from the present seems profoundly distorted. Often, time seems to stand still,

as the very idea of a potential change of the current state seems absent from experience. Again, the distortion of the patients’ sense of capability might account for this pathological change. Finding oneself unable to act and thus unable to effect any change in the world undermines the practical basis of the very idea of a change of situation (and correlatively, of a change of one’s own state). Being unable to act amounts to one’s being tied to the current state of affairs and thus to the present moment. The experienced present may in this way extend indefinitely and turn into what Heidegger

has called a “standing now” (cf. Heidegger, 1995, pp. 123–126). In light of this it is not surprising that this altered experience can give rise to strong feelings of dread and despair—emotions quite often reported by depressed persons .8

Thus, in all, the condition of clinical depression seems to present an encompassing “mirror image” of human affectivity. Depression marks a distortion of the root dimension of our affective orientation in the world and therefore can help us gain insight into the complex structure of human feelings. With regard to affective self-awareness and self-construal, depressive experience seems to confirm the suggestion that a basic form of self-awareness comes in view as a modification of a person’s sense of ability and capacity, a sense

that, in nonpathological cases, is often not distinct from action itself—in that case, it is a fundamental sense of agency. However, in depression, the capacity to act is severely distorted, leaving the patient with a debilitating sense of inability and impossibility . This sense of incapacity and impossibility then “infects” other practical and experiential dimensions, negatively affecting the depressed person’s relationship to the world in general, to other people, and certainly to her own embodied existence—it is always a distorted awareness of oneself.

Internalized ableism is perpetuated through the academy, where passing is used by academics to hide their impairments so that they aren’t seen as needy or troublemakers. Absent the 1AC, the psychological violence of internalized ableism continues –which reproduces passing as the self protection of their communitiesCampbell 08 (Fiona Kumari Campbell, disability author and professor at Griffith University, “Exploring internalized ableism using critical race theory” Disability & Society, Vol. 23, No. 2, March 2008, 151–162) sbb

In the case of disability subjectification internalization of negative ontologies of disability contributes to the formation of a docile and readily pliable disabled body, continuing in various ways to inhibit performances of disability

acceptance and rehabilitation so demanded by the inclusivist impulses of liberal contract theory. Internalized ableism can mean the disabled subject is caught ‘between a rock and a hard place’; in order to attain the benefit of a ‘disabled identity’ one must constantly participate in processes of disability disavowal, aspiring towards normativity, a state of near ablebodiedness, or at very least to effect a state of ‘passing’ . As Kimberlyn Leary

(1999, 85) put it: Passing occurs when there is perceived danger in disclosure. … It represents a form of self-protection that nevertheless usually disables , and sometimes destroys , the self it means to safeguard . The workings of internalized ableism by way of ‘passing’ are only possible when viewed broadly, moving the focus from the impaired individual to the arena of

relationships. In the interactivity with the norm (such as an ableized able-bodied person) another form of erasure is required. Ableist passing is not just an individual hiding their impairment or morphing their disability; ableism involves a failure to ask about difference, i.e. disability/impairment. For internalized ableism to occur there needs to be an existing a priori presumption of compulsory ableness . Such passing is about keeping the colonizer happy by not disturbing the peace, containing the matter that is potentially out of place.1 An example of

Page 296: openev.debatecoaches.org€¦  · Web view1AC . Observation 1: If I could find the spot where truth echoesI would stand there and whisper memories of my children's future. I would

‘ passing ’ under these circumstances would be the conundrum encountered by some university academics with impairments who experience trepidation about revealing their impairment status, fearing stigma and tenure discrimination despite the fact that many argue that they and others would benefit from disability focused mentoring and networking arrangements (see Bishop 1999; Monaghan 1998).